[{"image": "VQAonline_00000001.png", "question": "What are the 25th-75th percentile ranges for the annual income for graduate students in the United States?", "context": "And are there differences in average income between different types of fields, or different types of schools?\nThe diagram below says that the average income for grad students is $17k/year. But I'm more commonly seeing incomes in the range of $25k-$30k/year on PhysicsGRE.com...\n\n", "answer": "Note that those statistics are often averaged over all possible disciplines, and therefore, since there is such a wide disparity of stipend levels between different schools, and between different disciplines at the same school. \nAt the institution I attended for graduate school, engineers had stipends approximately 30% higher than the science majors (chemistry, physics, math, etc.). Similarly, at the undergraduate institution I attended, a similar disparity existed between science and humanities graduate students. \nIt is also important to ask if master's students, who often don't receive a salary at all, are included in that average. (And, since humanities students tend to stay longer, they may skew the statistics even more than one might expect.)\n(One final note to directly address the question: competitive national fellowships in the US currently pay between $30,000 and $35,000 per year as a stipend. I would estimate, then, that most stipends are significantly below that amount. I'd say something in the range of $20,000-$30,000 would be appropriate in STEM fields, and probably $15,000-$20,000 for full-time humanities PhD's.)\n", "topic": "academia", "url": "https://academia.stackexchange.com/questions/684"}, {"image": "VQAonline_00000111.png", "question": "How to improve quality of figures after reviewers have mentioned it?", "context": "The paper we are working on was accepted and we got a comment saying \"Quality of Figures could be improved\".\nAll the figures that are given have been exported from matplotlib and have all the standard labels with different color schemes and have attached the sample as well.\n\nSo what do I do? make it more pretty? or use a different package?\nNote - This is just a sample, and the real one has a set of 6 plots big enough to fit in almost half a page\n", "answer": "The illustration you provide seems like a standard plot that can be seen in most journals. That said, it can certainly be improved; many journals do not seem to care what they publish. What is acceptable also varies from person to person. There are however some standards that should be followed and your figure fall short on a few points. Many of the comments made to your post outline some of the improvements and I will also reiterate them here.\n\nA figure should be understandable when shown without its figure caption. Your figure falls short of this. By understandable I do not mean the deep scientific significance of what is shown but to understand what is shown. This is a point made by Tufte (2001) and is a good guideline for finishing any figure.\nAxis labels should include units. You have latitude and longitude which would be in degrees. The color scale is more problematic because I do not understand what it should be showing, units would obviously help.\nAxis labels should start with capital letters so Lat/Lon or Latitude/Longitude.\nThe heading of the subplots containg a time stamp should be consistent. The ISO standard for date/time is yyyy-mm-dd HH:MM:SS. So I would suggest using this consistently or (which might be better for a more general audience) use an unambiguous date and time format such as \"dd month, yyyy, HHMM hrs\" (e.g. 29 April, 2022, 1200 hrs). The problem with date and time is that almost every country has their own standard.\nThe third diagram in the top row has no x-axis label and tick labels\nThe number of tick labels could be reduced so that the axes are not overloaded with information. Every second label can be removed while keeping the tick marks\nThe resolution of the coloured fields should reflect the resolution of the data shown in the fields. If the data is as coarse as it looks in the figure, then that is how it should be shown.\nFigures must be made to work in their final size in the publication. Submitting vector based formats avoids many problems. When exporting to a bitmap one should consider the necessary resolution (in dpi) to make the illustration sharp in its final size. A figure such as yours that include bitmap fields should follow the same guidelines but obviously the fields in your subfigures will not change in apparent resolution as was stated in the previous point.\nThe data is latitude/longitude based running from c. 10 degrees N(?) to c. 80 degrees N(?). The width of the space between longitude lines will obviously decrease from S to N so that a square representation is not strictly correct. The plots are better representing reality if they are shown in some cartographic projection. This is clearly something many will ignore and maybe not even consider but it is one aspect to consider.\nAny graphing package or software is not perfect. It is always useful to learn to use for example the open source Inkscape to edit standard output from plotting packages or software to optimse your graphics before publication.\n\nConsidering the colour scale I cannot see anything wrong. Aspects to consider regarding choice of colour scale include what is standard (if any), what may make sense depending on what is shown and what can be read by someone with colour blindness.\nSo there are several aspects that can be improved or at least considered. At the same time providing a comment such as the one you appear to have received without any exemplification of what to do is quite useless on behalf of the reviewer/editor.\nReference\nTufte, E.R. 2001. The visual display of quantitative information. Second Edition. Cheshire, CT: Graphics Press.\n", "topic": "academia", "url": "https://academia.stackexchange.com/questions/184099"}, {"image": "VQAonline_00000078.png", "question": "What does the envelope sign after the author name mean?", "context": "I'm in the process of writing a paper for an upcoming conference. The conference provides a LaTeX template which can be found here: Information for Authors of Springer Computer Science Proceedings.\nLooking at the proceedings of the previous years I noticed a little envelope after the name of the first authors.\n\nHowever, I was not able to figure out what it means. So my question is what is the meaning of the little envelope after the name of the first author?\n", "answer": "With Springer journals, the envelope simply indicates that they are the nominated corresponding author for the paper. \nSources: this forum topic and a Springer 'Instructions to Authors' PDF.\n", "topic": "academia", "url": "https://academia.stackexchange.com/questions/115668"}, {"image": "VQAonline_00000032.png", "question": "Can you abbreviate the title of the Proceeedings of the IEEE?", "context": "I found this on Page 34 of IEEE EDITORIAL STYLE MANUAL, it's about how to edit the reference. I don't understand the following sentence: \n\nNOTE: The only exception to this rule is PROCEEDINGS OF THE IEEE, which never carried an acronym on the masthead\n\nDoes it mean when cite a paper from PROCEEDINGS OF THE IEEE, do not cite it in an abbreviation form. If it is, why the following example ( Page 35 of IEEE EDITORIAL STYLE MANUAL) gave an abbreviation form:\n\n", "answer": "The part you quote does not refer to the name of the Journal but to the vol. ##, no. ## section of the citation and is applicable to those earlier than 1988. \nSo the 1981 citation would be \n\nIEEE Trans. Automat. Contr., vol. AC-26, no.1, pp. 1\u201334, Jan. 1981.\n\nwhile a citation from the same journal past 1988 would be\n\nIEEE Trans. Automat. Contr., vol. 40, no.1, pp. 1\u201334, Jan. 1995.\n\nThe exception to the Proc. of IEEE applies here, as it never had an abbreviation in the volume and it has always been:\n\nProc. IEEE, vol. 11, no. 3, pp. 1-11, Jan. 1978.\n\nThe example they provide is correct. \n", "topic": "academia", "url": "https://academia.stackexchange.com/questions/38514"}, {"image": "VQAonline_00000062.png", "question": "Should grading take all students' performance into account?", "context": "This question is partially related to this question.\nBy trying to understand which one is the correct way to grade an exam I came out with this doubt. Should each exam be considered as independent or should a mark be assigned based on the other students' performance.\nAs an example\n\nif all the students final exams are from 80% to 100% correct should be considered the fact that the exam was too easy while grading it?\nif all the students final exams are from 30% to 60% correct should be considered the fact that maybe the exam was too hard while grading it?\n\nTo resume, while preparing/grading the exams should the results respect somehow the normal distribution or should each one be graded independently?\nEDIT:\nBased on my career as a student for every exam we had statistical results from all the students (min, max, avg) with the grades distribution. Most of them respected the normal distribution. So what it is not clear to me is if it depends on how the teacher create the exam, on how it is graded or just on how usually students perform.\n\n", "answer": "If the goal is to help students learn, then you should grade against an absolute standard (often called a rubric), and you should not grade by comparing students to each other. Experimentally, it has been found that if students believe their grades are under their own control, they will learn more. If students believe they cannot control their grades (because it depends on classmates' performance) then they will learn less. The difference in attitudes is known as a locus of control in psychology. According to the locus of control research, grading against an absolute standard will help students.\nIf you goal is to limit the number of students who pass, you might chose differently from what I suggest. But I don't agree with that goal.\n", "topic": "academia", "url": "https://academia.stackexchange.com/questions/86301"}, {"image": "VQAonline_00000002.png", "question": "Contribution of gifts and hospital services to university revenue", "context": "This question is based on the pie-charts shown in this link, such as this one for the University of Michigan.\n\nIn most of these charts, around 5% of the revenue comes from gifts. What are these exactly? \nHow do hospitals and clinical services play such a critical role for the revenue? \n\n\n", "answer": "Gifts are donations, typically from former students but they could come from anyone. That's just how higher education works in the US, and the tradition of making these donations is a big factor in how some US universities have become very wealthy (some of the donations are spent each year, but many are added to the endowment). The donations qualify for a tax deduction, so there's an element of government subsidy as well.\nHospitals and clinical services can loom large because university teaching hospitals are huge and medical services are very expensive. (You can have thousands of employees, some of them very well paid, and elaborate equipment.) However, this is really not relevant. The medical revenue is typically paying for the hospital activities and medical education, but not used to subsidize the rest of the university. Many universities do not have hospitals or medical schools.\n", "topic": "academia", "url": "https://academia.stackexchange.com/questions/1660"}, {"image": "VQAonline_00000109.png", "question": "A small part of my transcript is hard to read; can I get into trouble?", "context": "My university does not provide official transcripts in English, so I sent mine to a certified translator for translation. There is a footer on each page in Persian, and unfortunately, on one page, this footer covers the cumulative GPA and total passed credits of a summer semester; it is not unreadable but hard to read.\n\nSince December 15 is the deadline of some of the universities I want to apply for, there is no time for me to ask for another certified translated transcript. Do you think applying with this transcript might be a problem?\n", "answer": "Apply with this transcript and to be on the safe side make sure you mention your grades (especially the parts that are hard to read) somewhere else where they are easy to read. In this case, someone who wants to 'confirm' your declaration can 'easily' (or with a little effort) check with the image you showed.\nPerhaps also provide the originals? I am assuming the grades will be in the same numerals so at least the numbers can be read.\n", "topic": "academia", "url": "https://academia.stackexchange.com/questions/178719"}, {"image": "VQAonline_00000240.png", "question": "How exactly is $Pr(s \\rightarrow x, k, \\pi)$ deduced by \"unrolling\", in the proof of the policy gradient theorem?", "context": "In the proof of the policy gradient theorem in the RL book of Sutton and Barto (that I shamelessly paste here):\n\nthere is the \"unrolling\" step that is supposed to be immediately clear\n\nWith just elementary calculus and re-arranging of terms\n\nWell, it's not. :) Can someone explain this step in more detail?\nHow exactly is $Pr(s \\rightarrow x, k, \\pi)$ deduced by \"unrolling\"?\n", "answer": "The unrolling step is due to the fact you end up with an equation that you can keep expanding indefinitely.\nNote that we start with calculating $\\nabla v_\\pi(s)$ and arrive at\n$$\\nabla v_\\pi(s) = \\sum_a\\left[ \\nabla \\pi(a|s) q_\\pi(s,a) + \\pi(a|s) \\sum_{s'}p(s'|s,a) \\nabla v_\\pi (s') \\right]\\;,$$\nwhich contains a term for $\\nabla v_\\pi(s')$. This is a recursive relationship, similar to the bellman equation, so we can substitute in a term for $\\nabla v_\\pi(s')$ which will be a term similar just with $\\nabla v_\\pi(s'')$. As I mentioned, we can do this indefinitely which leads us to\n$$\\nabla v_\\pi(s) = \\sum_{x \\in \\mathcal{S}} \\sum_{k=0}^\\infty \\mathbb{P}(s\\rightarrow x, k, \\pi) \\sum_a \\nabla \\pi(a|x) q_\\pi(x,a)\\;.$$\nWe need the term $\\sum_{x \\in \\mathcal{S}} \\sum_{k=0}^\\infty \\mathbb{P}(s\\rightarrow x, k, \\pi)$ because we want to take an average over the state space, however due to unrolling there are many different $s_t$'s that we need to average over (this comes from the $s',s'',s''',...$ in the unrolling) so we also need to add the probability state of transitioning from state $s$ to $x$ in $k$ time steps, where we sum over an infinite horizon due to the repeated unrolling.\nIf you are wondering what happens to the terms $\\pi(a|s)$ and $p(s'|s,a)$ terms and why they are not explicitly shown in this final form, it is because this is exactly what the $\\mathbb{P}(s\\rightarrow x, k, \\pi)$ represents. The average over all possible states accounts for the $p(s'|s,a)$ and the fact that we follow policy $\\pi$ in the probability statement accounts for the $\\pi(a|s)$.\n", "topic": "ai", "url": "https://ai.stackexchange.com/questions/22094"}, {"image": "VQAonline_00000228.png", "question": "How can reinforcement learning be unsupervised learning if it uses deep learning?", "context": "I was watching a video in my online course where I'm learning about A.I. I am a very beginner in it. \nAt one point in the course, the instructor says that reinforcement learning (RL) needs a deep learning model (NN) to perform an action. But for that, we need expected results in our model for the NN to learn how to predict the Q-values. \nNevertheless, at the beginning of the course, they said to me that RL is an unsupervised learning approach because the agent performs the action, receives the response from the environment, and finally takes the more likely action, that is, with the highest Q value. \nBut if I'm using deep learning in RL, for me, RL looks like a supervised learning approach. I'm a little confused about these things, could someone give me clarifications about them?\n\n", "answer": "Supervised learning\nThe supervised learning (SL) problem is formulated as follows. \nYou are given a dataset $\\mathcal{D} = \\{(x_i, y_i)_{i=1}^N$, which is assumed to be drawn i.i.d. from an unknown joint probability distribution $p(x, y)$, where $x_i$ represents the $i$th input and $y_i$ is the corresponding label. You choose a loss function $\\mathcal{L}: V \\times U \\rightarrow \\mathbb{R}$. Then your goal is to minimize the so-called empirical risk \n$$R_{\\mathcal{D}}[f]=\\frac{1}{N} \\sum_{i=1}^N \\mathcal{L}(x_i, f(x_i)) \\tag{0}\\label{0}$$\nwith respect to $f$. In other words, you want to find the $f$ that minimizes the average above, which can also be formally written as\n$$\nf^* = \\operatorname{argmin}_f R[f] \\tag{1}\\label{1}\n$$\nThe problem \\ref{1} is called the empirical risk minimization because it is a proxy problem for the expected risk minimization (but you can ignore this for now).\nReinforcement learning\nIn reinforcement learning, you typically imagine that there's an agent that interacts, in time steps, with an environment by taking actions. At each time step $t$, the agent takes $a_t$ in the state $s_t$, receives a reward $r_t$ from the environment and the agent and the environment move to another state $s_{t+1}$. \nThe goal of the agent is to maximize the expected return \n$$\\mathbb{E}\\left[ G_t \\right] = \\mathbb{E}\\left[ \\sum_{i=t+1}^\\infty R_i \\right]$$ \nwhere $t$ is the current time step (so we don't care about the past), $R_i$ is a random variable that represents the probable reward at time step $i$, and $G_t = \\sum_{i=t+1}^\\infty R_i $ is the so-called return (i.e. a sum of future rewards, in this case, starting from time step $t$), which is also a random variable.\nIn this context, the most important job of the programmer is to define a function $\\mathcal{R}(s, a)$, the reward function, which provides the reinforcement (or reward) signal to the RL agent. $\\mathcal{R}(s, a)$ will deterministically or stochastically determine the reward that the agent receives every time it takes action $a$ in the state $s$. (Note that $\\mathcal{R}$ is different from $R_i$, which is a random variable that represents the reward at time step $i$).\nWhat is the difference between SL and RL?\nIn RL, you (the programmer) need to define the reward function $\\mathcal{R}$ and you want to maximize the expected return. On the other hand, in SL you are given (or you collect) a dataset $\\mathcal{D}$, you choose $\\mathcal{L}$ in \\ref{0}, and the goal is to find the function $f^*$ that minimizes the empirical risk. So, these have different settings and goals, so they are different!\nHowever, every SL problem can be cast as an RL problem. See this answer. Similarly, in certain cases, you can formulate an RL as an SL problem. So, although the approaches are different, they are related.\nIs RL an unsupervised learning approach?\nIn RL, you do not tell the agent what action it needs to take. You only say that the action that was taken was \"bad\", \"good\" or \"so so\". The agent needs to figure out which actions to take based on your feedback. In SL, you explicitly say that, for this input $x_i$, the output should be $y_i$.\nSome people may consider RL is an unsupervised learning approach, but I think this is wrong, because, in RL, the programmer still needs to define the reward function, so RL isn't totally unsupervised and it's also not totally supervised. For this reason, many people consider RL an approach that sits between UL and SL.\nWhat is deep learning?\nThe term/expression deep learning (DL) refers to the use of deep neural networks (i.e. neural networks with many layers, where \"many\" can refer to more than 1 or 1000, i.e. it depends on the context) in machine learning, either supervised, unsupervised, or reinforcement learning. So, you can apply deep learning to SL, RL and UL. So, DL is not only restricted to SL.\n", "topic": "ai", "url": "https://ai.stackexchange.com/questions/20891"}, {"image": "VQAonline_00000199.png", "question": "On-policy state distribution for episodic tasks on Sutton & Barto, page 199", "context": "In Sutton & Barto's \"Reinforcement Learning: An Introduction\", 2nd edition, page 199, they describe the on-policy distribution for episodic tasks in the following box:\n\nI don't understand how this can be done without taking the length of the episode into account. Suppose a task has 10 states, has probability 1 of starting at the first state, then moves to any state uniformly until the episode terminates. If the episode has 100 time steps, then probability of the first state is proportional to $1 + 100\\times 1/10$; if it has $1000$ time steps, it will be proportional to $1 + 1000\\times 1/10$. However, the formula given would make it proportional to $1 + 1/10$ in both cases. What am I missing?\n", "answer": "Let's first assume that there is only one action so that $\\pi(a|s) = 1$ for every state - action pair which simplifies the discussion.\nNow let's consider a case with 100 time steps, 10 states and uniform distribution for starting state $s_0$ with $h(s_0) = 1$. The result would be\n\\begin{align}\n\\eta(s_0) &= 1 + \\sum_{i = 0}^9 \\eta(s_i) \\cdot p(s_0|s_i) =\\\\\n&= 1 + \\sum_{i = 0}^9 10 \\cdot \\frac{1}{10} = 11\n\\end{align}\nNow let's consider a case with 1000 time steps where other settings are the same as in the first case.\n\\begin{align}\n\\eta(s_0) &= 1 + \\sum_{i = 0}^{9} \\eta(s_i) \\cdot p(s_0|s_i) =\\\\\n&= 1 + \\sum_{i = 0}^{9} 100 \\cdot \\frac{1}{10} = 101\n\\end{align}\nIn the first case\n\\begin{equation}\n\\mu(s_0) = \\frac{11}{9\\cdot 10 + 11} = 0.1089\n\\end{equation}\nand in the second case you have\n\\begin{equation}\n\\mu(s_0) = \\frac{101}{9\\cdot 100 + 101} = 0.1009\n\\end{equation}\nso it looks like you are correct that $\\mu(s)$ depends on the length of the episode, but they didn't really say that it doesn't. Obviously as the length of the episode increases so will the number of times a certain state was visited so you could say that formula implicitly depends on the number of time steps. If $h(s_i)$ is equal for every state then results would be the same in both cases regardless of number of time steps. Also, as the number of possible states grows very large, as it usually is in real problems, the results would be approaching each other as the number of states grows.\n", "topic": "ai", "url": "https://ai.stackexchange.com/questions/16616"}, {"image": "VQAonline_00000121.png", "question": "What is the variant of the minimax tree with 3 types of nodes called?", "context": "I have a task on my class to find all the nodes, calculate their values and choose the best way for the player on the given game graph:\n\nEverything is fine, but I have no idea what these dots are. Is this a third player, or just a 'split' for player1 move? Some kind of heuristics?\n", "answer": "The triangles pointing up are Max' nodes. We assume it starts. Then follows a random choice of moves at the circles, for instance, with a die. The triangles pointing down are from Min. This variant is called Expectiminimax, see https://en.wikipedia.org/wiki/Expectiminimax_tree.\nAt that circles you have to multiply the possibilities on the edges below that nodes to your current value and sum all products up. The circles in your picture mean that Min dices.\n", "topic": "ai", "url": "https://ai.stackexchange.com/questions/2760"}, {"image": "VQAonline_00000119.png", "question": "Understanding the perceptron algorithm in the book \"A Course in Machine Learning\"", "context": "The following text is from Hal Daum\u00e9 III's \"A Course in Machine Learning\" online text book (Page-41).\n\nI understand that $D$ is the size of the input vector $x$.\n\nWhat is $y$? Why is it introduced in the algorithm? How/where/when is the initial value of $y$ given? \nWhat is the rationale of testing $ya \\leq 0$ for updating weights?\n\n", "answer": "Y is the desired output of the perceptron (often referred to as target) , for the given set of input vectors.\nRationale behind Y.a<=0 :\nPrerequisite knowledge :\n\nA=A-B : Moves vector A away from direction of vector B\nA=A+B : Moves A in the direction of B\nA (.) B >0 ; A vector is directed acutely (<90 deg.) towards B vector\nA (.) B <0 ; A vector is directed away from (>90 deg.) from B vector [(.) denotes dot (scalar) product and bold letters indicate vectors]\nW is augmented vector (includes threshold as another weight along with normal input weights)\nX is augmented input (includes -1 as extra input (corresponding to threshold) along with other normal inputs\na(activation) = W (.) X\na >=0 ; Perceptron output 1\na<0 ; Perceptron output -1 (Not zero as implicit in the given algorithm I think)\n\nNow Rationale :\n(Y.a<0) \nThis means \nEither of the following :\n\nY=-1 and a>0 ; in this case the target output is -1 but as a>0 so the Perceptron outputs 1. So we must move the weight vector away from this set of input vector, so that angle between them increases and the dot product (a) becomes < 0 so that we can get the target output.\nHence : W=W-X\n\nOr ,W=W + (-1)*X\nOr, W=W+YX\n\nY=1 and a<0\n\nThis means the target output is 1 but as the activation is <0 so Perceptron is outputting -1. So we must move the weight vector close to this set of input vector so that the activation can become >0 (angle decreases) and the Perceptron can output the desired output.\nSo :\nW = W+X\nOr W=W + YX\nAgain , Y.a=0 is a boundary case.\nBy now I think you can understand the rationale behind Y.a=0. If any doubt , comment to this answer , I will explain it.\nSorry for so much long answer though. :) :)\n", "topic": "ai", "url": "https://ai.stackexchange.com/questions/2545"}, {"image": "VQAonline_00000195.png", "question": "What does the notation $[m]=\\{1, \\ldots, m\\}$ mean in the equation of the empirical error?", "context": "The empirical error equation given in the book Understanding Machine Learning: From Theory to Algorithms is \n\nMy intuition for this equation is: total wrong predictions divided by the total number of samples $m$ in the given sample set $S$ (Correct me if I'm wrong). But, in this equation, the $m$ takes $\\{ 1, \\dots, m \\}$. How is this actually calculated, as I thought it should be one number (the size of the sample)?\n", "answer": "This is a commonly used notation in theoretical computer science.\n$[m]$ is not the variable $m$, but is instead the set of integers from $1$ to $m$ inclusive. The empirical error equation thus reads in English:\nThe cardinality of a set consisting of the elements $i$ of the set of integers $[m]$ such that the hypothesis given input $x_i$ disagrees with label $y_i$, normalized by $m$. \n", "topic": "ai", "url": "https://ai.stackexchange.com/questions/15852"}, {"image": "VQAonline_00000187.png", "question": "Why does GLIE+MC Control Algorithm use a single episode of Monte Carlo evaluation?", "context": "GLIE+MC control Algorithm:\n\nMy question is why does this algorithm use only a single Monte Carlo episode (during PE step) to compute the $Q(s,a)$? In my understanding this has the following drawbacks:\n\nIf we have multiple terminal states then we will only reach one (per Policy Iteration step PE+PI).\nIt is highly unlikely that we will visit all the states (during training), and a popular scheduling algorithm for exploration constant $\\epsilon = 1/k$ where $k$ is apparently the episode number, ensures that exploration decays very very rapidly. This ensures that we may never visit a state during our entire training.\n\nSo why this algorithm uses single MC episode and why not multiple episodes in a single Policy Iteration step so that the agent gets a better feel of the environment?\n", "answer": "I feel the general answer is that we want to be as efficient as possible in learning from experience. \nPolicy improvement here always produces an equivalent or better policy, so delaying the improvement step to gather more episodes will only slow down learning.\nI would note too that often a different kind of Monte Carlo learning is used. Instead the speed of the update is typically controlled with a new hyper parameter $\\alpha$, instead of keeping track of the visit counts. The Q estimate is then updated something like:\n$$\nQ \\leftarrow Q + \\alpha \\left (G - Q \\right)\n$$\nThe value of $\\alpha$ then lets you tune how much evaluation vs improvement happens. This is called constant alpha Monte Carlo. Often this is used as a stepping stone to introduce TD methods, e.g., in 6.1 of the Sutton and Barto book.\n", "topic": "ai", "url": "https://ai.stackexchange.com/questions/13307"}, {"image": "VQAonline_00000639.png", "question": "What's the coordinate ability?", "context": "I'm on chapter 57 on Attack on Titan, and a \"coordinate ability\" keeps getting mentioned -- here by Hanji, but also earlier by either Bertholdt or Reiner (who apparently didn't want it to fall into the wrong hands). But what exactly is this coordinate ability?\nI skipped a lot of the chapters up to 34 because they were just rehashings of the stuff in the anime, and I skimmed over a lot of the Survey Corps action in the woods, since it was getting too repetitious for me, so it's possible I might just have missed something.\nEdit (taken from my comment in the answers): As an explanation to other readers, I was confused about this since at least in English, \"coordinate\" is pretty ambiguous without any further qualification. (It could be something geographical.) From the way Reiner and Bertholdt were talking about it, it sounded a bit like it could simply be a piece of crucial information that Eren might have in his lost memory.\n\nMoreover, because of how Hanji was describing it merely as a \"coordinate ability\", it still wasn't clear to me.\n", "answer": "Titans can have different types of abilities, like shifting back to human form (aka Titan Shifters), or putting armor on themselves (eg: Armored Titan, Annie hiding the back of her neck, ...). The third ability that was shown, was that of the Coordinate.\nThe Coordinate ability is basically the ability to control Titans. If the ability is controlled properly it can command all Titans to do whatever you want them to do.\n\n This is how humans were able to built the walls as sturdy and as big as they are right now.\n\nAs far as the story goes, it does not seem to work on Titan Shifters and there is supposed to be only one Titan with the Coordinate ability.\n\n Although the Beast Titan was shown to have a similar ability.\n\n", "topic": "anime", "url": "https://anime.stackexchange.com/questions/21157"}, {"image": "VQAonline_00000887.png", "question": "What does the word \"jiken\" on young Luffy's shirt mean in Japanese?", "context": "In One Piece manga, young Luffy wears a shirt containing the word \"jiken\".\n\nWhat does \"jiken\" mean in Japanese?\n", "answer": "In this context, jiken (\u4e8b\u4ef6) means \"incident\" or \"case\".\nThe full picture indicates tamago jiken which literally means \"the egg incident\" where a chick is killed due to the egg getting cracked.\nThere might not be any deeper meaning considering Oda's nature of humor.\n", "topic": "anime", "url": "https://anime.stackexchange.com/questions/57122"}, {"image": "VQAonline_00000818.png", "question": "Exactly what is Aizawa-sensei's quirk?", "context": "In My Hero Academia, chapter 6:\n\n Midoriya throws the ball, and Aizawa erases his Quirk, making him fail a first time.\n\nDuring this time, he grabs the student with his scarf, while not touching it. The drawn version is ambiguous, but the anime version is clear about that statement.\nHow so ? His quirk is only quirk erasing, not something about gravity or scarf controlling. Is there even a valid reason about that ?\nLinks to evidence:\n\nVideo extract (sorry for the commercial channel)\n", "answer": "Aizawa-sensei's quirk is Erasure:\n\nShota's Quirk gives him the ability to nullify anyone's Quirk by looking at them.\n\nThe cloth like material is not a quirk nor is it quirk controlled. It is called the Capturing Weapon:\n\nThe Capturing Weapon is a cloth-like equipment made from carbon fibers and a special metal alloy. Shota Aizawa wears it around his neck at all times.\nHe is able to manipulate the weapon to ensnare his targets and immobilize them. He uses the weapon as a powerful combo with his Erasure Quirk and as a precaution for Mutant-type Quirk users who are immune to his nullification power.\n\n", "topic": "anime", "url": "https://anime.stackexchange.com/questions/43598"}, {"image": "VQAonline_00000826.png", "question": "What's the song when Mikasa talks to her squad leader while on the mission helping Eren to seal the wall with a rock in episode 12?", "context": "On season 1 episode 12, there's a song when Mikasa talks to her squad leader while on a mission helping Eren to seal the wall with a rock, as shown on the following picture.\n\nWhat's that song?\n", "answer": "It\u2019s Vogel im Kafig. It\u2019s on the OST or you can find it on YouTube here: https://youtu.be/M2LdF1RlSL0.\n", "topic": "anime", "url": "https://anime.stackexchange.com/questions/44909"}, {"image": "VQAonline_00000533.png", "question": "Why is Ulquiorra the only Espada with a second release?", "context": "In Bleach Ulquiorra is the only Espada with a second release, and he admits he has not shown Aizen. So I have come to a few questions:\n\nWhy would he not show Aizen how powerful he is?\nWhy is he the only Espada with a second release?\nIf Aizen knew about his second release, where would he be in the Espada rankings?\n\n\n", "answer": "Answer to first question:\n\nA. Just in case.\nUlquiorra\u2019s a pretty careful guy. He likes to make plans. And stick to them. So it\u2019s possible that he hides his second release just because he knows that it\u2019s good to have an ace in the hole, just in case he ever needs it. Not that he\u2019s planning to use it for any nefarious purposes - he just figures, why tell if he didn\u2019t have to?\n\nUlquiorra: Also, I\u2019m not really a talker.\n\nB. As a tiny act of rebellion.\nOr you could read Ulquiorra as more of a Grimmjow-figure. You know, someone may recognize that Aizen is too powerful to actively rebel against, but who needs to do SOMETHING a little rebellious just to stay sane. Grimmjow leaves meetings early and sneaks off to the world of the living; Ulquiorra hides his second release. And this allows them to put up with working for a freakin\u2019 shinigami.\n\nGrimmjow: Um\u2026I\u2019m pretty sure that rebellion doesn\u2019t count if nobody knows about it.\nUlquiorra: And that\u2019s why you get spiritual pressure slammed to the floor, and I don\u2019t.\n\nC. Because he was actively disloyal to Aizen.\nIt\u2019s possible, though, that this wasn\u2019t a tiny act of rebellion, but rather a big one - that Ulquiorra had some sort of secret plan to rebel against Aizen someday, and he was saving his second release for that time. In which case letting Aizen go off to the WOL with the top three espada was a damn good plan. Ulquiorra would have had the castle to himself if he hadn\u2019t been killed by Ichigo. So close, Ulquiorra.\n\nUlquiorra: And I was really looking forward to sitting on that throne, too.\nAizen: Yeah, white marble is comfier than you\u2019d think.\n\n\nSource: \"Why does Ulquiorra keep his second release secret?\" on bleachlists.tumblr.com\n\nAnswer to second question:\nI came across an answer while scavenging the net (and I think it's pretty good):\n\nWell..the arrancars sword and the soul reapers swords are the same except for how they are released or how to manifest themselves after they are released. Arrancars have the power of soul reapers and soul reapers swords have 2 different forms shikai (1st form) and then bankai (2nd Form or release state). Now most Arrancars only have 1 sword release thats their resurreccion, resurreccion means resurrection and if u notice all arrancar release states are in spanish for some weird reason, but they only have one b/c arrancars have only been around for only about 1 year or a couple of months b/c they didn't come into existence till azien stole hogyoku out of Rukia's chest. Seeing how they're a new form of hollow but with soul reaper powers they probably have not explored all of their powers unlike soul reapers who have been around for over a 1000 years which means they would know more about their swords and how they operate. So basically the 2nd form Resurreccion segunda, which means 2nd resurrection, is the 2nd form of the sword similar to a soul reapers bankai. So in other words Arrancars haven't been around for a while which is why a lot of them don't know the other form unlike soul reapers who know both forms b/c they've been around forever. Now every arrancar has one but Ulquiorra was just the first to learn it. \n\nSource: \"Why does Ulquiorra have two forms?\" on Yahoo Answers\n\nAnswer for Third one:\nOfcourse Aizen knew.\n\n...[m]aybe Ulquiorra just wanted to know how much Aizen really watched, or how much he could just deduce by being insanely smart. So he kept his release hidden to see if Aizen could figure it out for himself. And like any guy who tests a god, this just didn\u2019t end well for poor Ulquiorra.\n\nAizen: Is there really any doubt that I knew?\n\n\nand about the ranking:\n\nMaybe he wanted to stay out of the top 3.\nMaybe Ulquiorra just didn\u2019t want to be in the top 3, and knew that with his second release taken into account, he would be. Ulquiorra\u2019s smart - he might have guessed that the people who went with Aizen to Fake Karakura Town wouldn\u2019t be coming back. Or maybe he just really likes the number 4.\n\nUlquiorra: Actually, I just know how important the number 3 is to Halibel.\nHalibel: \u2026\nUlquiorra: Don\u2019t worry, we\u2019re not having a moment.\nHalibel: Oh, good!\n\n\nSource: \"Why does Ulquiorra keep his second release secret?\" on bleachlists.tumblr.com\n", "topic": "anime", "url": "https://anime.stackexchange.com/questions/8836"}, {"image": "VQAonline_00000579.png", "question": "Is this girl really from Bleach?", "context": "On some sites which have ads I've been seeing the following image\npossibly NSFW if you look hard enough\n\n \n\nNow I've only seen up to episode 130 and I've never seen her before. However she kind of looks like Lighting (AKA Clare Farron) from Final Fantasy XIII.\nI am wondering: is this girl really from Bleach? If so when would I see her in the series (which episode or chapter), and if not where is she from?\n", "answer": "This isn't a Bleach character. This is, as you guessed, Lightning from Final Fantasy XIII, at least according to this page on TheAnimeGallery. \n", "topic": "anime", "url": "https://anime.stackexchange.com/questions/13821"}, {"image": "VQAonline_00000580.png", "question": "Why do Aya and Youko consider going to a ramen shop alone so amazing?", "context": "Consider the following page from Kiniro Mosaic.\n\nSome context, in case you don't know the series: The three girls are in high school. Karen is the blonde, Aya is the girl with black twin-tails, and Youko is the girl with short light hair. Karen is half-British, has never lived in Japan before, and still has some trouble with the language and culture. That's why she says \"It might be something normal for you two Japanese people, but for someone like me, it was a big adventure.\"\nIt was shown earlier that Karen is pretty outgoing, but the joke here seems to be that Karen thinks going to a ramen shop is something that's only a big deal to her because she's foreign, and it turns out that it's also a big deal to natives Aya and Youko. But why do Aya and Youko consider this \"amazing\" and \"grown-up\"? Youko's line about fast food restaurants suggests there's some cultural reference concerning ramen shops that I'm missing.\n", "answer": "Shall I assume that these characters are in high school or younger? In that case, yes, it not so common for high schoolers to hang out at a ramen restaurant compared to a fast food place like McDonald's or a family restaurant like Jonythan's where they can stay for a long time using the drink bar (all-you-can-drink soft drinks/juice/coffee/tea), study there, chat with friends, etc. Ramen restaurants are generally not upscale and, as you can see in the illustration, strangers may sit next to each other at bar-like seating, so this makes it convenient for middle-aged salarymen to eat lunch there alone (again, this is shown in the illustration). Ramen shops are not very conducive for having conversations, hanging out, and staying past when you empty your bowl. Thus, it's not a place that usually has a family or young people vibe.\nIf these characters are university students, it is less likely that the Japanese girls would consider Karen fearless, because university students tend to want to eat cheap and quick food, so ramen joints work for that, but it is still that case that many female college students wouldn't go to a ramen shop alone, and possibly wouldn't even go there in a group unless that group includes a male friend, because they will assume that the place will be full of college guys and older men. It's more common for Japanese university girls to eat at regular restaurants and cute caf\u00e9s.\n", "topic": "anime", "url": "https://anime.stackexchange.com/questions/13828"}, {"image": "VQAonline_00001103.png", "question": "How is interstellar gas density mapped from GAIA data?", "context": "I found the image below in Space.com's article This 3D Color Map of 1.7 Billion Stars in the Milky Way Is the Best Ever Made.\nThe caption for this image reads:\n\nThe Gaia spacecraft gathered observations for this all-sky view of the Milky Way and neighboring galaxies between July 2014 and May 2016, releasing the data on April 25, 2018. This image shows all the stars' colors and brightness (top), the total density of stars (middle) and the distribution of interstellar gas and dust across the galaxy (bottom). Credit: ESA/Gaia/DPAC\n\nThe first two maps reflect data obtained from analysis of GAIA's measurement of starlight as explained in this answer, but the third map is described as \"the distribution of interstellar gas and dust across the galaxy.\"\nAs far as I understand, GAIA does not transmit all complete image data to Earth, but instead does a significant amount of image processing, object detection, classification and analysis, and other data reduction in order to extract positions and radial velocity measurements for point-like objects such as stars, though of course small solar system bodies such as comets and asteroids end up in the raw data as well. \nQuestion: How is interstellar gas density obtained from GAIA data?\n\n", "answer": "Two techniques immediately spring to mind.\nFor the stars you detect, you can compare their colours and luminosities (Gaia provides photometric colours and distances) with what you expect for a star of that type at that distance. The difference between what you expect and what you observe tells you the reddening and extinction caused by interstellar dust, integrated in a column towards that star. By going through this process for lots of stars in different directions you build up a 3D picture of the distribution of dust.\nThe measurements are mostly insensitive to gas - simple extrapolation between dust and gas is often assumed.\nA second technique would be just to compare how many stars you can see and with what brightness distribution, with a model for the Galactic stellar population and dust distribution. This is an old-fashioned way of estimating dust extinction, works well locally, but I suspect not what is used in that map, for the simple reasons that Gaia also provides the distances to stars, and one of the purposes of the mission is to improve the Galactic model, not assume it.\n", "topic": "astronomy", "url": "https://astronomy.stackexchange.com/questions/26057"}, {"image": "VQAonline_00001302.png", "question": "What is the thickness of the Solar System disk?", "context": "What is the thickness in AU of the all planets' orbital plane height combined in the Solar System?\nExcluding Pluto.\nLooking for h\n\n", "answer": "The thickness of the planetary disc is dominated by Neptune, due to its large orbital radius.\nWe can calculate a planet's maximum distance from the ecliptic $h$ from the inclination angle of its orbit $\\theta$ and its aphelion distance $r$. We get a right triangle, with $r$ as the hypotenuse, so\n$$h = r\\sin\\theta$$\nThe table below was calculated using data from the NASA Planetary Fact Sheet. Angles are in degrees, distances are in millions of kilometres.\nPlanet distance from the ecliptic plane.\n\n\n\n\nName\nInclination\nAphelion\nDistance\n\n\n\n\nMercury\n7.0\n69.8\n8.506\n\n\nVenus\n3.4\n108.9\n6.458\n\n\nEarth\n0.0\n152.1\n0.000\n\n\nMars\n1.9\n249.2\n8.262\n\n\nJupiter\n1.3\n816.6\n18.526\n\n\nSaturn\n2.5\n1514.5\n66.062\n\n\nUranus\n0.8\n3003.6\n41.937\n\n\nNeptune\n1.8\n4545.7\n142.784\n\n\n\n\nSo the total thickness of the disc is $2\u00d7142.784 = 285.568$ million kilometres, which is almost $1.91$ au.\n\nHere's the Python code I used to create that table:\nfrom math import sin, radians\n\nnames = (\n 'Mercury', 'Venus', 'Earth',\n 'Mars', 'Jupiter', 'Saturn',\n 'Uranus', 'Neptune',\n)\n\n# Orbit data from https://nssdc.gsfc.nasa.gov/planetary/factsheet/\n# Inclination to ecliptic plane\ninc = [7.0, 3.4, 0.0, 1.9, 1.3, 2.5, 0.8, 1.8]\n# Aphelion in millions of kilometres\naph = [69.8, 108.9, 152.1, 249.2, 816.6, 1514.5, 3003.6, 4545.7]\n\nprint(\"|Name | Inclination | Aphelion | Distance|\")\nprint(\"|-|-|-|-|\")\nfor n, th, r in zip(names, inc, aph):\n # Perpendicular distance to eciptic\n h = r * sin(radians(th))\n print(f\"|{n} | {th} | {r} | {h:.3f}|\")\n\nHere's a live version of the script running on the SageMathCell server.\n\nAs John Holtz mentions in the comments, the true $h$ value for a planet may be smaller than the value shown in my table. The table's $h$ value only occurs if the planet's argument of periapsis is \u00b190\u00b0. Fortunately, Neptune's argument of periapsis is currently ~272\u00b0, so my $h$ value should be fairly close to the true value.\n\nJames K has supplied a list of orbit inclinations to the Solar System's invariable plane. Here's the table using those values.\nPlanet distance from the Solar System invariable plane.\n\n\n\n\nName\nInclination\nAphelion\nDistance\n\n\n\n\nMercury\n6.34\n69.8\n7.708\n\n\nVenus\n2.19\n108.9\n4.161\n\n\nEarth\n1.57\n152.1\n4.167\n\n\nMars\n1.67\n249.2\n7.262\n\n\nJupiter\n0.32\n816.6\n4.561\n\n\nSaturn\n0.93\n1514.5\n24.582\n\n\nUranus\n1.02\n3003.6\n53.468\n\n\nNeptune\n0.72\n4545.7\n57.121\n\n\n\n\nThat reduces Neptune's $h$ considerably! Uranus may even be the current \"winner\", depending on their arguments of periapsis with respect to the invariable plane.\n", "topic": "astronomy", "url": "https://astronomy.stackexchange.com/questions/41927"}, {"image": "VQAonline_00001126.png", "question": "What are these objects near Mirach", "context": "Night sky objects always interested me, nowadays Im trying to observe and photograph some of these objects. My phone is iPhone 6S, i used SkyGuide app to find positions, NightCap to photograph. Im sure brightest star is Mirach according to SkyGuide app. \nIt's photographed at 00:30 at my local time (Istanbul, London + 3), SkyGuide says Mirach's azimuth was 77\u00b0 11\u2019 28\u201d .\n\n", "answer": "If this is really Mirach, then I would say the point-like object above and a little to the left of it is the star mu And (another designation: 37 And) in the same constellation of Andromeda. Then the next point-like object slightly further above and to the left from mu And would be the star nu And (35 And). And a barely visible fuzzy object above nu And would be the Andromeda galaxy (M31). Then in the left lower corner we see another star Nembus (51 And), and above it the star phi And (42 And). And at the same height with nu And farther to the right from it at the edge of the building there is the star pi And (29 And).\nI would strongly suggest you to use Stellarium (www.stellarium.org) for identification of nearly any night sky object visible at your or any other location on Earth at any time.\n", "topic": "astronomy", "url": "https://astronomy.stackexchange.com/questions/27399"}, {"image": "VQAonline_00001366.png", "question": "Camera settings for Astrophotography", "context": "I own an Orion 10015 StarBlast 4.5 Astro Reflector Telescope and I recently bought a Lumix G7 camera then I bought the adapter needed to attach the camera to the telescope.\nI tested it today at morning while it was still dark and for some reason I cannot focus the things I am seeing through the camera attached to the telescope.\nI was trying to focus Venus and other objectives but I got this, do the camera need to have some specific settings for it? The telescope by itself works as usual, I use it almost every day.\nThanks in advance.\nThese are the adapters.\n\nhttps://www.amazon.com.mx/gp/product/B07DB8B7VN/ref=ppx_yo_dt_b_asin_title_o03_s00?ie=UTF8&psc=1\nhttps://www.amazon.com.mx/gp/product/B0000665V6/ref=ppx_yo_dt_b_asin_title_o04_s00?ie=UTF8&psc=1\n\n\n", "answer": "The Celestron adapter you are using is designed for telescopes having focal length greater than 650mm or \"slow\" telescopes, or to be more specific, you will have greater travel of the knob to achieve focus which suits this kind of barlow but your telescope is of 450mm or \"fast\", which is the opposite of the latter. So this will actually impact the focus and it has nothing to do with camera settings, what you can do is try to lengthen the adapter and if the problem is still not solved, buy an adapter that is designed to do the job for your telescope. Get a Universal T-adapter, which is an adapter and also a barlow lens that will suit almost every telescope. See if the telescope is collimated, although it has nothing to do with the focus adjustment it might improve viewing experience a lot, If you are not familiar with the term collimation, see a bright star you can see in your night sky and zoom it with eyepiece on until you see the secondary mirror obstruction and the spider vanes, if the dark circle is at the center it is collimated and you don't need to worry, if not adjust the screws in the secondary mirror until you see the circle at the center. And for some astrophotography tips see my answer here\n", "topic": "astronomy", "url": "https://astronomy.stackexchange.com/questions/49006"}, {"image": "VQAonline_00001062.png", "question": "What are these dark spots on Mars' north pole?", "context": "I was exploring Mars in Google maps and I found these dark spots, like tree shadows on the north pole. I don't have any idea what they might be.\n\nView this in Google maps.\n", "answer": "They look similar to the \"dark dune spots\" of the southern hemisphere, associated with accumulation of melting liquid at the bottom of dunes during warming seasons: https://en.wikipedia.org/wiki/Geyser_(Mars)\n", "topic": "astronomy", "url": "https://astronomy.stackexchange.com/questions/21324"}, {"image": "VQAonline_00000927.png", "question": "Why does Jupiter have so many moons?", "context": "Jupiter has a great many moons - in the hundreds, and they're still being discovered.\nWhat is the current theory for where all these moons came from? Are they rocks flying through space captured by Jupiter's gravity?\n\n", "answer": "Mass.\nThe more massive a body, the larger the gap between its lowest and highest orbit; the range of speeds at which a random body entering its gravity is likely to remain as its satellite. Sun has millions of satellites if you count all the asteroids; smaller planets tend to have one or two moons at most (Pluto with five being a notable and not fully explained exception)\nTo a lesser degree there's a matter of shape too. A regularly round body will have more regular and stable orbit than a potato-shaped one. Jupiter, being a gas giant is perfectly round. This doesn't play that much of a role though, especially with higher orbits.\nAnd last but not least, no destabilizing influence of other bodies. It's very hard to maintain a lunar orbit - artificial satellites around our Moon last only a couple years each, because relatively close neighborhood of Earth tends to destabilize orbit of anything orbiting the Moon. Jupiter being a single massive planet with relatively tiny (relatively to its mass) moons doesn't have them influence each other all that much. \n", "topic": "astronomy", "url": "https://astronomy.stackexchange.com/questions/321"}, {"image": "VQAonline_00001214.png", "question": "Are Tharsis Montes and Hellas Basin a result of the same event?", "context": "Looking at Mars map:\n\nthe highest, biggest mountains and their associated highlands are essentially antipodes of the deepest, huge basin. Roughly 180 degrees away, similar latitude except opposite sides of equator... this immediately brings to mind the idea that an asteroid impacted the area of Hellas Basin, and the shockwave traveled clear across the planet causing a bulge of Tharsis Rise, and causing multiple volcanoes to sprout there.\nIs there anything to this impression, or am I entirely wrong?\n", "answer": "\nAre Tharsis Montes and Hellas Basin a result of the same event?\n\nYou were not the first to have seen that the Hellas Basin and parts of the Tharsis Rise are roughly antipodal (Peterson 1978, Williams and Greeley 1994). The Tharsis Montes themselves are far too close to the equator to be considered antipodal to the Hellas Basin. The shield volcano Alba Mons however is almost exactly antipodal to the Hellas Basin.\nThis shield volcano is a bit to the north of the Tharsis Rise proper, and well to the north of the Tharsis Montes. Both Peterson and Williams suggested that the Hellas Basin impact might well have triggered the vulcanism that resulted in Alba Mons.\nThere's an issue with this hypothesis, which is that the apparent ages do not align. Improved remote observations of Mars has provided a tool for estimating ages of Mars features: crater density. Based on crater density, the Hellas Basin appears to be very old, at least 3.8 billion years old (Carr 2010). Alba Mons, while also quite old, isn't that old. It formed at least 200 million years after the Hellas Basin impact and perhaps as much as a billion years after (Ivanov 2006).\nBottom line: If the huge impact that formed the Hellas Basin did have any antipodal effects, those effects are buried under the magma that formed Alba Mons several hundred million years later.\n\nReferences:\nCarr, Michael H., and James W. Head III. \"Geologic history of Mars.\" Earth and Planetary Science Letters 294.3-4 (2010): 185-203.\nIvanov, Mikhail A., and James W. Head. \"Alba Patera, Mars: Topography, structure, and evolution of a unique late Hesperian\u2013early Amazonian shield volcano.\" Journal of Geophysical Research: Planets 111.E9 (2006).\nPeterson, J. E. \"Antipodal effects of major basin-forming impacts on Mars.\" Lunar and Planetary Science Conference. Vol. 9. 1978.\nWilliams, David A., and Ronald Greeley. \"Assessment of antipodal-impact terrains on Mars.\" Icarus 110.2 (1994): 196-202.\n", "topic": "astronomy", "url": "https://astronomy.stackexchange.com/questions/34749"}, {"image": "VQAonline_00002230.png", "question": "Why are the engines of the Airbus A400M slightly tilted down?", "context": "\nPicture: Youtube.\nFirstly, are the engines really tilted down? I have tried to check another picture over the net, seems they are really tilted down slightly. But need confirmation from you experts, however.\nSecond, if it is true, what is the reason?\n", "answer": "Just about all propeller driven airplanes big and small have the engine thrust line canted down a couple degrees from the wing chord line (not necessarily the longitudinal axis - that depends on the wing incidence), roughly equal to the cruising AOA, so that the thrust line is aligned a little closer to the free stream at cruise. On the A400 it looks like the nacelles are creating a bit of an optical illusion that exaggerates the effect, although the propellers do appear to be canted a bit relative to the fuselage. Thrust lines may also be offset to mitigate undesired pitching moments caused by thrust.\nSingle engine airplanes with the engine on the nose also often cant the engine to the side several degrees as well, to compensate for P factor and torque effects, reducing the amount of fin offset required at the tail. Look straight down on a typical single like a 172, with the prop stopped horizontally, and you can see the prop isn't quite parallel to the wings. Once you gain awareness of this you start to notice that most single engine planes you see have engines that are cockeyed, as if they were bent.\n", "topic": "aviation", "url": "https://aviation.stackexchange.com/questions/69935"}, {"image": "VQAonline_00002076.png", "question": "How do you know the maximum load a control surface can withstand?", "context": "\nsource\nThis is a separate strength testing (load testing) of the elevator of an airplane, how was the load determined? How did the engineers determine the maximum elevator load?\n", "answer": "Elevator loads are the function of the design and flight envelope. Looks like a small plane, so look up for example CS-VA 391 (or the correct national code for your category) how to calculate the loads. Base air force distribution on the horizontal tail can usually be assumed to be triangular, and increment from control surface deflection need to be added, also usually approximated as a triangular distribution acting from the hinge line.\n", "topic": "aviation", "url": "https://aviation.stackexchange.com/questions/55663"}, {"image": "VQAonline_00002111.png", "question": "What are these \"pimples\" on the jet engine for?", "context": "When looking into the General Electric CJ 805-23B jet engine, I have discovered that all forward section of the compressor is covered by a number of near identically looking \"pimples\". What are these tiny devices for, what are they doing? And why only front section of the compressor has them?\n\n", "answer": "They are the roots of the variable stator vanes that control the direction of the airstream as it enters each compressor stage to manage and optimize the angle of attack of the compressor blades in each stage. The vanes are connected to the ring shaped gang bar and somewhere around the circumference there is an hydraulic actuator, or a series of them, that drives the gang bar rings to move all the vanes connected to it at the same time. The hydraulic actuator is usually powered with pressurized fuel and is operated by the fuel control system to keep the vanes at the optimal rotational position.\nThis video (thanks, @Mackk) shows how the vanes operate:\n\n\n\n", "topic": "aviation", "url": "https://aviation.stackexchange.com/questions/57923"}, {"image": "VQAonline_00002492.png", "question": "Why is FAA Aircraft Certification Service department named as AIR?", "context": "The acronym AIR doesn't match with the functional name \"Aircraft Certification Service\"\n\"AIR (the Aircraft Certification Service) is the department within the FAA that develops and administers safety standards for aircraft and related products that are manufactured in the United States or are used by operators of aircraft registered in the United States\"\n\nRef1\nRef2\n", "answer": "Because it's not an acronym. It's an administrative designation code for the office. While many are acronyms, others are not. The key is to make sure they are unique across the agency. They are in effect tied to mail distribution in large government buildings. It's a lot easier to write AIR-130 than Aircraft Certification Service, Engineering Directorate.\nOnce you get out of headquarters, the regional/local offices tend more to the standard acronym for the office: Aircraft Certification Office (ACO), Flight Standards District Office (FSDO), Manufacturing and Inspection District Office (MIDO).\nIf you dig through the FAA organization you'll see that the major office codes have two common properties: they all start with an \"A\" and are three characters long.\nWhile Air Traffic Organization (ATO), Human Resource Management (AHR), Commercial Space Transportation (AST), are at least an approximate acronym; Airports (ARP), Finance and Management (AFN), and Aviation Safety (AVS) are more of an abbreviation.\nFlight Standards (AFS) and Aircraft Certification Office (AIR) are under AVS.\nAs to why AIR was chosen, I can only speculate as it was done a very long time ago and from experience in other government agencies, when these are set it's typically some staffer to the front office that proposes the structure and the boss signs off on it. So at most you'll find a directive that says what it is but is unlikely to specify the rational for each.\nPersonal speculation would be that AIR comes from 'Airworthiness' or looking way back, it was originally the Aeronautics Branch. AIR could be drawn from that or an early mission statement that tied them to 'Inspection and Regulation'. Take your pick.\n", "topic": "aviation", "url": "https://aviation.stackexchange.com/questions/92714"}, {"image": "VQAonline_00001433.png", "question": "Can I log time in a Cri-Cri twin-engine aircraft?", "context": "I was looking at potential experimental projects when I read this fascinating website about a tiny aerobatic-capable twin-engine airplane. It's light enough to be an ultralight, but much too fast:\n\nAside from the obvious fun of flying this little plane, I wondered whether:\n\nI'd be able to log time in the Cri-Cri as multi-time? My guess is yes.\nAssuming I'm MEL-IFR, could I log multi-IFR with a two-way radio, altimeter, Dynon-type AI, HI and at least one cert. VOR & glide slope? An approach cert. GPS setup would be too heavy I assume. My guess is this is wishful thinking...\n\n", "answer": "What a positively delightful looking aircraft!\nI see no reason you couldn't log time in this as multi-engine time - it clearly has two engines, with two separate sets of controls (at least throttles, from what I can see).\nSimilarly I see no reason you couldn't fly it IFR as an experimental aircraft, provided it's \"properly equipped\" (consult the FARs for all the details on what you need), passes an IFR pitot/static test, and has appropriate radios (Transponder which you'd likely need anyway, Com, and Nav).\nWhether you'd want to take it up in real weather is another matter though :-) \nFor that matter if you want GPS you could certainly have it without much weight penalty: throw in a Garmin GNS 430W and cover the Nav/Com and GPS in one ~7-8lb box. Whether or not that kind of investment in what's basically a single-seat super-ultralight is \"worth it\" is a judgment call only you could make, but it definitely seems possible.\n", "topic": "aviation", "url": "https://aviation.stackexchange.com/questions/2791"}, {"image": "VQAonline_00001399.png", "question": "Is it legal to smoke in your own plane?", "context": "I was just curious, in the past, smoking on planes was permitted.\nWhat is the law surrounding smoking in your own private plane?\n\n", "answer": "The short answer to \"Can I smoke in my own private aircraft?\" is \"Yes, usually\".\nAs Promised, I looked up the relevant FAR (23.853 - Passenger & Crew Compartment Interiors). \nThe actual rule is long and verbose, but pretty common sense. The important regulatory bits for most personal aircraft are just two points though. You must:\n\nHave \"an adequate number\" of self-contained ash trays\nYou don't want flicked ash or a snubbed cigarette to start an in-flight fire.\nHave \"No Smoking\" signs you can illuminate, if the passenger/crew compartments are separated\nThis doesn't apply to most of us since our \"personal\" airplanes are light singles, but if you've got a fancy twin with a curtain or door between you and the passengers you'll need those signs.\n\nFor \"commuter category\" aircraft there are even more requirements. Those requirements don't apply to most \"personal\" planes, so even though it's probably a good idea to meet some of those requirements if you're going to allow smoking in your personal aircraft they're not legally required.\n\nAll that said, while it's likely legal (unless you've removed your ashtrays or your plane didn't come with them) common sense generally dictates you don't want to smoke in your plane - It's bad for the vacuum-driven instruments' filters, will require even more attention to cleaning the inside of your windows, and a loose cigarette in turbulence or a bunch of ashes blown about by an open air vent can be a pretty substantial distraction.\n", "topic": "aviation", "url": "https://aviation.stackexchange.com/questions/613"}, {"image": "VQAonline_00002203.png", "question": "Does a skinless fuselage add to the likeliness of an unrecoverable spin?", "context": "While at Air Venture this week, I happened to hear a pilot of a small experimental airplane say that he liked the skinless (Naked) look his airframe had at one point during the build, but that he believed flying it that way made it more susceptible to an unrecoverable (I assume flat) spin. \n\nThinking about it later, I realized that I wasn't sure I understood how this could be. My understanding of airplane behaviour is that a flat spin is mostly a result of CG being to far aft. Anyway, I was wondering if anyone knew this?\n", "answer": "Well, I would solve with transparent Monokote if it was a model. No accounting for people's taste. \nAlthough some very early aircraft featured that look, designers soon realized that covered \"slab\" sides not only reduced drag, but also improved directional stability. Note that covered area aft of CG acts as an extension of the empennage, allowing for a smaller Vertical stabilizer. Many 100 years ago were comparitively tiny.\nAs far as behavior in a spin, what he has on the tail may be sufficient for safety, as well as good CG management. But I would certainly not recommend denuding your fuselage for a certain \"look\" without some expert advice and testing.\nDesigners do compare side area ahead of CG to aft of CG to estimate yawing tendency in a slip. So he may be OK.\n", "topic": "aviation", "url": "https://aviation.stackexchange.com/questions/66903"}, {"image": "VQAonline_00002575.png", "question": "Blocky video artifact problem on Canon 600D", "context": "When shooting video with my Canon 600D I get artefacts like the following in the pictures: (not throughout the whole video but only for a second and then its gone again, but comes back after a few seconds)\n\nThey are always in the bottom part of the image. (Although not visible on the LiveView Screen when viewed in camera)\nI tried to view the video on several devices and its always the same.\nI use a Class 10 SD Card, so this should not be an issue.\nSearching on google for the problem had no success. \n", "answer": "This is dropped data. It may be a problem with either your memory card or your camera itself. h.264 data, which is the format your video is recorded in, works off of a concept known as Group of Pictures. Basically, many of the frames of video don't store information about what they look like, but rather what they look like related to previous or future frames. The entire group of pictures is then decoded together. A group of pictures typically covers somewhere around half a second to 3/4 of a second of video.\nAdditionally, image data is encoded in blocks within each frame. When the data is encoded properly with sufficient bandwidth, these blocks blend seamlessly and you can't tell they are there. If the data rate is too low or something gets corrupted, then they become obvious though.\nThis is what has occurred in your video. During the recording, some data was dropped, resulting in the remaining data being shifted over and thus when the video is decoded, the image blocks are given information that doesn't line up correctly, resulting in the blocky look you are seeing. \nSince full frames are only stored every 15 frames or so, the video can't recover from the error until the next full frame (I frame) is reached. This is because the decoder only knows what changed from the neighboring frame for B (based on former and next frame) and P (based on previous frame) frames. Thus, it works from the corrupt frame and produces further corrupt frames.\nIt is possible that the camera is trying to record proper data, but the SD card is not bothering to write it down correctly. It is also possible that the encoder within the camera is corrupting the data and that the SD card is writing the data it is given faithfully. The best bet to troubleshoot is to try a different SD card and see if the problem continues in the same manner.\nIf the SD card does not fix the problem, I would suggest contacting Canon's customer support for further assistance with getting your camera fixed.\n", "topic": "avp", "url": "https://avp.stackexchange.com/questions/11989"}, {"image": "VQAonline_00002567.png", "question": "How to get a crisp, exact color match for lossless screen-cast?", "context": "Background\nRecording a screen-cast (no audio) in Xubuntu Linux. The following image shows a side-by-side comparison of the video (left) versus the desktop (right):\n\nThe screen-cast is recorded using:\nffmpeg -r 30 -f x11grab -s $WIN_GEO -i :0.0+$WIN_XY -vcodec huffyuv -preset ultrafast -qp 0 output.mkv\n\nHere, $WIN_GEO stores the browser dimensions (1030x794) and $WIN_XY has the browser location (1020x221).\n$ cat /etc/issue\nUbuntu Trusty Tahr (development branch) \\n \\l\n\nProblem\nThe colours in the video differ from the colours shown on the desktop, resulting in a slightly blurry playback. I have read that this could be due to conversion between RGB and YUV colours.\nQuestion\nHow do you record a desktop video (screen-cast) that when played back (e.g., using mplayer, VLC, or ffplay) results in an exact match to the original desktop colours and crispness?\nRelated\nRelevant pages:\n\nhttp://www.unixuser.org/~euske/vnc2swf/\nhttp://technically.us/pictures/test.html\nhttp://ubuntuforums.org/showthread.php?t=1392026\nhttps://askubuntu.com/questions/347574/how-to-do-screencasting-desktop-recording-with-high-quality-audio-and-video\nhttp://step4wd.com/2012/08/03/recording-screencast-in-ubuntu-linux/\nhttps://unix.stackexchange.com/questions/73622/how-to-get-near-perfect-screen-recording-quality\nhttps://unix.stackexchange.com/questions/65970/washed-out-colours-with-screencast-software-but-not-screenshot-on-ubuntu-all-ve\nhttps://trac.ffmpeg.org/wiki/How%20to%20grab%20the%20desktop%20%28screen%29%20with%20FFmpeg\nhttp://rayslinux.blogspot.ca/2012/04/capturing-linux-desktop-via-command.html\n\n", "answer": "Capturing the video frames in XWD format then converting to PNG yields results that are almost pixel perfect when encoded with ffmpeg.\nInstall xvidcap:\nmkdir -p $HOME/dev\ncd $HOME/dev\nsvn checkout svn://svn.code.sf.net/p/xvidcap/code/trunk xvidcap-code\ncd xvidcap*\n./autogen.sh\nLIBS=\"-ldl -lX11 -lXext\" ./configure --prefix=/usr/local\nmake && sudo make install\n\nCapture stills at 25 frames per second:\nmkdir -p $HOME/video\ncd $HOME/video\nxvidcap --fps 25 --quality 100 --cap_geometry 1024x768+768+288 --file out%05d.xwd\n\nThis creates a series of X Windows capture files inside $HOME/video. Convert the files to PNG format using ImageMagick then combine into a video:\nfor i in out*.xwd; do convert -define png:color-type=2 $i $(basename $i .xwd).png; done\nffmpeg -i out%05d.png -c:v huffyuv -preset veryslow -qp 0 -qscale:v 0 output.avi\n\nAn example script:\necho \"Click window to capture...\"\n\nINFO=$(xwininfo -frame)\nWIN_GEO=$(echo $INFO | grep -oEe 'geometry [0-9]+x[0-9]+' | grep -oEe '[0-9]+x[0-9]+')\nWIN_XY=$(echo $INFO | grep -oEe 'Corners:\\s+\\+[0-9]+\\+[0-9]+' | grep -oEe '[0-9]+\\+[0-9]+' | sed -e 's/\\+/,/' )\n\nOUTPUT_DIR=$HOME/video\n\necho Capturing to $OUTPUT_DIR...\nrm -rf $OUTPUT_DIR\nmkdir -p $OUTPUT_DIR\ncd $OUTPUT_DIR\n\nWIN_XY=$(echo $WIN_XY | tr , +)\n\nxvidcap --fps 25 --quality 100 --cap_geometry $WIN_GEO+$WIN_XY --file out%05d.xwd\n\n# Convert captured images to PNG format\necho \"Converting XWD to PNG...\"\nfor i in out*.xwd; do convert -define png:color-type=2 $i $(basename $i .xwd).png; done\n\nffmpeg -i out%05d.png -c:v huffyuv -preset veryslow -qp 0 -qscale:v 0 output.avi\n\nThis creates a high-quality video that looks sharp with VLC, can be edited in OpenShot, and uploaded to various video hosts. Playback with MPlayer yields slightly fuzzy and off-coloured results.\n", "topic": "avp", "url": "https://avp.stackexchange.com/questions/10428"}, {"image": "VQAonline_00002558.png", "question": "placing text in video filmed environments", "context": "I really love mixing reallife environment with floating text over buildings or virtual paintings on the walls.\nWhen the camera moves, the text also moves and looks like it is really attached to the surface. I hope you have the idea of what I'm trying to tell. Here are some questions: \n\nWhat (1 entry level, 1 professional) Software can be used or is typically used?\nDoes the Software detect the movement or is it neccessary to move the decal every frame?\n\n\n", "answer": "The general technique you're asking about is called motion tracking, of which there exist different types: Feature (\"point\") tracking, planar tracking, and 3D match moving. After Effects (and the bundled Mocha) is probably the best entry level software to each of these methods, but it definitely leaves room for professional growth. Apple's Motion includes a more basic toolset, is only suited to feature tracking, but is more beginner-friendly.\nWhen text is flatly attached to a simple, clearly visible surface such as a picture frame, you can use Feature tracking to track the corners of the object, and \"corner pin\" an image or text into the frame. The software will do its best to automate the process of tracking each corner, but if, for instance, an object passes in front of one of the points, you may have to manually keyframe its movement.\nPlanar tracking goes a step further and tracks the movement of full flat surfaces. Mocha, bundled with After Effects, is a planar tracker. It's fast and flexible, and you can achieve better results more reliably, but it's a little more complicated. To mask out an obstruction, for instance, you must rotoscope out the offending object.\nYou would only use 3D matchmoving if you wanted your text to be three dimensional, cast realistic shadows, receive image-based lighting, reflections, transparencies, in other words, be a realistic object in your scene. This is the most complicated method because it requires a back and forth workflow through tracking, 3d modeling, and compositing software. Different combinations of software packages will lump one or more of these steps within the same application.\nAn example of a dedicated professional motion tracking application is Boujou, although there are others (PFtrack comes to mind).\nHope this gives you a place to start.\n", "topic": "avp", "url": "https://avp.stackexchange.com/questions/7340"}, {"image": "VQAonline_00002652.png", "question": "How do I resize the canvas of a image layer in After Effects?", "context": "I'm using After Effects CC 2017. I've imported a PNG image with transparency, then I added a simple CC Light Wipe effect:\n\nThe problem is that the range of the lights produced by this effect are limited to the image size, so it produces an annonying thingy. Is there a way to resize the canvas of the layer of the image or another way to solve this (avoiding the need to resize the image canvas using external programs like Photoshop)?. \nI really tried to find out the answer researching in Google before asking here but I only found questions about resizing the entire compossition in the compossition settings...\n", "answer": "Add an adjustment layer on top (right-click in empty area of the layer names. Go to New menu) Apply the effect to that.\n", "topic": "avp", "url": "https://avp.stackexchange.com/questions/20225"}, {"image": "VQAonline_00002546.png", "question": "Why is my video alternating between blurry and sharp?", "context": "I notice this on a regular basis while watching videos I've downloaded. The image will alternate pretty quickly between blurry and sharp and sometimes the effect is quite drastic. Below is an example image of a tree and the difference between two frames.\n\nWhat is this effect called and why does it happen? I'm fairly certain it has to do with compression but what is going on?\nIf you receive a video like this and there's nothing you can do with how it was rendered or processed, is there anything you can do after the fact to minimize the effect? Settings in your video player maybe?\nThis particular screen shot was taken in Windows Media Player on Windows 7 playing an AVI file. The effect is still visible in VLC and I first noticed this while playing video through my PS3 off a harddrive. I can't say for sure, but I'm fairly certain that it's happened on multiple file types as well.\n", "answer": "The effect that you've seen is a side effect of the compression algorithm that was used to encode the video.\nThe most common compression algorithms (MPEG-4 and MPEG-2 among them) compress different frames with different methods, and that is why two very close and relatively similar frames may look so different.\nThe frame that looks pretty good is likely a keyframe. This is a frame that has all its pixels represented in the encoded stream. The compression introduces some loss of quality, but since the whole image is encoded the result typically looks good.\nThe image that looks bad is a frame that is not fully represented in the encoded stream. To make videos smaller, for certain parts of the picture the encoder references parts of previous frames that the player/decoder keeps stored in memory, so only the parts that are different are stored for the frame. Because of this these frames end up being a sort of collage. If the bitrate is too low, then you end up with really washed up images like the one you show. But increasing the bitrate enough helps dissimulate this effect, to the point that it is barely or not noticeable. Consider that all the TV that you watch (digital over the air, satellite, FIOS, cable, etc.) is encoded in this way.\nFor MPEG-2 it is common practice to insert a keyframe in the stream every half a second (15 frames for 29.97fps video, 12 frames for 25fps). For H.264 sometimes they go even longer than half a second.\nIn general, the image quality of a compressed video is at its highest at the keyframe points, and then slowly starts to degrade, going up again with the next keyframe.\nFor a more detailed explanation of frame types used in compression algorithms see the Wikipedia article Video compression picture types.\nUnfortunately a player cannot improve the quality of the video, since the problem is that the information to reproduce the frame with better quality isn't there. The only ways to address this is by increasing the bitrate when encoding and/or increase the number of keyframes. \nOn the player side I think the only thing you can do is play the video in a smaller window, so then these artifacts will be less visible.\n", "topic": "avp", "url": "https://avp.stackexchange.com/questions/4054"}, {"image": "VQAonline_00002632.png", "question": "How to get 'outer' composition's current time?", "context": "Suppose I've got 2 sequences, one named OUTER and one named INNER. Composition INNER is inside composition OUTER. INNER is shorter than OUTER and is placed in the middle of it's timeline. Inside INNER there is a text object which I want to display OUTER's current time - but I found out this is not so straightforward as I thought. My first idea was to create a solid namer TIMER inside OUTER and link the INNER's text object 'Source Text' to comp(\"OUTER\").layer(\"TIMER\").time and despite that this layer lasts from the beginning to the end of OUTER, the text shown from OUTER's perspective is always starting with 0.0 when INNER starts.\nFor the sake of clarity, here is the pic:\nIs there a proper way to get linked composition's current time?\n", "answer": "If you change the inner text layer's source text to this it will work:\nthisCompLayer = comp(\"outer\").layer(thisComp.name); \nthisCompLayer.time + thisCompLayer.startTime\n\nYou just offset the time to cope with the later start time. \nLooks like when AE calculates values from another composition it just calculates them for the current comp's time. It makes sense, because there could be multiple copies, or no copies at all of the inner comp in the outer comp.\n", "topic": "avp", "url": "https://avp.stackexchange.com/questions/17550"}, {"image": "VQAonline_00002674.png", "question": "My colors look awful, I can't tell why", "context": "I'm an amateur that recently made a video clip using the DJI Mavic Pro drone.\nThe footage itself turned out great, but the coloring is off.\nThe problem is, I can't tell what exactly is off, it just looks awful.\nI tried to do some color correction in my video software, but I couldn't get it right. \nHere is a single frame of the video, with no color correction:\n\nI came here to ask what parameters I have to consider to make it look more natural, or if that isn't enough, what I should change when recording the video. (exposure, filters?)\n", "answer": "'Awful' is pretty extreme - I don't actually think it looks that bad.\nPerhaps it's a little bit over exposed (the car is almost glowing, it's that white) so you might want to consider an ND filter if you're filming outdoors (and you can get one of those for your drone).\nColour-wise, it does look like it's tipping towards the blue-end of the spectrum. Maybe try throwing a little more yellow/orange into the mix. However, that could just be my monitor. Do, please, check your monitor settings, too, they may be throwing off your perception.\nThis guy seems to know his stuff in regards to Mavic colour grading: \n\n\nMaybe give that a watch and see if it helps :)\n", "topic": "avp", "url": "https://avp.stackexchange.com/questions/23924"}, {"image": "VQAonline_00002755.png", "question": "What dishes pair well the Maltese Harruba liqueur?", "context": "What food stuffs pair well with the Maltese liqueur Harruba?\nA friend of mine has several bottles of this sweet liqueur and we are hoping to make it into a meal of something more than just the ordinary.\n\n\nThe carob tree, in Maltese called Harruba (Ceratonia siliqua) has been a prominent component of the Maltese vegetation for several centuries. The fruit beans of this tree, also known as \u2018St. John\u2019s bread\u2019 formed a substantial part of the diet of the local population during the hard times of World War II.\n Carob beans, harvested by Maltese farmers, are crushed, roasted and boiled to produce a syrupy liquid with aromas and colour reminiscent of cocoa to which orange extracts are then added. Zeppi\u2019s Harruba liqueur can be served chilled, on the rocks or can also be taken neat to close off an exquisite meal. Harmless sediment may form at the bottom of the bottle due to the nature of the product. -\n Mediterranean Maltese Liqueur\n\n", "answer": "I'd say it is probably too sweet to be enjoyed as part of a meal except as an aperitif as @Eric suggested.\nSince Carob tastes like chocolate and this is a sweet strong liqueur. Maybe consider using it in a Tiramisu recipe as a substitute for Marsala? I have used Amaretto in this way before and it was delicious! \nCarob is often used as a substitute for chocolate so any liqueur based recipes where you want a hint of chocolate flavour would benefit from this. Maybe in a beef based dish (as we know a hint of dark chocolate in a beef casserole or chilli is a great trick!)\nI found this article online which could give you some inspiration.\n", "topic": "beer", "url": "https://beer.stackexchange.com/questions/7833"}, {"image": "VQAonline_00002747.png", "question": "Other than the USA, do any other countries have dry counties?", "context": "Other than the USA, do any other countries have dry counties?\nI understand that the term county may not exist in some counties, so one can adopt a different term that may be better suited for a particular countries.\nWhat is a dry county?\n\nA dry county is a county in the United States whose government forbids the sale of any kind of alcoholic beverages. Some prohibit off-premises sale, some prohibit on-premises sale, and some prohibit both. Hundreds of dry counties exist across the United States, a majority of them in the South. A number of smaller jurisdictions also exist, such as cities, towns, and townships, which prohibit the sale of alcoholic beverages. These are known as dry cities, dry towns, or dry townships.\n\n\nMap showing dry (red), wet (blue), and mixed (yellow) counties in the United States as of March 2012.\n", "answer": "Yes. Currently the list of Countries that have prohibition (and their counties) are:\n\n Afghanistan\n Bangladesh\n Brunei\n Iran\n Indonesia (prohibition in small business shops)\n India (prohibition in the states of Kerala, Gujarat, Bihar, Nagaland, Manipur and the union territory of Lakshwadeep)\n Libya\n Kuwait\n Maldives (excluding non-Muslims)\n Mauritania\n Pakistan (excluding non-Muslims)\n Saudi Arabia\n Sudan\n Somalia\n United Arab Emirates (prohibition in the emirate of Sharjah)\n Yemen\n\n\nMost of these are in Muslim-majority countries while India only prohibits alcohol in some states and their residing counties. \nI am interchanging county and country because not all countries that have counties or states that have been divided into counties.\n", "topic": "beer", "url": "https://beer.stackexchange.com/questions/6759"}, {"image": "VQAonline_00002743.png", "question": "Vaping Alcohol - fad or here to stay?", "context": "Just recently I discovered that not only can I vape in stead of smoking, but I can also vape instead of drinking. What's vaping alcohol all about? Well, you put a shot of spirits into an orb, which sits over a small candle (tea-light), and then through a glass or metal straw inhale the results.(check it out here with buzzfeed)\nThis is what the Vaportini looks like - credit Vaportini\n\nI understand that the alcohol bypasses the liver stage and just goes straight to your head and lungs. So, none of that 'oops, I've drunk too much - excuse me whilst I visit the bathroom' nonsense, with this method you apparently don't get to expell what your body can't handle.\nWith all this in mind, is there a limit (average) that can be vaped without doing myself some serious brain damage? What also is the effect on the lungs? Is this just another fad or is this the future?\n", "answer": "Limit: So, I would speculate that if you're \"vaping\" the same quantity of alcohol as you might instead drink, then you're facing the same risk. It's the same as drinking alcohol - the risk is cumulative and hard to define - it really depends on how much you drink and how often. \nEffect on lungs: Frankly this is hard to say, as I am quite confident there have been no large scale (AKA reputable) studies (nor can I find any) performed on the effect of vaping alcohol on lung tissues. I would expect the effects to be minimal, but then again, personally, I'd probably still just stick to drinking my alcohol.\nI can't provide any scientific basis to this, but I am thoroughly confident that vaping is simply a fad or novelty to enjoy - think of the increased costs and lack of accessibility compared to just drinking alcohol - you simply need a bottle of beer and a fridge, or a bottle of spirits and a glass versus a specialised glass, tea candle, vapour catching vessel and a glass or metal straw (seriously, who keeps one of them around?).\n", "topic": "beer", "url": "https://beer.stackexchange.com/questions/6641"}, {"image": "VQAonline_00002750.png", "question": "What to do with 50 (?) years old sake?", "context": "I found a bottle of sake in my grandfathers basement.\n\nAs there are no Latin characters indicating the age of the bottle (and I don't speak Japanese), I used google Translator and found the following on a card which I assume can be sent back to the manufacturer to give feedback.\n\n\u5dee\u51fa\u6709\u52b9\u671f\u9593\u662d\u548c45\u5e7412\u670814\u65e5\u8fc4\uff08\u5207\u624b\u306f\u3044\u308a\u307e\u305b\u3093\uff09\n\nwhich is translated to:\n\nSent validity period Until December 14, 1970 (stamps are not required)\n\nSo I guess it must be produced somewhere around 1970\nDoes sake age well?\nCan I sell the bottle, as in: is it worth anything due to its age?\nHow do I know if I can still drink it safely?\n", "answer": "Due its high alcohol content, you will still be able to drink the sake without worrying about your health.\nHowever, the recommended consumption period is usually one year after bottling. After that, the maker cannot guarantee the flavor of the sake. Whatever the flavor of your sake is right now, whether it aged well or went plain ugly, is not the flavor of the sake as advertised by the sake maker. Oh, and if the bottle is open, like wine, sake will oxidize rapidly.\nSake may or may not age well, depending on the sake. I don't have enough knowledge to say what those differences are, though from my experience, old sake can become either a tad acidic or more rich in its original flavor.\nYou may be able to sell the bottle so someone interested, but I don't think there is a giant market for it outside of Japan (though I may be wrong about that). In Japan, there would be plenty demand for old sake there.\nBy law, there should be a date on the bottle (not sure if they print that on the box) which indicates the \"month of bottling\". Although, that law may not have existed 50 years back...\nNote there is a chance it will go bad in the case it is nama (fresh) sake, which you can tell if you see \u751f somewhere on the label. Nama sake is sake that does not undergo the two pasteurization processes after being made, so the yeast is still alive. Taste guarantee is only about a half a year, and something 50 years old will probably taste yogurty or cheesy.\n", "topic": "beer", "url": "https://beer.stackexchange.com/questions/7146"}, {"image": "VQAonline_00002735.png", "question": "Beers that are popular but hyper-localized", "context": "I grew up near Baltimore, Maryland (USA), and spent a few years living in the city. One very popular beer in Baltimore is National Bohemian (\"Natty Boh\" in local parlance). You can see the beer's one-eyed mascot, Mr. Boh, all over the city. Apparently more than half a million cases of Natty Boh are sold in the city of Baltimore each year, which seems pretty high for a city with population 620,000.\n\u00a0\u00a0\u00a0\u00a0\u00a0\u00a0\u00a0\u00a0\u00a0\u00a0\u00a0\u00a0\u00a0\u00a0\u00a0\u00a0\u00a0\u00a0\u00a0\u00a0\u00a0\u00a0\u00a0\u00a0\u00a0\u00a0\u00a0\u00a0\u00a0\u00a0\u00a0\u00a0\u00a0\u00a0\u00a0\u00a0\u00a0\u00a0\nOne thing that I've found very odd is that outside of the small state of Maryland, you really can't find the beer at all, and 90% of the beer's sales are within the city of Baltimore. Thus, while the beer is reasonably popular, the interest is hyper-localized to a single city.\nAre there other examples of beers that have high sales volume but are hyper-localized, or are Baltimore and Natty Boh unique in this respect? Is there some sort of economic or marketing phenomenon behind hyper-localized popularity of a high-volume beer?\n", "answer": "Most of the long running local brands have been acquired by the conglomerates and continue to operate and distribute in a limited range, as there is little to no financial motive to further the distribution of these brands. \n", "topic": "beer", "url": "https://beer.stackexchange.com/questions/4578"}, {"image": "VQAonline_00002736.png", "question": "Cause of seasonal difference in beer production", "context": "I was recently crunching some numbers about beer production in the United States (data from the Alcohol and Tobacco Tax and Trade Bureau), and I noticed pretty significant seasonality in the production of beer:\n\nAverage production peaks in June (on average 18.1 million barrels) and is lowest in November and December (on average 14.1 million barrels). Are these ~25% seasonal swings in production based on some technical aspect of the brewing process, or are they based on something else (e.g. demand)?\n", "answer": "One thing you should take into consideration is that this chart certainly represents macro breweries production, once they still have the vast majority of the market-share, as opposite to craft breweries. So, we are talking here mainly about light lagers, known to be consumed as a refreshing beverage.\nSo, to me, it's pretty clear that the production increases as the weather warms up, peaking at the summer, and then falls down again towards chiller seasons. Considering distribution and other delaying factors, the peak at June would match the peak consumption at mid-July through mid-August.\nPS: I don't live in USA, and this is merely a theory I came up with, but which seems feasible to me.\n", "topic": "beer", "url": "https://beer.stackexchange.com/questions/4580"}, {"image": "VQAonline_00002749.png", "question": "Things people put in bottles of liquor or liqueur and are sold commercially?", "context": "People put the strangest things in bottles of liquor to be sold commercially. We have all probably heard of liquor stores, pubs or other drinking establishments occasionally selling tequila with a worm in the bottle. Be what the history of the mescal worm is: true or false is not my question.\nThe other day, I was in a local liquor store and noticed bottle of liquor called Eau de Vie de Poire in the Bottle with a real full sized pear inside the bottle containing 40% (ABV) of pear brandy. One can see how they get the pear in a bottle here and here. \n \nEau de Vie de Poire Pear In The Bottle\nMy question is quite simple: What other things are sold in bottles of liquor or liqueur commercially other than fruit or possibly worms (tequila)?\n", "answer": "You can see pretty much anything in the bottle. This article provides a good first look (trigger warning!).\nBesides the stuff described there I witnessed bottles with exotic fish, giant prawns, and kelp. I even tasted some of them; we were not amused.\n", "topic": "beer", "url": "https://beer.stackexchange.com/questions/7030"}, {"image": "VQAonline_00003321.png", "question": "Identify year of specialized s-works tarmac?", "context": "Can anyone identify this specialized s-works tarmac? The seller says it's a 2013, but I haven't found it.\n\n", "answer": "Looking on bikepedia.com, I can only find listings from 2012 and 2013 for Specialized Tarmac SL4 Pro, or S-Works Tarmac SL4. The frame in the photo seems to have decals for both 'S Works' AND 'Pro' (on the seat tube). This seems inconsistent with any photographs I can see on google images. SWorks frames tend to have 'SL4' on the seat tube instead but I have only seen photos taken from the drive side.\nAlso, the colour of the red decals on the bike seem a suspiciously similar shade to those on the wheels, which makes me skeptical even further. I haven't seen any google images results in this paint scheme, and expect that you haven't either, hence the question here.\nI would ask the seller for more information, for warranty documentation, proof of purchase, a frame number that you can check with Specialized etc. Maybe it's all legit and original, or maybe it's been resprayed. If so, the owner shouldn't have any issue with answering your questions. But if it seems too good to be true, be prepared to walk away.\n", "topic": "bicycles", "url": "https://bicycles.stackexchange.com/questions/55806"}, {"image": "VQAonline_00002861.png", "question": "Can't remove crank arm on older Mercier", "context": "I'm rebuilding an older Mercier and have decided to tackle the bottom bracket. I've read a few articles and watched a few videos on how to remove the crank but none of them look anything like what I've got on this bike.\nBasically, there are bolt threads on one side and a 'nub' on the other side that would indicate that this is what I would remove the crank arm with, but I'm not having any luck removing the bolt.\nI've attached a picture as I'm sure I'm probably not using the correct terms for some of these parts.\n", "answer": "What you are looking at is called a \"Cottered Crank\". The shaped cotter pin holds the crank in place and they can be next to impossible to remove without significant effort or a cotter pin press.\nSheldon Brown has an excellent article on removing cottered cranks using a hammer and pipe, which is a method I have used before, albeit multiple decades ago.\nWhen you get it off you may want to consider replacing the bottom bracket and cranks with a more modern design that is easier to maintain, unless this is a restoration job and you want to stay true to the original.\n", "topic": "bicycles", "url": "https://bicycles.stackexchange.com/questions/10856"}, {"image": "VQAonline_00003142.png", "question": "How to mount TT shifters and TT brake levers together?", "context": "I am starting a new project to build a custom bike with bullhorn handlebar. It'd most likely be made either with road or hybrid type of frame. I am planning on having both front and real derailleurs, and brakes (writing all this because, you know, fixies :) There are many unknowns/choices (obviously!), but I am really struggling with one detail in particular: shifters and brake lever mount to the bullhorn handlebar. I was searching a lot and came across this bike: \n\nThis is way aero for my taste, but check out that brake lever/shifter combo on the right side of the handlebar. Does anyone know what these components are? I searched Vision's site, but couldn't find TT brake level with opened top to which TT shifter could be mounted. And the lever doesn't look to be crosstop one; so I am really confused.\nDoes anyone know how can this be done? Thanks!\n", "answer": "I can't tell for sure what's going on in that picture, but I think it's a modified older Vision lever that clamps on the OD rather than inside the bars, and then the shifter is just stuck out the end. Ways of doing it with off the shelf parts that I know of are the Jtek Aerobrake, Ultegra ST-6871 and DA ST-9071, the Record and Athena tri/tt EPS levers, and SRAM eTap blips (their equivalent, kind of). \n", "topic": "bicycles", "url": "https://bicycles.stackexchange.com/questions/41998"}, {"image": "VQAonline_00003006.png", "question": "Steel rims responsible for slicing tubes?", "context": "I have an old Giant dutch bike and it has steel rims. Since I'm a big boy I felt that the 60PSI on the tyres is not enough and pumped my tyres to the maximum of 85PSI which is what my Schwalbe Marathon Plus tyres were supposed to be able to take in. It was great, riding was so much easier all of a sudden! Not the slowest guy on the road anymore, really fast if I wanted to in fact! I loved it.\nUnfortunately a few days after I came to find one of the tubes neatly sliced. The tyre untouched. I wasn't sure what had happened but walked to work that day and planned to bring it to the workshop asap. When the day came I exited my house and found the other tube sliced in the exact same way.\n\nI have meanwhile talked to lots of people. Some say it has to do with the steel rims even though I can't make any sense of that. I have also encountered people that said I just need the right tubes and tyres and it shouldn't happen anymore.\nI'm now thinking of either buying a new bike, exchanging the wheels(which would be more expensive than the initial price tag of the bike) or give new tubes/new rimband and maybe different tyres a shot?\nDoes anyone have experience with this sort of thing or any sort of input?\nthis didn't help: Tire popping out of rim when inflating tube \n", "answer": "OK if the tire was blown off the rim then I highly suspect that is the whole story.\nWhen the tube leaks out the because tire bead has come out the tube will burst and you will see a slit. \nA tube outside a tire cannot take much pressure. \nMost likely when you went from 60 to 85 psi it was enough pressure to push the tire off the rim (bead). \nI have seen this with some older bikes. The rim is a little small.\nLook for tires that run small and run at lower pressures. \nYou might also have a mismatch like a 27 on a 700c.\nA Schwalbe Marathon Plus is a pretty hefty tire and tend to fit a little tight so this is kind of strange. But it did happen after you went to 85 PSI. \nMaybe give it a try again at 60 PSI. Even if you get a new bike you would have this as a backup.\n", "topic": "bicycles", "url": "https://bicycles.stackexchange.com/questions/29255"}, {"image": "VQAonline_00003388.png", "question": "Is this a crack on the carbon frame?", "context": "I have just noticed this on the top of my saddle bar and where the seat post clamp is. Is it a crack probably caused by the saddle pressure?\n\n", "answer": "Doesn\u2019t look like a crack, looks more like a surface void that formed during the resin injection molding. These types of imperfections are not uncommon, especially on the inside of the frame where you can\u2019t see them. Less common to see them on the outside as they are usually caught in quality control inspections. While they are not ideal, as areas with voids will be somewhat weaker than areas without voids (depending on the size and depth of the void), they are not critical like a crack.\n", "topic": "bicycles", "url": "https://bicycles.stackexchange.com/questions/60186"}, {"image": "VQAonline_00003710.png", "question": "Cannot steer my bike with with new adult training wheels", "context": "I just had adult training wheels put on my bike and I cannot steer it properly. The roads in my neighborhood are all cambered (raised in the middle) and the bike keeps pulling strongly to the right. I have to lean my body way out to the left to conteract the pull to the right. The training wheels are set almost right on the ground. Should I move them higher so there is space between the wheels and the ground? Or am I doing something wrong? Thanks.\nZaffer\nAddemdum: I made a diagram (see attached). I think I need to raise both training wheels at least an inch and get used to the idea of having zero or only one training wheel on the ground at a time.\n\n", "answer": "Bicycles (and other two-wheeled vehicles like motorcycles) constantly need righting while riding. That is done mostly by the rider. If the bicycle starts to tip to the side, you automatically steer to the same side so you induce an opposite effect. This is helped by the head angle, the wheel trail and the gyroscopic effect, but active steering plays the biggest role. If a bicycle leans to one side for a longer time, it starts to turn or it falls over. (In fact, this is how you steer a bike in a corner: you steer in the opposite direction to make it lean into the corner and then you just keep it leaning until you've taken the turn.)\nIf you're cycling on the right side of a cambered road, the part of the road you're riding on slopes to the right. On a normal bike this isn't a problem since you can ride upright regardless of the slope.\nCompared to this, a bicycle with training wheels is a completely different vehicle. It doesn't need righting and it doesn't need to lean into corners. In fact, if you try to ride it like a normal bike it leads to the experience you described. Your training wheels make the bike lean to the right, perpendicular to the road surface. Your natural reaction to the bike leaning to the right is to steer to the right to counteract the leaning. You then trick your brain into thinking the bike is already upright by leaning left relative to the bicycle.\nIf you would lift the training wheels a little, you would create a vehicle that behaves like a bicycle when it is upright but makes it impossible to lean into corners and actually throws you off if you try to go into a corner at any speed.\nThe solution is to remove the training wheels completely. Training wheels suck and they don't teach you how to ride but they also hinder your progress.\n", "topic": "bicycles", "url": "https://bicycles.stackexchange.com/questions/85604"}, {"image": "VQAonline_00003376.png", "question": "(Light?) rust removal from chain", "context": "Situation:\nI'm new to the world of trike maintenance. My son and I purchased recumbent trikes at the end of last summer. The weather for the past couple of months has been cold, snowy, cold, and...cold (Northern Pennsylvania climate.) So I cleaned them, tried my hand at lubricating the chains for the third time since owning them, and put them up in the shed to keep them out of the sun and precipitation.\nThis weekend looks like the first time in awhile that we might be able to go for a ride without freezing the fingers off. I took a look at the trikes, anticipating having to add air to tires and maybe clean the disc brakes, and saw the chains:\n\nThe top is one trike, the bottom two are the other...\nOn soliciting some info, I was told that is \"definitely rust.\" Apparently I didn't do something right when I put them in the shed for a month of storage, or rust is not easy to fend off. As I said, I'm new to this, and trying to improve maintenance skills. I'm hoping this isn't so bad that the chains require removal or replacement at this point.\nQuestions:\n\nHow bad in shape are these chains, relatively speaking? \nI have yet to find a definitive answer to this; how do I clear off the rust? WD-40, followed by degreaser and relubricate them?\nThese are chains that came stock with the trikes. I wonder if the future fix is to get a particular type of rust-resistant chain?\n\nBeing winter in PA, the roads will no doubt be salty from road melt material. I'm trying to keep the trikes cleaned, touch up paint where chips are found, wipe them down, etc...but there's a lot of contradictory advice on how to do certain things. Any advice/fixes for the questions above would be appreciated... \n", "answer": "As long, as you can easily move the links of your chain, there's no problem. Just lube them and ride them, and they'll be fine. The movement will rub any rust from the critical places. You may need to work the chain for a while before the links become free, a creeping oil should help get the links moving again.\nThat said, there are two ways to ensure that a chain won't rust (as quickly as yours):\n\nMake sure they are always well-lubed. A surface that's covered by oil is not covered by humid air, and cannot rust. May work well during storage, not so well when you actually use the bike.\nUse chains with an anti-rust coating. I have quite good experiences with this, though the chains are a bit more expensive. I'm not sure how robust the anti-rust coating is against sliding through a front deraileur, though (I only ride IGH). In any case, I've never had a rust-proof chain go stiff.\nOf course, anti-rust coated chains still like being kept well-lubed...\n\n", "topic": "bicycles", "url": "https://bicycles.stackexchange.com/questions/59467"}, {"image": "VQAonline_00003840.png", "question": "How to increase the size of the shapes that denotes effect sizes of different datasets and metaanalysis?", "context": "I am doing metaanalysis with forestplot from ggforestplot package. I have 7 different datasets from 7 different countries. After metaanalysis, I have 7 Effect sizes and a Meta-analysis effect size, i.e., total of 8 effect sizes. Now, I have generated a forest plot using this code:\n# Forestplot\nmy_plot <- forestplot(\n df = final,\n estimate = estimate,\n logodds = FALSE,\n colour = country==\"metaanalysis\",\n shape = country,\n title = \"Associations to disease accross populations\",\n xlab = \"Effect Size\",\n #xlim = c(-1.5, 1.5),\n xtickbreaks = c(-1.0, -0.5, -0.45, -0.4, 0, 0.4, 0.45,0.5, 1.0)\n)+\n # You may also want to add a manual shape scale to mark meta-analysis with a\n # diamond shape\n ggplot2::scale_shape_manual(\n values = c(5L, 0L, 1L, 2L, 8L, 19L, 4L, 3L),\n \n )\n\nBut, the shapes denoting the countries and the metanalysis seem very small. How can I make them larger for better visualization? Can anyone please help me?\nRepresentative data for replication:\nfinal <- data.frame(\n stringsAsFactors = FALSE,\n name = c(\"RIBOSYN2-PWY__flavin_biosynthesis_I_(bacteria_and_plants)\",\n \"PWY-6385__peptidoglycan_biosynthesis_III_(mycobacteria)\",\n \"PWY-6151__S-adenosyl-L-methionine_cycle_I\",\n \"PWY-6387__UDP-N-acetylmuramoyl-pentapeptide_biosynthesis_I_(meso-diaminopimelate_containing)\",\n \"PWY-6700__queuosine_biosynthesis\",\n \"PEPTIDOGLYCANSYN-PWY__peptidoglycan_biosynthesis_I_(meso-diaminopimelate_containing)\",\n \"PWY-6386__UDP-N-acetylmuramoyl-pentapeptide_biosynthesis_II_(lysine-containing)\",\n \"PWY-5667__CDP-diacylglycerol_biosynthesis_I\",\n \"PWY0-1319__CDP-diacylglycerol_biosynthesis_II\",\n \"PWY-6122__5-aminoimidazole_ribonucleotide_biosynthesis_II\",\n \"PWY-6277__superpathway_of_5-aminoimidazole_ribonucleotide_biosynthesis\",\n \"PWY-6163__chorismate_biosynthesis_from_3-dehydroquinate\",\n \"TRPSYN-PWY__L-tryptophan_biosynthesis\",\n \"PANTOSYN-PWY__pantothenate_and_coenzyme_A_biosynthesis_I\",\n \"GLYCOCAT-PWY__glycogen_degradation_I_(bacterial)\",\n \"P161-PWY__acetylene_degradation\",\n \"FUC-RHAMCAT-PWY__superpathway_of_fucose_and_rhamnose_degradation\",\n \"P108-PWY__pyruvate_fermentation_to_propanoate_I\",\n \"PWY-7328__superpathway_of_UDP-glucose-derived_O-antigen_building_blocks_biosynthesis\",\n \"TCA__TCA_cycle_I_(prokaryotic)\",\n \"PWY-6901__superpathway_of_glucose_and_xylose_degradation\",\"PWY66-400__glycolysis_VI_(metazoan)\",\n \"FUCCAT-PWY__fucose_degradation\",\n \"PRPP-PWY__superpathway_of_histidine,_purine,_and_pyrimidine_biosynthesis\",\n \"PWY-7234__inosine-5'-phosphate_biosynthesis_III\",\n \"TRPSYN-PWY__L-tryptophan_biosynthesis\",\n \"PWY-6122__5-aminoimidazole_ribonucleotide_biosynthesis_II\",\n \"PWY-6277__superpathway_of_5-aminoimidazole_ribonucleotide_biosynthesis\",\n \"RIBOSYN2-PWY__flavin_biosynthesis_I_(bacteria_and_plants)\",\n \"PWY-6163__chorismate_biosynthesis_from_3-dehydroquinate\",\n \"PWY-5667__CDP-diacylglycerol_biosynthesis_I\",\n \"PWY0-1319__CDP-diacylglycerol_biosynthesis_II\",\n \"PANTOSYN-PWY__pantothenate_and_coenzyme_A_biosynthesis_I\",\n \"RIBOSYN2-PWY__flavin_biosynthesis_I_(bacteria_and_plants)\",\n \"PWY-6386__UDP-N-acetylmuramoyl-pentapeptide_biosynthesis_II_(lysine-containing)\",\n \"RIBOSYN2-PWY__flavin_biosynthesis_I_(bacteria_and_plants)\",\n \"PANTOSYN-PWY__pantothenate_and_coenzyme_A_biosynthesis_I\",\n \"PWY-6385__peptidoglycan_biosynthesis_III_(mycobacteria)\",\n \"PWY-6387__UDP-N-acetylmuramoyl-pentapeptide_biosynthesis_I_(meso-diaminopimelate_containing)\",\"PWY-6700__queuosine_biosynthesis\",\n \"PWY-6151__S-adenosyl-L-methionine_cycle_I\",\n \"PWY0-1319__CDP-diacylglycerol_biosynthesis_II\",\n \"PWY-5667__CDP-diacylglycerol_biosynthesis_I\",\n \"PEPTIDOGLYCANSYN-PWY__peptidoglycan_biosynthesis_I_(meso-diaminopimelate_containing)\",\n \"PWY-6387__UDP-N-acetylmuramoyl-pentapeptide_biosynthesis_I_(meso-diaminopimelate_containing)\",\n \"PWY-6385__peptidoglycan_biosynthesis_III_(mycobacteria)\",\n \"PANTOSYN-PWY__pantothenate_and_coenzyme_A_biosynthesis_I\",\n \"PEPTIDOGLYCANSYN-PWY__peptidoglycan_biosynthesis_I_(meso-diaminopimelate_containing)\",\"PWY-5667__CDP-diacylglycerol_biosynthesis_I\",\n \"PWY0-1319__CDP-diacylglycerol_biosynthesis_II\"),\n estimate = c(0.492903365186722,\n 0.490116714405755,0.470681520902659,0.465726944992238,\n 0.461786439528495,0.455469435286562,0.449174326308655,\n 0.433755737833203,0.433670791448977,0.412009484469875,\n 0.412009484469875,0.409849212159568,0.407818381473059,\n 0.406187518931948,-0.405573831841341,-0.407806403622317,\n -0.418351108232386,-0.419107645158175,-0.420781760133428,\n -0.441930843508614,-0.444723954813865,-0.451224756237618,\n -0.504642907784859,-0.515843696383127,-0.523823484265933,\n 1.01203740887784,0.860911808112254,0.860911808112254,\n 0.782824395343453,0.779871038531055,0.763412247455786,\n 0.763412247455786,0.744965434413444,0.735086809796288,\n 0.733132943355509,0.731855318166865,0.730523378560539,\n 0.728356885227315,0.694487466356475,0.694335496605963,\n 0.68469489116557,0.680753784497535,0.680732580641012,\n 0.668909750729078,0.665107737526883,0.662352062210176,\n 0.643767077562848,0.640204416487105,0.639460661647805,\n 0.637911093993277),\n se = c(0.10858902830706,\n 0.101375498952004,0.101193730594653,0.101213265518358,\n 0.101165178275411,0.101149604926044,0.101114528367103,\n 0.101177617463096,0.101176196339295,0.10102421433221,\n 0.10102421433221,0.100910703206783,0.101046445558438,\n 0.12265509469116,0.10486856029419,0.103267024648877,0.100867566762125,\n 0.11467836793049,0.10790947678297,0.100997454994136,\n 0.101684644439116,0.103275688295495,0.101358559584742,\n 0.101485340026678,0.131050876787633,0.12672527384678,\n 0.0911987727585976,0.0911987727585976,0.0473284627336248,\n 0.120947206065826,0.120594420271743,0.120594420271743,\n 0.120207965256539,0.12000489747134,0.119965054342127,\n 0.0890575546896603,0.0468935746714197,\n 0.0890043416222531,0.119198789457309,0.0466102635462743,\n 0.119011209638725,0.0883056120686039,0.0883053113523379,\n 0.0881390968551948,0.0880862640207957,0.118593197976584,\n 0.0877953030586674,0.118192523540133,0.0462081105569776,\n 0.0461972348759043),\n country = c(\"metaanalysis\",\"metaanalysis\",\n \"metaanalysis\",\"metaanalysis\",\"metaanalysis\",\n \"metaanalysis\",\"metaanalysis\",\"metaanalysis\",\"metaanalysis\",\n \"metaanalysis\",\"metaanalysis\",\"metaanalysis\",\n \"metaanalysis\",\"metaanalysis\",\"metaanalysis\",\"metaanalysis\",\n \"metaanalysis\",\"metaanalysis\",\"metaanalysis\",\n \"metaanalysis\",\"metaanalysis\",\"metaanalysis\",\"metaanalysis\",\n \"metaanalysis\",\"metaanalysis\",\"Swedish\",\"Kazakh\",\"Kazakh\",\n \"Japanese\",\"Swedish\",\"Swedish\",\"Swedish\",\"Swedish\",\n \"Swedish\",\"Swedish\",\"Kazakh\",\"Japanese\",\"Kazakh\",\n \"Swedish\",\"Japanese\",\"Swedish\",\"Kazakh\",\"Kazakh\",\n \"Kazakh\",\"Kazakh\",\"Swedish\",\"Kazakh\",\"Swedish\",\"Japanese\",\n \"Japanese\")\n)\n\nOutput:\n\nThanks\n", "answer": "A quick look at the code for forestplot shows that geom_effect is what needs to be modified. If you add a value for fatten changes the size of the shapes in the the figure (but not in the legend), or a value for size will change the shape, line and legend sizes (add to not geom_effect not to the aes layer)\nThis may be easier said than done as forestplot also seems to call an unexposed function (function quo_is_null). A couple of simple hacks are possible; copy and make your own forestplot in the default environment and remove the call to the function (it seems to just check if pvalue is set), or change the environment of the \"new\" forestplot to ggforestplot\nenvorionment(forestplot) <- envornment(ggforestplot::forestplot)\n\nThere may be other less awkward methods, but this will work. Fatten changed to size 6 which is massive.\nforestplot <- \nfunction (df, name = name, estimate = estimate, se = se, pvalue = NULL, \n colour = NULL, shape = NULL, logodds = FALSE, psignif = 0.05, \n ci = 0.95, ...) \n{\n stopifnot(is.data.frame(df))\n stopifnot(is.logical(logodds))\n name <- enquo(name)\n estimate <- enquo(estimate)\n se <- enquo(se)\n pvalue <- enquo(pvalue)\n colour <- enquo(colour)\n shape <- enquo(shape)\n args <- list(...)\n const <- stats::qnorm(1 - (1 - ci)/2)\n df <- df %>% dplyr::mutate(`:=`(!!name, factor(!!name, levels = !!name %>% \n unique() %>% rev(), ordered = TRUE)), .xmin = !!estimate - \n const * !!se, .xmax = !!estimate + const * !!se, .filled = TRUE, \n .label = sprintf(\"%.2f\", !!estimate))\n if (logodds) {\n df <- df %>% mutate(.xmin = exp(.data$.xmin), .xmax = exp(.data$.xmax), \n `:=`(!!estimate, exp(!!estimate)))\n }\n if (!quo_is_null(pvalue)) {\n df <- df %>% dplyr::mutate(.filled = !!pvalue < !!psignif)\n }\n g <- ggplot2::ggplot(df, aes(x = !!estimate, y = !!name))\n if (logodds) {\n if (\"xtickbreaks\" %in% names(args)) {\n g <- g + scale_x_continuous(trans = \"log10\", breaks = args$xtickbreaks)\n }\n else {\n g <- g + scale_x_continuous(trans = \"log10\", breaks = scales::log_breaks(n = 7))\n }\n }\n g <- g + theme_forest() + scale_colour_ng_d() + scale_fill_ng_d() + \n geom_stripes() + geom_vline(xintercept = ifelse(test = logodds, \n yes = 1, no = 0), linetype = \"solid\", size = 0.4, colour = \"black\")\n g <- g + geom_effect(ggplot2::aes(xmin = .data$.xmin, xmax = .data$.xmax, \n colour = !!colour, shape = !!shape, filled = .data$.filled), \n position = ggstance::position_dodgev(height = 0.5),\n fatten=6) + \n ggplot2::scale_shape_manual(values = c(21L, 22L, 23L, \n 24L, 25L)) + guides(colour = guide_legend(reverse = TRUE), \n shape = guide_legend(reverse = TRUE))\n if (\"title\" %in% names(args)) {\n g <- g + labs(title = args$title)\n }\n if (\"subtitle\" %in% names(args)) {\n g <- g + labs(subtitle = args$subtitle)\n }\n if (\"caption\" %in% names(args)) {\n g <- g + labs(caption = args$caption)\n }\n if (\"xlab\" %in% names(args)) {\n g <- g + labs(x = args$xlab)\n }\n if (!\"ylab\" %in% names(args)) {\n args$ylab <- \"\"\n }\n g <- g + labs(y = args$ylab)\n if (\"xlim\" %in% names(args)) {\n g <- g + coord_cartesian(xlim = args$xlim)\n }\n if (\"ylim\" %in% names(args)) {\n g <- g + ylim(args$ylim)\n }\n if (\"xtickbreaks\" %in% names(args) & !logodds) {\n g <- g + scale_x_continuous(breaks = args$xtickbreaks)\n }\n g\n}\nenvironment(forestplot) <- environment(ggforestplot::forestplot)\n\n", "topic": "bioinformatics", "url": "https://bioinformatics.stackexchange.com/questions/15804"}, {"image": "VQAonline_00003856.png", "question": "Identify side chain atoms in BioPandas dataframe", "context": "I am currently implementing some metrics I could use for comparing two conformations of the same protein in Python.\nFor example, I know I could use the RMSD of all protein ATOMS in BioPandas using this:\nbiopandas.pdb.PandasPdb.rmsd(conformation1.df['ATOM'], conformation2.df['ATOM'])\n\nAre there some out-of-the-box tools in Python for measuring the RMSD between the side chains of the two conformations? When trying to do this manually, the main issue is how to identify side chain atoms, which is not clear for me. I know how to get all atoms from a residue using BioPandas:\naminoacids = list(PandasPdb().read_pdb(my_pdb_filename).df['ATOM'].groupby(['chain_id', 'residue_number', 'residue_name']))\n\nNow aminoacids[i] is a dataframe containing all the atoms of the i-th aminoacid, like this:\n\nFrom here, it's not clear how to separate back bone atoms from side chain atoms. It's clear that the one with atom_number 'CA' will be the alpha-Carbon, but I am not sure how to identify the others. But once the separation is done, the side chain RMSD could be measure for example using Bio.SVDSuperimposer.SVDSuperimposer which works directly on the array of 3D coordinates of the atoms.\nThanks!\n", "answer": "As you suggest one way of solving your problem would be by selecting all atoms that don't have backbone atoms names. In a pdb file I believe backbone atoms would be named 'CA', 'HA', 'N', 'HN' or 'H', 'C' and 'O'. Beware of the N-terminal (where the hydrogens would be named either 'H1', 'H2' & 'H3'; or 'HN1', 'HN2' and 'HN3') and the C-terminal (the other oxygen would be named 'OXT').\nOne way of doing it is by using the handy selections from pandas dataframes. It's probably a more elegant way than making a list of dataframes.\nfrom biopandas.pdb import PandasPdb\n\nbpdf_1 = PandasPdb().fetch_pdb('1t48').df['ATOM']\nbpdf_2 = PandasPdb().fetch_pdb('1t49').df['ATOM']\n\n# Function returns a list of boolean. True when it's not a backbone atom.\nsel = lambda df: df['atom_name'].str.contains('[^(CA|HA|N|C|O|HN|H)]$')\n\n# The number of atom is not the same, so I only align the first 200\nsidechain_rmsd = PandasPdb.rmsd(bpdf_1[sel][:200], bpdf_2[sel][:200])\n\nThe regex says something like \"(not (CA or HA or \u2026)) end of line\". The end of line is important, otherwise any atom containing N, C, O or H in their name would be excluded (so most of them).\nNote that with this method it's still possible to use the PandasPdb selectors. For example, to only consider heavy atoms (no hydrogen):\nsidechain_rmsd = PandasPdb.rmsd(bpdf_1[sel][:200],\n bpdf_2[sel][:200],\n s='heavy')\n\nEDIT:\nAnother way to do it would be:\nsidechain_rmsd = PandasPdb.rmsd(bpdf_1[:200],\n bpdf_2[:200],\n s='main chain',\n invert=True)\n\nYou don't need any selection in this case, BUT this is a bad idea if you have hydrogens, as it will also align them (including the backbone ones).\n", "topic": "bioinformatics", "url": "https://bioinformatics.stackexchange.com/questions/18690"}, {"image": "VQAonline_00003846.png", "question": "GROMACS RMSF Result Analysis Problem", "context": "This is RMSF for protein from GROMACS after 100ns simulation-\n\nI have few questions-\n\nResidue numbers are starting from 1650 to 1850, has it ignore LIG\nand HOH as residues?\nThere are three lines, so are there three chains and their residue number is same?\nAre the straight lines connecting these chains or this is kind of loop something?\nSince I am seeing 2nm or 20A fluctuation, is the protein quite disturbed?\n\n", "answer": "\nThe residue numbering in a PDB file mainly depend on the person who deposited it. There are usually reasons for how things are numbered, but ultimately you should look at PDB file, and check the details on how the structure was obtained. To give you an example, during my PhD I worked on a protein that had some missing residues at the N-terminus. That was because the N-term was very unstable and they didn't manage to crystallise it properly.\nYou need to check the PDB, but it is possible that the 3 chains have the same residue numbering (A1-200, B1-200). This is common in structures where you have multiple identical sub-units (e.g. dimers).\nThe lines are merely an artefact. Those lines are the attempt of the engine to draw a single line for all the chains. You go from residue 1850 of chain 1 to residue 1650 of chain 2, and so on. Some tools handle this behaviour, others don't. I would suggest you to parse the PDB yourself and plot (e.g. ggplot) each chain as a layer of the same figures .\nIt is indeed, but only you can tell if this is good or bad. It really depends on how you calculate the RMSF. Was it on the backbone or on the side-chains? How is your dynamics? High temperature? Does you protein partially unfold? are the chains separating?\n\n", "topic": "bioinformatics", "url": "https://bioinformatics.stackexchange.com/questions/17602"}, {"image": "VQAonline_00003788.png", "question": "Highly heterozygous reads mapping", "context": "I have short (67bp) Hi-C reads from a highly heterozygous organism (~15% between-haplotype SNP divergence) and I have both reference haplotypes.\nI wanted to try and compare different haplotyping softwares for Hi-C reads using these reads as benchmarking dataset. When mapping the reads separately to each haplotype, I get good mapping statistics. When I map the reads to a single haplotype with all the heterozygous SNPs masked (into N's), I get very poor mapping rates.\nI would like to be able to map the reads when the real haplotypes are unknown (reference is a mix of haplotypes).\nI use minimap2 to map the reads with the sr preset. I tried decreasing the mismatch penalty (-B) to 1 and increasing the --score-N value, but this had no effect.\nAs shown in the attached IGV screenshot, coverage drops to 0 when the SNP density increases.\nIs it feasible to map reads with such a high heterozygosity on a single (masked) reference ? Should I use another tool ?\n", "answer": "I believe you may be able to map your reads, but I don't know how to do that with minimap2. \nI recommend running gsnap, which is more SNP tolerant and provides a number of parameters which are likely to help.\nFor instance, I believe that most aligners will treat 'N' characters as mismatches when you align. GSNAP has parameters to account for this. \n--query-unk-mismatch=INT Whether to count unknown (N) characters in the query as a mismatch\n (0=no (default), 1=yes)\n--genome-unk-mismatch=INT Whether to count unknown (N) characters in the genome as a mismatch\n (0=no, 1=yes (default))\n\nIt also has a mismatch parameter similar to the one you described for minimap2.\n-m, --max-mismatches=FLOAT Maximum number of mismatches allowed\n\nTry running with the 'genome-unk-mismatch' parameter above (with the masked reference). That may be your best bet. There are also other parameters that may help, but this should be a good start.\n", "topic": "bioinformatics", "url": "https://bioinformatics.stackexchange.com/questions/6791"}, {"image": "VQAonline_00003867.png", "question": "reverse translation from amino acid string to DNA strings", "context": "what is the opposite of .translate() function calls ?\nI mean let's say I am given an amino acid string CYCLIC, how do I obtain all the possible combinations of DNA strings before the translation process ?\nNote: I was told that degeneracy has an important role in this particular aspect, but I am not entirely sure about the degeneracy calculation in the following screenshot highlighted in yellow color.\n\nThe degenerate code is as follows:\ntable = {\n 'ATA':'I', 'ATC':'I', 'ATT':'I', 'ATG':'M',\n 'ACA':'T', 'ACC':'T', 'ACG':'T', 'ACT':'T',\n 'AAC':'N', 'AAT':'N', 'AAA':'K', 'AAG':'K',\n 'AGC':'S', 'AGT':'S', 'AGA':'R', 'AGG':'R', \n 'CTA':'L', 'CTC':'L', 'CTG':'L', 'CTT':'L',\n 'CCA':'P', 'CCC':'P', 'CCG':'P', 'CCT':'P',\n 'CAC':'H', 'CAT':'H', 'CAA':'Q', 'CAG':'Q',\n 'CGA':'R', 'CGC':'R', 'CGG':'R', 'CGT':'R',\n 'GTA':'V', 'GTC':'V', 'GTG':'V', 'GTT':'V',\n 'GCA':'A', 'GCC':'A', 'GCG':'A', 'GCT':'A',\n 'GAC':'D', 'GAT':'D', 'GAA':'E', 'GAG':'E',\n 'GGA':'G', 'GGC':'G', 'GGG':'G', 'GGT':'G',\n 'TCA':'S', 'TCC':'S', 'TCG':'S', 'TCT':'S',\n 'TTC':'F', 'TTT':'F', 'TTA':'L', 'TTG':'L',\n 'TAC':'Y', 'TAT':'Y', 'TAA':'_', 'TAG':'_',\n 'TGC':'C', 'TGT':'C', 'TGA':'_', 'TGG':'W',\n }\n\n", "answer": "Given the table above the solution for CYCLE is:\nfrom itertools import product\n\nprot = 'CYCLE'\ndegenDict = dict()\nfor v,k in table.items():\n if k in degenDict:\n degenDict[k].append(v)\n else:\n degenDict[k]=[]\n degenDict[k].append(v)\nnucs = [degenDict[resid] for resid in prot]\nfor degenNuc in product(*nucs):\n print(''.join(degenNuc))\n\n\nTGCTACTGCCTAGAA\nTGCTACTGCCTAGAG\nTGCTACTGCCTCGAA\nTGCTACTGCCTCGAG\nTGCTACTGCCTGGAA\nTGCTACTGCCTGGAG\nTGCTACTGCCTTGAA\nTGCTACTGCCTTGAG\nTGCTACTGCTTAGAA\nTGCTACTGCTTAGAG\nTGCTACTGCTTGGAA\nTGCTACTGCTTGGAG\nTGCTACTGTCTAGAA\nTGCTACTGTCTAGAG\nTGCTACTGTCTCGAA\nTGCTACTGTCTCGAG\nTGCTACTGTCTGGAA\nTGCTACTGTCTGGAG\nTGCTACTGTCTTGAA\nTGCTACTGTCTTGAG\nTGCTACTGTTTAGAA\nTGCTACTGTTTAGAG\nTGCTACTGTTTGGAA\nTGCTACTGTTTGGAG\nTGCTATTGCCTAGAA\nTGCTATTGCCTAGAG\nTGCTATTGCCTCGAA\nTGCTATTGCCTCGAG\nTGCTATTGCCTGGAA\nTGCTATTGCCTGGAG\nTGCTATTGCCTTGAA\nTGCTATTGCCTTGAG\nTGCTATTGCTTAGAA\nTGCTATTGCTTAGAG\nTGCTATTGCTTGGAA\nTGCTATTGCTTGGAG\nTGCTATTGTCTAGAA\nTGCTATTGTCTAGAG\nTGCTATTGTCTCGAA\nTGCTATTGTCTCGAG\nTGCTATTGTCTGGAA\nTGCTATTGTCTGGAG\nTGCTATTGTCTTGAA\nTGCTATTGTCTTGAG\nTGCTATTGTTTAGAA\nTGCTATTGTTTAGAG\nTGCTATTGTTTGGAA\nTGCTATTGTTTGGAG\nTGTTACTGCCTAGAA\nTGTTACTGCCTAGAG\nTGTTACTGCCTCGAA\nTGTTACTGCCTCGAG\nTGTTACTGCCTGGAA\nTGTTACTGCCTGGAG\nTGTTACTGCCTTGAA\nTGTTACTGCCTTGAG\nTGTTACTGCTTAGAA\nTGTTACTGCTTAGAG\nTGTTACTGCTTGGAA\nTGTTACTGCTTGGAG\nTGTTACTGTCTAGAA\nTGTTACTGTCTAGAG\nTGTTACTGTCTCGAA\nTGTTACTGTCTCGAG\nTGTTACTGTCTGGAA\nTGTTACTGTCTGGAG\nTGTTACTGTCTTGAA\nTGTTACTGTCTTGAG\nTGTTACTGTTTAGAA\nTGTTACTGTTTAGAG\nTGTTACTGTTTGGAA\nTGTTACTGTTTGGAG\nTGTTATTGCCTAGAA\nTGTTATTGCCTAGAG\nTGTTATTGCCTCGAA\nTGTTATTGCCTCGAG\nTGTTATTGCCTGGAA\nTGTTATTGCCTGGAG\nTGTTATTGCCTTGAA\nTGTTATTGCCTTGAG\nTGTTATTGCTTAGAA\nTGTTATTGCTTAGAG\nTGTTATTGCTTGGAA\nTGTTATTGCTTGGAG\nTGTTATTGTCTAGAA\nTGTTATTGTCTAGAG\nTGTTATTGTCTCGAA\nTGTTATTGTCTCGAG\nTGTTATTGTCTGGAA\nTGTTATTGTCTGGAG\nTGTTATTGTCTTGAA\nTGTTATTGTCTTGAG\nTGTTATTGTTTAGAA\nTGTTATTGTTTAGAG\nTGTTATTGTTTGGAA\nTGTTATTGTTTGGAG\n\n", "topic": "bioinformatics", "url": "https://bioinformatics.stackexchange.com/questions/19498"}, {"image": "VQAonline_00003831.png", "question": "TCGA: gene IDs not appearing and other concerns", "context": "I want to download RNA-seq datasets of 4 or 5 different types of cancer from the TCGA to investigate what is happening with my gene of interest. The problem is that I'm a physician and I don't know people used to deal with Big Data to help me.\nI'm using a Bioconductor program called TCGAbiolinks and I'm able to download some datasets using the following script:\nif (!requireNamespace(\"BiocManager\", quietly = TRUE))\n install.packages(\"BiocManager\")\n\nBiocManager::install(\"TCGAbiolinks\")\n\nbrowseVignettes(\"TCGAbiolinks\")\n\nlibrary(TCGAbiolinks)\nlibrary(dplyr)\nlibrary(DT)\n\nquery <- GDCquery(project = \"TCGA-BRCA\", \n data.category = \"Gene expression\",\n file.type = \"normalized_results\",\n data.type = \"Gene expression quantification\",\n experimental.strategy = \"RNA-Seq\", \n barcode = c(\"TCGA-AN-A046\", \"TCGA-AC-A23H\"),\n legacy = TRUE)\n\nGDCdownload(query, method = \"api\", files.per.chunk = 10)\n\n# GDCprepare transforms data in R format\n\n# The default GDCprepare output is a SummarizedExperiment (SE) object,\n# but you can also set to output the data frame.\n\ndata <- GDCprepare(query, summarizedExperiment = FALSE)\n\n\nThe problems is that: 1) the gene list is not appearing in the first column; 2) I don't know how many datasets I'll be able to download and analyse without a server.. I'm working with my Macbook..\nCan someone familiar with the TCGA website or with the TCGAbiolinks help me with this project? I've been trying to progress in the last months, but the TCGA is a complex website and I'm feeling lost. I use R, I don't know SQL or other programming languages..\n", "answer": "I had to play around a little to understand the object, but here's what I got:\nR version 4.0.2 (2020-06-22) -- \"Taking Off Again\"\nCopyright (C) 2020 The R Foundation for Statistical Computing\nPlatform: x86_64-apple-darwin17.0 (64-bit)\n\nR is free software and comes with ABSOLUTELY NO WARRANTY.\nYou are welcome to redistribute it under certain conditions.\nType 'license()' or 'licence()' for distribution details.\n\n Natural language support but running in an English locale\n\nR is a collaborative project with many contributors.\nType 'contributors()' for more information and\n'citation()' on how to cite R or R packages in publications.\n\nType 'demo()' for some demos, 'help()' for on-line help, or\n'help.start()' for an HTML browser interface to help.\nType 'q()' to quit R.\n\n> library(TCGAbiolinks)\n> query <- GDCquery(project = \"TCGA-BRCA\",\n+ data.category = \"Gene expression\",\n+ file.type = \"normalized_results\",\n+ data.type = \"Gene expression quantification\",\n+ experimental.strategy = \"RNA-Seq\",\n+ barcode = c(\"TCGA-AN-A046\", \"TCGA-AC-A23H\"),\n+ legacy = TRUE)\n--------------------------------------\no GDCquery: Searching in GDC database\n--------------------------------------\nGenome of reference: hg19\n--------------------------------------------\noo Accessing GDC. This might take a while...\n--------------------------------------------\nooo Project: TCGA-BRCA\n--------------------\noo Filtering results\n--------------------\nooo By experimental.strategy\nooo By data.type\nooo By file.type\nooo By barcode\n----------------\noo Checking data\n----------------\nooo Check if there are duplicated cases\nooo Check if there results for the query\n-------------------\no Preparing output\n-------------------\n\n\n> GDCdownload(query, method = \"api\", files.per.chunk = 10)\nDownloading data for project TCGA-BRCA\nGDCdownload will download 3 files. A total of 1.306357 MB\nDownloading chunk 1 of 1 (3 files, size = 1.306357 MB) as Wed_Aug_19_12_20_11_2020_0.tar.gz\n\nDownloading: 580 kB\n\n\n> data <- GDCprepare(query, summarizedExperiment = TRUE)\n------------------\noo Reading 3 files\n------------------\n|====================================================|100% Completed after 0 s\n------------------\noo Merging 3 files\n------------------\nAccessing grch37.ensembl.org to get gene information\nDownloading genome information (try:0) Using: Human genes (GRCh37.p13)\nStarting to add information to samples\n => Add clinical information to samples\n => Adding TCGA molecular information from marker papers\n => Information will have prefix 'paper_'\nbrca subtype information from:doi.org/10.1016/j.ccell.2018.03.014\n\n\n> head(data)\nclass: RangedSummarizedExperiment\ndim: 6 3\nmetadata(1): data_release\nassays(1): normalized_count\nrownames(6): A1BG A2M ... RP11-986E7.7 AADAC\nrowData names(3): gene_id entrezgene ensembl_gene_id\ncolnames(3): TCGA-AN-A046-01A-21R-A034-07 TCGA-AC-A23H-11A-12R-A157-07\n TCGA-AC-A23H-01A-11R-A157-07\ncolData names(83): barcode patient ... paper_PARADIGM Clusters\n paper_Pan-Gyn Clusters\n\nI see from this representation that I can use metadata(data), assays(data), etc to get more information.\n> head(SummarizedExperiment::assays(data))\nList of length 1\nnames(1): normalized_count\n\nAlright, so this is a named list. I know how to deal with lists: I can use either the index ([[1]]) or the name ($normalized_count)\n> head(SummarizedExperiment::assays(data)$normalized_count)\n TCGA-AN-A046-01A-21R-A034-07 TCGA-AC-A23H-11A-12R-A157-07 TCGA-AC-A23H-01A-11R-A157-07\nA1BG 121.9879 38.5795 13.6874\nA2M 9703.8622 37640.8373 4767.0490\nNAT1 8488.0727 114.8582 270.4753\nNAT2 6.0583 4.2938 2.3804\nRP11-986E7.7 33016.6641 11308.3460 3894.9639\nAADAC 1.1359 219.6976 1.1902\n\nI can now get more information on the samples using colData(). I'm truncating the output below because there are 83 columns.\n> SummarizedExperiment::colData(data)\nDataFrame with 3 rows and 83 columns\n barcode patient sample shortLetterCode definition sample_submitter_id sample_type state\n \nTCGA-AN-A046-01A-21R-A034-07 TCGA-AN-A046-01A-21R-A034-07 TCGA-AN-A046 TCGA-AN-A046-01A TP Primary solid Tumor TCGA-AN-A046-01A Primary Tumor released\nTCGA-AC-A23H-11A-12R-A157-07 TCGA-AC-A23H-11A-12R-A157-07 TCGA-AC-A23H TCGA-AC-A23H-11A NT Solid Tissue Normal TCGA-AC-A23H-11A Solid Tissue Normal released\nTCGA-AC-A23H-01A-11R-A157-07 TCGA-AC-A23H-01A-11R-A157-07 TCGA-AC-A23H TCGA-AC-A23H-01A TP Primary solid Tumor TCGA-AC-A23H-01A Primary Tumor released\n sample_id is_ffpe sample_type_id tissue_type submitter_id days_to_collection oct_embedded pathology_report_uuid\n \nTCGA-AN-A046-01A-21R-A034-07 57aff09f-0e97-4e65-a227-3dc7a8516367 FALSE 01 Not Reported TCGA-AN-A046 27 true 1304FB17-A20A-4EBC-9CC8-1554808AC1F6\nTCGA-AC-A23H-11A-12R-A157-07 7df59ca8-7e51-4581-9d7f-8bba0395ce17 FALSE 11 Not Reported TCGA-AC-A23H 478 false NA\nTCGA-AC-A23H-01A-11R-A157-07 d7e3b628-d5fd-4e79-9c4a-6409330fb8a7 FALSE 01 Not Reported TCGA-AC-A23H 478 false A7C7D409-D086-4A9B-8C8F-E7E231D5891D\n\nI can get more information on the genes using rowData():\n> SummarizedExperiment::rowData(data)\nDataFrame with 19947 rows and 3 columns\n gene_id entrezgene ensembl_gene_id\n \nA1BG A1BG 1 ENSG00000121410\nA2M A2M 2 ENSG00000175899\nNAT1 NAT1 9 ENSG00000171428\nNAT2 NAT2 10 ENSG00000156006\nRP11-986E7.7 RP11-986E7.7 12 ENSG00000273259\n... ... ... ...\nRASAL2-AS1 RASAL2-AS1 100302401 ENSG00000224687\nLINC00882 LINC00882 100302640 ENSG00000242759\nFTX FTX 100302692 ENSG00000230590\nTICAM2 TICAM2 100302736 ENSG00000243414\nSLC25A5-AS1 SLC25A5-AS1 100303728 ENSG00000224281\n\nYou want the assay data as a data.matrix, I guess. That'd be:\n> data_as_matrix <- data.matrix(SummarizedExperiment::assays(data)$normalized_count)\n\nMy session Info:\n> sessionInfo()\nR version 4.0.2 (2020-06-22)\nPlatform: x86_64-apple-darwin17.0 (64-bit)\nRunning under: macOS Catalina 10.15.5\n\nMatrix products: default\nBLAS: /Library/Frameworks/R.framework/Versions/4.0/Resources/lib/libRblas.dylib\nLAPACK: /Library/Frameworks/R.framework/Versions/4.0/Resources/lib/libRlapack.dylib\n\nlocale:\n[1] en_US.UTF-8/en_US.UTF-8/en_US.UTF-8/C/en_US.UTF-8/en_US.UTF-8\n\nattached base packages:\n[1] stats graphics grDevices utils datasets methods base\n\nother attached packages:\n[1] TCGAbiolinks_2.16.1\n\nloaded via a namespace (and not attached):\n [1] bitops_1.0-6 matrixStats_0.56.0 bit64_0.9-7.1 doParallel_1.0.15 RColorBrewer_1.1-2 progress_1.2.2\n [7] httr_1.4.2 GenomeInfoDb_1.24.2 backports_1.1.8 tools_4.0.2 R6_2.4.1 DBI_1.1.0\n [13] BiocGenerics_0.34.0 colorspace_1.4-1 tidyselect_1.1.0 gridExtra_2.3 prettyunits_1.1.1 bit_1.1-15.2\n [19] curl_4.3 compiler_4.0.2 rvest_0.3.6 Biobase_2.48.0 xml2_1.3.2 DelayedArray_0.14.0\n [25] rtracklayer_1.48.0 scales_1.1.1 survMisc_0.5.5 readr_1.3.1 genefilter_1.70.0 askpass_1.1\n [31] rappdirs_0.3.1 stringr_1.4.0 digest_0.6.25 Rsamtools_2.4.0 foreign_0.8-80 R.utils_2.9.2\n [37] rio_0.5.16 XVector_0.28.0 pkgconfig_2.0.3 dbplyr_1.4.4 readxl_1.3.1 rlang_0.4.7\n [43] ggthemes_4.2.0 RSQLite_2.2.0 generics_0.0.2 zoo_1.8-8 jsonlite_1.7.0 BiocParallel_1.22.0\n [49] zip_2.0.4 car_3.0-8 dplyr_1.0.0 R.oo_1.23.0 RCurl_1.98-1.2 magrittr_1.5\n [55] GenomeInfoDbData_1.2.3 Matrix_1.2-18 Rcpp_1.0.5 munsell_0.5.0 S4Vectors_0.26.1 abind_1.4-5\n [61] lifecycle_0.2.0 R.methodsS3_1.8.0 stringi_1.4.6 carData_3.0-4 SummarizedExperiment_1.18.1 zlibbioc_1.34.0\n [67] plyr_1.8.6 BiocFileCache_1.12.0 grid_4.0.2 blob_1.2.1 parallel_4.0.2 ggrepel_0.8.2\n [73] forcats_0.5.0 crayon_1.3.4 survminer_0.4.8 lattice_0.20-41 haven_2.3.1 Biostrings_2.56.0\n [79] splines_4.0.2 GenomicFeatures_1.40.0 annotate_1.66.0 hms_0.5.3 knitr_1.29 pillar_1.4.6\n [85] ggpubr_0.4.0 GenomicRanges_1.40.0 ggsignif_0.6.0 codetools_0.2-16 biomaRt_2.44.1 stats4_4.0.2\n [91] XML_3.99-0.5 glue_1.4.1 downloader_0.4 data.table_1.13.0 vctrs_0.3.2 selectr_0.4-2\n [97] foreach_1.5.0 cellranger_1.1.0 gtable_0.3.0 openssl_1.4.2 purrr_0.3.4 tidyr_1.1.0\n[103] km.ci_0.5-2 assertthat_0.2.1 ggplot2_3.3.2 openxlsx_4.1.5 xfun_0.16 xtable_1.8-4\n[109] broom_0.7.0 rstatix_0.6.0 survival_3.1-12 tibble_3.0.3 iterators_1.0.12 KMsurv_0.1-5\n[115] GenomicAlignments_1.24.0 AnnotationDbi_1.50.1 memoise_1.1.0 IRanges_2.22.2 ellipsis_0.3.1\n\n", "topic": "bioinformatics", "url": "https://bioinformatics.stackexchange.com/questions/14166"}, {"image": "VQAonline_00003791.png", "question": "wget for links inside html pages", "context": "I am trying to download a file from the following repository: https://trace.ncbi.nlm.nih.gov/Traces/sra/?run=SRR7276474 \nAs you can see, there are several layers to the webpage. For example, clicking on the download tab doesn't change the URL and, the link is not a 'download link' per ce - where simply clicking the link automates a download. I have tried some of the answers on the forum, where they have advised using quotes and the operation: \nwget \"url/?target=link\". This however does not work in the following instance.\nBasically, I want to download the file labeled 'P1TLH.bam', in the download tab of the link provided:\n\n", "answer": "If you want to download a file, you need to use the link to that file. Your original attempt, wget https://trace.ncbi.nlm.nih.gov/Traces/sra/?run=SRR7276474 wouldn't work since that's a link to the trace page of the relevant run. If you want to download something else, just right click on the link (the one in the screenshot in your question), copy the URL and use that:\nwget https://sra-download.ncbi.nlm.nih.gov/traces/sra64/SRZ/007276/SRR7276474/P1TLH.bam\n\n", "topic": "bioinformatics", "url": "https://bioinformatics.stackexchange.com/questions/6949"}, {"image": "VQAonline_00004314.png", "question": "What causes \"ear rumbling\"?", "context": "The video Ear rumbling happens below the range of human hearing demonstrates that the \"ear rumbling\" sound is actual sound and can be recorded by a sensitive microphone near the ear.\nWhat is it exactly that produces this vibration and how is it radiated as sound waves into the air?\nIf it is subsonic, then why does it sounds like noise rather than being inaudible?\n\n\n", "answer": "I'm able to create rumbling in my ears at will. Unlike the poster of the video, I don't need to yawn to do so; I can do it without my face appearing to move. I actually discovered this alongside another ability of mine when I was little: the ability to create warmth flowing from the base of my neck outwards into the rest of my body. After some research I figured out that the warmth I'm able to create is actually part of voluntary piloerection (its use for warmth is mentioned here), while the ability to create a rumbling sound is simply the contraction of the tensor tympani, a muscle in the ear. Voluntary control over either one of these things is supposed to be rare, but I've found claims of several other people online who can do both (example1, example2).\nAnyways, Wikipedia gives a brief explanation of how the ear rumbling works, which also specifically mentions \"yawning deeply\" as a trigger:\n\nContracting muscles produce vibration and sound. Slow twitch fibers produce 10 to 30 contractions per second (equivalent to 10 to 30 Hz sound frequency). Fast twitch fibers produce 30 to 70 contractions per second (equivalent to 30 to 70 Hz sound frequency). The vibration can be witnessed and felt by highly tensing one's muscles, as when making a firm fist. The sound can be heard by pressing a highly tensed muscle against the ear, again a firm fist is a good example. The sound is usually described as a rumbling sound.\nSome individuals can voluntarily produce this rumbling sound by contracting the tensor tympani muscle of the middle ear. The rumbling sound can also be heard when the neck or jaw muscles are highly tensed as when yawning deeply. This phenomenon has been known since (at least) 1884.\n\nIt also seems like this type of noise can happen with conditions such as Tonic Tensor Tympani Syndrome.\nThe tensor tympani connects directly to the handle of the malleus, so I'd assume that's why it can be heard by the person doing it. Apparently this also causes the eardrum to move. This means it's classified as objective tinnitus.\nHere are some relevant studies I found:\n\nVoluntary contraction of the tensor tympani muscle\nand its audiometric effects\nVoluntary Eardrum Movement : A Marker for Tensor Tympani Contraction? (paywall)\nTonic contractions of the tensor tympani muscle: a key to some\nnon-specific middle ear symptoms? Hypothesis and data from\ntemporal bone experiments (paywall)\n\n", "topic": "biology", "url": "https://biology.stackexchange.com/questions/80419"}, {"image": "VQAonline_00004255.png", "question": "A strange insect", "context": "I saw this ant-like insect in Brazil, close to Rio de Janeiro. It was around 2 cm long. I tried to use google's search by image, but no luck. Does any one know the name of the species? \n\n", "answer": "Insects like this are commonly called \"velvet ants\", but they are wasps, not ants. This insect is in the family Mutillidae, and it's called Hopolocrates cephalotes. \n(https://www.inaturalist.org/taxa/629302-Hoplocrates-cephalotes/browse_photos)\n", "topic": "biology", "url": "https://biology.stackexchange.com/questions/71526"}, {"image": "VQAonline_00004155.png", "question": "How close genetically is the most human-like chimpanzee to the most chimp-like human?", "context": "I understand that:\n\nChimpanzees are the closest species to humans genetically. Only 1%-6% of their genes are different.\nWithin any species there is genetic diversity, i.e. no two individuals have the same exact DNA sequence.\nThis variability applies to humans and chimps. \nThus, there exists a pair consisting of a human and a chimp that will have the smallest number of different (edit: genes) DNA base-pairs within the two populations. One can say that the pair forms an \"inter-species genetic gap\".\n\nQuestion: What is the smallest estimated inter-species genetic gap between humans and chimpanzees?\nEdit: I changed the last point to base-pairs instead of genes. Most of the comments seem to suggest the population genetic variabilities are much, much smaller than the genetic distance between the populations. Visually, that looks something like this:\n\nIs this a fairly accurate picture of the human-chimp genetic distance?\n", "answer": "You may be interested in this 2005 Nature paper from the Chimpanzee Sequencing and Analysis Consortium: Initial sequence of the chimpanzee genome and comparison with the human genome. It breaks down the most common categories of genetic variation:\n\nSingle-nucleotide substitutions occur at a mean rate of 1.23% between copies of the human and chimpanzee genome, with 1.06% or less corresponding to fixed divergence between the species.\n\nSo if we accept their analysis, ~1% (it says \"or less\", but elsewhere in the paper they estimate a lower bound of ~0.96%) of the genome difference consists of fixed single-nucleotide differences between species.\n\nSmall insertions and deletions: On the basis of this analysis, we estimate that the human and chimpanzee genomes each contain 40\u201345 Mb of species-specific euchromatic sequence, and the indel differences between the genomes thus total ~90 Mb. This difference corresponds to ~3% of both genomes and dwarfs the 1.23% difference resulting from nucleotide substitutions; this confirms and extends several recent studies.\n\nSo another ~3% comes from small insertions and deletions, which gives around a ~4% fixed divergence between populations. This rough estimate of the fixed genetic distance between humans and chimpanzees is probably a reasonable estimate of the distance between the nearest human-chimpanzee pair.\nIf you also want to know how far apart the two furthest humans are, as @Remi.b mentioned, the populations we've sequenced don't span the whole diversity of our species, but it is likely much smaller. You could look at some of the HapMap or Human Genome Diversity Project papers. For reference, when Watson's genome was sequenced, they reported around 0.1% of the sequence to be different from the reference genome, but that isn't a particularly diverse comparison.\nYou may also be interested in this paper comparing the human, chimpanzee, and bonobo genomes: http://www.nature.com/nature/journal/v486/n7404/full/nature11128.html.\n", "topic": "biology", "url": "https://biology.stackexchange.com/questions/52088"}, {"image": "VQAonline_00004096.png", "question": "Coefficient of Inbreeding - implementation issue", "context": "The formula for the coefficient of inbreeding is as follows...\n\nI have a family tree going back 9 generations. Say I find a common ancestor X in the 4th generation on the mothers side and in the 5th generation on the fathers side. I can work out that constituent of F... but what do I do about all the ancestors of X (they will all also be common ancestors). Do I also work out their values and add them to F?... or do I ignore them? The descriptions of the COI I have seen do not make this clear.\n", "answer": "The $F_i$ term is the term that accounts for inbreeding of your common ancestor X. If that common ancestor is not inbred then the term is 0 and the calculation is a little easier. Using the example you have given:\n X \u2500\u252c\u2500 Y \n \u2502\n \u250c\u2500\u2500\u2534\u2500\u2500\u2510\nGGGF GGM\n \u2502 \u2502\nGGF \u2502 \n \u2502 GM\nGF \u2502\n \u2502 \u2502\n F \u2500\u252c\u2500 M \n \u2502\n Mick\n\nIn this case, the common ancestors are not inbred so the $F_i$ terms go away ($1+F_i = 1$). There are two paths of common ancestry (via X and Y), therefore the calculation of the two paths from the father (F) and mother (M) to X and Y is:\n$F_{Mick} = \\frac12^{n_{F,X}+n_{M,X}+1}(1+F_X)+\\frac12^{n_{F,Y}+n_{M,Y}+1}(1+F_Y) $\n$F_{Mick} = \\frac12^{4+3+1}(1+0)+\\frac12^{4+3+1}(1+0) $\n$F_{Mick} = \\frac{1}{256} + \\frac{1}{256} = 0.0078 = 0.78\\% $\n\nIn a more complex case, such as where ancestor X was inbred, $F_i$ cannot be ignored. In the following example, X's parents were first cousins:\n U \u2500\u252c\u2500 V\n \u2502\n \u250c\u2500\u2500\u2534\u2500\u2500\u2510\n XGF XGM \n \u2502 |\n XF\u2500\u252c\u2500XM\n \u2502 \n X \u2500\u252c\u2500 Y \n \u2502\n \u250c\u2500\u2500\u2534\u2500\u2500\u2510\n GGGF GGM\n \u2502 \u2502\n GGF \u2502 \n \u2502 GM\n GF \u2502\n \u2502 \u2502\n F \u2500\u252c\u2500 M \n \u2502\n Mick\n\nFor this, we must first calculate $F_X$ (i.e. how inbred X is):\n$F_X = \\frac12^{n_{XF,U}+n_{XM,U}+1}(1+F_U)+\\frac12^{n_{XF,V}+n_{XM,V}+1}(1+F_V) $\n$F_X = \\frac12^{2+2+1}(1+0)+\\frac12^{2+2+1}(1+0)$\n$F_X = \\frac{1}{32} + \\frac{1}{32} = 0.0625 = 6.25\\% $\nThen we can input this $F_X$ into the same calculation as done before for the simple case:\n$F_{Mick} = \\frac12^{n_{F,X}+n_{M,X}+1}(1+F_X)+\\frac12^{n_{F,Y}+n_{M,Y}+1}(1+F_Y) $\n$F_{Mick} = \\frac12^{4+3+1}(1+0.0625)+\\frac12^{4+3+1}(1+0)$\n$F_{Mick} = 0.00415 + \\frac{1}{256} = 0.0081 = 0.81\\% $\n\nA good explanation of these calculations with further examples can be found here: The Coefficient of Inbreeding (F) and its applications. The calculations can get quite complicated even with relatively simple inbreeding.\n", "topic": "biology", "url": "https://biology.stackexchange.com/questions/41024"}, {"image": "VQAonline_00004160.png", "question": "By just looking at an unlabeled picture, how do you know it is a protein?", "context": "I'm learning about proteins and need to understand this concept. What makes this colourful ball of squiggles identifiable? Why is it a protein, and not any other biomolecule (like DNA for example)? What are the colours showing? What are the variations in the shape of the image?\nBasically, the question is: What is this image trying to tell me about this protein?\n\n", "answer": "Shapes\nThis is a common way of illustrating a protein and is often called a \"protein cartoon\". There are other common ways of illustrating proteins.\nThis cartoon shows you where \u03b1 helices are (spirals) and where \u03b2 sheets are (arrows) which are important structural elements. The strings connecting those structural elements are flexible \"loop\" regions.\nIf you want to query this against other structures and see which protein it is, check out PhyreStorm. You'll need the PDB file, not just an image.\nColour\nThe colouring is also common. The N-terminus (the first amino acid) is coloured blue, and the colours go through the rainbow to red, the C-terminus (the last amino acid in the polypeptide chain).\n", "topic": "biology", "url": "https://biology.stackexchange.com/questions/52426"}, {"image": "VQAonline_00004422.png", "question": "How can SNP arrays be used to detect deletions within a gene?", "context": "I am reading a journal paper where the researchers are studying the effect of disease-causing mutations in the IL1RAPL1 gene. In the first figure of this paper, they show pedigrees of families where they have family members with the disease-causing mutations.\n\nFor the BMC family, they found using a SNP array that a deletion of ~200 kb between intron 5 and 6 of the IL1RAPL1 gene results in an in-frame deletion of exon 6.\nI am having trouble interpreting the graph in Figure D, which is the results of the SNP array analysis. Aren't SNP arrays used for detecting single nucleotide polymorphisms? I am confused because in this paper they are using a SNP array to detect a gene deletion.\nAny insights are appreciated.\n", "answer": "In the materials and methods of the paper, you will observe the following statement regarding the CNV (deletion) analysis:\n\nCNV analysis was performed using CNV-WebStore (38).\n\nThat citation points to this paper, a software package designed to infer CNVs such as deletions from array data.\nRecall that SNP arrays are essentially asking \"how much DNA from the sample is sticking to each of N probes against specific sequences\". The SNP is inferred by comparing how much DNA sticks to one allele vs the other allele on the array. If a specific sequence assayed by the array is simply missing, then you will observe a corresponding decrease in signal.\nThis is signified by panel D of the figure that you include, where signal in the pink region is dramatically lower.\n", "topic": "biology", "url": "https://biology.stackexchange.com/questions/107947"}, {"image": "VQAonline_00003977.png", "question": "Is there some genetic variance underlying music appreciation?", "context": "Is there any research done on the genetic variance for Music appreciation?\nIf not, why is there no genetic variance for this trait?\n\n", "answer": "In 2013, \nDr. Liisa Ukkola-Vuoti, University of Helsinki, Finland, done a detailed GWCNV(Genome-Wide Copy Number Variation) Analysis of certain group of people for musical creativity and aptitude.\nGenome-Wide Copy Number Variation Analysis in Extended Families and Unrelated Individuals Characterized for Musical Aptitude and Creativity in Music\n\nThey did a Genome Wide Copy Number Variations (CNVs) in five extended\npedigrees and in 172 unrelated subjects characterized for musical\naptitude and creative functions in music\nMuscial Aptitude is taken as Sum of Scores of Auditory structuring ability, Seashores test for pitch and for time. Along with data on creativity in music was surveyed using a web-based questionnaire.\nSeveral CNVRs containing genes that affect neurodevelopment, learning\nand memory were detected. \nA deletion at 5q31.1 covering the protocadherin-\u03b1 gene cluster (Pcdha 1-9) was found co-segregating with low music test scores (COMB) in both sample sets. Pcdha is involved in neural migration, differentiation and synaptogenesis.\nCreativity in music was found to co-segregate with a duplication covering glucose mutarotase gene (GALM) at 2p22. GALM has influence on serotonin\nrelease and membrane trafficking of the human serotonin transporter.\nGenes related to serotonergic systems have been shown to associate not only with psychiatric disorders but also with creativity and music perception. \nBoth, Pcdha and GALM, are related to the serotonergic systems influencing cognitive and motor functions, important for music perception and practice. \nA 1.3 Mb duplication was identified in a subject with low COMB scores in the region previously linked with absolute pitch (AP) at 8q24. \nNo differences in the CNV burden was detected among the high/low music test scores or creative/non-creative groups. \n\n\nIn summary, CNVs and genes found in this study are related to\n cognitive functions. Our result suggests new candidate genes for music\n perception related traits and supports the previous results from AP\n study.\n\nSource:\n[1] Genome-Wide Copy Number Variation Analysis in Extended Families and Unrelated Individuals Characterized for Musical Aptitude and Creativity in Music\n[2] Musical Aptitude Is Associated with AVPR1A-Haplotypes\n", "topic": "biology", "url": "https://biology.stackexchange.com/questions/19924"}, {"image": "VQAonline_00004440.png", "question": "City card and predicting when Supply piles will run low", "context": "This card kind of has me stumped. In my experience, the vast majority of games deplete the Province or Colony pile before any other, ending the game. Which means in most circumstances, this card is basically an overpriced Village, which isn't a very strong card to begin with.\nIt seems to me the key is knowing when Supply piles are likely to run low. Are there any things to look for in the setup of 10 Kingdom cards to help predict when this might happen? Or are you better off waiting until it actually does happen before investing in this card?\n\n", "answer": "More supply piles tend to go in 5 or 6 player games.\nCards that synergize with themselves tend to get bought out quickly - like Caravan, Treasure Map, and Laboratory. Extra buys also seem to increase the rate at which Kingdom cards get bought out.\nTerminal actions (ones that do not give you +1 Action) tend to get bought less frequently, as each player would only consider 1-2 copies anyway. It seems lower cost cards (2 and 3) aren't bought out very much. 4-5 costs appear to go fastest.\nI have noticed City itself is frequently the first one to get bought out if any will, and that immediately boosts all of them. Generally I tend to wait on City until I notice people are grabbing Kingdom cards, but I also generally lose to the guy who bought 5 of them.\n", "topic": "boardgames", "url": "https://boardgames.stackexchange.com/questions/1262"}, {"image": "VQAonline_00004566.png", "question": "What is this descent figure?", "context": "I was wandering around on boardgamegeek and I saw this picture of a guy painting Descent figures, but the post doesn't exists anymore - thank you google - and the figures are just amazing\nI'm not sure this is descent figure, but photos are taken on Descent v2 tiles so I guess it may be an extension I don't know.\n\n", "answer": "It is the Ice Wyrm, from Descent second edition.\n", "topic": "boardgames", "url": "https://boardgames.stackexchange.com/questions/28481"}, {"image": "VQAonline_00004654.png", "question": "Carcassone - placing barn figure in field", "context": "Game : Carcassone with expansion \"Abbey & Mayor\". \nQuestion - theoretically barn can be placed:\nA) only if my farmer has already been placed in field?\nB) regarding if any farmers already exist in fields?\n\n", "answer": "According to the rules, the barn can be placed in a field as long as that field doesn't already have a barn. Farmers are no concern when placing the barn:\n\nYou may place your barn on a field already occupied by farmers. You may not,\n however, place your barn on a field already occupied by another barn. Once\n placed, a barn remains in play until the end of the game.\n\n", "topic": "boardgames", "url": "https://boardgames.stackexchange.com/questions/44369"}, {"image": "VQAonline_00004720.png", "question": "Canceling King Arthurs boosted attack on cards with no effects", "context": "In Unmatched: Battle of Legends Vol 1 King Arthur has a special ability:\n\nWhen King Arthur attacks, you may BOOST that attack. Play the BOOST\ncard, face down, along with your attack card.\nIf your opponent cancels the effects on your attack card, the BOOST is\ndiscarded without effect.\n\nOne of King Arthurs cards is Excalibur. There is no effect on the card just a title. If my opponent plays a card that happens to cancel any card effects would a Boosted Excalibur lose the boosted bonus?\n\n", "answer": "From Jenn S. at Restoration Games\n\nEssentially, Arthur's special ability is an effect that can get\ncanceled, so the Boost would be discarded.\nThere is some further discussion on it in this BoardGameGeek thread:\nFeint vs boosted Excalibur.\n\nTL;DR even with Excalibur, the boost would be discarded if an opponent's card discards effects.\n", "topic": "boardgames", "url": "https://boardgames.stackexchange.com/questions/54384"}, {"image": "VQAonline_00004584.png", "question": "What's the use of blinking a creature out (such as with Essence Flux)?", "context": "Essence Flux says:\n\nExile target creature you control, then return that card to the battlefield under its owner's control. If it's a Spirit, put a +1/+1 counter on it.\n\nSo, say I have a 1/1 Sanctuary Cat cast onto the battlefield. According to above, I would exile it, then return it right back? Is that right? I don't see what purpose this serves unless it was a spirit. Could you clarify if I have this right and what the benefit is?\n\n", "answer": "There are many uses for so-called \"flicker\" effects like from Essence Flux.\nThe namesake Flicker was the first card to exile and immediately return the target. This is where the mechanic got its unofficial name.\nThe creature that returns is a whole new object without any relation to its previous existence. That means that, for example:\n\nThe creature will trigger all applicable \"enters the battlefield\" and \"leaves the battlefield\" abilities.\nIf the creature you blink is being targeted by a spell or ability, then it will lose track of the creature and not affect it, or will fizzle if that creature was the only target.\nAuras and equipments attached to the creature will fall off of it, and counters get removed.\nIt will return untapped.\nAll temporary effects, such as from Giant Growth or Death Wind, will be removed.\nThe new creature has no damage marked on it, so it's like an improved regeneration effect.\nIf you blink the creature during combat, it will be removed from combat and will neither deal nor take any damage, but (if it was already assigned as a blocker), the creature blocked by it will still be blocked.\n\nOther blink cards such as Flicker can also target enemy creatures (though not at instant speed), which can be a very valuable play at the right time.\nIn short, blink effects are a powerful trick that can throw a wrench in your opponent's plans at a crucial moment. The +1/+1 counter for spirits is more of a bonus in most cases, but of course it allows you to get even more utility out of this spell.\n", "topic": "boardgames", "url": "https://boardgames.stackexchange.com/questions/32379"}, {"image": "VQAonline_00004681.png", "question": "Bonus traps from Essen Spielmesse 2019, what are the rules for them?", "context": "I was at the Essen Spielmesse 2019 and enjoyed the game Obscurio. I bought the game and at the booth they gave us some bonus traps to use in the game. However no rules were given with the traps and we have no idea how to use them. We could hazard a guess, but would love some official guidelines on how they should be played if they are pulled out of the bag.\n\n", "answer": "Someone asked the same question on boardgamegeek, and someone found the rules on the publisher's website, they are:\n\nMusic hall (Music note):\n\nAffects step 2 and 4\nThe wizards can only communicate by singing, the Grimoire is not affected by this token\n\nRaving mad (Cards with the arrows around the outside and question mark on them):\n\nAffects step 2\nTake the loyalty card from each wizard and mix them facedown. Deal them to each Wizard. This token is then removed from the game and cannot be pulled again\n\nAphonia (Speech bubble with the X in it):\n\n\nAffects step 2 and 4\nThe wizards can only communicate through gestures and mimes, they may not talk, sing, nor make any verbal noise of any kind. The Grimoire is not affected by this token.\n\n\n", "topic": "boardgames", "url": "https://boardgames.stackexchange.com/questions/49173"}, {"image": "VQAonline_00004505.png", "question": "Do I have to sacrifice non-token dragons to Day of the Dragons?", "context": "The Card Day of the Dragons says the following:\n\n\nWhen Day of the Dragons leaves the battlefield, sacrifice all Dragons you control. Then return the exiled cards to the battlefield under your control.\n\nNote the part I've highlighted. What exactly does that mean?\nFor example, if I invoke a dragon card after using the enchantment (Dromar, the Banisher for example), when the enchantment leaves play do I have to sacrifice both my 5/5 Dragon tokens and Dromar?\n", "answer": "Yes, you would have to sacrifice all dragons creatures and creature tokens under your control, regardless of where they came from or when they were cast. All Dragons means all permanents (creatures, tokens) that have have Dragon in their supertype or subtype. \"Under your control\" should be clear.\n", "topic": "boardgames", "url": "https://boardgames.stackexchange.com/questions/13093"}, {"image": "VQAonline_00004747.png", "question": "Is there a Technique or Algorithm for Building Spheres?", "context": "There are times when I would like to build a sphere or a dome like this one:\n\nFrom Small Lego Sphere Project\nWhile this example is great, I find it difficult to scale it to another size, whether smaller or larger, without it...not looking roundish anymore.\nIs there a tool available that can design a sphere or dome based on a size?\n", "answer": "This website generates a sphere in LDraw format: http://lego.bldesign.org/sphere/\nHere's a link if you want to learn how it can be done: http://groups.csail.mit.edu/graphics/classes/6.837/F04/assignments/assignment5/\n(there are other ways too)\n", "topic": "bricks", "url": "https://bricks.stackexchange.com/questions/549"}, {"image": "VQAonline_00004941.png", "question": "How do I build this robot car?", "context": "I want to start using more LEGO Technic, but I'm not sure what the best way is to buy the parts without breaking the bank.\nI'd really like to make a robot like the ones shown in this video.\nMaking Lego Car CLIMB Obstacles\n\nThey identify these parts:\nSBrick https://www.sbrick.com/sbrick.html\nBrickController 2 app (Android)\nPlayStation 4 wireless controller\nBut where do I get the rest? Their channel recommends BrickLink, but that mostly seems to be individual pieces and vintage bits of Lego (unless I'm using it all wrong?)\nIs there a low-cost way to buy most of the things shown here? Should I buy a kilogram of used Technic elements on eBay and hope for the best?\nWhich motors, gearing systems, and batteries are they using here? How are they connected to the SBrick? Is there a single kit that gets me 90% of the way there?\n", "answer": "There are several ways you can start with LEGO Technic.\nIf you have no elements whatsoever the best (cost-efficient) approach is to buy a set that contains most of the parts. Or a couple of sets. Bulk lots could also be useful, however, they may not contain the most expensive elements (like motors). Specific elements could be acquired from Bricklink or BrickOwl.\nIt is also cost-efficient to buy sets that contain electronic elements already since these are usually more expensive when bought individually.\n\nWhich motors, gearing systems, and batteries are they using here?\n\nMotor - Power Functions M-Motor (as a \"set\") or Electric, Motor 9V Power Functions M with Dark Bluish Gray Bottom (as an \"element\")\nGears - multiple have been used in a video. You can find all of them in the Technic, Gear section. These are used in the pictured model:\n\nTechnic, Gear 24 Tooth (2nd Version - 1 Axle Hole)\n\nTechnic, Gear Worm Screw, Long, Axle Opening Type II\n\nTechnic, Gearbox 2 x 4 x 3 1/3\n\nTechnic, Gear 40 Tooth\n\n\nBattery - battery box used in the video is Electric 9V Battery Box Power Functions (Rechargeable) with Dark Bluish Gray Bottom. It is rather expensive and I suggest much cheaper and commonly found Electric 9V Battery Box Power Functions (Non-Rechargeable) with Dark Bluish Gray Bottom instead.\nPlease be advised both motors and battery boxes belong to the Power Functions system which has been recently discontinued by LEGO.\n\nHow are they connected to the SBrick?\n\nSBrick is a controller and has connections compatible with the ones used by the Power Functions system. Each motor has a cable with a connector, so no extra cables are needed here.\n\nIs there a single kit that gets me 90% of the way there?\n\nThe Pictured model doesn't use a huge bunch of elements, so it is rather hard to define what 90% is. Personally, I would probably look for some used sets with motors and battery box. And acquire the rest from Bricklink.\n", "topic": "bricks", "url": "https://bricks.stackexchange.com/questions/16742"}, {"image": "VQAonline_00004905.png", "question": "What is the meaning of the available color lists in Bricklink searches?", "context": "Recent AFOL; figuring out Bricklink.\nI'm trying to understand the color lists that are retrieved by searches in Bricklink.\nUsing Bricklink, I searched for part 3068a (2x2 tile without Groove), and looking at availability, and so forth.\nFour categories of color lists are given in Bricklink:\nLots for Sale\nOn Wanted list\nPrice Guide info\nKnown Colors\nThe colors displayed in these categories puzzle me. For example, how is it possible for a color to be in the \"Lots for Sale\" list, and not be in the \"Known Colors\" list? Other possible discrepancies exist in these lists.\nWhat do these lists of colors represent?\nHere's a link to the available color lists for part 3068a:\nhttps://www.bricklink.com/v2/catalog/catalogitem.page?id=17085#T=C\nFor part 3068a, I scraped the data, looking for differences.\nHere's a screenshot of part of the results, highlighting a couple of colors that are in lots for sale and are priced, but are not in the known color list:\n\n", "answer": "Lots for Sale - this contains colors in which current part is being listed for sale.\nOn Wanted list - this shows which colors Bricklink members have added to their \"wanted list\". It doesn't mean parts in this color have existed. Most likely this being used to add non-existing color to list of wanted items and mark for notification. In such case if wanted item ever to appear for sale (and seller pushed this notification) person with desired item would be notified.\nPrice Guide info - here you can see sale/price history for each color for the past 6 months as well as items currently listed for sale.\nKnown Colors - here you can find colors which appear in released sets. Bricks in this list are tied to set inventories produced by community of Bricklink. If there ever be an error with inventory or it has alternative selection of bricks, this is being addressed by fixing or adding an alternative inventory for the same set. Every Bricklink member is welcome to make these fixes.\nThere could be several reasons why there are more colors for sale than on a list of known colors:\n\nAn error is an obvious thing. It is quite easy to make a mistake while listing individual part for sale if it has been produced in similar colors, or has variety in shape (with/without groove). Some shades of Blue and Pink are hard to distinguish. Shape differences aren't always so easy to notice too.\nSome parts you see for sale may have been prototypes, test bricks or\nproduced for specific project (like life-size LEGO Bugatti has a ton of non-existing parts) which may have made their way somehow from the factory. This is not something you could request for as LEGO is trying to not let these items into the market.\n\nOne more thing to note. If you find a part in odd color and it isn't listed as HTF (Hard To Find) or Special Color, this is most likely a mistake while listing. Odd parts have quite a high value (depends on availability). Although it doesn't hurt to contact the seller to confirm the case before ordering. \n", "topic": "bricks", "url": "https://bricks.stackexchange.com/questions/13458"}, {"image": "VQAonline_00004742.png", "question": "Why did Technic bricks have holes in the studs?", "context": "Lego Technic (/ Expert Builder) bricks have always had a very different design than the normal Lego brick, with peg holes lining the sides serving fairly obvious purposes, but they also have had (as long as I can remember) hollow studs on the top:\n\nNow that these pieces are appearing in non-Technic sets I've seen them used often to hold things - they're about the same size as a minifigure's grip, so wrenches, guns, gems, and other decorative elements that need to stand are often stuck into them.\nBut Technic has no minifigures and features only sparse decoration. I don't recall ever seeing a Technic set that uses the holes. It seems like they would be much more difficult to mold especially as far back as Technic's introduction in the late 1970s.\nSo why were they introduced in Technic? (Or am I wrong, and they were not introduced in Technic / Expert Builder?) Did they serve a functional purpose, or were they just another visual marker to separate it from \"regular\" Lego during its introduction?\n", "answer": "From principles of molding, you want a uniform part thickness throughout if possible. This facilitates plastic flow as well as dimensional stability (you want uniform shrinkage on cooling.) Removing a divot from the underside of a stud serves this purpose in a regular brick. The Technic brick with through holes would have had a large amount of solid plastic under the stud compared to regular bricks, which could create undesirable shrinkage and warping from stress in cooling. So removing the top divot helps there. \nHowever, if you examine a 1x16 beam from Expert Builder, you can observe a plastic channel running along the underside between studs 4 to 13 (which are injection gates; also stud 8, or 9.) The 1x8 doesn't have this. This channel would let the plastic flow away from the injection gate without turning too many corners. It's also possible LEGO used a reciprocal molding action where plastic is pressed in one gate, then another, in a rocking action, to facilitate welding of the multiple injection streams. Solid studs might lessen the effectiveness of this process, but the channel helps it. If the channel was necessary to produce a 1x16 beam back in the day, and the channel forced hollow studs, then the hollow stud style was the only way to make a 1x16 beam. The technical difficulty of making a new, longer brick (beam) may have influenced the look of the whole Expert Builder system. \nThat's only my theory, but you can still see the channel for yourself. \nMy tests also indicate the hollow stud separates from another beam more easily than a common solid stud would.\nIt remains possible though that they were just maximizing the usefulness of the part by making the stud hollow. Though the fun part of being an engineer is maximizing more than one property at once. \nSo my thought is the hollow stud came from various molding principles. Cost of plastic comes after that (and yes, if you can save a bit, you do.)\nAnother interesting trivia: the Technic studs were each freehand drafted in the early Expert Builder instruction booklets. I joked about reproducing this while showing some Lego employees (in 2001) what could be done with non-representational rendering, for instance a pen-and-ink renderer with random jitter.\n", "topic": "bricks", "url": "https://bricks.stackexchange.com/questions/196"}, {"image": "VQAonline_00004892.png", "question": "How do I run the LEGO Technic V6 engine using a Lego electric motor?", "context": "I am not into LEGO, don't know anything about it, so I will try to be as specific as possible.\nMy goal is to spin the LEGO Technic V6 engine (preferably at some realistic RPM's like thousands).\nSince I can't combust air and gasoline inside, my simple idea was to run it with LEGO electric motor from LEGO Technic Power Functions Motor Set 8293.\nObviously, I need to connect them somehow and make them run. I don't have any other LEGO pieces available. What do I need to buy (the minimum) to get this running?\nHere's the type of V6 that I'm referring to:\n\nHope that explains what I want and I hope someone can help me, I will be very thankful!\n", "answer": "Assuming that your motor already has a Technic axle available, you'll need:\n\nAny Technic motor. You probably want an M motor (8883).\nAn appropriate battery box or other power source. If you selected the above motor, you probably want the PF battery box that uses AA batteries (8881).\nAny Technic elements that you want to actually mount the motor onto the V6. In the case of the above, you could get away with a shorter axle and a few Technic pins. If I were you, I'd either purchase a small Technic set (e.g. https://brickset.com/sets/42088-1/Cherry-Picker), or an assortment of Technic elements on Bricklink.\n\nMost Technic motors aren't especially fast (~400 RPM). If you want an RPM up in the thousands, you'll need at least a couple of gears as well.\n", "topic": "bricks", "url": "https://bricks.stackexchange.com/questions/11367"}, {"image": "VQAonline_00004854.png", "question": "How can I find brick part number from a picture?", "context": "I found some books I like, but the book only give pictures of the parts. For example, this one\n\nHow can I know the part number (for example in bricklink) from the picture?\n", "answer": "You'll need to start learning the nomenclature for those pieces.\nFor example the long straight pieces you have there with holes on top are \"Technic, Brick 1 x 12 with Holes\"\nThe pieces with rounded ends are lift arms. You can browse through the list of them here or take a stab at their descriptions using the width and number of pin holes. For example the purple on you have there is\n\n1 hole wide\n\n9 holes long\n\nas think as a normal LEGO brick (some are half that thickness)\n\nBent with 6 on one line and 4 on the other\nso \"Technic, Liftarm 1 x 9 Bent (6 - 4) Thick\"\n\n\n", "topic": "bricks", "url": "https://bricks.stackexchange.com/questions/9073"}, {"image": "VQAonline_00004901.png", "question": "Can you identify these column decorators?", "context": "I want to decorate some columns with these little round ionic scroll-looking things. Does anyone know the part number?\n\n", "answer": "Brick, Modified 1 x 1 with Scroll with Hollow Stud\nhttps://www.bricklink.com/v2/catalog/catalogitem.page?P=20310&idColor=2\n", "topic": "bricks", "url": "https://bricks.stackexchange.com/questions/13262"}, {"image": "VQAonline_00004983.png", "question": "What raised mudra is depicted on the Buddha rupa at the London Peace Pagoda and what does it mean?", "context": "\nWhat raised mudra is depicted on this particular Buddha rupa at the London Peace Pagoda and what does it mean? Also, which Buddha is this?\n", "answer": "This guide -- The Battersea Park Peace Pagoda -- says,\n\nIn this statue the Vitarka Mudra, associated with explaining the Buddha\u2019s teachings, with the right hand right raised and the tips of the forefinger and thumb touching, is combined with the Tarjani Mudra for warding off evil, with forefinger and little finger outstretched.\n\n", "topic": "buddhism", "url": "https://buddhism.stackexchange.com/questions/44988"}, {"image": "VQAonline_00004981.png", "question": "Does 'karma' mean that everything that happens to us is under our control, or only that we are responsible for it?", "context": "Does 'karma' mean that everything that happens to us is under our control, or only that we are responsible for it? I thought that only substantial beings could be completely in control of everything that happens to them, and not just because things are impermanent.\nAlso, conditioned things are a result of past karma, which is often said to be an unconscious process of seeds becoming ripe. So surely it would only be under out control over the course of very many lives, at least?\nThe Points of Controversy -- theravada -- refute the claim that everything is from karma, including karma, of the rajagirikas and siddhatthikas. But it does not show that for anything but new karma. The sautrantikas taught that there is no life faculty sustaining events between life and death, because karma alone is \"sufficient\" to account for all destinies.\n\n\nthis is accepted by all Buddhists... theravada or mahayana\n\nThe Lioness in Bloom, p33\nFurther, Bodhidharma exhorted\n\nthe practice of following conditions, sentient beings lack a self and\nare all whirled around by conditions and karma; suffering and joy are\nto be equally accepted, for both arise from conditions. If I\nencounter excellent karmic recompense, such as honor and so forth, it\nis in response to causes in my past lives. Even if I should encounter\nsuch recompense in the present, the necessary conditions for it will\nexhaust themselves, and it will again cease to exist. What is there\nto be joyful about in its existence? Gain and loss follow conditions.\nMind has neither increase nor decrease. Unmoved by the winds of joy,\none is mysteriously in accordance with the path. Therefore, it is\ncalled the practice of following conditions.\n\n\nBodhidharnma, Two Entrances\n\n", "answer": "Not everything is caused by karma. Some things are caused by other reasons, like the weather. The Sivaka Sutta talks about this. There's a summary at the bottom:\n\nBile, phlegm, and also wind,\nImbalance and climate too,\nCarelessness and assault,\nWith kamma result as the eighth.\n\nTo quote the sutta in detail:\n\n\u201cSome feelings, S\u012bvaka, arise here originating from phlegm disorders \u2026\noriginating from wind disorders \u2026 originating from an imbalance of the\nthree \u2026 produced by change of climate \u2026 produced by careless behaviour\n\u2026 caused by assault \u2026 produced as the result of kamma: that some\nfeelings arise here produced as the result of kamma one can know for\noneself, and that is considered to be true in the world. Now when\nthose ascetics and brahmins hold such a doctrine and view as this,\n\u2018Whatever a person experiences, whether it be pleasant or painful or\nneither-painful-nor-pleasant, all that is caused by what was done in\nthe past,\u2019 they overshoot what one knows by oneself and they\novershoot what is considered to be true in the world. Therefore I say\nthat this is wrong on the part of those ascetics and brahmins.\u201d\n\nThe Acintita Sutta states that you can't work out karma and its results precisely:\n\n\"The [precise working out of the] results of kamma is an\nunconjecturable that is not to be conjectured about, that would bring\nmadness & vexation to anyone who conjectured about it.\n\nWhat's more skillful is to develop one's virtue (sila), concentration (samadhi) and wisdom (pa\u00f1\u00f1a) as stated in the Lonaphala Sutta. That will mitigate the ill effects of past karma (actions).\nAs for future karma (actions), to avoid accumulating karma debts, it is skillful to adopt the following skillful thinking from the Themes Sutta:\n\n\u201cAnd for the sake of what benefit should a woman or a man, a\nhouseholder or one gone forth, often reflect thus: \u2018I am the owner of\nmy kamma, the heir of my kamma; I have kamma as my origin, kamma as my\nrelative, kamma as my resort; I will be the heir of whatever kamma,\ngood or bad, that I do\u2019? People engage in misconduct by body, speech,\nand mind. But when one often reflects upon this theme, such misconduct\nis either completely abandoned or diminished. It is for the sake of\nthis benefit that a woman or a man, a householder or one gone forth,\nshould often reflect thus: \u2018I am the owner of my kamma, the heir of my\nkamma; I have kamma as my origin, kamma as my relative, kamma as my\nresort; I will be the heir of whatever kamma, good or bad, that I do.\u2019\n\n", "topic": "buddhism", "url": "https://buddhism.stackexchange.com/questions/39874"}, {"image": "VQAonline_00004953.png", "question": "Why is Kwan Yin riding a sea turtle?", "context": "And maybe to save two related questions, why does Samantabhadra ride an elephant and Majursri ride a lion?\nI was meditating with Kwan Yin statute and that was the main distraction, I had no idea why Kwan Yin is on a sea turtle.\n\n", "answer": "Okay, I did my own research and here is what I've come up with so far. Interestingly, the artist is making a reference to this Pali suttra:\n\n\"Monks, suppose that this great earth were totally covered with water,\n and a man were to toss a yoke with a single hole there. A wind from\n the east would push it west, a wind from the west would push it east.\n A wind from the north would push it south, a wind from the south would\n push it north. And suppose a blind sea-turtle were there. It would\n come to the surface once every one hundred years. Now what do you\n think: would that blind sea-turtle, coming to the surface once every\n one hundred years, stick his neck into the yoke with a single hole?\"\n \"It would be a sheer coincidence, lord, that the blind sea-turtle,\n coming to the surface once every one hundred years, would stick his\n neck into the yoke with a single hole.\"\n\"It's likewise a sheer coincidence that one obtains the human state.\n It's likewise a sheer coincidence that a Tathagata, worthy & rightly\n self-awakened, arises in the world. It's likewise a sheer coincidence\n that a doctrine & discipline expounded by a Tathagata appears in the\n world. Now, this human state has been obtained. A Tathagata, worthy &\n rightly self-awakened, has arisen in the world. A doctrine &\n discipline expounded by a Tathagata appears in the world.\n\"Therefore your duty is the contemplation: 'This is stress...This is\n the origination of stress...This is the cessation of stress...This is\n the path of practice leading to the cessation of stress.'\"\n\nhttp://uwf.edu/wmikulas/webpage/leaves/website/canon/samyutta/sn56-48.html\nOther Bodhisattvas and Buddhas are often described as riding a particular animal.\nSamantabhadra is explicitly described as riding an elephant in his sutra. Samantabhadra's elephant appears to be another awe inspiring flourish. (At least from my reading of the excerpt of Samantabhadra Meditation S\u016btra in Wikipedia.\nWikipedia says of Manjusri and his lion: \"Ma\u00f1ju\u015br\u012b is often depicted as riding on a blue lion, or sitting on the skin of a lion. This represents the use of wisdom to tame the mind, which is compared to riding or subduing a ferocious lion.\" Wikipedia doesn't explain why the lions, but elsewhere it's suggested that lions were sort a family symbol of the Sakyas, or it's just a reference to the pali sutra about \"The lions Roar\", ref: http://www.accesstoinsight.org/lib/authors/nanamoli/wheel390.html\nhttp://en.wikipedia.org/wiki/Shakyasimha\n", "topic": "buddhism", "url": "https://buddhism.stackexchange.com/questions/1541"}, {"image": "VQAonline_00004966.png", "question": "How is it wrong to believe that a self exists, or that it doesn't?", "context": "\nthe one place where the Buddha was asked point-blank whether or not\n there was a self, he refused to answer. When later asked why, he said\n that to hold either that there is a self or that there is no self is\n to fall into extreme forms of wrong view that make the path of\n Buddhist practice impossible.\n\nIt seems to me, and I may be wrong, that the skandhas can be identified as a self, as long as they aren't then taken to be in any way unchanged from moment to moment\n\nEncyclopedia of Buddhism, By Damien Keown, Charles S. Prebish\n\nSo I was wondering if that above silence means that any dharma can fulfill the role of that empirical self.\n\n\nAn example would be: Adam tastes the apple\nSupposing this \"taste\" can be considered a kind of self, then if Adam sees smells touches feels nothing, just tastes the apple, there is \"continuity\" and he's still Adam. But as long as he does have other senses, and from moment to moment, the taste of apple doesn't make him who is he.\nI've never read any commentary which claims this is what the empirical self is in Buddhism, so would be really surprised that this counts as a categorical denial of substantial self.\n", "answer": "Buddha kept silent in response to Vacchagotta\u2019s question because answering it in either way, it would have been misunderstood. This nature of the self is beyond the level of understanding of Vacchagotta. He is not yet at that stage in his spiritual development. Buddha never denies the existence of the self. He rejects annihilationism. Read the Alagaddupama Sutta (MN 22), Yamaka Sutta (SN 22.85), Kalama Sutta (AN 3.65), Ananda Sutta SN 44.10. In fact Buddha explained this when asked by the Venerable Ananda about his silence regarding Vacchagotta\u2019s question. He said that this would have lead Vacchagotta to misinterpret the answer in a way that would bring him further attachment.\nWith regard to self, the Buddha said to Ananda in the Mahaparinibbana Sutta, to stay as those who have the self as island, as those who have the self as refuge, as those who have no other refuge. (DN 16). As per the scriptures existence is real, but it is transitory.\n", "topic": "buddhism", "url": "https://buddhism.stackexchange.com/questions/15898"}, {"image": "VQAonline_00004977.png", "question": "In the movie Spring Summer Fall Winter and Spring, why does the master cry?", "context": "Here is the movie: Spring Summer Fall Winter and Spring 2003\nThe plot is that the student having attachment to a woman, leave the temple and marries her. Years fly by, and the student returns because he kills her, after her affair. The master helps him to let the anger out before being caught by the police, and then he immolates himself. After that, he leaves some \u015bar\u012bra.\nMy question is: why would the master cries before the immolation?\n \nMy answer is that he disappoints with the student. But isn't that disappointment an attachment?\n", "answer": "\nMy question is: why would the master cries before the immolation?\n\nThe \"Winter\" segment of the movie kind of gave the hint. Notice in this segment, after being paroled from prison for the crime of killing his wife, the now middle-age apprentice returns to the floating monastery and begins to take on the role of his previous master. And as with the movie's title's central theme, which reflects the cyclical nature of Samsara, the same exact history repeats itself with the scene of a woman abandoning her baby son, then the baby becomes the boy apprentice who is shown tormenting a turtle in the Spring-Again segment,... etc. Putting together all the pieces of the puzzle, it points to the original old master's own dark past with its own dark kamma before he renounced it to become a good monk. So although after years of training and gaining certain level of attainment, he's still not free of his kammic debt. And so he cries not only out of sadness for his student but also for himself, that his self immolation doesn't mean total and final complete liberation, there's still \"more work to be done\" and life will go on and on until all kammic debts' been completedly exhausted, hence the image of the water snake swimming away after the self immolation. \n", "topic": "buddhism", "url": "https://buddhism.stackexchange.com/questions/35686"}, {"image": "VQAonline_00004980.png", "question": "Does the Buddha deny genetic differences?", "context": "In MN.98, the Buddha seems to deny the ethnic/racial/genetic differences between humans. He seems to indicate that differences in physical traits between individuals are not determined by birth as in the rest of the animal world. This seems totally contrary to genetics:\nI. Humans are genetically part of the animal world and follow the same laws of heritability.\nII. Whether we speak of race or ethnicity, there are many identifiable genetic clusters within humanity.\nIII. Even without going that far it is obvious that between two individuals there will be physical differences according to their genetics (height, eye colors, features, hair, etc). The Buddha seems to deny this completely.\nHe says for example that the differences in eyes/nose/shoulders, etc., between two individuals are not defined by birth but by convention... this is totally false: children inherit the traits of their parents and two parents with the Asian phenotype do not give birth to an African child by pure chance or convention. I thought it was because he wanted to say that all these genetic differences were conventional and I would have agreed, but it can't be that since he's only saying that for humans and recognizes the validity of these differences for animals and plants... I don't understand this sutta?\n\nWhile the differences between these species are defined by their birth, the differences between humans are not defined by their birth. Not by hair nor by head, not by ear nor by eye, not by mouth nor by nose, not by lips nor by eyebrow, not by shoulder nor by neck, not by belly nor by back, not by buttocks nor by breast, not by genitals nor by groin, not by hands nor by feet, not by fingers nor by nails, not by knees nor by thighs, not by color nor by voice: none of these are defined by birth\n as it is for other species. In individual human bodies you can\u2019t find such distinctions. The distinctions among humans are spoken of by convention.\n\nThe Buddha\n\n\n\n", "answer": "MN 98 is about \"jati\" or \"social identity\". The Buddha says physical birth or \"ancestry\" is unrelated to jati; that the jatis are verbal designations assigned due to kamma (actions). \nFor example, Michael Jordan's great great grandparents were probably slaves. Where as Michael Jordan was a sporting superstar & billionaire. Jordan was born at Cumberland Hospital in Fort Greene, Brooklyn, to Deloris, who worked in banking, and James R. Jordan Sr., an equipment supervisor. These distinctions of \"slave\", \"banker\", \"supervisor\" vs \"basketball billionaire\" are not related to physical birth or ancestry. They are related to social kamma (activity). \nAlbert Einstein was a German-born theoretical physicist. His parents were Hermann Einstein, a salesman and engineer, and Pauline Koch. The Einsteins were non-observant Ashkenazi Jews, and Albert attended a Catholic elementary school.\n\nYou\u2019re a brahmin by your deeds, Kammun\u0101 br\u0101hma\u1e47o hoti,\nand by deeds a non-brahmin. kammun\u0101 hoti abr\u0101hma\u1e47o.\nYou\u2019re a farmer by your deeds, Kassako kammun\u0101 hoti,\nby deeds you\u2019re a professional; sippiko hoti kammun\u0101;\nyou\u2019re a trader by your deeds, V\u0101\u1e47ijo kammun\u0101 hoti,\nby deeds are you an employee; pessako hoti kammun\u0101.\nyou\u2019re a bandit by your deeds, Coropi kammun\u0101 hoti,\nby deeds you\u2019re a soldier; yodh\u0101j\u012bvopi kammun\u0101;\nyou\u2019re a sacrificer by your deeds, Y\u0101jako kammun\u0101 hoti,\nby deeds you\u2019re a ruler. r\u0101j\u0101pi hoti kammun\u0101.\nIn this way in accord with truth, Evameta\u1e43 yath\u0101bh\u016bta\u1e43,\nthe astute regard deeds. kamma\u1e43 passanti pa\u1e47\u1e0dit\u0101;\nSeeing dependent origination, Pa\u1e6diccasamupp\u0101dadass\u0101,\nthey\u2019re expert in deeds and their results. kammavip\u0101kakovid\u0101.\nDeeds make the world go on, Kammun\u0101 vattati loko,\ndeeds make people go on; kammun\u0101 vattati paj\u0101;\nsentient beings are bound by deeds, Kammanibandhan\u0101 satt\u0101,\nlike a moving chariot\u2019s linchpin. rathass\u0101\u1e47\u012bva y\u0101yato.\nMN 98\n\n", "topic": "buddhism", "url": "https://buddhism.stackexchange.com/questions/37490"}, {"image": "VQAonline_00004972.png", "question": "Dhamma riddle? What does it mean?", "context": "I found a dhamma riddle below. \nWhat do we think it means? \n\n", "answer": "The first one represents desire, grasping something whose roots are impermanent.\nThe next ones look like Tathagatagarbha.\n", "topic": "buddhism", "url": "https://buddhism.stackexchange.com/questions/21474"}, {"image": "VQAonline_00005930.png", "question": "Does aniline react with diazonium ions at C or N?", "context": "\nIn the azo coupling of aniline with benzenediazonium cation, I thought of two possible products 1 and 2 that could be formed. However, I can't work out which will be preferred. What would be the major product and why?\n", "answer": "This is a really interesting question and the answer is that the reaction of benzenediazonium chloride with aniline is a bit different to most of the reactions of benzenediazonium salts in that the initial product is compound 1, diazoaminobenzene. It is possible to run the reaction to isolate diazoaminobenzene prep here.\nThese diazoaminobenzene compounds are unstable with respect to reversion to a diazonium salt + nucleophile, and so many references suggest that compound 2, 4-aminoazobenzene, is directly produced. The procedure for the transformation of diazoaminobenzene 1 to 4-aminoazobenzene 2 is by heating to 50\u00a0\u00b0C with aniline (prep here).\n", "topic": "chemistry", "url": "https://chemistry.stackexchange.com/questions/107344"}, {"image": "VQAonline_00005010.png", "question": "Is this the lowest chair conformer of Beta D-galactose", "context": "I drew the conformer (below) and I'm wondering if this is the most stable chair conformation.\nThree of the bonds are equatorial so it looks like the most stable to me.\n", "answer": "The chair conformation that you have drawn (4C1) is likely to be the most stable one, as it minimizes the number of heavy axial groups.\nTo determine the chair conformation of a hexose, it is generally easiest to draw it and compare it with \u03b2-D-glucose, where all heavy groups are equatorial and the conformation is 4C1. If the number of heavy axial groups becomes smaller when the conformation is changed to 1C4 (all equatorial groups in 4C1 become axial and vice versa), then it is likely that the conformation is 1C4. \n", "topic": "chemistry", "url": "https://chemistry.stackexchange.com/questions/2488"}, {"image": "VQAonline_00005325.png", "question": "Generating mol files from a molecular structure image?", "context": "I have a question regarding mol files.For example I have this molecule-\n\nThe mol file for this is-\n Sample\n\n 22 23 0 0 0 0 0 0 0 0999 V2000\n -2.5962 2.2535 0.0000 C 0 0 0 0 0 0 0 0 0 0 0 0\n -2.5962 0.7512 0.0000 C 0 0 0 0 0 0 0 0 0 0 0 0\n -1.2958 0.0000 0.0000 C 0 0 0 0 0 0 0 0 0 0 0 0\n 0.0000 0.7512 0.0000 N 0 0 0 0 0 0 0 0 0 0 0 0\n 0.0000 2.2535 0.0000 C 0 0 0 0 0 0 0 0 0 0 0 0\n -1.2958 3.0047 0.0000 N 0 0 0 0 0 0 0 0 0 0 0 0\n -4.6479 1.7042 0.0000 F 0 0 0 0 0 0 0 0 0 0 0 0\n 1.3005 3.0047 0.0000 O 0 0 0 0 0 0 0 0 0 0 0 0\n -1.2958 4.5070 0.0000 C 0 0 0 0 0 0 0 0 0 0 0 0\n -1.2958 -1.5023 0.0000 O 0 0 0 0 0 0 0 0 0 0 0 0\n 1.3005 0.0000 0.0000 C 0 0 0 0 0 0 0 0 0 0 0 0\n 1.3005 -1.5023 0.0000 C 0 0 0 0 0 0 0 0 0 0 0 0\n 2.6009 -2.2535 0.0000 C 0 0 0 0 0 0 0 0 0 0 0 0\n 3.8967 -1.5023 0.0000 C 0 0 0 0 0 0 0 0 0 0 0 0\n 3.8967 0.0000 0.0000 C 0 0 0 0 0 0 0 0 0 0 0 0\n 2.6009 0.7512 0.0000 C 0 0 0 0 0 0 0 0 0 0 0 0\n 5.1972 -2.2535 0.0000 Cl 0 0 0 0 0 0 0 0 0 0 0 0\n 2.6009 -3.7559 0.0000 C 0 0 0 0 0 0 0 0 0 0 0 0\n -3.8967 3.0047 0.0000 C 0 0 0 0 0 0 0 0 0 0 0 0\n -5.1972 3.7559 0.0000 F 0 0 0 0 0 0 0 0 0 0 0 0\n -3.1455 4.3052 0.0000 F 0 0 0 0 0 0 0 0 0 0 0 0\n 3.9014 -4.5070 0.0000 O 0 0 0 0 0 0 0 0 0 0 0 0\n 1 2 2 0 0 0 0\n 2 3 1 0 0 0 0\n 3 4 1 0 0 0 0\n 4 5 1 0 0 0 0\n 5 6 1 0 0 0 0\n 1 6 1 0 0 0 0\n 1 19 1 0 0 0 0\n 5 8 2 0 0 0 0\n 6 9 1 0 0 0 0\n 3 10 2 0 0 0 0\n 4 11 1 0 0 0 0\n 11 12 1 0 0 0 0\n 12 13 2 0 0 0 0\n 13 14 1 0 0 0 0\n 14 15 2 0 0 0 0\n 15 16 1 0 0 0 0\n 11 16 2 0 0 0 0\n 14 17 1 0 0 0 0\n 13 18 1 0 0 0 0\n 19 7 1 0 0 0 0\n 19 20 1 0 0 0 0\n 19 21 1 0 0 0 0\n 18 22 2 0 0 0 0\nM END\n\nSo,I have read that lines 5-26 are for atoms.first three columns represent x,y and z coordinates of atoms.So,my question is how are the coordinates of atoms decided?What is taken as the origin?Also,the atoms are large,so they may occupy more than one coordinate,so how do we decide what x,y,z value to take?Also,is H atoms not taken into account for creating mol files?\n", "answer": "The MOL format specification doesn't specify an origin, so presumably any origin can be chosen, as long as the coordinate values are within the allowed range. (The format allows a range of roughly -10,000 to +100,000, but I don't know if all programs support that large a range.) For example, in one of the examples in the specification the atom closest to the origin is over 17.6 units away. Where the origin is actually set would depend on the software used to create the file.\nThe coordinates for each atom specify the nucleus of the atom. I think you have a misconception about atomic size. The size of an atom, and thus the space it takes up, can be defined in different ways. The representation of an atom as a sphere (with the nucleus at the center) is a convention, only approximating reality, and the size of the sphere is arbitrary (but usually based on measurable properties). The nucleus is the only point in the atom that has a well-defined location, and thus the only part that meaningful coordinates can be defined for.\nIn MOL files, the presence of hydrogen atoms can be implied from valence and number of other bonds. (They can also be specified explicitly if needed.) In the long set of zeros at the end of each atom line (just after the element), the fourth zero specifies to add however many single bonds to hydrogen atoms are needed for the atom's valence. The sixth zero indicates to use the element's normal valence. For example, the carbon of the $\\mathrm{CH_3}$ group has only one explicit bond, so three bonds to hydrogen are implied by carbon's normal valence of 4. How implicit hydrogens are handled depends on the software (e.g., this molecule has four implicit hydrogens that, per the usual convention for skeletal formulas, are not shown in your image).\nThe specification for the MOL format is available from Accelrys. The this answer is based on the specification and my knowledge of chemistry. Also, it only describes the MOL V2000 format, which is what you sample file is (based on the header); there is a newer V3000 format.\n", "topic": "chemistry", "url": "https://chemistry.stackexchange.com/questions/33392"}, {"image": "VQAonline_00006063.png", "question": "Carcinogenicity of polynuclear hydrocarbons and benzene rings", "context": "\n(1933) established that polynuclear aromatic hydrocarbons (PAHs) were\ncarcinogenic components of pitch. ... Although less is known about\ntheir health effects than about those of PAHs, it is clear that some\nnitro-PAHs are potent mutagens and carcinogens.(1)\nBenzene and polynuclear hydrocarbons containing more than two benzene\nrings fused together are toxic and said to possess cancer producing\n(carcinogenic) property. Such polynuclear hydrocarbons are formed on\nincomplete combustion of organic materials like tobacco, coal and\npetroleum.(2)\n\nFrom the two sources cited above, I keep seeing that polybenzenoid compounds like 1,2-benzanthracene, 3-methylcholanthrene and 1,2- benzpyrene are mentioned to be carcinogens.\n\nIn general, why are polynuclear compounds carcinogenic? I google searched on this topic and all I could find is their effects and concentrations in various substances.\nMy questions are:\n\nWhy are they carcinogenic\n\nIs there a reason for a given organic compound to be carcinogenic?\n\n\nReferences\n\nPotential Carcinogenic Effects of Polynuclear Aromatic Hydrocarbons and Nitroaromatics in Mobile Source Emissions, STEPHEN S.HECHT(link)\npg403, NCERT Class XI part 2(pdf link)\n\n", "answer": "\nWhy are they carcinogenic?\n\nFrom my point of view as a medicinal chemist, polycyclic aromatic hydrocarbons compounds tend to be metabolized by specific cytochrome P450 (CYP 450) isoforms to form highly electrophilic species, which are highly reactive and can react with the DNA of the cell and damage it. Specifically, they can react with the nucleophilic nitrogen in the DNA bases such as adenine.\n\nIs there a reason for a given organic compound to be carcinogenic?\n\nMechanism for carcinogenicity can vary, in general, any compound damaging or altering the DNA can be carcinogenic. For example, highly reactive electrophiles (such as formaldehyde).\nReference\n\nhttps://socratic.org/questions/how-do-polycyclic-aromatic-hydrocarbons-cause-cancer\n\n", "topic": "chemistry", "url": "https://chemistry.stackexchange.com/questions/126030"}, {"image": "VQAonline_00005353.png", "question": "What angle limits should apply to Graham condensers?", "context": "I have seen Graham condensers set at an angle (perhaps as low as 30 degrees from horizontal). I guessed this wouldn't be a problem because siphoning would clear the condensate. But I recently saw an argument that Graham condensers should only be used at near-vertical angles. Thinking harder this makes sense to me, because as soon as enough liquid condenses in the first loop to form a trap the condenser can't work on any other vapor. Absent some vacuum on the receiving end the only way it could clear itself is if there happened to be enough vapor to eventually close a loop and start the siphon effect, or if enough pressure built up on the boiling end to push the liquid through which would be bad!\nBut I still see diagrams and photos of Graham condensers set at shallow angles. Here's an example from the first few Google image results:\n\nSo when and why, if ever, is it considered a good practice to run a Graham condenser in a non-vertical orientation?\n", "answer": "I have no idea why there are so many photos showing Graham condensers in a non-vertical position. Maybe just because they look so cool and photos tend to be landscape. The only time that might be safe is if there were a vacuum pulling to the collection side.\nGraham condensers should be used vertically, precisely for the reasons you stated: Particularly because otherwise they will bubble, bump, and potentially explode.\n", "topic": "chemistry", "url": "https://chemistry.stackexchange.com/questions/35578"}, {"image": "VQAonline_00006151.png", "question": "Why the formation of a fog is observed when triethylamine is added?", "context": "In the procedure for the synthesis of N-anisoyl-2-pyrrolidinone that was given to me, it is written that when triethylamine is added, a \"fog\" is observed inside the flask.\n\nTriethylamine is added to neutralize the hydrochloric acid that is generated during the reaction, forming triethylammonium chloride.\nI searched for similar procedures involving an acyl chloride and an amine, but the formation of a \"fog\" is never mentioned.\nA quick search on the internet gave me no clue.\nIn my experience I have never witnessed the formation of a \"fog\" with this kind of reactions.\nSince I cannot enter the lab due to the COVID-19 restrictions, I cannot perform the reaction now, can someone help me to understand if and possibly why in this case a \"fog\" is formed?\n", "answer": "You are adding Et3N to a solution that contains HCl as a reaction product. That means there will be HCl vapour above the solution. The Et3N is fairly volatile so there will be Et3N vapour around the addition stream/droplets and this will react with the HCl vapour as it is added to give triethylammonium chloride. This gives you the fog.\n", "topic": "chemistry", "url": "https://chemistry.stackexchange.com/questions/139729"}, {"image": "VQAonline_00005424.png", "question": "Temperature change when volume halved", "context": "For this multiple question:\n\n\n\nThe correct answer is B. However, I don't understand.\nYou are halving the volume\nso for the original solution\nthe number of moles of HCl is 0.1 moles\nand in the new solution, the number of moles of HCl is 0.05 if we are using the same concentration.\nTherefore as there are less number of moles, won't the temperature increase (ie. energy release) get halved?\n", "answer": "The correct answer is B in both the case the final concentrations are not changing. Therefore same will be the heat of rise. But heat released will be less but volume of the final solution also got halved. So temp. raise will be same. \n", "topic": "chemistry", "url": "https://chemistry.stackexchange.com/questions/43909"}, {"image": "VQAonline_00006354.png", "question": "how to break through using pawn storm", "context": "This is a game I played against my phone. I think I have an attacking chance with pawn storm. But staring at the situation for a bit, I can't find a plan to bring down black's defense.\nWhen attacking with g and h pawns against a flat f7/g7/h7 structure, where do my pawns usually go (g5/h6 or g6/h5?)? How to coordinate my other pieces?\nAny general suggestion or illustrative games I can follow to see how this is done?\nThanks\n\n", "answer": "The idea of a pawn storm is to exchange pawns and open files; so the 'usual' defence against a pawn storm is to keep the files closed, which often entails responding to an offer to swap pawns by advancing your own, e.g. if white plays h6, black will want to play ...g6.\nSo from the attacker's viewpoint, you will want to make use of the weaknesses created by any change in your attacker's pawn structure, or force a pawn exchange. In the position you have given, your pieces are not too well placed; only your knight plays a role in the attack on the king, and the queen is ready. The bishop on f4 is a liability in lines where the f-file is opened, and you would much rather have a rook on g1.\nPersonally, I would prefer to start with Rg1 because in almost all cases, you will need to put a rook on the g-file. In fact, after this move you are threatening 2. Qd4, forcing 2 ...f6 where black's position collapses after 3. Nxg7. You are threatening 2. h5 and 3. g6, after which your position will be crushing, because of the piece sacrifices you have available.\nYou usual concern as the attacker (especially when material is equal) is the counterattack of the opponent, as the success of your attack is only a matter of time. In the case of this position, black's a4-a3 advance is too slow (you can even ignore it and allow ...axb2 and your position is not immediately busted yet), so you can even ave the luxury of making another prepatory move like Be3.\nFor any attack against the king, the attacker generally wants to force the advance of the pawns covering the king, so pawns on f7-g7-h7 are usually the most resistant to an attack. Often he does this by tempting the move ...h6 by placing a piece on g5. This position looks much like the Petroff after 1. e4 e5 2. Nf3 Nf6 3. Nxe5 d6 4. Nf3 Nxe4 5. Nc3 Nxc3 6. dxc3, and is probably what you played if I had to guess. I recall Morozevich winning a game in this line as white, which you could probably look up.\n", "topic": "chess", "url": "https://chess.stackexchange.com/questions/9872"}, {"image": "VQAonline_00006521.png", "question": "Why this game is a draw?", "context": "\nChess.com gives a draw result. I can't understand why.\n", "answer": "Black has no legal move, which is an automated draw by design.\nEvery move at Black's hand would result in its king in check.\n", "topic": "chess", "url": "https://chess.stackexchange.com/questions/39150"}, {"image": "VQAonline_00006496.png", "question": "What do these game durations mean?", "context": "There are different game duration on chess.com. What do options 1|1, 2|1 etc. mean?\n\n", "answer": "For example 3|2 means each player starts with 3 minutes and gets 2 seconds added to the clock for making a move (this is called an \"increment\").\nWhen there is no \"|\", you do not get any time for making a move, and each player gets the displayed time in minutes at the start of the game.\nAnother notation you might see uses a \"+\", for example 3+2.\n", "topic": "chess", "url": "https://chess.stackexchange.com/questions/34534"}, {"image": "VQAonline_00006505.png", "question": "What is meant by \"Licensed\" in FIDE website?", "context": "I was browsing the candidates' profiles of the FIDE World cup on the official website and noticed something interesting.\nOn the profile page of GM Vladislav Kovalev, a Belarusian chess GM, has an interesting declaration, called \"Licensed\", which I didn't find on the profile of any other players.\n\nInitially, I thought that this is because he is playing under FIDE and not Belarus. So I checked the profile of GM Alireza Firouzja and there was NO such declaration whatsoever. But, I have noticed another disparity in Alireza's profile. He is enlisted under the Federation of France but his national ranking is calculated under FIDE.\nCan anyone please help me to understand about what's going on?\n", "answer": "\nCan anyone please help me to understand about what's going on?\n\nYes, it is nice that we can look up our entries on the FIDE rating website but actually there is a business need for this information for arbiters and tournament organizers to avoid fines from FIDE.\nWhenever a FIDE rated tournament is rated by FIDE (standard time control only because rapid and blitz, for the moment are free) FIDE charges the federation 1 euro per player for Swiss tournaments and lower rated (average rating < 2230) round robins. The full details are given in Appendix 2 of the FIDE 02. Financial Regulations / Financial Rules (effective from 1 Jan 2021) document.\nEach of the players must normally be registered with one of the federations and those federations also pay FIDE an annual membership fee which is detailed in Appendix 1 of the above document. All national, over-the-board players are normally registered with a federation and so are regarded as licensed. In exceptional circumstances, e.g. a player seeks asylum, a player may for a time be without a national federation and be registered with FIDE as their federation until they settle on a new federation.\nAdditionally FIDE has its Online Arena where players can register directly with FIDE and play online. Online players who aren't already registered OTB with a federation get a FIDE online registration. This licenses them to play in FIDE's Online Arena but does not license them to play rated FIDE OTB tournaments.\nNormally players pay a registration fee to their national federation which covers them for playing FIDE rated tournaments. If players could register directly with FIDE for free and still play OTB FIDE rated tournaments then this would deprive the federations of the money they need to operate.\nTherefore if your federation is FIDE (for whatever reason) then you need to pay FIDE a registration fee (which ideally is more than you would pay a normal federation) before you can be licensed to play OTB.\nThese rules regarding licensing are defined in the FIDE document Regulations on Registration & Licensing of Players / Regulations for Registration & Licensing of Players (effective from July 1, 2015).\nHere are some useful extracts:\n\n2.5 Players with no Federation are automatically considered to be \u2018FIDE flag players\u2019 and their website profile cards indicate FID as their Federation.\n2.6 Players registered as FID are provisionally licensed to play over the board rated tournaments after paying the \u2018FIDE license fee\u2019 of 60 euros. \u2018FIDE flag players\u2019 are encouraged to join a National Federation rather continuing to play as FID.\n2.7 The provisional license under FIDE flag expires on 30 June of the following year.\n2.8 Before 1 July, the license must be renewed with a payment of 60 euros to FIDE, without further notification from FIDE, failing which the license is annulled.\n2.9 \u2018FIDE flag players\u2019 with a valid provisional license are referred to as \u2018licensed\u2019 on their FIDE website profile cards\n\nIf an arbiter or organizer submits a tournament for FIDE rating which includes one or more unlicensed FID players then the federation has to pay the license fees, basically a fine. Hence this information (licensed or not) is vital for arbiters and organizers.\nHere is the FIDE profile for one of these unlicensed players, Abbagouni, Raja. You will see that in the top right hand corner it says:\n\nNot Licensed Player.\nExtra fees for tournament Organizers are applicable.\nPlease refer to FIDE Handbook or proceed to get license\n\n", "topic": "chess", "url": "https://chess.stackexchange.com/questions/35591"}, {"image": "VQAonline_00006437.png", "question": "Why is Na5 not played in this line of the French Defense, Advance Variation?", "context": "I find Na5 a good resource for black in this line of the french, profiting from the fact that white's b-pawn is binned to the bishop. As far as I can understand this position with my limited chess, I find it a strong move, winning space on queenside with the plan of trying Bd7-Rc8-Nc4 on c-file. \n[FEN \"r1b1kbnr/pp3ppp/1qn1p3/2ppP3/3P4/2P2N2/PP3PPP/RNBQKB1R w KQkq - 3 6\"]\n\n6. a3 Nh6 7. b4 cxd4 8. cxd4 Nf5\n9. Bb2 Na5\n\nHowever, I don't find this move at the database of chesstempo I use to use for my correspondence games.\n\nStockfish neither considers it. It even don't play 10....Nbd2 (it plays Nc3) to defend on c4 (which I would expect), and doesn't wonder at all about Nc4 (it simply trades his bishop for the knight; apparently it should result an advantage on the center).\nWhat's my misunderstanding here?\n", "answer": "The plan you outlined with ...Bd7, ...Rc8, ...Nc4 is a good strategical plan, but there are a few concrete reasons it doesn't work well:\n\nIn the opening you should be focusing on developing all your pieces and castling. Your plan with maneuvering the knight to c4 will move the same piece three times.\nAt depth 33, Stockfish gives 10.Nc3 Bd7 11.Rb1!, breaking the pin on the b2-bishop and attacking the a5-knight. Now you have to move the knight before your a8-rook manages to reach c8. If you go ahead with 11...Nc4 then 12.Bxc4 dxc4 is poor for you. Your new pawn on c4 is mainly a weakness at the moment, and White may blast open the centre with d5 to attack your uncasted king.\nAlso, White could always play 10.Qa4+. If you block with 10...Bd7 then 11.Qxa5 wins the knight. So you have to respond with retreating the knight: 10...Nc6.\n\n", "topic": "chess", "url": "https://chess.stackexchange.com/questions/24692"}, {"image": "VQAonline_00006444.png", "question": "The move Bg4 in the Pirc defense", "context": "I am having trouble understanding a move studied in an opening book. The move is Bg4 in the Pirc defence in this position.\n\nWhat is the point of aiming to exchange the bishop on f3? The point d4 does not seem to be the reason.\n", "answer": "Compare that position with other variations where White does not allow ...Bg4 to be played. Black often struggles with the development of the c8 bishop.\nAlso, the central dark squares are indeed a target, so ...Bg4 is clearly helping there (what else could he possibly do in a different square?) Pawn breaks with ...e5 or ...c5 are a possibility as well\n", "topic": "chess", "url": "https://chess.stackexchange.com/questions/25230"}, {"image": "VQAonline_00006438.png", "question": "King or Queen-Which piece is which?", "context": "I recently bought a set of wooden chess pieces, which I liked for its simplicity.\nI'm not sure, however, which one is supposed to be the king and which one is supposed to be the queen.\nI would say that the one with the spike is the queen, and the one more flat is the king. Am I right, or is it the opposite?\n\n", "answer": "In general, a chess set has the king as the tallest piece, followed by queen, bishop, knight, rook and pawn in that order. Notice in the starting position how the piece height decreases smoothly from the centre to the edge. (Also, when buying a chess set, usually the height of the king is given as a guide to the size of the chessmen.)\nThus I would say the king is the taller of the two pieces, which from your picture looks to be the one with the spike on the left.\nThis ordering of height may not hold for particularly ornate decorative sets.\nHowever, if you're not going to use the set to play against other people in tournaments (where usually Staunton pieces are mandated anyway), the correct answer would probably be: the king is whichever piece you think it is.\n", "topic": "chess", "url": "https://chess.stackexchange.com/questions/24719"}, {"image": "VQAonline_00006766.png", "question": "What does \u65e9\u5f85\u4e0d\u4f4f\u4e86 mean in \u8981\u5728\u5f80\u5e38\uff0c\u6211\u4e00\u5b9a\u65e9\u5f85\u4e0d\u4f4f\u4e86\u2026\u2026?", "context": "\n\n\u8981\u5728\u5f80\u5e38\uff0c\u6211\u4e00\u5b9a\u65e9\u5f85\u4e0d\u4f4f\u4e86\uff0c\u60e6\u8bb0\u7740\u67d0\u67d0\u4fa6\u63a2\u7684\u6848\u4ef6\u662f\u4e0d\u662f\u6709\u4e86\u7a81\u7834\uff0c\u2026\u2026\nGoogle Translate: As usual, I must have been unable to stay any longer, wondering if there was a breakthrough in the case of Detective XX,...\n\u6807\u51c6\u6559\u7a0bHSK6\u4e0a\uff0c87\u9875\n\nI don't understand what the part in bold above means. Maybe it's something along the lines of \"As usual, I was definitely prematurely restless, ...\". It's not clear at all to me.\nQuestion: What does \u65e9\u5f85\u4e0d\u4f4f\u4e86 mean in \u8981\u5728\u5f80\u5e38\uff0c\u6211\u4e00\u5b9a\u65e9\u5f85\u4e0d\u4f4f\u4e86\u2026\u2026?\n", "answer": "\n\u8981\u662f signifies a hypothetical construction. \u5f80\u5e38 does not only mean \u2018as usual\u2019 (which only corresponds to \u5e38, as in \u901a\u5e38, \u6b63\u5e38, \u5e38\u898f); \u5f80 here specifies the past, as in \u5f80\u65e5, \u5f80\u4e8b, \u5f80\u6614. Together they mean \u2018normally in the past\u2019.\n\n\u65e9 is an adverb meaning \u2018already\u2019. Think of that as saying, something happened well in the past (early in the timeline).\n\n\u4ed6\u65e9(\u5c31)\u96e2\u958b\u4e86\u3002 He left already.\n\n\n\u5f85 is a verb meaning \u2018to stay\u2019. \u4e0d\u4f4f is an add-on to it, literally meaning \u2018not to a still\u2019. Figuratively, \u5f85\u4e0d\u4f4f means you can\u2019t contain yourself, or you\u2019ve got itchy feet. Perhaps you're excited or anxious about something.\n\nThe whole sentence means \u2018I\u2019 in the past would\u2019ve not been able to control myself in coming up with the many racing thoughts in my head (unsolved cases, scientific breakthroughs, lottery etc.). My mind is so irritable that these thoughts would have occurred very easily and very soon (hence the use of \u65e9). But now my mind is completely at peace, so they are nowhere to be found.\n\n\n", "topic": "chinese", "url": "https://chinese.stackexchange.com/questions/43915"}, {"image": "VQAonline_00006642.png", "question": "What does \u4e50\u7fa4 (l\u00e8q\u00fan) mean?", "context": "Among the many banners for new students at Nankai Uni is this one:\n\nIt says:\n\n\u7231\u56fd \u656c\u4e1a \u521b\u65b0 \u4e50\u7fa4\n\u00c0igu\u00f3 j\u00ecngy\u00e8 chu\u00e0ngx\u012bn l\u00e8q\u00fan\n\nThe first three I can translate:\n\n\u7231\u56fd (\u00e0igu\u00f3\uff09 = patriotism (literally \"love country\")\n\u656c\u4e1a (j\u00ecngy\u00e8) = dedication to one's job\n\u521b\u65b0 (chu\u00e0ngx\u012bn\uff09 = innovation\n\nBut the fourth one doesn't appear on dict.cn. (And I think I've managed to identify the hanzi characters here.)\nQuestion: What does \u4e50\u7fa4 (l\u00e8q\u00fan) mean?\nIf we break it down:\n\n\u4e50 (l\u00e8) = happy\n\u7fa4 (q\u00fan) = group\n\nSo I guess it means something like \"happy group\", which sounds a bit strange.\n", "answer": "Baidu Baike gives the origin of this expression as from \u300a\u793c\u8bb0\u00b7\u5b66\u8bb0\u300b, and explains \u4e50\u7fa4 as:\n\n\u4e50\u7fa4\uff1a\u4e50\u4e8e\u4e0e\u597d\u670b\u53cb\u76f8\u5904\n\nWilling to socialize with friends.\n", "topic": "chinese", "url": "https://chinese.stackexchange.com/questions/26476"}, {"image": "VQAonline_00006596.png", "question": "Pronunciation Of \u4f5b in \u56db\u5ddd\u8bdd", "context": "The 1986 edition of\u300a\u56db\u5ddd\u65b9\u8a00\u8bcd\u5178\u300bhas the following entry:\n\n\n\u4e50\u5c71\u7684\u5927\u4f5b\u2014\u2014\u8001\u77f3\u4eba\nLo2 san1 ni1 da4 fu3\u2014\u2014nao3 si2 ren2\n\u4e50\u5c71\u5927\u4f5b\u8ddd\u4eca\u5df2\u6709\u4e00\u5343\u591a\u5e74\u5386\u53f2, \u6240\u4ee5\u662f\u201c\u8001\u77f3\u4eba\u201d\u3002\u8c10\u201c\u8001\u5b9e\u4eba\u201d\u3002 \u25b7\u4f60\u4ee5\u4e3a\u4ed6\u662f\uff5e\u55e6? \u4ed6\u624d\u9634\u5012\u4e0d\u8001\u5b9e\u3002\n\n\u4f5b is given the pronunciation of fu3\nThe 2014 edition of the same dictionary gives \u4f5b the pronunciation fu2\n/note: this dictionary is riddled with errors(!)/\nLogically I would have assumed the pinyin to have been written fo2 - which is pretty much how it is pronounced, as far as I can tell.\n\u300a\u56db\u5ddd\u65b9\u8a00\u8bcd\u5178\u300bhas fu3 and fu2\n\nAre Either fu3 or fu2 correct pronunciations for \u4f5b in \u56db\u5ddd\u8bdd?\n\n", "answer": "I tried the \u6f22\u8a9e\u65b9\u8a00\u5927\u8fad\u5178, and there are many places with the fu pronunciation in the southwest mandarin area, for example \u4f5b\u8c46 (= \u8836\u8c46, fava bean) is pronounced fu tou in \u7562\u7bc0; \u4f5b\u74dc (\u74e0\u74dc gourd) is pronounced fu gua in \u81ea\u8ca2, etc. Not really surprising; \u5f17 is fu in Mandarin.\n", "topic": "chinese", "url": "https://chinese.stackexchange.com/questions/14122"}, {"image": "VQAonline_00006773.png", "question": "What does this MYSQL image say? (Database Locks)", "context": "I am reading a mysql article about different kind of locks and this image appeared:\n\nUnfortunately, I cannot copy paste the words because it's in an image. I also don't know how to reproduce this with my keyboard.\nIf anyone could kindly translate this for me, would be very kind.\n", "answer": "Databases are not my main domain, but I double checked all terms. Here you go.\nBlue central box\n\u9501 = lock\nGreen box\n\n\n\n\nBox\nZH\nEN\n\n\n\n\nmain\n\u52a0\u9501\u673a\u5236\nlock engine\n\n\nupper leaf\n\u4e50\u89c2\u9501\noptimistic lock\n\n\nlower leaf\n\u60b2\u89c2\u9501\npessimistic lock\n\n\n\n\nPurple box\n\n\n\n\nBox\nZH\nEN\n\n\n\n\nmain\n\u517c\u5bb9\u6027\ncompatibility\n\n\nupper leaf\n\u5171\u4eab\u9501\nshared lock\n\n\nlower leaf\n\u6392\u4ed6\u9501\nexclusive lock\n\n\n\n\nOrange box\n\n\n\n\nBox\nZH\nEN\n\n\n\n\nmain\n\u9501\u7c92\u5ea6\nlock granularity\n\n\nupper leaf\n\u8868\u9501\ntable lock\n\n\nmiddle leaf\n\u9875\u9501\npage lock\n\n\nlower leaf\n\u884c\u9501\nrow lock\n\n\n\n\nYellow box\n\n\n\n\nBox\nZH\nEN\n\n\n\n\nmain\n\u9501\u6a21\u5f0f\nlock mode\n\n\n1\n\u8bb0\u5f55\u9501\nrecord lock\n\n\n2\ngap\u9501\ngap lock\n\n\n3\nnext-key\u9501\nnext-key lock\n\n\n4\n\u610f\u5411\u9501\nintention lock\n\n\n5\n\u63d2\u5165\u610f\u5411\u9501\ninsert intention lock\n\n\n\n", "topic": "chinese", "url": "https://chinese.stackexchange.com/questions/45466"}, {"image": "VQAonline_00006558.png", "question": "What does \u9905\u4e7e mean in Taiwan?", "context": "I'm unfamiliar with the variety of Mandarin spoken in Taiwan, so I was surprised to see a chilli-snack \u9999\u8106\u6912 referred to as \u9905\u4e7e:\n\nhttp://mall.tw100years.com/goods.php?id=9\n\u771f\u4e0d\u6562\u76f8\u4fe1\u9019\u771f\u662f\u7f8e\u5473\u7684\u9905\u4e7e\uff01\u770b\u8d77\u4f86\u662f\u7d05\u7d05\u7684\u8fa3\u6912\uff0c\u5403\u8d77\u4f86\u662f\u9999\u3001\u8106\u5916\u52a0\u9ede\u6de1\u6de1\u7684\u9e79\uff0c\u4e0d\u4f46\u80fd\u5e6b\u60a8\u6392\u53bb\u5bbf\u4fbf\uff0c\u9084\u80fd\u990a\u984f\u7f8e\u5bb9\uff01\n\nThe definition of \u9905\u4e7e that I'm familiar with is biscuit or cracker, a baked pastry food. This snack doesn't fit that definition; it is made of fried chilli peppers:\n\n", "answer": "I am Taiwanese, and I have even had this \"\u9905\u4e7e\" once.\nGenerally, we can refer to almost every snack that is made with flour and \"cracks\" in your mouth as \u9905\u4e7e. So when you say you want some \u9905\u4e7e, people will not only give you crackers, but also cookies, potato chips, wafer cookies, wafer rolls, mille feuille, etc. These things have their own specific names, of course!\nEven these are \u9905\u4e7e: Jagabee Fries, and Pocky.\nI would also like to clarify that this sentence is not in Taiwanese. It is in Standard Chinese, but we have our own accent and wording. Just like the differences between American English and British English. We normally speak two languages in everyday life, i.e. Chinese and Taiwanese, a Min dialect. \n", "topic": "chinese", "url": "https://chinese.stackexchange.com/questions/5287"}, {"image": "VQAonline_00006616.png", "question": "Is this a rare/variant character (combining \u7c73 and \u56e3), or not really a Chinese character?", "context": "\nThis character is on the label of a riceball sold at 7-11 in Taipei. I believe this kind of riceball is originally Japanese.\nIt seems to be some kind of variant of \u7cf0 / \u56e2 but is actually composed of \u7c73 on the left plus \u56e3 on the right. But I can't find it anywhere on the Internet or in Unicode so far.\nIs it a known variant, or is it possibly normally considered a Japanese character? On Wiktionary, \u56e3 is listed only as a Japanese shinjitai character. (Shinjitai is a simplified character made in Japan.)\nOr maybe it's some kind of fake character made up by 7-11, like a trademark?\n", "answer": "\u7cf0 is \u7c73 + \u5718. \u5718 simplified is \u56e2. \u56e2 has a variant character \u56e3.\nThus \u7c73 + \u56e3 = \u7cf0.\n\u56e3 itself is a variant chacter of \u7cf0.\nOn the other hand it could just be a clever way of writing \u7c73\u5718 = \u7c73\u56e2 = rice ball.\nEdit: It is written \u5fa1\u98ef\u5718 in all instances I can find regarding the ad. It is a private/funny simplification that has no formal recognition.\n", "topic": "chinese", "url": "https://chinese.stackexchange.com/questions/20827"}, {"image": "VQAonline_00006646.png", "question": "What is the meaning of \u539f\u6e6f\u5316\u539f\u98df\uff1f", "context": "In the show \u4eba\u6c11\u7684\u540d\u7fa9 the suspect doesn't want to accompany the protagonist on account of his soup getting cold. The protagonist replies \"\u8b1b\u7a76\u554a\"\uff08meticulous\uff01\uff09and then:\n\n\u539f\u6e6f\u5316\u539f\u98df\n\nNone of my many Pleco dictionaries lists \u539f\u6e6f or \u539f\u98df\uff0calthough \u539f\u6e6f was my phonetic keyboard's first suggestion!\n\n", "answer": "\u539f\u6c64\u5316\u539f\u98df is a kind of eating habit, especially exists in northern areas; means drinking the soup of the dumplings (or noodles etc) after eating them. e.g. for the case of dumplings, \u539f\u6c64 refers to the water (or you can think it as soup) which used for boiling dumplings, \u539f\u98df means dumplings.\n\n\u4e00\u822c\u6765\u8bb2\uff0c\u201c\u539f\u98df\u201d\u6307\u7684\u662f\u6dc0\u7c89\u7c7b\u98df\u7269\uff0c\u6bd4\u5982\u997a\u5b50\u3001\u9762\u6761\u3001\u9984\u9968\u3001\u6c64\u5706\u7b49\uff0c\u201c\u539f\u6c64\u201d\u5c31\u662f\u6c34\u716e\u8fd9\u4e9b\u98df\u7269\u540e\u5f97\u5230\u7684\u6c64\uff0c\u800c\u201c\u5316\u201d\u6709\u201c\u6d88\u5316\u201d\u7684\u610f\u601d\u3002\u6240\u4ee5\u201c\u539f\u6c64\u5316\u539f\u98df\u201d\u7684\u610f\u601d\u662f\uff0c\u6c64\u80fd\u591f\u5e2e\u52a9\u6dc0\u7c89\u7c7b\u98df\u7269\u6d88\u5316\u5438\u6536\u3002\n\n(In general, \u539f\u98df refers to starchy foods, like dumplings, noodles, wontons, glue puddings etc, \u539f\u6c64 refers to the water which used for boiling these foods. \u5316 means digestion, so \u539f\u6c64\u5316\u539f\u98df means the soup could help digesting starchy foods.)\nThere're several various explanations about why people doing this, someone believe \u539f\u6c64 could help digestion; someone believe \u539f\u6c64 is nourishing. I prefer to that it's just a habit; people like drinking something after eating and with \u539f\u6c64 you don't need to cook the soup separately. And I think this article gave a nice summary:\n\n\u6240\u4ee5\uff0c\u771f\u8981\u8f83\u771f\u7684\u8bdd\uff0c\u201c\u4e13\u5bb6\u201d\u4eec\u63d0\u4f9b\u7684\u201c\u539f\u6c64\u5316\u539f\u98df\u201d\u7684\u201c\u79d1\u5b66\u9053\u7406\u201d\u5b9e\u5728\u662f\u5f88\u7275\u5f3a\u9644\u4f1a\u3002\u5f53\u7136\uff0c\u9762\u6c64\u672c\u8eab\u5e76\u6ca1\u6709\u4ec0\u4e48\u4e0d\u597d\uff0c\u4e0e\u767d\u5f00\u6c34\u76f8\u6bd4\uff0c\u5b83\u5176\u4e2d\u8fd8\u6709\u4e00\u4e9b\u6dc0\u7c89\u751a\u81f3\u7ef4\u751f\u7d20\u3002\u5403\u5b8c\u9762\u540e\u559d\u70b9\u9762\u6c64\uff0c\u591a\u5c11\u4e5f\u7b97\u662f\u8282\u7ea6\u7cae\u98df\u7684\u8868\u73b0\u3002\u5982\u679c\u771f\u7684\u559c\u6b22\u559d\uff0c\u201c\u559c\u6b22\u201d\u5c31\u662f\u6700\u597d\u7684\u7406\u7531\u3002\u5982\u679c\u975e\u8981\u5806\u780c\u4e00\u4e9b\u79d1\u5b66\u672f\u8bed\u6765\u7ed9\u5b83\u4e00\u4e2a\u201c\u79d1\u5b66\u89e3\u91ca\u201d\uff0c\u53ef\u80fd\u5c31\u662f\u5f04\u5de7\u6210\u62d9\u3002\n\n", "topic": "chinese", "url": "https://chinese.stackexchange.com/questions/27088"}, {"image": "VQAonline_00006837.png", "question": "What is meant by \"men have been raised up from the fall to far greater heights?\"", "context": "In Catholic theology it is said that \"men have been raised up from the fall to far greater heights.\"1\nWhat is meant by this and how are the \"heights\" of a different degree than our first parents' state in the Garden of Eden?\nA fuller answer will also include the Orthodox meaning should they have the same theology.\n1. E.g. Francisca Javiera del valle in \"ABOUT THE HOLY SPIRIT\".\n\n[...] because the mercy of the divine Word decided to raise him up from\n his fall to even greater heights.\n\n\n", "answer": "From the Catechism of the Catholic Church: \n\nCCC 412 But why did God not prevent the first man from sinning? St. Leo the Great responds, \"Christ's inexpressible grace gave us blessings better than those the demon's envy had taken away.\" and St. Thomas Aquinas wrote, \"There is nothing to prevent human nature's being raised up to something greater, even after sin; God permits evil in order to draw forth some greater good. Thus St. Paul says, 'Where sin increased, grace abounded all the more'; and the Exsultet sings, 'O happy fault,. . . which gained for us so great a Redeemer!'\".\n\nIn brief, in Eden we had no Saviour, the Grace had not been shown to the fullest, and our \"greater good\" was out of reach.\n", "topic": "christianity", "url": "https://christianity.stackexchange.com/questions/34480"}, {"image": "VQAonline_00006869.png", "question": "What does Mary look like in the Fatima Apparitions?", "context": "\nBeginning in the spring of 1917, the children reported apparitions of\n an Angel, and starting in May 1917, apparitions of the Virgin Mary,\n whom the children described as \"the Lady more brilliant than the Sun\".\n (quoted in Wikipedia) *.\n\n*Was this report accurate? How did Mary really look like in the Fatima Apparitions according to trusted Catholic sources? \n\n(We are not alone in the Universe, Konrad Kulczyk, Sixth Books 2012)\n", "answer": "What does Mary look like in the Fatima Apparitions?\nYour quotation from \u201cWe are not alone in the Universe\u201d by Konrad Kulczyk, seems to be at odds with Catholic statements about Our Lady and her apparitions at F\u00e1tima in 1917.\nKonrad Kulczyk seems to almost make the whole situation the imaginations of three little children. Nothing is further from the truth.\nYes, it may be possibly true that both Lucia and Blessed Jacinta never stated that the Lady never called herself the Mother of Jesus nor the Mother of God however she did describe herself as \u201cthe Lady of the Rosary\u201d in her final apparition!\n\nIt was Mary's final appearance, on Oct. 13, 1917, that became the most famous. In his book \"Looking for a Miracle,\" Joe Nickell states that \"an estimated 70,000 people were in attendance at the site, anticipating the Virgin's final visit and with many fully expecting that she would work a great miracle. As before, the figure appeared, and again only to the children. Identifying herself as 'the Lady of the Rosary,' she urged repentance and the building of a chapel at the site. After predicting an end to [World War I] and giving the children certain undisclosed visions, the lady lifted her hands to the sky. Thereupon Lucia exclaimed, 'The sun!' As everyone gazed upward, and saw that a silvery disc had emerged from behind clouds, they experienced what is known [as] a 'sun miracle'.\" - The Lady of F\u00e1tima & the Miracle of the Sun\n\nWikipedia\u2019s quotation is very close to how Lucia described Our Lady of the Rosary, who is no other than Mary the Mother of Jesus! Nothing more is known.\n\nOn May 13, 1917, ten-year old Lucia Santos and her cousins Jacinta and Francisco Marto were herding sheep at a location known as the Cova da Iria near their home village of Fatima, Portugal. Lucia described seeing a woman \u201cBrighter than the sun, shedding rays of light clearer and stronger than a crystal ball willed with the most sparkling water and pierced by the burning rays of the sun\u201d. \n\nAs for Kulczyk, never have I come across any statement comparing Our Lady to the imagery that Mary must have seemed like a 3\u20198\u201d of 1.1 m tall to the imagination of some 12 or 15 year old child. What nonsense. I have read many, many books on F\u00e1tima and have never read about any such conclusions by a Catholic author on this subject. Kulczyk is definitely wet behind the ears.\nSorry Kulczyk: Mary wore a blue dress all the way down to feet and not simply a skirt just to her knees like you mention.\n\nThe first Marian apparition occurred on 13 May 1917 as the children were tending to their sheep. They had had their lunch, and, as was their habit, had started to pray the rosary when they heard \u201clightning in a clear sky.\u201d Fearing a storm was approaching, they decided to head home. As they prepared to leave, a strange light in the sky moved toward them. The children reported that, although astonished, they felt no fear. They claimed a beautiful lady appeared before them, \u201cabove the holm oak tree, wearing a pure white mantle, which was edged with gold, which went to her feet. In her hands the beads of a rosary shone like stars, with its crucifix the most radiant gem of all.\u201d During this first visit the apparition told the children not to be afraid, that she came from Heaven. She asked them to pray the rosary and to return to the same spot on the thirteenth of every month. Then the apparition returned the same way it had come. - The Marian Apparitions in F\u00e1tima as Political Reality: Religion and Politics in Twentieth-Century Portugal\n\n", "topic": "christianity", "url": "https://christianity.stackexchange.com/questions/77865"}, {"image": "VQAonline_00006868.png", "question": "Wall numbers in German church / during German burial ceremony", "context": "I have just came about a 10 years old photo of my aunt's burial ceremony that took place in June 2009 in one of the German churches (in a small city close to Oldenburg).\nUnfortunately, relation both with my aunt and her living family was/is very weak, so I can't even say, if she was Christian or Protestant and thus, in which church following photo was taken?\n\nDoes anyone know what are these numbers printed on the wall in the right part of the photograph?\nMy first and only guess was that these are song numbers that will be sing during the ceremony. But then, what are those additional numbers (or actually some kind of value ranges)? And why to print song numbers permanently on the wall, if they keep to changed (depending on ceremony, day of year etc.) and thus should rather be displayed using some non-permanent way. Like a display etc.\n", "answer": "The numbers are on little (wooden) panels that can be pushed in a (wooden) rack. In your case both panels and rack have the same colour a the wall, so they are a bit difficult to see.\nThe numbers give the songs to sing during the service. The red numbers give the verses to sing.\nOn the denomination: First I thought it is a Catholic Church (esp. because the seats in the sanctuary), but the numbers given lead to songs in the Catholic song book \"Gotteslob\" (then used 1975 edition) not fitting on a burial. The songs of the protestant \"Evangelisches Gesangbuch\" fit better. So probably it was a protestant (near Oldenburg: lutheran) service, but I may be mistaken.\n", "topic": "christianity", "url": "https://christianity.stackexchange.com/questions/71102"}, {"image": "VQAonline_00006848.png", "question": "Is there a customary way to go around rosary beads?", "context": "The picture below depicts one way to go around the rosary beads when praying the Holy Rosary.\n\nIf a child were to ask, as they typical do, as to why Catholics when praying the Holy Rosary customarily go around the rosary beads one way rather than the other - if that's what they do, how would one answer? Is there a customary way to go around rosary beads? If yes, what is the reason for that order?\n\nA question in the \"same series\" as: Is there a particular order in lighting candles on an advent wreath?\n", "answer": "Either direction is acceptable\nThis might be semi-authoritative since it comes straight from the only approved Marian apparition site in the United States. Up near Green Bay, Wisconsin there is an outdoor rosary walk with the mysteries visible for people who traverse in a right-hand or left-hand direction.\nFor instance, when you walk around clockwise, you see 1st glorious mystery, 1st luminous. Etc... On a big banner, but if you look at the backside of the banner, its got 5th glorious, 5th luminous, etc... So no matter whether you walk around clockwise or counter-clockwise, you will see the mysteries in their proper order. Which I remarked to my wife, \"that's mighty clever, those Catholics are always thinking...\"\n", "topic": "christianity", "url": "https://christianity.stackexchange.com/questions/46337"}, {"image": "VQAonline_00006862.png", "question": "Would Mormons accept communion, administered by a representative of another Christian denomination, as valid?", "context": "There are considerable differences in some beliefs between Catholic and Protestant churches on one hand, and the Mormon church on another hand.\nOne of these is the belief in Trinity due to which, according to the answers to this question and this question, Lutheran/Protestant churches do not view Mormons as Christian. Another difference is the symbols used with the communion, being that Mormons use water instead of wine (as is explained in the answers to this question).\nNow considering some more exceptional situation, such as when serving in U.S. military during a war, as in the book cover shown below, would Mormons accept communion from military chaplains that may not be Mormons?\nOr would a Mormon in this case not participate in the communion, or have the communion on their own (or possibly request a Mormon chaplain)?\n\n", "answer": "A better answer would come from an actual LDS war veteran, but I would imagine that a Mormon in the military may participate in an available ceremony as a symbol of friendship, unity, and support with the rest his regiment. Although they may participate, Latter-day Saints maintain that ordinances are only authorized when exercised by the Priesthood of God, which is had His Church only (the LDS Church). That's not to say the ceremony by the non-LDS chaplain is not beneficial to the Latter-day Saint\u2014it still provides him a blessed opportunity to worship with other Christians who share the same fundamental beliefs\u2014it just is not binding in heaven. In fact, I think any decent member of the LDS Church would encourage, not disparage, participation in available Christian worship services.\nYou can find out more about the LDS Church and military relations on its website: https://www.lds.org/callings/military-relations/organization-leader-responsibilites?lang=eng and https://www.lds.org/callings/military-relations/military-chaplains?lang=eng. There's nothing particularly quotable but it should give good background. I think the point is that this is up to the individual member's conscience. Other questions and answers on this site go into detail about the Priesthood authority necessary for ordinances to be valid in heaven.\nAs an aside (that may or may not be relevant), I have personally met and informally interviewed a Latter-day Saint astronaut who commanded a couple Shuttle missions. I asked about how he worships in space. Even he, a Priesthood holder, needed to obtain authorization from his local Priesthood leader (bishop or stake president) before the mission to be able to administer the Sacrament if the opportunity arose. Of course, this was done privately in the extremely limited personal time allotted. But the point is, the administration of ordinances that are considered officially valid before the Lord is well organized, even... in space.\n", "topic": "christianity", "url": "https://christianity.stackexchange.com/questions/60888"}, {"image": "VQAonline_00006816.png", "question": "Crucifixion -- torture stake or cross?", "context": "The Jehovah's Witnesses state that Jesus was not nailed on a cross, but on a torture stake. The New World Translation of the Bible is phrased accordingly. For example:\n\nMatthew 10:38 (NWT)\nAnd whoever does not accept his torture stake and follow after me is not worthy of me.\n\n(All the other translations (that I know of) translate cross instead.)\nWhat is significant about the difference between a torture stake and a cross, and on which one was Jesus crucified?\n\nAn illustration of what might be a torture stake:\n\nCrux Simplex by Justus Lipsius (1547-1606). Wikimedia Commons.\n", "answer": "There is comparatively little evidence in the NT for the shape of the object. There is not much etymological help from the Greek words \u03c3\u03c4\u03b1\u03c5\u03c1\u03cc\u03c2 or \u03be\u03cd\u03bb\u03bf\u03bd, which do not clearly indicate either shape.\nSome of the textual hints we do have are:\n\nBut [Thomas] said to them, \u201cUnless I see the nail marks in his hands and put my finger where the nails were, and put my hand into his side, I will not believe.\u201d (John 20:25, NIV)\n\nThis suggests that there was more than one nail placed through the hands, which is more likely on a cross than on a stake - though I could imagine a nail through each hand, with the hands side by side. But the JWs assert that the plural is referring to two nails, one through both hands and the other through both feet. The Greek reads \"\u1f10\u03bd \u03c4\u03b1\u1fd6\u03c2 \u03c7\u03b5\u03c1\u03c3\u1f76\u03bd \u03b1\u1f50\u03c4\u03bf\u1fe6 \u03c4\u1f78\u03bd \u03c4\u03cd\u03c0\u03bf\u03bd \u03c4\u1ff6\u03bd \u1f25\u03bb\u03c9\u03bd\", which is \"in his hands the imprint of the nails\", clearly linking multiple nails to the hands specifically.\n\nAbove his head they placed the written charge against him: THIS IS JESUS, THE KING OF THE JEWS. (Matthew 27:37, NIV)\n\nIf Jesus were crucified as in the picture, the sign would be obscured by his arms. If it were also placed above his hands, Matthew could have written \"Above his hands...\" - but he didn't.\n\nCarrying his own cross, he went out to the place of the Skull, which in Aramaic is called Golgotha. (John 19:17, NIV)\n\nSeveral different crucifixion methods were used by the Romans, including the traditional cross and the simple vertical stake. But the reference to carrying suggests that Jesus carried the crossbeam or patibulum of his cross - contemporary sources indicated that condemned people could be made to carry their own patibulum, but not the entire cross or stake, and of course a stake has no crossbeam at all. (Now-familiar depictions of Jesus carrying the whole cross come along later.)\nIt is common ground between Jehovah's Witnesses and the rest of us that for most of Christian history, Christ's crucifixion has almost always been regarded as being on a cross. The problem that the JWs have with it is that they think that the cross is a pagan symbol which was mistakenly (or maliciously) adopted early on in Christian history, as Christian belief and practice departed from its original form. As a restorationist group, they believe that in rejecting the cross shape they are returning to Christianity as it was meant to be. Insistence on the 'torture stake' is not particularly important in itself, but it is a prominent reminder of their distinctiveness, and of the error into which they think others have fallen.\n", "topic": "christianity", "url": "https://christianity.stackexchange.com/questions/2109"}, {"image": "VQAonline_00006830.png", "question": "Why was Dan removed from the list of 12 Israel tribes in Rev 7?", "context": "Here is the list of 12 tribes of Israel from Genesis, Numbers and Revelation.\n\nI sorted them according to the list in Genesis 29-30.\nIn Numbers, Levi is replaced by Ephraim and Manasseh who replaced Joseph also.\nBut in Revelation 7 Dan is removed and replaced by Manasseh.\nWhy are these list different? Why is Dan removed from the list?\n", "answer": "Dan fell into idolatry in Judges 17 and 18, and was thus removed:\nJudges 18:30-31:\n\nAnd the people of Dan set up the carved image for themselves, and\n Jonathan the son of Gershom, son of Moses, and his sons were priests\n to the tribe of the Danites until the day of the captivity of the\n land. So they set up Micah's carved image that he made, as long as the\n house of God was at Shiloh.\n\nAlso, Ephraim is missing as well. Hosea 4:17 explains why:\n\nEphraim is joined to idols; leave him alone.\n\nSee this link for more details. \n", "topic": "christianity", "url": "https://christianity.stackexchange.com/questions/14725"}, {"image": "VQAonline_00006900.png", "question": "What do these large bubbles in my French press indicate?", "context": "I've seen these bubbles when pressing coffee that I've home roasted in two different presses with two different water sources. I've seen it with coffee from multiple roasts now. The beans are the same in all cases (Rwandan single origin). I am grinding with a burr grinder. Do these large bubbles in the bloom indicate anything about the bean or the roast?\n\n", "answer": "The bubbles are the result of \"degassing\" \u2014 carbon dioxide escaping from the coffee. Degassing will be especially dramatic in the first 24 hours after roasting, during which 40% of the CO2 leaves the bean. See this blog post: Understanding Degassing.\n", "topic": "coffee", "url": "https://coffee.stackexchange.com/questions/2952"}, {"image": "VQAonline_00006889.png", "question": "What is the purpose of the crimp on a moka funnel?", "context": "Spurred by this recent moka question...\nThe funnel on my moka pot has a \"crimp\" -- a little ring-shaped indentation -- around the middle, similar to this one from Bialetti:\n\nWhat is the purpose of this crimp?\nIt is indented toward the middle of the funnel (i.e., not protruding out of the funnel). It's far too low to indicate recommended \"fill line\" for quantity of grounds (for my taste anyway!) There's no other attachment that fits into it, nor does it engage with any other part on the pot (e.g., the top lip of the funnel sits on the moka pot base). It's one piece of sheet metal; e.g., this isn't a point where two pieces attach. Some moka pots have funnels that have this crimp, and others are solid/flat around the side.\n", "answer": "The crimp is there to support the reducer! The reducer slots into the funnel, and sits on the ridge. You then put coffee on top of it, allowing you to use less coffee in larger pots.\n", "topic": "coffee", "url": "https://coffee.stackexchange.com/questions/2025"}, {"image": "VQAonline_00006894.png", "question": "What is the standardized naming of this type of coffee?", "context": "What is the \"standard\" name for this type of coffee in the various countries (I mean made from these type of machines with the paper filter)?\n\nI have noticed when travelling that in some coffee spots in various countries in Europe, this type of coffee is not available. Unless it has a different naming than I am aware.\nIs this called American coffee? Because I know that Americano is a type of espresso so it is not the same\n", "answer": "The drip-type coffee is available in many countries in Europe. A list of names:\n\nFilterkaffee in Germany / Austria / Switzerland \nfilterkaffe in Denmark and Sweden\nfilterkoffie in the Netherlands\nkawa filtrowana in Poland\ncaf\u00e9 filtre in France / Switzerland / Belgium\ncaf\u00e9 filtro in Spain\ncaff\u00e8 preperato con il filtro in Italy / Switzerland\n\nCan you see a pattern here: Always a combination of coffee and filter/filtered. The terms are so closely related that if you order \"filtered coffee\" an attentive waiter should be able to help you even if you don't know the precise term at your destination.\nNote that if you order \"coffee\" without specifying the preparation, you will be served the local \"default\". This will actually be filtered coffee in many countries of Northern and Central Europe, in Italy, as counter-example, you will get what you'd call \"espresso\", likewise in Spain or Portugal: small, strong. \nAsking for caff\u00e8 americano will perhaps get you an espresso diluted with hot water in some places in Italy (sources vary whether it's a remenant from WW II or started with international tourism), sometimes caff\u00e8 all'americana (\"coffee American-style\") will get you drip coffee. Likewise in touristy areas of Spain, caf\u00e9 americano might get you a diluted coffee.\nFurther north you won't be understood. They will probably direct you to the nearest Starbucks.\n\nSource:\nLiving and travelling in Europe for decades plus some research for the precise terms.\n", "topic": "coffee", "url": "https://coffee.stackexchange.com/questions/2275"}, {"image": "VQAonline_00006892.png", "question": "Where can I get Koiwai coffee milk delivered to Europe?", "context": "Since I went to Japan on holidays I've been craving the coffee milk that I've had over there, but I haven't been able to get it at home.\n\nNo local asian stores stock it and I haven't been able to find any online stores (although admittedly I have been looking at only English-speaking sites) that have any available - or deliver to Europe.\nI'm not sure what it is about it, but i've been thinking about it ever since. Is there somewhere online where I can order some?\n", "answer": "You can buy them from Rakuten. They ship internationally, with the exception of some countries, due to them not sufficing to the standards. \nI don't believe there to be such a limit in Europe though, so you should be fine.\nIt costs \u00a5 3,369 which comes down to roughly $27. And no account is required to order. \n", "topic": "coffee", "url": "https://coffee.stackexchange.com/questions/2084"}, {"image": "VQAonline_00006925.png", "question": "Are these white bits mold?", "context": "I bought some coffee beans today. When I poured them into a jar, I found some white bits. Are they mold? Thanks\n\n", "answer": "Based on the picture I believe that these white spots are something form when he beans have been harvested or they have been sugar and sprinkled when the manufacture was selling this product.\n", "topic": "coffee", "url": "https://coffee.stackexchange.com/questions/5898"}, {"image": "VQAonline_00006902.png", "question": "What is a FlavorLock\u2122 bag?", "context": "What is a FlavorLock\u2122 bag? Why it is important for wrapping coffee product or even pet food? Does it have any effect to the product inside if we use an ordinary bag? \n\n", "answer": "This question is actually answered on the website of the coffee producer.\nTo summarize, the roasting process of coffee produces carbon dioxide inside the beans, which takes a while to find its way out. Sealing the coffee in tight containers would be problematic, as the carbon dioxide would stay in the beans in this way (making the coffee sour) or may let the bag burst over time.\nHowever, not sealing coffee bags is not a solution either, as exposure to air lets the coffee get stale after a while. As supermarket coffee is unfortunately often stored for long period of time before sold, this would be problematic.\nSo the solution is to either (1) use a one-way valve that allows the carbon dioxide to escape from the bag without the air coming into the bag, or (2) to let the beans degas before sealing them airtight in bags. Solution (1) is probably best for fresh coffee, so it is used here. I don't know what problem is solved with such valves in pet food containers.\n", "topic": "coffee", "url": "https://coffee.stackexchange.com/questions/3081"}, {"image": "VQAonline_00006921.png", "question": "What type of coffee maker is this?", "context": "I got a stove top coffee maker, from my grandfather (not antique, he bought it recently). I am not sure what type it is. I claims to be a percolator, but functions more like a moka pot. They have given the nozzle to pour, in the top chamber. It heats water in bottom chamber, and the water passes through coffee powder, placed in between, and a brown liquid falls in the top, which is what I drink.\nThis is an Amazon.in link to that piece:\nhttps://www.amazon.in/Pigeon-Xpresso-Stainless-Coffee-Perculator/dp/B072F17LRP/\n\n", "answer": "This is a typical moka pot.\nThe confusion may stem from the fact that the principle of a moka pot is somewhat similar to a percolator, just without a back flow (sort of a \u201cone way percolator\u201d), and in some descriptions, including the German Wikipedia article on moka pots, the expression \u201cpercolator\u201d is used.\n", "topic": "coffee", "url": "https://coffee.stackexchange.com/questions/5484"}, {"image": "VQAonline_00006989.png", "question": "What is the difference between apparent motion and apparent movement?", "context": "A definition for apparent movement is:\n\nThe perception of movement produced by stimuli that are stationary but are presented first at one position and then, at an appropriate time interval, presented at a different position.\n\nExample:\n\n\nAnd apparent motion is defined as:\n\nMotion can be detected even if there is no real motion\n\nExample:\nFlip Books\n\nNow to my actual question: I know that those two forms of perception are very close; so what is the actual difference between the two? Is there a pure definition of the difference?\nI feel that both apparent movement and apparent motion are according to Gleitman et al. (2010) pretty much the same, and could be applied to both sorts of examples. So if presented with a stimulus/illusion, how would one approach them to discern which is which?\n", "answer": "There is no difference between the two; illusionary movement and motion are interchangeable terms. For example, a definition of apparent motion is (Oxford Index): \n\nA sensation of movement in the absence of actual movement, [for example] visual illusions [...]\n\nAnd a definition for apparent movement (Psychology Dictionary) is:\n\n[A]n illusion of motion or change of size that is cued by visual tricks.\n\nIn other words, apparent movement and apparent motion are the same thing. The two visual illusions you provide, though, are caused by two different neurophysiological phenomena altogether. \nThe flip book relies on persistent vision where multiple images are blurred into a coherent perception over time. \nThe rotating spokes illusion is thought to be caused by asymmetric luminance steps that trigger the motion detectors in the brain.\n", "topic": "cogsci", "url": "https://cogsci.stackexchange.com/questions/16089"}, {"image": "VQAonline_00006990.png", "question": "Brain Right hemisphere is random and left hemisphere is linear? really?", "context": "For long I've seen uncountable number of blogs, websites, tutorials, booklets about career-guide, self-improvement, meditation, parenting etc. claiming; \n\n\"Right hemisphere loves to work random, creative way; in contrast left-hemisphere loves to follow instructions and discipline.\" \n\"Right hemisphere takes part in lateral-thinking; in contrast left-hemisphere takes part in vertical thinking.\" \n\"Right hemisphere deals with pictures, visions, senses, feelings etc; in contrast, Left hemisphere deals with formula, equation, numbers, names, letters, calculations etc.\" \n\"Right hemisphere works in nonlinear, meshy manner; in contrast, Left hemisphere works in linear, rigid, sequential way. \n\"Right hemisphere see the whole, big-picture. Left hemisphere see one part only\".\n\netc. \nSome example here , here, here , here, here, here et cetera. \nThus I developed a concept about behavior of 2 hemisphere. \n\nNow While I was going through following research article (of 2000): \nCerebral specialization and interhemispheric communication by M. Gazzaniga (pdf) , \nI stuck on a diagram telling the exact opposite. \n\n(URL: https://d1gqps90bl2jsp.cloudfront.net/content/brain/123/7/1293/F16.medium.gif)\n(I've slightly edited the screenshot to enlarge the text and utilize the whitespace). \nThe only thing of the paper maches with popular view, is; the right hemisphere is taking important part in understanding pictures, geometric shapes, extrapolating to fill a gap in an image, etc. \nBut according to this diagram; actually Left hemisphere is site for viewing the 'big-picture' of a subject ('living things'), finding 'similarity' (sings, eats, has a etc). Also From personal experience, the way of thinking shown as relational-map at left hemisphere, is the cause of 'randomness' in me, so that I keep many books and files scattered while working; or was broken line (que) in school to collect a grass or stone. The thing shown at left side forces me to collect various objects and arrange them... not the rightward one. Also I zone-out too-much and day-dream because of the left-one... not the right-one. \nIn contrast, clearly the right-hemisphere in this diagram, is processing information in much rigid, linear or sequential, disciplined way; and looking a small-part... not the big-picture. \nSo, please could anyone explain me, why should I still have to consider left hemisphere as a rigid linear servant and right-hemisphere as rulebreaker daydreaming artist?\n\n(P.S. most of these popular-series's tell that Right hemisphere is intuitive and Left hemisphere is logical. But I can't believe this because I can't separate logic and intuition. Logic is the bridge between intuition and reality.)\n", "answer": "You shouldn't. It is one of those stubbornly persistent myths. It is almost entirely false. It is true that in some tasks one half of the brain is more prominent (although even in supposed classic cases like speech both halves are actually involved, just one half more than the other).\nBut the idea that one half of the brain is logical and the other intuitive or emotional is simply false. At most, one half the brain is slightly more prominent in processing some tasks that may be perceived by us as more logical or more intuitive, but that is really as far as it goes.\n", "topic": "cogsci", "url": "https://cogsci.stackexchange.com/questions/16096"}, {"image": "VQAonline_00006993.png", "question": "What challenges the ideasthesia balance theory has to answer?", "context": "Ideasthesia \"can be defined as a phenomenon in which activation of concepts produces phenomenal experience\". The examples vary from the Kiki/Bouba phenomenon applied to (possibly) everyone to actually seeing colors in graphemes like in synesthetic people. According to the theory, \"there is a strong similarity between semantics and sensations\".\n\n(source: ieet.org)\nFigure: Sensation vs. meaning balance shown for hypothetical pieces of entertainment (red), art (yellow) and science (white), as they are broken down into their components. Only an art piece is consistently located in the gray area.\nGiven that this theory is pretty young (first creation in Wikipedia was in 2012 by presumably one avid contributor on the field, based on their username), I wonder if there are any review or critic on the theory. What challenges does it need to answer? Which theory is it incongruent with?\nSource:\n\u2022 Wikipedia: Ideathesia\n\u2022 TED-Ed: Ideasthesia: How do ideas feel? - Danko Nikoli\u0107\n\u2022 IEEE: Danko Nikoli\u0107, Ideasthesia and Art\n\n", "answer": "Short answer\nIdeaesthesia does not comprise a theory or even a hypothesis. It is merely a term meaning 'sensing concepts' or 'perceiving meaning'. \nBackground\nAs Steven Jeuris also comments, ideaesthesia is not a theory, it is simply a term. It is a combination of two Greek words, namely 'concept' or 'idea', and the other is 'sensation', or 'aisthesis'. In translation, ideaesthesia means sensing concepts or perceiving meaning (source: Danko Nikoli\u0107).\nIt it used to denote phenomena as the Kiki/Bouba effect (Milan et al., 2013; Martinez & Milan, 2015) and synesthesia (J\u00fcrgens & Nikoli\u0107, 2012) as you write. These phenomena are associated with assigning shapes to names, and colors to graphemes (and the likes), respectively. As far as I can see, ideaesthesia does not explain anything, it is not an observation, it is not a hypothesis, let alone a theory. It is a term used to group phenomena where people associate seemingly unrelated concepts.\nFurther, for what it's worth, a cursory Google Scholar search on 'ideaesthesia' yields few hits (82), and those hits that pop up, in part referenced here, point sometimes to questionable journals or to irrelevant papers. I would hence use the term cautiously as it doesn't seem to be widely used. \nReferences\n- J\u00fcrgens & Nikoli\u0107, Translat Neurosci (2012); 3(1): 22-7\n- Martinez & Milan, V Int Conf Synesthesia: Science and Art. Alcal\u00e0 la Real de Ja\u00e9n. Espa\u00f1a. 16\u201319th May 2015\n- Milan et al., J Consciousness Studies, 20(1-2 \n", "topic": "cogsci", "url": "https://cogsci.stackexchange.com/questions/17933"}, {"image": "VQAonline_00006986.png", "question": "How many times on average do people wake up per night?", "context": "It would be best if the answer could be broken down by age and sex, and even by other variables (alcohol-drinker vs teetotaller, etc).\nIf there's a difference between self-report data and data from studies that observe people sleeping, I'm interested in that too.\nThis image, \n\nwhich I took from here, implies that waking up several times is normal, but the article doesn't provide any information on averages.\nIt may be that the median is more useful than the mean in this context.\n", "answer": "Based on this paragraph from the Wikipedia article on hypnograms\n\nOn a hypnogram, a sleep cycle is usually around 90 minutes and there are four to six cycles of REM/NREM stages that occur during a major period of sleep.\nMost SWS occurs in the first one or two cycles; this is the deepest period of sleep. The second half of the sleeping period contains most REM sleep and little or no SWS and may contain brief periods of wakefulness which can be recorded but are not usually perceived.\nThe stage that occurs before waking is normally REM sleep.\n\nAnd this image (also from wiki):\nSample hypnogram showing one sleep cycle (the first of the night) from NREM through REM\nA normal and healthy person usually wakes for a brief time after his/her REM stage was finished in each sleep cycle. So, if we consider A 7\u20138-hour sleep probably includes four cycles, and each cycle include at least one REM, then each person at least wake four times.\n", "topic": "cogsci", "url": "https://cogsci.stackexchange.com/questions/15926"}, {"image": "VQAonline_00007009.png", "question": "Why does \"bulk\" yellow ink look red?", "context": "A transparent blue inkjet cartridge looks deep blue, a red one looks deep red, but a yellow one looks red. Tea also looks yellowish when it's shallow and reddish otherwise. Red is another colour, a different wavelength, so why does deep yellow look red?\n\n", "answer": "Not all liquid or things that are yellow become red when they exist in large quantity. For example, beer, vegetable oil, and normal urine never look red even in large quantity, like in a vat. It really depends on the optical properties of the liquid and the suspended particles in it.\nThe optical properties of something depend on both its intrinsic factors such as its reflectance, transmittance, and absorptance of the light and the extrinsic factors such as its thickness, the surface that it\u2019s on, the angle of light incidence, and the polarization of the incident light. (This is not to mention psychological factors like the mood when you look at that thing or the surroundings colors or compositions of that thing, which can make that thing look different.)\nThat\u2019s why a thin film of water is colorless but a large body of water like the lake looks blue. And that\u2019s why there is a phenomenon called iridescence \u2013 the phenomenon that certain surfaces appear to change color as the angle of view or the angle of illumination changes \u2013 like changing rainbow colors on a soap bubble or on a colorless CD surface.\nIn your case, I think the color reflection and transmission properties of the ink are different. The ink probably reflects mainly the yellow color, so it looks yellow when reflection of colors is the major event \u2013 as when the ink is painted on a surface. But the ink probably transmits more of the red color than the yellow color, so it looks red when transmission of colors is the major event \u2013 as when you look through it.\nN.B. I think you\u2019ll probably get better answers than mine by posting the question on the Physics forum.\nReferences.\n\nII.8. Reflection, Transmission, and Absorption Gigahertz-Optik.\nElements that Affect the Appearance of Color Konica Minolta.\nIridescence Wikipedia.\n\n", "topic": "cogsci", "url": "https://cogsci.stackexchange.com/questions/24942"}, {"image": "VQAonline_00006966.png", "question": "Effective sampling rate in human visual system", "context": "The [stroboscopic effect][1] is often explained as one of the problems of sampling. If sample rate is too low, you might have the impression of the signal frequency being low or even reverse. There are many animations on the internet describing the process but I find static sinus function graph more comprehensible:\n\nYou definitely know this effect from car wheels or plane propeller in movies. That's because the movie captures certain amount of frames. If the event is significantly shorter than capture rate, the output doesn't make any sense anymore.\nMy question is, how can I observe the stroboscopic effect in full light with my very own eyes? I thought there's no fixed frame-rate for human vision. I thought it's an analogous process (continuous stream), rather than digital (timed sampling).\n\nQ: How can be human vision in non-flashing (such as sun) light be subject to stroboscopic effect? Where does the damage happen?\n\n", "answer": "Yes, you can experience the stroboscopic effect under continuous illumination. This wikipedia article does a nice job of summarizing relevant findings.\nAs mentioned in the article, there are two competing theories for how the stroboscopic effect happens under continuous illumination in the human visual system. The first theory is that human visual processing is like a movie camera and it operates on roughly discrete frames. The second theory is that neural adaptation is responsible: the brain adapts to the very regular patterns of motion, which can cause visual after-effects where the opposite pattern is perceived. Overlaying the actual motion with the visual after-effect will 'cancel out' the perceived motion.\nKline and Eaglemen (2008) present evidence against the discrete sampling hypothesis and supporting the adaptation hypothesis. Macdonald et al. (2014) present evidence for the discrete sampling account.\n", "topic": "cogsci", "url": "https://cogsci.stackexchange.com/questions/8716"}, {"image": "VQAonline_00007006.png", "question": "How do we know what colors animals perceive?", "context": "Humans have three kinds of photoreceptors corresponding to different wavelength-spectra:\n\nIf say photons with wavelengths of around 530 nanometers hit the human retina, this would lead to the perception of the color conventionally called \"green\". The color \"green\", however, is subjective in the sense that it is bound to a certain cortical physiology. \nSo my question is: Given that we don't have access to how animals perceive color, as we can't infer such qualitative properties purely from physical stimulus properties, how can we conclude that some animals are, say, perceiving ultra-violet colors given certain items or wavelengths? What if they would perceive entirely different colors than known to us?\n", "answer": "There are some levels of confusion in this in question, but basically we can test\n\nwhat wavelengths animals perceive (simple behavioral tests, e.g. training & testing them to distinguish a certain wavelength)\ninspect their cones to determine what wavelengths they could perceive. The latter doesn't necessarily imply that if an animal has say 4 different cones, it has tetrachromatic vision, because there's some degeneracy in some species, which unlike in humans (which have 3 types of cones and trichromatic vision) don't really use all their cones.\n\nTo give you a nearly random example of such a paper (this species doesn't have degeneracy), a (freely available) paper on chicks' vision:\n\nThe colour vision of domestic chicks (Gallus gallus) was\n investigated by training them to small food containers\n decorated with tilings of grey and coloured rectangles.\n Chicks learn to recognise the colour quickly and\n accurately. Chicks have four types of single-cone\n photoreceptor sensitive to ultraviolet, short-, medium- or\n long-wavelength light. To establish how these receptors\n are used for colour vision, stimuli were designed to be\n distinguished only by specific combinations of receptors.\n We infer (1) that all four single cones are used, and (2) that\n their outputs are encoded by at least three opponency\n mechanisms: one comparing the outputs of ultraviolet- and\n short-wavelength-sensitive receptors, one comparing the\n outputs of medium- and long-wavelength receptors and a\n third comparing of the outputs of short- and long- and/or\n medium-wavelength receptors. Thus, the chicks have\n tetrachromatic colour vision. These experiments do not\n exclude a role for the fifth cone type, double cones, but\n other evidence suggests that these cones serve luminance-based tasks, such as motion detection, and not colour\n recognition.\n\nIf you want to get philosophical qualia discussions, akin to whether all humans describe the same color with same language, that's a whole different kettle of fish. I don't know of any studies like that in the absence of language. I'm guessing you might be familiar with the dress.\n", "topic": "cogsci", "url": "https://cogsci.stackexchange.com/questions/21883"}, {"image": "VQAonline_00007379.png", "question": "Why do I have lumps in my caramel? (NOT crystallized sugar.)", "context": "There are a lot of caramel questions. Hopefully I'm not repeating anyone...\nI've made caramel sauce several times with the recipe from Sally's Baking Addiction. (But using a different beginning procedure: Dissolve sugar in ~1/2 cup of water, not stirring. Put the lid on for a few minutes to clean the sides. After that it's the same.) For some reason, though, whenever I make it I end up with lots of tiny lumps suspended in my caramel. As I mention in the title, these lumps are not little sugar crystals. I've had my caramel crystallize before, and this is very different. They're soft, for one thing, and so I'm not sure what's happening.\nAt first I thought the little lumps were just air bubbles. That's what they look like. But they actually have some body to them. It's almost like I have tiny bits of cooked egg or something (which is obviously impossible.) They have a gelatinous quality. So my guess is that the cream is doing something weird? Flavor is good, thickness is good, everything is good except these stupid little lumps.\nDoes anyone know what's happening, and how I can make it stop...? I'm about to take an immersion blender to the mess and let God sort it out.\nEDIT/UPDATE: So... I now have a theory that I tend to get my caramel so dark (more of a crimson/burnt sienna than an amber. Probably around 380F) that some of the dairy solids in the butter (maybe even the cream?) cook/coagulate as soon as I add it, leading to those gelatinous lumps. I haven't had the time (or a good reason) to try cooling my syrup with some water before I add the butter yet, but as I said in the original question, this definitely isn't sugar crystallization, and it's been a persistent problem. When my mom's made the recipe before, she didn't have the problem, but she's also terrified of burning her caramel and tends to add the butter much, much sooner.\nI've noticed brown specks when I've made chewy caramels and toffee as well, so I'm wondering if cooked and browned dairy solids are my whole problem here.\n\n", "answer": "So.... I've finally figured it out.\nMy cream's been curdling because I get my syrup too hot and too dark. \nMost caramel recipes recommend heating the initial sugar syrup to no more than 380 degrees, and for the most part, people will get scared and pull it off the heat and add their cream before it even gets close to 380. I know I did when I first started! But without realizing it, I moved beyond those fears, enjoying progressively darker caramels. \nHowever, one thing I was not aware of (and should have learned sooner) is that as sugar decomposes further, creating darker caramel, it becomes more acidic. Maybe not acidic enough in its own right to curdle heavy cream, but heat accentuates the curdling effect of acidity on casein and it's friends. I currently have a jar of caramel \"sauce\" in my fridge with a layer of dark caramel liquid on the bottom, and lighter, solid dulce de Leche reminiscent curds on top. \nTo which I said \"the FRIG is going on???\" I busted out the thermometer and actually added my cream at a wussy 380. It's wayyyy too sweet for my taste, but only a few shreds of cooked/curdled cream floated to the top. That seems significant. \nWhile I can't find a decent chart that measures the pH of caramel at various stages of decomposition, I found lots of references to increasing acidity, and even to milk curdling as a result. Cream is more stable than milk, but the anecdotal stuff I found is still relatable. I have tiny curds instead of big ones, but that doesn't change their nature. \nEasy fixes? Don't cook my sugar so hot is the obvious. One idea I haven't tried yet is to make my caramel with a pinch of baking soda, although I might not like the results as much. (I'd try and see, but two batches of caramel in 24 hours is enough for me) \nThe reason the answer wasn't obvious to anyone is because the recipe I started with never discusses temperature, so I never checked mine, and that's what made my situation unusual. I was trying to make a caramel sauce with a sugar solution approaching baker's caramel. \nThe moral of this story: if you're EVER following a recipe that instructs you to eyeball something, and then it comes out wrong, check your eyeballs. That means take precise measurements of what you're currently eyeballing and compare to accepted standards in other recipes. \nEg, I should have measured the temperature of my caramel syrup when I added the cream and took it off the heat, as well as noting my recipe's proportions, then compared to the temperatures and proportions of other accepted recipes to see how mine was different. \nI could have saved myself a LOT of curdled caramel if I'd realized that 400 degrees is not caramel sauce standard sooner. \n", "topic": "cooking", "url": "https://cooking.stackexchange.com/questions/80770"}, {"image": "VQAonline_00007068.png", "question": "Desperately Seeking Chicken Haddies", "context": "A staple of my Gran's kitchen when I was a child, Chicken Haddies (unbelievably I found a graphic of the product she favoured, below) was boneless haddock meat wrapped in wax paper then canned (a process unique in my experience.) \n\nShe was raised in Southern Ontario but spent decades of her life in mining camps in Ontario, Qu\u00e9bec, and Newfoundland. My question is, is anyone aware if Chicken Haddies are still sold anywhere? An internet search finds references to several canneries in Atlantic Canada but the companies have no internet presence. \nPlease help, Chicken Haddies made the best chowder EVER!\n", "answer": "Apparently the chicken haddie plant had fire damage and we have not been able to buy it for over a year now in New Brunswick, Canada. I too grew up with it for making the best fish cakes. Anyway, the plant is up and running again as of January 2012 and I've been buying at Sobeys.\n", "topic": "cooking", "url": "https://cooking.stackexchange.com/questions/16949"}, {"image": "VQAonline_00007512.png", "question": "What is the white substance floating in my fresh mozzarella brine?", "context": "I purchased fresh mozzarella bocconcini from Whole Foods six days ago. The mozzarella is sold in a brine solution. I opened the package when I bought it and have kept it refrigerated since, using the original packaging and brine solution. Today, I opened the package again to see some white substance floating in the brine as shown below. The mozzarella smells fine, and based on eating one bite, also tastes normal.\n\nWhat is this substance, and is my mozzarella still safe to eat?\n", "answer": "Those are just milk solids.\nQuality mozzarella is sold floating in whey, the leftover liquid from making cheese. While most of the milk solids are gone from the whey -- that's what the cheese is made from -- not all of them are. As such, some will coagulate in the fridge, floating on top of the whey.\nNow, if the floaters were any color other than white, or if they were fuzzy, that would mean mold. But those are really just cheese bits.\n", "topic": "cooking", "url": "https://cooking.stackexchange.com/questions/100302"}, {"image": "VQAonline_00007360.png", "question": "Bumps on carbon steel seasoning, can I repair it", "context": "I've just got a new De Buyer carbon steel pan and I'm 3 seasons in (oven method) and I'm noticing quite an uneven surface (especially little bubbles). I've tried as best as possible to keep the layers of oil used very thin. I'm wondering if I should strip it down and start over or just forge ahead?\n\n", "answer": "I think you are fine. I personally feel the best way to continue to season a pan is to actually just to cook in it. The first things I like after the oven or range top seasoning is to use onions, green onions and other inexpensive ingredients. Perhaps also to deep fry some onion rings or french fries as well. This will have the correct heat and also plenty of oil to seep into the pan. Hope this helps. \n", "topic": "cooking", "url": "https://cooking.stackexchange.com/questions/77250"}, {"image": "VQAonline_00007237.png", "question": "Old-Fashioned Candy/Deep-Fry Thermometer - How Shallow Can I Go?", "context": "I have a thermometer like this:\n\nToday I'm making tostadas. I don't need or want deep oil for that, just over a centimeter would be plenty. In order to submerge the bulb of this thermometer in oil that shallow, the bulb would almost be touching the bottom of the pan. Would I get even close to an accurate reading that way? Is there an accepted minimum depth for using a thermometer of this type? \n", "answer": "That is probably not deep enough to get an accurate reading. It varies with the thermometer, but usually you need at least a couple of centimetres to be submerged to get an accurate reading. Also, unless you are super careful, it will be nearly impossible to stop the thermometer touching the bottom of the pan. In my experience the clips on these doohickies are terrible for adjusting the angle of the thermometer.\nWhen shallow frying in this manner I usually just go by rules of thumb. One that has never let me down is to use a wooden chopstick. When you think the oil is getting hot enough, place the tip of the chopstick in the oil. If bubbles rapidly appear around the chopstick, you're hot to trot.\n", "topic": "cooking", "url": "https://cooking.stackexchange.com/questions/49654"}, {"image": "VQAonline_00007410.png", "question": "Is this a bad apple?", "context": "I cut my apple open this morning and found that it had an unexpected consistency emanating from the core. I couldn't find much about what's going on here other than maybe this is some kind of mold (seems unlikely). \nWhat's going on with this apple?\nThis particular apple was part of an Amazon Fresh delivery. I've had about 10 of them up to this point, all of which were good apples.\nI took a bite before throwing it out. It tasted fine and the texture is the same as far as I can tell (dense, fibrous). Ultimately I decided to toss it erring on the side of caution.\n\n", "answer": "This appears to be watercore, specifically radial watercore, from the image you have posted. It should be safe to eat, according to this site and this site.\nAccording to an article from Washington State University, watercore is a disorder of some apple variants, which can cause the internal flesh to appear glassy [shiny and translucent] later into the growing season. In some studies, low levels of calcium have been shown alongside the condition. As well, it should be noted that the condition 1) does not occur after harvesting 2) may cause an increase of sweetness, and 3) cause the apple to quickly degrade during storage.\nAs to the causes of this disorder, there are a number of theoretical factors for causation: Genetic defects, water regiment, temperature, minerals, leaf-to-fruit ratio, maturation and ripening, sorbitol metabolism and altered transport. These are discussed to some degree in both the WSU article and this MSU article\n", "topic": "cooking", "url": "https://cooking.stackexchange.com/questions/84939"}, {"image": "VQAonline_00007261.png", "question": "Ginger starting to sprout - can I still use it?", "context": "I have two beautiful ginger roots (rhizomes) which I had planned to turn into gari (pickled ginger). But I also have two children and the ever-changing schedule that goes with this. So the ginger sat in a bowl for a while, waiting.\nWhile Wikipedia states that the rhizome\n\n...it is immediately scalded, or washed and scraped, to kill it and prevent sprouting.\n\nmine obviously weren't and appear to have gotten spring fever and started to sprout:\n\nNow, I could put them in a planter, let them grow and harvest in fall when they start to wither, but before I sneakingly sell them to hubby as the latest addition of our ever-growing collection of plants I'm wondering:\nCan I use sprouting ginger? Just like regular/dormant ginger? Or is there something to keep in mind?\nMy main focus is on the culinary aspect: While sprouted ginger isn't toxic, I'm wondering whether it requires special or different preparation, how to handle the sprouts, and do sprouting (\"growing\") and mature (\"dormant\") ginger taste differently in dishes.\n(Should I decide to plant my ginger, I will ask the guys over at Gardening SE for advice, if required...) \n", "answer": "From a culinary perspective, I find that when sprouted, the ginger just loses a bit of flavour, that's all. If you let it grow for a very long time and it becomes all shriveled, then you are talking about a flavourless piece of ginger. I've even used such shriveled and even moldy (cut the mold off, though) pieces... no flavour at all!\n", "topic": "cooking", "url": "https://cooking.stackexchange.com/questions/57645"}, {"image": "VQAonline_00007790.png", "question": "Paper's preservation (fixative not working)", "context": "Apparently, my (acid-free) paper (from 'SoHo Studio'; shown below) is yellowing, I have Krylon Workable Fixatif Spray Coating.\nObviously, fixatives prevent this issue to happen, what should I do?\nIs there anything special I should do to my paper?\nI don't think you can see the yellowing, but just trust me, it is yellow\n\n", "answer": "Paper will yellow over time because of lignin in the pulped wood.\nChoosing an archival, acid-free fixative means that the fixative itself should not yellow over time. Unfortunately, the primary purpose of the fixative is to prevent smudging of the media that is on your paper. While it can slow down paper yellowing over time (by reducing the surface area of the paper that is exposed to air), it doesn't neutralize those oxidizing chemicals.\nThere are ways to treat the paper to prevent the yellowing from getting worse, and you can also put it behind UV-blocking glass to protect it from further oxidization due to air and light. However, the current yellowing may not be reversible.\n", "topic": "crafts", "url": "https://crafts.stackexchange.com/questions/9018"}, {"image": "VQAonline_00007748.png", "question": "Advice on preserving an old painting", "context": "The image attached is of a painting which is deteriorating around the edges of the frame. The paint appears to be falking off of the canvas. \nIt is between 70 and 80 years old, I don't know what type of paint although I would guess oil. \nIs there anything I can do to prevent further damage?\n\n", "answer": "You should consult a professional conservator.\nIf you're not even sure what kind of paint was used for this painting, chances are high anything you do'll cause more damage.\nA conservator can see what kind of paint was used, what may have caused the damage (moisture, too much tension, deterioration of the paint?) and how to effectively stop the damage from spreading. Usually they would have a look at the painting and give you an estimate of the current value as well as an cost estimate for the preservation or repair before you have to decide whether or not you want to engage them.\n", "topic": "crafts", "url": "https://crafts.stackexchange.com/questions/5763"}, {"image": "VQAonline_00007677.png", "question": "How do you properly store tins of turpentine?", "context": "Is there a proper way to store tins of turpentine? I have attached a picture of the type of container my turpentine comes in:\n\nIn the past where ever I store the container, turpentine smell would inevitably leak out and it's not really a pleasant smell. Initially I had stored it in the corner of my bedroom closet. I quickly moved it to a storage closet built into the stairwell. It's dark and cooler than room temperature usually.\nI also made the mistake of putting the tin of turpentine inside several plastic bags. This helped with the smell that leaked out but when I took the tin out of the plastic bags, the whole tin was sticky with what I assume is the dried turpentine gas trapped inside the plastic bag.\n", "answer": "In a general sense it is a very good practice to refer to the material safety data sheets (MSDS) for products like this. The product labels, in theory, should also contain similar information. \nMSDSs will always contain more information though. They will break down all the information needed about the product when it comes to safety. It should be safe to assume that householdproducts.nlm.nih.gov is a good source of this information. I located the sheet for your specific product. Most other MSDS's for turps cover the exact same sentiment as far as storage and handling is concerned. For completeness sake I have also included some basic handling as well in the information quoted below.\n\nHANDLING:\n\nIsolate from oxidizers, heat, sparks, electric equipment & open flame. \nUse only with adequate ventilation. Avoid breathing of vapor or spray mist. Avoid prolonged or repeated contact with skin. Wear OSHA Standard goggles or face shield. Consult Safety Equipment Supplier. Wear gloves, apron & footwear impervious to this material. Wash clothing before reuse. \nAvoid free fall of liquid. Ground containers when transferring. \nDo not flame cut, saw, drill, braze, or weld. Empty container very hazardous! Continue all label precautions!\n\nSTORAGE:\n\nKeep in fireproof surroundings. \nKeep separated from strong oxidants, incompatible materials. Keep cool. Keep inside a well-ventilated room.\nKeep container tightly closed & upright when not in use to prevent leakage.\n\n\nText is copied as is. Formatting and emphasis is my own.\nKeep it cool, ventilated and away from sources of oxidants and sources of ignition. Ensure the container is seal and cleaned between uses. \n\nI am not sure about the cause of the leaking. Chris Johns has a good theory though. Assuming you follow the instructions above it should not happen. \nRelated I found a discussion between users and the smells created by turpentine like yours and artist grade. Consensus is that the \"cheap\" stuff has a more offending odor \n", "topic": "crafts", "url": "https://crafts.stackexchange.com/questions/598"}, {"image": "VQAonline_00007668.png", "question": "How can I laminate with contact paper without leaving bubbles?", "context": "I have to cover some books with adhesive contact paper. Although I try to do it slowly and with the help of a ruler, I systematically get some bubbles between the book and the cover.\nIs there a good trick to do that properly?\n\n", "answer": "Try removing the paper off the plastic foil during the process, not before like removing a little bit of foil, sticking it on the edge, then pulling away the paper flat on the book.\n", "topic": "crafts", "url": "https://crafts.stackexchange.com/questions/183"}, {"image": "VQAonline_00007691.png", "question": "How to keep scratch art paper clean and free of pencil and eraser marks?", "context": "I was introduced to scratch-art in high school and have loved it ever since. The problem I repeatedly encounter is in keeping the surface of the paper unmarked where I want it to remain black. I have tried a number of solutions as far as a storage is concerned, like keeping it between two pieces of paper and even buying a folder carrier for my projects to keep them protected. I don't wear jewellery when I work and that helps a lot.\nI find though, that my faint pencil sketch I do before starting often shows up and is difficult to erase. the pencil is very shiny and reflects a lot more light than the paper does. The eraser makes more ugly marks than the pencil does. Further, I haven't figured out what to do if I do make a mistake. \nDoes anyone have tips for preventing ugly marks on scratchboard paper or fixing them once they've occurred?\nIs there a suggested medium to sketch with on this sort of paper to lay out the outline of a design?\nHere's an example of one of my projects:\n\n", "answer": "I have not done any sort of serious scratch-board art, but here are some suggestions, to answer your two part question:\nUse black colored pencil instead of regular pencil.\u2014 it isn't as reflective as regular pencil, so you will just barely be able to see it while you're working.\nPaint over the nicks/stray pencil marks\u2014 If you paint over the scratches and stray pencil marks with ink, it will probably cover it up and not show. You should try this on a scrap piece of paper first, to be sure.\n", "topic": "crafts", "url": "https://crafts.stackexchange.com/questions/2342"}, {"image": "VQAonline_00007765.png", "question": "How to Trim Poofy Sleeves", "context": "I purchased a boho maxi dress a while ago. \nI lost weight and it is now big for me.\nHow to narrow the sleeves to maintain their poofy shape?\nCan I simply trim along the seam of the sleeve, or is there something more involved?\nI know how to alter the bodice as I've had successful experience (based on this tutorial).\nAnd I'm unsure if I'd attempt to narrow the skirt (length is absolutely perfect though) - I'll save that for another question if the time comes.\nHere is a pattern of the dress in question (note the poofy arms)\n\n", "answer": "An assembled sleeve isn't much more than a tube of fabric, but completely disassembled, it looks more complicated.\nIf you are inexperienced with tailoring, this \"quick and dirty\" tutorial looks like a good start: How to tailor a sleeve. It requires you to put on the shirt (or dress) inside out, gather the surplus fabric at the existing seams and pin it to your desired size. You then open the original seams, sew along the pin lines and trim and neaten your new seams.\nWhat it doesn't explain is how you match the new sleeve seam to the existing side seam of the bodice.\nIf you want to reduce the size of the bodice as well, it's perfect to do both at the same time. Otherwise you either need to match the new sleeve seam up to the original side seam (which preserves the puffiness at the shoulders) or you need to sew the sleeve seam in a crooked line back down to the side seam (which artificially narrows the arm hole). \nI suggest going with the first solution, because it's the neatest and an armhole that's too narrow is very uncomfortable. This solution is shown in this video: Resize oversized sleeves\n\nThe professional way is to dissassemble the sleeves, adjusting the cut of the pieces and reattatching them to the dress. Of course this is more complicated, but if the \"quick and dirty\" solution doesn't look good, this is the only way to make it look perfect.\nFirst of all, mark which sleeve is the left one and which is the right one, because they may differ very slightly from each other and sewing the left sleeve to the right armhole looks strange and feels uncomfortable.\nThen take your measurements as described in Create custom sleeve pattern. Keep in mind that you want to measure your desired sleeve, so if you don't want it to be skin tight, don' t measure skin tight.\nFollow that same instruction and construct your new sleeve on a big sheet of paper. Once you're done, measure the circumference of the armhole in your bodice. Then measure the length of the \"sleeve crown\", the curved line on top, from seam to seam. The sleeve crown should be slightly longer than the circumference of the arm hole.\nIf your sleeves have cuffs like shown in the picture, reattach them after closing the long sleeve seam.\nAt the end, sew the sleeves back onto the bodice as shown here: How to insert sleeves.\n", "topic": "crafts", "url": "https://crafts.stackexchange.com/questions/7522"}, {"image": "VQAonline_00007774.png", "question": "Is it okay to cast off for neckline near crosses (cables)?", "context": "My wife is knitting me this wonderful cabled sweater pattern: https://knitrowan.com/en/free-patterns/cole and she is about to finish the back panel. However, the very last RS row before casting off for the neck has a Cr12F and a Cr12B in it. Then there was a WS row, and then casting off for the neck. Before she continues doing the front as well, the cast-off stitches look a bit twisted because of these two large crosses. Will this fix itself when picking up these cast-off stitches and knitting the ribbing around the neck, or will this cause a strangely twisted shape in the whole neck? If so, what is the best way to fix this? \nE: added a picture\n", "answer": "Looking at the photo of the front, there is a crossing in the cable just two rows below the cast off/pick up for the neck.\nI would go with the pattern, and see how the stitches work out in the casting off.\nWhen you see them pulling so much that you do not like it you can unpick that bit of casting off, drop all the stitches of the cable, let them run down to just below that last cross and pick them up in knit without the twist. A crochet needle and a (short) cable needle will make the picking up easier.\nAlternatively, you can just forget to make that last crossing, (not following the pattern,) but do so on both sides of the front and/or back, not just on the left or on the right (easy mistake to make.)\nYou can make the last crossing on the front but not on the back, if you think that is best. \n", "topic": "crafts", "url": "https://crafts.stackexchange.com/questions/8118"}, {"image": "VQAonline_00008270.png", "question": "Recurrence equation calculation", "context": "I have a recurrece equation\n\nChanged notations:\n$\\qquad\\displaystyle G(W) = \\max \\{ G(W - s_i) + v_i \\mid 0 \\leq i \\leq n, w_i \\leq W \\}$,\nI am not sure if I understand $\\ w_i $ correctly. \nOn each iteration my $ w_i \\leq W $ . So my $ w_i $ will be previous $ W-s_i $ since G takes one argument?\n", "answer": "Here is pseudocode for computing your function:\n\nParameters:\n\nInteger $n$\nArrays $W,V,S$ of length $n$\n\nfunction G(w):\n\nIf $w = 0$, return $0$\nLet $\\mathrm{max} \\gets -\\infty$\nFor $i$ from $1$ to $n$:\n\nIf $W[i] \\leq w$:\n\n\n\nLet $x \\gets G(w-S[i])+V[i]$\n\n\n\n\nIf $x > \\mathrm{max}$, let $\\mathrm{max} \\gets x$\n\n\nReturn $\\mathrm{max}$\n\n\nI used the notation $W[i],V[i],S[i]$ rather than $w_i,v_i,s_i$.\n", "topic": "cs", "url": "https://cs.stackexchange.com/questions/90662"}, {"image": "VQAonline_00008023.png", "question": "Confusion in CLRS's version of Prim's algorithm", "context": "The algorithm is as follows:\nMST-PRIM(G,w,r)\n1 for each u \u2208 G.V //initialization\n2 u.key = \u221e\n3 u.\u03c0 = NIL\n4 r.key = 0\n5 Q = G.V //end initialization\n6 while Q \u2260 \u2205\n7 u = EXTRACT-MIN(Q)\n8 for each v \u2208 G.Adj[u]\n9 if v \u2208 Q and w(u,v) < v.key\n10 v.\u03c0 = u\n11 v.key = w(u,v)\n\n\nAll vertices that are not in the tree reside in a min-priority queue Q based on a key attribute.\nv.key is the minimum weight of any edge connecting v to a vertex in the tree\nv.\u03c0 points to the parent of v in the tree\nG is a graph, w is a weight function, r is a root\n\nThe example given is as follows:\n\nI am confused, why in step (c), edge $(b,c)$ is added instead of edge $(a,h)$. Below the diagram, there is a note saying:\n\nIn second step, the algorithm has a choice of adding either edge $(b,c)$ or edge $(a,h)$ to the tree since both are light edges crossing the cut.\n\nHowever still I think that according to the line 8 in algorithm, all adjacent vertices of $a$ not in the tree must be added first to the tree. Thus $h$ should also get added immediately after $b$, before $c$. \nThe line 8 says for **each v** \u2208 G.Adj[u](i.e. for each adjacent vertex $v$ of $u$), why only adjacent node $b$ of $a$ is considered, but not $h$ (which is also adjacent to $a$).\nIf what author says should occur, then there should be something like break construct on line 12 inside the if construct's body, which will result in exiting for loop and hence grabbing next u = EXTRACT-MIN(Q).\nAm I correct? or I must be missing something very stupid. What's that?\n", "answer": "\nQuestion 1: However still I think that according to the line 8 in algorithm, all adjacent vertices of $a$ not in the tree must be added first to the tree. Thus $h$ should also get added immediately after $b$, before $c$.\n\nYou are confusing the resulting MST tree (denoted $T$) with the intermediate priority queue $Q$. \n\nAll the vertices are already in $Q$ at the end of initialization (Lines 1 ~ 5). However, $T = \\emptyset$ at this time point.\nA vertex $u$ is extracted from $Q$ and becomes a member of tree $T$ at line 7.\nIn the loop starting from Line 8, vertice $v$ satisfying the requirements of Line 9 are not inserted into the tree $T$; instead their keys in the priority queue $Q$ are changed (in fact, decreased).\n\n\nQuestion 2: \"My point is when the line 8 says \"for each adjacent vertex $v$ of $u$\", why only adjacent node $b$ of $a$ is considered, but not $h$ (which is also adjacent to $a$).\"\n\nYes, $h$ is also considered, but in the way different from what you describe. In the iteration for $a$, both $b$ and $h$ are considered (satisfying Line 8 and Line 9): their keys are updated (i.e., decreased) in the priority queue $Q$ (Line 11) and their parents are updated accordingly (Line 10). (Note that these updates are not finalized and may be updated again during the algorithm.) \nThe key point is: neither $b$ nor $h$ has been added into the resulting MST tree $T$. Vertex $b$ will be extracted from $Q$ and added into $T$ in the next iteration at Line 7 (shown in step (b)).\nAlso note that the figure in CLRS only shows the changes on $T$; it does not show the states of $Q$.\n", "topic": "cs", "url": "https://cs.stackexchange.com/questions/50964"}, {"image": "VQAonline_00008274.png", "question": "How to find vertices of bounded region made by intersection of lines", "context": "Suppose we have random lines made with 2 points. and point has (x,y)\nFor example:\n\nNow when we draw a random line you will see many lines intersect with each other. This eventually gives rise to bounded regions. How to find the vertex points for each bounded region?\nMy approach:\nFirst we find all the intersection points of the various lines. In some cases multiple lines intersect at the same point.\nHowever I am not sure how to pick the points that make up a bounded region.\nThanks in Advance.\n", "answer": "Finding the all the vertices for all of the bounding regions is roughly the same amount of work as creating a doubly connected edge list (DCEL) that represents your regions, so I will show how to construct a DCEL from a set of line segments. After that, finding all the boundary points for each region from a DCEL is a simple operation.\nFirst, we find all line segment intersections between the segments we start with. This can be done efficiently with the standard sweepline algorithm for line segment intersection, which is described in various references, such as in these slides. This can be done in $O(n\\log n + K)$ time, where $n$ is the number of line segments and $K$ the number of intersections between these lines.\nFor the next part, we construct a set of endpoints $P$ and a set of line segments $L$, where for each segment $l\\in L$, we also store the pair of points $(p,q)$ from $P$ that are the endpoints of segment $l$. First, add all original line segments to $l$ and the corresponding endpoints to $P$.\nThen, at every intersection point $p$, split the segments that intersect at that point into two segments 'separated' by $p$, store the point $p$ in the set $P$, remove the old segment from $L$ and add the new segments to $L$, setting their endpoints accordingly.\nIn the end, we have represented our regions with a graph $G=(P,L)$, where the vertex set $P$ are all points in our 2D space (including the intersection points) and the set of edges $L$ contains all segments connecting these points. Note that the graph $G$ doesn't have any crossing edges by construction, so it is planar.\nFrom this planar graph, we can construct a DCEL, as shown in this answer, which is mostly bookkeeping.\nIn the DCEL, each region is associated with a representative half-edge. From that half-edge, we can find all half-edges that form the border of the region. Then, the vertices of the region are all vertices of those half-edges.\n\nIt is possible to skip the DCEL and find the vertices directly, but likely you need to do almost the same thing. The main advantage of using a DCEL as an intermediate step is that someone else already has made an algorithm to construct a DCEL.\n", "topic": "cs", "url": "https://cs.stackexchange.com/questions/90880"}, {"image": "VQAonline_00008338.png", "question": "Why does RAID-5 require an additional disk for parity blocks?", "context": "I know that RAID-5 consists of block-level striping across multiple disks, but using an additional parity-check block on each disk .. and that at least two disks are required for striping.\nAnd it's obvious that each parity block is specific to each disk it belongs to (and so there is no need for allocating an additional disk).\n\nImage from Wikipedia.\nHowever I've been unable to understand why in fact there is an additional disk required for parity checks, as I found on this article:\n\nThe minimum number of disks in a RAID 5 set is three (two for data and\n one for parity).\n\nAny idea?\n", "answer": "I think you've misunderstood what the parity data is. They're not parity checks, so it's not true that \"each parity block is specific to each disc it belongs to.\" The parity data is to allow recovery from a failed disc.\nLet's go back to RAID-4 for a second, and assume we have three discs: discs $0$ and \u00a0$1$ are data and disc\u00a0$2$ is parity. \"Parity\" means that the $b$th block of disc\u00a0$2$ is the xor of the $b$th blocks of discs $0$ and\u00a0$1$. The point is that, if any single disc fails, we can recover its data because the $b$th block of any disc is the xor of the $b$th block on the other two discs. For this to work, it's crucial that the parity data is on a separate discs. If you only had two discs and put the parity data on those discs (e.g., each disc was two-thirds data blocks and one-third parity blocks) then the failure of a single drive would destroy some blocks and their corresponding parity data, so you'd be unable to recover the data using just what was left on the remaining disc.\nRAID-5 is the same idea except that, instead of putting all the parity data on the last disc, it's spread across all the discs. So, for a three-disc set-up, a third of the blocks would have parity data on disc\u00a0$2$, a third on disc\u00a0$1$ and a third on disc\u00a0$0$.\nThe point of using RAID-5 rather than RAID-4 is that every time you write data, the corresponding parity block must be updated. If all parity data is on the same disc, that disc will be written to much more than the other discs ($k$\u00a0times as much, in a $k$-disc sytem), so it will fail faster. Spreading the parity data across the discs evens out the wear on them.\n", "topic": "cs", "url": "https://cs.stackexchange.com/questions/99180"}, {"image": "VQAonline_00008253.png", "question": "timed automata - advance only in certain states", "context": "I am currently looking into timed automata for a project.\nI am thinking about a timed automaton where a clock only advances when the automaton is in a certain state.\nHowever, my little knowledge in the domain makes me wonder whether somebody already defined timed automata like that.\nAs an example:\nI.e. I have a timed automaton with two states: L0 and L1.\nMy clock is c - the total time that the automaton spent in time L1.\nThe clock c should only advance when the automaton is in state L1.\n(see upper part of drawing)\nSolution with my existing knowledge (not very elegant):\nI can see a way where I could use event-clock automata (1) to achieve this, by triggering an event a and on entering L1 the value of c is set to:\nc := c - xa\nwhere xa is the time since event a.\n(See lower part of the drawing)\nHowever, I'm not convinced of this solution as it is inelegant, further I'm not sure whether timed automata allow for the setting of clocks. So far I only read about reset to zero.\n(1) http://pub.ist.ac.at/~tah/Publications/a_determinizable_class_of_timed_automata.pdf\n\n", "answer": "So, after looking deeper into the subject I found the solution. Posting in case somebody else needs an answer and is unfamiliar with the subject.\nWhat I was looking for is called a Stopwatch Automaton (SWA) and allows the time to stop or advance (by specifying the derivative of a clock c' = {0, 1}).\nReference: The Impressive Power of Stopwatches. Franck Cassez, Kim Larsen\nhttp://citeseerx.ist.psu.edu/viewdoc/download?doi=10.1.1.123.669&rep=rep1&type=pdf\nFurther I discovered that Hourglass automata also allow time to stop, but additionally also to reverse: c' = {-1, 0, 1}.\nThey however require the specification of a maximum clock value after which \"all the sand passed through the hourglass\" and the clock does not advance further.\nReference: Hourglass Automata.\nYuki Osada, Tim French, Mark Reynolds, Harry Smallbone\nhttps://arxiv.org/abs/1408.5965\n", "topic": "cs", "url": "https://cs.stackexchange.com/questions/88411"}, {"image": "VQAonline_00007931.png", "question": "Finding recursion for runtime of code", "context": "This is the first time we have to do recursive/closed form expressions WITH code in class and I really have no idea how to approach this. My course notes that the prof put up don't really help as he put up a basic example, but this one has lots of recursive calls in it.\nI know that the first part len(lst)$ \\leq 1$ will be linear time because of it will just return whatever in the conditional. How do I tackle the rest, and ultimately come up with the recursive $T(n)$? The question is below:\n\n", "answer": "Base cases: $T(0) = T(1) = a, T(2) = b$. How did we get these base cases? Well, from the code:\nif len(lst) <= 1:\n return\n\nThis tells us that for n < 2, T doesn't depend on the input list size; hence, the function takes some constant amount of time for these input sizes.\nif lst[0] > lst[-1]:\n ...\n\nif len(lst) >= 3:\n ...\n\nThis part tells us that for input sizes between 1 and 3, exclusive (i.e., for 2) we do some more work than in the n <= 1 case but not as much as in the n >= 3 case. THe amount of work done still doesn't depend on the input size, so it's constant, but it might be difference from that of the n <= 1 case.\nNow we consider the n >= 3 case.\nRecursive part: $T(n) = 3T(\\frac{2}{3}n) + c$. Notice that the three recursive calls each use a list two thirds the size of the input list. To see this, let's consider the code:\nif len(list) >= 3:\n split = len(lst) // 3\n do_something(lst[0..len(lst) - split - 1])\n do_something(lst[split..len(lst) - 1]\n do_something(lst[0..len(lst) - split - 1])\n\nFirst off, we see that split will be equal to one third the length of the input list, rounded down (we assume // means integer division). We then have three recursive function invocations. To get the input size for these recursive calls, we have to see how big the lists we're passing are. All three are slices of the original input list; we can determine the size of the lists being passed to recursive calls by computing the indices used to determine slices. Observations:\n\nThe first and third slices use the same indices.\n0 .. len(lst) - 1 is the entire list, size n.\nsplit is one third the list size, equal to n/3.\nThe second slice uses split .. len(lst) - 1; to get the size of this slice, we can use simple subtraction of the size of the smaller interval (0..split) from the bigger interval (0..len(lst) - 1): n - n/3 = 2/3 n.\nThe first and third slices work similarly: if we take a slice of size n and stop it n/3 indices earlier, we get n - n/3 = 2/3 n.\n\nNow that you have a recurrence relation, you can resolve it using any of a several of techniques. \n", "topic": "cs", "url": "https://cs.stackexchange.com/questions/31850"}, {"image": "VQAonline_00007844.png", "question": "Transform inverse result", "context": "let us assume that:\nf is a grayscale image of size NxN, and F is the Fourier transform of f.\n\nG is a 2Nx2N transform obtained by inserting 0 between every value in F:\n\nWhat is the inverse transform of G?\nsolution:\nso i have programmed it, and the picture will be 4 times bigger with the picture 4 times in it. g= [f f;f f]\ncan you please explain why this is the answer? \n", "answer": "Think of it the other way around. \nIf you duplicate an image then you get zeros in between the transform (\nThis can intuitively be explained: \nduplicating an image is like convolving with 2 deltas (comb) of wavelength 1/2 the new image size. so is equivalent under convolution theorem to multiplying with comb of freq size of image / 2 which gives a comb with delta every other pixel -> every other pixel is zero.\n", "topic": "cs", "url": "https://cs.stackexchange.com/questions/9449"}, {"image": "VQAonline_00008641.png", "question": "NP-completeness of a problem using a \"T-gadget\"", "context": "Working on a problem I came up with the following \"T-gadget\":\n\nIt has 3 connectors (A, B, C); each connector has two wires (A1,A2; B1,B2; C1,C2);\nit can be rotated (0, 90, 180, 270 degrees);\ntwo gadgets X and Y, can be linked together (one connector of X is joined with one connector of Y);\nthe wires used to link the T-gadgets cannot overlap.\nA signal is propagated on the wires; and it can use 1 or 2 connectors to traverse a T-gadget. Suppose that it enters through wire A1, then it can:\n\ngo back through wire A2, OR\nchoose among connector B or C - suppose that it chooses connector B; then it can exit the gadget through wire B1, re-enter through B2 and exit again through A2\n\nWhen a signal enters a gadget and exits from the same connector, the gadget is \"activated\".\nPossible traversals of a T-gadget:\n(OUT) - A1 - A2 (OUT)\n(OUT) - A1 - B1 - (OUT) - B2 - A2 - (OUT)\n(OUT) - A1 - C1 - (OUT) - C2 - A2 - (OUT)\n(OUT) - B1 - B2 (OUT)\n(OUT) - B1 - A1 - (OUT) - A2 - B2 - (OUT)\n(OUT) - B1 - C1 - (OUT) - C2 - B2 - (OUT)\n(OUT) - C1 - C2 (OUT)\n(OUT) - C1 - B1 - (OUT) - B2 - C2 - (OUT)\n(OUT) - C1 - A1 - (OUT) - A2 - C2 - (OUT)\n\nThe signal starts on wire A1 of a gadget (with connector A unlinked) and must return to A2.\nQuestion: is the problem: \"Given a set of T-gadgets linked together, does exist a path that activates all of them?\" $NP$-complete?\nIf we omit option 1 (go back through wire A2) I think that the problem is $NP$-complete (through reduction from Hamiltonian path problem on cubic planar graphs).\n", "answer": "You're asking whether the underlying 3-regular graph has a Hamiltonian path, which is NP-complete. Your activation signal must traverse every edge in this path exactly twice.\nSee also this question and this one.\n", "topic": "cstheory", "url": "https://cstheory.stackexchange.com/questions/10405"}, {"image": "VQAonline_00008693.png", "question": "Series-parallel DAG characterization", "context": "According to this Wikipage series-parallel graphs are characterized like this:\n\n2-connected series-parallel graphs are characterized by having no subgraph homeomorphic to $K_4$.\n\nI couldn't find a similar characterization for series-parallel directed acyclic graphs (DAGs), however I think that the prohibited subgraph in this case can be a little simpler ($K_4$ minus one edge plus specific orientation):\n\nIs it correct? Is it proven anywhere?\n", "answer": "The two-connected series-parallel DAGs are also called 2-terminal series-parallel graphs. The graph in the figure is also known as Wheatstone graph and is known as the example graph for demonstrating Braess' paradox in algorithmic game theory.\nIt was shown [1, Theorem 5.3] that a DAG is 2-terminal series-parallel if and only if it does not contain a subgraph that is homeomorphic to the Wheatstone graph.\n1 Pietro Cenciarelli, Daniele Gorla, Ivano Salvo: Inefficiencies in network models: A graph-theoretic perspective. Information Processing Letters 131 (2018) 44\u201350.\n", "topic": "cstheory", "url": "https://cstheory.stackexchange.com/questions/47645"}, {"image": "VQAonline_00008634.png", "question": "Dealing with duplicates in Genetic Algorithm", "context": "I'm using Genetic Algorithm to create a rota for home-care organisation. All the groundwork is complete, I'm getting the results, but results are not as good as expected. \nIf I calculate fitness for existing rota made by somebody, I get about \u00a36000 (I measure fitness in money - great point for management). \nWhen I run my GA, the best I manage to get is \u00a38K after about 30K generations. \nSo I know the better solution exists and I'm trying to get better result. \nI have tried different approaches - big population, small population, different number of crossover points, swap-mutation, random-mutation, hill-climber-mutation. \nAnd nothing improved the results. \nThe resulting curve looks like this: \n\nI can run GA much longer, but it does not give me any better results - the curve is just flat horizontal to the right.\nI figured that I get quite a lot of duplicated individuals in final population. And that slows down the improvement of the solution. \nI learned how to find duplicate individuals. But what do I do with them? I have tried replacing them with randomly created solutions, but usually their fitness is much worse then every body else in the solution space, so they get \"killed\" quite soon without giving any offsprings or other influence to the solution space. \nAny ideas/suggestions/papers will be highly appreciated, as I've run out of ideas and research papers and almost no time left. \nUPD: some description of the problem: \nThere are number of visits(shifts) that are located in different geographical places (distances between locations are known). Currently I work with 700 visits per week with durations from 30 minutes to 5hrs. There are workers with different levels of contractual arrangements: some have 0 contracted hours - relief workers, there are part-timers and full-timers.\nI need to allocated workers for every shift in a best possible manner with this factors in mind:\n\nMinimise travel distance for workers. When they have number of shifts in a day, the locations should be roughly in the same area\nWorkers should not do a lot of overtime hours\nWorkers should not work less than their contract says\nWorkers with 0-contracts should be allocated no or very limited working hours\nMinimise waiting time between shifts. So no situations when somebody starts one visit at 9am to 10am, and next visit is at 6pm.\nThere is a limit of total number of hours that can be worked per day\nThere is a minimum of working hours per day: nobody should be deployed for only 30-minute visit and no other visits per day. \nAt least 2 days off allocated for anybody, but can be more\nAt least 2 days off must be adjacent.\n\nAnd other minor fitness function adjustments.\nI hope this is sufficient for description.\n", "answer": "What you've hit upon is the basic mechanism that drives GAs -- convergence to some fixed point that you hope is near optimal. In your case, clearly it isn't. So the name of the game is to try and balance this tendency to drive convergence with the competing interest of exploring the space looking for other solutions. There's no silver bullet for this problem -- you always end up tuning things to suit your particular problem.\nIn general, there are two ways to do this. One is to delay convergence for longer, thus giving the algorithm more time to find better solutions. You can switch to a weaker selection operator (binary tournament selection for example), you can increase your mutation rate, etc. The other option is to allow the algorithm to converge, but figure out a sensible way to restart the search (as you mentioned, putting random points in tends not to work because their fitness is too low to allow them to survive and contribute to the search).\nI'll give an example of the latter approach. There is a relatively-little known algorithm called CHC (Eshelman 1991) that tends to work really well for a lot of problems. Unfortunately, one reason it's little-known is that the only way to access the original paper is to buy the printed proceedings of the first FOGA conference, but you can probably find a little bit of information online, hopefully enough to get you started. I can give you something of a quick overview though. In rough pseudocode, CHC looks like\ndelta = k/4 # k = chromosome length\nwhile not done\n create new child population\n for i = 1 to n/2 # n = population size\n select p1, p2 from population without replacement\n if hamming_distance(p1, p2) > delta\n c1, c2 = HUX crossover(p1, p2)\n insert c1, c2 into child pop\n end if\n end for\n if child pop is empty\n delta = delta - 1\n else\n take best n individuals from union of parent and child populations as next population\n end if\n if delta < 0\n keep one copy of best individual in population\n generate n-1 new population members by flipping 35% of the bits of the best individual\n delta = k/4\n end if\n\nThe HUX crossover operator is like uniform crossover, but instead of choosing randomly between the parents independently for each bit, it exchanges precisely half the differing bits from the parents. \nYou then just keep running this loop until you decide to quit, either because you've found a good enough solution, you've exhausted the amount of time you have to spend on the problem, or whatever other criteria you desire.\n", "topic": "cstheory", "url": "https://cstheory.stackexchange.com/questions/7849"}, {"image": "VQAonline_00008623.png", "question": "One-shot quantum hitting times", "context": "In the paper Quantum Random Walks Hit Exponentially Faster (arXiv:quant-ph/0205083) Kempe gives a notion of hitting time for quantum walks (in the hypercube) that is not very popular in the quantum walk literature. It is defined as follows:\n\nOne-Shot Quantum Hitting Time: A discrete-time quantum walk has a $(T,p)$ one-shot $(|\\Psi_0\\rangle,|\\Psi^f\\rangle)$-hitting time if $|\\langle\\Psi^f|U^T|\\Psi_0\\rangle|^2 \\geq p$ where $|\\Psi_0\\rangle$ is the initial state, $|\\Psi^f\\rangle$ is the target state, and $p>0$ is the hitting probability.\n\nNormally you would like to know the minimum $T$ such that $p>0$. It is not possible (correct me if I'm wrong) to define a notion of average hitting time because you will need to make measurements during the walk, and that would collapse it to a classical walk. That's why we have the one-shot notion. In the same piece of work, there is an application to quantum routing (cf. section 5).\nIn order to know that the walk arrived at the target vertex, you need to make a measurement only at that node. For example, in the $n$-dimensional hypercube with $2^n$ nodes if you start at node $|\\Psi_0\\rangle=|00\\dots00\\rangle$ and have as target node $|\\Psi^f\\rangle=|11\\dots11\\rangle$, the paper shows that $T=O(n)$ with bounded error probability, i.e. $p\\to 1$ as $n$ becomes very large. So in order to detect that the walk arrived at $|11\\dots11\\rangle$ you make a measurement after $\\Omega(n)$ steps. This is an exponential speed-up.\nQuestions:\n\nTo use this notion of hitting time for search you need to know at least the distance of the target vertex from the origin, because that's how you know when to apply your measurement. Let's say that you have a graph $G$, and set as initial vertex $v_0$ and want to reach $v^f$. Assume also that $T=O(dist(v_0,v^f))$ and $p\\geq 1/2$. Well, $T$ is obvious because you need at least that many steps to reach it. Does it make any sense using this hitting time for search? If you know where the node is there is no meaning in searching, but having a piece of information like \"distance from the starting vertex\" but not knowing exactly where the target is, does this notion of hitting time gives any interesting (worth to study) search algorithm?\nDoes the application to quantum routing makes any sense? In the paper it says that it can be used for routing packages, but it seems to me that you can only send 1 bit, e.g. does it arrived at destination or not? Can you actually send a quantum state in this framework? In the paper this issue is not being addressed.\nThis is maybe a silly question to ask, but here it goes. Can you use this notion of hitting time for constructing a \"Generalized Mach-Zender Interferometer\"?\n\nI'm aware of the other notions of hitting times for quantum walks (like Szegedy's or Ambainis's). I'm particularly interested in this specific hitting time.\nUpdate (9/24/2010): Thanks to Joe Fitzsimons questions 2 and 3 were completely answered. Although question number 1 still remains. First, I will restate question 2 in more specific terms now that I finished reading the paper that Joe recommended me and a couple more (for example see arXiv:0802.1224), and then I'll give a concrete example of what I have in mind for question 1.\n2'. If you are sending a concrete message (like a sequence of classical bits), you can use a more complicated unitary that will copy this information during the steps of the walk. To send quantum states you need something more. The spin-chains channel uses a linear array of qubits with a fixed coupling. You can put the state (pure state, I don't know if it works for mixed states) you want to transmit in one end and it goes to the other end with high fidelity according to numerical results. I still have to give it more thought but I have two ideas: i) put a chain on each link of the graph, or ii) make the walk, find the target state, then make the channel between initial state and target and then send the state. Are any of these approaches plaussible? Does it work with mixed states?\n1'. Consider a walk on a 2-dimensional grid centered in the origin with $n$ nodes with each side with length $\\sqrt{n}$. Set the initial state at $v_0=(0,0)$ and the target state at $v^f=(\\sqrt{n}-1,a)$ where $a=0,\\dots,\\sqrt{n}-1$. Because the walk is symmetric we have that the same hitting time and hitting probabilities hold for any target somewhere on the border of the grid as shown below.\n\nTherefore the information we have is that $dist(v_0,v^f)=\\Omega(\\sqrt{n})$. We can use this to know when to make the measurement. Can the one-shot hitting time be used to search this grid? Here you need that information. An open problem in searching a grid is that we know that $\\Omega(\\sqrt{n})$ is a lower bound for search, and for grids the best upper bound is $O(\\sqrt{n\\log n})$. Either we are not being able to find a better algorithm, or the techniques for proving lower bounds when you use them on grids are giving a weak lower bound. Can you show that the only way to go below $\\sqrt{n\\log n}$ is having \"a piece of information\" as the one in the question? This would imply a way of proving a lower bound for grids. Does it make any sense?\n", "answer": "I'm not so familiar with this paper, but I will try to give a rough answer to each of your questions after a cursory skim.\n\nGrover's algorithm can indeed viewed with this notion of hitting time. You need to decide when to measure the system, and even though T is constant for all results, it is still important to calculate. Here T is certainly not $O(\\mbox{dist}(v_0, v^f))$ (which in this case is 1), but rather $O(\\sqrt{n})$, so your assumption that $T=O(\\mbox{dist}(v_0, v^f))$ is not valid here.\nI assume the author is taking an entire packet to do the random walk. Obviously this requires a somewhat more complicated unitary, but I don't really see an issue. Alternately, Burgarth and Bose have a very nice scheme for encoding information across identical graphs which would work too if you simply replace their 1d chains with the network of choice (quant-ph/0406112).\nWell, you don't quite need this notion of hitting time. Hypercubes have perfect state transfer (see for example quant-ph/0309131 and quant-ph/0411020), so you can view transport on a hypercube as an interferometer with the Mach-Zender interferometer corresponding to the 2d case.\n\n\nUPDATE: (To answer the updated question regarding random walks on a grid or other lattice)\nOne approach to the measurement issue you highlight with the spatial search problem is to simply make a measurement at each timestep such that it returns 1 iff the the vertex the walker is currently at (say $v_t$) is equal to $v^f$ and the current timestep t is the hitting time for that vertex. This should avoid the issue of collapsing the wave function, as the measurement is only made for each vertex once the hitting time is reached, and it only registers collapses onto a location if that location is the correct result.\n", "topic": "cstheory", "url": "https://cstheory.stackexchange.com/questions/1400"}, {"image": "VQAonline_00008696.png", "question": "What's the logical counterpart to jumps with arguments on CPS terms?", "context": "It's well known that the CPS (continuation-passing style) translation often employed in compilers corresponds to double negation translation under the Curry-Howard isomorphism. Though often the target language of a CPS translation is the same as the source language, sometimes it's a specialized language which only allows terms in CPS form (i.e., there are no direct style functions anymore). See, e.g., this or this.\nAs an example, consider Thielecke's CPS-calculus, where commands are defined as either jumps or bindings:\n$$b ::= x\\langle\\vec{x}\\rangle\\ |\\ b\\ \\{\\ x\\langle\\vec{x}\\rangle = b\\ \\}$$\nAnd one-hole contexts (commands with holes) are defined as follow:\n$$C ::= [-]\\ |\\ C \\ \\{\\ x\\langle\\vec{x}\\rangle = b\\ \\}\\ |\\ b\\ \\{\\ x\\langle\\vec{x}\\rangle = C\\ \\}$$\nIf we try to see these languages under the Curry-Howard isomorphism, we don't have implication anymore, but rather we use negations and products alone. The typing rules for such languages demonstrate we're trying to derive a contradiction:\n$$\\frac{\\color{orange}{\\Gamma\\vdash} k{:}\\ \\color{orange}{\\neg\\vec{\\tau}}\\quad\\quad\\color{orange}{\\Gamma\\vdash}\\vec{x}{:}\\ \\color{orange}{\\vec{\\tau}}}{\\color{orange}{\\Gamma\\vdash} k\\langle\\vec{x}\\rangle}(J)$$\n$$\\frac{\\color{orange}{\\Gamma,}k{:}\\ \\color{orange}{\\neg\\vec{\\tau}\\vdash} b\\quad\\quad\\color{orange}{\\Gamma,}\\vec{x}{:}\\ \\color{orange}{\\vec{\\tau}\\vdash} c}{\\color{orange}{\\Gamma\\vdash} b\\ \\{\\ k\\langle\\vec{x}\\rangle=c\\ \\}}(B)$$\n(Note that these look similar to the (AXIOM) and (CUT) rules from linear logic, though on the other side of the sequent: we have a conjunction rather than a disjunction.)\nReduction rules in intermediate languages such as the ones above allow jumps to be performed to bound continuations, immediately replacing arguments (hence the name \"jump with arguments\" sometimes employed). For the CPS-calculus, this can be represented by the following reduction rule:\n$$\\frac{}{C[\\color{blue}{k\\langle \\vec{x}\\rangle}]\\ \\{\\ k\\langle\\color{red}{\\vec{y}}\\rangle=\\color{red}c\\ \\} \\longrightarrow C[\\color{red}{c[\\color{blue}{\\vec{x}}/\\vec{y}]}]\\ \\{\\ k\\langle\\color{red}{\\vec{y}}\\rangle=\\color{red}c\\ \\}}$$\n$$\\frac{a\\longrightarrow b}{C[a]\\longrightarrow C[b]}$$\n...though similar languages have similar notions of jump. I'm not totally sure, but I believe that the reduction rule would corresponde to a cut inference rule similar to the following (quickly sketched):\n\n...where we're allowed to copy a bound proof tree and replace the jump subtree with it (in the example above, replacing subtree a with a copy of subtree b, though with a different context).\nI'm interested on how such an intermediate language could be seen by the Curry-Howard isomorphism. So, my actual question is twofold:\n\nHas a similar implication-free subset of some logic (e.g., propositional logic) been studied somewhere? I mean, has a \"logic without implication\" been proposed?\nWhat is the equivalent of a jump with arguments in logic? Assuming the cut rule I sketched above is correct (and it corresponds to the reduction rule), has something similar to it appeared elsewhere?\n\n", "answer": "\nSuch a logic of continuations (or a syntax of continuation that arose from logical considerations) would be Laurent's \u201cpolarised linear logic\u201d (LLP): Olivier Laurent, \u00c9tude de la polarisation en logique (2002). A good explanation of what is going on from a categorical perspective is given in Melli\u00e8s and Tabareau, Resource modalities in tensor logic (2010). A detailed description of the correspondence between LLP and CPS along the lines of your question appears in my PhD thesis (2013) (Chapter III, pp.91-95,153-199). (There are a lot of other references in this area; the bibliographies should provide you with a good starting point.)\n\nThe two rules (J) and (B) you wrote are derived as follows (in the notations of Laurent's PhD thesis):\n\\begin{array}{c}\n\\dfrac{\\dfrac{\\vdash\\mathcal{N},!(P_{1}^{\\bot}\\mathbin{\u214b}\\cdots\\mathbin{\u214b}P_{n}^{\\bot})\\qquad\\dfrac{\\dfrac{\\vdash\\mathcal{N},P_{1}\\quad\\cdots\\quad\\vdash\\mathcal{N},P_{n}}{\\vdash\\mathcal{N},\\dots,\\mathcal{N},P_{1}\\otimes\\cdots\\otimes P_{n}}}{\\vdash\\mathcal{N},\\dots,\\mathcal{N},?(P_{1}\\otimes\\cdots\\otimes P_{n})}}{\\vdash\\mathcal{N},\\mathcal{N},\\dots,\\mathcal{N}}}{\\vdash\\mathcal{N}}\\\\\n\\dfrac{\\dfrac{\\vdash\\mathcal{N},?(P_{1}\\otimes\\dots\\otimes P_{n})\\qquad\\dfrac{\\dfrac{\\vdash\\mathcal{N},P_{1}^{\\bot},\\dots,P_{n}^{\\bot}}{\\vdash\\mathcal{N},P_{1}^{\\bot}\\mathbin{\u214b}\\cdots\\mathbin{\u214b}P_{n}^{\\bot}}}{\\vdash\\mathcal{N},!(P_{1}^{\\bot}\\mathbin{\u214b}\\cdots\\mathbin{\u214b}P_{n}^{\\bot})}}{\\vdash\\mathcal{N},\\mathcal{N}}}{\\vdash\\mathcal{N}}\n\\end{array}\nLLP is a sequent calculus and as such is finer-grained: it makes explicit structural rules and (what corresponds to) left-introduction of negation.\nVisible above, it also internalizes duality (all formulae on the right-hand side, like in linear logic). While Laurent does not cite Thielecke's works, Thielecke's PhD thesis previously suggested that having a \u201cduality functor\u201d could be useful to clarify the duality aspects of CPS.\nLaurent's graphical syntax will let you interpret linear substitution, but one would have to look at the details to see if it is exactly the same as the one you mention.\n\n\n", "topic": "cstheory", "url": "https://cstheory.stackexchange.com/questions/50964"}, {"image": "VQAonline_00008650.png", "question": "Is the D-Wave architecture a close implementation of quantum interactive proof?", "context": "A very high level architecture is, as mentioned here, shown in this picture.\n\nThe component on the left is classical while the one on the right is the D-Wave box. I understand that in QIP, Arthur is BQP. Here the classical component is always in BQP if we are not too restrictive. On the other hand, the D-Wave box although don't have unbounded computational resources is not violating any law of quantum mechanics. Let's consider this as not-so-strong Merlin.\nCan we say that this architecture is an restricted implementation of quantum interactive proof system? My motivation for this question is that if we want to do the complexity analysis of the D-Wave architecture should we model it as QIP?\n", "answer": "This question seems a little confused. The class of decision problems solvable efficiently on a quantum computer is BQP, while on a classical computer it is either P or BPP depending on exactly how you define things. An interactive proof is something entirely different. It is a protocol which allows a prover to prove, beyond reasonable doubt, the outcome of some decision problem, and allows the prover and verifier to interact. QIP is the class of interactive proofs where the verifier, not the prover, has access to a quantum computer. It turns out that this doesn't help very much since you can prove exactly the same things to a fully classical verifier. I think what you have in mind is something different: an interactive proof where the prover is limited to quantum computation. This is something that has been studied in recent years (see for example arXiv:0807.4154, arXiv:0810.5375 and arXiv:1209.0449). The extent of our current knowledge is that you can basically prove anything in BQP if you have either some limited quantum power for the verifier (the ability to prepare single qubit states, for example) or if you have several provers who do not communicate, but share entanglement.\nThe DWave machine does not implement anything like these schemes, and does not constitute any form of interactive proof. Actually, the problem of implementing such a scheme on DWave hardware has been something that has interested me for a while. There are a few hurdles to doing this. All of the known schemes are based in the circuit model or the closely related measurement based model, while the DWave machines use a very different model of computation. This introduces a few problems, as their system is not a general purpose quantum computer, and so cannot implement any of the known schemes directly. Further, it cannot really accept any kind of quantum input, which rules out many of the tricks used to construct the known schemes.\nHowever, I don't mean this to be a criticism of DWaves device: virtually no other quantum device has been used to implement such a proof. The closest you get are things like violations of Bell's inequalities and the recent experimental demonstration of blind quantum computing in linear optic (arXiv:1110.1381). At this point, no one has yet published an experimental demonstration of a quantum prover interactive proof, so DWave isn't any better or worse in this respect than anyone else. \nBy the way, I apologise for linking to two of my own papers in the above, but this is a question I have been very interested in in recent years, and I think those papers are relevant to the question at hand. It's not intended as self-publicity.\n", "topic": "cstheory", "url": "https://cstheory.stackexchange.com/questions/17679"}, {"image": "VQAonline_00008625.png", "question": "Degree sets for linear extension graphs", "context": "A linear extension $L$ of a poset $\\mathcal{P}$ is a linear order on the\nelements of $\\mathcal{P}$, such that $x \\leq y$ in $\\mathcal{P}$ implies $x \\leq y$ in $L$ for all $x,y\\in\\mathcal{P}$.\nA linear extension graph is a graph on the set of linear extensions of a poset, where two linear extensions are adjacent exactly if they di\ufb00er in one adjacent swap of elements.\nOn the following picture there is the poset known as $N$-poset, and its linear extension graph, where $a=1234, b=2134, c=1243, d=2143, e=2413$. \n(This figure is taken from the work.)\nWhen you study linear extension graphs (LEG) you can come up with an idea (conjecture) that if $\\Delta$ - maximal degree of a LEG, $\\delta$ - respecrively, minimal degree, then the degree set of any LEG consists of $\\Delta,\\delta$ and each natural number between them. For example, let's take a poset, known as chevron, then in its LEG $\\mathcal{G}$ with $\\Delta(\\mathcal{G})=5$ and $\\delta(\\mathcal{G})=2$, and also, according to our conjecture, vertices with the degrees 4 and 3 are contained in the graph. So, the question is can we prove or disprove this conjecture?\nAbout LEGs and how do they look like one can read in the dissertation of Mareike Massow here. Chevron and its LEG can be seen on the page 23 of the dissertation.\nOn the degree sets there is the classical paper \"Degree sets for graphs\" by Kapoor S.F. et al.\n", "answer": "I think I proved it yesterday. Thus here goes the sketch of the proof.\nAt first, the following lemma is proved.\nLemma. Let $\\mathcal{P}$ - a partial order, $G(\\mathcal{P})$ - its linear extension graph and $v_1,v_2$ - two adjacent vertices of $G(\\mathcal{P})$. Then $|deg(v_1)-deg(v_2)|\\leq 2$.\nThe sketch of the proof.\nAt the same time, $v_1,v_2$ are linear extensions of $\\mathcal{P}$ such that one of them, say $v_1$, can be transformed into $v_2$ by one transposition of adjacent elements (adjacent transposition). It is easy to see (consider, for instance, $d$ and $e$ from the above figure) that any element $x_i$ of any linear extension $L=x_1x_2\\dots x_n$ can change the number of incomparable adjacent elements on at most two:\n\nIf $x_i$ can be transposed at all then at least one its neighbor, say $x_{i+1}$, is incomparable to it ($x_i\\parallel x_{i+1}$, if comparable then $x_i\\perp x_{i+1}$). Note: before transposing we have $L_1=\\dots x_{i-1}x_ix_{i+1}x_{i+2}\\dots$ and immediately after - $L_2=\\dots x_{i-1}x_{i+1}x_{i}x_{i+2}\\dots$.\nLet us consider how the number of incomparabilities (degree of the linear extension as the vertex in $G(\\mathcal{P})$) in $L$ could change. We consider at first the pair $x_ix_{i+2}$. For $x_{i-1}x_{i+1}$ the same conclusion follows by symmetry.\n\nIf $x_{i+1}\\parallel(\\perp) x_{i+2}\\land x_{i}\\parallel(\\perp) x_{i+2}$, then $deg(L)$ doesn't change.\nIf $x_{i+1}\\perp(\\parallel) x_{i+2}\\land x_{i}\\parallel(\\perp) x_{i+2}$, then $deg(L)$ increases (decreases) by one.The sketch of the proof is completed.\nTheorem. Let $G(\\mathcal{P})$ - a linear extension graph. If $G(\\mathcal{P})$ contains vertices $v_1,v_2$ with $deg(v_1)=k,deg(v_2)=k+2$, then there is $v_3\\in G(\\mathcal{P})$ such that $deg(v_3)=k+1$.\nThe sketch of the proof.\nSuppose $v_1,v_2,deg(v_1)=k,deg(v_2)=k+2$ are adjacent in $G(\\mathcal{P})$, otherwise any vertex with degree $k$ in $G(\\mathcal{P})$ is adjacent with some vertex if such exists with degree $k+1$.\nLet us consider the case where we have $L_1,L_2$ from the previous lemma such that\n$$x_{i+1}\\perp x_{i+2}\\land x_{i}\\parallel x_{i+2},$$\nand\n$$x_{i-1}\\perp x_{i}\\land x_{i-1}\\parallel x_{i+1},$$\nThus $deg(L_2)=deg(L_1)+2$.\nLet us now start transpose $x_{i+1}$ in the direction of $x_1$. It is easy to see that eventually we could stop at the position where\n$$x_{j}\\perp x_{i+1}\\land x_{i+1}\\parallel x_{j+1},$$ for some $j 0$$\nI'm asked to derive the isoquant curve which yields the following diagram:\n\nCould anyone please explain to me how the form of the graph was found?\nI have 5 unknowns in this equation and I just can figure out how would it be possible to plot the graphs.\nThank you very much in advance for your help!\n", "answer": "Each curve shows the rate at which K can be substituted for L, or vice versa, while keeping output constant. The Marginal Rate of Technical Substitution (MRTS) equals the absolute value of the slope. The MRTS tells us how much one input a firm can sacrifice while still mainting a certain output level. Then the MRTS is equal to the following ratio: $$\\frac{\\text{Marginal Productivity of Capital } (MP_{K})}{\\text{Marginal Productivity of Labour } (MP_{L})}$$\nThen the substitution of Labour (L) for Capital (K) is given by: \n$$ MRTS_{K,L} = -dL/dK = MP_{K}/MP_{L}$$\nNow, note that in your equation $\\alpha$, $\\beta$ and $A$ are exogenous parameters (i.e. given to you). What your equation will then defined is, given these parameters, how your inputs (K and L) result in some amount of output given your production function. \nYou can find the isoquant curve that yields that diagram by varying the fixed level of output and by \"playing with the fixed parameters\". \n", "topic": "economics", "url": "https://economics.stackexchange.com/questions/27687"}, {"image": "VQAonline_00012349.png", "question": "Income Elasticity of Demand vs. shift in demand curve", "context": "I've just read an econ textbook and want to verify a statement/definition about income elasticity of demand.\nChanges in income shift the demand curve, and we can measure the responsiveness of demand to income changes by calculating income elasticity of demand as follows:\n\nQuestion: Where does this change in quantity demanded come from? Is it from a movement along demand curve, or from a shift of demand curve?\n", "answer": "Short answer:\nDenoting income by $I$, price by $p$, the demand function by $D(I,p)$ and income elasticity of demand by $\\eta$, the definition of point elasticity of income is\n$$\n\\eta = \\frac{\\text{d}D(I,p)}{\\text{d}I}\\frac{I}{D(I,p)}.\n$$\nSo the change in demand comes from a change in income.\nLong answer:\nWithout specifying your coordinate system the terms \"movement along the curve\" and \"shift of the curve\" are meaningless.\nConsider the function $f(a,x) = a/x$. Suppose this function determines the value of a variable $y$, that is $y = f(a,x)$. Given $a$ we can plot $f(a,x)$ in the $(x,y)$ coordinate system. If $x$ changes there is movement along the curve in this coordinate system. If $a$ changes the curve shifts in this coordinate system.\nHowever treating $a$ as a variable and given $x$ we could also plot $f(a,x)$ in the $(a,y)$ coordinate system. Then change in $a$ would mean movement along the curve and change in $x$ would shift the curve.\nAs quantity demanded usually depends on both income and price, you face a similar situation, where $q = D(I,p)$. Income elasticity examines a change in income, but without specifying if your curve is ploted in $(p,q)$ or $(I,q)$ you cannot classify the change in $I$ as either \"movement along the curve\" or \"shift of the curve\".\n", "topic": "economics", "url": "https://economics.stackexchange.com/questions/20273"}, {"image": "VQAonline_00012286.png", "question": "How to determine regressive tax", "context": "I have a mock-up exam that my teacher gave me and one of the questions was the following:\n\nGiven these four graphs, which one represents regressive tax?\n\nI'm quite sure it's graph A, but I don't seem to be able to find a proper reason why. Any help and/or tips are welcome!\nThanks in advance\nEDIT: As per request. How I interpret this: regressive means the higher the income, the lower the tax. Thus: tax drops with higher incomes. But I can't really determine to which graph this applies.\n\n(T is tax, Y is income)\n", "answer": "You're correct it is graph A. Regressive tax means that the lower income you have the higher %tax you have. If you imagine at the lowest level of income, you still have a flat tax(The intercept).\nSince the slope is constant the %tax component is the same for all income levels.\nTherefore this flat tax component is the part you need to look at. The flat tax represents a larger proportion for lower income earners. As a whole (%tax + Flat tax) the tax is then a higher % for lower income earners, hence regressive taxation.\n", "topic": "economics", "url": "https://economics.stackexchange.com/questions/5902"}, {"image": "VQAonline_00013012.png", "question": "The remaining parts of an ancient city", "context": "What do you call the remaining parts of an ancient city like what you see within the picture that sometimes archeologists discover and unearth? \n\n1- Remnants of a city \n 2- Remainings of a city \n\nTo me they both work, but I am sure there is a fixed term to it. Also, I wonder if there is a better choice. \n\n", "answer": "Cities do not have remnants. Remaining parts of something does exist in general but is not an archaeological term per se.\nRemnants is a word applied to objects. Not a site. \nThe proper term is remains:\n\"Sites may range from those with few or no remains visible above ground, to buildings and other structures still in use.\" [Wikipedia]\nThe remains of a city or archaeological remains of a city.\nremains\n", "topic": "ell", "url": "https://ell.stackexchange.com/questions/215303"}, {"image": "VQAonline_00012857.png", "question": "To pinch or to grab someone's face?", "context": "\n\"She pinched my face. That hurt.\"\n\"She grabbed my face. That hurt\"\n\nWhat word is more suitable for the action illustrated in the image below?\n\n", "answer": "I'd call that a pinch. You can be more specific and say \"she pinched his cheek\", which means exactly what's shown in the picture.\n", "topic": "ell", "url": "https://ell.stackexchange.com/questions/146191"}, {"image": "VQAonline_00013016.png", "question": "What's the idiomatic way of saying \"hide your lips\"?", "context": "\nWhat's the idiomatic way of saying \"hide your lips\"? I thought \"bite your lips\", but you're not biting your lips, your putting your lips inside your mouth. So what's the proper way to say it? I added a pic of someone doing it.\n", "answer": "Lambie's answer describes the action of curling one's lips inwards. But if the OP is looking for a shorter expression, then I would say that the young mother in the photo appears tight-lipped.\n\n1 : having the lips closed tight (as in determination)\n 2 : reluctant to speak : TACITURN\nMerriam-Webster \n\nCambridge Dictionaries has\n\nSomeone who is tight-lipped is pressing their lips together to avoid showing anger, or is refusing to speak about something: \n\n", "topic": "ell", "url": "https://ell.stackexchange.com/questions/216508"}, {"image": "VQAonline_00012945.png", "question": "All vs. For all", "context": "According to the fourth edition of English Grammar in Use by R.Murphy, we can't use for + all:\n\nHowever, I can often hear people use for + all. They might say, for example:\n\nI wanted to remember it for all my life.\nI've been waiting for this moment for all my life.\n\nBut as for \"all day\", I think it's really better to day \"I've been working all day\" or \"I've been working for the whole day\".\nSo, do people break the grammar rule saying something like \"For all my life, you'll be the one\"?\n", "answer": "\n\"However, I can often hear people [...] say, for example:\n I wanted to remember it for all my life.\n I've been waiting for this moment for all my life.\"\n So, do people break the grammar rule saying something like \"For all my life, you'll be the one\"?\n\nLet's cut to the chase. Yes, in each case, these examples, represent people breaking rules of grammar. However, I would qualify this by saying that in spoken English, either form is acceptable.\nThis subject has been canvassed in this place at least once previously Is there a grammatical rule impeding the presence of 'for' in \u201cI have lived here 'for' all my life\u201d?. On that occasion there were views for and against. Some relying on standard academic publications to reinforce their views. The most popular answer was \"I think you just have to accept that when all comes before words that express any length of time, it cannot be preceded by for\". \nJ.R (reputation >94K in this community) also entered the fray and his comments are most relevant to this question. He said \"Unless the for is not starting a prepositional phrase (as in, \"That is what I've been hoping for all my life,\" or, \"This is what I've been waiting for all summer\").\" \nThe topic has also been canvased, to a lesser extent, on \"English language and usage\" Difference between \u201cmy whole life\u201d and \u201call my life\u201d?. Interestingly, the responses there were somewhat less didactic\nThe questioner asks specifically about the sentence \"For all my life, you'll be the one\". I don't think there is any doubt that this breaks the formal rule of not using \"for\" in a prepositional phrase. But how easy is it to imagine two lovers saying these words to each other? At that time and place, do the laws of grammar apply? Which is why I believe that in spoken English the rules are not so hard and fast as they might be for written English.\nIn any event, though I am a stickler for grammar (or at least the bits that I know of), I think grammar rules are made to be broken. How else does language relate and respond to daily life, new experiences and events, if not through constant change? The rules of grammar are written at a given point in time and space in the world. And even in that time and space, one might find that within a radius of several kilometres, or even several hundred kilometres, there are marked cultural and language differences which reinforce that often such \"rules\" are written by and for the benefit of the reigning middle and upper classes, and have little relevance to the working man's use of language.\n", "topic": "ell", "url": "https://ell.stackexchange.com/questions/183364"}, {"image": "VQAonline_00012849.png", "question": "Vegan fish fillet or vegetarian fish?", "context": "We normally say \"vegan fish fillet\" shown in the image below. Here is the product\nList of ingredients:\n\nWater, Soybean Fiber, (soybean protein, wheat starch, wheat protein, soybean oil), Soybean Protein, Soybean Oil, Tapioca Starch, Spice,Raw Cane Sugar, Sea Salt, White Pepper Powder, Laver, Yeast Peptone Powder, Natural Vegan Flavor.\n\nCan we instead say \"vegetarian fish\" to describe this food?\n\n", "answer": "Yes, it sounds fine to me. The word vegetarian plays the role of an adjective and it combines with fish to create a compound noun. The noun implies the fish is for vegans or made by/belongs to a vegan.\n", "topic": "ell", "url": "https://ell.stackexchange.com/questions/143314"}, {"image": "VQAonline_00012592.png", "question": "what is the opposite of the stretch?", "context": "As I have checked some online dictionaries, I have found 'shorten' and 'shrink' as opposites of the 'stretch' word. But, I am not sure, yet.\nAs an example, suppose that shrink is the correct word, then I say in this figure\nI am shrinking the plot from top to bottom or stretching vice-versa.\n", "answer": "Shrink can work. I would also consider \"compress\" as a good option. \"Reduce\" can also work\n\nI reduced the plot vertically\n\nYou could ask this on English Language and Usage to get additional ideas.\n", "topic": "ell", "url": "https://ell.stackexchange.com/questions/46791"}, {"image": "VQAonline_00012539.png", "question": "What does \"something are down\" mean?", "context": "I saw this picture on the Net...\n\nWhat does \"The computers are down\" mean? Thanks.\n", "answer": "Literally, \"computers are down\" means that those computers are off-line, usually because of some kinds of problems (such as power failure, software failure, operating system crash, etc.).\nI don't know if there are any deeper meaning. But it seems to me that the comic hints that people nowadays rely too much on computers. Even a guru still needs computers to provide a good answer. (In this case, \"the meaning of life\".)\n\nPS. This reminds me of a novel by Douglas Adams, Hitchhiker's Guide to the Galaxy. In order to get \"the answer to life, the universe, and everything\", a supreme being had to create a computer to provide such an answer. After a long time past, the computer gave the answer, 42. (The computer that was created in the story was actually our Earth, including us human beings.)\n", "topic": "ell", "url": "https://ell.stackexchange.com/questions/13927"}, {"image": "VQAonline_00013261.png", "question": "What does \"Do you haz teh codez?\" mean?", "context": "There is a banner in StackOverflow pointing to the careers website (programmers):\n\nDo you haz teh codez?\n\nMy questions about the sentence/question are: \n\nWhat's the question in plain English?\nWhy do they write it like that?\nIs it like a word game?\nHow should I read it in terms of pronunciation?\n\n\n", "answer": "Literal translation is \"Do you have the code?\" but what is implied is \"Please can you write my application for me?\" or \"Please solve my problem so I don't have to put any effort in.\"\nIt is written like that as a mockery of people who ask questions on Stack Overflow, and other such websites, where their question is a thinly veiled, or completely unveiled, attempt to get someone to do their work for them. So the questioner won't have even have tried to solve their problem and will possibly be asking for an entire software application to be written.\nI'm not sure what you mean by \"Is there like a word game?\" If you mean is this like a word game then not really, it's just a joke.\nIt should be pronounced \"Do you haz (as in Mazda) teh (as in meh) codes?\" and in IPA: /du\u02d0ju\u02d0h\u00e6zt\u025bk\u0259\u028adz/.\nFor an example of the type of thing this is mocking: http://thedailywtf.com/Articles/plz-email-me-teh-codez.aspx\n", "topic": "english", "url": "https://english.stackexchange.com/questions/13231"}, {"image": "VQAonline_00013900.png", "question": "\"Neither of us are\" -vs- \"Neither one of us is\"", "context": " This cartoon was recently posted on Facebook. My sister (who is a retired HS English teacher) says the grammar is wrong and that the correct wording should be: \"Neither one of us is.\" but I disagree. \nI am not an English teacher but I believe that \"US\" relates to \"WE\" and the correct wording for that pronoun is \"WE ARE\". On the other hand \"ONE OF US\" relates to \"I\" and \"YOU\" but the correct wording for those pronouns would be \"I AM\" or \"YOU ARE\" ... There absolutely is no place in English where we correctly use \"I IS\", \"YOU IS\", or \"WE IS\" so why would \"US IS\" be correct?\n", "answer": "Your sister appears to be correct. 'Neither of us' is understood as 'neither one of us' and is treated as singular. From the Commnet \"Guide to Grammar and Writing\":\n\nThe pronouns neither and either are singular and require singular verbs even though they seem to be referring, in a sense, to two things.\nNeither of the two traffic lights is working.\nWhich shirt do you want for Christmas?\nEither is fine with me.\n\nAccording to Ngram, it is also the most conventional usage.\n", "topic": "english", "url": "https://english.stackexchange.com/questions/470158"}, {"image": "VQAonline_00014010.png", "question": "What do you call the strips on shoes that are often used instead of laces?", "context": "Some shoes uses laces and so you lace them. But some shoes use these long strips (see picture below for an example).\n\nHow do you call these strips and what is the proper alternative for the verb to lace in this case?\n", "answer": "It is called Velcro strips:\n\nthe brand name of a type of fastening tape consisting of opposing pieces of fabric, one piece with a dense arrangement of tiny nylon hooks and the other with a dense nylon pile, that interlock when pressed together, used as a closure on garments, luggage, etc., in place of buttons, zippers, and the like.\n\n(Dictionary.com)\nUsage examples from Lexico.com\n\nI can't manage things like zips, so they took the zips out and put Velcro in instead.\nYou may find yourself going out and buying shoes that have Velcro fastenings instead of laces as the child cannot grasp how to tie their own laces.\n\n", "topic": "english", "url": "https://english.stackexchange.com/questions/581514"}, {"image": "VQAonline_00013985.png", "question": "What is ridin in \"We're ridin' with Biden\"?", "context": "What is ridin in \"We're ridin' with Biden\"?\n\n\n\n\n\nSource: [Dantri.com]\n", "answer": "Ridin\u2019 means riding. It is meant figuratively: It does not mean literally riding in the car, just \u201cwe are along with him- we support his candidacy.\u201d\nThe term for dropping a sound (in this case g) is elision.\n\u201cRidin\u201d has the benefits of rhyming (close enough) with Biden and also having a folksy sound, which is part of the soon to be president\u2019s persona.\nEDIT: Thanks to comments by shoover: Ridin(g) also alludes to Biden riding the train from Delaware to DC and the sign is from a transit workers\u2019 unit.\n", "topic": "english", "url": "https://english.stackexchange.com/questions/558038"}, {"image": "VQAonline_00013846.png", "question": "Toasters don't toast toast, toast toasts toast, or does toast toast toast?", "context": "I saw this funny meme from someeecards:\n\nIt has me a little confused: To me it sounds like toast toasts toast, not toast toast toast. Is this meme wrong or am I missing something Either they (toast) toast toast. Or, it (the idea of toast) toasts toast?\n", "answer": "Your interpretation is correct, but so-called Internet memes are not overly concerned about grammaticality or idiomacity or logic. \nThe humor fails on multiple levels. As Edwin and Dan point out in the comments, there are various practical objections, for example, that toasters toast bread. But it is an attempt to be humorous, after all, not really a political argument, so the bad analogy is beside the point.\nThe attempt lies primarily within the thrice-repeated toast. This forces the reader to pause to work out how each toast functions. Toast is not the grammatical verb form, however, which spoils the attempt. Guns and people are plural whereas the middle toast, being used as an uncountable noun here, is treated as singular. The grammatical form doesn't succeed as there is no confusion as to which words serve which roles in toast toasts toast as compared, for example, to Buffalo buffalo Buffalo buffalo buffalo buffalo Buffalo buffalo.\nYou need a noun which shares a form with a related verb and which can also serve as its object or other complement. Consider \n\nFishermen don't fish fish, fish fish fish.\n\n", "topic": "english", "url": "https://english.stackexchange.com/questions/428900"}, {"image": "VQAonline_00013430.png", "question": "What is a \"hood\" in this context?", "context": "A forum poster was giving an account of his visit to his friend who is an owner of a burger shack, and in one part he said:\n\nWe smoked some of the Mr. Nice I brought along to give him and his employee for the trouble and blew it up into the hood in classic poverty restaurant style :coolface:.\n\nWhat is a hood? Is that the ceiling of the restaurant? I was thinking this couldn't be the folding hood of the car.\nEdit: \nThis is the picture of the kitchen he posted (couldn't scale it down unfortunately I'm on iPad :(), not sure if that device on the top left corner is the extractor or not, but could possibly be since I think there's what it looks like a fryer on the far right below it.\n\n", "answer": "In this context, sense 9 of hood applies: \u201cA metal covering that leads to a vent to suck away smoke or fumes\u201d.\nThe \u201cclassic poverty restaurant\u201d / burger shack will have a big heated stovetop, or grill, with a noisy fan to move smoke from grease burning on the grill \u2013 or in this case from the joints they smoked \u2013 out of the restaurant.\n", "topic": "english", "url": "https://english.stackexchange.com/questions/122264"}, {"image": "VQAonline_00013633.png", "question": "A Word For A Typographical Illusion or Typo Blindness?", "context": "Sometimes typographical errors are exceedingly difficult to catch - often because of typographical illusions.\nThis image is a good example of what I mean:\n\nYou may have missed the double \"the\" as many do!\nI am curious if a word exists for this strange process of the brain \"glossing over\" the typo. Humorously, these typos are sometimes difficult to spot even when another person points them out.\nI've heard this described as \"Typographical blindness\" but am looking for something more succinct.\n", "answer": "The specific illusion would probably fall under Future Perception, where the second \"the\" is consumed as we have already guessed what will come next.\nhttps://en.wikipedia.org/wiki/Optical_illusion#Future_perception\nA term I have seen for this specific occurrence concerning written text, is Typoglycemia. -- It does have an urban dictionary entry, so take it for what you will.\nhttps://en.wikipedia.org/wiki/Typoglycemia\n", "topic": "english", "url": "https://english.stackexchange.com/questions/289485"}, {"image": "VQAonline_00014033.png", "question": "How do you say \"empanada\" in Esperanto?", "context": "How do you say \"empanada\" in Esperanto? I've tried to find a term for it but failed.\nFrom Wikipedia:\n\nAn empanada is a type of baked or fried turnover consisting of pastry\n and filling, common in Latin American and Filipino cultures.\n\n\nImage source link\n", "answer": "I didn't find the exact word for empanada but I think (fritita) pastopo\u015do is close enough. According to PIV if the empanada is small and the filling is sweet it is named dariolo and if the filling is meat, fish, etc. risolo. \nFrom PIV:\n\ndariol/o Fritita kuka\u0135o, speco de pastopo\u015do kun, interne, dol\u0109a\u0135o.\nrisol/o Pastopo\u015deto a\u016d bulo, entenanta muelitan viandon, fi\u015da\u0135on a\u016d \u0109asa\u0135on, k fritita en graso a\u016d oleo.\n\n", "topic": "esperanto", "url": "https://esperanto.stackexchange.com/questions/5451"}, {"image": "VQAonline_00014026.png", "question": "\u0108u estas bona traduko por la angla vorto \u00abgate\u00bb?", "context": "Kiel oni povas traduki la anglan vorton \u00abgate\u00bb, ekzemple por priskribi \u0109i tiun objekton (la pordon en la barilo):\n\nLa vortaroj de Lernu kaj John Wells havas pordego, sed al mi tio \u015dajnas ne ta\u016dga \u0109ar tiu pordo fakte estas pli malgranda ol normala pordo. Eble tio estas \u0109efe por io kiel la Brandenburga pordego.\n", "answer": "Pordego is a large door for closing off a gateway - this is often appropriate for large gates in exterior walls.\nBarila pordo is a small door through a fence.\nPordejo or is a gateway, and seems appropriate for the picture, since it doesn't look like it can be closed.\n", "topic": "esperanto", "url": "https://esperanto.stackexchange.com/questions/2678"}, {"image": "VQAonline_00014027.png", "question": "Kiu nivelo de domo estas la \"unua eta\u011do\"?", "context": "La maniero per kiu oni kalkulas eta\u011dojn estas malsama en la germana kaj la sveda. En la sveda, la normo estas ke la \"unua eta\u011do\" staras sur la grundo. Tie kie oni eniras la domon estas la unua eta\u011do. Mi lernis ke en la germana la \"unua eta\u011do\" estas super la surgrunda eta\u011do. Kiel oni nombras eta\u011dojn en Esperanto? Kiu estas la unua eta\u011do? \u0108u a a\u016d b?\n\n", "answer": "Vortaro.net anka\u016d mencias \u0109i tiun problemon:\n\nLa maniero kalkuli la eta\u011dojn ne estas la sama por \u0109iuj popoloj: iuj inkluzivigas la tereta\u011don (do nomatan unua), aliaj ekskluzivigas \u011din.\n\n", "topic": "esperanto", "url": "https://esperanto.stackexchange.com/questions/3219"}, {"image": "VQAonline_00014032.png", "question": "Nomo por skota bovo-specio: \"Highland cow\"", "context": "Kio estas la kutima nomo de la jena skota bovo-specio en Esperanto? Mi ne sukcesis trovi ion ajn interrete. Mi kontrolis en Vikipedio, PIV, ReVo kaj la vortaro de John Wells. En la angla oni nomas \u0109i tiun beston \"Highland cow\", en la nederlanda oni diras \"Schotse hooglander\". Mi improvize uzis la esprimon \"skota altlandulo\", sed eble ekzistas pli tradicia esprimo.\nWhat is the usual name of the following Scottish cow species in Esperanto? I couldn't find anything online. I checked Wikipedia, PIV, ReVo and John Wells' dictionary. In English this animal is called \"Highland cow\", in Dutch it's called \"Schotse hooglander\". I improvisationally used the term \"skota altlandulo\", but maybe there's a more traditional expression.\n\n", "answer": "La\u016d mia scio, Skota Altlandulo estas la plej senca kaj tute ta\u016dga nomo por tiu besto, kun aliaj alternativoj ekzemple \"Skota Altlanda Bovo\". \u011cenerale plej gravas ke la uzita esprimo komuniku ta\u016dge la koncepton al la parol-partnero.\n", "topic": "esperanto", "url": "https://esperanto.stackexchange.com/questions/5395"}, {"image": "VQAonline_00014030.png", "question": "Why does 'feli\u0109e' mean \"unfortunately\" whilst 'feli\u0109a' means \"happy\"?", "context": "The dictionary is Esperanto-English Dictionary 1.\n\n", "answer": "That dictionary contains an error. Search the word \"malfeli\u0109e\" in that dictionary, it's translated by the same word \"unfortunately\".\n", "topic": "esperanto", "url": "https://esperanto.stackexchange.com/questions/5080"}, {"image": "VQAonline_00014024.png", "question": "How do you say \"a meme\" in Esperanto?", "context": "Some words combined with an image can go viral and really stay in your head. This is a pretty universal concept. So how does one say it in the international language?\n \nHow do you say \"a meme\" in Esperanto?\n", "answer": "According to the Esperanto version of Wiktionary, it's memeo:\n\nmeme - memeo\n\n", "topic": "esperanto", "url": "https://esperanto.stackexchange.com/questions/1701"}, {"image": "VQAonline_00014025.png", "question": "If I can't find a word in Vortaro.net, should I cease using that word?", "context": "For example, I've seen the word dumtempe before used to mean for a while or for some time. \nI used the word today, then started second-guessing myself and looked it up at Vortaro.net, but it showed no results. Isn't Vortaro.net basically an online version of PIV?\nWas I wrong to use the word? Have I been led astray concerning the word?\nThis has happened multiple times before. \n", "answer": "Remember that a dictionary is essentially a tool to help you understand a word. You don't have to not use a word because it is not in a dictionary. PIV, while influential, is just a dictionary among others, and is regularly updated in order to reflect the language usage. People should be able to speak Esperanto without having to use a dictionary everytime. I don't remember the last time I used a dictionary in French. If you understood a new word without the help of a dictionary, and the word is not suspicious, then it should be safe to reuse the word without checking a dictionary.\n", "topic": "esperanto", "url": "https://esperanto.stackexchange.com/questions/1914"}, {"image": "VQAonline_00014039.png", "question": "What is the \"/\" in the addresses in Scotland stand for?", "context": "The numbers in the picture below is confusing to me, I have lived in different flats in London and never came across a similar system.\nDo they refer to the floor? the flat number?\nRecently I have sent a parcel to Glasgow from Mail Boxes Etc office and it came back because the address is so confusing that they themselves have got it wrong even when they send hundreds of parcels everyday. \nAdd to that that various high-profile companies/websites like Argos reject the \"/\" when you enter the address manually, so you are either lucky to have the address already in their database or you need to substitute the \"/\" with an equivalent of letters. I have used \"of\" three times so far, the postman hasn't understand that twice.\n \n", "answer": "As mkennedy says, it's the floor/flat.\nWhen I lived in Glasgow it was in the top floor flat (4 floors) on the right, so the address was \nFlat 4/2, 129 XXXXX Road, Glasgow G12 XXX\nThere isn't an explicit numbering in the flats, but usual to number from the left (stair) side, so left would be /1, and so on until you include all flats on the floor.\n", "topic": "expatriates", "url": "https://expatriates.stackexchange.com/questions/7724"}, {"image": "VQAonline_00014036.png", "question": "UK EEA Family permit refused for spouse - reapply or appeal", "context": "My wife applied for EEA family permit for coming to join me in UK.\nShe is from India and I also lived in India before coming to UK. I acquired my Portuguese nationality and then came to UK in July 2015.\nWe are married to each other since December 2014 but we were in a relationship since December 2013.\nWe provided every evidence to support her application except the conversation medium which we used after I arrived in UK.\nThis is what the refusal letter says :\n\nNow for reason 1 I understand that we did not provide sufficient evidence, so next time I am going to put every thing from our chat history in Skype, Whatsapp, Viber and phone logs.\nBut I do not understand how to justify or provide evidence for the second reason.\nCan someone please advice?\nAlso, we got the refusal last week and I am thinking to reapply in next instead of going for appeal as this takes much more time to process. Do you know if there any specific time limit under which I cannot make a new visa application if I am already refused visa.\n", "answer": "I would be inclined to appeal, though I say that without enough awareness of the associated costs, so I can't actually recommend one way or the other whether you should appeal or reapply.\nThe refusal strikes me as scandalous. Consider the guidance for evaluating your application under which the entry clearance operator was working (emphasis added):\n\nEUN2.10 What if I suspect a marriage / civil partnership of convenience?\nThe definition of \u2018spouse\u2019 and \u2018civil partner\u2019 in the EEA Regulations does not include someone who has entered into a marriage / civil partnership of convenience.\nWhen a marriage / civil partnership of convenience is suspected, the burden of proof is high and rests with the ECO. However, in these cases the ECO is entitled to interview the applicant. Factors to consider include:\n\nan adverse immigration history;\ndoubts about the validity of documentation;\napplication follows soon after the marriage / civil partnership;\nno previous evidence of the relationship.\n\nThe ECO should not consider the following cases as marriages / civil partnerships of convenience where:\n\nthere is a child of the relationship;\nthere is evidence to suggest cohabitation.\n\n\nSource: https://www.gov.uk/government/publications/eea-family-permits-eun02/eea-family-permit-eun02#eun210-what-if-i-suspect-a-marriage--civil-partnership-of-convenience\nThe key phrase here is the burden of proof is high and rests with the ECO. In fact, the ECO has responded as if the burden of proof were on you (\"reasonable to expect ... no evidence provided to demonstrate\").\nThe fact that your application came a few months after your marriage is, I suppose, sufficient to invoke the third \"factor to consider,\" but the ECO should have made some effort to develop evidence to satisfy his or her burden of proof, and did not.\nHave the two of you ever lived together? If so, and if either you reapply or your appeal raises the question of additional evidence, I would skip the chat histories and phone logs, and instead rely on the last bullet point, which should be much easier and quicker to show. In addition, it will be a more reliable defense against any suggestion that yours is a marriage of convenience.\nFor an EU perspective on the question, see https://eumovement.wordpress.com/2014/10/16/eu-free-movement-handbook-on-alleged-marriages-of-convenience/. A selection (emphasis removed):\n\nBurden of proof is on the national authorities to prove abuse\nMarried couples cannot be obliged or required, as a rule, to present evidence that their marriage is not abusive.\nEU citizens and their family members enjoy the benefit of assumption, meaning that they do not need to provide evidence that their marriage is genuine. To require this would go beyond the requirement to present proof that their marriage is valid.\nThis reflects the principle of law that the person who lays charges has to prove the charges (\u2018semper necessitas probandi incumbit ei qui agit\u2018).\nThe burden of proof clearly rests on the national authorities who suspect that a non-EU national has entered into a marriage of convenience with an EU citizen for the sole purpose of being granted an EU right to free movement to prove that the marriage is of convenience.\n\nHere is a story of a successful appeal, in which the court awarded the cost of the appeal to the appellant: http://www.immigrationboards.com/eea-route-applications/won-vs-marriage-of-convenience-details-inside-now-t124572.html. A selection:\n\nApplied in June for EEA family permit.\nRejected - marriage of convenience\nAppealed and Reapplied\n2nd application Rejected in July - same reason with very aggressive and bad wording\nToday i got Letter from HM court and Tribunal\nThey decided in my favor and gave my 80 Euro back\n\nIf you read down a bit, you can see the (very poorly scanned) text of the appeal decision. That makes it clear that the ECO has a burden to prove a reasonable suspicion that the marriage is one of convenience, at which point the burden shifts to the visa application to prove that it is not. I do not know whether the amount of time between your marriage and the application is sufficient evidence of reasonable suspicion.\n", "topic": "expatriates", "url": "https://expatriates.stackexchange.com/questions/7324"}, {"image": "VQAonline_00014034.png", "question": "How to read I-797c form (notice of action for H1B)", "context": "I am an Indian citizen and currently a resident of France (on Scientific/D visa until 30/04/2015). A university in the USA that hired me as a faculty recently petitioned for an H1B for me. I received the I-797C form from the USCIS notifying me/the university that the H1B application was submitted.\nHowever, some details on page 1 of the I-797C form elude me. In the screen shot below (with personal details redacted), what exactly does \"Class Consulate / POE OCC 90\" signify?\nA cursory search of the intergoogles revealed not much.\n\n", "answer": "From Immihelp and Department of Labor OCC 90 is an occupation code:\n\nOCCUPATIONS IN EDUCATION\n090 OCCUPATIONS IN COLLEGE AN UNIVERSITY EDUCATION\n091 OCCUPATIONS IN SECONDARY SCHOOL EDUCATION\n092 OCCUPATIONS IN PRESCHOOL, PRIMARY SCHOOL, AND KINDERGARTEN EDUCATION\n094 OCCUPATIONS IN EDUCATION OF PERSONS WITH DISABILITIES\n096 HOME ECONOMICS AND FARM ADVISERS\n097 OCCUPATIONS IN VOCATIONAL EDUCATION\n099 OTHER OCCUPATIONS IN EDUCATION\n\nThe rest seems to be more of an issue with formatting the form then anything else and should read:\nClass | Consulate / POE | OCC\n", "topic": "expatriates", "url": "https://expatriates.stackexchange.com/questions/3023"}, {"image": "VQAonline_00014051.png", "question": "Suggestion needed for uk visa 2nd attempt", "context": "Respected Sir/Mam, \nI have applied for uk Tire 4 visa for LSE summer schools but i have got a refusel . I need suggestions before going for secound attempt . Let me draft a picture of visa application . \nI have fully paid the accomodation as well as tution fee . I was self sponsored i got the funds from the companies where i was invested eariler . \nSo what changed should i make so that i will get visa . \nIf my father will be my sponsor will that be going to work ? \n\nI will be really thankfull if i will get to know the reason s for refusal. \n", "answer": "The letter seems pretty clear. They found inconsistencies in your application regarding your bank statement and claims about your income. This, possibly with other circumstances, has led to the conclusion that they think you will overstay. Your best bet is to work with a professional immigration lawyer who has expertise in these matters.\n", "topic": "expatriates", "url": "https://expatriates.stackexchange.com/questions/11310"}, {"image": "VQAonline_00014053.png", "question": "Tax status when doing micro job in Germany", "context": "I'm working in Germany and I do have a Tax Number.\nRecently I found a mobile app that allows me to do small jobs (for a couple of EUR) by making pictures of things that are of interest for the client.\nNow I want to receive the money I have earned but it asks me to tell them if I'm a private individual or a small company.\nI believe that the first case is right for me.\n\nNow my question:\nIs \"private individual\" the right case here or \"the contractor\" is better (and why)?\nExtra points for explaining how to put this into tax declaration.\n", "answer": "The question is really complicated. To my understanding, if you are getting money for it, there should be either a so-called mini-Job-contract or you should become an entrepreneur.\nPlease read the following materials (in German) for details:\nhttp://www.esteuerpartner.de/steuerinfo/GruenderBasics-crowdsourcing-Recht-Steuern\nhttp://www.ikosom.de/2014/02/08/welche-steuern-muss-ich-beim-crowdfunding-zahlen/\nAnother important point for you is that if you are a foreigner, then, depending on you residence/work permit, an entrepreneurship can be forbidden for you.\nFinally, if you are working, you contract with your employer can also forbid you to do a job aside.\n", "topic": "expatriates", "url": "https://expatriates.stackexchange.com/questions/12571"}, {"image": "VQAonline_00014057.png", "question": "ESTA application with passport issued in UK", "context": "If I'm an Australian citizen, I need to apply for an ESTA before flying to the US for a week-long conference. I'm travelling on an Australian passport, which I recently renewed while I was studying in the UK (I lodged an application to the London passport office).\nSo when I fill in the ESTA form below, what do I put as the answer for \"What country issued your passport?\" Is it \"GBO\" or just Australia?\n\n", "answer": "It is an Australian passport, so the answer is \"Australia.\" \n", "topic": "expatriates", "url": "https://expatriates.stackexchange.com/questions/15607"}, {"image": "VQAonline_00014048.png", "question": "How to pay water bill in Thailand?", "context": "I got my first water bill\n\nOn a bill I am used to getting bank account number into which I need to pay money due. I then usually can go and pay online, however I couldn't find any bank numbers on the slip...\nHow does one pay water bill in Thailand?\n", "answer": "Many condo buildings allow basic utilities (but not always electricity) to be paid at the condo management office. Looking at your bill, it looks like this is how you should pay this bill.\n", "topic": "expatriates", "url": "https://expatriates.stackexchange.com/questions/10157"}, {"image": "VQAonline_00014172.png", "question": "Why does my fitness watch show calories whereas calories on food packages are in kilo calories?", "context": "I am trying to track the calories which I burn during my exercises using wearable fitness tracker watch (Gear Fit2 Pro), but I am pretty confused by the amount of calories which the watch shows me.\nBurning Calories:\nMy watch says, I am burning almost 3000 Calories everyday.\nReceiving Calories:\nI get 373K Calories (or 373,000 Cal) from my dinner and lunch. \nI obtained this number from the rice and meat package.\nIf it is true, it means that, I am receiving much more calories than what i am burning, and definitely I must be a fat guy, however I am 180cm and 74KG (31 year old). So, I am almost fit.\nSo, I guess, something is wrong! Either the watch must also show more burned calories or the food packages are lying!!!\nAnyways, please tell me, how should I track the calories which I burn and receive?\n\n", "answer": "They're actually the same thing as far as this is concerned.\nA \"Calorie\" and \"Kilocalorie\" is the same when talking about nutrition and exercise. Your watch says you burned 3000 Calories in the day and your dinner was 373 Calories. What food companies choose to put on food packaging is regional and usually dictated by the government. In the United States, they show everything as \"Calorie\" but other countries may choose the other. Fitness trackers aren't bound by the same laws, so they may just use \"Calorie\" because it's easier.\n\nTechnically, the calorie the amount of heat energy needed to raise the temperature of 1 gram of water by 1 degree Celsius. This is also referred to as the \"gram calorie\" or \"small calorie\". It's still used in chemistry and other science fields.\nA Kilocalorie the amount of heat energy needed to raise the temperature of 1 kilogram of water by 1 degree Celsius. This is referred to as the \"Big Calorie\".\nThe big Calorie is used in nutrition because the amount of small calories food contains is actually in the thousands. So rather than saying something is \"100,000 cals\" you just say \"100 Cals\". Likewise, because the amount of calories people burn is in relation to food, so you'd use the big Calorie. \n", "topic": "fitness", "url": "https://fitness.stackexchange.com/questions/42168"}, {"image": "VQAonline_00014160.png", "question": "Practical applications of the earthquake/bamboo bar", "context": "My gym recently purchased this weird bar:\n\nI have yet to see anyone actually use it. From what I can tell, the point is to hang weights off the sides and they swing at each rep. The idea (I guess) is to make the bar unstable.\nHowever, it seems to me that, as it pertains to strength training, using a bar like this would either have too much weight to be safe or not enough weight to be very effective.\nWhat use does this type of bar have? What would be an effective way of using it?\n", "answer": "Wow, you're lucky! Kudos to your gym for acquiring one of these. I wish my gym had one too.\nWhat is that?\nIt's commonly referred to as a \"bamboo bar\" because of its resemblence. It's actually plastic, as I'm sure you've surmised if you've tried it out.\nEric Spoto (one of the best bench pressers in the world) is a strong advocate for incorporating the bamboo bar bench press into your routine, because of the instability it causes, which in turn forces you to activate a lot of the balance muscles that usually don't get a lot of focus.\nHow do I use it?\nHe advocates using quite low weights, and going for 3-5 sets of 10-15 reps. There's really no point trying to measure your 1-rep max on such an auxilliary exercise. This is for volume, and to train those hard-to-engage muscles.\nIn terms of \"how do I even?\", you use it like you do with a regular bench press. Same setup routine, same prep, same execution. It's just more unstable.\nAnother benefit is that if your bench technique isn't very good, it will let you know very quickly that you have some flaws in your setup. And more importantly, working on this variation will help resolve those issues.\nI highly recommend watching this video, where Eric Spoto himself takes the time to instruct and demonstrate this movement. https://www.youtube.com/watch?v=5AHCpJecV2U\n", "topic": "fitness", "url": "https://fitness.stackexchange.com/questions/37409"}, {"image": "VQAonline_00014196.png", "question": "name of this exercise", "context": "I search for the name of this exercices :\n(I am very strong in drawings)\n\n\nthe arm is against the body with the dumbbell in suspension\nkeep the elbow glued to the body and raise the forearm\nwe finish the movement with the dumbbell aligned to our arm, one weight on the shoulder the other pointing in front, the elbow pointing in front\n\n", "answer": "That sounds like a Hammer Curl as per the list here.\n\nThis bicep curl variation is like the standard biceps curl with one small adjustment. Turn the palms towards the midline of the body with the thumbs up. Curl the weight up towards the shoulder creating flexion only at the elbow. Extend the elbow and lower the weight back to the starting position.\nThe rotation of the palms adds a focus on the brachialis and brachioradialis in a way that many other variations cannot.\n\nHow-To Video\nAdding the elbow floating away turns it into a \"cheat curl\" variation. Here, the \"cheat\" is not necessarily meant pejoratively. It just means that, by letting the elbow move, you're recruiting other muscles, such as the deltoids in your back, which reduces the biceps isolation, but will allow you to lift more weight, which can translate to a bump in your ability to lift heavier weight in a \"strict\" hammer curl.\n", "topic": "fitness", "url": "https://fitness.stackexchange.com/questions/44958"}, {"image": "VQAonline_00014082.png", "question": "What can explain my feet cramping while using swim fins when I swim in the afternoon/evening?", "context": "I've been swimming for the last couple years and currently swim 4 times a week. Two of those days I swim in the morning, the other two are in the evening. Whenever I swim in the evening and swim a set using swim fins my feet and toes tend to cramp up the next swim set.\n\nI've tried lots of things, potassium and water because I read they help with cramps as well as stretching before and after the swim fin set. So far, neither has worked. My only other thought was perhaps the fins are the source of the problem. Swimming in the evening without using the fins doesn't bother me, so its not just because I swim in the evening. At the same time, if I use the fins in the morning, I don't cramp up either.\nI was just hoping someone here might have a suggestion and/or solution as to what may be causing this.\n", "answer": "There are various reasons for using fins. Don't give up on them as they can be an excellent training tool.\nAs a swim coach, I do not take a 'one-fin-fits-all' approach, especially with training adults. Yes, foot size does matter and if the fin is ill-fitted (too small OR too big) the foot and ankle and foot will have to flex/tense more to use the fin.\nIn addition, the level of resistance the fin provides will also influence cramping. If you have never used fins before or your ankles are weak (in respect to this exercise), a larger blade on the fin will increase the chances of fatigue and/or cramping.\nStarting off with a split fin or, even better, a cut or short fin, will help you begin to increase ankle/calf strength. You can progress to a larger or firmer blade from there.\nHere are a couple of fins I like for starting off:\nhttp://www.swimoutlet.com/product_p/1647.htm\nhttp://www.swimoutlet.com/product_p/1267.htm?color=206\nI've personally taken my long fins and cut them to the size I want.\nSo you know, I have no allegiance to any one particular brand, just know what I've found works best during training (for myself and my swimmers)\nAlso, wearing socks to make the fin 'snug' helps - can eliminate blisters, too.\n", "topic": "fitness", "url": "https://fitness.stackexchange.com/questions/3928"}, {"image": "VQAonline_00014093.png", "question": "What are the primary muscles worked when hitting a Heavy Bag?", "context": "What are the primary muscles used while hitting the heavy bag (boxing focus sessions as compared to kickboxing where kicks are involved). And how do you improve the overall hitting/power of those strikes from accessory exercises?\n\n", "answer": "Start at your head, and go to your feet. That's the muscles involved. :p\nSeriously, a good punch will involve most of the muscles in the body. The primary single movers in a punch are going to be\n\nPecs (Chest)\nTriceps (Arm)\nDeltoids (Shoulder)\n\nYou could throw a punch just using those muscles, but it won't be a hugely powerful punch (Think a jab in boxing). The triceps straighten the arm, and the delts/pecs move the arm \"forward\" and in towards the center.\nTo that, you can add the muscles of the abdomen (abdominals and internal/external obliques) for rotation in the execution of the punch, muscles of the leg (calves and hamstrings/quads) used for driving and transmitting ground force through the punch, forearm muscles for rotation and stabilization of the lower arm, and lats/biceps for oppositional force and arm retraction after the punch. Clenching your jaw at the point of impact will tense/stabilize and add force, although a permanent clench is counter productive.\nThink of a right cross, from the point of view of a right handed boxer (lead hand is left). Starting position for a right handed boxer is the left foot and left shoulder forward, right leg back, so that your hips and upper body are angled to your opponent. A few boxers will fight more square, though. \nAs the boxer settles from movement/shuffle, the feet get planted. The rear leg starts hip rotation and gives a \"push\" from the feet upwards, involving the calf and thigh muscles. As the hip and torso continue to rotate (obliques and abdominals, some lats) the upper body continues the rotation, and the arm starts to extend out (pecs/triceps/delts), and the hand/lower arm rotate so that at the point of contact, the fist is palm down as opposed to facing inward as in a typical sparring/protection stance.\nAs far as hitting power, any basic weightlifting routine will work all the muscles involved, as most of them are primary movers for the core exercises such as pullups/dips, squats, situps and rotational exercises (abs and wood choppers for example). Where a LOT of the power comes from is simply technique, learning how to plant and rotate, keeping the arm as relaxed as possible until point of impact, things of this nature. You could have one person that can outlift a second person by a considerable margin, but not be able to punch as effectively because of a lack of technique.\n", "topic": "fitness", "url": "https://fitness.stackexchange.com/questions/7523"}, {"image": "VQAonline_00014120.png", "question": "Looking for equivalent workout to Upright Barbell Row excersie", "context": "i'm looking for an equivalent workout to this shoulder workout i've been practicing. \nI had to stop practicing this because my wrists hurt! (both my wrists were broken when i was a kid) \nAny help is greatly appreciated ! \n\n", "answer": "Since upright rows stress the traps and the shoulders, I would switch to using barbell or dumbbell shrugs. Your shoulders won't get as much work, but, if done correctly, the traps should benefit. Additionally, either exercise should reduce any strain on your wrists.\n", "topic": "fitness", "url": "https://fitness.stackexchange.com/questions/18565"}, {"image": "VQAonline_00014193.png", "question": "Unable to catch the Clean", "context": "I've started to perform the Clean from a month, and I often have an issue: when I try to catch the bar, it lands on my upper chest instead on my collarbone.\nThe following picture shows the point where the bar touches me when I try to catch it (in red). I do not do this mistake every time, but when I do it I instinctively stay for a while in a sort of isometric contraction (about 1/2 second) desperately trying to bring the bar to my collarbone for the rack position. When it happens, I probably use my arms and I feel pain in the blue points in the following picture.\n\nTo make it clear, I've uploaded a video there.\nMy Opinion: possible causes are\n\nMy legs aren't able to pull the bar high enough to get under it. However, sometimes this problem occurs also with a light weight (despite less frequently)\n\nI'm not fast enough to get under the bar. From my video I think I'm not able to go under the bar. However, I think this can't be the only mistake. In fact, as far as I'm concerned, the Muscle Clean exercise allows the athlete not to go under the bar. So, even if the purpose was to make a normal clean, this technique is not inherently harmful. But I feel pain in the blue points, so there is probably a mistake I cannot understand.\n\nMy elbows are not quick at raising up\n\n\nWhat do you think is the cause and do you have any suggestions to fix it?\n", "answer": "Catching the bar on your chest can be caused by any of the options you laid out in your question. From your video, it seems that you are trying to pull the bar too high, and allowing your forearms to come forward. This creates a rounded bar path and slows the turnover of your elbows.\nA few things to try:\nTry using straps (even with low weights) to remove the tension in your arms, and do high pulls and muscle cleans keeping your elbows up not back (ie keep your forearms vertical). The bar should rise to about nipple height.\nTry catching your clean a little lower. Instead of pulling the bar very high to catch almost standing, try to catch it with a little more bend in the knees. This should give you more leeway to catch properly.\nYou may be limited by your mobility. If you find you cannot get into a good rack position, try stretching your lats and triceps as well as mobilizing your T spine.\n", "topic": "fitness", "url": "https://fitness.stackexchange.com/questions/44563"}, {"image": "VQAonline_00014313.png", "question": "What is the correct way of describing a standard brick pattern?", "context": "In English the standard brick pattern (where each brick lies half across the one below it - see below) is technically called \"Running Bond\" but might be described as \"staggered\" or \"overlapping\" by the layman.\nHow might I describe this in French (for example if I wanted tiles arranged like this)? I see the following translations for staggered: \"\u00e9chelonn\u00e9\", \"\u00e9tal\u00e9\" and \"d\u00e9cal\u00e9\" and for overlapping: \"superpos\u00e9\" but I am pretty sure that isn't right. Alternatively, a translation for saying \"brick-pattern\" might get the message across.\n\n", "answer": "Apr\u00e8s quelques recherches il semble que beaucoup de sites emploient le terme \"en quinconce.\"\nJ'emploierais personnellement ce terme pour peu que les briques ne soient pas align\u00e9es, m\u00eame si elles ne sont pas d\u00e9cal\u00e9es exactement de la moiti\u00e9 de leur longueur, mais je serais sans-doute dans l'erreur car il semble que le terme d\u00e9signe sp\u00e9cifiquement l'alignement tel que sp\u00e9cifi\u00e9 dans la question (les briques sont align\u00e9es une ligne sur 2)\nNotons que l'une des pages li\u00e9es dans la d\u00e9finition wikip\u00e9dia renvoie vers une formation de l'arm\u00e9e romaine o\u00f9 l'on parle effectivement de disposition en quinconce et le sch\u00e9ma correspond lui aussi.\n", "topic": "french", "url": "https://french.stackexchange.com/questions/40415"}, {"image": "VQAonline_00014227.png", "question": "Are there any textbooks based on the natural method?", "context": "Question: Are there any textbooks for learning French that are based on the natural method?\nWhat's the natural method, you say? It is a method of learning the language like a child would, that is the textbook would be all in French and would be very easy to read at the beginning, even with no previous knowledge of the language, and then progresses to more advanced texts. There are no translation excercises, no explanations of words or grammar in your mother tongue \u2013 the point is to get you to immerse yourself into the language, to start to think in the language. Words that might be difficult are explained by illustrations. Hans \u00d8rberg's Lingua Latina per se illustrata (something like \"Latin illustrated in itself\") is a perfect example:\n\n", "answer": "@citizen: That's a great question... The only book that I have been able to find that matches your description, is a book called \"Easy French Reader,\" by R. de Roussy de Sales (ISBN 978-0-07-142848-4). The book starts off with \"easy\" stories, and the level of difficulty progresses with each chapter. There is a small glossary, and on average about three of the more difficult words appear on the right margin of the page. The book starts off with fictional stories, moves on to stories about the history of France, and ends with excerpts from stories written by French authors. \nI've found this book to be extremely helpful, but I've had difficulty finding more that are like it. If you can find any more books like this one, post the info--I would greatly appreciate it. Whether this book is what you're looking for or not, I wish you the best of luck, and happy reading!\n", "topic": "french", "url": "https://french.stackexchange.com/questions/9255"}, {"image": "VQAonline_00014306.png", "question": "Le \u00ab tin tie coffee bag \u00bb ?", "context": "Comment nomme-t-on ou d\u00e9crit-on un \u00ab tin tie coffee bag \u00bb ? On utilise souvent ce genre de sac que ce soit de papier ou en plastique pour le vente de caf\u00e9 moulu ou en grains. On a parfois des emballages sous vide de caf\u00e9 qui une fois ouverts disposent d'un \u00ab syst\u00e8me \u00bb similaire pour la fermeture. L'\u00ab \u00e9l\u00e9ment \u00bb du haut sert \u00e0 \u00e9viter que ne se d\u00e9roule le rebord qu'on a roul\u00e9. Il s'agit non seulement de traduire les termes mais encore de se demander si la pr\u00e9position suffira pour agencer le sac avec le \u00ab m\u00e9canisme \u00bb servant \u00e0 le fermer ou s'il faudra un verbe (participe pass\u00e9), tout en \u00e9vitant l'\u00e9sot\u00e9rique. On notera que ce qui se trouve fix\u00e9 au rebord sup\u00e9rieur du sac existe aussi sous diff\u00e9rents formats comme objet \u00e0 part enti\u00e8re et sert \u00e0 des fonctions similaires avec d'autres types de sacs ; on aura un nom est une \u00e9pith\u00e8te, comme dans tin tie.\n\nPour fins d'illustration.\n", "answer": "Je connais en France le syst\u00e8me de fermeture de paquets alimentaires o\u00f9 l'on fait coulisser une baguette (ou tige) en plastique sur le bord sup\u00e9rieur qui a \u00e9t\u00e9 soigneusement repli\u00e9 et aplati.\nIl me semble que ce qui est d\u00e9crit dans la question est un syst\u00e8me o\u00f9 la baguette est beaucoup plus longue que la largeur du paquet de fa\u00e7on \u00e0 pouvoir en replier les extr\u00e9mit\u00e9s de chaque c\u00f4t\u00e9 du paquet. Je n'ai jamais vu en France ce syst\u00e8me (ce qui ne veut pas dire que \u00e7a n'existe pas, bien s\u00fbr), que la fermeture soit fix\u00e9e ou non au paquet.\nEn cherchant sur des sites de vente en ligne j'ai trouv\u00e9 des \u00ab\u00a0barrettes de fermeture\u00a0pour sacs en papier\u00a0\u00bb vendus en anglais sous le nom de tin ties.\nUn autre site propose ce m\u00eame syst\u00e8me de fermeture sous le nom \u00ab\u00a0d'attache twist adh\u00e9sive\u00a0\u00bb. Dans la mesure o\u00f9 le syst\u00e8me est assez souple pour \u00eatre repli\u00e9 je trouve que le mot d'\u00ab\u00a0attache\u00a0\u00bb convient mieux que celui de \u00ab\u00a0barrette\u00a0\u00bb, la\u00a0barrette \u00e9voquant pour moi quelque chose de pus rigide. Il est pr\u00e9cis\u00e9 par ailleurs que ces liens adh\u00e9sifs sont en \u00e9tain.\nIl existe aussi des sacs \u00e0 fermeture zip int\u00e9gr\u00e9e au sac mais je ne crois pas que ce genre de fermeture soit vraiment r\u00e9utilisable apr\u00e8s premi\u00e8re ouverture. \nSi je devais faire la traduction de tin tie bag pour un site de vente en ligne je proposerais certainement quelque chose comme : \u00ab\u00a0sac refermable avec un syst\u00e8me d'attache en \u00e9tain repliable\u00a0\u00bb, ou \u00ab sac refermable par syst\u00e8me d'attache en \u00e9tain repliable \u00bb. Et en ajoutant peut-\u00eatre \u00ab\u00a0fixe\u00a0\u00bb ou \u00ab\u00a0adh\u00e9sif\u00a0\u00bb apr\u00e8s \u00ab syst\u00e8me \u00bb selon le cas. \u00ab\u00a0Sachet\u00a0\u00bb pouvant \u00e9ventuellement \u00eatre mis \u00e0 la place de \u00ab\u00a0sac\u00a0\u00bb, (ce serait bag dans tous les cas en anglais).\nNous avons donc encore un exemple de la concision de l'anglais qui peut exprimer en trois mots tin tie bag (analysables je suppose comme un substantif pr\u00e9c\u00e9d\u00e9 de deux \u00e9pith\u00e8tes qui servent \u00e0 d\u00e9crire le syst\u00e8me de fermeture) ce que le fran\u00e7ais exprime en cinq ou plus, une pr\u00e9position (\u00ab\u00a0avec \u00bb ou \u00ab\u00a0par\u00a0\u00bb) introduisant le m\u00e9canisme de fermeture.\n\nJe me suis amus\u00e9 \u00e0 chercher comment d'autres sites en ligne anglophones pouvaient chercher \u00e0 vendre ce type de produit \u00e0 des francophones, je vous livre quelques r\u00e9sultats plut\u00f4t amusants (ou d\u00e9solants ?).\nApr\u00e8s un passage rapide sur les \u00ab\u00a0tin tie sacs\u00a0\u00bb, clair et transparent pour quiconque parle anglais, la \u00ab bo\u00eete cravate \u00bb ou le \u00ab\u00a0sac de cravate \u00e9tain \u00bb m'ont quand m\u00eame demand\u00e9 quelques secondes de r\u00e9flexion.\n", "topic": "french", "url": "https://french.stackexchange.com/questions/37352"}, {"image": "VQAonline_00014264.png", "question": "What is a \u2018t\u00e9moin agent(e)\u2019 (\u2018witness-agent\u2019/\u2018agent-witness\u2019)?", "context": "As previously mentioned, I\u2019m translating excerpts from some letters from the 1830s which are written in a somewhat nonstandard orthography, very idiosyncratic punctuation, and sometimes with rather peculiar expressions.\nI\u2019m currently at a bit which describes a woman with the phrase (if a phrase it is) t\u00e9moin agente, and I\u2019m having a bit of trouble figuring what exactly that is. The context would seem to imply that it\u2019s simply a witness, but since I can find exactly one instance of the word being used on the Internet, I don\u2019t have any way of verifying that this is in fact so.\nThe phrase occurs at the beginning of a letter:\n\n\nConstance, ma ch\u00e8re Natalie, me prie de vous dire que ses esprits ne sont pas assez concentr\u00e9s pour pouvoir se mettre auteur, en rapportant les sc\u00e8nes, auquelles elle a \u00e9t\u00e9 t\u00e9moin agente depuis ses derniers ouvrages en forme de lettres; toutes plaintes ont cess\u00e9s, car elle a re\u00e7u 4 lettres de la part de sa ch\u00e8re Maman\u2026\n\nThe \u2018scenes\u2019 referred to here are mainly the fact that Constance\u2019s infant child had just gone through a terrible ordeal, nearly dying from some unknown illness. Saying that she doesn\u2019t feel up to writing letters just yet describing the scenes she\u2019s witnessed sounds quite reasonable and logical.\nAccording to TLFi,\n\nL\u2019emploi au f\u00e9m. est exceptionnel. Ac. 1878 signale que \u201elorsque le mot est pris en mauvaise part, on lui donne quelquefois un f\u00e9minin\u201c avec les ex. suiv.\u00a0: \u201eElle est leur principale agente. Je d\u00e9couvris que dans cette intrigue, elle \u00e9tait la principale agente.\u201c\n\nGoing by that, the usage in the letter is additionally odd in that it uses the decidedly nonstandard feminine form of the word. Looking for t\u00e9moin agent in the masculine doesn\u2019t reveal anything more useful, though\u2014everything I can find is examples like \u201cle t\u00e9moin, agent de police, \u00e9tait\u2026\u201d, where the two words just happen to be right next to each other, but have nothing really to do with each other.\nAgent itself comes from the Latin \u0101g\u0113ns, which is the present participle of \u0101g\u014d \u2018act, do\u2019. If we take that straight down to Modern French, one might assume that it means an \u2018acting/active witness\u2019 (someone who is an active part in the goings-on described, I suppose)\u2014but the Modern French outcome of \u0101g\u014d is agir, whose present participle is agissant, not agent, so that doesn\u2019t seem a likely explanation either.\nSo what does agente add to t\u00e9moin in the particular context in my letter here? What does it mean to be t\u00e9moin agent(e) to something?\n", "answer": "The passage you quote \"Constance...\" is a bit cryptic and hard to understand, but I would venture the following possible interpretation, that you might be able to validate from the context. \nI would interpret t\u00e9moin agente as meaning that Constance is/was both a witness and an active participant in the events she should write about. That would be in contrast to being an ordinary t\u00e9moin or an ordinary agent, who are not both witness and participants, and might have special legal value. Here, possibly, Constance was a witness to the events (the complaints? the letters?), but also the recipients of the letters, maybe? I would need more context at this point. \n", "topic": "french", "url": "https://french.stackexchange.com/questions/25218"}, {"image": "VQAonline_00014229.png", "question": "Sens de la phrase \u00ab descendre l'escalier \u00bb", "context": "Comment distingue-t-on entre sens de la phrase \u00ab descendre l'escalier \u00bb ? Je crois que ceci peut signifier mouvement de sujet ( 1 ) aussi bien que mouvement de l'escalier se ( 2 ).\n\nNouvelle information :\nIl y a une \u00e9chelle dans deuxi\u00e8me cas. Ma question est maintenant sur la phrase \u00ab descendre l'\u00e9chelle \u00bb :-)\n", "answer": "A ma connaissance il n'y a pas de moyen de savoir si la phrase \"Descendre l'escalier\" signifie Se d\u00e9placer dans l'escalier vers le bas ou effectuer un d\u00e9placement de l'escalier en direction du bas.\nTout est une affaire de contexte mais heureusement dans l'immense majorit\u00e9 des cas, cela correspondra \u00e0 la premi\u00e8re signification. Un rare cas contraire qui me vient \u00e0 l'esprit est celui des avions, o\u00f9 on parle parfois de \"descendre l'escalier\" pour d\u00e9signer l'ouverture des portes-escaliers. \nDans le cas d'une \u00e9chelle qui est par d\u00e9finition moins statique, les deux sens peuvent \u00eatre possibles et seul le contexte permet de faire la distinction.\n(Bien entendu, il en va de m\u00eame pour \"monter l'escalier/l'\u00e9chelle\"...)\nA noter que l'expression \"descendre de l'escalier/l'\u00e9chelle\" est elle plus claire puisqu'elle n'a qu'un seul sens, diff\u00e9rent des deux autres : celui de quitter par le bas l'escalier (ou l'\u00e9chelle) o\u00f9 on se trouve. \n", "topic": "french", "url": "https://french.stackexchange.com/questions/11656"}, {"image": "VQAonline_00014236.png", "question": "Le jeu de plateau et l'id\u00e9e du \u00ab paragraph driven \u00bb ?", "context": "Certains jeux de plateau (un jeu de table, un jeu de soci\u00e9t\u00e9), utilisent des paragraphes num\u00e9rot\u00e9s dont le texte met en sc\u00e8ne les cons\u00e9quences d'un choix qu'avait fait le joueur auparavant dans la s\u00e9quence de jeu.1 \nContrairement au concept du livre-jeu, et plus particuli\u00e8rement du \u00ab livre dont vous \u00eates le h\u00e9ros \u00bb, o\u00f9 le choix est cantonn\u00e9 aux paragraphes qu'on encha\u00eene selon les indications, ici le paragraphe n'est qu'une composante d'un jeu plus \u00e9labor\u00e9; avec par exemple un plateau, des figurines, des cartes etc. L'emploi du paragraphe est assur\u00e9ment une particularit\u00e9 servant \u00e0 sa classification mais n'en constitue pas son \u00ab interface \u00bb.2\n.\n\u00c0 gauche, un grille et des paragraphes d'un jeu de plateau; \u00e0 droite, deux paragraphes d'un livre-jeu. J'aimerais distinguer la \u00ab caract\u00e9ristique \u00bb du \u00ab format \u00bb.\nComment exprimer de mani\u00e8re nuanc\u00e9e (et possiblement succincte) le recours aux paragraphes au niveau du sc\u00e9nario et leur relation avec ce jeu, dans la m\u00eame phrase? \n\nIl s'agit d'un jeu de plateau + [?] + [\u00ab paragraph driven \u00bb/une s\u00e9rie\n de paragraphes].\n\n\n1. En anglais, on a l'expression \u00ab paragraph driven \u00bb comme m\u00e9canisme, dans le contexte d'aventure solo par exemple; autre exemple.\n2. Voici un vid\u00e9o traitant de la place du paragraphe dans la s\u00e9quence d'un jeu (provenant d'une liste de jeux ayant cette particularit\u00e9). On explique que d'une carte, on en arrive \u00e0 une situation, \u00e0 laquelle on associe un choix, et o\u00f9 l'on croise des valeurs dans un index auquel on ajoute un \u00e9l\u00e9ment al\u00e9atoire, pour enfin en arriver \u00e0 un paragraphe particulier (\u00e0 17:00) o\u00f9, d\u00e9pendamment des caract\u00e9ristiques du joueur, il se passe alors \u00ab quelque chose \u00bb.\n", "answer": "Si je comprends bien, dans le type de jeu que tu d\u00e9cris\u202f:\n\nle livre de paragraphes est une composante importante du jeu, puisqu'il faut la mentionner\nmais pas au point que \u00e7a soit le format du jeu. Id\u00e9alement, la description met en \u00e9vidence que le livre n'est le format du jeu, au contraire de ceux \u00ab\u202fdont vous \u00eates le h\u00e9ros\u202f\u00bb.\nla relation entre le livre et le jeu est que le livre raconte (narre, met en mots) et guide (pilote, dirige) l'action qui se d\u00e9roule sur le plateau.\n\nJe parlerais donc d'un jeu de plateau\u202f:\n\npilot\u00e9 par les fragments de sc\u00e9nario du livre\nencadr\u00e9 par un syst\u00e8me de passages narratifs\ndont l'action est dirig\u00e9e par des paragraphes de r\u00e9cit\n\n", "topic": "french", "url": "https://french.stackexchange.com/questions/13832"}, {"image": "VQAonline_00014225.png", "question": "Hawaii et Hawa\u00ef", "context": "J'ai cru comprendre lors de mes recherches sur internet que la bonne orthographe \u00e9tait celle avec le i tr\u00e9ma. Pour ma part j'avais l'habitude d'\u00e9crire les deux i.\nJ'ai vu sur le Wiktionnaire que les deux i \u00e9taient en fait une variante. J'aimerais savoir dans quelle mesure cette variante est-elle correcte et d'o\u00f9 vient-elle ?\n\n\u00c9dit : graphique repr\u00e9sentant l'utilisation des variantes dans la litt\u00e9rature Fran\u00e7aise.\n\n", "answer": "L'orthographe fran\u00e7aise est Hawa\u00ef, mais on retrouve \u00e9galement l'orthographe anglaise Hawaii. L'orthographe anglaise est elle-m\u00eame d\u00e9riv\u00e9e de l'orthographe hawa\u00efenne Hawai\u2018i. \u00c0 Hawa\u00ef m\u00eame, l'orthographe hawa\u00efenne domine.\nLe dictionnaire de l'Acad\u00e9mie fran\u00e7aise ne contient pas d'entr\u00e9e sous Hawaii ou Hawa\u00ef. Il y a, cependant, une entr\u00e9e sous hawa\u00efen qui se lit comme suit\u00a0:\n\nHAWA\u00cfEN, -ENNE (wa se prononce oua) adj. XIXe si\u00e8cle. De Hawa\u00ef, nom polyn\u00e9sien de la plus grande des \u00eeles Sandwich.\n Relatif aux \u00eeles Hawa\u00ef. Les ports hawa\u00efens. Le climat hawa\u00efen. Guitare hawa\u00efenne, voir Guitare. Subst. Un Hawa\u00efen, une Hawa\u00efenne, personne habitant les \u00eeles Hawa\u00ef ou qui en est originaire. \u2022\u00a0G\u00c9OL. Volcan hawa\u00efen, volcan dont l'activit\u00e9 est continue et dont le crat\u00e8re est empli d'une lave bouillonnante qui d\u00e9borde lors des \u00e9ruptions. La \u00ab\u00a0chaudi\u00e8re de Kilauea\u00a0\u00bb est un volcan hawa\u00efen.\n\nIl n'y a pas d'entr\u00e9e sous hawaiien.\nLe coup de glotte (l'apostrophe invers\u00e9e) est ce qui dicte la s\u00e9paration des deux i dans la prononciation anglaise et le tr\u00e9ma en fran\u00e7ais. Aux \u00c9tats-Unis, on prononce le h initial, on met l'accent tonique sur la deuxi\u00e8me voyelle (le \u00ab\u00a0wai\u00a0\u00bb) et on prononce le i final sans accentuation particuli\u00e8re\u00a0:\u00a0/h\u0259\u02c8wa\u026a.i\u02d0/. En fran\u00e7ais, la prononciation /aw\u0251.i/ est, \u00e0 mon avis, une des meilleures approximations.\n", "topic": "french", "url": "https://french.stackexchange.com/questions/8889"}, {"image": "VQAonline_00015616.png", "question": "How does the new hacking mini-game work?", "context": "In the Eve Online expansion Odyssey, a hacking mini-game for hacking relic sites was added which looks like this;\n\nClicking through this game, it seems that there are multiple types of nodes that do varying amounts of damage to your \"hacking software\" and you need to find the system core to win the mini-game. This leads me to a few questions about how this game works;\n\nHow many different kinds of nodes are there?\nIs the damage caused by specific types of node consistent or random?\nAre the rewards based on how many nodes you light up while looking for the system core?\nIf you lose, can you try again?\n\nObviously, any other information you can provide me on how these mini-games work would also be appreciated!\n", "answer": "The devblog on hacking covers most of the basics, and via personal experience and a quick refresher at the UniWiki Odyssey page I have this to say:\n\nThere are five different kinds of nodes: Empty, System Core, Data Cache, Defensive Subsystem and Utility Subsystem - with the defensive Subsystem and Utility Subsystem having a few flavors each. Data Caches are either Defensive or Utility Subsystem.\nThe damage is consistent per node, but different types of the same node are random within a specific set - i.e. the same node will always hit you for the same amount, but another node of the same type may be different.\nNo.\nYes, but the number of released containers increases, without increasing the amount of available loot, meaning each individual container will have less in it. However if you fail too many times (currently 2) then the loot will be destroyed.\n\nAdditionally, the profession containers can be cargo scanned. If the loot isn't overly valuable, skip it. The ISK/hr may be better without wasting the time.\nExtra extra additionally, Ali Arias wrote a detailed piece of the entire process over at themittani.com. It's a bit of a read, but it covers all of the salient points.\nOnce you've won the mini-game and the look has exploded into space you have about 10-15 seconds before the last of it disappears. \nThe individual containers are divided into types (currently Data, Materials, Parts, and Scraps) with their contents being determined according to their type. Scraps containers are effectively worthless, don't waste time on these and reliably collecting all the loot requires at least 2 characters doing it.\n", "topic": "gaming", "url": "https://gaming.stackexchange.com/questions/119414"}, {"image": "VQAonline_00016867.png", "question": "Complex message templates in Dark Souls 3", "context": "Occasionally I come across messages which seem to follow a template which I don't have available to me (for example, see the below screenshot).\nHow do you leave messages like this? Can you acquire more complex message templates somehow?\n\n", "answer": "While in the message menu, press \"Y\" (Xbox) or Triangle (PlayStation) to cycle through the different template/gesture combinations. \nThis includes the \"simple\" template (one phrase with a blank to fill), and the \"complex\" template (two phrases with blanks to fill, linked by a conjunctive word/phrase).\n", "topic": "gaming", "url": "https://gaming.stackexchange.com/questions/269059"}, {"image": "VQAonline_00017750.png", "question": "Can I stop another Civ's trader from crossing my land?", "context": "I have an enemy who is crossing my land to trade with another civ. Can I stop them from crossing without going to war?\n\n", "answer": "No, you cannot prevent another nation's traders from crossing your territory.\nOnce you research the Early Empire civic, you gain the ability to enforce borders against military units and allow friendly nations to cross them.\nHowever, traders and religious units are exempt from closed borders, and can wander freely into your territory. See Civ VI Border Rules for more information.\n", "topic": "gaming", "url": "https://gaming.stackexchange.com/questions/362076"}, {"image": "VQAonline_00017744.png", "question": "Why am I seeing so many Lairon recently?", "context": "For the last few days, I've been seeing a considerable number of wild Lairon. I don't recall ever seeing a wild Lairon before - only Aron - but over the last few days I must have seen dozens.\n\nI don't particularly mind, as I have an Aggron with near-perfect IVs that could use the Candy. But why are so many of them appearing all of a sudden? I can only assume this is part of an event, but the only one I know of that's happening right now is the Evolution Event, and Lairon doesn't have any special evolutionary requirements, so it's not that.\n", "answer": "It is part of this Evolution Event indeed, even though it has no special evolution requirement. There are more Pokemon that appear more often, for example Haunter, Dragonair and Graveler. Make use of it while it lasts!\n", "topic": "gaming", "url": "https://gaming.stackexchange.com/questions/361341"}, {"image": "VQAonline_00015052.png", "question": "How do I become the Puppet Master?", "context": "I'm trying to get the achievement Puppet Master for the Witch Doctor:\n\nI've got the following:\n\n4 Zombie Dogs (requires the Zombie Handler passive skill)\nGargantuan\nFetish from Big Bad Voodoo\nMy Templar Follower\n\nThis seems like it should be enough, right? Furthermore, the pets I've got correspond to the ones in the achievement's picture. However, the achievement has not yet popped. What am I doing wrong?\n", "answer": "Big Bad Voodoo doesn't really count as a pet,it is more like AOE ward.\nOn lvl 30 you will get fetish army and it makes an easy achievement.\nAlso additional followers you get from quests like Tyrael,Leah... don't count.But main followers templar,enchantress and scoundrel do.\n", "topic": "gaming", "url": "https://gaming.stackexchange.com/questions/68373"}, {"image": "VQAonline_00018071.png", "question": "What game is this character creation screen from?", "context": "Does anyone recognize this game?\n\nI saw this image and it looks rather cool. I was thinking divinity but don't remember this class.\n", "answer": "Similar images seen here suggest it's Baldur's Gate 3.\nBacking Video: \n\n\n", "topic": "gaming", "url": "https://gaming.stackexchange.com/questions/399569"}, {"image": "VQAonline_00014414.png", "question": "What do the values on the Minecraft debug screen represent?", "context": "If you hit F3 while playing Minecraft, an overlay appears. Some of the items, such as fps, x, y, and z are obvious, but I'm curious about some of the other values. What do they all represent?\n\n", "answer": "Debug numbers\n\nC is the number of renderers* being rendered over total number loaded\nF is number of renderers being clipped (i.e. not viewable)\nO is the number of renderers removed through occlusion culling\nE (first line) is the number of renderers skipped\nE (second line) is the number of rendered entities over total entities\nB is the number of entities hidden\nI is the number of individually removed entities (the remainder of B and the numerator of E combined)\nP is the number of particles\nT: All is the total number of entities\nMultiplayerChunkCache is the number of chunks currently loaded into memory\n\n\nx and z are your current coordinates along the horizontal plane\ny (feet pos) is the current coordinate along the vertical plane of the floor you're walking on \ny (eyes pos) is the current coordinate along the vertical plane of your eye level (Steve's eyes being 1.620 blocks above the ground)\nf is the orientation of Steve: 0 is west, 1 is north, 2 is east, 3 is south\nlc is the height level of the highest non-air block within the current chunk\nb is the name of the biome you're currently in\nbl is the light level of the block your head is in, from block-based lights like torches\nsl is the level of sunlight/moonlight on the block your head is in (independent of the current time of day)\nrl is the maximum of bl and sl\nws is your walk speed\nfs is your flying speed\ng is whether your feet are on the ground (true if you are, false otherwise)\n\nGraphs\nThe bar graph charts frame rate over time inversely (e.g., higher spikes = lower frame rate). Green plots are when your frame rate is 60 FPS or higher.\nThe pie graph charts the distribution of CPU time across different parts of Minecraft:\n\n[0] root (white) is essentially a sanity check: should always be 100%\n[? probably 1]unspecified (green) is the time spent on things not already enumerated\n[2] render (blue) is the time spent on rendering the world \n[3] tick (purple) is the time spent on the game clock (heartbeat)\n[4] sound (yellow) is the time spent on the sound engine\n[5] updatelights (salmon) is the time spent on updating the lighting engine (usually at sunset and sunrise)\n\nPressing the 1-9 keys will show sub-divisions of the numbered slices of the pie chart (in addition to switching items as usual), and pressing 0 will go back up.\nNotes\n\nA chunk, if you're not familiar with the term, is a section of the world, equal to 16x128x16 blocks.\nGiven the context in which it appears in RenderGlobal.java, I believe \"renderers\" refers to a single block.\nUnder the Minecraft coordinate system, X and Z are the horizontal axes and Y is the vertical axis, which confuses players used to other coordinate systems.\n\n", "topic": "gaming", "url": "https://gaming.stackexchange.com/questions/14990"}, {"image": "VQAonline_00019509.png", "question": "Using PON as main substrate for fiddle leaf figs", "context": "Is Lechuza pon a suitable substrate for fiddle leaf figs ?\nI just received a fiddle leaf fig tree which is 3 plants in a single 36 cm nursery pot. I bought a self watering 43 cm pot and I am planning to repot.\n\n", "answer": "I have seen many different members of the Ficus family including the fiddle leaf fig grown in similar substrates. The problems that could occur are:\n\nthese plants get quite tall. In the wild they grow up to 45 feet tall so indoors they need to be cut back frequently or they will be up to the ceiling. So much top weight makes them harder to keep balanced\nall the hydroponic solutions for plants work better if the solution can be changed. The old style indoor hydroponics using baked clay were in pots that did not drain. The solution was never changed and became saturated with dissolved salts in low light situations\n\nIn your question I don't see any reason why you want to go with a hydroponic solution. It's extra work and cost with the same result as these plants grow quite well in a soil less potting mix\n", "topic": "gardening", "url": "https://gardening.stackexchange.com/questions/57027"}, {"image": "VQAonline_00019218.png", "question": "What's this plant with fragrant leaves", "context": "This plant has very thin and fragile stems. It has very good smelling leaves. No flowers. Is there any way i can propagate it?\n \n", "answer": "I think it's Helichrysum petiolare, common name liquorice plant; note that it is not edible, despite the common name. In the UK, this is a seasonal, frost tender plant used for decorative purposes in pots and tubs with summer bedding, but in other, warmer parts of the world, it is a short lived perennial or sub shrub. It usually flowers in its second year and may seed itself, otherwise, you can take cuttings, 4-6 inches long, with soft growth at the tip and woodier stems at the base - full instructions here https://living.thebump.com/propagation-helichrysum-petiolare-6685.html\nFor an image and general information on this plant, see here https://kumbulanursery.co.za/plants/helichrysum-petiolare\n", "topic": "gardening", "url": "https://gardening.stackexchange.com/questions/42966"}, {"image": "VQAonline_00018782.png", "question": "Can it be too late to aerate the lawn?", "context": "So, it's mid october, the plug-pulling lawn Aerator that I ordered in early august got here today. In the past two weeks, as is pretty normal in my neck of the woods, the weather has looked like this, and is quickly getting colder.\n\nI'm thinking about still aerating my lawn, but don't want to expose roots to winter temperatures. Should I hold off until the spring, or can you aerate pretty much all year long?\n", "answer": "Fall is the best time to aerate. I wouldn't do it when temperatures at night are regularly at or below freezing but your temperatures look balmy, now. I am assuming you have cool season grasses since you are looking forward to a winter and freezing, so go ahead and aerate. Leave those plugs right where they fall. They will disintegrate adding soil and bacteria to the surface that will greatly help with reducing thatch (if any). Core aeration reduces compaction and also gets more air into the lawn's bed to boost your micro and macro flora and fauna health.\nBy the temperatures you have shown, your fall is looking just fine for aerating. You are right to worry about subjecting the roots to freezing temperatures. Believe it or not, fall is the best time for aeration. Is this aerator rented or did you purchase your very own? (Make some money by helping your neighbors, aerating is one of the easiest lawn responsibilities). \nBe sure you do not cut your grass any shorter than 3\" at any time of the year. This length is critical to maintain always! The reason is, our cool season grass species that make up our lawns have huge root systems genetically. If you've been watering correctly (watering deeply and allowing to dry out before watering deeply again) those roots will be 4-6\" deep. Necessary for making your lawn drought resistant and eliminating any weeds that might have germinated. In between watering the weeds won't be able to grow as the surface is dry and your grasses are enjoying moist soils down deeper. Also, the height of your grass at 3\" (not 2 1/2\" or 2 3/4\" but 3\") keeps the surface soils shaded further inhibiting grow of weed seeds and reducing evaporation.\nThe fall is also an important time to do the last fertilizing. The formulation is different than earlier in the season. Do not use a fast acting, high nitrogen fertilizer. You don't want fresh, new, more vulnerable grass photosynthetic matter going into the winter. This will set you up for major fungal disease problems in the sping.\nI like the 'organic' (I am not a fan of the oft used words 'organic', 'natural'...chemicals are chemicals) brands that are extended release, slower to take effect, and come with bacteria for thatch decomposition and mycorrhizae (for symbiotic support of the roots). Make sure you look for the formulation for FALL. It will be lower in Nitrogen or at least equal in percentage to the Phosphorus and Potassium: For instance 7-8-9 or 10-10-10. Check out Dr. Earth...that is the one I am familiar with and used for client's lawns as well. Worth ever cent. You'll find this type of fertilizer is more expensive but by using this expensive fertilizer you only need to do 2 or 3 applications not the normal 4 applications. Follow the directions closely. It takes more fertilizer per application so it all evens out. I'd use this stuff if it were 3X more expensive!! Worth having the prettiest, healthiest crop of lawn you've ever seen!\n", "topic": "gardening", "url": "https://gardening.stackexchange.com/questions/29571"}, {"image": "VQAonline_00018340.png", "question": "How should I prune my Cussonia Bonsai?", "context": "I have had this Cussonia for couple of years. As you can see on the picture, the branches grew somewhat unevenly (I marked the branches with a blue line). I am thinking how to trim it, so the bonsai won't become unbalanced and overgrow the space I have for it.\nI was told by the store that I should trim it from time to time. However I am not sure how to do it. The main branch of the longest branch already developed a thick layer so no new branches can come out of it. I'm worried cutting it in the middle will cause this long branch to stop growing at all.\n\n", "answer": "Cut that tall branch about halfway up your blue line. I am sure there is a healthy leaf on the outside of that branch (you don't want branches that angle towards the center of the plant). Use a pair of by-pass (not anvil-type) hand pruners, sterilize them with alcohol and cut 1/8\" above that leaf at an angle so water runs off away from the bud/leaf you are choosing as your new terminal bud. This is a bonsai, it is important to keep roots and plant in proportion. \nLater, there might be an even better leaf to choose that is a little lower. Don't go crazy as this cut is at least a 1/3 of the plant and it might get stressed a bit. Pruning on Bonsais must be done a little at a time.\nMost of the energy in that branch is at tip. You are cutting off that energy expenditure and all that energy will be diverted to the rest of that branch to 'lateral buds' you can't see right now. This extra energy will tell that branch to wake up more lateral buds...they are there. Careful and educated pruning will make your plant healthier. Take a bonsai class! They depend on you for everything. One mistake and it'll die. Bonsai are the furthest thing from 'natural'...learning to care for a bonsai will teach you an awful lot and humble you. Plants are amazing lifeforms! Very different than animals!!\n", "topic": "gardening", "url": "https://gardening.stackexchange.com/questions/9701"}, {"image": "VQAonline_00018758.png", "question": "Help growing a pine seedling", "context": "Earlier on in the year (February I believe) I found a pine cone on the path. On the off chance that it might have a viable seed in it I took it home. I recovered just one seed and planted it in a small 4\" pot.\nIt germinated and started to grow. This picture was taken on the 1st June.\n\nNow (28th August) it is still the same size. It doesn't seem to be doing anything, although it appears to be alive.\nIs this normal for pines to only grow such a small amount in the first year? Or is there something I can do to encourage more growth?\nIt's placed in an area where it gets afternoon direct light and indirect shaded light in the morning. I'm in the south of England.\n", "answer": "no, nothing you can do to hurry growth along - the only observation I'd make is the pot its in appears to be way too big for it - its always a good idea to pot things into a pot about twice the size of the rootball, and then pot up into something larger as it grows. If your seedling had a long root, then a narrower, taller pot would have been best. Not sure its worth decanting it after this much time and disturbing its roots, but the risk, with so much compost surrounding such a small amount of root, is that it becomes sour and somewhat toxic because, in order to keep the seedling supplied with water, the whole pot needs to be wetted thoroughly, and that's a lot of potting medium sitting damp with nothing growing in it. \nI can't tell from the picture whether you have a tray beneath the pot which catches water - if you do, empty that out 30 minutes after watering so the pot isn't sitting in water.\n", "topic": "gardening", "url": "https://gardening.stackexchange.com/questions/28979"}, {"image": "VQAonline_00018955.png", "question": "What is this black and red bug?", "context": "I have these bugs in and around my yard:\n\nThis is in Los Angeles. Are these harmful to plants, pets, or humans? I've found them on some of our plants (Salvia Lantana, Naga Peppers, etc.) I've even found a couple \"gettin' busy\" so they're pretty active.\n", "answer": "Meet the Small Milkweed Bug (Lygaeus kalmii). \nThis little fellow is mostly harmless, unless you are a milkweed breeder. Its favorite food are milkweed seeds, but it won't be too picky about other seeds or sources of protein. In a balanced garden, they allow you to have milkweed for the butterflies without being overrun by it as weeds. (They will eat the occasional butterfly egg though, but also other insects.)\nThey are completely harmless for humans and only if you feel they are a nuisance or you can't tolerate the minimal feeding damage some intervention may be warranted - manually removing large clusters should suffice.\nMore links: 1, 2, Wikipedia (available only in German, not English)\n", "topic": "gardening", "url": "https://gardening.stackexchange.com/questions/35341"}, {"image": "VQAonline_00018741.png", "question": "Why would one plant two olive trees right next to each other?", "context": "I just bought \"an\" olive tree and, to my surprise, it had two trunks that obviously belonged to two different plants planted right next to each other.\nLooking around the shop, this appears to be their default MO with olive trees.\nWhy would they do that?\nDo I have to worry about it harming the tree(s)?\nEDIT picture\n\n", "answer": "(Many) olive cultivars will not produce olives (very well) without a pollinator (an olive tree of a different cultivar)... so basically, you want/need two trees to produce (many) olives. (source 1)(source 2)(source 3)\n", "topic": "gardening", "url": "https://gardening.stackexchange.com/questions/27430"}, {"image": "VQAonline_00019680.png", "question": "Finding identity of Eliza Ann Wills who witnessed marriage of Grace Martin Wills and Matthew Nettell in 1865 at Redruth, Cornwall?", "context": "My 2nd great grandmother Grace Martin Wills married Matthew Nettell on 23 Sep 1865 at Redruth, Cornwall. Grace gave her age as 17 (I think it was just over 16 and half), and Matthew gave his as 19.\nOn their Marriage Certificate the two witnesses are clearly written (see below):\n\nGeorge Wills which I suspect would be her father (1806 - 1873) or brother (1841 - ?)\nEliza Ann Wills\n\n\nI would like to try and determine the identity of Eliza Ann Wills.\nShe does not appear to be a sister (and perhaps not a sister-in-law) to Grace because her siblings appear to have been:\n\nFrances (1836 \u2013 ?)\nMary Jane (1838 \u2013 ?)\nGeorge (1841 \u2013 ?) may have married Ann about 1865 (based on first child born 1866)\nRichard (1843 \u2013 ?) married Louisa Trevorah in 1863 and they appear to have been living in New Zealand at the time\nWilliam (1845 \u2013 ?) may not have married\nEliza Davey (1847 \u2013 1910) - one candidate but middle name does not match and appears to have been living in New Zealand at the time\nSarah Ellen (1851 \u2013 1868)\nSusan Mary (1853 \u2013 1875)\nEdith Ann (1857 \u2013 1894) - suggested by a cousin but was only 8 years old at the time\nCharles Henry Martin (1859 \u2013 1916) was only 6 years old at the time\n\nIf not a sister or sister-in-law then I am thinking that perhaps she was a cousin to Grace, or maybe her aunt or married to her uncle. So far I have found no likely candidate.\nThe last theory I have is that she is simply a friend who shares the same surname.\nBefore I try to exhaustively examine the Family and Neighbours (FAN) of Grace, I thought I would ask this question to see if anyone is aware of an Eliza Ann Wills living at the right time and right place to possibly have been the witness to this marriage?\n", "answer": "I think that I have found my answer via FindMyPast:\nIn Q3 1865 at Redruth:\n\nGEORGE WILLS married one of these people GRACE MARTIN WILLS, ELIZA ANN\n WHITBURN\n\nIt looks like Grace Martin Wills married Matthew Nettell soon after her brother George Wills married Eliza Ann Whitburn.\nI may need to order the Marriage Certificate (Volume 5C,Page 418) of George and Eliza Ann to be more certain but I think the possibility of a double wedding with George and Eliza Ann being married first should not be discounted. I'll see if I can find a newspaper report too.\nAs commented by @user3310902 this appears to be the baptism record of Eliza Ann:\n\n\"England and Wales Non-Conformist Record Indexes (RG4-8), 1588-1977,\"\n index, FamilySearch (https://familysearch.org/pal:/MM9.1.1/FWF5-5Q9 :\n accessed 25 February 2015), Eliza Ann Whitburn, 06 Dec 1836, Birth;\n citing p. 50, West Cornwall, record group RG6, Public Record Office,\n London.\n\n", "topic": "genealogy", "url": "https://genealogy.stackexchange.com/questions/7918"}, {"image": "VQAonline_00019760.png", "question": "What type of journey from Plymouth to Exeter in 1751 might cost 1 pound 5 shillings?", "context": "My 6th great-grandfather Robert Osment was baptised on 8 Mar 1703, married (to Elizabeth Simons) on 3 Jan 1729, and buried on 6 May 1756, with these life events all occurring at Plymouth Charles, Devon, England.\nIn the Devon, Plymouth Borough records 1519-1905 at FindMyPast there is a very clear image in the Borough Receivers' Accounts that says the amount of 1 pound 5 shillings was:\n\nPaid Robert Osment for a Journey to Exeter\n\n\nThe entry is undated but it is near the end of a long list of such entries for Charges and Disbursements from 29 Sep 1750 to 29 Sep 1751. FindMyPast dated the event as being in 1750 (see below) but I suspect that it was probably around Aug-Sep 1751.\nGiven the wording and amount paid, can anything be inferred about the type of journey (from Plymouth to Exeter) that may have been paid for and thus the likely occupation of Robert Osment?\nEvent type Borough Receivers' Accounts\nRecord set Devon, Plymouth Borough Records 1519-1905\nFirst name(s) Robert\nLast name Osmont\nEvent year 1750\nCounty Devon\nCountry England\nPlace Plymouth\nYears 1739-1784\nArchive Plymouth & West Devon Record Office\nArchive reference 1/134\nCategory Census, land & surveys\nSubcategory Courts & legal\nCollections from England, Great Britain\n\n", "answer": "In \"The Transport Revolution in Industrializing Britain: A Survey\" (pdf) by Dan Bogard at UC Irvine, a table on page 14 gives Stagecoach Fares in shillings per passenger mile, with a value of 0.23 in 1760, which is reasonably close to 1751.\nOn Google Maps, Plymouth to Exeter is around 45 miles by current roads, but that could be somewhat higher using 1750s \"roads\". If we say 50 miles travelled, that gives a cost of a return journey of 50 x 0.23 x 2 = 23 shillings, or one pound three shillings. That's very close to the quoted 25 shillings cost, so the cost is consistent with a stagecoach return journey if no other substantial costs were involved.\nAt an average of 2.6 miles per hour, each way's travel would have taken around 20 hours, so would have been split over at least two days.\nThis doesn't exclude other transport options of similar or lower cost, of course. Robert could have gone alone on horseback (probably cheaper, but perhaps with added danger from highwaymen!), or even gone by sea, although that's not a straight route.\nAnd it doesn't help with why he went. He may have been a passenger taking something to Exeter, or meeting someone there, or bringing something back. Or he may have been the carrier/driver, who charged the borough for taking someone to Exeter. (Purely speculatively, Plymouth is within the diocese of Exeter, so taking someone/something to the Bishop of Exeter could have been a valid reason for such a chargeable trip.)\nIs there anything on Robert's burial record or the baptism records of his children that refers to his occupation?\n", "topic": "genealogy", "url": "https://genealogy.stackexchange.com/questions/13652"}, {"image": "VQAonline_00019720.png", "question": "What does \"ads\" stand for in a Chancery Court record index?", "context": "I found the following index page at Family Search:\n\nI would like to know what \"ads\" stands for in the unlabeled column and what it implies. Does it stand for \"administrator\"and imply that that person to the right administered the estate of the person to the left?\n", "answer": "These are chancery court cases.\nAds is an abbreviation for ad sectam, Latin for \"at the suit of\". The alternative is vs for versus.\nThe difference is that for papers associated with the defendant entering the plea, the names are written [defendant] ads [plaintiff]. For papers associated with the plaintiff filing charges, the names are written [plaintiff] vs [defendant].\nFor example, the first file on the snippet given, Shannon ads. Newman, Joseph Shannon is the defendant at the suit of plaintiff Alex Newman. \n", "topic": "genealogy", "url": "https://genealogy.stackexchange.com/questions/10597"}, {"image": "VQAonline_00019791.png", "question": "Finding place named St Lagille, France in 1914?", "context": "A 1914 USA immigration form says Marie Paudois was born in \"St. Lagille, France\" but a web search only turns up \"Rue St. Lagille\" in various French cities. Can't find a village or department by that name.\u00a0 My suspicion from other documents is that it's in Bretagne or another western department.\nIs there an online gazetteer that is likely to help? We have almost two weeks unplanned and we're already in northern Spain, so we'd like to go through there.\n\n", "answer": "Based on sempaiscuba's answer, Julien PAUDOIS is most likely:\nJulien Joseph Emile PAUDOIS, born in Saint-Mars-la-Jaille (probably what was later turned into \"Saint Lagille\", as it sounds similar) on the 6th of May 1875, son of Julien PAUDOIS and Jos\u00e9phine BOSS\u00c9.\nSee https://www.archinoe.fr/v2/ark:/42067/c7024a4c2466721865b2b84824e58edc\n", "topic": "genealogy", "url": "https://genealogy.stackexchange.com/questions/15402"}, {"image": "VQAonline_00019868.png", "question": "Reading old German script about Henry Reimer", "context": "I found a DNA match to someone who has an ancestor Henry Reimer born about 1839, probably my 2G Grandparent Diderich Reimer's brother. They both immigrated to Cleveland, Ohio (separately).\nThese scripts are hard for me to read. I am using pattern recognition more than reading. I believe that I have a parent match, but I can't read the child's name. I am hoping that one of the forenames is Heinrich.\nFrom Archion.de. Top record is my 2G Grandfather Diderich Reimer. Bottom record may be his brother Henry.\n\n", "answer": "Here are the names as I read them, and I think you have a match:\nRecord from 1833\n\nJohannes Joachim Friedrich Reimer, Weberknecht zu Beselin\n\n\nSophia Christina Margarethe geb. Schr\u00f6der\n\n\nDiderich Carl Christoph Johann Joachim Theodor\n\nRecord from 1840\n\nJohann Joachim Friedrich Reimer, Weber zu Fresendorf\n\n\nSophia Christina Margarethe geb Schr\u00f6der\n\n\nChristopher Heinrich Friedrich\n\n\"Weber\" and \"Weberknecht\" both mean weaver. Fresendorf is quite close to Beselin (about 3km across the river K\u00f6sterbeck according to Google).\n", "topic": "genealogy", "url": "https://genealogy.stackexchange.com/questions/18904"}, {"image": "VQAonline_00019821.png", "question": "Deciphering names from Spanish death certificate", "context": "This is a follow on from:\n\nDetermining cause of death from Spanish death certificate\nDeciphering Work Profession from Spanish death certificate\n\nThis is the first part of my Great Great Grandmothers death certificate:\n\nI have already discussed this certificate with my mother (who is Spanish) and we have come up with the following translation so far:\n\nIn the city of Corella, province of Navarra, at 12 noon on 28th April\n1944, before D. Santiago [Frances Segura], Municipal Judge and D.\nPablo [????? ?????], Secretary, proceed to register the death of Da.\nJuana Sanz Cueva, born in Corella, province of Navarra, on 6th May\n1857 (86 years old), daughter of D. Blas Sanz and Da. Felipa Cueva,\nresided at the street, of [????? ?????], number 40, of profession\n[????? ?????] and state [?????] of D. Ciriaco Bienzobas [????? ?????\n????? ?????] children [?????] Leon, Villar, Maria [?????] and Felipa.\n\nThis was the best we could come up with. We are struggling with some of the names. Ideally I would like to decipher and complete the whole segment but in particular I am interested in the names of Great Great aunts and uncles. I know for sure:\n\nMaria\nVillar\nLeon\nFelipa\n\nBut there are other words and / or names in that segment that I am not sure about.\nIf anyone knows Spanish and is good at this kind of thing then I welcome your assistance. I included the whole snippet from the certificate so you could see more handwriting.\nUpdate\nBased on the comments I have adjusted the information like this so far:\n\nIn the city of Corella, province of Navarra, at 12 noon on 28th April\n1944, before D. Santiago [Frances Segura], Municipal Judge and D.\nPablo [????? ?????], Secretary, proceed to register the death of Da.\nJuana Sanz Cueva, born in Corella, province of Navarra, on 6th May\n1857 (86 years old), daughter of D. Blas Sanz and Da. Felipa Cueva,\nresided at Tajadas Street, number 40, of profession [????? ?????] and\nwidow of D. Ciriaco Bienzobas of which marriage are left five\nchildren [?????], Leon, Villar, Maria [?????] and Felipa.\n\n", "answer": "If I count correctly, there are five missing blocks:\n\nIn the city of Corella, province of Navarra, at 12 noon on 28th April 1944, before D. Santiago [Frances Segura], Municipal Judge and D. Pablo [????? ?????], Secretary, proceed to register the death of Da. Juana Sanz Cueva, born in Corella, province of Navarra, on 6th May 1857 (86 years old), daughter of D. Blas Sanz and Da. Felipa Cueva, resided at Tajadas Street, number 40, of profession [????? ?????] and widow of D. Ciriaco Bienzobas of which marriage are left five children [?????], Leon, Villar, Maria [?????] and Felipa.\n\nI would go for:\n\nD. Pablo Salvatierra \u00bfLasa?\nprofesi\u00f3n sus labores (housewife)\nare left five children llamados (called)\nMar\u00eda Mauricia\n\n", "topic": "genealogy", "url": "https://genealogy.stackexchange.com/questions/16163"}, {"image": "VQAonline_00019663.png", "question": "Does the name on this record say Nikiefor?", "context": "I found a strange name on a baptismal record from Bia\u0142y Kamie\u0144 which is now part of Ukraine. The records are mostly in Polish and Latin.\n\nIt looks like \"Nikiefor\" but that's a very strange name. I've never seen that name before and I've viewed decades worth of records for the town. Plus there are only 169 results for it on Google.\nExtra points if you can tell me what the origin of the name is.\n", "answer": "Your reading of this given name as \"Nikiefor\" seems to be reasonable. Behind the Name lists Nikifor (without an \"e\") as the Russian, Bulgarian, Macedonian and Polish form of the Greek name Nikephoros.\nNote 1: the full image of the register page, which would show additional examples of writing, is only viewable at a Family History Center or to LDS church members. See the collection description Ukraine, Western Ukraine Catholic Church Book Duplicates .\nNote 2: many of the Google hits using \"Nikiefor\" spelling are error pages of various kinds. \"Nikifor\" has more than 3 million hits.\n", "topic": "genealogy", "url": "https://genealogy.stackexchange.com/questions/3935"}, {"image": "VQAonline_00019970.png", "question": "Umlaut over consonant preceding a vowel", "context": "In the photo below, the word \"v\u00f6lliger\" is spelled with the umlaut over the v:\n\nIs this a typo, a design choice, or is there a linguistic explanation?\n", "answer": "This simply is wrong.\nIt should be:\n\nBEI V\u00d6LLIGER DUNKELHEIT ENTWICKELN\n\nThere is a letter \u00bbV\u0308\u00ab, but it is not part of the German language. (Some Oceanic languages use this letter, for example Araki)\n", "topic": "german", "url": "https://german.stackexchange.com/questions/37730"}, {"image": "VQAonline_00019953.png", "question": "What is the name of this type of chair that can be found on beaches?", "context": "I'm looking for a German translation of this type of chair, that can be found on the northern French beaches. We call them \"Chaises \u00e0 capuchon\", where chaise is \"chair\" and capuchon would be \"hood\" or \"cap\". \nI couldn't find that word used in German on the Internet, an approximate realistic translation would be highly appreciated.\n\n", "answer": "\nder Strandkorb\n\nThe literal translation is beach basket \u2014 which should be pretty obvious if you look at it, and where it is usually found. They are extremely common on beaches on the North Sea in Germany, where weather and wind are usually even worse than on northern French beaches.\n", "topic": "german", "url": "https://german.stackexchange.com/questions/30998"}, {"image": "VQAonline_00019966.png", "question": "What is the German word of this small boat that you usually rent for two people?", "context": "What is the German word of this small boat that you usually rent for two people?\n\n", "answer": "Even if swan-shaped, I can't imagine of any other word than\n\nTretboot\n\nWhile this would also apply to other small and somewhat bigger boats (I recall having seen ones with 4 seats and possibly a small area to lay down after taking a swim), any construction like Schwanentretboot would be understood, but is unlikely to be used broadly.\n", "topic": "german", "url": "https://german.stackexchange.com/questions/36294"}, {"image": "VQAonline_00019957.png", "question": "Using the verb \"erscheinen\" just like in english", "context": "I would like to know,could I use \"erscheinen\" for \"appear\" like its english equivalent.\nContext:\n\nAnd my translation attempt:\nIn dem Satz '...', welche der folgenden erscheint nicht?\n", "answer": "Though erscheinen is not really wrong here (i.e. you will be understood, and probably there are folks who would not even take notice), it seems more idiomatic to me to use auftreten. I would also recommend to not follow the original sentence structure 1:1, but rather rephrase it like e.g.\n\nWas tritt in dem Satz \"...\" nicht auf?\n Welche Wortart tritt in \"...\" nicht auf?\n Untersuchen Sie den Satz \"...\". Welche der folgenden Wortarten tritt darin nicht auf?\n\nTwo other alternatives would be vorkommen (thanks to PerlDuck for suggesting it) and vertreten sein, so e.g. \n\n... Welche der folgenden Wortarten kommt darin nicht vor?\n ... Welche der folgenden Wortarten ist darin nicht vertreten?\n\nThe verb erscheinen (in the meaning that does matter here) always comes with a sense of dynamic or change (\"it wasn't there before, but now it is\"), but here we are talking about something completely static.\n", "topic": "german", "url": "https://german.stackexchange.com/questions/32699"}, {"image": "VQAonline_00020018.png", "question": "Verb position with modal verbs", "context": "I have this sentence that I think is wrong:\n\nMeine Mitbewohner sagen, ich soll vorsichtig sein mit der Gasheizung.\n\nIsn't the verb \"sein\" supposed to come at the end of the sentence like this?\n\nMeine Mitbewohner sagen, ich soll mit der Gasheizung vorsichtig sein.\n\nHere is the page from the book Learn German with short stories: Caf\u00e9 in Berlin by Andre Klein\n\n", "answer": "While it is true that that a modal verb plus infinitive form a sentence bracket, and the infinitive usually is at the end of a sentence, it is possible for parts of the sentence to appear after the sentence bracket. This is called ausklammern.\nThis usually happens with parts of the sentence introduced by a preposition, like in your example.\nThis page discusses this phenomenon and lists other examples.\n", "topic": "german", "url": "https://german.stackexchange.com/questions/55787"}, {"image": "VQAonline_00020026.png", "question": "German word for a specific situation in an image", "context": "\nI know the word Pechstr\u00e4hne (streak of bad luck) or maybe even Pechvogel (unlucky person) would fit the case, or even Schwein gehabt! if he survives (or Schadenfreude for what some observers may feel)\nThen one German speaker told me the following word: Clownenschadenmitbananaundsafeleiderkaputt.\nIs this a valid word that can be used?\n", "answer": "German compound nouns are often made fun of because of their length. To nonspeakers, they look monstrous and incomprehensible. But in reality, they are quite simple.\nThere's always at least two parts, with the first part determining the second and the second part being a noun.\n\nVerkehrsunfall: accident in traffic\n Nasenspray: spray for nose\n Klimaleugner: denier of climate (i.e. climate change)\n\nThen there's recursion: A compound may be used as the first part of another compound.\n\nVerkehrsunfallstatistik: statistic about accidents in traffic\n Nasenspraysucht: addiction to spray for nose\n Klimaleugnerthese: proposition by denier of climate\n\nThese compounds may be well-established, such as Verkehrsunfallstatistik, they may be occasional, such as Nasenspraysucht, or they can be created ad hoc, such as Klimaleugnerthese.\nAs the literal translations indicate, these compounds are quite easily understood if read from right to left. However, when reading quickly, proper segmentation (at least for long or rare compounds) is sometimes a problem even for native speakers, which is why spellings with a hyphen, such as Nasenspray-Sucht, are becoming more popular.\nThe problem with\n\nclownenschadenmitbananaundsafeleiderkaputt\n\nis that it does not follow the rules for compound nouns: There's a prepositional phrase (mit Banane), a conjunction (und), an adverb (leider) and an adjective (kaputt) all mushed together, without any nouns that could serve as second part.\n", "topic": "german", "url": "https://german.stackexchange.com/questions/56604"}, {"image": "VQAonline_00019871.png", "question": "Hatten das runde \u201es\u201c und das lange \u201e\u017f\u201c die gleiche Aussprache?", "context": "Gab es einen Unterschied in der Aussprache von rundem \u201es\u201c und langem \u201e\u017f\u201c, oder war der Unterschied nur typographisch?\n\n", "answer": "Dadurch, dass das runde s nur am Auslaut vorkommt, gilt nat\u00fcrlich die Auslautverh\u00e4rtung, weswegen man es gew\u00f6hnlich nicht als\u00a0/z/ sprechen konnte, sondern immer nur /s/. Genauso wenig ist (standardsprachlich) die Aussprache als\u00a0/\u0283/ m\u00f6glich, da Kombinationen /\u0283t/ und\u00a0/\u0283p/ nicht \u00fcber Wortgrenzen (u.\u202f\u00c4.) gesprochen werden.\nF\u00fcr das lange s l\u00e4sst sich eine derartige Einschr\u00e4nkung nicht treffen, da es sowohl im An- als auch im Auslaut auftreten konnte. Es kann also als /z/\u00a0(\u017facht), /s/\u00a0(lu\u017ftig) oder /\u0283/\u00a0(\u017fteil) ausgesprochen werden. Ebenso kann das lange\u00a0s Bestandteil eines Digraphen sein, z.\u202fB. \u017fch (ausgesprochen als\u00a0/\u0283/, z.\u202fB. in ra\u017fch) oder \u017f\u017f (ausgesprochen als \\s\\, z.\u202fB. in la\u017f\u017fen).\nDas runde s wird also im Grunde so ausgesprochen wie ss und\u00a0\u00df.\nGenerell gilt, dass das lange\u00a0s nicht anders ausgesprochen wird als ein rundes\u00a0s an derselben Stelle, \u017fy\u017ftemati\u017fch spricht sich also genau so wie systematisch.\n", "topic": "german", "url": "https://german.stackexchange.com/questions/1397"}, {"image": "VQAonline_00021266.png", "question": "Link coreldraw characteristics", "context": "Is there any option to \"link\" paragraph content and characteristics something similar to object style\nposted an example where i have to copy some contents from left to the empty right side\n\n", "answer": "Probably you could use Edit > Clone if you just want to duplicate the text from one side to the other.\nOr if you want to import data from a text cvs text file use the File > Print Merge comand.\n", "topic": "graphicdesign", "url": "https://graphicdesign.stackexchange.com/questions/137594"}, {"image": "VQAonline_00020784.png", "question": "What is this image effect (similar to viewing a 3d film without the glasses) called?", "context": "If you look at this site, the designer(s) made use of a an effect similar to what a 3d film would look like without the 3d glasses (if your remember those days). How would I replicate this effect in Photoshop for a photo?\nImage reference:\n\n", "answer": "The 3D effect is called anaglyph 3D. Though in this case ist is just simply a color channel offset. So it might also be simulating a mis-registration (see this post).\nWhat you do is you duplicate the artwork run a red filter on the other (multipy by red) offset that and then blue (actually for full color cyan) filter the other copy. If you want a real effect you will need 2 images.\n", "topic": "graphicdesign", "url": "https://graphicdesign.stackexchange.com/questions/61991"}, {"image": "VQAonline_00021026.png", "question": "User Input Color Restrictions", "context": "What is an appropriate restriction to place on a color gamut for user defined colors?\nI am helping build a website that will offer organizations to create their own pages. We want to allow them the option to customize the background color, as well as the 'highlight' color on their page.\nThe 'highlight' color defines the color of all icons and buttons on their page. Because of this, we cannot allow the highlight color to be too light/faint/bright.\nWe offer the following UI to choose a color:\n\nadaptation of https://bgrins.github.io/spectrum/#details-ieImplementation\nYou can see the selected color is reflected in the 'choose' button, and would also be applied to other buttons in the UI.\nMy potentially naive plan was to restrict the color spectrum in two different ways:\n\nEvaluate the selected color in RGB - check if the combined value of R, G, and B is less than 255 (on a 0 to 255 scale)\nEvaluate the selected color in HSL - extract the L value (luminance or lightness) and check if the value is less than 0.5\n\nWhen a user selects a color that is not in the proper range an error message appears to notify that they need to select a darker color.\nThis implementation seems to work pretty well. All bright colors get caught by the above restrictions with a few exceptions (full green appears brighter to my eyes than full red for example... not sure why)\nMy question is if there is a standard restriction that is put in place on similar selections. Is there a better way I should be evaluating and restricting the selected colors?\n", "answer": "I'd stay away from showing all color options in a color picker like this for a couple reasons: 1) it's frustrating for a user to see (or even choose) options that they can't end up having. And more importantly 2) most people would be confused by this setup, at least initially. If a user is using your product in the first place, chances are that they would prefer something simpler but less powerful.\nWhat I'd recommend instead is either give them a choice from pre-made colors, or give them a choice of pre-made pallettes (color combinations). You could also optionally allow customisability later or more hidden for more \"power users\". I do things like this in my applications all the time. By doing this, you can have more full control while not frustrating users (as often).\n", "topic": "graphicdesign", "url": "https://graphicdesign.stackexchange.com/questions/96610"}, {"image": "VQAonline_00020574.png", "question": "Google map icon- is it free to use?", "context": "Can I use the google map icon on my website, if I link to google maps?\n\n", "answer": "Per Google after a little search:\n\nThings not permitted:\nDon't copy or imitate Google's trade dress, including the look and\n feel of Google web design properties or Google brand packaging,\n distinctive color combinations, typography, graphic designs, product\n icons, or imagery associated with Google.\n\nSource\n", "topic": "graphicdesign", "url": "https://graphicdesign.stackexchange.com/questions/35962"}, {"image": "VQAonline_00020321.png", "question": "How to scan charcoal and graphite drawings?", "context": "I was trying to scan some of my sketches in order to post them on some community sites, but the effects are quite bad. An example could be seen here:\n\nBasically, the contrast is terribly off. All soft shading done with a hard pencil is gone, the white is too bright and too \"aggressive\" and bleeding all over. All the shading had been \"flattened down\" in the process of scanning.\nSo, my question is, are there some techniques or procedures I should follow, some typical settings for the scanner I should use in order to get good quality scans without the brightness/contrast distorion?\n", "answer": "(upgraded comment to answer)\nScott, brendan and tim human all provide good advice regarding scanners.\nI actually do a lot of work with paintings and drawings, and photographing them is almost always a better option than scanning. I have a low-end professional flatbed scanner on my desk, but I almost never use it anymore because I get better results in less time using a DSLR (even for the rare times I need to scan office documents). Then again, I have a dedicated station, with alignment marks on the floor and table, so the photo equipment is pretty much set up and ready to go.\nI use 2 photo lights, polarizing filters to reduce glare, and a Digital SLR using RAW format. I use a low ISO setting to minimize noise, which usually requires longer exposures and therefore a tripod. Additionally, I set the aperture small to increase the depth of field as a way to compensate for the auto-focus and ensure that larger items are in focus across the entire surface (it is not always easy to get an old painting perfectly parallel). Usually this means 15-25 second exposures.\nThe polarizers are really only needed for items with glass or high reflectance--drawings are probably not going to exhibit glare. They do alter the color and/or saturation and they can mess with auto focus settings in some cameras.\nIf you only have a consumer pocket camera, consider a tripod at least. The key is decent consistent lighting across the composition, and as \"straight-on\" a shot as you can get. You want the plane of the camera's CCD to be parallel to the plane of the drawing to avoid having to distort or fix the perspective. Not so bad for a one-off, but if you need to fix 30 at a time, you are better off taking the time working on your set up first.\nFor consumer cameras, and non-RAW format, if you don't like the results, check to see if there is a \"custom white balance\" setting in the camera menus: you take a shot of a white piece of paper in the light setting you are using, and then set the custom white balance to that photo. This will help reduce the color cast of any lights you are using (regular lights usually cast yellow-red, fluorescents usually cast blue)\n", "topic": "graphicdesign", "url": "https://graphicdesign.stackexchange.com/questions/16294"}, {"image": "VQAonline_00020373.png", "question": "Combining angled shapes in Photoshop CS6 causes antialiasing on all", "context": "When I combine two shapes where at least one has a non-right angle or curve, both shapes become anti-aliased. This appears to be some new to CS6. Are there any ways to get around this? The photo below shows two sets of combined shapes, one with a 45 degree angle which causes the blurring and one with right angles which does not blur.!\nExample of Blurring\n\n", "answer": "Photoshop will always anti-alias shapes, which in most situations is what you want. When designing icons and other small-scale items, quite often that's not what you want.\nOne way to avoid the problem is to change your workflow slightly. Create and combine paths, rather than shape layers, then stroke and fill the paths. \n\nFor strokes, use the Pencil tool and \"Stroke Path\" from the Paths flyout menu. \nFor fills, use \"Fill Path\" and turn off Anti-Alias.\n\n", "topic": "graphicdesign", "url": "https://graphicdesign.stackexchange.com/questions/19107"}, {"image": "VQAonline_00021281.png", "question": "Which slash is correct for \u2018Old Money\u2019", "context": "Introduction\nUnicode defines multiple strokes; of interest to this question is U+002f (solidus: \u2018/\u2019), U+2044 (fraction slash: \u2018\u2044\u2019) and U+2215 (division slash: \u2018\u2215\u2019). I cannot\u2014however\u2014find these well defined. In Unicode 12.1.0 the strokes are defined as such:\n\nFraction Slash. U+2044 fraction slash is used between digits to form numeric fractions,\nsuch as 2/3 and 3/9. The standard form of a fraction built using the fraction slash is defined\nas follows: any sequence of one or more decimal digits (General Category = Nd), followed\nby the fraction slash, followed by any sequence of one or more decimal digits. Such a fraction should be displayed as a unit, such as \u00be or . The precise choice of display can depend on additional formatting information.\nIf the displaying software is incapable of mapping the fraction to a unit, then it can also be\ndisplayed as a simple linear sequence as a fallback (for example, 3/4). If the fraction is to be\nseparated from a previous number, then a space can be used, choosing the appropriate\nwidth (normal, thin, zero width, and so on). For example, 1 + thin space + 3 + fraction slash + 4 is displayed as 1\u00be.\n\nOther notes on the stroke include the following:\n\nSeveral punctuation marks, such as colon, middle dot and solidus closely resemble mathematical operators, [\u2026]\n\nAnd finally:\n\nTwo small form variants from CNS 11643/plane 1 were unified with other characters outside the ASCII block: 213116 was unified with U+00B7 middle dot, and 226116 was unified with U+2215 division slash.\n\nQuestion\nThis specific question relates to the historical usage of a stroke to denote shillings and pence, the so-called \u2018old money\u2019. Which stroke is the typographically correct stroke to be used for this? I am quite sure that I somewhere read that the regular stroke (U+2f) is wrong, that one should rather use the division slash (U+2215). In any case, the fraction slash (U+2044), as it is designed for fractions and thus triggers specific behaviours in word processors, web browsers and the like when frac or afrc is called for.\nTwo follow-up questions are in order: Should zero pence be written with an n-dash or hyphen? And which space should be used to separate pounds from shillings, if one uses a space rather than a stroke?\nAlternatives, all with thinspaces and n-dashes:\n\nU+002f (\u2018solidus\u2019):\n\n\u00a31\u20095/\u2013\n\u00a31/5/\u2013\n\n\nU+2215 (\u2018division slash\u2019):\n\n\u00a31\u20095\u2215\u2013\n\u00a31\u22155\u2215\u2013\n\n\n\nIt might well be that a hairspace would be desired for the final two examples.\n", "answer": "In predecimal currency notation in the UK, the symbol should be the solidus symbol, because the word \"solidus\" comes from the Latin name of a coin. Pounds shillings and pence were also denoted with \u00a3, s, and d, which came from the Roman silver coinage denominations librae, solidi (plural of solidus), and denarii (plural of denarius).\nThe solidus wasn't used as a divider. It actually stands for shillings/solidi\nSo you would never write \u00a31/5/- It would be written as \u00a31 5/-\nPennies were added after the solidus, for example \u00a31 5/9d, sometimes with or without the d.\nIt could also be written as \u00a31-5s-9d, sometimes with a centre dot as separators instead of a hyphen, or a long dash instead of a hyphen. Sometimes just a space separated pounds from shillings. For example \u00a31 5/-\nIt's also interesting to note that instead of \u00a31 5/- people would often convert it to shillings when writing it - for example 25/- or 25s.\nMy sources: An old person (82) who I just asked. Also if you look at some old newspaper adverts you will see how they actually did it. Sometimes the solidus was something more akin to a long apostrophe. There's an example here.\nAnother example here for a higher priced item priced in shillings. In this case 79/6, instead of \u00a33 19/6d.\nAnother interesting thing to note is that very high priced items such as a television set were often priced in guineas - abbreviated GNS. A guinea was \u00a31 1/-. So in this example 78GNS would be equivalent to \u00a378 and 78 shillings, or \u00a381 18/-\nAll I can say is thank God for decimalisation. Imagine having to work this all out in your head! 240 pennies to the pound, 12 pennies to the shilling, 20 shillings to the pound, 21 shillings to the guinea. There were also halfpennies to contend with, \u00bdd, and prior to 1961 farthings, \u00bcd. It was completely insane!\n", "topic": "graphicdesign", "url": "https://graphicdesign.stackexchange.com/questions/140097"}, {"image": "VQAonline_00021453.png", "question": "Should I add call signs with /R suffix to the logbook?", "context": "Sometimes in FT-8 mode I receive calls like this one:\n\nIt happens 2-3 times a week.\nThe problems are: 1) I never could look up such call signs 2) I tried to reply, but never got answer 3) I don't know what /R means.\nAre these real calls or maybe just a noise that WSJT-X somehow managed to decode? So far I didn't log such events as QSO's. Maybe I should?\nUPD: As Marcus pointed out, the checksum collision in FT-8 is very unlikely. Also I managed to figure out that \"/R\" means a repeater https://en.wikipedia.org/wiki/Amateur_radio_call_signs I believe it means that the signal was sent automatically by the device, not that the operators works through the repeater. Still I wonder - should I add repeaters to the logbook?\n", "answer": "FT8 decoding can use a technique called a priori (AP) whereby it uses naturally accumulating information for the purposes of increasing apparent sensitivity by about 4 dB. There is an increased chance of false decodes when AP is enabled since AP is essentially sophisticated guess work. The technique looks at its guessed result and compares it to the parity information and displays the guess if there is a specified level of confidence of it being correct. There is no assurance that it is correct. The term often used for these false AP decodes is exotica - referring to a rare, decoded call sign that doesn't exist.\nWhile it is entirely possible that the sender is a pirate station, the fact that the V0 prefix is not allocated by the ITU is a good indicator that you are experiencing exotica. The /R is likely part of the exotica decode - not an indication of a repeater. Another good indication of an exotica decode is a decode of an unlikely grid square.\nBecause the decoded call sign does not exist, when you respond, the other station does not recognize the exotica call as their call sign so no QSO takes place.\nIf you wish to avoid exotica, at the expense of some loss of apparent sensitivity, simply disable the AP feature under the decode menu.\n", "topic": "ham", "url": "https://ham.stackexchange.com/questions/12930"}, {"image": "VQAonline_00021383.png", "question": "Trap dipole tuning up on the wrong band", "context": "I'm building a trap dipole for the high HF bands, 10 / 12 / 15 / 17.\nSo far what I've got in the air is: \n\nThe 10 meter section (4.92 meters, 16.14 feet, resonant at 29Mhz) \nThe 10 meter traps (8pF cap in parallel with a 4\u00b5H inductor, resonant at \u224828Mhz) \nThe 12 meter section (80 cm, 31.5 inch)\n\n 8pF/4\u00b5H 8pF/4\u00b5H\n ----{10M}------------{Balun}------------{10M}----\n 0.4M 2.46M \u2551 2.46M 0.4M\n \u2551\n \u2551 30.4M coax\n\nNow take a look at the output from my antenna analyzer:\n \nWhhaaaat... Well, I'm happy with the 10 meter resonant point (given by M1)\nbut what the heck happened next? I read somewhere that traps would electrically shorten all subsequent dipole sections, but i'm not sure what to make of this. Why would the 12 meter section be so far out of tune. Any ideas? \n", "answer": "Traps require that the sections that follow them to be shortened because they look like loading coils. We can demonstrate this by calculating the impedance of your trap at 28MHz and 24.9MHz.\nThe impedance of a parallel LC circuit is given by:\n$$ Z(\\omega) = \n-j \\left( 1 \\over C \\right)\n\\left( \\omega \\over \\omega^2 - \\omega_0^2 \\right )$$\nWhere\n\n$\\omega = 2 \\pi f$\n$\\omega_0 = 1 / \\sqrt{LC}$\n\nSo for your trap:\n$$ \\omega_0 = {1 \\over \\sqrt{4\\:\\mathrm{\\mu H} \\cdot 8\\:\\mathrm{pF}}} = 176776695 $$\nThis is the resonant frequency in radians per second; if we divide by 2\u03c0 we see that it's 28.13MHz as expected. We can then calculate the impedance at the relevant frequencies:\n$$ Z(28\\:\\mathrm{MHz}\\cdot 2 \\pi) = j73.6\\:\\mathrm k\\Omega \\\\\nZ(24.9\\:\\mathrm{MHz}\\cdot 2 \\pi) = j2.89\\:\\mathrm k\\Omega $$\nIn the case of 28MHz, the trap is a high impedance, and this serves to reduce the current on the section past the trap to some negligible level.\nAt 24.9MHz the impedance is much lower, but it's not nothing. Additionally, it is no longer a quarter-wave down the antenna, but something less. Because this impedance is purely imaginary and positive, it's an inductive impedance. We can divide the reactance (the imaginary part of impedance) by \u03c9 to get an effective inductance:\n$$ {2.89\\:\\mathrm k\\Omega \\over 2 \\pi \\cdot 24.9\\:\\mathrm{MHz}} = 18.5\\:\\mathrm{\\mu H} $$\nThus, at 24.9MHz (and just that one frequency), the trap looks just like a 18.5\u03bcH inductor. Just like a loading coil.\nBecause the coil is not at the base of the antenna but closer to the tip, its effect on the feedpoint impedance somewhat diminished. I don't know the exact math, although you can find loading coil calculators around the net. Normally we use a loading coil when we don't have room for a full-sized antenna, but here we are reasoning in the opposite order: because you have a loading coil, your antenna can't be full-size.\n", "topic": "ham", "url": "https://ham.stackexchange.com/questions/3605"}, {"image": "VQAonline_00021476.png", "question": "Can these in-band spurs from rpitx be eliminated somehow?", "context": "I'm putting together a 2m NBFM transmitter/receiver combo using rpitx and rtl-sdr. During testing my low-pass filter I discovered that rpitx is generating spurious signals within and around the band (see screenshot below). They're only about -30dBc so I need to remove them before transmitting. Removing them with a band-pass filter would be difficult and restrict the frequency range. What could be causing these spurs, and can anything be done to clean up the signal?\nI've overlayed two screenshots, one with no signal (green), and one with the rpitx signal at 146MHz (red), taken from GQRX+RTLSDR which show some of the spurs.\n\nThe spurs are frequency-dependent and generally span +/-5MHz or so. I've ruled out the rtl-sdr as the problem as I can see them on my scope too. The signal is generated with rpitx/tune -f 146000000 and is on the BCM4 GPIO. My Pi is a 3B+.\n", "answer": "I can't dig up my source for this, but I'm pretty sure these spurs are just the result of rpitx using a fractional-N PLL (that isn't in any way optimized for phase noise) to generate a square wave. You get pulses of different (discrete) widths, which means you get spurs at predictable intervals.\nThere isn't really anything you can do to make them go away, but their relative strength will be dependent on your chosen frequency and its relationship to the master oscillator frequency (which I believe is 19.2 MHz). If you're willing to do some trial-and-error, you can scan through the band hoping to find a center frequency where all of the spurs are at acceptable levels (unlikely, but possible) or one where all of the significant spurs are far enough away from the carrier that using a bandpass filter becomes practical.\n", "topic": "ham", "url": "https://ham.stackexchange.com/questions/16667"}, {"image": "VQAonline_00021384.png", "question": "Will I harm batteries by only trickle charging them, or connecting them to a power supply?", "context": "I have some 12V, 12Ah Enercell batteries from RadioShack, RS part number 2301219. As far as I can tell these are AGM SLA batteries.\nI picked up this 3-stage charger from Harbor Freight and as far as I can tell it's charging the batteries (which were probably on the shelf for years and not touched).\nThe goal is to build a portable 12V (and hopefully 36Ah) battery box. I've wired the batteries in parallel:\n\nsimulate this circuit \u2013 Schematic created using CircuitLab\nThe batteries need 14.5-14.9V (max 3.6A) for \"normal charging\" and 13.6-13.8V (max 1.8A) for \"trickle charging\". The packaging says not to use automotive or marine chargers. So for single batteries, the only thing I've found has been something like this charger from PowerStream.\nWill I harm these batteries by only charging them on a trickle charger? If I hook these up to my 15A MFJ power supply for a short period of time, would that hurt these batteries?\n", "answer": "You're not getting answers for multiple reasons.\n1) Batteries are a science but its surrounded by marketing and astrological tradition (concrete floors, blah blah)\n2) If something bad happens they are destroyed or blow up the house or start a fire and no one wants to get involved in that mess. I guess I'm just crazy.\n3) You're using stuff from mainland China, not exactly legendary for reliability. So the charger you have is spec'd by its manual which I downloaded and read, to output never higher than 13.2 volts to float... and from http://www.powerstream.com/SLA.htm its impossible to outgas explosive hydrogen unless the voltage at room temp (it varies with temp) is around 14.5 volts, so all good there, and most SLA recombinators won't vent until about C/3 charge rate and your Chinese charger theoretically only outputs 0.75 amp so its safe on any battery larger than about 3 amphours and you're at 36. So you've got a factor of safety of more than a volt and over ten times the capacity, so IF it works to it's written specs, THEN you're totally safe. At 3/4 amp rate, and 36 theoretical aH capacity, thats going to take more than two days to charge a completely dead battery, but it'll get there eventually.\nThe most effective way to kill a SLA is over volt it causing plate corrosion, and cheap 1970s car chargers don't voltage limit because flooded cell batteries from 1970 can have distilled water added when they outgas (unlike a SLA). So the battery mfgr covers all bases by saying \"don't use a car charger\" But per the specs your modern trickle charger even worst case should be very safe if you compare it to typical SLA battery mfgr specs.\nWith no current limiting directly wiring the MFJ supply would just try to supply like 100 amps and blow a fuse if the battery is low. If it doesn't blow a fuse and you do draw 15 amps (which it won't, for reasons beyond this discussion) then thats about C/2 charge rate and the recombinators in SLAs are usually rated to handle C/3 (optimistically) so you'll pop the vent and leak some acid and get to buy new batteries, which is annoying. In summary, no, don't wire it directly to a power supply without some kind of circuitry in between, like a charger or charge controller type gadget.\nThe powerstream charger looks awesome and would make a great charger.\nIn other news \"which were probably on the shelf for years and not touched\" lead acid cells die by abuse (not a problem here) cycles (not a problem) or old age (oh oh thats a problem). You only get \"less than a decade\" starting the instant acid hits lead back in the factory. Just saying if it sat on the shelf for five years already, they don't really have that long to live anymore. So on one hand if they die \"soon\" it's probably not your fault, and on the other hand if you do something bad to them they were going to the recycler soon enough anyway so you've not got much to lose.\nAnyway 73, best of luck, de N9NFB K\n", "topic": "ham", "url": "https://ham.stackexchange.com/questions/3627"}, {"image": "VQAonline_00021491.png", "question": "Use (TV?) Log-Periodic antenna for VHF", "context": "I found this old log periodic antenna that I assume was from a TV. Could I, or how would I, use this on the amateur radio bands? My main focus would be VHF, with UHF being secondary and HF would also be nice. I'd like to keep it as cheap as possible.\n\nLargest: 4' 5\" (134.62 cm) - seems to be electrically connected to form a 8' 10\" element?\nsmallest: 5\" (12.7 cm) (unfolds to be parallel with another 5\" element)\n\"Other\" smallest: 6\" (15.24 cm) (by itself)\n", "answer": "I'm sorry to say that's not a simple log-periodic antenna, it's a hybrid monstrosity to meet some particular need.\nGuessing from the element lengths:\n\n136 cm each side would work at about 50 MHz, so low-band TV. Two-element log-P with a reflector. OK.\n12 cm each side would work at about 550 MHz, so mid UHF TV. Some sort of Yagi\n15 cm total would be a director for about 900 MHz, so the top end of UHF TV.\n\nIt doesn't seem to cover the VHF ham band, and it probably only starts working above the 440 MHz ham band.\nYou could probably re-use the mounting and feed hardware, cut the feed down to work at 145 MHz, and then install new directors to give it some gain. But you are basically building a new yagi, and with very non-ideal director spacing.\nIf you need a directional antenna (for satellites?) rather build a new yagi from scratch. You can make a nice one in a few hours with some bits of brazing rod and a broomstick.\n", "topic": "ham", "url": "https://ham.stackexchange.com/questions/17621"}, {"image": "VQAonline_00021464.png", "question": "What dot means in JTDX FT8 received messages?", "context": "What dot near country means in JTDX? Maybe LOTW?\n\n", "answer": "Found on the JTDX groups.io (link won't work without login, unfortunately):\n\nsybmol \u2022 - this operator loaded his log into LoTW\n symbol \u00b0 - this operator loaded his log into LoTW and this is Hint decode (FT8 AP or matched filter based decoder in JT65 JT9 T10 modes)\n symbol * - Hint decode (FT8 AP or matched filter based decoder in JT65 JT9 T10 modes)\n\nSo the solid dot and the open dot both mean LoTW user. Star doesn't (it means the same thing that it does in wsjt-x).\n", "topic": "ham", "url": "https://ham.stackexchange.com/questions/15378"}, {"image": "VQAonline_00021456.png", "question": "Where does this common spurious transmission come from? Is there a quality difference?", "context": "I decided to see what my ham transmissions look like on an SDR and was kind of shocked to see how much leaks out. The waterfall display is split between two handhelds during transmit. There are a couple large energy spikes at +/- 800kHz to the 431.5MHz transmitting frequency.\nGiven that both radios spurious emissions are lined up and similar I'm guessing it is common and due to some sort of harmonic effect, possibly from implementing similar architecture.\nIs there any negative to the less dispersed transmission of the bottom device/waterfall? The top half of the waterfall display is from a handheld transceiver that is much higher in quality than the one captured on the bottom half. Interestingly the SDR picks up the better handhelds audio much better even though voice communication between handheld devices is fine.\n\n", "answer": "This is very likely to be be due to overload of the receiver. A quick, rough way to tell the difference:\n\nTune your SDR receiver so the waterfall center frequency is not the same as the transmitter's frequency.\nTransmit.\nCheck whether the spurs you see are symmetric about the transmitter's frequency or the receiver's frequency. This tells you which side generated them.\n\nAlso, if transmitting causes the entire waterfall to rise up then you definitely have an overload problem.\nHow to eliminate overload: First, make sure your receiver's RF gain is set to the minimum level, and AGC (if any) is off. Then try these things:\n\nReplace the transmitter's antenna with a dummy load.\nReplace the receiver's antenna with a dummy load or terminator, or if neither is available, just leave the antenna port unconnected.\nGet more distance.\n\n", "topic": "ham", "url": "https://ham.stackexchange.com/questions/13163"}, {"image": "VQAonline_00021543.png", "question": "Is the spit test for candida legitimate?", "context": "My friend sent me a test to determine if one has too much candida. I called her gullible and she got mad and sent me several other sites which amounted to the same thing, like http://www.candidayeastinfection.com/candida-test-how-to-tell-if-you-have-candida/. Is the spit test for candida legitimate? I tried it and during the day it was negative and during the morning it was only semi-positive.\n\n", "answer": "You're correct, your friend is indeed gullible.\nOropharyngeal candidiasis can be diagnosed by an experienced clinician by looking at the mouth to see characteristic plaques, or by taking swabs for microbiological culture. Blood cultures can be taken for systemic candidiasis. Antibody profiles are suggested as a method to also assist in the diagnosis. The differential movement in water and sedimentation rate of saliva has no known medical use, and is promoted as a way to sell naturopathic anti-candida treatments. Since the test will often be \"positive\" even in the absence of a candida infection, the promoters get to sell more product.\n", "topic": "health", "url": "https://health.stackexchange.com/questions/5472"}, {"image": "VQAonline_00021572.png", "question": "How does a herniated spinal disk correct itself?", "context": "Looking at the picture below, nothing in the herniated disc seems to be able to correct itself over time. Yet, the article I took the picture from says that surgical treatment is used only if the pain is not resolved within a few weeks.\nMy question is: How exactly can this condition be resolved within a few weeks, with drugs only? What happens to the mechanical deformation - the root cause?\n\n", "answer": "The herniated part of the disc can be resorbed by the inflammatory process, which means that inflammatory cells, such as macrophages, can remove the damaged tissue. The healing time can range from several weeks to months.\nConservatively treated massive prolapsed discs: a 7-year follow-up (The Royal College of Surgeons, 2010):\n\nThe mechanism by which herniated discs are resorbed is not fully\n understood. It is generally thought that an immune response develops\n to the disc tissue and inflammation helps to remove the invading\n tissue.\nMassive disc herniations usually reduce in volume and by 6 months most\n are only a third of their original size. Several studies have shown\n that the largest discs appear to have the greatest tendency to resolve.\n\nLong-Term Course to Lumbar Disc Resorption Patients and Predictive Factors Associated with Disc Resorption (Evidence-Based Complementary and Alternative Medicine, 2017):\n\nOf 505 participants, 19 did not show disc resorption, while 486 did. A\n total of 220 displayed resorption rates of \u226550%.\n\nThere is insufficient evidence that any treatment, including analgesics, physical therapy, spinal manipulation, steroid injections and surgery result in greater symptom relief 1-2 years after the onset of symptoms than no specific treatment at all (BMJ, 2007, European Spine Journal, 2007).\nEarly surgery should be considered when symptoms are unbearable, are bilateral, involve the perineum (the area between the legs) or bladder or bowel dysfunction (cauda equina syndrome).\n", "topic": "health", "url": "https://health.stackexchange.com/questions/19519"}, {"image": "VQAonline_00021548.png", "question": "Positive ANA test suggestive of Lupus/SLE, but is 1:160 still ambiguous?", "context": "I was referred to a Rheumatologist earlier this year due to all my symptoms: constant fatigue, getting sick all the time by every little thing, having unexplained fevers, having constant joint pain, swollen joints, digestive issues, nausea, sensitivity to light, erratic menstruation cycles, chest pain, uptick in my migraines, being in a brain fog, coldness in extremities, etc. I already had been diagnosed with fibromyalgia and osteoarthritis, but my Dr. was suspicious that I might I have Lupus so he ordered an ANA (antinuclear antibody) test. \nMy test result is seen below: Abnormal 1:160. My WBC was also soaring, but, my Dr. was undecided on whether or not to diagnose me with Lupus -- though the laboratory technicians had recommended the diagnosis. \nSo is a positive 1:160 ANA test result still considered ambiguous for diagnosis Lupus? He labeled me border-line Lupus. Is that a real thing? What other diagnostic testing is needed to diagnosis definitively that I have or do not have Lupus? \nOr should I see a different rhematologist? \n\n", "answer": "Your doctor may use the term \"possible lupus\" if you meet part, but not all of the diagnostic criteria for lupus [1]. I'm assuming in the remainder of this post that you do not meet criteria for \"probable\" lupus (>= 4 criteria including 1 clinical and 1 immunologic criteria).\nA positive ANA is considered 1 immunologic criteria for the diagnosis of lupus. The general expert consensus is that to be considered \"possible lupus,\" you need at least one criteria plus at least two other features that may be associated for, but are not specific for SLE [1]. These may include (along with some associated symptoms):\n\nOptic neuritis (loss of vision, blurry vision, loss of color perception)\nAseptic meningitis (stiff neck, nausea, vomiting, headache, abnormal lumbar puncture)\nGlomerular hematuria (blood and protein in urine seen under microscope)\nPneumonitis, pulmonary hemorrhage, or pulmonary hypertension, interstitial lung disease (shortness of breath, coughing up blood, abnormal pulmonary function tests, abnormal chest CT scan, abnormal heart echo)\nMyocarditis (chest pain, shortness of breath, abnormal heart echo), verrucous endocarditis (complex manifestations, diagnosed on heart echo)\nAbdominal vasculitis (may manifest as severe acute attacks of abdominal pain)\nRaynaud phenomenon (fingers or toes becoming vividly white when exposed to cold)\n\nNote that all of these symptoms are extremely non-specific and all have causes that are much more common than those listed above, e.g. the overwhelming majority of patients with headaches and abdominal pain and shortness of breath do not have lupus, since those are all symptoms commonly experienced by the general population.\nTo answer your questions: yes, \"borderline lupus\" (possible lupus) is a real thing. Further tests to rule out probable lupus would include a complete physical exam; complete blood count; urinalysis; anti-DNA, anti-Smith, and antiphospholipid antibodies; complement levels; and a direct Coombs test. \nPatients with \"possible lupus\" may or may not progress to eventually have \"probable lupus.\" Your doctor may discuss potential treatment strategies to prevent progression depending on your symptoms and the rest of your evaluation.\n", "topic": "health", "url": "https://health.stackexchange.com/questions/9653"}, {"image": "VQAonline_00021546.png", "question": "What are those white things in my juice & can it cause me any harm?", "context": "So i got this white weird condensed things into my juice once i added some milk into it & i got worried whether it is a normal thing or can it have some risk on my health ! Is this a safe juice & what can cause this? \n\n", "answer": "Fruit juice is acidic, and acid makes milk curdle. This is the end result. It's harmless. It's pretty much just juice-flavored cottage cheese.\n", "topic": "health", "url": "https://health.stackexchange.com/questions/9283"}, {"image": "VQAonline_00021591.png", "question": "Child got A+ blood group when both parents are O+", "context": "Based on my understanding of Blood group tables, such as the one below a child can only get O blood group when both parents have O as their blood groups. In a close relative's case, while her parents and two siblings have O+ as their blood groups, she ended up having A+. We are puzzled as how this could happen?\n\n", "answer": "Although such cases are undoubtedly rare, they are not impossible. Here's a case report from 2005:\n\nApparent deviation from Mendelian rules of blood group inheritance is rarely observed. Blood group O parents with children expressing weak A subgroups have occasionally been described but not explained. A detailed serological investigation of such a family is described here. [...]\nThe propositus' RBCs were very weakly agglutinated with monoclonal anti-A but distinctly with polyclonal anti-A,B, i.e. typical for Ax. Serum anti-A1 (titre 4) and -B were present. Her parents' blood groups were both clearly O, with titres of serum anti-A1, and -A at 16 and 4, respectively. Adsorption/elution studies demonstrated A antigen on the daughter's cells only. The ABO genotypes were: mother, AxO1; father, O1vO2; and propositus, AxO2. The Ax allele was an A1-O1v hybrid allele with a crossing-over breakpoint between positions 235 and 446 in intron 6 (Ax-4). Compared to the A1 glycosyltransferase, this allele predicts a protein with two amino acid substitutions (Phe216Ile and Met277Val) known to yield either weakly expressed or no A antigen on RBCs.\nThis study suggests that the nature of the ABO allele in trans can influence A antigen expression, a phenomenon previously described as allelic enhancement (or reinforcement).\n\n", "topic": "health", "url": "https://health.stackexchange.com/questions/29381"}, {"image": "VQAonline_00021554.png", "question": "Basic CPR practice without a dummy", "context": "I would like to practice basic CPR with my children, but do not want to invest in a CPR doll. I would be interested in suggestions as to how else one could do this. My idea was to practice the bits which are not dangerous with a person and to substitute a firm cushion (or several, to get a feeling for different resistance levels) for the actual pressing, with another at the top for the head. I suspect there must be something better though.\nFor information I am sufficiently qualified to teach first aid at the organisation I volunteer for, but have not been taught the didactics of it.\nSorry if the question is too imprecise for this site, but I hope not.\nUpdate: here is a photo of the result.\n\n", "answer": "With a very small budget\nIf you do not want to spend any (or very little) money on it, I recommend filling a small garbage bag with wet sand and then placing a cushion or pillow over it. I'm guessing if you want to add a realistic touch, you could place some small sticks of wood in the garbage bag to act as ribs. \nUnfortunately, the above answer is all based on my guessing; but seems like it would be fun to experiment with. However, the below answer and link will give you instructions to make a DIY manikin that a CPR instructor has made and uses. \nSlightly bigger budget\nWith some patience, time and approximately $14 USD, you can make a manikin out of a plunger, lid, foam, plaster of Paris and a few other common items. I will not go into detail about it, but here is a link to the blog where you can find detailed instructions, pictures and a list of materials. \nYou mention having the children practice some skills on a human. As long as they are old enough to understand the concept of not actually doing these skills on a person who doesn't need them, I encourage you to teach them on a human. Teach them the anatomical landmarks, how to check for breathing and a pulse and how to open an airway(1) on a real person. \nHope this is helpful to you. If you have any additional questions, please let me know! \n\n(1) Note: If you plan to teach them how to open an airway, be sure to only do the head-tilt chin-lift. If you wish to teach the jaw thrust, that is best done on a doll. \n", "topic": "health", "url": "https://health.stackexchange.com/questions/12611"}, {"image": "VQAonline_00021592.png", "question": "Why are Drugs.com side-effect incidence rates so different from manufacturer/FDA reports?", "context": "Entries at Drugs.com have a section on \"Side Effects... For Healthcare Professionals\" which present percentages for incidence rates of side effects. However, I've found that the rates reported there are wildly at odds with those reported on manufacturer and FDA-approved drug sheets, or any other reference I can find. (Note that Drugs.com collaborates directly with the FDA, and in fact the FDA directs readers to Drugs.com for information on drugs.)\nHere's a slice of one example. Researching Reclast, among the numerous side effects listed on Drugs.com, the following are included:\n\nGastrointestinal\nVery common (10% or more): Nausea (29.1% to 46%), vomiting (14% to 32%), constipation (26.7% to 31%), diarrhea (17.4% to 24%), abdominal\npain (14% to 16.3%), dyspepsia (10%)...\nMusculoskeletal\nVery common (10% or more): Bone pain (55%), myalgia (23%), arthralgia (21%), back pain (15%), and limb pain (14%)...\n\nI'll compare to the prescribing information document, produced by the manufacturer Novartis and approved by the FDA, and available on both of their websites (e.g., at FDA). The following is an excerpt from Table 1.\n\nComparing a few of the side-effect rates noted (Drugs.com vs. Prescribing Information document):\n\nNausea: 29.1-46% vs. 4.5-8.5%.\nVomiting: 14-32% vs. 3.4-4.6%\nDiarrhea: 17.4-24% vs. 5.2-6.0%\nBone pain: 55% vs. 3.2-5.8%\nMyalgia: 21% vs. 4.9-11.7%\n\nThe Prescribing Information document has a few other tables reporting on other studies (for osteopenia, for men, and for Paget's disease), but the incidence rates are all quite similar to those in Table 1.\nOn Drugs.com, individual numbers are not given specific citations, although there is a general list of references at the bottom of the page. In the case of Reclast, there are five: (1) the Product Information document from Novartis, (2) UK summary, (3) Australian APPGuide online, (4) Australian product information, and (5) Kidney Int report (specific to tubular necrosis). Having looked at all of them, the UK and Australian reports give only qualitative descriptors for incidence rates (no percentage numbers), and the APPGuide Online doesn't appear to exist anymore.\nIn short: I can't find any source for the incidence rates shown on Drugs.com, and the numbers shown there are dramatically different from those in the prescribing information document from Novartis and the FDA. In some cases the rates are off by up to 50 percentage points.\nFurthermore, this is true for all of the larger list of side effects shown for Reclast. And if I look at any other drug I see the same situation (e.g., currently investigating osteoporosis drugs: Fosamax, Boniva, Forteo, and Prolia).\nSo: Why are the side-effect incidence rates shown on Drugs.com so different from the FDA-approved prescribing information report? Where are Drugs.com getting these numbers? Which one should be considered more reliable?\n", "answer": "The numbers are different because the package insert rates are based on two clinical trials while the information on Drugs.com is determined by Cerner Multum, a commercial drug database.\nAdverse events frequencies reported in the drug prescribing information is typically determined by initial clinical trial data. The table included in your question indicates two studies as the source of these rates.\nHowever, the very prescribing information you linked notes:\n\nBecause clinical trials are conducted under widely varying conditions, adverse reaction rates observed in the clinical trials of a drug cannot be directly compared to rates in the clinical trials of another drug and may not reflect the rates observed\nin practice.\n\nTurning to Drugs.com, editorial policy page says:\n\nThe information available on the Drugs.com website is displayed under a number of licensing agreements with various publishers. For our drug-database information, we rely on the solid reputation of our suppliers: Cerner Multum, Micromedex and American Society of Health-System Pharmacists.\n\nIndeed at the bottom of the Drugs.com zoledronic acid (Reclast) page we find:\n\nReferences relevant to chosen section (Show all references)\n\nCerner Multum, Inc. \"UK Summary of Product Characteristics.\"\n\n\nIf we review the informational page about Multum on Drugs.com\n\nMultum's content is written by full-time associates who have no affiliations with drug companies, using a combination of sources. Initial References include the Product Information/Package Insert, primary literature and information from standards groups such as the Centers for Disease Control and Prevention (CDC), the World Health Organization (WHO) and the American Academy of Pediatrics.\nPhysicians and pharmacists review the new information and make any necessary changes; all leaflets are reviewed by one or more practicing clinical specialists. Content is verified using secondary references that include standard authoritative medical textbooks and the Multum Expert Review Panel.\n\nCerner itself is the second largest vendor of electronic health record technology in the United States, and so it is also possible that adverse events may be supplemented by information gained by their customers.\nWhich source should be considered more reliable is difficult to say and probably opinion based.\n", "topic": "health", "url": "https://health.stackexchange.com/questions/29399"}, {"image": "VQAonline_00021648.png", "question": "Dating Babylonian Destruction of Jerusalem w/ Only the Bible", "context": "I am interested to see if it's possible to obtain an approximate date of when Jerusalem was destroyed (secular sources say 587 BCE using cuneiform tablets, astronomy, etc.) with only the Bible and its internal dating (e.g. number of years of Kingly rulerships, when prophecies were given, age of people, etc.) I put the following chart together but still can't find a solution.\n\nMy thought was Daniel (who was a \"youth\" during the 3rd year of King Jehoiakim) and only lived after the return of exile could have been used, but we have no information about his age other than what I mentioned. I also thought Jehoiachin + Ezekiel could have provided some clues, but seems nothing to tie the loop.\n", "answer": "The short answer is no, because all Biblical dates that we can convert to modern calendars come via either astronomical observations that we can back-calculate OR by conversion to the *Julian calendar.\nTwo examples:\n\nWe have Babylonian astronomical observations paired with Babylonian calendar dates. The laws of motion allow us to calculate exactly how long ago a particular astronomical phenomenon occurred, thereby tying a Bablyonian date to a date on a modern calendar. This is how we know when Nebuchadnezzar's reign began and when a number of events during his reign occurred.\n\nWe have Roman officials whose terms of office were known, such as the proconsul Gallio before whom Paul appeared while in Corinth. We know when Gallio was in office (+/- about a year), in terms of the regnal years of Emperor Claudius, and we know on the Julian calendar when Claudius was in office. From which we can calculate the timing of many events in Paul's ministry.\n\n\nPerhaps what you're looking for, though, is whether the fall of Jerusalem can be dated by something other than Babylonian dates. Possibly...but in ancient chronology the more certain dates are used to ascertain the less-certain dates, so we would get a less accurate approximation by trying to date the fall of Jerusalem by back calculating from something later, like known events from the Persian Empire.\nBut even then, we'd still have to tie the Biblical chronology back somewhere to an external record of an astronomical event, or the Julian calendar. The Bible doesn't provide direct links to the Gregorian calendar, we always use an intermediary to do that (whether it's the Babylonians or the Persians or the Greeks etc, there's no getting around that).\nEven if we developed a thoroughly internally consistent Biblical calendar using only Biblical texts, and could thereby determine how many years apart two Biblical events were, without astronomy or secular history we couldn't figure out how many years there are between those events and today.\n\n*We could also mention the Greek Olympiad dates or the chronology from Josephus, but these are known by conversion through the Julian calendar as well.\n", "topic": "hermeneutics", "url": "https://hermeneutics.stackexchange.com/questions/60840"}, {"image": "VQAonline_00021602.png", "question": "In Genesis 2:22, what is the significance of the verb \u05d5\u05b7\u05d9\u05b4\u05bc\u05d1\u05b6\u05df (lemma: \u05d1\u05bc\u05b8\u05e0\u05b8\u05d4)?", "context": "The Hebrew text of Gen. 2:22 states,\n\n\u05db\u05d1 \u05d5\u05b7\u05d9\u05b4\u05bc\u05d1\u05b6\u05df \u05d9\u05b7\u05d4\u05b0\u05d5\u05b6\u05d4 \u05d0\u05b1\u05dc\u05b9\u05d4\u05b4\u05d9\u05dd \u05d0\u05b6\u05ea \u05d4\u05b7\u05e6\u05b5\u05bc\u05dc\u05b8\u05e2 \u05d0\u05b2\u05e9\u05b6\u05c1\u05e8 \u05dc\u05b8\u05e7\u05b7\u05d7 \u05de\u05b4\u05df \u05d4\u05b8\u05d0\u05b8\u05d3\u05b8\u05dd \u05dc\u05b0\u05d0\u05b4\u05e9\u05b8\u05bc\u05c1\u05d4 \u05d5\u05b7\u05d9\u05b0\u05d1\u05b4\u05d0\u05b6\u05d4\u05b8 \u05d0\u05b6\u05dc \u05d4\u05b8\u05d0\u05b8\u05d3\u05b8\u05dd\n\nwhich may be translated into English as,\n\nAnd Yahveh God built a woman with the rib that he took from the man, and He brought her to the man.\n\nMost translations obscure the actual meaning of the verb \u05d5\u05b7\u05d9\u05b4\u05bc\u05d1\u05b6\u05df by translating it as \u201cmade.\u201d1 But, the Hebrew verb meaning \u201cto make\u201d is \u05e2\u05b8\u05e9\u05c2\u05b8\u05d4 (asah).2 Instead of \u05e2\u05b8\u05e9\u05c2\u05b8\u05d4, we see a conjugation of the verb \u05d1\u05bc\u05b8\u05e0\u05b8\u05d4 (banah).\nAccording to Heinrich Friedrich Wilhelm Gesenius, the primary meaning of this verb is \u201cto build.\u201d3\n\nThe translators of the LXX translated the Hebrew verb \u05d5\u05b7\u05d9\u05b4\u05bc\u05d1\u05b6\u05df into Greek as \u1fa0\u03ba\u03bf\u03b4\u03cc\u03bc\u03b7\u03c3\u03b5\u03bd (\u014dkodom\u0113sen), a conjugation of the Greek verb \u03bf\u1f30\u03ba\u03bf\u03b4\u03bf\u03bc\u03ad\u03c9 (oikodome\u014d), also meaning \u201cto build.\u201d4 Moses certainly could have used the Hebrew verb \u05e2\u05b8\u05e9\u05c2\u05b8\u05d4, but why didn't he? What significance is there in using the verb \u05d1\u05bc\u05b8\u05e0\u05b8\u05d4?\n\nFootnotes\n1 For example, ASV, ESV, KJV, NET, NIV, NKJV, NLT, RSV.\n2 cp. Gen. 1:7\n3 p. 127-128\n4 (Wilke) p. 439-440\nReferences\nGesenius, Heinrich Friedrich Wilhelm. Gesenius\u2019s Hebrew and Chaldee Lexicon to the Old Testament Scriptures. Trans. Tregelles, Samuel Prideaux. London: Bagster, 1860.\nWilke, Christian Gottlob. A Greek-English Lexicon of the New Testament: Being Grimm Wilke\u2019s Clavis Novi Testamenti. Trans. Thayer, Joseph Henry. Ed. Grimm, Carl Ludwig Wilibald. Rev. ed. New York: American Book, 1889.\n", "answer": "The first two occurrences of 'made' in the Bible (\u05e2\u05e9\u05c2\u05d4 \u2018asa) indicate that although similar to 'built' (\u05d1\u05e0\u05d4 bana) it is more general with respect to the creation process. 'Built' seems to almost imply the gathering of pieces and joining them together as an architect would. 'Made' may describe things created but is more general to encompass virtually any work that produces something else.\nFor example the first two occurrences of 'make' is in Genesis 1:11 and again in 1:12 is talking about trees making fruit (bearing fruit). However 'made' also refers to God's general creation of everything he 'made':\n\nAnd God saw everything that he had made, and behold, it was very good. And there was evening and there was morning, the sixth day. (Genesis 1:31, ESV)\n\nThe word 'built' first occurs when God made Eve from Adams rib and the second occurrence is here:\n\nThen Noah built an altar to the Lord and took some of every clean animal and some of every clean bird and offered burnt offerings on the altar. (Genesis 8:20, ESV)\n\nThe next occurrence is when mention is made of the building of Nineveh in Gen 10:11\nTherefore it seems 'build' is used as we use it in English, to make things out of other objects in a kind of construction and compacting of materials together.\nThere is the obvious suitability to the word 'built' with respect to Eve as God began the construction with a rib, which signifies the beginning of arranging and compacting materials together to form something new. However, in addition to this I would not be surprised if God already inlaid scripture with the notion of the invisible church as typified by Eve. As the church is thought of as a holy building , where as a temple made of various stones perfectly fitted together as a habitation of God, the building may have been prefigured in Eve. Also as a tree with the root of Jesse as its stump, the believing Jews as its trunk and believing Gentiles as grafted in branches, the rib could be seen as similar to the root used to build a unique and well cared for tree getting its life and nourishment from the root.\n", "topic": "hermeneutics", "url": "https://hermeneutics.stackexchange.com/questions/2830"}, {"image": "VQAonline_00021645.png", "question": "In John 1:14 what does \u1f10\u03c3\u03ba\u03ae\u03bd\u03c9\u03c3\u03b5\u03bd mean?", "context": "\n\u00a0\u039a\u03b1\u1f76 \u1f41 \u03bb\u03cc\u03b3\u03bf\u03c2 \u03c3\u1f70\u03c1\u03be \u1f10\u03b3\u03ad\u03bd\u03b5\u03c4\u03bf \u03ba\u03b1\u1f76 \u1f10\u03c3\u03ba\u03ae\u03bd\u03c9\u03c3\u03b5\u03bd \u1f10\u03bd \u1f21\u03bc\u1fd6\u03bd,* \u03ba\u03b1\u1f76 \u1f10\u03b8\u03b5\u03b1\u03c3\u03ac\u03bc\u03b5\u03b8\u03b1 \u03c4\u1f74\u03bd \u03b4\u03cc\u03be\u03b1\u03bd \u03b1\u1f50\u03c4\u03bf\u1fe6, \u03b4\u03cc\u03be\u03b1\u03bd \u1f61\u03c2 \u03bc\u03bf\u03bd\u03bf\u03b3\u03b5\u03bd\u03bf\u1fe6\u03c2 \u03c0\u03b1\u03c1\u1f70 \u03c0\u03b1\u03c4\u03c1\u03cc\u03c2, \u03c0\u03bb\u03ae\u03c1\u03b7\u03c2 \u03c7\u03ac\u03c1\u03b9\u03c4\u03bf\u03c2 \u03ba\u03b1\u1f76 \u1f00\u03bb\u03b7\u03b8\u03b5\u03af\u03b1\u03c2.\n(John 1:14, NA28)\n\nUsually commentators point out that \u03c3\u03ba\u03b7\u03bd\u03cc\u03c9 has the idea of dwelling in a tent, and \u03c3\u03ba\u03b7\u03bd\u03b7 is used in the New Testament and LXX for the tabernacle. However, \u03c3\u03ba\u03b7\u03bd\u03cc\u03c9 is very similar to the Hebrew word \u05e9\u05b8\u05c1\u05db\u05b7\u05df,\n(\u0161akan, imperfect waw consecutive \u05d5\u05b7\u05d9\u05b4\u05bc\u05e9\u05b0\u05c1\u05db\u05b9\u05bc\u05a4\u05df, wayi\u0161kon) which also means dwell and is also used for Mount Sinai.\n\nThe glory of the LORD dwelt on Mount Sinai, and the cloud covered it six days. And on the seventh day he called to Moses out of the midst of the cloud.\n(Exodus 24:16, ESV)\n\nThe context seems to fit John making a parallel to Mount Sinai.\n\nFor the law was given through Moses; grace and truth came through Jesus Christ.\n(John 1:17, ESV)\n\nHere's a chart of the roots of the word in the LXX translating \u05e9\u05b8\u05c1\u05db\u05b7\u05df. Note: most of these roots had the preposition \u03ba\u03b1\u03c4\u03b1 prefixed. The root of \u1f10\u03c3\u03ba\u03ae\u03bd\u03c9\u03c3\u03b5\u03bd is predominant.\n\nTranslations with other than dwell.\nThe Word became flesh \nand took up residence among us. (HCSV)\n\n\nThe Word became a human being and, ..., lived among us. (GNB)\n\n\nSo the Word became human and made his home among us. (LT)\n\n\nThe Word became a human and lived among us. (NCV)\n\n The Word became flesh and blood,\n and moved into the neighborhood. (Message))\n\n\nAnd the Word (Christ) became flesh (human, incarnate) and tabernacled (fixed His tent of flesh, lived awhile) among us; (Amp.)\n\n\nThe Word became flesh and tabernacled among us. (ISV)\n\n\nAnd the Word became flesh, and did tabernacle among us (YLT)\n\nMany more had \"lived.\"\n\n\u05d4\u05b7\u05d3\u05b8\u05bc\u05d1\u05b8\u05e8 \u05e0\u05b4\u05d4\u05b0\u05d9\u05b8\u05d4 \u05d1\u05b8\u05bc\u05e9\u05b8\u05c2\u05e8 \u05d5\u05b0\u05e9\u05b8\u05c1\u05db\u05b7\u05df \u05d1\u05b0\u05bc\u05ea\u05d5\u05b9\u05db\u05b5\u05e0\u05d5\u05bc;\n-- ha-Berit ha-\u1e25adashah. (2000). (John 1:14). Israel: The Bible Society in Israel.\n\n", "answer": "The verb \u03c3\u03ba\u03b7\u03bd\u03cc\u03c9 is literally \"I encamp\", but idiomatically, \"I dwell\" or \"I live among\", etc.\nThis John 1:14 is correctly translated by most versions as \"made his dwelling among us\".\nIn my opinion, the primary precedent is to make a complete contrast with one text and a continuation of another text.\nContrast Precedent:\n\nDan 2:11 - What the king requests is so difficult that no one can tell\nit to him except the gods, whose dwelling is not with mortals.\u201d\n\nContinuation Precedent:\n\nEx 24:16 - The glory of the LORD dwelt/settled on Mount Sinai, and the\ncloud covered it six days. And on the seventh day he called to Moses\nout of the midst of the cloud.\n\n\nEx 25:8 - \"Then have them make a sanctuary for me, and I will dwell\namong them.\n\n\nEx 29:45 - Then I will dwell among the Israelites and be their God.\n\n\nEze 43:9 - Now let them put away from me their prostitution and the\nfuneral offerings for their kings, and I will live among them forever.\n\n\nLev 26:12 - I will walk among you and be your God, and you will be my\npeople.\n\n[There are many OT verse discussing God being among the people.]\nThus, John pointedly contrasts the false gods of Babylon (and others) with the true God of Israel who, via both the shekinah-glory and Jesus' incarnation, lived among us. On this topic, we may also quote 1 Cor 14:25 -\n\nas the secrets of their hearts are laid bare. So they will fall down\nand worship God, exclaiming, \"God is really among you!\"\n2 Cor 6:16 - As God has said: \"I will live with them and walk among\nthem, and I will be their God, and they will be my people.\"\n\n(This verse quotes Lev 26:12; Jer 32:38; Eze 37:27.)\n", "topic": "hermeneutics", "url": "https://hermeneutics.stackexchange.com/questions/58806"}, {"image": "VQAonline_00021612.png", "question": "Does k\u00f4k\u0101b\u00eem (\u05db\u05d5\u05b9\u05db\u05b8\u05d1\u05b4\u05d9\u05dd) in Book of Daniel refer to any bright objects in the sky?", "context": "In Daniel 8:10 we read:\n:\n\nIt grew great, even to the host of heaven. And some of the host and\nsome of the stars it threw down to the ground and trampled on them. [English Standard Version]\nIts power reached to the heavens, where it attacked the heavenly army,\nthrowing some of the heavenly beings and some of the stars to the\nground and trampling them. [New Living Translation]\n\nThis sounds like a heaven war, but obviously proper stars are too big to fall to the Earth.\nCan the Hebrew word for \u201cstars\u201d (\u05db\u05d5\u05b9\u05db\u05b8\u05d1\u05b4\u05d9\u05dd, k\u00f4k\u0101b\u00eem or k\u014d\u00b7w\u00b7\u1e35\u0101\u00b7\u1e07\u00eem) refer to any bright objects or a celestial bodies in the sky similar to the Greek word \u1f00\u03c3\u03c4\u03ad\u03c1\u03b5\u03c2 (ast\u1e17res)?\n\n\n", "answer": "Rashi, here, explains that in Daniel's vision, the horns of the goat represent Persia and Media, and the stars it stomps represents Israel. God, speaking to Abraham, also compared the children of Israel to the stars. See, e.g. Gen. 15:5. See also Deut. 1:10 (\"the Lord your God hath multiplied you, and behold, ye are this day as the stars of heaven for multitude\" (Translation, Hertz Chumash)).\n", "topic": "hermeneutics", "url": "https://hermeneutics.stackexchange.com/questions/13408"}, {"image": "VQAonline_00021642.png", "question": "Is this the commonly known Red Sea?", "context": "In Exodus 13:18 (NASB)\n\nTherefore God led the people around by way of the wilderness to the Red Sea; and the sons of Israel went up in battle formation from the land of Egypt.\n\nDoes this passage reference the commonly known Red Sea?\n\n", "answer": "This cannot be answered with any certainty. Almost every Bible atlas and Bible commentator offers a different theory about the route of the Exodus.\nHere is what can be known:\n1. The Hebrew name for this is \"yam suph\" = \"sea of reeds\". Note the differences in probable reference as from BDB for the word \u05e1\u05d5\u05bc\u05e3 (suph)\n\na. to Gulf of Suez Exodus 10:19; Joshua 2:10 (both J), Exodus 13:18;\nExodus 15:4,22; Exodus 23:31 (all E), Deuteronomy 11:4; Joshua 4:23\n(D), Numbers 33:10,11 (P), elsewhere late Nehemiah 9:9; Psalm 106:7;\nPsalm 106:9; Psalm 106:22; Psalm 136:13; Psalm 136:15.\nb. sometimes to Gulf of Akaba 1 Kings 9:26, and \u05d3\u05b6\u05bc\u05e8\u05b6\u05da\u05b0 \u05d9\u05b7\u05dd\u05e1\u05bf\u05d5\u05bc\u05e3\nNumbers 21:4 (E), probably also Numbers 14:25 (E), Deuteronomy 1:40;\nDeuteronomy 2:1; perhaps Judges 11:16; Jeremiah 49:21; possibly read\n\u05de\u05b4\u05d9\u05b7\u05bc\u05dd\u05e1\u05bf\u05d5\u05bc\u05e3 for \u05f3\u05de\u05d5\u05ba\u05dc \u05e1 Deuteronomy 1:1\n\n2. Comments from Wikipedia https://en.wikipedia.org/wiki/Red_Sea\n\nThe association of the Red Sea with the biblical account of the\nIsraelites crossing the Red Sea is ancient, and was made explicit in\nthe Septuagint translation of the Book of Exodus from Hebrew to Koine\nGreek in approximately the third century B.C. In that version, the Yam\nSuph (Hebrew: \u05d9\u05dd \u05e1\u05d5\u05e3\u200e, lit. 'Sea of Reeds') is translated as Erythra\nThalassa (Red Sea). Although reeds do not grow in the Red Sea today\n(reeds do not grow in salt water), Professor Colin Humphreys explains\nthe discrepancy on the basis that a freshwater marsh of reeds could\nhave existed around Aqaba.[15]\n\nNote the description in Ex 14:22 -\n\nand the Israelites went through the sea on dry ground, with walls of\nwater on their right and on their left.\n\nThus, the \"sea\" must have been deep enough to have \"walls of water\" suggesting more than just a marsh or swampy area.\nThere has been a huge amount of ink expended on this question and we are unlikely to resolve it here. There is not enough data to establish this beyond question.\n", "topic": "hermeneutics", "url": "https://hermeneutics.stackexchange.com/questions/54818"}, {"image": "VQAonline_00021615.png", "question": "How does John 5:28-29 fit with the dispensational hermeneutic", "context": "The dispensational hermeneutic has been summarized as:\n\u201cConsistently literal or plain interpretation is indicative of a dispensational approach to the interpretation of the Scriptures,\u201d declared Charles Ryrie in 1965. (See here.)\nDispensationalism teaches that there will be two resurrections, one of the righteous (at the bodily return of the Lord Jesus Christ) and one of the unrighteous (at the final judgement). These two resurrections are separated by the millennial kingdom. \n \nHowever if we take a literal or plain sense reading of John 5:28-29 where do we see this temporal gap between the two resurrections? \n\n\"Do not marvel at this; for the hour is coming in which all who are in\n the graves will hear His voice and come forth-- those who have done\n good, to the resurrection of life, and those who have done evil, to\n the resurrection of condemnation.\" (John 5:28-29 NKJ)\n\nNotice that we are told that, \"the hour is coming\". This is a singular noun, so the plain sense is that it refers to one single hour and in that hour \"all who are in the graves will hear His voice and come forth\" and then John proceeds to add more detail to that, so that we understand who he means when he says 'all' - \"those who have done good, to the resurrection of life, and those who have done evil, to the resurrection of condemnation.\" \nIt seems to me that a \"consistently literal or plain interpretation\" means we must take John 5:28-29 to be saying that the resurrection of the just and resurrection of the unjust both happen at the return of the Lord Jesus Christ and they happen in the same hour. \nSo, my question is how does dispensationalism (or any futuristic position that separates these two resurrections by a millennium) consistently expound this text? \nI know some commentaries seem to brush over the implications of these words, take for example:\n\n5:28\u201329. Jesus switched suddenly to physical resurrection and\n identified a major doctrine of the New Testament\u2014the concept of two\n resurrections, one for the righteous and one for the wicked. The\n second resurrection is a resurrection to damnation (Rev. 20:13), but\n many interpreters believe there are three parts to the first\n resurrection outlined in Scripture: Christ the firstfruits (1 Cor.\n 15:20, 23); the saints (church) at the rapture; and Old Testament\n believers at the beginning of the millennium. Passages like this\n should lay to rest the false doctrine that death ends all life and\n essentially serves as a cessation of existence. There will not only be\n resurrection; resurrection will be followed by judgment.[Gangel, K. O.\n (2000). John (Vol. 4, p. 103). Nashville, TN: Broadman & Holman\n Publishers.]\n\nThis commentary points out that the text teaches two resurrections, but it does not deal with the time of those resurrections that John appears to be very explicit on. However I am sure some dispensationalist writers must have dealt with this passage in more detail than that, so how does a dispensenational hermeneutic deal consistently with this text?\n", "answer": "One Way\nAt present I only have time to outline how I particularly would fit this text with my dispensational view about two categories of resurrection separated by time periods. There are actually two possible ways the verse can fit (or even combining both ideas):\n\nThe \"hour\" is not necessarily definite, for it lacks the article (\u1f14\u03c1\u03c7\u03b5\u03c4\u03b1\u03b9 \u1f65\u03c1\u03b1), so \"an hour is coming ...,\" which leaves open the possibility that there is an hour in which one group (or technically even one individual) hears, and an hour in which another hears, just so long as \"all\" have an hour in which they do hear His voice.\nThe word \u1f65\u03c1\u03b1 (\"hour\") can be restricted in meaning to roughly a 60 minute period of time, but far more frequently in Greek literature is used to simply refer to a distinct \"period\" of time, hence why the word is also translated \"season\" (as the KJV does in Jn 5:35 for this word) or \"time\" (as the NIV and NET do for v.28), for the word has a much more generic meaning in Greek associated to it.\n\nThe meaning of a term is always a key starting point for the grammatical-historical hermeneutic upon which dispensationalists build their understanding of the text.\nThe \"Plain\" Reading\nSo the \"plain\" reading is that there is \"coming a period of time in which all who are in the graves will hear and come forth.\" \nThis period may refer to the \"type\" of period it is, that is to many separate but distinct individual periods of resurrection \"calls\" (\"an hour in which all in the graves will hear\"). The emphasis in this case is placed on there being a time in which each individual hears the call, and he or she comes forth. That is one way it could be handled.\nHowever, I lean toward one distinct and whole period (\"the hour in which all in the graves will hear\"), partly because it does make for a more plain reading still of the text. This period would be equal to that of the \"last days,\" the period of time in which Christ is reclaiming direct (i.e. physically present) rulership and judging the earth. This period would include Christ's first call of resurrection at the rapture (1 Thes 4:15-16; many dispensationalists would consider this the \"start\" of the period as a pretribulation call, but \"in\" this period fits with any rapture timing view) to the last call to judgment immediately preceding the destruction of the current heaven and earth (Rev 20:11-12).\nWhile either view above fits a plain reading of the text of John 5:28-29, for the dispensationalist, the important point is that it also fits the passages that do indicate timing differences of resurrection.\nOther Dispensationalists*\nLewis Sperry Chafer considers the period of \"Christ's second advent\" as \n\nfrom Christ\u2019s coming \u201cas a thief in the night\u201d (Matt. 24:43; Luke\n 12:39\u201340; 1 Thess. 5:2; 2 Pet. 3:10; Rev. 16:15) to the passing of the\n heavens and the earth that now are and the melting of the elements\n with fervent heat (Systematic Theology, 7:110)\n\nThis passage from Chafer, Dwight Pentecost quotes in Things to Come as an explanation for \"such passages as John 5:28-29\" (174). Pentecost addresses the John 5 passage again later, quoting \"Everett F. Harrison's 'The Christian Doctrine of Resurrection,' p. 46\" (400, reference in n.4):\n\nIt must be granted, however, that the language [of John 5:28-29] does not demand coincidence in the resurrections. John's use of the word ... (hora) in 5:25 allows for its extension over a long period. The same is true of 4:21, 23.\n\nIn short, neither the language itself, nor the period referenced, prevent the dispensational reading, nor do they violate a \"plain\" reading of the text, since \"hour\" is such an indistinct term in its usage.\nConclusion\nObviously the John 5:28-29 passage alone gives no distinct indication of a timing difference in resurrections, as its focus is upon the quality and end result differences of the two types of resurrections, but it does leave open the possibility of a timing difference, explicitly made clear in other Scripture, by the indistinct language used.\n\n* When I get more time, I will do additional research how other dispensationalists have explicitly handled it.\n", "topic": "hermeneutics", "url": "https://hermeneutics.stackexchange.com/questions/19242"}, {"image": "VQAonline_00021635.png", "question": "In John 11:33 what does \u1f10\u03bd\u03b5\u03b2\u03c1\u03b9\u03bc\u03ae\u03c3\u03b1\u03c4\u03bf \u03c4\u1ff7 \u03c0\u03bd\u03b5\u03cd\u03bc\u03b1\u03c4\u03b9 \u03ba\u03b1\u1f76 \u1f10\u03c4\u03ac\u03c1\u03b1\u03be\u03b5\u03bd \u1f11\u03b1\u03c5\u03c4\u1f78\u03bd mean in this context?", "context": "How you translate the verbs in \u1f10\u03bd\u03b5\u03b2\u03c1\u03b9\u03bc\u03ae\u03c3\u03b1\u03c4\u03bf \u03c4\u1ff7 \u03c0\u03bd\u03b5\u03cd\u03bc\u03b1\u03c4\u03b9 \u03ba\u03b1\u1f76 \u1f10\u03c4\u03ac\u03c1\u03b1\u03be\u03b5\u03bd \u1f11\u03b1\u03c5\u03c4\u1f78\u03bd in 11:33 affects how you interpret the surrounding verses, especially when answering the question why did Jesus weep (11:35). Translations vary significantly on how they translate the verb \u1f10\u03bd\u03b5\u03b2\u03c1\u03b9\u03bc\u03ae\u03c3\u03b1\u03c4\u03bf.\n\n\u1f10\u03bc\u03b2\u03c1\u03b9\u03bc\u03ac\u03bf\u03bc\u03b1\u03b9c: to have an intense, strong feeling of concern, often with the implication of indignation\u2014\u2018to feel strongly, to be indignant.\u2019 \u1f38\u03b7\u03c3\u03bf\u1fe6\u03c2 \u03bf\u1f56\u03bd \u1f61\u03c2 \u03b5\u1f36\u03b4\u03b5\u03bd \u03b1\u1f50\u03c4\u1f74\u03bd \u03ba\u03bb\u03b1\u03af\u03bf\u03c5\u03c3\u03b1\u03bd \u03ba\u03b1\u1f76 \u03c4\u03bf\u1f7a\u03c2 \u03c3\u03c5\u03bd\u03b5\u03bb\u03b8\u03cc\u03bd\u03c4\u03b1\u03c2 \u03b1\u1f50\u03c4\u1fc7 \u1f38\u03bf\u03c5\u03b4\u03b1\u03af\u03bf\u03c5\u03c2 \u03ba\u03bb\u03b1\u03af\u03bf\u03bd\u03c4\u03b1\u03c2, \u1f10\u03bd\u03b5\u03b2\u03c1\u03b9\u03bc\u03ae\u03c3\u03b1\u03c4\u03bf \u03c4\u1ff7 \u03c0\u03bd\u03b5\u03cd\u03bc\u03b1\u03c4\u03b9 \u2018then when Jesus saw her weeping and saw those Jews who were with her weeping, his feeling was intense\u2019 or \u2018\u2026 he was indignant\u2019 Jn 11:33.\nLouw, J. P., & Nida, E. A. (1996). Greek-English lexicon of the New Testament: based on semantic domains (electronic ed. of the 2nd edition., Vol. 1, p. 293). New York: United Bible Societies.\n\nSome translations emphasize strong feelings of concern, while others emphasize indignation or even anger. Can we translate these phrases before we interpret this passage?\nLooking at the senses from Logos Bible Software:\n\nespecially combined with \u03c4\u1ff7 \u03c0\u03bd\u03b5\u03cd\u03bc\u03b1\u03c4\u03b9, the meaning appears to be restrain oneself or a smoother translation *held back his emotions. Thus, Jesus' weeping in 11:35 was reserved compared to his deep feelings.\n", "answer": "Many thanks, Perry Webb, for this superbly crafted question. I am not sure this answer it but it may be the spark that helps someone else along to finish the job.\nThe verb \u1f10\u03bc\u03b2\u03c1\u03b9\u03bc\u03ac\u03bf\u03bc\u03b1\u03b9 (embrimaomai), as the OP would be aware, only occurs five times in the NT, Matt 9:30, Mark 1:42, 14:5, John 11:33, 38. BDAG offers little to advance our understanding beyond listing the common translations in the most popular versions.\nAll occurrences of this verb use either middle or passive voice and none is active. In such circumstances I like to examine the subtle but literal translation of David Bentley Hart who often employed brilliant (but Herculean) efforts to render the nuances of the Greek verb which are listed below including the verb tense, mood and voice. \n\nMatt 9:30 (Aorist Indicative Passive - 3rd Person Singular) Jesus sternly commanded them \u2026 \nMark 1:43 (Aorist Participle Middle - Nominative Masculine Singular) And, sternly admonishing him \u2026 \nMark 14:4 ( Imperfect Indicative Middle or Passive - 3rd Person Plural) some who expressed indignation to one another \nJohn 11:33 (Aorist Indicative Middle - 3rd Person Singular) He groaned in His Spirit and yielded Himself to His turmoil \u2026 \nJohn 11:38 (Present Participle Middle or Passive - Nominative Masculine Singular) So Jesus, again groaning within Himself, come to the tomb \u2026 \n\nI think the key to understanding this very nuanced verb is (as usual) the context. Note that in Matt and Mark, the situation is some one expressing emotion to some one else; while in John, it is Jesus that experiences the deep emotion without trying to convey that to anyone else (although they notice his emotional state).\nThe root of the verb is clearly deeply felt passion. But note the surrounding language - Jesus is clearly deeply moved by the emotion, specifically grief, of His surrounding friends. In John 11:32, 33, it is Mary's obvious grief that moved Jesus; Jesus was \"deeply moved in his spirit and was troubled in himself\" (my translation).\nThus, John's use of \u1f10\u03bc\u03b2\u03c1\u03b9\u03bc\u03ac\u03bf\u03bc\u03b1\u03b9 (embrimaomai) appears to be emotionally internal; while Matthew and Mark use the same verb in an attempt to convey deeply felt passion to others. Thus we find the versions desperately trying to covey this translating John 11:33 as \"deeply moved\", or \"groaning\", within himself (or similar); while Matt and Mark are translated, \"sternly warned\", \"scolded\", etc.\n", "topic": "hermeneutics", "url": "https://hermeneutics.stackexchange.com/questions/48024"}, {"image": "VQAonline_00021757.png", "question": "Are all Samadhis are required in order OR can be practiced irrespective of each other?", "context": "In Yog sutra Patanjali mentioned 10 types of samadhis :\n\nSeems all are related and are inter-related to each other. \nSo questions arise in my mind are:\n\nIs there any text that suggest the order in which we should try Samadhis?\nDo all samadhis are required or based on personality some are optional ?\nIs there any side effect of Samadhi?\nIs that possible to achieve Samadhi without guidance of in-person Guru just by knowledge by books & try to practice that?\n\nPlease forgive my ignorance if something seems foolish to ask since I am seeking knowledge and yet to advance this stage experientially but I wish to seek that if that is possible. Please guide me. \n", "answer": "Your question is based on Yog Sutras of Patanjalii - Sutra No. 1.17. Below is the sanskrit sutra. \n\n\u0935\u093f\u0924\u093e\u0930\u094d\u0915\u0935\u093f\u091a\u093e\u0930\u093e\u0928\u0928\u094d\u0926\u0938\u094d\u092e\u093f\u0924\u093e\u0930\u0941\u092a\u0928\u0941\u0917\u092e\u093e\u0924\u094d\u0938\u092e\u094d\u092a\u091c\u094d\u091e\u093e\u0924 : || 1.17||\nSamprajnata , the Samadhi of Wisdom , occurs through the accompaniment\n of the appearance (RP ,BB) of gross thought ( Vitarka) ,Subtle thought\n ( Vichara) , ecstasy (ananda) and I-am- ness (asmita).\n\nHere Patanjali is describing various types of Samadhi's or meditation a Yogi should gradually practice in order to purify his mind for achieving Moksha or self-Realization or enlightenment. The soul purpose of all these all types of samadhis is to gradually develop Dispassion in his mind and to isolate him from his material as well as subtle world and to lead him towards his final goal. This is very difficult and laborious task. \nFor answring purpose i am taking help of commentary provided in Yoga-Sutras of patanjali with the exposition of Vyasa - Translation and commentary by- Pandit Usarbudh Arya- Vol.1 .\n\nDo all samadhis are required or based on personality some are optional?\nFrom commentary we come to know that all these steps are sequential and gradual , although not dependent on each other but are necessary in achieving final goal. And why are all these levels or types are necessary in achieving final liberation the answer can be found in below passage ,where it is said that in Practicing samadhi or meditation a yogi experiences some faults in each level of realization and thus he develops dispassion towards each of these types or levels until the final realization occurs. \nIt is also said that just mastering each of the stage or type is not important but before going to next level or type a full dispassion towards last type or level should be achieved .The purpose of each type is to lead a yogi sequentially to get free from all material as well as emotional bondages step by step. And in the process he overcomes the previous level and develops dispassion towards previous level and the outcome of that level .i.e. siddhis etc. \n\nWhat then is the purpose of such realization at these various levels\n in Samprajnata Samadhi? They are to fulfil the purpose described in YS\n 1.15 ,16 : to develop dispassion towards each level of the material evolutes. At each level one feels that the next subtler level is\n purer. But upon examination by realization it is found that the purity\n is only relative , at best derived from sattva ,which is an attribute\n of matter. Each level is successively found to be attended by some\n faults and blemishes (NB) that have been describe earlier. Thus a\n Vairagya towards each successive state develops . Simply perfecting\n the level of realization is not conductive to progress towards\n isolation and liberation . It is only when full dispassion develops\n towards that very experience and realization that the next step can be\n taken towards a yet subtler object of concentration. - [Page -222 - Second paragraph.]\n\n\nSo we can see that achieving Kaivalya or Moksha or enlightenment is a gradual or sequential process. And during all types of samadhi a Yogi is sequetially gets freed from various assumed bondages.And in the last samadhi stage i.e. in a cognitive ( asamprajnata) samadhi , final enlightenment happens. \n\nAs the process of samadhi advances , the assumption of false\n identifications with each shealth are dropped and gradual freedom from\n morality ,action, mentation,ignorance ,pain and bondage is achieved.\n One is freed from\n\n\nOne can achieve final state of Samadhi Or Moksha through grace of God Directly.\nA Yogi can bypass all the stages of Samadhis and can directly reach to final stage by the grace of the God through path of Devotion i.e. Bhakti etc. avoiding all the labour. In such a case he do not experience the other types of samadhis and their outcome i.e. siddhis etc. He directly gets enlightenlemt from God. \nHowever it is suggested that these stages or types are there and one should go through them in order to achieve the grace of the god , they make a yogi's mind pure step by step and makes him fit for grace. \n\nThe order in which the four stages are practised and mastered are\n important (BG NB).It is clearly stated however (BG) that , if a yogi\n makes spontaneous ascent to a higher ground by the grace of the God ,\n he then need not climb methodically and laboriously over the lower\n steps (BG) .Grace , then is a shortcut which bypass the method. It\n must be remembered however ,that when one climbs to a higher plateau\n through force such Grace, the lower siddhis attendant upon the\n practice of earlier steps do not occur .It is only for some reasons\n one need those siddhis - even they are relevant to one's spiritual\n attainment ,station and progress.\nFor the purpose here it is assumed that a method is generally\n requisite for mental purification and mastery in order that one may\n become deserving vessel for Grace. -[ Page - 243 -Second Paragraph.]\n\n\nSo in conclusion all these types of Samadhis are gradual and sequential. They are designed so that a yogi step by step can gets isolated from all his bondages of material as well as subtle world. leading him towards final stage. Although he can directly achieve enlightenment through grace of God , these types helps him to become fit for the Grace , so all these types are important even one wants the Moksha through Grace via Bhakti etc. \nSo it is assumed that a Yogi should practice all these types in order to gradually purify himself and becoming entirely free from this material as well as emotional world . Even in case of attempt of direct enlightenment these types helps and makes him fit for Grace of God. \n", "topic": "hinduism", "url": "https://hinduism.stackexchange.com/questions/29122"}, {"image": "VQAonline_00021724.png", "question": "Why Lord Krishna says he is Bhrigu in sages?", "context": "Why Lord Krishna says he is Bhrigu in sages whereas Bhrigu rishi put leg on the Lord Vishnu's chest? Is there any reason behind calling himself as Sage bhrigu?\nreference geeta ch-10 verse 25\n\n", "answer": "Prabhup\u0101da notes that Bh\u1e5bgu is the most powerful among all of Brahm\u0101's sons, so that's why K\u1e5b\u1e63\u1e47a says he's Bh\u1e5bgu among the \u1e5b\u1e63i's in BG 10.25:\n\nBrahm\u0101, the \ufb01rst living creature within the universe, created several sons for the propagation of various kinds of species. Among these sons, Bh\u1e5bgu is the most powerful sage ...\n\nAlso, according to the Mah\u0101bh\u0101rata, Bh\u1e5bgu was also reborn out of fire:\n\nThe great and blessed saint Bhrigu, we are informed, was produced by the self-existing Brahma from the fire at the sacrifice of Varuna.\n\nAnd a couple of verses earlier (BG 10.23), we see that K\u1e5b\u1e63\u1e47a is comparing himself with fire (Agni).\n\nOf all the Rudras I am Lord \u015aiva, of the Yak\u1e63as and R\u0101k\u1e63asas I am the Lord of wealth [Kuvera], of the Vasus I am \ufb01re [Agni], and of mountains I am Meru.\n\n", "topic": "hinduism", "url": "https://hinduism.stackexchange.com/questions/16717"}, {"image": "VQAonline_00021686.png", "question": "Who created Datta Bavani?", "context": "Who created Datta Bavani? Please provide some introduction of the creator.\n\n", "answer": "Rang Avadhoot Maharaj Created Datta Bavani. He also gave one mantra\nParaspara Devo bhava\nmeans we humans should behave as devas. means we should not harm anybody.\nBhakti should be without sho off.\n", "topic": "hinduism", "url": "https://hinduism.stackexchange.com/questions/10198"}, {"image": "VQAonline_00021699.png", "question": "What is the story of Somanath Jyotirlinga?", "context": "Lord Shiva has been worshipped in linga form from time beginning as described in answer here. Godesses Saraswati in Linga Purana states:\n\n\u0938\u0930\u094d\u0935\u0902 \u0932\u093f\u0902\u0917\u092e\u092f\u0902 \u0932\u094b\u0915\u0902 \u0938\u0930\u094d\u0935\u0902 \u0932\u093f\u0902\u0917\u0947 \u092a\u094d\u0930\u0924\u093f\u0937\u094d\u0920\u093f\u0924\u092e\u094d \u0964 \n\u0924\u0938\u094d\u092e\u093e\u0924\u094d\u0938\u0930\u094d\u0935\u0902 \u092a\u0930\u093f\u0924\u094d\u092f\u091c\u094d\u092f \u0938\u094d\u0925\u093e\u092a\u092f\u0947\u0924\u094d\u092a\u0942\u091c\u092f\u0947\u091a\u094d\u091a \u0924\u0924\u094d \u0964\u0964 \nThe whole world is identical with the Linga. Everything is founded on the Linga. Hence, one shall eschew everything, install the Linga and worship it.\n\nThe above verses itself establishes glory of Linga. Among the Lingas also there are 12 major Jyotirlingas which are most important. Shiva Purana in Koti Rudra Samhita, DwadashaJyotirlinga Mahatmya Chapter sates:\n\n\u0938\u094c\u0930\u093e\u0937\u094d\u091f\u094d\u0930\u0947 \u0938\u094b\u092e\u0928\u093e\u0925\u0902 \u091a \u0936\u094d\u0930\u0940 \u0936\u0948\u0932\u0947 \u092e\u0932\u094d\u0932\u093f\u0915\u093e\u0930\u094d\u091c\u0941\u0928\u092e\u094d \u0964\n\u0909\u091c\u094d\u091c\u092f\u093f\u0928\u094d\u092f\u093e\u0902 \u092e\u0939\u093e\u0915\u093e\u0932\u092e\u094b\u0902\u0915\u093e\u0930\u0947 \u092a\u0930\u092e\u0947\u0936\u094d\u0935\u0930\u092e\u094d \u0964\u0964\n\u0915\u0947\u0926\u093e\u0930\u0902 \u0939\u093f\u092e\u0935\u0924\u094d\u092a\u0943\u0937\u094d\u0920\u0947 \u0921\u093e\u0915\u093f\u0928\u094d\u092f\u093e\u0902 \u092d\u0940\u092e\u0936\u0919\u094d\u0915\u0930\u092e\u094d \u0964\n\u0935\u093e\u0930\u093e\u0923\u093e\u0938\u094d\u092f\u093e\u0902 \u091a \u0935\u093f\u0936\u094d\u0935\u0947\u0936\u0902 \u0924\u094d\u0930\u092f\u092e\u094d\u092c\u0915\u0902 \u0917\u094c\u0924\u092e\u0940\u0924\u091f\u0947 \u0964\u0964\n\u0935\u0948\u0926\u094d\u092f\u0928\u093e\u0925\u0902 \u091a\u093f\u0924\u093e\u092d\u0942\u092e\u094c \u0928\u093e\u0917\u0947\u0936\u0902 \u0926\u093e\u0930\u0941\u0915\u093e\u0935\u0928\u0947 \u0964\n\u0938\u0947\u0924\u0941\u092c\u0928\u094d\u0927\u0947 \u0924\u0941 \u0930\u093e\u092e\u0947\u0936\u0902 \u0918\u0941\u0936\u094d\u092e\u0947\u0936\u0902 \u091a \u0936\u093f\u0935\u093e\u0932\u092f\u0947 \u0964\u0964\n\u0926\u094d\u0935\u093e\u0926\u0936\u0948\u0924\u093e\u0928\u093f \u0928\u093e\u092e\u093e\u0928\u093f \u092a\u094d\u0930\u093e\u0924\u094d\u092f\u0938\u094d\u0924\u094d\u0925\u093e\u092f \u092f\u0903 \u092a\u0920\u0947\u0924\u094d \u0964\n\u0938\u0930\u094d\u0935\u092a\u093e\u092a\u0935\u093f\u0928\u093f\u0930\u094d\u092e\u0941\u0915\u094d\u0924\u0902 \u0938\u0930\u094d\u0935\u0938\u093f\u0926\u094d\u0927\u093f\u092b\u0932\u0902 \u0932\u092d\u0947\u0924\u094d \u0964\u0964 \nSomanath in Saurastra, Mallikarjuna in Shri Saile, Mahakala in Ujjaiyana and Parameshawara also manifested as Omkara. Kedara in Himavat and Bhima Shankara in Dakinya. Vishewaswara in Varanasi and Trayambaka in side of Gautami. Vaidyanath in ChitaBhumi and\u00a0 Nageshwara in forest of Daruka. Rameshwara in Setubandha and Ghusmeshwara in Shivalaya. These name of 12 Jyotirlingas who recites in morning, he gets freed from all sins and is eligible to get all Siddhis.\n\nThere are twelve Jyotirlingas mentioned in another similar popular Shloka too which is as:\n\nSaur\u0101\u1e63\u1e6dre Soman\u0101tha\u1e43 ca \u015ar\u012b\u015baile Mallik\u0101rjunam\nUjjayiny\u0101\u1e43 Mah\u0101k\u0101lam Omk\u0101ram Mamle\u015bhwaram\nParaly Vaidyan\u0101tha\u1e43 cha \u1e0c\u0101kiny\u0101\u1e43 Bh\u012bma\u015bha\u1e45karam\nSetubandhe tu R\u0101me\u015ba\u1e43 N\u0101ge\u015bha\u1e43 D\u0101ruk\u0101vane\nV\u0101r\u0101\u1e47asy\u0101\u1e43 tu Vi\u015bve\u015ba\u1e43 Tryambaka\u1e43 Gautam\u012bta\u1e6de\nHim\u0101laye tu Ked\u0101ra\u1e43 Ghu\u015bme\u015ba\u1e43 ca \u015aiv\u0101laye\net\u0101ni jyotirli\u1e45g\u0101ni s\u0101ya\u1e43 pr\u0101ta\u1e25 pa\u1e6dhennara\u1e25\nsaptajanmak\u1e5bta\u1e43 p\u0101pa\u1e43 smara\u1e47ena vina\u015byati\n\nThe first Jyotirlinga mentioned is Somanatha Jyotirlinga. I want to know, what is the story of formation of Somanath Jyotirlinga? Why Lord Shiva manifested himself in Somanath Jyotirlinga? And how this Jyotirlinga get its name?\nPlease mention the scriptural reference.\nSomanath temple is situated in Saurashtra (Gujarat).\n\nBy Anhilwara - Own work, Public Domain, https://commons.wikimedia.org/w/index.php?curid=18227742\n", "answer": "Somnatha Jyotirlinga is first of the Jyotirlingas on Earth (Bhooloka). Story of Somesvara is described in detail in both Shiva Mahapurana and Skanda Maha Purana.\nThis is how Chapter 14 of RudraSamhita of Shiva Purana describes the story.\nDaksha's 27 daughters were married to Soma, the moon God. But Moon God always loved Rohini more compared to other wives. This made other wives infuriated and they explained their tale of suffering to their father Daksha.\n\n\u0938\u092a\u094d\u0924\u0935\u093f\u0902\u0936\u0928\u094d\u092e\u093f\u0924\u093e\u0903 \u0915\u0928\u094d\u092f\u093e \u0926\u0915\u094d\u0937\u0947\u0923 \u091a \u092e\u0939\u093e\u0924\u094d\u092e\u0928\u093e\u0964 \u0924\u0947\u0928 \u091a\u0928\u094d\u0926\u094d\u0930\u092e\u0938\u0947 \u0926\u0924\u094d\u0924\u093e\n\u0905\u0936\u094d\u0935\u093f\u0928\u094d\u092f\u093e\u0926\u094d\u092f\u093e \u092e\u0941\u0928\u0940\u0936\u094d\u0935\u0930\u093e\u0903\u0965 \u096b\u0965 \n\n\nO great sage, his (Daksha) daughters,\ntwenty-seven in number, A\u0161vini and others, were given in marriage to\nthe moon by Dak\u0161a, the great soul. \n\u091a\u0928\u094d\u0926\u094d\u0930\u0902 \u091a \u0938\u094d\u0935\u093e\u092e\u093f\u0928\u0902 \u092a\u094d\u0930\u093e\u092a\u094d\u092f \u0936\u094b\u092d\u092e\u093e\u0928\u093e \u0935\u093f\u0936\u0947\u0937\u0924\u0903\u0964 \u091a\u0928\u094d\u0926\u094d\u0930\u094b\u093d\u092a\u093f \u091a\u0948\u0935 \u0924\u093e\u0903\n\u092a\u094d\u0930\u093e\u092a\u094d\u092f \u0936\u094b\u092d\u0924\u0947 \u0938\u094d\u092e \u0928\u093f\u0930\u0928\u094d\u0924\u0930\u092e\u094d\u0964\u0964 \u096c \u0964\u0964 \n\n\nAchieving the moon as their\nlord, all the girls felt graceful. The moon also felt graceful with\nthem. \n\u0939\u0947\u092e\u094d\u0928\u093e \u091a\u0948\u0935 \u092e\u0923\u093f\u0930\u094d\u092d\u093e\u0924\u093f \u092e\u0923\u093f\u0928\u093e \u0939\u0947\u092e \u091a\u0948\u0935 \u0939\u093f\u0964 \u090f\u0935\u0902 \u091a \u0938\u092e\u092f\u0947 \u0924\u0938\u094d\u092f \u092f\u091e\u094d\u091c\u093e\u0924\u0902\n\u0936\u094d\u0930\u0942\u092f\u0924\u093e\u092e\u093f\u0924\u093f\u0965\u096d\u0965\n\n\nThe gold looks graceful with the gem and same is\nthe case of gems with the gold. Now you listen to what happened\nthereafter.\n\u0938\u0930\u094d\u0935\u093e\u0938\u094d\u0935\u092a\u093f \u091a \u092a\u0924\u094d\u0930\u0940\u0937\u0941 \u0930\u094b\u0939\u093f\u0923\u0940\u0928\u093e\u092e \u092f\u093e \u0938\u094d\u092e\u0943\u0924\u093e\u0964 \u092f\u0925\u0948\u0915\u093e \u0938\u093e \u092a\u094d\u0930\u093f\u092f\u093e\n\u091a\u093e\u0938\u0940\u0924\u094d\u0924\u0925\u093e\u093d\u0928\u094d\u092f\u093e \u0928 \u0915\u0926\u093e\u091a\u0928\u0965\u096e\u0965 \n\n\nOf all the twenty-seven wives, he did\nnot love any one so much at anytime as he loved Rohi\u0146\u012b. \n\u0905\u0928\u094d\u092f\u093e\u0936\u094d\u091a \u0926\u0941\u0903\u0916\u092e\u093e\u092a\u0928\u094d\u0928\u093e\u0903 \u092a\u093f\u0924\u0930\u0902 \u0936\u0930\u0923\u0902 \u092f\u092f\u0941\u0903\u0964 \u0917\u0924\u094d\u0935\u093e \u0924\u0938\u094d\u092e\u0948 \u091a \u092f\u0926\u094d\u0926\u0941\u0903\u0916\u0902 \u0924\u0925\u093e\n\u0924\u093e\u092d\u093f\u0930\u094d\u0928\u093f\u0935\u0947\u0926\u093f\u0924\u092e\u094d\u0965\u096f\u0965 \n\n\nThe rest of the wives of the moon went to\ntheir father, feeling painful and took refuge with him. They explained\nthe tale of their sufferings to their father. \n\nDaskha tried to convince Soma, but moon God didn't heed. Daksha cursed the Soma to get consumed by disease (waning).\n\n\u0926\u0915\u094d\u0937\u0936\u094d\u091a\u0948\u0935\u0902 \u0938 \u0938\u092e\u094d\u092a\u094d\u0930\u093e\u0930\u094d\u0925\u094d\u092f \u091a\u0928\u094d\u0926\u094d\u0930\u0902 \u091c\u093e\u092e\u093e\u0924\u0930\u0902 \u0938\u094d\u0935\u092f\u092e\u094d\u0964 \u091c\u0917\u093e\u092e \u092e\u0928\u094d\u0926\u093f\u0930\u0902\n\u0938\u094d\u0935\u0902 \u0935\u0948 \u0928\u093f\u0936\u094d\u091a\u092f\u0902 \u092a\u0930\u092e\u0902 \u0917\u0924\u0903\u0964\u0964 \u0967\u0969\u0965\nS\u016bta said, \u201cThus requesting to moon, his son-in-law, Daksa returned to\nhis place feeling satisfied.\n\u091a\u0928\u094d\u0926\u094d\u0930\u094b\u093d\u092a\u093f \u0935\u091a\u0928\u0902 \u0924\u0938\u094d\u092f \u0928 \u091a\u0915\u093e\u0930 \u0935\u093f\u092e\u094b\u0939\u093f\u0924\u0903\u0964 \u0936\u093f\u0935\u092e\u093e\u092f\u093e\u092a\u094d\u0930\u092d\u093e\u0935\u0947\u0923 \u092f\u092f\u093e\n\u0938\u092e\u094d\u092e\u094b\u0939\u093f\u0924\u0902 \u091c\u0917\u0924\u094d\u0965 \u0967\u096a\u0965 \nThe moon on the other hand, having been over-powered with the delusion\nof Siva,did not bother about the advice of Daksa. \n\u0936\u094d\u0930\u0942\u092f\u0924\u093e\u0902 \u091a\u0928\u094d\u0926\u094d\u0930! \u092f\u0924\u094d\u092a\u0942\u0930\u094d\u0935 \u092a\u094d\u0930\u093e\u0930\u094d\u0925\u093f\u0924\u094b \u092c\u0939\u0941\u0927\u093e \u092e\u092f\u093e\u0964 \u0928 \u092e\u093e\u0928\u093f\u0924\u0902 \u0924\u094d\u0935\u092f\u093e\n\u092f\u0938\u094d\u092e\u093e\u0924\u094d\u0924\u0938\u094d\u092e\u093e\u0924\u094d\u0924\u094d\u0935\u0902 \u091a \u0915\u094d\u0937\u092f\u0940 \u092d\u0935\u0964\u0964 \u0967\u096e \u0964\u0964\nDaksa said, \u201cO Moon, you listen, I have made several requests to you\nwhich remain unheaded. Therefore you will suffer from the disease of\nconsumption.\u201d \n\u0938\u0942\u0924 \u0909\u0935\u093e\u091a \u0907\u0924\u094d\u092f\u0941\u0915\u094d\u0924 \u0924\u0947\u0928 \u091a\u0928\u094d\u0926\u094d\u0930\u094b \u0935\u0948 \u0915\u094d\u0937\u092f\u0940 \u091c\u093e\u0924\u0903 \u0915\u094d\u0937\u0923\u093e\u0926\u093f\u0939\u0964\n\u0939\u093e\u0939\u093e\u0915\u093e\u0930\u094b \u092e\u0939\u093e\u0928\u093e\u0938\u0940\u0924\u094d\u0924\u0926\u0947\u0928\u094d\u0926\u094c \u0915\u094d\u0937\u0940\u0923\u0924\u093e\u0902 \u0917\u0924\u0947\u0965 \u0967\u096f\u0965 \nS\u016bta said, \u201cAt these words of Daksa, the moon atonce attracted the\nconsumption disease, with the moon\u2019s attracting the ailment of\nconsumption, there was a turmoil everywhere. \n\nAll the sages with Devas being unsettled by changes went to Prabhasa Khanda and said to Soma to worship Shiva with Mahamrutyunja mantra.\n\n\u0938\u094d\u0925\u093e\u092a\u094d\u092f \u091a\u0928\u094d\u0926\u094d\u0930\u0902 \u092a\u094d\u0930\u092d\u093e\u0938\u0947 \u091a \u0938\u094d\u0935\u0902 \u0938\u094d\u0935\u0902 \u0927\u093e\u092e \u092f\u092f\u0941\u0930\u094d\u092e\u0941\u0926\u093e\u0965\u0969\u096e\u0965 \nInvoking all the holy places there, including Sarasvati, they started\nadoring Siva by Mrtyu\u00f1jaya-pujan-vidhi. The gods and the ascetics,\nwith the spotless intentions, seated the moon at the Prabh\u0101sa-kSetra,\nand went back to their respective places. \n\u091a\u0928\u094d\u0926\u094d\u0930\u0947\u0923 \u091a \u0924\u092a\u0938\u094d\u0924\u092e\u0902 \u0937\u0923\u094d\u092e\u093e\u0938\u0902 \u091a \u0928\u093f\u0930\u0928\u094d\u0924\u0930\u092e\u094d\u0964 \u092e\u0943\u0924\u094d\u092f\u0941\u091e\u094d\u091c\u092f\u0947\u0928 \u092e\u0928\u094d\u0924\u094d\u0930\u0947\u0923\n\u092a\u0942\u091c\u093f\u0924\u094b \u0935\u0943\u0937\u092d\u0927\u094d\u0935\u091c\u0903\u0964\u0964 \u0969\u096f \u0964\u0964\nThe moon performed the adoration of Siva reciting the\nMrtyu\u00f1jaya-mantra and performed tapas continuously for six months.\n\n\nPleased with his penance, Shiva appeared to Soma and asked for a boon. Soma asked to forgive him and cure from that disease of consumption. Shiva said he would go through period of waning and waxing for period of fortnight continuously.\n\n\u0936\u093f\u0935 \u0909\u0935\u093e\u091a \u092a\u0915\u094d\u0937\u0947 \u091a \u0915\u094d\u0937\u0940\u092f\u0924\u093e\u0902 \u091a\u0928\u094d\u0926\u094d\u0930! \u0915\u0932\u093e \u0924\u0947 \u091a \u0926\u093f\u0928\u0947 \u0926\u093f\u0928\u0947\u0964 \u092a\u0941\u0928\u0936\u094d\u091a\n\u0935\u0930\u094d\u0926\u094d\u0927\u0924\u093e\u0902 \u092a\u0915\u094d\u0937\u0947 \u0938\u093e \u0915\u0932\u093e \u091a \u0928\u093f\u0930\u0928\u094d\u0924\u0930\u092e\u094d\u0964\u0964 \u096a\u096b \u0964\u0964 \n\u015aiva said, \u201cIn a fortnight, you will get reduced by a digit daily and\nin the next fortnight, it would go on increasing in the same\nsequence.\u201d \n\nPleased with devotion of moon and at request of Devas, Shiva then took form of Linga there as Somnathesvara.\n\n\u091a\u0928\u094d\u0926\u094d\u0930\u0938\u094d\u092f \u092f\u0936\u0938\u0947 \u0924\u0924\u094d\u0930 \u0928\u093e\u092e\u094d\u0928\u093e \u091a\u0928\u094d\u0926\u094d\u0930\u0938\u094d\u092f \u0936\u0919\u094d\u0915\u0930\u0903\u0964\u0964 \u096b\u0966\u0965 \u0938\u094b\u092e\u0947\u0936\u094d\u0935\u0930\u0936\u094d\u091a\n\u0928\u093e\u092e\u094d\u0928\u093e\u093d\u093d\u0930\u0940\u0926\u094d\u0935\u093f\u0916\u094d\u092f\u093e\u0924\u094b \u092d\u0941\u0935\u0928\u0924\u094d\u0930\u092f\u0947\u0964 \u0915\u094d\u0937\u092f\u0915\u0941\u0937\u094d\u0920\u093e\u0926\u093f\u0930\u094b\u0917\u093e\u0923\u093e\u0902 \u0928\u093e\u0936\u0915:\n\u092a\u0942\u091c\u0928\u093e\u0926\u094d\u0926\u094d\u0935\u091c\u093e\u0903\u0964\u0964 \u096b\u0967\u0964\u0964 \n\n\nSiva then felt happy with the gods. In order to\nenhance the glory and greatness of the region, besides that of the\nmoon, \u0160iva established himself there in the form of Some\u015fvara and\nbecame famous in the three worlds. O Br\u00e4hmanas, by his worship, \u0160iva\nbecomes the destroyer of the ailments like consumption, leprosy and\nothers. \n\nStory of Somnatha is also described in Chapter 63 Nagara Khanda of Skanda Purana (Pg: 259) and Chapter 22 and 23 Section one of Prabhasa Kandha of Skanda Purana.\nGreatness of Somnatha Linga is described in much detail in Chapter 6 Section one of Prabhasa Kandha of Skanda Purana.\n", "topic": "hinduism", "url": "https://hinduism.stackexchange.com/questions/14213"}, {"image": "VQAonline_00021817.png", "question": "Who is this goddess in the picture", "context": "I found this below pic in instagram. Who is this goddess? Is any mention about this goddess in Hindu Puranas?\n\n", "answer": "Who is this goddess?\n\nPadmavati Devi\n(Ps- Do not confuse her with Padmavati Devi of Tirumala & Tiruchanur, or with the Padmavati Yakshini of Jainism.)\nShe is one of the 700 Mahavidyas or 13 Shaktis of Durga Saptashati or the Angavidya of Navarna Chand\u012b and thus she is the ruling deity of the 6th chapter (Dhumralochana Vadha).\nIs any mention about this goddess in Hindu Puranas?\n\nYes, she appears in Durga Saptashati or Markandeya Purana.\nHer dhyan is mentioned in the 6th chapter of Uttam Charitra, Durga Saptashati, as below-\n\n\u0950 \u0928\u093e\u0917\u093e\u0927\u0940\u0936\u094d\u200d\u0935\u0930\u0935\u093f\u0937\u094d\u091f\u0930\u093e\u0902 \u092b\u0923\u093f\u092b\u0923\u094b\u0924\u094d\u0924\u0902\u0938\u094b\u0930\u0941\u0930\u0924\u094d\u200d\u0928\u093e\u0935\u0932\u0940-\n\u092d\u093e\u0938\u094d\u0935\u0926\u094d\u0926\u0947\u0939\u0932\u0924\u093e\u0902 \u0926\u093f\u0935\u093e\u0915\u0930\u0928\u093f\u092d\u093e\u0902 \u0928\u0947\u0924\u094d\u0930\u0924\u094d\u0930\u092f\u094b\u0926\u094d\u092d\u093e\u0938\u093f\u0924\u093e\u092e\u094d\u0964\n\u092e\u093e\u0932\u093e\u0915\u0941\u092e\u094d\u092d\u0915\u092a\u093e\u0932\u0928\u0940\u0930\u091c\u0915\u0930\u093e\u0902 \u091a\u0928\u094d\u0926\u094d\u0930\u093e\u0930\u094d\u0927\u091a\u0942\u0921\u093e\u0902 \u092a\u0930\u093e\u0902\n\u0938\u0930\u094d\u0935\u091c\u094d\u091e\u0947\u0936\u094d\u200d\u0935\u0930\u092d\u0948\u0930\u0935\u093e\u0919\u094d\u200c\u0915\u0928\u093f\u0932\u092f\u093e\u0902 \u092a\u0926\u094d\u092e\u093e\u0935\u0924\u0940\u0902 \u091a\u093f\u0928\u094d\u0924\u092f\u0947\u0965\nOm, I meditate Upon The Ultimate goddess Padmavathi, who reside in the body of the Lord of all wisdom, Sarjneshwarbhairava. Her body is efflugent with the brilliance of multitude of jewels, she is adorned by the hood of the king of snakes, upon whom she is reclining. Her lustre is like that of the sun and her three eyes are brilliant. In her hands she holds, a rosary, a pitcher, a skull and a lotus, & the radiant half moon is the shining crown upon her head.\n\nThere are numerous tantric references too. An exclusive kavacham is dedicated to Padmavati Devi in the Deviyamal tantra. And she appears in Rudrayamal too. Almost every Devi Sahasranaam has her name, be it of Kali, Dhumavati, Laxmi, Bala, Bhuvaneshwari, Ganga, etc.\nDeviyamala tantra,\n\n\u0924\u094d\u0930\u093f\u0937\u094d\u091f\u0941\u092a \u091b\u0928\u094d\u0926 \u0907\u0924\u093f\u0916\u094d\u092f\u093e\u0924\u0902 \u0926\u0947\u0935\u0940 \u092a\u0926\u094d\u092e\u093e\u0935\u0924\u0940 \u0938\u094d\u092e\u0943\u0924\u093e \u0964\nWith the famous trishtup chanda, meditated upon Devi Padmavati.\n\nRudrayamal tantra,\n\n\u0938\u0902\u092a\u0924\u094d\u0915\u0930\u0940 \u091a \u0938\u093e\u092e\u094d\u0930\u093e\u091c\u094d\u092f\u0932\u0915\u094d\u0937\u094d\u092e\u0940: \u092a\u0926\u094d\u092e\u093e\u0935\u0924\u0940 \u0936\u093f\u0935\u093e\u0964\n\u0926\u0941\u0930\u094d\u0917\u093e \u092d\u0926\u094d\u0930\u093e\u0915\u0943\u0924\u093f\u0903 \u0915\u093e\u0932\u0940 \u0915\u093e\u0932\u0930\u093e\u0924\u094d\u0930\u093f\u0903 \u0938\u0941\u092d\u0926\u094d\u0930\u093f\u0915\u093e\u0964\u0964\n\nSarvajneshwar Bhairava (the all-knower/the lord of all wisdom) is the consort of Padmavati Devi.\nDeviyamala tantra mentions this in a shloka,\n\n\u0936\u094d\u0930\u0940\u0938\u0930\u094d\u0935\u091c\u094d\u091e\u0947\u0936\u094d\u0935\u0930\u0938\u0939\u093f\u0924\u093e \u0936\u094d\u0930\u0940\u092a\u0926\u094d\u092e\u093e\u0935\u0924\u0940\u0926\u0947\u0935\u0924\u093e\nPadmavati devta, along with Shree Sarvajneshwar.\n\nAlso in the above dhyan mantra, this was mentioned,\n\n\u0938\u0930\u094d\u0935\u091c\u094d\u091e\u0947\u0936\u094d\u200d\u0935\u0930\u092d\u0948\u0930\u0935\u093e\u0919\u094d\u200c\u0915\u0928\u093f\u0932\u092f\u093e\u0902 \u092a\u0926\u094d\u092e\u093e\u0935\u0924\u0940\u0902 \u091a\u093f\u0928\u094d\u0924\u092f\u0947\u0965\nI meditate upon Padmavati who resides in the body of Sarvajneshwara Bhairava.\n\n", "topic": "hinduism", "url": "https://hinduism.stackexchange.com/questions/52194"}, {"image": "VQAonline_00021678.png", "question": "What scriptures mention Shiva's Dakshinamurthi form?", "context": "One of the most popular forms of Shiva worshipped in South India is the sage Dakshinamurthi, whose name literally means \"the South-facing form\". Dakshinamurthi is the subject of the famous Dakshinamurthu Stotram, a hymn commonly attributed to Adi Shankarcharya (although this is disputed) which says this:\n\nIt is strange to see the very old disciples and the very young teacher,\n who sit under a banyan tree, with the teacher always observing silence, and the students getting all the doubts cleared.\n\n\nThe story as I understand it is that Shiva incarnated as a young sage named Dakahinamurthi, who sat under a banyan tree and silently imparted wisdom to the Sanatkumaras, four sages who are mind-born sons of Brahma.\nMy question is, what scriptures mention the sage Dakshinamurthi? I assume he's mentioned in various Shaiva Agamas, but it's very hard to date Agamic texts. And he is discussed in the Dakshinamurthi Upanishad, but that's believed by many to be a late text.\nIs he discussed in the Puranas? All I've found is a passing reference in this excerpt from the Narada Purana, which says that a hymn to Shiva called the Shiva Nyasa was heard from the gods by Dakshinamurthi:\n\nThe deity of this (nyasa) is Ardhanarisha, the sage Dakshinamurti and the metre is Gayatri. The application of this mantra is for attaining everything.\n\nI suppose it's akin to how the sage Narayana, an incarnation of Vishnu, was the seer of mantras to Vishnu like the Purusha Sukta of the Rig Veda, which I discuss here.\nAlso, this forum post claims that Dakshinamurthi is discussed in the \"Suta Samhita\" of the Skanda Purana, but the Motilal Banarsidass translation of the Skanda Purana is divided into Khandas, not Samhitas, so I'm not sure what part of the text constitutes the Suta Samhita.\nDoes anyone know any other scriptures which discuss Dakshinamurthi?\n", "answer": "Dakshinamurty form of Lord Shiva is described in Suta Samhita of Skanda Purana in Mochak Kathanam of Mukti Khanda. Here is the description of form:\n\n\u0917\u0921\u093c\u094d\u0917\u093e\u0927\u0930\u0902 \u0936\u093f\u0935\u0902 \u0936\u093e\u0928\u094d\u0924\u0902 \u0932\u0938\u0924\u094d\u0915\u0947\u092f\u0942\u0930\u092e\u0923\u094d\u0921\u093f\u0924\u092e\u094d \u0964\n \u0938\u0930\u094d\u0935\u093e\u092d\u0930\u0923\u0938\u0902\u092f\u0941\u0915\u094d\u0924\u0902 \u0938\u0930\u094d\u0935\u0932\u0915\u094d\u0937\u0923\u0938\u0902\u092f\u0941\u0915\u094d\u0924\u092e\u094d \u0964\u0964 \n\u0935\u0940\u0930\u093e\u0938\u0928\u0947 \u0938\u092e\u093e\u0938\u0940\u0928\u0902 \u0935\u0947\u0926\u092f\u091c\u094d\u091e\u094b\u092a\u0935\u0940\u0924\u093f\u0928\u092e\u094d \u0964\n \u092d\u0938\u094d\u092e\u093e\u0927\u093e\u0930\u093e\u092d\u093f\u0930\u093e\u092e\u0902 \u0924\u0902 \u0928\u093e\u0917\u093e\u092d\u0930\u0923\u092d\u0942\u0937\u093f\u0924\u092e\u094d \u0964\u0964\n \u0935\u094d\u092f\u093e\u0918\u094d\u0930\u091a\u0930\u094d\u092e\u093e\u092e\u094d\u092c\u0930\u0902 \u0936\u0941\u0926\u094d\u0927\u0902 \u092f\u094b\u0917\u092a\u091f\u094d\u091f\u093e\u0935\u0943\u0924\u0902 \u0936\u0941\u092d\u092e\u094d \u0964\n \u0938\u0930\u094d\u0935\u0947\u0937\u093e\u0902 \u092a\u094d\u0930\u093e\u0923\u093f\u0928\u093e\u092e\u093e\u0924\u094d\u092e\u093e\u091c\u094d\u091e\u093e\u0928\u093e\u092a\u0938\u094d\u092e\u093e\u0930\u092a\u0943\u0937\u094d\u0920\u0924 \u0964\u0964\n \u0935\u093f\u0928\u094d\u092f\u0938\u094d\u0924\u091a\u0930\u0923\u0902 \u0938\u092e\u094d\u092f\u091c\u094d\u091e\u093e\u0928\u092e\u0941\u0926\u094d\u0930\u093e\u0927\u0930\u0902\u0939\u0930\u092e\u094d \u0964\n \u0938\u0930\u094d\u0935\u0935\u093f\u091c\u094d\u091e\u093e\u0928\u0930\u0924\u094d\u0928\u093e\u0928\u093e\u0902 \u0915\u094b\u0936\u092d\u0942\u0924\u0902 \u0938\u0941\u092a\u0941\u0938\u094d\u0924\u0915\u092e\u094d \u0964\u0964\n \u0926\u0927\u093e\u0928\u0902 \u0938\u0930\u094d\u0935\u0924\u0924\u094d\u0935\u093e\u0915\u094d\u0937\u092e\u093e\u0932\u093f\u0915\u093e\u0902 \u0915\u0941\u0923\u094d\u0921\u093f\u0915\u093e\u092e\u092a\u093f \u0964\n \u0938\u094d\u0935\u093e\u0924\u094d\u092e\u092d\u0942\u0924 \u092a\u0930\u093e\u0928\u0928\u094d\u0926 \u092a\u0930\u093e\u0936\u0915\u094d\u0924\u094d\u092f\u0930\u094d\u0927\u0935\u093f\u0917\u094d\u0930\u0939\u092e\u094d \u0964\u0964\n \u0927\u0930\u094d\u092e\u0930\u0941\u092a \u0935\u0943\u0937\u094b\u092a\u0947\u0924\u0902 \u0927\u093e\u0930\u094d\u092e\u093f\u0915 \u0935\u0947\u0926\u092a\u093e\u0930\u0917\u0948 \u0964\n\u092e\u0941\u0928\u093f\u092d\u093f: \u0938\u0902\u0935\u0943\u0924\u0902 \u092e\u093e\u092f\u093e\u0935\u091f\u092e\u0942\u0932\u093e\u0936\u094d\u0930\u093f\u0924\u0902 \u0936\u0941\u092d\u092e\u094d \u0964\u0964\n \u0907\u0936\u093e\u0928\u0902 \u0938\u0930\u094d\u0935\u0935\u093f\u0926\u094d\u092f\u093e\u0928\u093e\u092e\u0940\u0936\u094d\u0935\u0947\u0930\u0936\u094d\u0935\u0930\u092e\u0935\u094d\u092f\u092f\u092e\u094d \u0964\n \u0909\u0924\u094d\u092a\u0924\u094d\u0924\u094d\u092f\u093e\u0926\u093f\u0924\u093f\u0928\u093f\u0930\u094d\u092e\u0941\u0915\u094d\u0924\u092e\u094b\u0902\u0915\u093e\u0930\u0915\u092e\u0932\u093e\u0938\u0928\u092e\u094d \u0964\u0964\n\u0938\u094d\u0935\u093e\u0924\u092e\u0935\u093f\u0926\u094d\u092f\u093e\u092a\u094d\u0930\u0926\u093e\u0928\u0947\u0928 \u0938\u0926\u093e \u0938\u0902\u0938\u093e\u0930\u092e\u094b\u091a\u0915\u092e\u094d \u0964\n \u0930\u0941\u0926\u094d\u0930\u0902 \u092a\u0930\u092e\u0915\u093e\u0930\u0941\u0923\u094d\u092f\u093e\u0924\u094d\u0938\u0930\u094d\u0935\u092a\u094d\u0930\u093e\u0923\u093f\u0939\u093f\u0924\u0947\u0930\u0924\u092e\u094d \u0964\u0964\n \u0909\u092a\u093e\u0938\u0915\u093e\u0928\u093e\u0902 \u0938\u0930\u094d\u0935\u0947\u0937\u093e\u092e\u092d\u0940\u0937\u094d\u091f\u0938\u0915\u0932\u092a\u094d\u0930\u0926\u092e\u094d \u0964\n\u0926\u0915\u094d\u0937\u093f\u0923\u093e\u092e\u0942\u0930\u094d\u0924\u093f\u0926\u0947\u0935\u093e\u0916\u094d\u092f\u0902 \u091c\u0917\u0924\u094d\u0938\u0930\u094d\u0917\u093e\u0926\u093f\u0915\u093e\u0930\u0923\u092e\u094d \u0964\u0964\n \u0938\u092e\u093e\u0917\u0924\u094d\u092f \u092e\u0939\u093e\u092d\u0915\u094d\u0924\u094d\u092f\u093e \u0926\u0923\u094d\u0921\u0935\u0924\u094d\u092a\u0943\u0925\u093f\u0935\u0940\u0924\u0932\u0947 \u0964\n \u092a\u094d\u0930\u0923\u092e\u094d\u092f \u092c\u0939\u0941\u0936\u094b\u0926\u0947\u0935\u0902 \u0938\u092e\u093e\u0930\u093e\u0927\u094d\u092f \u092f\u0925\u093e\u092c\u0932\u092e\u094d \u0964\u0964\n \u0930\u0941\u0926\u094d\u0930 \u092f\u0924\u094d\u0924\u0947 \u092e\u0941\u0916\u0902 \u0924\u0947\u0928 \u0926\u0915\u094d\u0937\u093f\u0923\u0902\u092a\u093e\u0939\u093f \u092e\u093e\u092e\u093f\u0924\u093f \u0964\n \u0909\u0915\u094d\u0924\u094d\u0935\u093e \u092a\u0941\u0928\u0903 \u092a\u0941\u0928\u0930\u094d\u0926\u0947\u0935\u0902 \u092a\u0942\u091c\u092f\u093e\u092e\u093e\u0938 \u092d\u0915\u094d\u0924\u093f\u0924\u0903 \u0964\u0964\n \u092a\u0941\u0928\u0930\u094d\u0926\u0947\u0935\u094b \u092e\u0939\u093e\u0926\u0947\u0935\u094b \u0926\u0915\u094d\u0937\u093f\u0923\u093e\u092e\u0942\u0930\u094d\u0924\u093f\u0930\u0940\u0936\u094d\u0935\u0930: \u0964\n \u092a\u094d\u0930\u0926\u0924\u094d\u0924\u094d\u0935\u093e \u0938\u094d\u0935\u093e\u0924\u094d\u092e\u0935\u093f\u091c\u094d\u091e\u093e\u0928 \u0924\u0938\u094d\u092e\u0948 \u0935\u093f\u092a\u094d\u0930\u093e\u092f\u0938\u0941\u0935\u094d\u0930\u0924\u093e \u0964\u0964\n \u0924\u0938\u094d\u092f \u0938\u0902\u0938\u093e\u0930\u0935\u093f\u091a\u094d\u091b\u0947\u0926\u092e\u0915\u0930\u094b\u0926\u092e\u094d\u092c\u093f\u0915\u093e\u092a\u0924\u093f\u0964 \n \u092c\u0939\u0935\u094b \u0926\u0915\u094d\u0937\u093f\u0923\u093e\u092e\u0942\u0930\u094d\u0924\u093f \u092a\u094d\u0930\u0938\u093e\u0926\u093e\u0926\u0947\u0935 \u091c\u0928\u094d\u0924\u0924\u0903 \u0964\u0964 \n The one bearing Ganga in his head, Shiva; is silent, with his hairs tied. He can give anything to anyone and he is full of all attributes. He is sitting in Virasana Position and wearing a Vedic Yajnopavita. He is covered by the ashes and his ornaments are snakes. He is sitting in tigerskin made for sitting in Yogic position. He is giving AtmaJnana to all creatures through silence. His left foot is in right thigh and his hand is on JnanaMudra. He is the embodiment of all jewels of knowledge. He is giving all tatwa just through his eyes. He himself is in fully trancsendental blissful state, transcending even Shakti. Sages which are like embodiment of Dharma tree and who have transcended even Vedas, they are sitting around that Banayan tree. Ishana himself is the all knowledge, he is the Lord and also unmanifested, He manifests just by the sound Om, the one who is sitting in Lotus. Providing knowledge of Self he is always the liberator of self. Such is Rudra and his compassion who always wants good for creatures. The one who do Austerity for him is sure to have fulfilled his desires, He is that Dakshinamurty divine and is the cause of this whole world. The great devotees salute him by keeping head on earth. I salute him and adore him with my limited abilities. Rudra I surrender to your form which is facing south. I worship again and again through my devotion. Oh Lord Mahadeva in the form of Dakshinamurti, you provide knowledge of self, thus intelligent worship you. Thus for liberating creatures from this cycle of Samsara, oh Lord of Ambika, you take the form of Dakshninamurti being like a fruit for all.\n\nLord Dakshinmurty is also described briefly in Lalita Sahasranama of Brahmanda Purana. There Shakti is called Dakshinmurty Rupini as Shakti is Shiva herself and Shiva just remains in unmanifested state without Shakti; (it is actually through Shakti Shiva assumes forns:)\n\n\u0938\u094d\u0935\u0924\u0928\u094d\u0924\u094d\u0930\u093e \u0938\u0930\u094d\u0935\u0924\u0928\u094d\u0924\u094d\u0930\u0947\u0936\u0940 \u0926\u0915\u094d\u0937\u093f\u0923\u093e\u092e\u0942\u0930\u094d\u0924\u093f \u0930\u0941\u092a\u093f\u0923\u0940 \u0964\n \u0938\u0928\u0915\u093e\u0926\u093f \u0938\u092e\u093e\u0930\u093e\u0927\u094d\u092f\u093e \u0936\u093f\u0935\u091c\u094d\u091e\u093e\u0928 \u092a\u094d\u0930\u0926\u093e\u092f\u0940\u0928\u093f \u0964\u0964140\u0964\u0964 \n You are independent. You are the presiding deity over all Tantras. You are the who takes form of Dakshinamurty who is adored by Sanaka and other sages and who provides knowledge of Supreme.\n\nThe form resembling Dakshinamurty form is also described in Srimad Bhagvatam from here although the name Dakshinmurty is not used:\n\nS.B.4.6.31,32 (Banayana tree); S.B.4.6.33 (Lord Shiva sitting under that tree); S.B.4.6.34 (Surrounded by Sanata Kumaras and other Saints and Silent), S.B.4.6.35 (Lord Shiva in full perfection); S.B.4.6.36 (Virasana and hand in Tarka mudra)\n\nA mantra adressing Lord Dakshinamurty is also in the Narada Purana (III.91) and Dakshinamurty is also described there:\n\nDakshinamurtaye should be uttered first, then homage unto you who reside at the root of the banayan tree, obeisance unto him who is deeply engrossed in meditation, then to Rudra and Sambhu. T\u00e3ra (Om) and Hrim should enclose the mantra. The mantra consists of thirty six syllables. It is the bestower of all desired gods. Suka is the sage and Anushtup is the metre. The deity is Sambhu Dakshin\u00e3murti. There is a lofty banyan tree on the ridge of the Him\u00e3layas frequented by Siddhas and Kinnaras. The tree has fruits shining like rubies. It has shadowed sunlight by its far extending branches. The full-blown trees are embraced by the clusters of creepers in full bloom. The place has been rendered cool by the winds blowing over the mountain streams Howing out of the rock crevasses. Groups of cdestial damels sing while the flocks of peacocks dance. Different quarters are resonant by the cooing of cuckoos. The region is frequented by the animals which have eschewed their mutual animosity. It is beautified by fragrant flowers of both aquatic and terrestrial origin. lt is conti- nuously and happily resorted to by the sages, Suka and others. It is frequently supervised by the gods, chief of whom is Indra and who are accompanied by their womenfolk and others. The banyan tree is beautilied by the thickly grown emerald-like leaves. It is embellished as though with hanging ornaments made of the nine precious gems. It is a wonderful tree the shade of which is capable of dispelling worldly distress. Shiva is seated at its root in an auspicious throne set beautifully, studded with jewels. His embellishments are immeasurable. His face resembles the autumnal moon. He resembles the mountain Kailasa, has three eyes and the crescent moon decorating his matted hair. He is seated in the heroic posture. He holds the posture of the hatchet, deer and has placed the delicate hand on the knee. He has kept a serpent firmly fixed under his arm pit. He should meditate on Siva in his pleasant mood. \n\n", "topic": "hinduism", "url": "https://hinduism.stackexchange.com/questions/7896"}, {"image": "VQAonline_00021747.png", "question": "Clarification about Kunti's statement on harlot", "context": "This answer says that\n\n\"Swairini (heanton)\" means \"Unbound\" & Harlot means \"prostitute\".\n\nAs per Adi Parva: Sambhava Parva of Mahabharata:\n\nThe celebrated Pandu, tempted by the desire of having more children\nwished to speak again unto his wedded wife (for invoking some other\ngod).\nBut Kunti addressed him, saying,\n'The wise do not sanction a fourth\ndelivery even in a season of distress. The woman having intercourse\nwith four different men is called a Swairini (heanton), while she\nhaving intercourse with five becometh a harlot. Therefore, O learned\none, as thou art well-acquainted with the scripture on this subject,\nwhy dost thou, beguiled by desire of offspring, tell me so in seeming\nforgetfulness of the ordinance?'\"\n\n\nNote: In this Hindi translation, it seems that Kunti is referring about having sons from different men as words \"is vidhi ke dvara\" are used. Also, there seems no limit of having only three or four sons as per scriptures. Many women had more than three/four sons. For example, Rukmini had more than 3/4 sons: Pradyumna, Charudeshna, Sudesna, Charudeha, Sucharu, Charugupta, Bhadracharu, Charuchandra, Vicharu and Charu.\nBut the count was already five :\n\nSurya\nPandu\nDharma\nVayu\nIndra\n\nI think no reference is needed for last three because Kunti must have referred them. Also, Pandu was cursed later, so he must have been in the list of four men whom Kunti referred else he must have given the reason to Kunti about his request of producing more children from other gods.\nAnd here is the reference for Surya:\n\nThe god Vivaswat (Sun) approaching her, said, 'Here I am, O black-eyed girl! Tell me what I am to do for thee.'\n\"Hearing this, Kunti said, 'O slayer of foes, a certain Brahamana gave me this formula of invocation as a boon, and, O lord, I have summoned thee only to test its efficacy. For this offence I bow to thee. A woman, whatever be her offence, always deserveth pardon.'\nSurya (Sun) replied, 'I know that Durvasa hath granted this boon. But cast off thy fears, timid maiden, and grant me thy embraces. Amiable one, my approach cannot be futile; it must bear fruit. Thou hast summoned me, and if it be for nothing, it shall certainly be regarded as thy transgression.'\n\"Vaisampayana continued, 'Vivaswat thus spoke unto her many things with a view to allay her fears, but, O Bharata, the amiable maiden, from modesty and fear of her relatives, consented not to grant his request.\nAnd, O bull of Bharata's race, Arka addressed her again and said, 'O princess, for my sake, it shall not be sinful for thee to grant my wish.' Thus speaking unto the daughter of Kuntibhoja, the illustrious Tapana--the illuminator of the universe--gratified his wish. And of this connection there was immediately born a son known all over the world as Karna accountred with natural armour and with face brightened by ear-rings. -Adi Parva: Sambhava Parva of Mahabharata\n\nThen why didn't Kunti say this statement before Arjuna's birth?\nOne possible reason that seems logical is:\n\nAnd after the birth of this child (Karna), the illustrious Tapana (Surya) granted unto\nPritha (birth name of Kunti) her maidenhood and ascended to heaven. -Adi Parva: Sambhava Parva of Mahabharata\n\nThis seems logical because this was the very reason which proved Karna's statement on Draupadi false and made him villain as Draupadi was blessed to gain her virginity.\nIs there any other reason for which Kunti didn't include Surya?\nOr as mentioned in this comment of Keshav Srinivasan \"I think it's posssible Pandu never had intercourse, and that she only had intercourse with Surya, Yama, Indra, and Vayu.\"\nIs it true that Pandu never had intercourse with Kunti? If so, was Pandu aware of that Karna was son of Kunti and Surya?\n", "answer": "It refers to giving birth and not about having intercourse. Secondly, she is referring to the Niyoga padati. So, that way its only 3. We can't consider Surya here, since he had granted a boon to her that her virginity (Kanyatva) will not change.\nYou can find it in Vyasa Mahabaratha - Chapter 111 - Verse 20 - MN Dutt Translation\n\nThe greatly effulgent Tapana, then giving Pritha her maiden-hood,\n again went to heaven. \n\nIn that case it would only be Yama, Vayu and Indra.\n", "topic": "hinduism", "url": "https://hinduism.stackexchange.com/questions/23413"}, {"image": "VQAonline_00022030.png", "question": "What was the primary motivation for a historical figure like Xenophon to create an extensive collection of written material?", "context": "Xenophon of Athens created a large amount of written material even by today's standards. Did he create this work knowing it would be read by future generations, or was it for his own satisfaction? I could imagine figures like Socrates, Aristotle, or Herodotus might expect their work to be copied many times, but maybe not? \nIs this similar to academia, where a document or book might be highly useful to a smaller audience? Even in this case, the physical effort to create and copy the work seems trivial compared to the same with Papyrus and copying by hand.\n\n", "answer": "Xenophon gave specific reasons for some of his works but for others he did not.\nXenophon (about 431 BC to 354 BC) produced a very wide range of work during his lifetime: historical, biographical, philosophical, instructional. He never stated a primary purpose for all his works and we can deduce that some of what he wrote was aimed at specific audiences. \nFor some of his works, he states a purpose (see below). For others we can sometimes deduce his intent with a certain degree of confidence, but with other texts we cannot be sure.\nThe historical works Anabasis and Hellenika, are the most problematic for, as John Marincola states in The Cambridge Companion to Xenophon,\n\nXenophon does not give us much help in interpreting these works. No\n major ancient historian is more reticent about the nature and purpose\n of his history...\n\nAs Xenophon read the works of some of those who came before him, he would certainly would have expected (and intended) that what he wrote would be copied and read by others. From the way that Xenophon returns to certain themes (such as leadership) in different works, we can deduce that he was perhaps partly motivated by what interested him and what he saw as important. \n\nWORKS WHERE XENOPHON STATES A REASON\nAgesilaos\nIn Agesilaos, (King Agesilaos II of Sparta, ruled c. 398 \u2013 c. 360 BC), Xenophon begins with:\n\nI know how difficult it is to write an appreciation of Agesilaus that\n shall be worthy of his virtue and glory. Nevertheless the attempt must\n be made. For it would not be seemly that so good a man, just because\n of his perfection, should receive no tributes of praise, however\n inadequate.\n\nThis laudatory account of Agesilaos is in contrast to some parts of Hellenika (see below) where Agesilaos is at times criticized.\nSymposium\nIn the Socratic dialogue Symposium, Xenophon, through a narrator, says:\n\nTo my mind it is worth while to relate not only the serious acts of\n great and good men but also what they do in their lighter moods. I\n should like to narrate an experience of mine that gives me this\n conviction.\n\nApology and Memorabilia\nThe Socratic dialogues Apology and Memorabilia are both defences of Socrates but are aimed at different audiences. In Apology, his intent was to make explicit something about Socrates' defence which he felt other writers hadn't:\n\nIt seems to me fitting to hand down to memory, furthermore, how\n Socrates, on being indicted, deliberated on his defence and on his\n end. It is true that others have written about this, and that all of\n them have reproduced the loftiness of his words,\u2014a fact which proves\n that his utterance really was of the character intimated;\u2014but they\n have not shown clearly that he had now come to the conclusion that for\n him death was more to be desired than life; and hence his lofty\n utterance appears rather ill-considered.\n\nThus, Apology is aimed at an audience which is already 'sympathetic' to Socrates. In contrast,\n\nThe audience Xenophon imagines for the Memorabilia , on the other\n hand, presumably includes readers still open to the attacks against\n Socrates; otherwise Xenophon would not spend so much time summarizing\n and refuting such arguments.\n\nConstitution of the Lacedaimonians\nXenophon's pro-Spartan sympathies frequently show through in his writing (see Hellenika, for example), though he is not entirely uncritical. In this treatise on the Spartan constitution, Xenophon started with:\n\nIt occurred to me one day that Sparta, though among the most thinly\n populated of states, was evidently the most powerful and most\n celebrated city in Greece; and I fell to wondering how this could have\n happened. But when I considered the institutions of the Spartans, I\n wondered no longer.\n\nXenophon, like many Greek writers, was concerned with good government and leadership. In Constitution, he praises Spartan institutions and aims to show how Sparta became a great power, but he also criticizes Spartans for not following their laws in his own time. Thus, he seems to be pointing to Sparta's decline, even before the catastrophic defeat at the Battle of Leuctra in 371 BC\n\nWORKS WHERE XENOPHON DOES NOT CLEARLY STATE A REASON\nCyropaedia\n(see also the section below on Anabasis) This multi-genre text is Xenophon's longest work and focuses on Cyrus the Great (although much of it is fiction) but it was intended neither as a history nor a biography but rather as a thesis on the training of a ruler. Cyropaedia is\n\nthe most enigmatic with regards to the author\u2019s intentions....Xenophon\n sets out to narrate certain noteworthy incidents and exceptional deeds\n (military and political) from the life of Cyrus the Great.... What he found most admirable and\n underlines already in the prologue of this work is the fact that Cyrus\n managed to rule a great empire with the willing obedience of his\n followers. The Cyropaedia is thus presented as having serious claims\n to contribute to the political discourse of Xenophon\u2019s time. It is no\n wonder that in antiquity it was considered a response to Plato\u2019s\n Republic...\n\nWhy did Xenophon choose a Persian, Cyrus? Melina Tamiolaki, in The Cambridge Companion to Xenophon argues that:\n\nAt least two reasons can be advanced to explain this choice: firstly,\n Xenophon was interested in paradigms of successful empires; obviously\n he could not find such paradigms in Greece, which experienced\n consecutive failures of empires. Secondly, he must have been impressed\n by specific features on which the propaganda about Cyrus insisted,\n mainly his qualities as a benevolent despot and his success in gaining\n the willing obedience of his followers. Greek leaders ostensibly\n lacked these qualities, for the simple reason that the regime of\n monarchy that by definition enables and fosters them, was absent in\n Greece. Xenophon wished to underline these qualities, but without\n implying a suggestion about constitutional change in Greece.\n\nHellenika\nHellenika is a continuation of Thucydides' work on the Peloponnesian War and then beyond. His style reveals that he had clearly read both Herodotus and Thucydides. He begins Hellenika with \"And after those things, not many days later...\". David Thomas, in his introduction to The Landmark Xenophon's Hellenika, observes:\n\nXenophon begins this way, expecting the reader to recognize that he is\n picking up roughly where Thucydides left off....Xenophon...is in some\n sense claiming to be a historian who can be compared to Thucydides.\n\nThucydides states why he wrote his work. It was because he thought\n\n\"it would be a great war, and more worthy of relation than any other\n that had preceded it\"\n\nXenophon may have felt the same way, particularly as he simply continues from where Thucydides left off. What else might have given him reason to write Hellenika we don't know with any certainty but Thomas suggests that Xenophon wished to put his own 'slant' on events, which might account for the differences between Xenophon's Hellenika and the Hellenica Oxyrhynchia (unknown author).\nAnabasis\nThis is similarly problematic in terms of motive. Perhaps he felt that the march of 10,000 Greeks through the heart of the mighty Persian empire was too good a story not to tell, but there is almost certainly more to it than that. In Anabasis, the theme of Leaders and Followers is evident, as it was in Cyropaedia, and Xenophon's desire to make known his thoughts on what he personally saw as something very important may well have been an important (but not only) reason for writing both Anabasis and Cyropaedia.\n\nOTHER WORKS\nHipparchicus, On Horsemanship and Hunting with Dogs\nHipparchicus, On Horsemanship and Hunting with Dogs can all be considered technical treatise. They are instructional, but that does not appear to be the only reason that Xenophon wrote them. Hipparchicus, for example, sees Xenophon once again dealing with leadership, while Hunting with Dogs\n\nis a definite outlier in Xenophon\u2019s corpus of smaller works and a\n difficult text. It is made up of three distinct parts: an elaborate,\n mythological preface; an extensive attack upon the sophists at the\n end; in between, a fairly straightforward practical section concerning\n hunting.\n\nSource: John Dillery (Chapter 10), in The Cambridge Companion to Xenophon\n", "topic": "history", "url": "https://history.stackexchange.com/questions/52680"}, {"image": "VQAonline_00022002.png", "question": "Was Col. John K. Waters (Gen. Patton's son-in-law) wounded in the buttocks or the stomach during Patton's raid on Oflag XIII-B near Hammelburg?", "context": "Background:\nI have researched Patton's raid on Oflag XIII-B near Hammelburg March 26-28, 1945 using the following four sources:\n\nWiki Article on Task Force Baum\nWar History\nOnline's article: The Real Fury: Patton's Disastrous 1945 Raid to\nRescue His Son-in-Law\nWarfare History Network's article: Top Secret Missions:\nLiberating General George S. Patton's Son-in-Law\nDon Moore's War Tales: Harry Long was a POW with Patton's\nson-in-law \n\nSummary:\nIn late March 1945, claiming he was worried about the Germans executing American POWs (ostensibly in the wake of the Malmedy massacre), General George S. Patton ordered a raid on Hammelburg's Oflag XIII-B POW camp to liberate the American officers imprisoned there. However, his real intent (based in part on letters to his family) was likely to rescue his son-in-law, Col. John K. Waters, who was a POW there. The assignment was given to Lt. Col. Creighton Abrams (whom the American M1 Abrams tank is now named after) of Combat Command B in the 4th Armored Division. Abrams wanted to proceed with a batallion or regimental sized force but Patton granted only a much smaller force for the rescue mission. Abrams could not go himself due to illness, so Capt. Abraham Baum was given the assignment and about a company of medium and light tanks and 300 infantry to penetrate deep behind enemy lines, liberate the camp, and return the POWs to safety.\nTask Force Baum encountered heavy fighting on the way and suffered many casualties and lost several tanks and vehicles, but made it to the POW camp. During the confusion of the battle at the camp, Col. John K. Waters (Patton's son-in-law) was seriously wounded when shot by a German guard, and could not be evacuated with the rest of the POWs when the camp was finally liberated. The nature of the wound gives rise to my question below. (The initial liberation of the camp is not the end of the story, there was worse to come for Task Force Baum and the POWs, but to get to my question\u2026)\nQuestion:\nWas Col. Waters wounded in the buttocks, or in the stomach? The first two sources above indicate the buttocks. The second two sources indicate the stomach. I cannot find a source that indicates multiple wounds (or a single hit with separate entry and exit wounds) so I don't think it is a case of both being true (though I grant that it is a possibility). Is there a superior information source which could clear up this discrepancy? I know it's only a minor detail in the larger picture of this incident, but I would like to know which account (if any) is accurate regarding the nature of Col. Waters' wound(s) in the raid.\nSpoiler (for the curious):\nMost of the POWs and Baum's task force were recaptured within a day following the raid, and taken back to Oflag XIII-B. Patton, who had not received permission from his superiors in Army Group 12 for the raid, was in trouble and faced Eisenhower's wrath. Patton deftly softened the blow of the tactically botched raid by pointing to the strategic success it offered through diverting large German forces away from the main body of Third Army, thus allowing an easier progression toward their more strategically important objectives. Ike was evidently mollified enough to not officially reprimand Patton for the incident.\n\nGen. George S. Patton visits his son-in-law Col. John K. Waters April 7, 1945 in Frankfurt hospital. Source: Wiki Commons\n", "answer": "In The Patton Papers: 1940-1945, Martin Blumenson cites the journal of Hobart R. Gay's journal entry for April 7th. Colonel (later General) Gay was Patton's Chief of Staff. The description is quite detailed and perhaps explains the conflicting accounts:\n\nCol. Odom returned with two cub planes, bringing Waters, shot through\n leg, bullett coming up through his buttocks and injuring his spine.\n Condition good, will live and probably not be paralyzed.\n\nAlso in April, Patton wrote to his daughter Beatrice (Waters' wife) with a more technical and specific description, saying the bullet struck Colonel Waters\n\nin the left groin but below the peritoneal cavity. The bullet went\n through the rectum, knocked the end off his coccyx, and came out his\n left hip...\n\nThe coccyx \"is the final segment of the vertebral column\".\n", "topic": "history", "url": "https://history.stackexchange.com/questions/47847"}, {"image": "VQAonline_00021911.png", "question": "What are the height units of contours on this WW2 topographic map?", "context": "What are the units seen on the contours and spot heights of this WW2 topographic map? I'm not sure if the map is of Russian or German origin: I've seen the same style and map symbols on Russian army maps and German army maps (they are often the same apart from transliteration between latin and cyrillic alphabets). Although this particular map was certainly used by the German army - there are annotations in German on it of military relevance.\nThe major contours (every 5th contour line) are labelled as 137, 154, 171. The spot heights show as e.g. 166,6 (comma being used as the decimal seperator in German, for instance) make me suspect that that it is 166.6 meters, but obviously the contour heights would be rather peverse in that case - each line is 3.4 meters. So presumably the original contours are based on some other unit system.\nAre the numbers here in meters? And what units have been used when the map was originally created?\n\n", "answer": "Those measurements are in meters. The area around \u0141agusz\u00f3w is about 160 meters above sea level.\nI do not see the divisions of 3.4 meters to which you are referring on the map. However, in old Germany there was local land measure called the rod (\"ruthe\") which was highly irregular and varied from place to place and could be anywhere from 3 to 5 meters in length. The Prussian standard Ruthe was 3.466m, but as I said, in actuality for a given place the Ruthe could vary considerably.\n", "topic": "history", "url": "https://history.stackexchange.com/questions/28544"}, {"image": "VQAonline_00022064.png", "question": "Who is this Asian figure?", "context": "I\u2019m looking for help in identifying this figure. I bought this box at a thrift store a few years back because it was silver and because it looked nice. Every once in a while, curiosity hits me as I look at it and wonder who the figure on the top is. There is no text or trademark on the box, only a very small line of text at the bottom that says \u201cSilver .925\u201d\n\nThe figure looks Asian, and it\u2019s art style is that of Chinese porcelain I\u2019ve seen in stores. The porcelain looks like it has been taken from a larger picture, since there is a small bit of paint at the left side that is abruptly cut off. I\u2019m very curious as to who this figure is, as he seems very iconic, but my historical knowledge is not so good. He\u2019s got what seems to be a sword on his back, a flag (possibly) in his left hand, and a long cloak-like outfit. Somewhat iconic hat, expression, and outfit.\nAny ideas would be much appreciated! Thanks in advance. =D\n", "answer": "It's a figure of Lu Dongbin (also spelled Lu Tung-Pin), one of the Eight Immortals of Taoism.\nQuoted from Learn Religions:\n\nLu Dongbin\u2019s emblem is the magic two-edged sword, which dispels evil spirits, and gives him the power of invisibility. He\u2019s also frequently shown carrying a fly-whisk and is dressed and honored as a scholar. He\u2019s known for being a \u201cladies\u2019 man\u201d and for being prone to bouts of drunkenness.\n\n", "topic": "history", "url": "https://history.stackexchange.com/questions/62918"}, {"image": "VQAonline_00022062.png", "question": "Why are pre-Harappan strata grouped together with Early Harappan strata? Does this not obfuscate the development of the civilization?", "context": "(1) I don't understand why some sites are called early Harappan because these sites are pre-Harappan so why we use wrong name with that. Can any one explain?\nFrom 'India's Ancient Past' by RS Sharma, Ch. 9: Chalcolithic Cultures, p67:\n\nChronologically, there are several series of Chalcolithic settlements in India. Some are pre-Harappan, others are contemporaneous with the Harappan culture, and yet others are post-Harappan.\nPre-Harappan strata on some sites in the Harappan zone are also called early Harappan in order to distinguish them from the mature urban Indus civilization.\nThus, the pre-Harappan phase at Kalibangan in Rajasthan and Banawali in Haryana is distinctly Chalcolithic. So too is the case with Kot Diji in Sindh in Pakistan. Pre-Harappan and post-Harappan Chalcolithic cultures and those coexisting with the Harappan have been found in northern, western, and central India.An example is the Kayatha culture c. 2000\u20131800 BC, which existed towards the end of the Harappan culture. It has some pre-Harappan elements in pottery, but also evidences Harappan influence. Several post-Harappan Chalcolithic cultures in these areas are influenced by the post-urban phase of the Harappan culture.\n\n(2) Harappan civilization has the following phases:\n(1) pre-Harappan\n\n(2) Early Harappan\n(3) Mature Harappan\n(4) Late Harappan \n\n(5) post-Harappan \n\nSo, when excavation begin, in the first soil layers we get post-Harappan culuture, and then Late, then pre, and so on.\nBut, if we combine pre-Harappan layers into Early Harappan (as above), we lose the distinction between the developing and the developed state of the civilization. Does this not lose information that would tell us about how the valley developed?\n(3) and here, Harappan zone means area around the Harappan city, am I right??\n(4) I have also provided diagram what I understand until now. Is this correct?\n\n", "answer": "The confusion here seems to stem from the use of a combined pre-Harappan/early Harappan terminology. We can see this usage repeated in the Wikipedia article for the Indus valley Civilization, in the table on Chronology. If we look at the meaning and locations of these terms it might be more clear.\nThe pre-Harappan era is the oldest, and seems to mainly occur at the Mehrgarh site. One of the distinctions of this culture would be that it is aceramic Neolithic, before pottery was in use here.\nThe next step, also found at the Mehrgarh site, but at different levels, is Early Harappan. One distinction here is that pottery is now in use.\nThe distinction between these two eras being the lack of pottery would imply a certain uncertainty when excavating a site such as Mehrgarh. The layers you draw are not quite that clear-cut, and if you find no pottery when digging, is it pre-Harappan (pre-pottery) or just an area where you were unlucky and didn't find any pottery samples.\nA safer label therefore would be the combined term of pre-Harappan/Early Harappan. Some regions with pottery represented, some without, but age indications placing the site definitely before the more developed culture labeled as the Mature Harappan.\nSometimes it is more accurate when describing something to use a 'fuzzy' definition then a more specific one which can't be supported by the evidence. (In genealogy, for instance, we often describe something undocumented as Before this date, or After another date-expressing uncertainty but giving information concerning what we may be sure of.)\n", "topic": "history", "url": "https://history.stackexchange.com/questions/62159"}, {"image": "VQAonline_00022089.png", "question": "How long has British women royals' solemn ceremonial dress included a sword?", "context": "In this photograph:\n\nfrom the recent funeral service of Queen Elizabeth II in London, England, in the row of attendees nearest to the queen's coffin, three attendees are wearing blue sashes. According to BBC News the three people wearing blue sashes are the King, the Princess Royal, and the Earl of Wessex. I notice that these people with blue sashes, including the Princess Royal, also have swords as part of their ceremonial dress.\nIs the blue sash related to the bearing of a sword, or an unrelated insignia? And, for how long has it been customary (or at least accepted) for women members of the British royal family to wear swords to solemn ceremonies such as state funerals?\n", "answer": "What appears to be missing from both question and answers is the fact that the Princess Royal is breaking ground for (royal) women.\nIf anyone has seen older videos of the Remembrance Day services at the Cenotaph, only male members of the Royal Family laid wreaths, in dress uniform, complete with sword, with the sole exception of Queen Elizabeth II, who laid the first wreath on behalf of the nation \u2014 but always in civilian dress. The other Royal ladies watched from the windows of the Foreign Office. (AFAIK, the only times the late Queen wore uniform was when she took the salute at the Trooping of the Colour, on horseback. When she took to using a carriage, she wore civilian dress.)\nDuring her life Princess Anne has held various honorary military ranks, and the integration of women into the military, including the bearing of arms, presumably aided the Princess's challenging of gender stereotypes. (If you watch recent Cenotaph ceremonies, HRH is always handed her wreath by a female aide-de-camp, which would scarcely have been possible at one time.)\nPrincess Anne has broken gender barriers in other ways: when Queen Elizabeth made her a Companion of the Order of the Garter, Anne said she would not be a Lady Companion, but a Knight Companion \u2014 KG instead of LG.\nIn some ways the Princess Royal's more visible presence in traditionally male military roles reflects the changes in society. Female soldiers, sailors etc on parade carry the same arms as the men: the Princess is simply a high profile example of this.\nThe blue sash is an insignia of The Most Noble Order of the Garter, (https://en.wikipedia.org/wiki/Order_of_the_Garter) the premier British chivalric order. It has no bearing on formal dress uniform or the wearing of a sword\n", "topic": "history", "url": "https://history.stackexchange.com/questions/69898"}, {"image": "VQAonline_00021935.png", "question": "Did Belgian detectives wear special shoes in 1939?", "context": "In the comic book Land of Black Gold originally drawn in 1939 but finished or redrawn some time in 1948 by the Belgian author Herg\u00e9 (Georges Remi), one of the bad guys can apparently detect from the soles that a person is a police officer of sorts. Here is an excerpt from the comic in question, with the relevant setting:\n\nThe location is an oil tanker on its way to a fictional middle eastern kingdom. The man in the first panel is the \"bad guy\", and the man searching the drawer is Thomson, a Belgian detective, a fact unknown to the bad guy.\nSince this is featured in a comic book, my guess is that it would be common knowledge for a teenager in 1939, but definitely not to me. My question is then, what is it actually he reacts to here?\nDid the police back then always wear a certain type of shoes, or is it that every sailor had to wear special shoes, and the shoes he sees are instead \"normal\" shoes?\n", "answer": "Not sure if specifically a police issue - but the image seems to indicate Thompson as wearing hobnail shoes (the dots patterning his soles). They were common in military shoes, but also for other purposes.\nHowever, they are not appropriate on ships with steel decks (especially in wet conditions), as the wearer would skid on the surface as they walked, so that might be a giveaway that Thompson is not a sailor.\n", "topic": "history", "url": "https://history.stackexchange.com/questions/35335"}, {"image": "VQAonline_00022210.png", "question": "Besides sparge water, what other water do all-grain brewers add in (and why)?", "context": "I have done a few extract kit beers and now a few partial mashes. I am starting the process of ramping up to all-grain.\nIn many online \"brewing calculators\", such as this mash water chemistry tool by Beersmith, they indicate that the total water volume is different than the mash water volume:\n\nI can understand that total water volume might be equal to mash volume plus sparge water volume, but is any other water (besides the sparge) added to the \"total\" at any point in the all grain brewing process? Why is it added, and when?\n", "answer": "Total water in your kettle is typically Mash liquor + sparge liquor - grain absorption, which you've essentially stated.\nI can't speak for this calculator, but for the purpose of simple all-grain brewing don't let it throw you off too much. These calculators are an assistant at best and will get close (and tbh have gotten much better over the years)- but will never really match up 100%\nThere are certain brewing processes in which one might start with a more concentrated wort solution and top off with water (among other things) but these are typically for pretty special processes.\n", "topic": "homebrew", "url": "https://homebrew.stackexchange.com/questions/25691"}, {"image": "VQAonline_00022099.png", "question": "What is this white stuff on the surface of my beer?", "context": "I've just gone to rack & bottle a witbier that has been in primary for about 5 weeks and noticed this white gunk floating on top of the beer.\nIt doesn't look too healthy, what is it? Is it safe? I'm hoping it's just some undisolved dry wheat extract or dry krausen.\n\nEDIT: After bottling / kegging the brew tasted & smelt fine - great in fact.\n", "answer": "It looks like normal Wit yeast byproducts to me. Wit yeast is a weird one anyway, imho. Does it smell like vinegar at all? A Wit should taste a little tangy, but you just need to verify that it's not infected. \nI'd recommend bottling now, but start checking the bottles for over-carbonation starting in about a week, and if you start getting gushers, then you might need to chuck the whole batch.\n", "topic": "homebrew", "url": "https://homebrew.stackexchange.com/questions/4470"}, {"image": "VQAonline_00022152.png", "question": "Mold growing on blow-off tube water -- batch ruined?", "context": "I'm brewing my first batch with a brooklyn brew show all-gain kit. I skipped the airlock and left the blow-off tube in place in a pot of bleachy water. It's been there for the past two weeks. \nToday, I go look at it to start bottling and I see mold growing on the water that the blow-off tube was plugged into! I guess the bleach evaporated..\nIs the batch ruined?\nAnd to avoid this in the future, I suppose I should just re-up the bleach every so often?\n", "answer": "It's not at all ruined. The airlock is working exactly as intended. Unless there is a growth of mold all the way up the blow-off tube and into the fermentor, you're fine.\nIt's not so much that the bleach evaporated as it was denatured by the copious blow-off material. You don't need to \"re-up\" on bleach (you really don't even need bleach) so much as discard that nasty-ass blow-off water after the ferment is far enough along that the blow-off tube is not really needed.\n", "topic": "homebrew", "url": "https://homebrew.stackexchange.com/questions/16343"}, {"image": "VQAonline_00022131.png", "question": "Saison fermentation. White residue and thin film. Infected?", "context": "I'm ferementing a saison with Wyeast 3724, and after the initial ferment, it died down, I then bought it into a warmer room and chucked some blankets over it (I'm also using an aquarium heater, but that alone wasn't working). I checked on it last night, and it seems to have fleks of white on top, as well as a thin film. The smell is also rather funky (I guess this is usual for a Saison). Here's what it looks like:\n\nIs this usual for a saison, or have I got an infection on my hands?\n", "answer": "Sorry to say that looks like the beginning of a pellicle, meaning your beer is infected. But if you drink it quickly, you may avoid the worst of it. Best case, it might even taste good! And NEVER put that heater in your beer! Put the beer in a tub of water and put the heater in the water.\n", "topic": "homebrew", "url": "https://homebrew.stackexchange.com/questions/12059"}, {"image": "VQAonline_00022197.png", "question": "Can I distill contaminated wine?", "context": "15 days ago I started a batch of wine from a Wine Kit. I went to measure the SG to begin the next step when I found the wine had a film layer at the top. Anyway I phoned the place that sold it to me and am getting a new wine kit tomorrow.\nBefore I pour this contaminated wine down the toilet, I am wondering if I could finish the process and distill it? Can I do anything to salvage this ~21L of bad juice?\nHere's a picture of the contamination, it looked more like a white film before I poked it with a thermometer then it cracked:\n\n", "answer": "Yes you can! You are just distilling the alcohol, not all the bad stuff. If you catch it before it turns to vinegar you should be fine.\n", "topic": "homebrew", "url": "https://homebrew.stackexchange.com/questions/24549"}, {"image": "VQAonline_00022143.png", "question": "How do I translate my water supply's \"Total Hardness\" to Mg and Ca or \"Effective Hardness\"?", "context": "My water supply comes through San Diego's Miramar treatment plant, and here's the data sheet. Here's the relevant part:\n Units Avg Range\nTotal Hardness ppm 214 142 - 243\nTotal Hardness gr/Gal 12.2 8.3 - 14.2\n\nAnd an excerpt from the footnotes:\n\nHardness is the sum of polyvalent cations present in the water, which is essentially the sum of magnesium and calcium. These cations are usually naturally occurring.\n\nOkay, now, I'm trying to use this scale under \"Determining the Beer Styles That Best Suit Your Water\" in Ch. 15.3 of John Palmer's How to Brew:\n\nWhat is my Effective Hardness? Is it the same as Total Hardness? (In which case, 214ppm seems very high on this scale\u2014Is it possible my water is that hard?) Or, since the footnotes say Total Hardness is a sum, is my best bet to split 214ppm by some rough, sane-looking proportions like 75ppm Mg and 150ppm Ca, thus crossing over on the scale at about 145ppm Effective Hardness?\nBy the way, my CaCO3 level is 98.2ppm. Not sure this is relevant to hardness though\u2014I imagine it's the non-alkaline Calcium that hardness is measuring.\nI do know a store near me where I can get reverse osmosis water for 35 cents a gallon which is basically nothing. I'm more just curious about tap water chemistry, especially for amateur brewing.\nThanks!\n", "answer": "In short, you can't.\nIf you dig into Palmer's spreadsheet version, you see that effective hardness is equal to:\n[ppm Ca / 1.4] + [ppm Mg / 1.7]*\n* See cell K15 on the Mash sheet\nNot getting too far into detail, this is a simplification of the Kolbach formula that Palmer refers to on the page you linked:\nResidual Alkalinity = mEq/L Alkalinity - [(mEq/L Ca)/3.5 + (mEq/L Mg)/7],\nused to determine how much the alkalinity is offset by the hardness (more specifically how much is left over, if any). mEq/L means milliequivalents per liter, which is just a way of expressing concentrations of different ions by first converting them to a common unit, similar to using common denominators with fractions.\nHere's the catch:\nThe total hardness of 214 ppm does not directly represent Ca or Mg concentrations, but rather the combination of their equivalent amounts of CaCO3, just as in the equation above. Put another way, it does not mean that the amount of Ca plus the amount of Mg in your water equals 214 ppm (e.g., 150 ppm Ca + 64 ppm Mg), but rather that whatever amount of Ca and Mg there is in your water, together, has an equivalent weight of 214 ppm expressed as CaCO3, telling you nothing about the concentration of either. A bit confusing? For sure. \nThe reason you can't apply the total hardness as your effective hardness is because Ca and Mg offset the alkalinity to different degrees, as you can see in the equations above. There's just no way to make this formula work without knowing the specific proportions of Ca and Mg. Indeed, this is a very common shortcoming of many municipal water reports, which are definitely not tailored to the brewer's needs. \n- edit -\nIf anyone's interested, here's how the equation in Palmer's spreadsheet is derived:\nFirst, the calcium and magnesium are converted to mEq/L by dividing ppm of each by their respective equivalent weights:\n1 ppm Calcium / 20\n1 ppm Magnesium / 12.2\n\nThese numbers are then multiplied by 50, the equivalent weight of CaCO3 so they can be directly compared to alkalinity:\nCalcium -> [50 / 20]\nMagnesium -> [50 / 12.2]\n\nTo finish it off, Kolbach's formula is applied to both:\nCalcium -> [50 / 20] / 3.5 = 0.714\nMagnesium -> [50 / 12.2] / 7 = 0.5855\n\nCheck that against Palmer's equations:\nCalcium -> 1 ppm / 1.4 = 0.714, checks out\nMagnesium -> 1 ppm / 1.7 = 0.588, close enough!\n\n", "topic": "homebrew", "url": "https://homebrew.stackexchange.com/questions/15025"}, {"image": "VQAonline_00022157.png", "question": "Help needed with recipe translation", "context": "I'm making the move from brewing all-grain kits to a \"full fledged\" recipe Weizen. However, I'm having trouble with some of the details: namely quantities and sparging. \nSome background: I've been doing small batches (4L) with fermentation on a demijohn. Only done 2 weiss so far. \nFor those 4L, the instructions I had ask for 3L of mash water and 4L for sparging (which I do by slowly pouring the water over the sieved grain). \nI've found the following 5 gallon recipe which I was intending to follow and convert. \nMy conclusions so far were (please correct me if I'm wrong):\nBatch size: 4L. \nBoil size: 4.9L (amount of wort ready to be boiled). \nRatio from 5 gallon to 4L is 5.683 (22.73L to 4L), which I used for converting grain and hops' quantities:\n\nWheat malt: 400g\nPale malt: 320g\nHops: 2.5g + 2.5g\n\nMy first question is, how much sparge water should I use? \nI believe the method I've used so far is called partial boil since the (kit) recipe says to add remaining water to fermenter to make up for any loss. How easy it'll be to make it a full boil (as most people seem to consider it \"better\"). \n\n(source: brewcraftbeer.co.uk)\nFinally, are my calculations correct?\nI guess the hops quantities will have to be adjusted according to the AAU of the hops I get? \n", "answer": "There are a bunch of mash water:grain ratios, but generally around 1.5:1 qt./lb. (that's about 1.5 litres per 500 grams).\nThe software BeerSmith recommends batch sparging your recipe with 4.1 and then 7.1 litres of water. So (assuming you're batch sparging) the first sparge-water addition would be 4.1 litres, and the second 7.1 litres. if you're continuously sparging, ideally you'd be close to finished after something like 4+7 => 11 litres of sparge water have passed through the grain bed.\nReally sparging is \"washing\" sugars from the grain. You want to keep sparging until the wort coming off the grain bed has a specific gravity, say around 1.010 - that is, when nearly all sugars are collected. Obviously you can't sparge all day, and you may not be able to boil that volume of liquid, so you stop at some point.\nThis is then boiled down to meet the correct gravity. If something went wrong, your volume of boiled wort may not be the amount you expected.\nFinally, FWIW: if you really want a \"German Style\" weizen (hefeweissbier), the type of yeast is very important. I suggest the \"Weihenstephan 3068\" yeast, available from both White Labs and Wyeast.\n", "topic": "homebrew", "url": "https://homebrew.stackexchange.com/questions/17176"}, {"image": "VQAonline_00022215.png", "question": "How certain is it that Lucas invented the Towers of Hanoi puzzle?", "context": "Wikipedia is unequivocal:\n\nThe puzzle was invented by the French mathematician \u00c9douard Lucas in 1883.\n\nI have no reason to doubt this, but given the many legends surrounding the topic,\nI wonder if there is not an earlier (even \"ancient\") source...\n\n\u00a0\n\u00a0\n\u00a0\n\u00a0\n\u00a0\n\n\n\u00a0\n\u00a0\n\u00a0\n\u00a0\n\u00a0\n(Wikipedia image.)\n\n", "answer": "The Tower of Hanoi \u2013 Myths and Maths gives a detailed historical account of the game, its predecessors and myths surrounding it, with references to original sources, it also goes in-depth into mathematical issues. The Temple of Brahma story was first reported in 1883 by Professor N. Claus (de Siam), mandarin of college Li-Sou-Stian, and was embellished by Henri de Parville, a popular science writer and chief editor of La Nature at the end of 19-th century. He first published it in Revue des Sciences in 1883, and then a year later, with pictures, in La Nature. If de Parville thought the fable would add to the game's popularity he was certainly right. Later authors made it even more colorful, adding details and modifications. \n\"A mandarin who invents a game about combinations, will incessantly think about combinations\", says de Parville. Permuting the letters in N. Claus (de Siam) of college Li-Sou-Stian reveals Lucas d'Amiens of lycee Saint-Louis. That would be Francois Lucas born in the city of Amiens, who taught at the lycee. W.W.R. Ball, a mathematician who translated the fable from French into English in Mathematical Recreations and Essays first published in 1892, called it \"a sufficiently pretty conceit to deserve repetition\". But if you are not convinced here is a comprehensive bibliography on the Towers of Hanoi. Not one entry dates from before 1883.\n", "topic": "hsm", "url": "https://hsm.stackexchange.com/questions/751"}, {"image": "VQAonline_00022227.png", "question": "Who is Hr\u00e6\u00f0raford, the \"learned clerk\" \"writing in modern chemical Latin\"?", "context": "Truesdell, C. Rational Thermodynamics. New York, NY: Springer New York, 1984. http://dx.doi.org/10.1007/978-1-4612-5206-1, p. 41 says:\n\n\u2026 in the numerous papers and books whose titles and topics include \"irreversible\"\u2014the learned clerk HR\u00c6\u00f0RAFORD, writing in modern chemical Latin, has recently called their field \"thermodynamice inreparabilis\"\u2014\u2026\n\nWho is this \"learned clerk\" Hr\u00e6\u00f0raford? Below, the relevant paragraph is shown: \n\n", "answer": "This is not exactly an answer, but too long for a comment. The quote, even in the attached context, is somewhat incomplete. It follows description of the few (including Duhem and Eckart), who in Truesdell's opinion positively contributed to thermodynamics after Gibbs. In other parts of the book he has pages long critique of what he calls \"Onsagerism\", i.e. Onsager's reciprocal relations in non-equilibrium (a.k.a. irreversible) thermodynamics. Actually, critique is too mild a word, one gets the idea from the second half of the displayed passage. \"HR\u00c6\u00f0RAFORD, writing in modern chemical Latin\" refers not to Hraedraford, whoever he is, so writing (as I originally misread), but to Truesdell writing his name in \"modern chemical Latin\". The whole passage is dripping with sarcasm about Onsager's formalism and notation. \nI was unable to find any references to this Hraedraford, but this is perhaps not so surprising because the only reason he is cited is for a Latin pun on \"irreversible\": \"thermodynamice inreparabilis\", the irreparable thermodynamics, which is what Onsager's formalism is in Truesdell's opinion. The \"numerous papers and books\" are not Hraedraford's either, so he is likely just some witty learned citizen who came up with a pun Truesdell liked. Interestingly, the introduction and Hraedraford in it only appear in the second 1984 edition of Rational Thermodaynamics, the first 1969 edition does not mention him despite already including a chapter long rage against the \"Onsagerism\" with an approving mention of Eckart.\n", "topic": "hsm", "url": "https://hsm.stackexchange.com/questions/3581"}, {"image": "VQAonline_00022233.png", "question": "Inscriptions on a 16th century 3-dimensional permutahedron sundial?", "context": "Does anyone know what the inscriptions, written presumably in Latin, are on this 3-D permutahedron sundial by Stefano Buonsignori (16th century) in the Medici collection presented by Museo Galileo?\n\n", "answer": "The following is an e-mail kindly sent to me by Giorgio Strano, a curator at the museum:\nBesides the usual \"Tropicus Cancri\", \"AEquator\", etc. the sundial contains the following sentences:\n\"Ad Latitudinem Graduum Quadragintatrium Minutorum Triginta\" (For the latitude of 43\u00b0 30')\n\"Sol idem semper, [nec non (?)] semper currit et ue[rtit ('?)]\" (The sun is always the same, [however ?] it always runs and [turns around ?])\n\"Non sic antiq[ui]s notus, nunc tempora monstro\" (I show the time of today, not as it was known to the ancients [i.e. the dial shows equal, not unequal hours])\n\"Surgentem ostendo solem, ostendoq[ue] cadentem\" (I show the rising and the setting Sun)\n[ (?)] indates plausible integrations of a sentence where the words in the sundial are scratched away\n[ ] indicates resolved Latin abbreviations\nI hope this information is what you were looking for.\nBest regards,\nGiorgio Strano\nCurator \n", "topic": "hsm", "url": "https://hsm.stackexchange.com/questions/5573"}, {"image": "VQAonline_00022221.png", "question": "Was Einstein the first person to think of the equivalence of a ball drop in an accelerated elevator and a ball drop on Earth?", "context": "Consider the following figure taken from Wikipedia.\n\nWas Einstein the first person to think of the equivalence of a ball drop in an elevator with acceleration of $g$ and a ball drop on Earth?\nTo be honest, I was also thinking of it before I learned relativity in high school.\n", "answer": "Einstein also considered weightlessness in free fall as a manifestation of equivalence between inertial forces and gravity. Although simple drum-rope elevators were constructed as early as 236 BC (by Archimedes) they started to spread after 1854, when Otis gave a dramatic demonstration of his safety catch invention by cutting holding ropes while people were inside. By 1900s a good number of people probably did drop balls in some of them, but as far as we know none of them analyzed the physics involved, or expressed their thoughts in print. This being said, there were authors who imagined what it would be like in a vertically accelerating vehicle, but they wither did not focus on the physics, or did not get it quite right. \nPossibly, the first was Islamic polymath Avicenna (c. 980-1037) with his Floating Man argument, the literal translation is \"falling man\". Avicenna was not concerned with physics, he argued that a person can be self-aware even if from birth deprived of all sensory input, including weight. It is unclear however what Avicenna thought cancels the effect of weight, free fall or air buoyancy. First definite account of weightlessness is attributed to Francis Godwin's novel Man in the Moone (1638) written before Newtonian mechanics was even established. The character is carried upward by birds, who \"strooke bolt upright, and never did linne towring upward, and still upward, for the space\" until \"the Lines slacked; neither I, nor the Engine moved at all, but abode still as having no manner of weight\". But Godwin's weightlessness is due to weakening of gravity rather than to inertia countering it:\"those things which wee call heavie, do not sinke toward the Center of the Earth, as their naturall place, but as drawen by a secret property of the Globe of the Earth, or rather some thing within the same, in like sort as the Loadstone draweth Iron, being within the compass of the beames attractive\". \nIt is unlikely that Einstein was aware of Godwin, but he might have encountered Jules Verne's novels From Earth to the Moon (1865), and Around the Moon (1870). They were quite popular at the time, and even inspired a 1902 silent film. In them travelers are launched to the Moon from a moongun in a projectile called Columbiad. Verne was aware of the overload forces, and protected his passengers from them with hydraulic and frangible shock absorbers, but it did not occur to him that they would be working like gravity. Moreover, he expected the passengers to feel the pull of gravity throughout the journey despite being mostly in free fall. Only near the \"neutral point\", where the gravity forces of the Earth and the Moon balance out, did he expect weightlessness:\"But that day, about eleven o'clock in the morning, Nicholl having accidentally let a glass slip from his hand, the glass, instead of falling, remained suspended in the air. \"Ah!\" exclaimed Michel Ardan, \"that is rather an amusing piece of natural philosophy\"\". \nIn Exploration of Outer Space by Means of Rocket Devices (1903) Tsiolkovsky presented detailed engineering calculations related to launching rockets into space. In particular, he concluded that the Verne's moongun would have to be impossibly long for the passengers to survive the acceleration overloads at take off, and offered rockets as the alternative. But he did not comment either on the gravity like behavior of the inertial forces, or on weightlessness.\n", "topic": "hsm", "url": "https://hsm.stackexchange.com/questions/2823"}, {"image": "VQAonline_00022226.png", "question": "Was \"Kepler's third law\" deduced from the Galilean moons, or from planetary motion?", "context": "I have read that Galileo was able to start observing the four large satellites of Jupiter in 1610. Did he ever attempt to estimate the relative sizes of the four orbits, and their periods? \nI made a plot of the period of the four satellites versus their orbit size, as a ratio to the diameter of Jupiter at the equator. That would have been easier to sketch/estimate than absolute size.\nIs there any record of a plot or graph somewhat like this - to visualize even qualitatively what kind of relationship there was? \nToday one would make a log-log plot and immediately \"discover\" the 3/2 slope, but logarithms weren't available yet. \nAbout eight years later, Kepler \"articulated\" what we now call Kepler's third law, the period squared being proportional to the diameter (semi-major axis) cubed. Did that realization come from these moons, or from looking at planetary motion instead?\nOne reason I ask about the Galilean moons - the eccentricity of orbits of these four moons is very low - their motion would be easily interpreted using just circles, while several planets have significant eccentricity and their positions would require more sophisticated math.\nSince the relationship of the planets and that of the Jovian moons would have a different constant - was it immediately interpreted as a measure of the ratio of masses of Jupiter and the Sun?\nThis is my first question here - the wording may seem colloquial, but my question is serious.\n\n", "answer": "The Wikipedia article on Harmonices Mundi states that Kepler gave only the conclusion. \nSince he had taken all of Brahe's observations, the presumption is that he used this data, for he was very familiar with it, and it was more than adequate for the task. His published result describes the relationship in terms of the sun and planets, but not planets and moons.\nSee also Was Jupiter's mass \u201cguessed at\u201d by Kepler or Galileo?\n", "topic": "hsm", "url": "https://hsm.stackexchange.com/questions/3574"}, {"image": "VQAonline_00022230.png", "question": "Introduction of $\\imath$ and $\\jmath$ notations for the imaginary unit", "context": "The imaginary unit is generally denoted $i$ or $\\imath$. I have learned that the term imaginary (\"imaginaires\") was coined by R. Descartes in 1637, and the \"i\" notation was introduced by L. Euler (cf. Short History of Complex Numbers). In engineering and physics, the notation $j$ or $\\jmath$ is often used. Some say it was used to limit confusions with the current, often denoted $i$ or \"I\", as noted at Electric current:\n\nThe conventional symbol for current is I, which originates from the\n French phrase intensit\u00e9 de courant, meaning current intensity .\n\nMy questions are:\n\nWas the $\\imath$ (or $i$) notation used before Euler?\nWho introduced the $\\jmath$ (or $j$) notation?\n\nEDIT: I just found, answering on a different topic about frequency, a mention of the word \"cisoid\" (abbreviated as $\\mathrm{cis}$), I just found Cisoidal Oscillations, 1911, by George A. Campbell, where he writes:\n\nThe use of $i$ (or Greek $\\imath$) for the imaginary symbol is nearly\n universal in mathematical work, which is a very strong reason for\n retaining it in the applications of mathematics in electrical\n engineering. Aside, however, from the matter of established\n conventions and facility of reference to mathematical literature, the\n substitution of the symbol $j$ is objectionable because of the vector\n terminology with which it has become associated in engineering\n literature, and also because of the confusion resulting from the\n divided practice of engineering writers, some using $j$ for $+i$ and\n others using $j$ for $- i$\n\n\nSo the story is not fully new, and we could date the $j$ before 1911.\n", "answer": "In answer to your second part of the question regarding $j$ for $\\sqrt{-1}$, this was introduced into text books describing Power System Analysis of AC power circuits in the early 1900s by Charles P. Steinmetz. I am not sure of the earliest date but my guess is between late 1890s and 1920s but certainly no later than 1923 as Steinmetz died in 1923.\nSteinmetz is to Power Systems Engineers (EEs) as Einstein and $E=m c\u00b2$ is to physicists (and everyone else for that matter).\nYou can read all about Charles Steinmetz here.\n", "topic": "hsm", "url": "https://hsm.stackexchange.com/questions/5082"}, {"image": "VQAonline_00022263.png", "question": "Greater-than symbol in Byrne's *The Elements of Euclid*", "context": "I was surprised to find that Oliver Byrne's 1847 marvelous The Elements of Euclid (color version)1 uses $\\sqsubset$ to mean \"greater than\" and $\\sqsupset$ to mean \"less than,\"\nin contrast our current $>$ and $<$ (p. xxvii).\nThis is especially puzzling because:\n\n\"The symbols < and > first appear in Artis Analyticae Praxis ad Aequationes Algebraicas Resolvendas (The Analytical Arts Applied to Solving Algebraic Equations) by Thomas Harriot (1560-1621).\"\nlink\n\nAny speculation on Byrne's motivation?\nTo me it seems so non-intuitive that $\\sqsubset$ should mean $>$ that I am surprised\nit was used in a book that tries to make the math easier to grasp.\nPerhaps $\\sqsubset$ was in common use in parallel with $>$?\n\n\u00a0\u00a0\u00a0\n\n1\nByrne, Oliver. The first six books of the Elements of Euclid: in which coloured diagrams and symbols are used instead of letters for the greater ease of learners. William Pickering, 1847.\n", "answer": "Byrne's symbols are variations of Oughtred's, contamporaneous with Harriot's, see Cajori, History Of Mathematical Notations, vol. I, p.192. They were adopted by Barrow, Newton's teacher, in his edition of Euclid, ibid., p. 210. Why Byrne went with Barrow's notation c. 1660 is hard to say, by his time Harriot's symbols were canonized by Euler, but their origin was not exactly intuition. Perhaps, he felt that it would be \"easier to grasp\" by those unburdened by entrenched standard. For example, Lambert in 1782 used < and > \"in reverse\" in his logic, as inspired by implication, see Lewis, Survey of symbolic logic.\nOn early experiments with various inequality symbols see Tanner, On the Role of Equality and Inequality in the History of Mathematics, p. 167-8. Incidentally, he attaches a confabulation on why Harriot's symbols look the way they do, which, spelled out or not, probably helped their adoption as \"intuitive\".\nHere is a later example of how our related \"intuitive\" symbols in set theory could have been reversed. Interpreting inference as inclusion of classes was common in the 19th century. Whatever can be said for the implication pointing in the direction it does, if one chooses that as \"intuitive\" then their inequality symbols should go the opposite way from our today's standard too. Indeed, if something is in A then it is in B (A \u2283 B) means that A must be smaller than B (A \u2282 B), see Why is there this strange contradiction between the language of logic and that of set theory?\nHarriot, or rather his editors, derived < and > from a tattoo on a native American's hand, which was symmetric, so what they chose to stand for what was rather random. Derived from this in Schr\u00f6der's Vorlesungen set inclusion symbol \u2282 then conflicted with the implication symbol \u2283, as used by Peano and later many others. Gergonne earlier derived a similar symbol from the looks of letter C (in \"contained\"), i.e. equally randomly, see Earliest Uses of Symbols of Set Theory and Logic.\nPeano, for his part, was consistent, and used \u2283 for inclusion, as well as implication, see Tou, Math Origins: The Logical Symbols, so his smaller would be >. Peirce, Schr\u00f6der's inspiration for much of his content, was consistent the other way, his implication looked something like this \u2014<, see Anellis, Peirce's Truth-functional Analysis and the Origin of the Truth Table.\nHilbert's later symbol \u2192 for implication similarly conflicts with <, especially when the arrowhead is drawn bigger.\n", "topic": "hsm", "url": "https://hsm.stackexchange.com/questions/14667"}, {"image": "VQAonline_00022301.png", "question": "Does a hadith \"believers who show the most perfect Faith are those who have the best behaviour\" exist?", "context": "The following image quotes a hadith about marriage. Is it an exact quote or rephrasing of another hadith?\n\nI found another hadith with similar wording but not the exact quote:\n\nAbu Hurairah (May Allah be pleased with him) reported: Messenger of\n Allah (\ufdfa) said, \"The believers who show the most perfect Faith are\n those who have the best behaviour, and the best of you are those who\n are the best to their wives\".\n[At-Tirmidhi].\n\nDoes a hadith with the exact wording exist?\n", "answer": "The hadith, as shown in the picture you posted, is in Jami' Al-Tirmidhi (Arabic: \u062c\u0627\u0645\u0639 \u0627\u0644\u062a\u0631\u0645\u0630\u064a), not in Sahih Muslim. Most likely, the picture is referring to this hadith:\n\n\u062d\u062f\u062b\u0646\u0627 \u0623\u0628\u0648 \u0643\u0631\u064a\u0628 \u0642\u0627\u0644: \u062d\u062f\u062b\u0646\u0627 \u0639\u0628\u062f\u0629 \u0628\u0646 \u0633\u0644\u064a\u0645\u0627\u0646\u060c \u0639\u0646 \u0645\u062d\u0645\u062f \u0628\u0646 \u0639\u0645\u0631\u0648 \u0642\u0627\u0644: \u062d\u062f\u062b\u0646\u0627\n \u0623\u0628\u0648 \u0633\u0644\u0645\u0629\u060c \u0639\u0646 \u0623\u0628\u064a \u0647\u0631\u064a\u0631\u0629 \u0642\u0627\u0644: \u0642\u0627\u0644 \u0631\u0633\u0648\u0644 \u0627\u0644\u0644\u0647 \u0635\u0644\u0649 \u0627\u0644\u0644\u0647 \u0639\u0644\u064a\u0647 \u0648\u0633\u0644\u0645: \u0623\u0643\u0645\u0644\n \u0627\u0644\u0645\u0624\u0645\u0646\u064a\u0646 \u0625\u064a\u0645\u0627\u0646\u0627 \u0623\u062d\u0633\u0646\u0647\u0645 \u062e\u0644\u0642\u0627\u060c \u0648\u062e\u064a\u0631\u0643\u0645 \u062e\u064a\u0631\u0643\u0645 \u0644\u0646\u0633\u0627\u0626\u0647\u0645\nAbu Hurairah narrated that The Messenger of Allah said: \"The most\n complete of the believers in faith, is the one with the best character\n among them. And the best of you are those who are best to your women.\"\n\u2014 Source: Jami' Al-Tirmidhi, Muhammad ibn Issa at-Tirmidhi,\n Vol. 1, Book 7, Hadith 1162)\n\nThe wording is almost identical, considering that both are translations. This is an authentic (sahih, Arabic: \u0635\u062d\u064a\u062d) hadith.\n", "topic": "islam", "url": "https://islam.stackexchange.com/questions/38238"}, {"image": "VQAonline_00022269.png", "question": "What is the significance of being resurrected as \"blind (from birth)\"?", "context": "In Surah Taha, Allah says:\n\n\nHe will say, \"My Lord, why have you raised me blind while I was [once]\n seeing?\" (Surah Taha, verse 125)\n\nFrom surah 'abasa, we know that the word used here and translated as blind, \u0623\u064e\u0639\u0652\u0645\u064e\u0649\u0670 (a3maa) means blind from birth. It does not refer to a person who acquired blindness during their lifetime after having the ability to see.\nThis seems interesting -- that a person who, by their own admission, could see in their dunya life, is now resurrected blind (from birth). What is the significance of the word a'maa here, as opposed to a different word that could have been used to describe \"could once see but is now blind\"?\n", "answer": "In tafseer, scholars commented that this means the person will have no proof to justify his deeds in dunya (world). There will be no evidence supporting him. \nMujahid, Abu Salih and As-Suddi said, \n\n\"This means he will have no proof.\"\n\n`Ikrimah said, \n\n\"He will be made blind to everything except Hell.\"\n\nIt is because of what Allah the Exalted says in Surah Al-Isra :\n\nAnd We shall gather them together on the Day of Resurrection on their\n faces, blind, dumb and deaf; their abode will be Hell. [Qur'an\n 17:97]\n\nSource: Tafseer Ibn Katheer\n", "topic": "islam", "url": "https://islam.stackexchange.com/questions/1569"}, {"image": "VQAonline_00022313.png", "question": "Is Non-vegetarian Cheese halal", "context": "My question is specifically regarding Pesto I have been having for a few weeks, until last Friday when I noticed the words Non vegetarian cheese.\nPlease see this image:\n\nI am wondering whether to continue enjoying this product or to throw it away. Both the cheese mentioned in this product, Padano Cheese & Pecorino Romano Cheese appear to be milk based. \nMy question is, is this halal to consume or should I err on the side of caution & throw it away.\nThank you.\n", "answer": "UPDATE:\nIt seems like (thanks to the answerer below) the product contains potentially one haram ingredient namely rennet. It is extracted from the stomach of animals.\n\nTraditional animal rennet is an enzyme derived from the stomachs of calves, lambs or goats before they consume anything but milk. Vegetable rennet is obtained from a type of mold (Mucur Miehei). However, even though it is derived from mold, there is no mold contained in the final product. It is an equivalent chymosin product which works equally well but is not animal derived. \n\nI've skimmed the following articles for something haram based on the ingredients shown on the image, it seems you're not good to go. The only thing I'm concerned with is that it also shows the use of European and non-european products but it isn't specified. Inshallah there will be nothing haram in it. \nSo, I'd say enjoy your new vegetarian pesto. ;)\nApologies for the mistake. \nSources: \nhttps://islamqa.info/en/114129\nhttps://islamqa.info/en/219137\nhttps://en.m.wikipedia.org/wiki/Grana_Padano\nhttps://en.m.wikipedia.org/wiki/Glucono_delta-lactone\nhttps://en.m.wikipedia.org/wiki/Pecorino_Romano\nhttps://www.cheesemaking.com/learn/faq/rennet.html\nL.S.:\nIf you want to get more info about the etiquette of eating in islam, feel free to go to the link below.\nhttps://islamqa.info/en/13348\n", "topic": "islam", "url": "https://islam.stackexchange.com/questions/49176"}, {"image": "VQAonline_00022283.png", "question": "Can water be halal or haram?", "context": "On a flight from Tokyo to Sydney with Japan Airlines, I was given a bottle of water that was certified halal. (Apologies for the image quality)\nDo Muslims consider water as being potentially halal or haram, or was this a case of certification going overboard? (I can spot four other certifications on the bottle)\n\n", "answer": "Pure water by itself is fundamentally halal. If there are haram impurities involved, the water would also become haram; not because of the water itself, but because of what's in it. So, for example, if it was drawn from a small pool in which someone was storing their dead pig carcasses, we shouldn't drink it (most would chalk that up to common sense though).\nIt's always nice to know that there were no pig carcasses involved in the production of your water, but I would call the certification overkill in this case.\n\nOf note, if the water is not actually pure bottled water, but rather some brand of flavoured/enriched water, there is an increased chance that the additives themselves are haram. Given that a number of common additives are of animal origin, the certification makes more sense in such cases.\n", "topic": "islam", "url": "https://islam.stackexchange.com/questions/8579"}, {"image": "VQAonline_00022284.png", "question": "What is the name of the script used in the earliest Qur\u02be\u0101nic manuscripts?", "context": "I've seen it transliterated into English as ma'ili which supposedly means \"inclined\" or \"slanting.\" I'm looking for the Arabic word. Is it \u0645\u0627\u0626\u0644\u064a?\n\n", "answer": "It is written \u0645\u0627\u0626\u0644 (maa'il) in Arabic, which does indeed mean slanted. Another name for this early script is \u062d\u062c\u0627\u0632\u064a\u200e (hijazi) in reference to it originating from the Hijaz region of the Arabian Peninsula.\nLater Quranic manuscripts switched to Kufic which is a reference to the city of Kufa in Iraq where it was first developed.\n", "topic": "islam", "url": "https://islam.stackexchange.com/questions/8608"}, {"image": "VQAonline_00022309.png", "question": "How to accurately translate \u062a\u0631\u0643 in the context of the hadith \"Between man and polytheism and unbelief is the [abandonment/negligence] of prayer\"?", "context": "There are two similar ahadith with different translations:\n\n\u0625\u0650\u0646\u0651\u064e \u0628\u064e\u064a\u0652\u0646\u064e \u0627\u0644\u0631\u0651\u064e\u062c\u064f\u0644\u0650 \u0648\u064e\u0628\u064e\u064a\u0652\u0646\u064e \u0627\u0644\u0634\u0651\u0650\u0631\u0652\u0643\u0650 \u0648\u064e\u0627\u0644\u0652\u0643\u064f\u0641\u0652\u0631\u0650 \u062a\u064e\u0631\u0652\u0643\u064e \u0627\u0644\u0635\u0651\u064e\u0644\u0627\u064e\u0629\u0650\nVerily between man and between polytheism and unbelief is the negligence of prayer.\nSahih Muslim 82 a\n\u0628\u064e\u064a\u0652\u0646\u064e \u0627\u0644\u0631\u0651\u064e\u062c\u064f\u0644\u0650 \u0648\u064e\u0628\u064e\u064a\u0652\u0646\u064e \u0627\u0644\u0634\u0651\u0650\u0631\u0652\u0643\u0650 \u0648\u064e\u0627\u0644\u0652\u0643\u064f\u0641\u0652\u0631\u0650 \u062a\u064e\u0631\u0652\u0643\u064f \u0627\u0644\u0635\u0651\u064e\u0644\u0627\u064e\u0629\u0650\nBetween man and polytheism and unbelief is the abandonment of salat.\nSahih Muslim 82 b\n\nIt looks like the relevant Arabic word is \u062a\u0631\u0643, which Wiktionary translates to to leave, to relinquish. The impression I get from the English translation is that it means permanently.\nQuestion: How to accurately translate \u062a\u0631\u0643 in the context of the hadith \"Between man and polytheism and unbelief is the [abandonment/negligence] of prayer\"?\nThis is relevant to my previous question here: Can someone end up leaving Islam through neglect?\nGoogle Translate gives a ranking of translations, favouring \"leave\" over \"neglect\":\n\n", "answer": "Tark \u062a\u064e\u0631\u0643\u064e can mean to leave/abandon/release with an open probability to get back to something, whereas in the context of the hadeeth, The word Tark \u062a\u064e\u0631\u0643\u064e gives the notion of ceasing to pray, whether that is permanent or willingly temporary.\nDismissing Prayers\nA related verse that sheds light on the topic of the hadeeth is :\n\n\u0641\u064e\u062e\u064e\u0644\u064e\u0641\u064e \u0645\u0650\u0646 \u0628\u064e\u0639\u0652\u062f\u0650\u0647\u0650\u0645\u0652 \u062e\u064e\u0644\u0652\u0641\u064c \u0623\u064e\u0636\u064e\u0627\u0639\u064f\u0648\u0627 \u0627\u0644\u0635\u0651\u064e\u0644\u064e\u0627\u0629\u064e \u0648\u064e\u0627\u062a\u0651\u064e\u0628\u064e\u0639\u064f\u0648\u0627\n \u0627\u0644\u0634\u0651\u064e\u0647\u064e\u0648\u064e\u0627\u062a\u0650 \u06d6 \u0641\u064e\u0633\u064e\u0648\u0652\u0641\u064e \u064a\u064e\u0644\u0652\u0642\u064e\u0648\u0652\u0646\u064e \u063a\u064e\u064a\u0651\u064b\u0627 - 19:59\n\nsahih International translation\n\nBut there came after them successors who neglected prayer and pursued\n desires; so they are going to meet evil -\n\nYousuf Ali Translation\n\nBut after them there followed a posterity who missed prayers and\n followed after lusts soon, then, will they face Destruction,-\n\nQuranic wording\nAllah, exalted He is, eloquently used the term \u0623\u064e\u0636\u064e\u0627\u0639\u064f\u0648\u0627 which means originally to lose something... you lose due to negligence, or you lose if you are willingly not paying attention or taking care. I think you can see how this word covers the meaning, which is permanent or willingly temporary.\nTark \u062a\u064e\u0631\u064e\u0643\u064e in Quran\nFor your reference, the word Tark \u062a\u064e\u0631\u064e\u0643\u064e in Quran can be one of three meanings:\nTo reject/abandon/leave intentionally\n\n\u0648\u064e\u062a\u064e\u0631\u064e\u0643\u0652\u0646\u064e\u0627 \u0628\u064e\u0639\u0652\u0636\u064e\u0647\u064f\u0645\u0652 \u064a\u064e\u0648\u0652\u0645\u064e\u0626\u0650\u0630\u064d \u064a\u064e\u0645\u064f\u0648\u062c\u064f \u0641\u0650\u064a \u0628\u064e\u0639\u0652\u0636\u064d \u06d6 \u0648\u064e\u0646\u064f\u0641\u0650\u062e\u064e \u0641\u0650\u064a\n \u0627\u0644\u0635\u0651\u064f\u0648\u0631\u0650 \u0641\u064e\u062c\u064e\u0645\u064e\u0639\u0652\u0646\u064e\u0627\u0647\u064f\u0645\u0652 \u062c\u064e\u0645\u0652\u0639\u064b\u0627 -\n18:99 And We will leave them that day surging over each other, and\n [then] the Horn will be blown, and We will assemble them in [one]\n assembly.\n\nanother example of leaving willingly, on purpose\n\n\u0648\u064e\u0627\u062a\u0652\u0631\u064f\u0643\u0650 \u0627\u0644\u0652\u0628\u064e\u062d\u0652\u0631\u064e \u0631\u064e\u0647\u0652\u0648\u064b\u0627 \u06d6 \u0625\u0650\u0646\u0651\u064e\u0647\u064f\u0645\u0652 \u062c\u064f\u0646\u062f\u064c \u0645\u0651\u064f\u063a\u0652\u0631\u064e\u0642\u064f\u0648\u0646\u064e - 44:24 And\n leave the sea in stillness. Indeed, they are an army to be drowned.\"\n\nThe other meaning that appeared in Quran is to be forced/unwillingly to leave/abandon something\n\n\u0643\u064e\u0645\u0652 \u062a\u064e\u0631\u064e\u0643\u064f\u0648\u0627 \u0645\u0650\u0646 \u062c\u064e\u0646\u0651\u064e\u0627\u062a\u064d \u0648\u064e\u0639\u064f\u064a\u064f\u0648\u0646\u064d - 44:25 How much they left behind\n of gardens and springs\n\nThird Meaning is what a person leaves as inheritance after death\n\n\u064a\u064e\u0633\u0652\u062a\u064e\u0641\u0652\u062a\u064f\u0648\u0646\u064e\u0643\u064e \u0642\u064f\u0644\u0650 \u0627\u0644\u0644\u0651\u064e\u0647\u064f \u064a\u064f\u0641\u0652\u062a\u0650\u064a\u0643\u064f\u0645\u0652 \u0641\u0650\u064a \u0627\u0644\u0652\u0643\u064e\u0644\u064e\u0627\u0644\u064e\u0629\u0650 \u06da \u0625\u0650\u0646\u0650\n \u0627\u0645\u0652\u0631\u064f\u0624\u064c \u0647\u064e\u0644\u064e\u0643\u064e \u0644\u064e\u064a\u0652\u0633\u064e \u0644\u064e\u0647\u064f \u0648\u064e\u0644\u064e\u062f\u064c \u0648\u064e\u0644\u064e\u0647\u064f \u0623\u064f\u062e\u0652\u062a\u064c \u0641\u064e\u0644\u064e\u0647\u064e\u0627 \u0646\u0650\u0635\u0652\u0641\u064f \u0645\u064e\u0627\n \u062a\u064e\u0631\u064e\u0643\u064e \u06da \u0648\u064e\u0647\u064f\u0648\u064e \u064a\u064e\u0631\u0650\u062b\u064f\u0647\u064e\u0627 \u0625\u0650\u0646 \u0644\u0651\u064e\u0645\u0652 \u064a\u064e\u0643\u064f\u0646 \u0644\u0651\u064e\u0647\u064e\u0627 \u0648\u064e\u0644\u064e\u062f\u064c \u06da \u0641\u064e\u0625\u0650\u0646\n \u0643\u064e\u0627\u0646\u064e\u062a\u064e\u0627 \u0627\u062b\u0652\u0646\u064e\u062a\u064e\u064a\u0652\u0646\u0650 \u0641\u064e\u0644\u064e\u0647\u064f\u0645\u064e\u0627 \u0627\u0644\u062b\u0651\u064f\u0644\u064f\u062b\u064e\u0627\u0646\u0650 \u0645\u0650\u0645\u0651\u064e\u0627 \u062a\u064e\u0631\u064e\u0643\u064e \u06da \u0648\u064e\u0625\u0650\u0646\n \u0643\u064e\u0627\u0646\u064f\u0648\u0627 \u0625\u0650\u062e\u0652\u0648\u064e\u0629\u064b \u0631\u0651\u0650\u062c\u064e\u0627\u0644\u064b\u0627 \u0648\u064e\u0646\u0650\u0633\u064e\u0627\u0621\u064b \u0641\u064e\u0644\u0650\u0644\u0630\u0651\u064e\u0643\u064e\u0631\u0650 \u0645\u0650\u062b\u0652\u0644\u064f \u062d\u064e\u0638\u0651\u0650\n \u0627\u0644\u0652\u0623\u064f\u0646\u062b\u064e\u064a\u064e\u064a\u0652\u0646\u0650 \u06d7 \u064a\u064f\u0628\u064e\u064a\u0651\u0650\u0646\u064f \u0627\u0644\u0644\u0651\u064e\u0647\u064f \u0644\u064e\u0643\u064f\u0645\u0652 \u0623\u064e\u0646 \u062a\u064e\u0636\u0650\u0644\u0651\u064f\u0648\u0627 \u06d7 \u0648\u064e\u0627\u0644\u0644\u0651\u064e\u0647\u064f\n \u0628\u0650\u0643\u064f\u0644\u0651\u0650 \u0634\u064e\u064a\u0652\u0621\u064d \u0639\u064e\u0644\u0650\u064a\u0645\u064c - 4:176\nThey request from you a [legal] ruling. Say, \" Allah gives you a\n ruling concerning one having neither descendants nor ascendants [as\n heirs].\" If a man dies, leaving no child but [only] a sister, she will\n have half of what he left. And he inherits from her if she [dies and]\n has no child. But if there are two sisters [or more], they will have\n two-thirds of what he left. If there are both brothers and sisters,\n the male will have the share of two females. Allah makes clear to you\n [His law], lest you go astray. And Allah is Knowing of all things.\n\nAnd Allah knows best.\n", "topic": "islam", "url": "https://islam.stackexchange.com/questions/45098"}, {"image": "VQAonline_00022323.png", "question": "Is this bid\u2019ah ? Praying two volontary raka\u2019ats", "context": "I heard on a trusted Youtube channel (IloveUAllah) that to make shaytan hate to give you whispers and make you lazy for salat you can pray 2 rakats to make the shaytan less powerful.\nIs it bid\u2019ah to do it ? Is it bid\u2019ah because there is a limited amount of (waajib and sunnah and nafl) voluntary prayers one can do in a full day ? Jazzakumu Allah.\n", "answer": "Praying two raka'as nafl can never be a bid'ah because nafl (voluntary acts) have no limitation.\nFor example there's a known hadith (see here) about Bilal () using to perform two raka'as nafl after each wudu'.\nWhat could be a bid'ah is considering it a Sunnah of the prophet to pray two raka'as in order to weaken Shaytan without any proof. And what would be worse is telling others to follow this \"sunnah\".\nIt could rather be better (or sunnah) to consider doing similar to Bilal, as this was approved by our prophet ().\n", "topic": "islam", "url": "https://islam.stackexchange.com/questions/69179"}, {"image": "VQAonline_00022339.png", "question": "Italian from 1600 - what is this letter?", "context": "I bought an old book in Italian. It's a collection of letters, published in 1600 in Venezia. What is this strange letter (and word)? It's the first letter of the second word. \nHo comprato un libro antico italiano. \n\u00c8 una collezione di lettere, pubblicata nel 1600 a Venezia.\nQual \u00e8 questa strana lettera, all'inizio della seconda parola?\n\n", "answer": "I think it's the word \"perfetto\" (see this list of scribal abbreviations).\nMi sembra che sia la parola \"perfetto\" (vedi questo elenco di abbreviazioni scribali).\n", "topic": "italian", "url": "https://italian.stackexchange.com/questions/7414"}, {"image": "VQAonline_00022330.png", "question": "Why is perch\u00e9 sometimes written perch\u00e8 instead of perch\u00e9?", "context": "If using a grave accent as opposed to an acute accent on top of letters e and o in Italian is used to denote a difference in pronunciation when these appear on the last syllable of the word, and such syllable happens to be stressed, with \u00e8 being an open e and \u00e9 a closed e, then why is the Italian word perch\u00e9 sometimes written as perch\u00e8 across the Internet instead of as perch\u00e9?\nAfter all, I've always heard the last vowel of this word being pronounced as a closed e. Could the reason for this orthographic rule be due to differences in regional pronunciations, or is there some other reason?\n\nUPDATE:\nIn response to a comment in one of the answers given below, his is an image of a magazine for kids that was popular in the late 1980s and early 1990s, perhaps even later (not sure if it still exists, haven't checked!). Anyways, it illustrates two different spellings of one particular word, one using the grave accent, and the other using the acute accent, on the very same place (and this is not a misprint, this is a sample representative of thousands of magazines each of whose cover page is similar): PI\u00d9 and Pi\u00fa. Of course, in the case of 'u' there is only way to pronounce this vowel in Italian; there is no distinction made between an open pronunciation and a closed pronunciation as is the case with the letters e and o, so perhaps this does not matter much here.\n\n(SOURCE: Link to the page where I downloaded this image)\n", "answer": "There's more than one reason for that.\nThe structure of the Italian keyboard\nI believe that the major reason has to be researched in the Italian keyboard: indeed, the key for \u00e8 and \u00e9 is the same. If it is pressed without any other keys, it outputs \"\u00e8\", while if it's used in combination with \"Maiusc\", it outputs \"\u00e9\".\nMy guess is that many people simply forget to press \"Maiusc\", they're too lazy to do that or they don't know that doing so will output the \"e\" with an other accent.\nLack of knowledge of the difference\nThere are plenty of Italians that simply don't know the difference between the two accents, hence they tend to always use the grave one.\nThis is mostly caused by the education in primary school: indeed, children are often taught the existance of just one, generic accent.\nBelief that the difference between the two accents is irrelevant\nItalian mother-tongues often don't need to distinguish between the two accents in order to spell a word (relatively) correctly, so they might think that the difference between them is so minimal that it's not worth distinguishing them.\n", "topic": "italian", "url": "https://italian.stackexchange.com/questions/3896"}, {"image": "VQAonline_00022346.png", "question": "Help to read Atto di nascita", "context": "I need help to read my grand-parent's 'atto di nascita'. See a piece of it in the attached image.\nFollowing is what I can understand. Please help me with '...' (maybe the last part is his parents address)\nLucia Serra di anni trentotto .... domiciliata in Spezzano e de ... Giovanni Cali\u00f2 di anni ... muratore\ndomiciliato in Spezzano ......... propria casa. \nThank you\n", "answer": "This is my best shot (what a peculiar handwriting!):\n\n[lo stesso \u00e8 nato] Da Lucia Serra di anni trentotto, filatrice, domiciliata in Spezzano e da suo marito Giovanni Cali\u00f2 di anni trentasei, muratore, domiciliato in Spezzano, nel d\u00ec ventisei detto mese ed anno alle ore due di notte nella propria casa.\n\nWhich translated roughly as\n\n[said person was born] from Lucia Serra, thirty-eight years old, spinner, resident in Spezzano and from her husband Giovanni Cali\u00f2, thirty-six years old, bricklayer, resident in Spezzano, on the twenty-sixth of the aforementioned month and year at two o'clock in the morning in their own house.\n\n", "topic": "italian", "url": "https://italian.stackexchange.com/questions/10619"}, {"image": "VQAonline_00022340.png", "question": "\"la sindaco\".. - nuova tendenza o semplicemente un typo?", "context": "Essendo di madrelingua italiana e vivendo in Germania, non mi scandalizza affatto la, tanto discussa, femminizzazione dei titoli, delle cariche e dei nomi di professione e simili a seconda del contesto (verso una lingua, quasi, asessuata oserei dire - opinione personale questa scaturita dal parallelo con Die Feministische Linguistik che incontrai a lezione, qualche anno fa, di tedesco qui in Germania). \nLeggendo qualche articolo sul sito dell Accademia della Crusca (e oltre), una interminabile discussione su wikipedia anche, Panico linguistico su nomi di cariche e professioni di genere femminile, e poi anche in italian.stackexchange, e sul sito della Treccani, mi ero abituato a usare \"la sindaca\" ed altri ancora, ma nemmeno il tempo di farci l'abitudine che mi ritrovo a leggere \"la sindaco\" da una pagina de \"Il Fatto Quotidiano\", ovvero:\n\nCome propone la sindaco di Barcellona Ada Colau, indicendo dal 9 all\u201911 giugno un incontro nella sua citt\u00e0, con la partecipazione degli aderenti al movimento Fearless Cities, citt\u00e0 senza paura.\n\n\nPersonalmente metterei in dubbio che si intenda \"la sindaco\" in senso ironico se non addirittura dispregiativo (Ada Colau non \u00e8 una donnetta); tanto meno riesco a pensare che sia un refuso/typo/lapsus durante una qualche misteriosa digitazione (anche se lo spero). \nAd ogni modo, \u00e8 possibile scrivere \"la sindaco\"? Qualora no, perch\u00e9? Pu\u00f2 sembrare banale come domanda, e se tale risulta chiedo venia, tuttavia dinanzi a questi nuovi termini femminili ancora oggi trovo non tanto immediato, in certi casi, una regola schietta che mi permetta di rispondere alla domanda suddetta aldil\u00e0 di un approccio da madrelingua e d'uso quotidiano. Per me la grande questione era se usare il maschile \"ovunque\" o ammettere termini femminili per le donne scritti secondo le regole, e l uso era tra \"il sindaco\" e il naturale \"la sindaca\" (e non \"la sindaco\"), ma non vorrei che quelli del Fatto Quotidiano scrivano correttamente e magari inaugurano un nuovo uso del termine...\n", "answer": "Le forme pi\u00f9 comuni sono la sindaca... o il sindaco... seguito dal nome femminile. La sindaco sembra un'espressione eccentrica ma non di uso comune. Personalmente la eviterei. \nLa sindaca \u00e8 donna, non una donna sindaco:\n\n\nFra tutti i dubbi, questo \u00e8 quello che meno ci tormenta. Complice la recente elezione di Raggi e Appendino, a Roma e Torino, possiamo affermare che il sostantivo che qualifica il primo cittadino (la prima cittadina, anzi!) abbiamo finalmente imparato a usarlo come si deve: facile facile, con la a, al posto della o; al femminile, cio\u00e8, se riferito a una donna. Vale lo stesso per l'utilizzo di ministra, sdoganato a tutti gli effetti. E a chi cerca le vie di mezzo e insiste a definire Boschi la ministro, ecco l'esempio a cui guardare: direste mai la maestro, pur avendo la versione femminile di quella parola, l\u00ec, a portata di mano?\n\n\n(www.repubblica.it)\nCome si dice correttamente sindaco al femminile? \u00c8 corretto chiamare \"sindaco\" anche una donna?\n\n\nNon siamo di fronte a una questione di correttezza grammaticale, ma di adeguatezza nel trattare i nomi di mestiere al femminile.\n *Chi scrive il sindaco Anna Rossi non sbaglia, semplicemente preferisce usare il nome di mestiere sindaco trattandolo come se fosse una sorta di \u201cneutro\u201d, inclusivo dei riferimenti di genere maschile e femminile, che si riferisce a una categoria professionale in termini di funzione generale.\n Chi scrive sindaca adopera con efficacia le risorse flessive messe tranquillamente a disposizione dalla nostra lingua: sindaco/sindaca, avvocato/avvocata, postino/postina, ecc. seguono la normale alternanza nominale di genere maschile/femminile, espressa attraverso le uscite -o e -a. \n\n\n(www.treccani.it)\nCi sono alcuni altri esempi dell'uso \"la sindaco\" come: \nDa Dalle pratiche di partecipazione all\u2019e-democracy: Analisi di casi concreti ...\n\n\n\nCinque anni dopo la sconfitta del 1999, per vincere le nuove elezioni contro la sindaco uscente, Giustina Destro, su consiglio del suo regista per la campagna elettorale, Marco Marturano, che cinque anni prima aveva portato alla vittoria la candidata Destro.\n\n\nDa \nEducazione democratica per una pace giusta:\n\n\nLa sindaco di Napoli trovava significativo che tutte le personalit\u00e0 studiate nel mio volume avessero \u00ab condiviso, seppure in tempi e contesti profondamente diversi\n\n\n", "topic": "italian", "url": "https://italian.stackexchange.com/questions/8188"}, {"image": "VQAonline_00022341.png", "question": "What are the meanings of these convincing phrases and where exactly do we use them in Italian?", "context": "\ndai\nsu\nforza\navanti\ncoraggio\n\nThe above words have the same meaning which is \"come on\". I like to know why there are so many words for just one phrase \"come on\" in Italian. What I want to know is their precise contexts. Here are some examples that I have found: \n\n", "answer": "I would say that something like \"dai\" is probably the most generic. I can imagine it being used in all those sentences correctly.\nCoraggio literally means courage, so I would imagine it more as being used in a situation when you need to build up a bit of \"courage\", even in a metaphorical way.\n\"Su\" literally means \"up\". It can be used as to cheer up someone, or to spur someone to do something. So, if I was asking a friend about how a dress looks on me, and she used \"su\", that would feel more like \"just take this dress and let's finish this, we've been around shops for three hours already!\". \n\"Forza\" literally means \"strength\". I can imagine it being used, again, when inciting someone, (\"forza, ancora qualche minuto di cammino e siamo arrivati\" -> \"Come on, some more minutes of walking and we'll get there\"), or to encourage. Actually, \"Coraggio\" would work exactly the same in this context, especially if the person you're speaking to really looks very tired. And also \"su\" and \"dai\" would be ok, although especially the latter would probably implicitly express \"stop complaining\".\n\"Avanti\" I think transmit more the message of \"go on\", which to be fair would make it suitable for this context.\nThere are also many other words that can be used, as someone already pointed, and I would say that the differences in using some more than others may be also regional, or even just personal choice actually more then anything else. \nAlso for these words I guess the intonation would matter as well to convey the message. \n", "topic": "italian", "url": "https://italian.stackexchange.com/questions/8374"}, {"image": "VQAonline_00022342.png", "question": "Perch\u00e9 questa frutta si chiama \"tabacchiera\"?", "context": "Durante un recente viaggio in Italia ho scoperto che la frutta della fotografia si chiama \"tabacchiera\", parola che mi \u00e8 sembrata molto curiosa. \n\u00a0 \u00a0 \u00a0 \u00a0 \u00a0 \u00a0 \u00a0 \u00a0 \u00a0 \u00a0 \u00a0 \u00a0 \u00a0 \u00a0 \u00a0 \n\u00a0 \u00a0 \u00a0 \u00a0 \u00a0 \u00a0 \u00a0 \u00a0 \u00a0 \u00a0 \u00a0 \u00a0 \u00a0 \u00a0 \u00a0 (Fonte della immagine)\nBeh, perlomeno \u00e8 cos\u00ec come la chiamava il fruttivendolo perch\u00e9 non ho trovato questa accezione in nessuno dei dizionari che ho consultato. Si trova qualcosa su Wikipedia, ma non so fino a che punto le informazioni che si includono l\u00e0 sono affidabili (la prima cosa che si vede \u00e8 una frase evidenziata e indicata come \"senza fonte\"). \nLa mia domanda \u00e8: da dove proviene questo nome, \"tabacchiera\", per questa frutta? Veramente si chiama cos\u00ec perch\u00e9 assomiglia a una tabacchiera? (personalmente non riesco a immaginare una scatoletta per tabacco con la forma di questa frutta). In spagnolo usiamo il termine \"paraguayo\" per designarla (in catalano sono \"paraguaians\", ma non so perch\u00e9 \u00e8 una di quelle parole che, perlomeno a Barcellona, quasi tutti dicono in castigliano): non so neanche da dove proviene questo nome (e non ho trovato nulla al riguardo), forse dovrei chiederlo in Spanish.SE.\n", "answer": "Su questo blog ho trovato:\n\n\u00c8 la pesca \u201cTabacchiera\u201d cos\u00ec chiamata per la sua forma piatta, a\n disco, che ricorda appunto quella di una tabacchiera.\n\nIn questo altro blog invece\n\nIl suo nome scientifico \u00e8 Prunis persica, variet\u00e0 platycarpa ossia dal\n corpo piatto: la tabacchiera appartiene a pieno diritto alla famiglia\n delle pesche. Fu introdotta in Europa attraverso la Persia, pur se le\n sue origini sono cinesi, come per diverse piante della famiglia delle\n Rosacee ormai diventate comuni in Europa quali il ciliegio e il melo.\nIn Sicilia l\u2019antica pesca tabacchiera La sua \u00e8 una storia curiosa. \u00c8 opinione comune che la si chiami tabacchiera nel Sud Italia e\n saturnia o saturnina al Centro Nord. Le forme pi\u00f9 antiche di questa\n pianta si sono sempre chiamate tabacchiera.\n\nCome utilmente sottolineato da @DaG anche su Zanichelli si pu\u00f2 trovare un riferimento:\n\npesca tabacchiera, variet\u00e0 a forma schiacciata, profumo intenso, polpa\n bianca, succosa e dolce, coltivata in aree limitate della Sicilia e\n della Romagna.\n\n", "topic": "italian", "url": "https://italian.stackexchange.com/questions/9815"}, {"image": "VQAonline_00022337.png", "question": "\"Bello bello, chiaro chiaro e tondo tondo\". What does this mean?", "context": "This phrase has appeared several times while I was looking up the word tondo.\n\nBello bello, chiaro chiaro e tondo tondo.\nPretty pretty, clear clear and round round?\n\nReally? Or is it just a repetitive sentence that sounds nice in Italian to calm someone or some animal down?\nWhy would you say this? What would be its closest meaning? Are there other phrases like this that are commonly used? Is THIS a commonly used phrase? \nOkay - I have been looking further... I am also finding the phrase \nchiaro e tondo which seems to mean clear and simple, or bluntly or pure and simple. Could it be that the original phrase is just \"verbal filler\"? \n", "answer": "Given the clarification in the comments, I am quite confident that this is simply a case of repetition for emphasis. The sentence in question is\n\nQuesto \u00e8 il fatto... bello bello, chiaro chiaro e tondo tondo. Mio padre mi insegn\u00f2 che \u00e8 obbligo dare soccorso a mare\n\n(Incidentally this sentence sounds a bit weird. It's not wrong but I would not consider it an example of \"good Italian\". Apparently it comes from some subtitle but I was unable to track down the movie in question. I have to assume that there is some visual context that explains it)\nMy interpretation is that it is a modification of the following\n\nQuesto \u00e8 il fatto... Bello, chiaro e tondo\n\nThat we can roughly translate as\n\nThis is the long and short of it, without any possible ambiguity\n\n(From chiaro e tondo, which means plainly, without mincing words, unambiguously, with an added bello for emphasis)\nThe speaker however repeats all the words in the expression twice in order to add further emphasis. This is not something I would commonly use. However the expression chiaro e tondo is quite common (ex: Te lo dico chiaro e tondo, I'm telling you plainly).\nJust to add to your initial motivation for looking up this word: tono tondo is not a common Italian turn of phrase either. I can imagine using it to discuss some singer or musician (as in Her voice has a beautiful, round, tone in that passage) but that's about it, and even this is fairly uncommon.\n", "topic": "italian", "url": "https://italian.stackexchange.com/questions/6580"}, {"image": "VQAonline_00022415.png", "question": "What's the joke in this 4\u30b3\u30de comic?", "context": "In this 4\u30b3\u30de\u6f2b\u753b{\u307e\u3093\u304c}, a four panel comic, a young attractive woman walks through a station ticket gate, but there's something wrong with her ticket. The station attendant calls after her, first by calling out \u304a\u5ba2{\u304d\u3083\u304f}\u3055\u3093\uff01, which is \"customer\" or \"passenger\", a usual way for a staff to refer to a patron. She does not respond. So then he calls out \u304a\u30cd\u30a8\u3055\u3093\uff01, which is something like \"miss\" or \"young lady\". She still does not respond. He calls out \u304a\u30cd\u30a8\u3055\u307e\uff01, which is again \"miss\" or \"young lady\" or something like that, just with the politeness upped a little. She does not respond.\nFinally, he calls out \u304a\u5b22{\u3058\u3087\u3046}\u3055\u307e\uff01. Which, as far as I've understood its usage, is still just a way of saying \"young woman\" or \"daughter\". My dictionary also says it means \"unmarried woman\".\nMy feeling is that this comic isn't wildly hysterical in any case, but there's some kind of nuance to the fact that she only responds to \u304a\u5b22{\u3058\u3087\u3046}\u3055\u307e\uff01 that is supposed to be kind of amusing. But to my non-native Japanese level of comprehension, it's just different ways of referring to a young woman, so I'm not seeing the extra connotations that support the humour.\nWhat is it about her responding only to \u304a\u5b22{\u3058\u3087\u3046}\u3055\u307e\uff01 that is in any way noteworthy? Is the fact that the man beside her is more distant from her in the last panel relevant (in that he is clearly not her husband)?\nLastly, I thought \u304a\u30cd\u30a8\u3055\u3093\uff01 would be too casual for a station attendant. Wouldn't it translate to something like \"hey, babe\"?\n\n", "answer": "About the nuance of \u304a\u5b22\u3055\u307e.\nThe difference is visual. Someone described as \u304a\u5b22\u69d8, besides being a young unmarried female, has also cultivated (or been raised to have) a sense of upper-class refinement, most immediately evident through her appearance and attitude. Perhaps in between Scarlett O'Hara and Holly Golightly?\nLooking closely at that \u30d5\u30b8\u4e09\u592a\u90ce comic strip, there's a suggestion of erectness in the straight shoulders; of an elegant gate, in the slight to-and-fro swish of the skirt; of luxury, in the chicness of the clothes, particularly the chain-link strap of the purse; and in the long straight hair and slight makeup, of a self-awareness of girlish attractiveness. It takes the ticket attendant a moment to recognize how these signs distinguish her from a woman who just happens to be young, i.e., an \u304a\u306d\u3048\u3055\u3093.\nPerhaps it's proof of this perception of mine that, until I looked it up just now on WWWJDIC, I had always thought \u5b22 meant \"princess\". \nThanks for the question and introducing me to a new comic strip.\n", "topic": "japanese", "url": "https://japanese.stackexchange.com/questions/12054"}, {"image": "VQAonline_00022453.png", "question": "What is a newspaper \u6bb5, exactly?", "context": "Does this bit contain 2\u6bb5, for instance? \n\nWould 3\u6bb5\u629c\u304d refer to a write-up containing 3 such \u6bb5? \n", "answer": "Your picture contains 5\u6bb5\u3010\u3060\u3093\u3011. In other words, this picture is 5\u6bb5\u3010\u3060\u3093\u3011\u7d44\u3010\u3050\u307f\u3011\u306e\u30ec\u30a4\u30a2\u30a6\u30c8.\n\u6bb5\u3010\u3060\u3093\u3011\u629c\u3010\u306c\u3011\u304d refers to a long heading or figure which spans multiple \u6bb5.\nIn your example, the purple heading with \"\u65e5\u672c\u8a9e\u7d44\u7248\u306e\u7279\u5fb4\u3068\u4e8b\u4f8b\" is called 2\u6bb5\u629c\u304d\u306e\u898b\u51fa\u3057, and the other one is 3\u6bb5\u629c\u304d.\nReference: Adobe InDesign CS5 \u6bb5\u629c\u304d\u898b\u51fa\u3057\n", "topic": "japanese", "url": "https://japanese.stackexchange.com/questions/23054"}, {"image": "VQAonline_00022618.png", "question": "what is the dakuten katakana character in the image?", "context": "what is the dakuten katakana character in the image ? \nSeem the image say X\u30b5?What is the X ?\n\n", "answer": "It's \u30b0\u30b5, a common onomatopoeia for a sound of \"stab\". It also describes how someone's harsh word sticks into your heart. It's listed on jisho.org in the form \u3050\u3055\u308a\u3068.\nIt looks like \u30ef, too, but \u30ef does not take dakuten, as you know.\n", "topic": "japanese", "url": "https://japanese.stackexchange.com/questions/56905"}, {"image": "VQAonline_00022580.png", "question": "What does \u305d\u3046\u3059\u308b\u306e mean here?", "context": "I came across this phrase in Yotsubato! manga. (ch.82, pg.35)\n\nSource: http://raw.senmanga.com/Yotsubato!/82/35\nWhat does \u305d\u3046\u3059\u308b\u306e mean here (middle panel)? As far as I can tell, the \u306e is most probably an explanatory \u306e. I think it means \"yea just like that\" or literally \"to do so\" but the literal meaning somehow doesn't feel right. I think it is most probably \"just like that\" from reading the text bubble that comes before it but I am confused here.\nI tried searching \u305d\u3046\u3059\u308b\u306e on the web but couldn't find much that relates to above context. As a phrase what does \u305d\u3046\u3059\u308b\u306e mean here? I couldn't find any more examples that use \u305d\u3046\u3059\u308b\u306e. Any examples would be welcome.\n", "answer": "The \u306e expresses \u547d\u4ee4 (imperative/command). According to \u30c7\u30b8\u30bf\u30eb\u5927\u8f9e\u6cc9:\n\n\u306e\n 2\u20e3\uff3b\u7d42\u52a9\uff3d\u6d3b\u7528\u8a9e\u306e\u9023\u4f53\u5f62\u306b\u4ed8\u304f\u3002\n \uff13 \u5f37\u304f\u6c7a\u3081\u3064\u3051\u3066\u547d\u4ee4\u3059\u308b\u610f\u3092\u8868\u3059\u3002\u300c\u4f59\u8a08\u306a\u3053\u3068\u3092\u8a00\u308f\u306a\u3044\u306e\u300d\u300c\u904a\u3093\u3067\u3070\u304b\u308a\u3044\u306a\u3044\u3067\u52c9\u5f37\u3059\u308b\u306e\u300d\n\nAnd \u660e\u93e1\u56fd\u8a9e\u8f9e\u5178 says:\n\n\u306e\n \u3281\u3018\u7d42\u52a9\u3019\n \u2779 \u8efd\u304f\u547d\u3058\u308b\u306e\u306b\u4f7f\u3046\u3002\u300c\u3055\u3063\u3055\u3068\u7740\u66ff\u3048\u308b\u306e\u300d\u300c\u5f37\u3044\u5b50\u306f\u6ce3\u304b\u306a\u3044\u306e\u300d\n \u25c8(\u8868\u73fe) \u3281\u306f\u591a\u304f\u5973\u6027\u304c\u4f7f\u3046... \u2463\u306f\u5b50\u4f9b\u306a\u3069\u76ee\u4e0b\u306e\u76f8\u624b\u306b\u5bfe\u3057\u3066\u4f7f\u3046\u3002\n\nThis \u306e is a sentence-ending particle (\u7d42\u52a9\u8a5e) expressing a relatively light command. It's often used by females, normally towards someone who is inferior to or younger than the speaker. \n\n\u305d\u3046\u3002\u305d\u3046\u3059\u308b\u306e\u3002\nlit. \"Yes. Do (it) that way.\" \n\nSo here the girl is talking to her dad a little arrogantly (but that might be how she usually talks to him?)\n", "topic": "japanese", "url": "https://japanese.stackexchange.com/questions/47606"}, {"image": "VQAonline_00022371.png", "question": "What is the first meaning of \u305f\u305f\u304d\u53f0?", "context": "\u305f\u305f\u304d\u53f0 means a springboard for discussion.\nAccording to my dictionary, this word's first meaning is \"chopping block\".\nOn Google Images, it does not look like the plastic/wood board I use to cut meat/vegetables:\n\nSo what is the non-abstract meaning of \u305f\u305f\u304d\u53f0?\nALC does not have it.\n", "answer": "There are a lot of meanings related to \u305f\u305f\u304d\u53f0 (as noted by the comments and posts above.)\nIn one case (from your picture posted above,) \u305f\u305f\u304d\u53f0 actually means something like \"a pounding stand\" for baseball gloves. (Along with a \u305f\u305f\u304d\u53f0, there is also another tool used when pounding baseball gloves like this, called a \u305f\u305f\u304d\u68d2; the actual glove-hitting tool.)\nWhen you buy a typical baseball glove, it is usually made out of some sort of leather. Leather usually requires some sort of maintenance from time to time (and especially before a glove, in this case, is first used.)\nOne type of glove maintenance is the art of \"creating a pocket\" so that a baseball fielder can more-easily catch a fly-ball... or field a ground ball... during a game of baseball.\nThe following (commercial) website describes what the process of \"creating a pocket\" can look like:\nhttp://bba.co.jp/kakou-teire_shinka.html (see step 7)\n", "topic": "japanese", "url": "https://japanese.stackexchange.com/questions/4859"}, {"image": "VQAonline_00022401.png", "question": "Words for Ginkgo in Japanese: \u304e\u3093\u304d\u3087\u3046 and \u304e\u3093\u306a\u3093", "context": "I decided to read about the words for Ginkgo in Japanese (as in Ginkgo biloba).\nI was surprised to learn that Ginkgo comes from Japanese! According to\nWikipedia, Engelbert Kaempfer, while writing the Amoenitatum exoticarum, read the kanji \u9280\u674f as \u304e\u3093\u304d\u3087\u3046. He romanized this reading as Ginkjo, which was then unfortunately misprinted as Ginkgo, resulting in the spelling used in many Western languages today.\n\nFrom the same article, I see that \u30a4\u30c1\u30e7\u30a6 is a common word for Ginkgo plants, apparently from the Chinese \u9d28\u811a. It seems that this word was assigned to the kanji \u9280\u674f as a jukujikun reading, and we can see that both readings existed 300 years ago, because Kaempfer also transcribed \u30a4\u30c1\u30e7\u30a6 (as Itsjo).\nIn fact, \u9280\u674f has a third reading, \u304e\u3093\u306a\u3093. Similar to \u304e\u3093\u304d\u3087\u3046, it appears that this reading was made by putting two on'yomi together: \u304e\u3093\uff0b\u3042\u3093\uff1d\u304e\u3093\u3042\u3093\u2192\u304e\u3093\u306a\u3093. But here's what makes me curious about the difference between the readings: \u5927\u8f9e\u6797 and \u5927\u8f9e\u6cc9 only list \u304e\u3093\u306a\u3093, not \u304e\u3093\u304d\u3087\u3046. And 300 years ago, Kaempfer only transcribed \u304e\u3093\u304d\u3087\u3046, not \u304e\u3093\u306a\u3093.\nIt seems that both \u304e\u3093\u304d\u3087\u3046 and \u304e\u3093\u306a\u3093 mean the same thing (Ginkgo plants or seeds). But what is the relationship between the two readings?\n\nDid \u304e\u3093\u306a\u3093 replace \u304e\u3093\u304d\u3087\u3046, making the latter obsolete?\nOr are both readings still used?\n\n", "answer": "\u674f has as three \u97f3\u8aad\u307f, namely \u30a2\u30f3, \u30ad\u30e7\u30a6, \u30b3\u30a6, of which Kaempfer transcribed the first two (including \u30ae\u30f3\u30ca\u30f3, as the picture shows). I don't consider too far a stretch of the imagination to think that Kaempfer simply asked for readings \u9280\u674f, and was told that you can read it as \u30ae\u30f3\u30ad\u30e7\u30a6, \u30ae\u30f3\u30ca\u30f3, or \u30a4\u30c1\u30e7\u30a6.\nThe text accompanying the picture says that the seed was called \u30ae\u30f3\u30ca\u30f3 (already in 1700). (He describes it resembling a \"Persian pistachio\", just twice as big.) He also says that \u30a4\u30c1\u30e7\u30a6 was the common name for the tree.\nIn current usage, too, \u9280\u674f is read \u304e\u3093\u306a\u3093 and means the seed of the Gingko tree. (In fact many supermarkets do sell Gingko seeds under the name \u9280\u674f.) The tree itself is called \u30a4\u30c1\u30e7\u30a6, which may also be written \u9280\u674f, by \u719f\u5b57\u8a13, like you said.\nI think we can conclude that \u30ae\u30f3\u30ca\u30f3 did not replace \u30ae\u30f3\u30ad\u30e7\u30a6, but rather \u30ae\u30f3\u30ad\u30e7\u30a6 has fallen out of use with the common name \u30a4\u30c1\u30e7\u30a6 surviving in its place.\n", "topic": "japanese", "url": "https://japanese.stackexchange.com/questions/11457"}, {"image": "VQAonline_00022505.png", "question": "definition and usage of ~\u3044\u305a\u308c\u3082", "context": "I was struggling to translate the following poster in a rigorous manner, more specifically to Korean, which is a LOT easier than translating Japanese to English. \n\nIt says \"2016\u5e74\u30ec\u30ae\u30e5\u30e9\u30fc\u30b7\u30fc\u30ba\u30f3\u516c\u5f0f\u6226\u306e\u897f\u6b66\u30d7\u30ea\u30f3\u30b9\u30c9\u30fc\u30e0\u958b\u50ac\u8a66\u5408\u306b\u304a\u3044\u3066\u3044\u305a\u308c\u3082\u5f53\u65e5\u3001\u6b8b\u5e2d\u304c\u3042\u308b\u5834\u5408\u306e\u307f\" \nNow I have divided the phrase \"-\u958b\u50ac\u8a66\u5408\u306b\u304a\u3044\u3066\u3044\u305a\u308c\u3082\u5f53\u65e5\" into\n\"-\u958b\u50ac\u8a66\u5408\u306b\u304a\u3044\u3066\" / \"\u3044\u305a\u308c\u3082\u5f53\u65e5\"\n\"Regarding Prince Dome Games\" / \"???? the day\" \nAnd from the context I've concluded that the phrase really means\n\"All Prince Dome games regardless of the day\" and subsequently,\n\"The game is free only if it is a regular season game @ Prince Dome, and if there are free seats left.\"\nWhich sounds pretty reasonable, but I still can't really pinpoint the definition of the word \"\u3044\u305a\u308c\u3082,\" and Googling didn't help at all. \nI was thinking \"all,\" but why the phrase when we have \u5168\u90e8 or \u3059\u3079\u3066? \n", "answer": "\u5168\u90e8 or \u3059\u3079\u3066 sound more like all of them (in a batch, or all together), whereas \u3044\u305a\u308c\u3082 sounds more like each of them, any of them (individually, separately). In the context of your example, \u3044\u305a\u308c\u3082 applies to the games -- not the days -- which have free seating only if \u5f53\u65e5\u3001\u6b8b\u5e2d\u304c\u3042\u308b -- i.e., seating in any given game is free if there are seats left on the day of that game.\nNote too that the \"free\" here is \u81ea\u7531, meaning that seats are not assigned. This is not \"free\" as in \u7121\u6599, meaning that there is no charge.\nPS: \u3044\u305a\u308c\u3082 and \u3069\u308c\u3082 are synonyms. Modern \u3069\u308c is a phonetic evolution of older \u3044\u305a\u308c.\n", "topic": "japanese", "url": "https://japanese.stackexchange.com/questions/33132"}, {"image": "VQAonline_00022859.png", "question": "Meya - mi yodeya?", "context": "\nWho knows one hundred?\nPlease cite/link your sources, if possible. At some point in the future, subject to holiday and user activity delay, I will:\n\nUpvote all interesting answers.\n\nAccept the best answer.\n\nGo on to the next number.\n\n\n", "answer": "Ok, 100, here we go!\n\nThe Heichal of the second Beish Hamikdash was 100 amos tall. -msh210\n..and 100 amos long.\n..and 100 amos wide.\nBrochos to say each day -JeremyR\n100 times \u05e9\u05f4\u05dd is written in Tehillim. -Yahu \n100 amos2 where the Kohanim walked on top of the exterior altar. -msh210\n100 children of Kayin. -Gershon Gold\n100 people that Elish fed miraculously. -msh210\n100 cc's in the Chazon Ish's K'beitzah -msh210\nShem was 100 years old when his son Arpachshad was born. -msh210\n100 prophets that OVadya hid from Izevel. -msh210\n100 is the number used for large examples in Rabbinic literature. -msh210\n100 amos that water must travel underground before it's considered a spring when it comes back up. -msh210\n100 amos tall was Adam. -msh210\n100 amos around a grave that there might still be body parts. -msh210\n100 amos is how far the carob tree moved. -msh210\n100 \u05d6\u05d5\u05d6 in a woman's second marriage kesubah. -Gershon Gold\n100 walk-amos is the duration of a light sleep. -Isaac Moses\n100 Shofar blasts on Rosh Hashana. -Isaac Moses\n100 species of impure birds in the east. -Isaac Moses\n100 is how much Yaakov paid for real-estate near Sh'chem. -msh210\n100 years will still be young at death says Yechezkel HaNavi. -msh210\n100 years old when Sarah still looked like 20. -msh210\n100 zuz wants another 100 zuz. -Isaac Moses\n100 silver that a \u05de\u05d5\u05e6\u05d9\u05d0 \u05e9\u05dd \u05e8\u05e2 pays. -Yahu\n100 times that \"\u05e9\u05de\u05d9\u05dd\" is written in Torah. -Yahu\nTorah learning is 100 times more valuable in difficult circumstances. -Isaac Moses\n100 times that you must have said Mashiv Haruach, before you can assume that you say the right one without thinking. -ArghMo\nHeter 100 Rabbis. -ArghMo\n100 silver sockets for the Mishkan. -YDK\n100 answers for the missing miracle question. -Isaac Moses\nMeah Shearim. -Yahu\nThe cross-sectional area of the Mishkan (viewed from the East or West) was 100 square cubits (10x10). -Isaac Moses\n100 times the chullin must outnumber the terumah to be able to use the mixture. -Dave\n100 years old was Avraham when Yitzchak was born. -jutky\n100 Rabbis from R' Moshe Feinstein's home town. -Alex\n100 amos was the length of the Mishkan's courtyard. -Dave\n100 zuz in a maneh. -Shalom\n100 times the word \u05d1\u05e0\u05d9\u05de\u05df is in Tanach. -Shalom\n100 times the word \u05d4\u05d0\u05d7\u05d3 is in Tanach. -Shalom\n100 reviews which is incomparable to 101 reviews (Chagiga 9b) -HodofHod because..\n100 reviews is what was common in Talmudic times (brought in Tanya Ch. 15) -HodofHod\n100 zuz is the value of virginity at the time of marriage.\n100 clusters of raisins that Tziva brought to Dovid. (2 Shmuel 16) -HodofHod \n100 summer fruits that Tziva brought to Dovid. (ibid.) -HodofHod\n\"The city that went forth a thousand shall have a hundred left\" (Amos 5:3) -HodofHod\n\"And five of you shall chase a hundred, and a hundred of you shall chase ten thousand\". (Vayikra 26:8). -HodofHod\n\u05d1\u05df \u05de\u05d0\u05d4 \u05e9\u05e0\u05d9\u05dd \u05db\u05d0\u05d9\u05dc\u05d5 \u05e2\u05d1\u05e8 \u05d5\u05d1\u05d8\u05dc \u05de\u05df \u05d4\u05e2\u05d5\u05dc\u05dd. -Dave\n\u05d0\u05b7\u05e8\u05b0\u05ea\u05b7\u05bc\u05d7\u05b0\u05e9\u05b7\u05c1\u05e1\u05b0\u05ea\u05b0\u05bc\u05d0 guaranteed Ezra 100 talents of silver. \n..and 100 measures of wheat.\n..and 100 baths of wine.\n..and 100 baths of oil. (Ezra 7:22) -HodofHod\n\"A rebuke entereth deeper into a man of understanding than a hundred stripes into a fool.\" (Mishlei 17:10) -HodofHod\n100 amos was the length of King Solomon's house. (1 Kings 7:2) -HodofHod\n100 sheep that King Solomon's court went through, a day! (1 Kings 5:3) -HodofHod\n100 bullocks that were sacrifice when the 2nd Beis Hamikdash was completed. (Ezra 6:16) -HodofHod\n100 talents of silver the Ammon gave to King \u05d9\u05d5\u05b9\u05ea\u05b8\u05dd. (Chronicles 1 27:5) -HodofHod\n100 chariots that Dovid reserved from what he took from \u05d4\u05b2\u05d3\u05b7\u05d3\u05b0\u05e2\u05b6\u05d6\u05b6\u05e8 \u05de\u05b6\u05dc\u05b6\u05da\u05b0-\u05e6\u05d5\u05b9\u05d1\u05b8\u05d4 . (Chronicles 1 18:4) -HodofHod\n\u05e4\u05b7\u05e8\u05b0\u05e2\u05b9\u05d4 \u05e0\u05b0\u05db\u05b9\u05d4 fined Israel 100 talents of silver during the reign of \u05d9\u05b0\u05d4\u05d5\u05b9\u05d0\u05b8\u05d7\u05b8\u05d6. (Kings 2 23:33) -HodofHod\nGematria of:\n\"\u05dc\u05db\u05df\" -msh210\n\"\u05de\u05d3\u05d4 \u05d1\u05de\u05d3\u05d4\" -msh210\n\"\u05de\u05d3\u05d5\u05df\" -msh210\n\"\u05dc\u05da \u05dc\u05da\" -Gershon Gold\n\"\u05de\u05d5\u05d3\u05d9\u05dd\" -Gershon Gold\n\"\u05d4\u05de\u05dc\u05db\u05d4\" -Gershon Gold\n\"\u05db\u05e3\" -Gershon Gold\n\"\u05e4\u05da\" -Gershon Gold\n\"\u05e2\u05dc\"\n\"\u05e1\u05dd\"\n\"\u05d9\u05de\u05d9\u05dd\"\n\"\u05db\u05dc\u05d9\u05dd\"\n\"\u05d1\u05df \u05d7\u05d9\u05dc\"\n\"\u05d9\u05d4\u05d5\u05e2\u05d3\u05d4\"\n\"\u05d9\u05de\u05dc\u05da\"\n\"\u05e1\u05db\u05da\"\n\"\u05de\u05e1\"\n\"\u05de\u05de\u05da\"\n\"\u05de\u05d9\u05db\u05dc\"\n\"\u05de\u05dc\u05db\u05d9\"\n\"\u05e1\u05dc\u05d9\"\n\"\u05de\u05db\u05dd\"\n\"\u05e2\u05d5\u05d3\u05da\"\n\"\u05d4\u05de\u05d9\u05de\u05d4\"\n\"\u05de\u05d9\u05de\u05d9\"\n\"\u05e0\u05dc\u05da\"\n\"\u05d9\u05dc\u05d9\u05df\"\n\"\u05d8\u05de\u05d0\u05d9\u05dd\"\n\"\u05d9\u05d2\u05d0\u05dc\u05e0\u05d5\"\n\"\u05d5\u05d1\u05d7\u05e1\u05d3\u05da\"\n\"\u05d5\u05d9\u05d7\u05dc\u05de\u05d5\"\n\"\u05d2\u05d1\u05d9\u05e2\u05d9\u05d4\"\n\"\u05d9\u05d9\u05e1\u05da\"\n\"\u05de\u05e0\u05d9\"\n\"\u05d1\u05dc\u05d7\u05de\u05da\"\n\"\u05dc\u05d9\u05e0\u05d9\"\n\"\u05db\u05dc\u05db\u05dc\"\n\"\u05d7\u05e6\u05d1\"\n\"\u05d5\u05d9\u05dc\u05d3\u05d9\u05dd\"\n\"\u05d9\u05e4\u05d9\"\n\"\u05d1\u05db\u05d5\u05db\u05d1\u05d9\u05dd\"\nAnswers in this answer.\n\n(Ok, I know the gematrias got a little out there, but you try coming up with 50 other references for 100!)\n", "topic": "judaism", "url": "https://judaism.stackexchange.com/questions/3027"}, {"image": "VQAonline_00022911.png", "question": "Difference between men and women regarding drinking water before the day kidush", "context": "In Chukas Hanoshim of the Ben Ish Hai chapter 43 (below) he writes that women cannot drink anything on Shabbos morning until kidush, but men can drink water before shachris. \nWhat might be his source/reasoning?\nDoes anyone argue with his decision?\n\n", "answer": "The Ben Ish Chai explains his own position in Year 2 Parashat Bereishit #18\n\n\u05e7\u05d9\u05d3\u05d5\u05e9 \u05d4\u05dc\u05d9\u05dc\u05d4 \u05d0\u05d9\u05e0\u05d5 \u05ea\u05dc\u05d5\u05d9 \u05d1\u05ea\u05e4\u05dc\u05ea \u05e2\u05e8\u05d1\u05d9\u05ea, \u05e9\u05d0\u05dd \u05d9\u05e8\u05e6\u05d4 \u05dc\u05e7\u05d1\u05dc \u05e9\u05d1\u05ea \u05de\u05d1\u05e2\u05d5\u05d3 \u05d9\u05d5\u05dd \u05d5\u05dc\u05e7\u05d3\u05e9 \u05d5\u05dc\u05d0\u05db\u05d5\u05dc \u05d5\u05dc\u05d4\u05ea\u05e4\u05dc\u05dc \u05e2\u05e8\u05d1\u05d9\u05ea \u05d1\u05dc\u05d9\u05dc\u05d4, \u05e8\u05e9\u05d0\u05d9, \u05d5\u05e8\u05e7 \u05e6\u05e8\u05d9\u05da \u05e9\u05d9\u05ea\u05d7\u05d9\u05dc \u05dc\u05d0\u05db\u05d5\u05dc \u05d7\u05e6\u05d9 \u05e9\u05e2\u05d4 \u05e7\u05d5\u05d3\u05dd \u05d6\u05de\u05df \u05e7\u05e8\u05d9\u05d0\u05ea-\u05e9\u05de\u05e2; \u05d0\u05d1\u05dc \u05e7\u05d9\u05d3\u05d5\u05e9 \u05d4\u05d9\u05d5\u05dd \u05ea\u05dc\u05d5\u05d9 \u05d1\u05ea\u05e4\u05dc\u05d4, \u05d3\u05db\u05dc \u05d6\u05de\u05df \u05e9\u05dc\u05d0 \u05d4\u05ea\u05e4\u05dc\u05dc \u05e9\u05d7\u05e8\u05d9\u05ea, \u05dc\u05d0 \u05d7\u05dc \u05e2\u05dc\u05d9\u05d5 \u05d7\u05d5\u05d1\u05ea \u05e7\u05d9\u05d3\u05d5\u05e9; \u05d5\u05dc\u05db\u05df, \u05d1\u05d9\u05d5\u05dd \u05e9\u05d1\u05ea \u05d1\u05d1\u05d5\u05e7\u05e8 \u05d9\u05d5\u05db\u05dc \u05dc\u05e9\u05ea\u05d5\u05ea \u05de\u05d9\u05dd \u05e7\u05d5\u05d3\u05dd \u05ea\u05e4\u05dc\u05d4, \u05de\u05e4\u05e0\u05d9 \u05e9\u05db\u05d9\u05d5\u05df \u05e9\u05dc\u05d0 \u05d4\u05ea\u05e4\u05dc\u05dc, \u05dc\u05d0 \u05d7\u05dc \u05e2\u05dc\u05d9\u05d5 \u05d7\u05d5\u05d1\u05ea \u05e7\u05d9\u05d3\u05d5\u05e9. \u05d5\u05db\u05dc \u05d6\u05d4 \u05d4\u05d5\u05d0 \u05d1\u05d0\u05e0\u05e9\u05d9\u05dd \u05d3\u05d7\u05dc \u05e2\u05dc\u05d9\u05d4\u05dd \u05d7\u05d9\u05d5\u05d1 \u05ea\u05e4\u05dc\u05d4 \u05d1\u05d1\u05d5\u05e7\u05e8 \u05d0\u05d7\u05e8 \u05e2\u05de\u05d5\u05d3 \u05d4\u05e9\u05d7\u05e8, \u05d0\u05d1\u05dc \u05d4\u05e0\u05e9\u05d9\u05dd, \u05de\u05e9\u05e2\u05dc\u05d4 \u05e2\u05de\u05d5\u05d3 \u05d4\u05e9\u05d7\u05e8 \u05d7\u05dc \u05e2\u05dc\u05d9\u05d4\u05dd \u05d7\u05d5\u05d1\u05ea \u05e7\u05d9\u05d3\u05d5\u05e9, \u05d5\u05d0\u05e1\u05d5\u05e8\u05d9\u05df \u05dc\u05e9\u05ea\u05d5\u05ea \u05de\u05d9\u05dd \u05e7\u05d5\u05d3\u05dd \u05e7\u05d9\u05d3\u05d5\u05e9; \u05de\u05e4\u05e0\u05d9 \u05d3\u05d9\u05e9 \u05d0\u05d5\u05de\u05e8\u05d9\u05dd: \u05d4\u05e0\u05e9\u05d9\u05dd \u05e7\u05d9\u05d9\u05de\u05d9 \u05d0\u05d3\u05d9\u05e0\u05d0 \u05d3\u05d0\u05d5\u05e8\u05d9\u05d9\u05ea\u05d0, \u05e9\u05d0\u05d9\u05df \u05dc\u05d4\u05dd \u05e0\u05d5\u05e1\u05d7 \u05e7\u05d1\u05d5\u05e2 \u05d5\u05dc\u05d0 \u05d6\u05de\u05df \u05e7\u05d1\u05d5\u05e2 \u05dc\u05ea\u05e4\u05d9\u05dc\u05d4, \u05d5\u05d1\u05e4\u05e2\u05dd \u05d0\u05d7\u05d3 \u05d1\u05d9\u05d5\u05dd \u05e9\u05d9\u05d0\u05de\u05e8\u05d5 \u05e0\u05d5\u05e1\u05d7 \u05ea\u05e4\u05d9\u05dc\u05d4, \u05d9\u05e6\u05d0\u05d5 \u05d9\u05d3\u05d9 \u05d7\u05d5\u05d1\u05d4, \u05d5\u05d0\u05dd \u05db\u05df \u05dc\u05d3\u05d9\u05d3\u05d4\u05d5, \u05d0\u05d9\u05df \u05d4\u05e7\u05d9\u05d3\u05d5\u05e9 \u05ea\u05dc\u05d5\u05d9 \u05d1\u05ea\u05e4\u05d9\u05dc\u05d4, \u05d5\u05dc\u05e4\u05d9\u05db\u05da, \u05de\u05e9\u05e2\u05dc\u05d4 \u05e2\u05de\u05d5\u05d3 \u05d4\u05e9\u05d7\u05e8 \u05d0\u05e1\u05d5\u05e8\u05d9\u05df \u05dc\u05d8\u05e2\u05d5\u05dd \u05de\u05d9\u05dd \u05e7\u05d5\u05d3\u05dd \u05e7\u05d9\u05d3\u05d5\u05e9, \u05d5\u05db\u05de\"\u05e9 \u05d4\u05e8\u05d1 \"\u05d7\u05e1\u05d3 \u05dc\u05d0\u05d1\u05e8\u05d4\u05dd\" \u05d1\u05ea\u05e9\u05d5\u05d1\u05d4, \u05d1\u05d0\"\u05d7, \u05e1\u05d9' \u05d5\u05d0\"\u05d5; \u05d9\u05e2\"\u05e9.\u200f\n ...women from dawn are obligated in Kiddush, and are [hence] forbidden to drink water before Kiddush, for there are those who say that women follow the biblical rule of having no fixed text or times for prayer, and in one time in the day when they say a prayer they fulfill their obligation, and if so for them Kiddush is not dependent on prayer, and therefore from dawn they are prohibited to taste water before Kiddush... (my translation of the part I italicized)\n\n", "topic": "judaism", "url": "https://judaism.stackexchange.com/questions/67090"}, {"image": "VQAonline_00022909.png", "question": "(How) Are these aravos kosher?", "context": "According to my understanding of Shulchan Aruch OC 647:1, the edges of the leaves of the aravos should not have serrated edges, but should rather be smooth:\n\n\u05e2\u05b7\u05e8\u05b0\u05d1\u05b5\u05d9 \u05e0\u05b7\u05d7\u05b7\u05dc \u05d4\u05b8\u05d0\u05b8\u05de\u05d5\u05bc\u05e8 \u05d1\u05bc\u05b7\u05ea\u05bc\u05d5\u05b9\u05e8\u05b8\u05d4 [...] \u05d5\u05bc\u05e4\u05b4\u05d9\u05d5 \u05d7\u05b8\u05dc\u05b8\u05e7\n\n(See also Mishna Berura there, note 1, which seems to support my reading of the Mechaber)\nWhen I went to the daled minim shuk tonight, all the aravos I saw appeared to have serrated edges, unlike the ones I'm used to from America (from my own backyard). The ones pictured below claim to be under the supervision of the Badatz Edah HaChareidis (you can see part of the logo, in reverse, in the picture). I also noticed some sets with the name of Rav Nissim Karelitz on them. (I don't have a picture of American aravos with me, at the moment)\n\nIs this a problem with the aravos? If they are, indeed, kosher for use, how should the above-cited portion of Shulchan Aruch be understood?\n", "answer": "Small serations are kosher, large ones are not. See Halachipedia's entry on aravos.\n", "topic": "judaism", "url": "https://judaism.stackexchange.com/questions/64186"}, {"image": "VQAonline_00022883.png", "question": "Which shita are those wearing extra long tallit katan following?", "context": "Some rabbonim are wearing really long and large talith katan. For example Rav Shmuel Auerbach shlita (picture below).\nRav Elyashiv ztl used to wear a very long one too. \nWhich shita are they following?\nThis is quite a bit longer than Chazon Ish shiur; I wonder if perhaps it has something to do with the Gra, as I read that he requires the talith katan to go down to the knees. (See note 10 here.)\n\n", "answer": "The answer is like you said that it is the opinion of the Gr\"a. It is quoted from Rabbi Chaim of Volozhin that the Gr\"a held any smaller sized Talis Katan to be Hotzaah on Shabbos. Reb Menashe Klein (Mishneh Halakhot) and Rav Moshe Sternbuch in Halichos Hagr\"a pg. 38 explain that the opinion is based on the fact that the Torah only required a garment that is of size one will use it to go into a marketplace. Therefore they explain that in the time of the Gr\"a they only wore long garments, so only with a long garment one fulfils tzitzis. However, in our time that it is the norm to where short jackets even the Gr\"a will hold you fulfil the mitzvah.\nAlso Rabbi Sternbuch brings that Rav Moshe Feinstein held that it is impossible for the Gr\"a to have said such a thing and he held it was the mistake of a scribe and one does not need to be concerned for this opinion. \nObviously the above is an answer to our minhag, however it would appear that the Rabbonim who where these tzitzis do so only as a Chumra.\n", "topic": "judaism", "url": "https://judaism.stackexchange.com/questions/30224"}, {"image": "VQAonline_00022890.png", "question": "Who is this (Haredi) Rabbi?", "context": "Who is this Rabbi?\n\nThis is a still taken from the beginning of a documentary on Haredi jews in Jerusalem: The Anti-zionist Jews of Jerusalem\nI am curious to learn who this Rabbi is, why he is dressed this way and what kind of festivity we are witnessing here?\n", "answer": "He is the Kaliver Rebbe.\n\n[Rabbi] Menachem Mendel Taub (born 1923) is the Rebbe of the Kaliv Hasidic dynasty. Born in Transylvania in 1923, he is seventh in a direct paternal line to the founder of the dynasty, Rabbi Yitzchak Izak of Kaliv, a disciple of the Rebbe Reb Elimelech of Lizhensk.\n\nAn extremely talented Rabbi who recently moved his headquarters from Bnei Braq to Jerusalem. He speaks quite a few languages, and as a survivor of Auschwitz, he regularly speaks passionately about the Holocaust and Jewish unity. As a twin, Dr Mengele experimented on him and his sister (who didn't survive). \nAs a Chassidic Rebbe, he regularly conducts tischen where his Chassidim will join him for his meal. This particular picture is likely on Tu B'Shvat when Rebbes are known to distribute fruit.\nAnd, video caption aside, he is NOT anti-Zionist!\nUPDATE: The Kaliver Rebbe passed away in Jerusalem, on Nisan 23, 5779 [April 28, 2019] (aged 95\u201396).\n", "topic": "judaism", "url": "https://judaism.stackexchange.com/questions/43749"}, {"image": "VQAonline_00022899.png", "question": "KORC - Kosher Symbol", "context": "I was in Costco today and saw chopped romaine lettuce with the KORC symbol. I was wondering if anyone knows if this symbol is accepted by the OU or other large Kashrus organizations.\n\n", "answer": "The Atlanta Kashrus Agency does not recommend the KORC.\n\nThe AKC does not recommend the KORC certification. Lettuce products\n with this certification have been found to have have insects and\n require additional washing and checking.\n\n", "topic": "judaism", "url": "https://judaism.stackexchange.com/questions/53795"}, {"image": "VQAonline_00022903.png", "question": "The Seder Plate according to the Rama", "context": "The most common Seder plate orders I have seen are the Rama, the Gra, and the Ari.\nBut when looking at just the Rama's Seder Plate, different places show even that differently.\nAccording to the Star K\n\nBut I have also seen in other places/web-sites/Haggadot that there are only five items on the Rama's Seder plate; often Salt Water is omitted and/or the three Matzos are omitted.\nWhy is there not one consistent view of what the Rama's Seder Plate looks like?\n", "answer": "The Rema is really very clear - there are six items on the seder plate plus the matzot. I've never seen anyone who tried to claim otherwise, but if you're seeing other descriptions in modern haggadot, my feeling is that the basic factor at play here is modern revisionism. With regard to leaving out the salt water, most of the other minhagim (Gra, Ari zl, etc) don't have salt water on the seder plate. Since most people do not follow the Rema's order for the seder plate, it's simply unusual. The only people I know of who have tried to keep the Rema's order are the Yekkes (German Jews). Since most people nowadays don't have the salt water on the seder plate, the authors of contemporary haggadot feel comfortable leaving it off \"because it's not done\" even if they're ignoring what the Rema actually said. Their goal is to give options that work for today's generation. As minhagim change, they print what they expect their audience to want.\nAs for the matzot - it's a similar story. There isn't room on the modern seder plate for today's modern sized matzot (I have no idea what size matzot the Rema had - they may have been a different size back then). Therefore, people leave it off. Even the German Jews who tried to keep the Rema's arrangement started making multi-layer seder plates in order to keep the matzot on the seder plate technically, but not have them interfere with the other items. Since it's just not done anymore, many authors feel comfortable providing something which is more usable (in their mind) even if it's not technically accurate.\n", "topic": "judaism", "url": "https://judaism.stackexchange.com/questions/56602"}, {"image": "VQAonline_00023008.png", "question": "Korean cosmetic-sample - what is it?", "context": "dear ones,\nI got this little cosmetic-sample, and have problem to translate it... What is it?\nCan somebody help me, please? \nBest regards, xoxo: Sophie \n", "answer": "It is product by brand calls \"TonyMoly\" and it is: Floria Brightening Foam Cleanser \n(Grapefruit extract to create clear and bright skin.\n)\nLink to read about product (choose the Grapefruit one to more information...)\nBTW you can translate image here: https://translate.yandex.com/ocr\n", "topic": "korean", "url": "https://korean.stackexchange.com/questions/5643"}, {"image": "VQAonline_00022995.png", "question": "What does this notification mean?", "context": "My Android phone from Korea has run into some serious software issues and is now flashing the notification below:\n\nMy limited Korean knowledge cannot understand what this means, except that the title reads 'Android upgrade has stopped' or so. I'd really appreciate if someone can help me by explaining the meaning.\nThanks in advance. \n", "answer": "Android is in the process of upgrading. (\uc911 means \"in the middle\")\nYou have 139 apps (9 are being optimized), so it may take some time. Please don't let your phone run out of battery (charge).\n", "topic": "korean", "url": "https://korean.stackexchange.com/questions/3071"}, {"image": "VQAonline_00022997.png", "question": "Is \"\uc62c\ub9bc\ud53d \uaca8\uc6b8 \uac8c\uc784\" a correct/natural way to say \"Winter Olympics\"?", "context": "\nI noticed some of the BBC's coverage of the Winter Olympics is calling them \"\uc62c\ub9bc\ud53d \uaca8\uc6b8 \uac8c\uc784\".\nA Korean friend remarked that this sounded a bit awkward, and that \ub3d9\uacc4\uc62c\ub9bc\ud53d was more normal, as per the title of the official site: \ud3c9\ucc3d\ub3d9\uacc4\uc62c\ub9bc\ud53d.\nNevertheless, could \"\uc62c\ub9bc\ud53d \uaca8\uc6b8 \uac8c\uc784\" be considered correct? Or is it a mistake arising from literally translating \"Olympic Winter Games\" word-by-word?\n", "answer": "Your friend is right. \nKorean call \"The Winter Olympics\", \"\ub3d9\uacc4 \uc62c\ub9bc\ud53d\".\n(\ub3d9\uacc4 means 'winter(\ub3d9) season(\uacc4)').\nI think '\uc62c\ub9bc\ud53d \uaca8\uc6b8 \uac8c\uc784' is just translated by word-by-word.\n", "topic": "korean", "url": "https://korean.stackexchange.com/questions/3288"}, {"image": "VQAonline_00022999.png", "question": "Etymology of Native Korean Numbers and Related Words", "context": "So I collected some native Korean numbers and their old forms into a few columns. The last two are words specifically for days.\n\nThe Roots column is what I observe to be common across the row, not something I know for a fact.\nThe dictionaries I consulted show the old forms, but do not mention etymological information earlier than that. I want to know if there have been any academic insights into the formation of these words.\n", "answer": "AFAIK, this is as far back as you can go. Korean is linguistically regarded as a language isolate, which has no sister languages anywhere else in the world. The comparative method, which is used for reconstructing older forms of languages, does not work for Korean.\nThe forms you find in the \uc61b\ud55c\uae00 columns are the oldest attested forms of the words (from the 15th century). There are no older forms of those words that has been discovered, and they likely will never be.\n", "topic": "korean", "url": "https://korean.stackexchange.com/questions/3356"}, {"image": "VQAonline_00022991.png", "question": "Is there any rule for the pronunciation of \ub2eb\ud788\ub2e4 as tachida not tathida?", "context": "Is there any rule for the pronunciation of \ub2eb\ud788\ub2e4 as tachida not tathida as in the picture below?\n\n", "answer": "This is called \uad6c\uac1c\uc74c\ud654 (palatalization), and it's one of several assimilation rules in Korean. It occurs when \u3137 or \u314c is in the \ubc1b\uce68, and is followed by a syllable beginning with \uc774:\n\n\uac19- + \uc774 = \uac19\uc774 [\uac00\uce58]\n\uad73- + \uc774 = \uad73\uc774 [\uad6c\uc9c0]\n\nIn the example you've given, the \u3137 + \u314e combination produce [\u314c] sound, but since it is followed by the \uc774 sound, palatalization occurs, resulting in [\uce58] pronunciation.\n(Note that this rule only applies when the \u3137/\u314c is in the \ubc1b\uce68 (and thus ending the morpheme). Historically, this sound change occurred across the board, but where it occurred in the middle of the word, the orthography was updated:\n\n\ud16c\ub514 -> \ucc9c\uc9c0\n\nNow, exceptions like \ub514\ub514\ub2e4 remain; this should be pronounced as [\ub514\ub514\ub2e4], without palatalization, because the \u3137 is not in the \ubc1b\uce68).\n", "topic": "korean", "url": "https://korean.stackexchange.com/questions/2479"}, {"image": "VQAonline_00022996.png", "question": "What is ~\uc5c6\uc5b4\uc11c\ub294, how is it used?", "context": "This sentence seemes to say the opposite of what it \"should\" mean.\n\n\ucc2c\uc591\uacfc \uc608\ubc30\uc758 \uc0b6\uc740 \uc2e0\uc559\uc778\ub4e4\uc5d0\uac8c \uc5c6\uc5b4\uc11c\ub294 \ud558\ub098\ub2d8\uacfc\uc758 \uad00\uacc4\uc774\uc790 \uc0dd\uba85 \uadf8 \uc790\uccb4\uc774\ub2e4\n\nJust to clarify, I'm not confused (hopefully) on these items:\n\n\ucc2c\uc591\uacfc \uc608\ubc30\uc758 \uc0b6\uc740 = a life of praising and worshiping\n\uc2e0\uc559\uc778\ub4e4\uc5d0\uac8c = to those that have a walk of faith\n\ud558\ub098\ub2d8\uacfc\uc758 \uad00\uacc4\uc774\uc790 \uc0dd\uba85 = both life and a relationship with God\n\uc790\uccb4\uc774\ub2e4 = (the very thing) itself\n\nI've heard \uc5c6\uc5b4\uc11c\ub294 \uc548\ub3fc like it's not okay for something to be absent, but I'm thrown by the position of \uc5c6\uc5b4\uc11c\ub294 in this sentence.\n\nIt seems to say that a life of praise and worship, if absent, are a relationship with God and life itself - which obviously must be wrong.\n\nWhen placed into Google translate, it comes out just a little too well. Perhaps the engine either just got lucky, or it didn't process the ~\uc5c6\uc5b4\uc11c\ub294 \n\nClarification and other examples of \uc5c6\uc5b4\uc11c\ub294 would be fruitful.\n", "answer": "I'm pretty sure \uc5c6\uc5b4\uc11c\ub294 is a typo of \uc788\uc5b4\uc11c\ub294.\n~\uc5d0(\uac8c) \uc788\uc5b4\uc11c\ub294 is a common idiom meaning \"when it comes to ...\" or \"to ...\" as in \"\uadf8\ub4e4\uc5d0\uac8c \uc788\uc5b4\uc11c\ub294 \ub9e4\uc6b0 \uc911\uc694\ud55c \ubb38\uc81c\uc600\ub2e4 (To them, it was a very important matter)\".\n", "topic": "korean", "url": "https://korean.stackexchange.com/questions/3178"}, {"image": "VQAonline_00022990.png", "question": "What's the correct representation of letter \"j\" in korean?", "context": "I started learning korean and the book shows c/j as , while in other online resources I found it as \u3148\nIs this the same, only the first is handwritten version and the second is print version?\nI found other letters (for eg. \"h\") that are also written differently.\nPlease explain this.\n", "answer": "Those two letters are the same letter - they just look different in different fonts. Also, in some fonts, like \ubc14\ud0d5\uccb4 you can see both forms - the first (\u3148) when it's in the \ubc1b\uce68 (bottom of the syllable) and the second when it's in the top of the syllable.\nIt is usually transliterated as j - that's how the Revised Romanization handles it, but the older McCune\u2013Reischauer transliterates it as 'ch', and I've seen linguistics papers that transliterate it as just 'c'.\n", "topic": "korean", "url": "https://korean.stackexchange.com/questions/2449"}, {"image": "VQAonline_00023066.png", "question": "Translation needed for 130 years old church document", "context": "\nI found this record of my great grandfather in a local church in Malaysia. Today, nobody use Latin anymore in this country. I should be much grateful if someone can help my family translating this document so that we can trace our root. My great grandfather's name on the certificate is Lam Fuk On.\n", "answer": "It's a record of a baptism, stating:\n\nthe date of the event (17 of April of 1887), which was four days after the baby was born\nthe minister officiating the ceremony (Father F.P. Sorin, a French missionary priest, buried at St. Anne's Church, just a few miles away from the church where the baptism took place) (more info about the priest here) \nthe place (the extinct Church of the Sacred Heart of Jesus, in Machang Bubok)\nthe name of the parents\nthe Christian name given to the baby (Justinus, or Justin, in English)\nthe name of the godparents (fifth line)\n\n", "topic": "latin", "url": "https://latin.stackexchange.com/questions/9180"}, {"image": "VQAonline_00023068.png", "question": "Understanding entries in Latin dictionary", "context": "\nI started learning Latin yesterday by myself using the Wheelock's Latin textbook\nMy question was why are there 4 variations given of a word but only one translated meaning? What do these words mean?\n", "answer": "These are called the principal parts of the verb. They're the same verb, but in different forms.\nBasically, one form isn't enough to know how to use a verb properly. Imagine if you didn't know English, and looked up the verb give in a dictionary. From that one form, how would you know the past tense? You'd probably guess *gived, but that would be wrong. How are you supposed to know it's actually gave?\nSo English verbs have three principal parts: if you look up give in a dictionary for English-learners, it'll list these parts as give, gave, given. From these three forms, you can create all the other forms you might need: he will give, I have given, you are giving, and so on.\nLatin has these same three principal parts: dare means \"give\", ded\u012b means \"gave\", and datum means \"given\". But there's a slight oddity in Latin that means you need one extra form as well: d\u014d specifically means \"I give\", because that one sometimes looks different from what you'd expect. So those are the four forms every dictionary will list.\n(P.S. Sometimes verbs won't have all these forms; you'll see some verbs listed with only three, or some with only two. This generally means certain forms are unattested: they've never been seen \"in the wild\", and are assumed not to exist. For example, memin\u012b, meminisse, \"remember\", only appears in the past tense. A good dictionary will also explain which forms are missing and how to work around that.)\n", "topic": "latin", "url": "https://latin.stackexchange.com/questions/9314"}, {"image": "VQAonline_00023100.png", "question": "ATM in Vatican City: \"Inserito scidulam quaeso ut faciundam cognoscas rationem\"", "context": "The automated teller machines in Vatican City show this screen when awaiting a card:\n\nCould someone, quaeso, break this down word by word? There are a number of things here that I find puzzling or surprising:\n\nWhy future imperative for inserito? I understand the future imperative to mainly be for things to do from now on, not for something to do right now.\n\nDoes quaeso here function like English \"please\"? But doesn't it anthropomorphize the machine, literally meaning \"I ask\"? Or has quaeso taken on a non-anthropomorphizing adverbial meaning in contemporary Latin as a polite softener for when a machine addresses a human in the imperative mood?\n\nDoes ratio faciunda mean bank account to operate on? Calculation to be performed? Something else?\n\nWhy is cognoscas in the 2nd person? I am to insert my card so that I know which bank account to use? (Maybe the answer to this question explains the previous one.)\n\nWhy does the word order emphasize rationem instead of placing cognoscas at the end of the clause?\n\n\n", "answer": "When I first saw this text it too struck me as unclear and unnatural. The first problem has since been resolved, the second however remains. Another reply mentions Foster himself translating it as 'Insert your card--scidulam--so you can access the operations allowed.' - that's certainly not an exact rendering of what the Latin says, which is instead:\n\n\"I kindly ask that you then insert your card in order to learn what approach needs to be done.\"\n\nFrom this follow my observations that will simultaneously try to answer the questions:\n\ninserit\u014d: the future imperative is the standard form of instruction in manuals; however, I don't think it's appropriate in an immediate address:\n\n\"The present imperative differs from the future imperative in -to, the semantic value of which is, certainly in Early Latin comedy and in legal texts, \u2018non-immediate realization of the action involved\u2019. When the present and future imperatives\nco-occur, the difference in immediacy becomes apparent...\" (Pinkster 2015: 515)\n\n\nscidula: \u03c3\u03c7\u1f77\u03b4\u03b1~\u03b7 seems to be a variant of \u03c3\u03c7\u1f73\u03b4\u03b7 already in Greek; sc(h)eda looks to be reflected by Italian sc/\u025b/da (no diphthong would mean a native Tuscan survival).\nquaes\u014d: I think it does anthrophomorphize the machine, and this is normal - we digital moderns anthropomorphize them all the time, and in Latin this is taken to eleven and one of the most common sources of neologisms. In this case, however, quaes\u014d might be a little too friendly-conversational (Cicero in his letters reserves it for the most personal requests) and rog\u014d might have been more appropriate - or in fact the even more impassionate rog\u0101mus.\n\nI disagree that if an English translation does not, this means that the original doesn't either - in many cultures animal names are employed metaphorically to refer to certain objectionable human qualities, like \"an ass=donkey\" for dim-witted and obstinate people, and the fact that in English \"ass\" refers to the buttocks while \"donkey\" isn't used as an insult bears no relation to the fact that calling someone \u043e\u0441\u0451\u043b in Russian is clearly zoomorphising, as is incidentally \u0441\u0443\u0301\u043a\u0430 \"bitch\" - less so in English.\n\n\nrati\u014d faciunda uses rati\u014d in the sense \"logical method, approach, way of proceding\", but the use of facere seems unnatural to me, which I illustrate in the translation \"to do an approach\". I've found no instances of this collocation. This is the main reason the sentence is difficult to understand - since this meaning of the noun that exhibits a great variety of idiomatic uses doesn't work with a verb whose meaning is itself extremely bleached, and there's no clue as to what other meaning could have been intended.\n\nI don't think it can be \"transaction\", because the ATM has no business telling the user which monetary transaction they need to perform - it's the other way around.\nnotice -und- instead of the more common -end- \u2013 their distribution was determined partly stylistically and partly word-by-word, and for facere in official contexts this form was indeed preferred, although this is not reflected by modern editions.\n\n\nc\u014dgn\u014dsc\u0101s is simply \"to get to know some information\", which is it's primary meaning. It promises to tell you how to proceed next.\nThe word order is the default one after a fronted faciundam, at least in written use. It exhibits what's commonly known in relation to Latin as 'hyperbaton' - a discontinuous noun phrase. In proper syntacticianese this is called 'movement', and in the case of leftwards movement 'fronting'. Here the underlying word order before fronting is c\u014dgn\u014dsc\u0101s rati\u014dnem faciundam, while its presence adds a certain amount of poetic flavour.\n\nI suppose the most unmarked word order is rati\u014dnem faciundam c\u014dgn\u014dsc\u0101s, with phrasal stress on the second word - the fronting of one member of the noun phrase is marked already, while faciundam rati\u014dnem c\u014dgn\u014dsc\u0101s is doubly so and places stress on rati\u014dnem - or can be an instance of right-movement of c\u014dgn\u014dsc\u0101s, thus delaying, suspending it (accompanied by a suspended intonation and possibly a pause).\n\n\n\n\nMy own rendering of 'Please insert your card to receive further instructions' would be:\n\nInsere schedulam, rog\u0101mus, ut agend\u012b rati\u014dnem accipi\u0101s.\n\nNotice the genitive nd-noun (\"gerund\") instead of the nominative nd-adjective (\"gerundive\") - this might be what Foster was going for, but he confused the two verbs and consequently the two constructions. Paradoxically enough I feel that this is the least marked word order in this case, since agend\u012b rati\u014d is a single, quasi-compound noun phrase, quite regularly so when it comes to generic-reference objective genitive expressions - indeed, (Spevak 2010: 99 & 182) concurs that \"genitives with generic referents [...] are often pre-nominal\" and \"objective genitives coming last are semantically prominent\".\nAs for translating 'so you can access the operations allowed', I think I'd be stretching the idiomatic resources of Latin and/or my knowledge of its financial jargon, but I think this would be sufficiently clear:\n\n...ut certior f\u012b\u0101s quae rati\u014dn\u0113s aut neg\u014dtia c\u016br\u0101r\u012b possint.\n\nRefs:\n\nPinkster H. (2015). The Oxford Latin Syntax\nSpevak O. (2010). The Noun Phrase in Classical Latin Prose\n\n", "topic": "latin", "url": "https://latin.stackexchange.com/questions/15786"}, {"image": "VQAonline_00023115.png", "question": "Struggling to translate baptism record", "context": "Researching the baptism record of a relative dating from the early 1800\u2019s where Catholic Priests used Latin in documenting the event. I apologize that I am a novice and hope its OK to post, but I have taken lots of time but can\u2019t seem to figure out two words and their meaning (see attached).\nI believe their naming or describing family relatives or close neighbors or friends??? Please let me know oif the attached is clear or if need additional info. Thank you for your consideration.\nPS Tried to attach full page but it exceeds size limit and if I shrink then resolution will eb an issue. Let me know if you want to see the full page and I can send / or post seperately.\n\nThank you all for the input, you helped me break a mental block. After your suggestions, I researched further and found another book which had overlapping baptismal events / dates, strange but true. The events were same basic date etc but the name was \"Snerigen\". I also found the Adam Schmith entry. Thank you again very much.\n", "answer": "I would read those words as Samuelis Snerigen. I think the first name is clear enough that it requires no explanation, the genitive \"of Samuel\".\nThe second letter of Snerigen does not look like an r nor a u, because this scribe seems to write those differently in a consistent way (see Martii in the top left, and fuit in the top centre. There is nothing else it could be but an n.\nThe third letter could perhaps be a c, but it looks fatter than his other c's. It looks similar to his neater e's. Since Snc- is not possible, it must be an e.\nThe fourth letter clearly resembles his other r's.\nThe fifth letter must be an i, because it is a single stroke. In addition, while the scribe does not dot all i's, there is a dot here, which arguably must belong to this stroke.\nThe seventh letter looks like a g, cf. the g in Legitima in the second line.\nThe eighth letter looks the same as the e earlier in this word.\nThe final letter could be an r in other manuscripts, but this author never seems to use that r. Besides, I think that r is mainly used in older manuscripts than this, or in Germany, perhaps. And the scribe consistently uses a different r. It does resemble the n in *Susceperunt in the line before last, and it has the typical little ending horizontal stroke for a final n or m, so it must be an n.\n\nWhen you Google, the surname Snerigen seems to exist in the 19th century in America..\nP.S. As Dbmag9 says, the word you suggested could be Adam is Mariae, \"of Maria\" (genitive case).\n", "topic": "latin", "url": "https://latin.stackexchange.com/questions/17557"}, {"image": "VQAonline_00023061.png", "question": "What's the translation of this Medieval document?", "context": "This is a page taken from a medieval breviary from 13th century Italy\nFound this document at The Antiquarium in Houston. Would like to know what it is describing. Translations as well as paraphrases will be equally appreciated.\n", "answer": "Here is a transcription starting from the first initial with only a few lacunae. I've mostly filled in the abbreviations and added punctuation, except when I didn't.\n\nCumque Anolinus huiuscemodi litteras accepisset, palam recitavit, sibique Nazarium offerri iussit. Dixitque ad eum, \"Iussit inclitus imperator incidi cervices tuas pariter et puerorum Gervasii et Prothasii et Celsi pueri.\" Nazarius respondit, \"Magnus Deus et magna opera eius qui nos absolvi iam iubet.\"\nR/ Isti sunt incliti martyres Christi Gervasius et Prothasius, qui monita Dei ardentes, spernendo mundum secuti sunt domini nostri Iesu Christi pia vestigia.\nV/ Nichil terrenum, nichil carnale concupiscentes in Mediolanensi urbe in Dei servitio perdurantes manserunt. Secuti [etc.]\n?: Iusti autem in perpetuum vivent et apud Dominum est merces eorum.\n\n[up to the initial, this next line is unclear and contains many technical abbreviations. I can only make out \"Vos servi? Domini.\" The red line starts with \"secundum.\" It continues:]\n\nIn illo tempore, egrediente Iesu de templo, ait illi unus de discipulis suis, \"Magister, aspice quales lapides et quales structurae [column break] templi...\" et rel[iqua]. (h/t Ben)\nIuxta historiam manifestus est sensus quia quadragesimo secundo anno\n post passionem Domini sub Vaspasiano et Tito Rommanorum principibus, ita funditus est ipsa civitas eversa cum templo suo magnificentissimo ut solo coaequaretur.\nR/ Quae Beato Ambrosio revelata atque ab eodem reperta in ecclesia quam ipse proprio fundavit studio ostensis miraculis sunt sepulta. Quae domini, etc.\nQuod etiam modo ambitus murorum qui ipsum locum circumdat, testatur eum locum Calvariae ubi Dominus extra passus est, et infra muros amplectitur. \nR/ Beatissimo Ambrosio Mediolanensi episcopo per visum apparuit quo loco laterent corpora Gervasiii et Prothasii. At ubi agnitio sancti sacerdotis divina ostensione condonavit, invenit sanctos martyres in profundo terrae sicut the[saurum], etc.\n\nSome notes:\n\nI'm not an expert in thirteenth century Medieval liturgy, but based on what I know of more recent centuries, this looks to be like a part of the office for the feast day of Sts. Protasius and Gervasius. It includes the usual variation between verses, responses, and readings, building on a general theme. \nSome of the narrative sections are taken, either word-for-word or in paraphrase, from the Analecta Bollandiana.\nOne strange part is that it launches into a Gospel before abruptly transitioning (starting with \"iuxta historiam\") to Jerome's Rabanus Maurus's (thanks, luchonacho!) commentary on that Gospel. I'm not sure why this is done.\n\n", "topic": "latin", "url": "https://latin.stackexchange.com/questions/6931"}, {"image": "VQAonline_00023123.png", "question": "What is the meaning of the idiom \"Discedere ab amicis\"", "context": "I found this in a Latin-English dictionary. Is the specific meaning known? The English translation does not give me any specifics in terms of the meaning. I was thinking it could mean self-isolation or even suicide.\n\n", "answer": "It means pretty much literally that: to abandon one's friends, to break off a friendship, either permanently or temporarily. Cicero uses it a few times in De Amicitia (12.42, 20.75):\n\n\u2026si in eius modi amicitias ignari casu aliquo inciderint, ne existiment ita se alligatos, ut ab amicis in magna aliqua re publica peccantibus non discedant\u2026\nIf unknowing people fall by some happenstance into friendships of this sort, they must not consider themselves to be so constrained that they cannot separate themselves from friends who are causing problems in important government affairs\u2026\n\n\nEt saepe incidunt magnae res, ut discedendum sit ab amicis; quas qui impedire volt, quod desiderium non facile ferat\u2026\nAnd often important matters come up where one must be separated from their friends; someone who wants to obstruct these matters because they cannot easily bear this grief\u2026[is weak-willed and also a bad friend]\n\nI don't think it was a special idiom, though, because Cicero uses plenty of synonyms too. When talking about ending friendships in the next paragraph (21), he uses demittere amicitiam instead.\n", "topic": "latin", "url": "https://latin.stackexchange.com/questions/18777"}, {"image": "VQAonline_00023049.png", "question": "What does 'ANGLORUM REGIS QUI COR LEONIS DICTUS' mean?", "context": "In the cathedral of Rouen I visited the grave of Richard I the Lionheart. It has an inscription:\n\n\nANGLORVM REGIS QVI COR LEONIS DICTVS\n\nSo what is the best translation?\n\nKing of England, who ruled with a lion heart?\nEnglish King, who ruled with a lion heart?\nMaybe something else?\n\n", "answer": "\"Here is interred the heart of Richard, King of the English, called Lionheart. Died in the year 1199.\"\n", "topic": "latin", "url": "https://latin.stackexchange.com/questions/5158"}, {"image": "VQAonline_00023161.png", "question": "Am I responsible for this car accident?", "context": "It is an accident that nearly occurred. It is in UK; so cars go on the left.\nI was standing at a junction, and there was busy traffic on my right. Occasionally, there are cars turning left, and I was looking for a gap to walk across. Then there is Taxi coming up on my right; it did not signal left, so I thought it was going straight. So I walk across, and the car turned left nearly hitting me. The driver was swearing at me after this near accident.\nSo my question is: if the driver did hit me, who is responsible for this accident?\nAppreciate any suggestion.\nEdit: I was not on a pedestrian passing, but in my town, people always go across like that. I think the taxi was driving at about 15 miles. I was also walking quickly and may not have allowed the driver to response. When I started to cross, the taxi is already ahead of me.\nSupplement:\nThe junction\n\n", "answer": "From Rule 170 of the Highway Code:\n\nwatch out for pedestrians crossing a road into which you are turning. If they have started to cross they have priority, so give way\n\nIf this was indeed the case, then it suggests that the OP had right of way, and the driver was at fault. In any case, the driver should have indicated before turning.\n", "topic": "law", "url": "https://law.stackexchange.com/questions/18197"}, {"image": "VQAonline_00023130.png", "question": "How much street space can restaurants occupy?", "context": "In Paris, France, what define how much street space can a restaurant occupy?\n\n", "answer": "According to the document Reglement des etalages et terrasses, page 15, such installations may not exceed 50% of the usable area of the sidewalk, and must leave a contiguous area of at least 1.6 meters in width for pedestrian traffic:\n\nLes installations peuvent \u00eatre autoris\u00e9es, soit d\u2019un seul tenant, soit scind\u00e9es, sans pouvoir exc\u00e9der 50% de la largeur utile du trottoir. Une zone contigu\u00eb d\u2019au moins 1,60 m\u00e8tre de largeur doit \u00eatre r\u00e9serv\u00e9e \u00e0 la circulation des pi\u00e9tons.\n\nIf you are interested in looking directly at the laws concerned, these can be found at the beginning of the same document:\n\nLes dispositions du pr\u00e9sent r\u00e8glement sont \u00e9tablies en application\n des articles L.2122-1 \u00e0 L.2122-3 du code g\u00e9n\u00e9ral de la Propri\u00e9t\u00e9 des personnes publiques, L.2512-13, L.2512-14 et L.2213-6 du code g\u00e9n\u00e9ral des Collectivit\u00e9s territoriales et de l\u2019article L.113-2 du code de la Voirie routi\u00e8re.\n\nRoughly translated, that means that the rules described in the present document are established by application of the articles L.2122-1 through L.2122-3 of the \"general code of property of public persons\", L.2512-13, L.2512-14 and L.2213-6 of the \"general code of territorial collectivities\" and of article L.113-2 of the highway code.\nMore links:\nL.2122-1\nL.2122-2\nL.2122-3\nL.2512-13\nL.2512-14\nL.2213-6\nL.113-2\n", "topic": "law", "url": "https://law.stackexchange.com/questions/867"}, {"image": "VQAonline_00023242.png", "question": "How are US grand juries instructed not to 'indict ham sandwiches' if there is no judge or opposing counsel?", "context": "In the May 26, 2021 video How a NY special grand jury could impact Donald Trump CNN\u2019s senior legal analyst and former federal prosecutor Elie Honig mentions the following in a background piece outlining the basics of a US Special Grand Jury. After about 01:17 they say:\n\nIt is a one-sided presentation; it\u2019s only the prosecutor, the grand jurors, and a court reporter. There\u2019s no judge, there\u2019s no defense lawyer. So it is extremely one-sided; you hear this expression \u2018the grand jury would indict a ham sandwich\u2019 there\u2019s some truth to that; it\u2019s not an adversarial process like you\u2019d have at trial.\n\nand the origin of the 'ham sandwich' reference is mentioned in Wikipedia's Grand Juries in the United States; Rubber Stamp for the Prosecution:\n\n[...]Grand jurors also often lack the ability and knowledge to judge sophisticated cases and complicated federal laws. This puts them at the mercy of very well trained and experienced federal prosecutors. Grand jurors often hear only the prosecutor's side of the case and are usually persuaded by them. Grand juries almost always indict people on the prosecutor's recommendation. An unnamed Rochester defense lawyer was quoted in a 1979 newspaper article claiming that a prosecutor could get a grand jury to \"indict a ham sandwich\", a saying subsequently repeated by the chief judge of New York State's highest court, Sol Wachtler. And William J. Campbell, a former federal district judge in Chicago, noted: \"[T]oday, the grand jury is the total captive of the prosecutor who, if he is candid, will concede that he can indict anybody, at any time, for almost anything, before any grand jury.\" (references therein)\n\nQuestion: With only prosecutor(s) and a court reporter, are jurors at all instructed to not simply \"rubber stamp\" the prosecution's desire to indict? Are the terms and concepts necessary for them to function as a jury (e.g. \"preponderance of evidence\") explained to them? If so, by who? Is there a standard \"script\" or talking points for such instruction?\n--\n\n", "answer": "\nWith only prosecutor(s) and a court reporter, are jurors at all\ninstructed to not simply \"rubber stamp\" the prosecution's desire to\nindict?\n\nNot really. They are told their job and the legal standards that they are required to act according to, however, and the federal judicial branch does have a short handbook for grand jurors.\n\nAre the terms and concepts necessary for them to function as a jury\n(e.g. \"preponderance of evidence\") explained to them?\n\nYes. But grand juries don't operate on a preponderance of the evidence standard. The sole evidentiary standard in a grand jury proceeding is the lower \"probable cause\" standard. Likewise, lots of evidence which would not be admissible in a criminal trial is admissible in a grand jury proceeding. The main kind of evidence not admissible in a grand jury proceeding is evidence which is \"privileged\" against court disclosure (e.g. attorney-client communications).\n\nIf so, by who?\n\nThe prosecutor and the handbook linked above.\n\nIs there a standard \"script\" or talking points for such instruction?\n\nProsecuting attorney's offices in jurisdictions that usually use grand juries typically have a set of forms.\nIn federal practice, Federal Rule of Criminal Procedure 6 is the primary source of guidance, and the Justice Department (which handles all or virtually all federal criminal prosecutions) has detailed policies and procedures for handling grand jury practice (and a 405 page practice manual) that his a matter of public record. There is some case law on what the content of an indictment must look like and on the grand jury process.\nSome state and local jurisdictions with routine use of grand juries probably have standard court forms for certain parts of grand jury process, and there may be some standard forms in local rules for some federal courts. But there isn't a national judicial branch sanctioned official set of court forms for grand juries in the federal criminal justice system.\nThe rather lax supervision of grand juries reflects the fact that they are a road bump in the criminal justice process with a pretty low threshold to meet that faces sterner tests later in the process:\n\nIs there probable cause to support the charges requested by the\nprosecutor against the named defendants?\n\nAbout half of U.S. states allow prosecutors to do this without a grand jury approval, unilaterally.\nEvery grand jury indictment must either be consented to by the defense, or approved by the court following a trial with a much higher proof beyond a reasonable doubt standard, to result in a conviction resulting in the imposition of punishment.\nA judge in a separate hearing determines if there will be pre-trial detention or bail conditions prior to the criminal trial.\n", "topic": "law", "url": "https://law.stackexchange.com/questions/65310"}, {"image": "VQAonline_00023151.png", "question": "Creative commons license vs copyright, who wins?", "context": "There's an image of a pokeball (from Pokemon, which is owned by Nintendo) that I found online.\n\nIt has a creative commons license 3.0.\nCould I sell this image without being sued by Nintendo? The creative commons license says I can sell it for commercial use, but I found articles online of Nintendo suing people for selling their stuff. So who owns this image? Am I safe from the law if I sell it?\n", "answer": "If the \"Pokeball\" image is copyrighted and/or a trademark of Nintendo/whoever makes the Pokemon games, then whoever put that image out there under CC 3.0 BY is in violation and can be sued and will probably lose, and you would be in violation and can be sued and will probably lose. Your penalty would almost certainly be less since your violation was \"innocent\", that is, you had no way of knowing that the \"Pokeball\" imagery was somebody's protected intellectual property.\n...\nExcept you kind of maybe should know that, unless whoever made the Pokemon games (Nintendo or other) put the image out there and you can verify that, that maybe this license could be bogus and you should consult with who you imagine the owner of that IP may be or an impartial professional who could tell you for sure. I'm thinking if it were me I would do a little more research - and maybe get a paid opinion - if I was really thinking about using this for any but private purposes.\n", "topic": "law", "url": "https://law.stackexchange.com/questions/13499"}, {"image": "VQAonline_00023216.png", "question": "Incident with another cyclist, repair costs too expensive", "context": "Past week I had a bike incident with another cyclist. The rider in front of me slowed down and when I looked back to confirm no one was behind, he stopped and I hit him. Totally my fault, and no one get hurt. Typical mistake. \nI gave my phone number to the other person and ask him if he wants to go to a bike shop at the moment but he refuse because it was late for him. Anyway, he contact me the day after asking for 370 euros for the repairs. I consider this amount of money ridiculous since the damage is only in the gear drive that is twisted (not broken). \nAt the moment I had some shots of the damage and a video of the person riding after with no problem apparently. \nMy question is if I should ask the person to take the bike to a shop repair that I trust and get a new budget for this? Or even buy the parts and install it by myself.\n\nThis is in Spain and I am not covered by insurance for this incident.\nThe person is very reluctant to detail the repairing. I told that I accept the costs and the repair shop but I need a detailed invoice. Somehow the pictures I took are helpful to clarify how damaged it was.\nThis was between two Bicycles. As additional info, we were not riding together.\n\n", "answer": "Great job taking the photo at the time, it could end up saving you some money.\nI can see an Acera rear derailleur(inexpensive), a seven speed freewheel (cheap), a rusty chain (poorly maintained), a bent derailleur hanger (possible damage) and a rear derailleur cable which is doing something slightly funny.\nFocusing on the seven speed freewheel, we can tell that this is a budget bike. I would expect you could replace the whole bike for 370 euros. Labour charges will make up a large portion of a repair bill, but it does seem like an inflated price.\nYou absolutely must insist on transparency upfront from the bike owner. If they are not prepared to tell you what they want to spend your proposed 370 euros on, then I am confident they are trying to rip you off. If they tell you what the money will be spent on and you are struggling to decide if it is reasonable, please ask us about it with the updated info, before you pay any money at all. \nAlso seek some regional legal input into what your obligations and time frame need to be (we aren't lawyers!). If you meet your legal obligations and do so in a timely manner, then you are free to be as generous or not as you like in returning the bike to the condition it was in before the incident.\n\nAs for the original questions (sorry). I would suggest having the quote for the repairs approved by someone you trust in advance, yes perhaps a third party bike shop you trust. I would suggest not doing the repair work yourself. Even though that could be cheaper, you don't want any follow-ups from the other person. So if the repair is done at the other person's choice of repair shop, with your prior agreement about the cost and extent of the work, then they cannot follow up and ask you for more money after the fact.\n", "topic": "law", "url": "https://law.stackexchange.com/questions/50951"}, {"image": "VQAonline_00023148.png", "question": "Why is PayPal blocking my account according to U.S. tax law, if I've a UK account only?", "context": "I've opened PayPal account for my business and in My Profile I've clearly selected the UK address. My phone is UK, my bank accounts are UK and the balance is shown in GBP. So this is very clear to PayPal that my account is UK based.\nHowever I've received the following e-mail which says:\n\nYour account access will remain limited until you add more information about your business.\nWhy do I need to add this information?\nThe Foreign Account Tax Compliance Act (FATCA), a U.S. tax law, requires financial institutions like PayPal to collect additional information from customers when a new business account is opened. You need to add this information even if your business or your customers are not United States taxpayers or located in the United States. We collect this information for regulatory and compliance purposes.\n\nBy adding \"more information\" they mean to download U.S. form W-8BEN-E, fill it in completely and upload or fax it to PayPal, however I don't want to fill some U.S. form, simply because I'm not from the U.S and I don't want to share my confidential personal details with the foreign country. To clarify, my business doesn't have any clients from the US and my business is registered in the UK. And as an owner of the business I've never been to the U.S.\nSo my question is, why am I affected by a U.S. tax law? Do I really need to comply with U.S. tax law (having the business in the UK)?\n\nThe above quote from the e-mail says that I need to add this information even though my business isn't located in the U.S. However on the FATCA FAQ page (requires log-in) it says:\n\nFor accounts held outside the United States that belong to non-US persons, financial institutions such as PayPal must gather information from account holders providing that they are not US persons.\n\nFor me this is contradictory to the previous quote (because here they ask me to prove I'm not a U.S. person) so I'm confused as to what I should do.\nIn other words, can my UK business somehow ignore FATCA requirements or not? Currently I'm forced to comply to the country compliance act tax (by filling W-8BEN-E) where I've nothing to do with it, which doesn't make any sense.\n\n\n", "answer": "The reason that you are being asked to comply with a US law is because PayPal, a US company, is required to comply with US laws. If you do not comply, it is likely that they will be non-compliant and subject to sanctions.\nFor more background, FATCA reporting is used to identify businesses that a company does business with. In this scenario PayPal does business with you, 'Kenorb Inc', and so must prove to the regulatory agencies that you are not a US based company, hence the need for the W-8BENE.\nIn a similar way - people from other countries cannot simply ignore UK law when dealing with your business.\nAs for your quotes - they are not contradictory. \n", "topic": "law", "url": "https://law.stackexchange.com/questions/11070"}, {"image": "VQAonline_00023261.png", "question": "How often do Davids fail to retain effective legal representation against Goliaths, in Private Law, even if David can pay?", "context": "I ask just about private (civil) law here, NOT criminal. David is the underdog layman, who needs a competent civil litigator against a powerhouse Goliath. Postulate that David can pay lawyers' fees and legal costs \u2014 rule this out as an issue. Postulate just good faith, skillful lawyers.\nHow likely will David fail to find \u2014 and be represented by \u2014 a skilled civil litigator \u2014 solely because Goliath is too powerful?\nFor example, Big Six (in Australia), Magic Circle (in U.K.), Seven Sisters (in Canada), White Shoe (in U.S.A.) law firms normally act for Goliath, like moneyed MultiNational Corporations. So they're conflicted from representing David.\nA bad faith, wealthy Goliath can deliberately instruct or retain (Which is the correct term?) most \u2014 if not all \u2014 law firms in a jurisdiction. Then Goliath can intentionally conflict all these lawyers from representing David, and deny David cost-effective realistic legal representation. Do any laws outlaw this kind of Lawyer/Regulatory Capture?\nThis is a genuine worriment, especially for teeny jurisdictions like Hong Kong or Singapore. Is this worriment less likely in big countries high on the Rule of Law Index like Canada, UK?\nIn theory, David can try small town or upstate lawyers in the boondocks?!?!??! The problem is that in some areas of the law, there are only so many lawyers who have deep experience and are really good at what they do. If a client has a complex matter which requires a particular expertise, it is not unusual for the client to discover that the number of knowledgeable lawyers who are available to take on the matter may be quite limited. [...] Moreover, I will readily acknowledge that there are many situations where most, if not all, of the best lawyers who practice in the relevant area of law will be in a large firm.\nMy child in Hong Kong was such a David.\nUnlike England that has Direct Access barristers, laymen cannot instruct barristers in Hong Kong. My kid couldn't find a solicitor on all of Hong Kong Island to represent him against Goliath! He asked at least 100 firms on Hong Kong Island \u2014 they all rejected him! He managed to find some solicitors in New Territories, but some appeared shoddy. Some didn't have websites. Some did not speak professional English. Their offices were run down.\nDonald Best asseverated that\n\nOver one hundred Ontario lawyers refused to represent me even as they acknowledged the veracity and power of my evidence. Many told me that while they personally sympathized with my situation facing injustice and corruption, they feared backlash and opprobrium from the profession if they harmed or even challenged the involved senior lawyers and their large Bay Street law firms.\n\n\n", "answer": "\nHow likely will David fail to find a skilled civil litigator to\nrepresent David \u2014 solely because Goliath is too powerful?\n\nIt almost never happens in reasonably urbanized areas. It is an issue that usually comes up, if at all, in jurisdictions that are highly rural, for example, Wyoming, or the Northwest Territories of Canada, where the \"Goliath\" is usually some representative of the national government or a firm representing a national business, and there are extremely few lawyers, few of which have relevant practices.\nEven then, it is almost always possible to get one's own out of town counsel, it is just more expensive than it would be if local counsel with appropriate skills could be found, and it takes more time since searching for counsel far from the place where the litigation will have to proceed is less convenient.\nIt isn't at all unusual for a tiny firm or sole practitioner who didn't attend a high prestige law school to take on a party represented by a large international law firm and win.\nFor example, I once represented a client in an attorney malpractice case against the largest law firm in the entire United States and secured a quite favorable settlement for them, working as co-counsel in a three lawyer firm, that has also won victories against other large firms.\nAs long as the client can afford to pay and the client has a case that isn't frivolous and isn't an absolute pain to deal with interpersonally (e.g. the client listens to the lawyer's advice), it is almost always possible to find counsel to represent an underdog against a large and powerful party.\nThis said, I can't necessarily speak to Hong Kong, which is in a very quirky and unique situation in a quite small market, in the post-Chinese takeover era, which is not the completely the usual open Western capitalist type legal system that it used to have. The Chinese Communist Party's pervasive influence over Hong Kong has changed the nature of legal practice there from the pre-takeover model to the kind of considerations that apply in a non-democratic, non-Western style legal regime even though the transition is not complete. The nature of contemporary legal practice in Hong Kong is totally unlike that nature of legal practice in Canada, the U.S., the U.K., France, Germany, Japan, Australia, New Zealand, or Italy, for example. It is more like that of a lawyer friend of mine who practiced law in Ivory Coast during a military coup regime.\n\nThe problem is that in some areas of the law, there are only so many\nlawyers who have deep experience and are really good at what they do.\nIf a client has a complex matter which requires a particular\nexpertise,\n\nWhile deep experience is desirable, it is rarely necessary in litigation work (for some kinds of specialized transactional work like oil and gas title work, or municipal bond underwriting, this is less true, but conflicts of interest aren't an issue in the same way). After all, all judges are basically generalists, so there are diminishing returns to having all that much more expertise than the judge. Any litigation lawyer has to boil down the law to arguments that the judge can handle.\nThere are economies of scale in litigation, but they max out somewhere in the vicinity of three or four lawyers and several paralegals. And, a lawyer supported by a couple of junior lawyers or a junior lawyer and a paralegal is usually more than adequate for a \"David and Goliath\" type case, although for a case between two \"Goliaths\" which is large and has voluminous facts or a class action case, one really needs to have more staff than that to be effective against a \"Goliath\". Scale mostly matters in cases which are evidence and discovery intensive (e.g. hundreds of thousand or millions of documents and dozens of witnesses are necessary to get the case resolved properly).\n\nA bad faith, wealthy Goliath can deliberately instruct or retain\n(Which is the correct term?) most \u2014 if not all \u2014 law firms in a\njurisdiction. Then Goliath can intentionally conflict all these\nlawyers from representing David, and deny David cost-effective\nrealistic legal representation.\n\nPossible, in theory, and conflicts of interest do happen, but I've never seen it happen for all firms in the market that can do the job in a quarter of a century outside very rural areas (like Wyoming). Big firms and small firms have different kinds of clients, so conflicts just aren't that common in this kind of scenario.\nOn the other hand, you have the weasel word \"cost-effective\" in there. Lawyers handling challenging litigation against a formidable opponent aren't cheap. In areas that I am familiar with, you are talking several lawyers at $250-$500 an hour each, for lots of hours, and big out of pocket charges for expert witness fees. It does cost a lot of money to fight a fight like that. But, it isn't because there aren't lawyers available to do it.\n\nDonald Best asseverates that Over one hundred Ontario lawyers refused\nto represent me even as they acknowledged the veracity and power of my\nevidence. Many told me that while they personally sympathized with my\nsituation facing injustice and corruption, they feared backlash and\nopprobrium from the profession if they harmed or even challenged the\ninvolved senior lawyers and their large Bay Street law firms.\n\nWith due respect, Mr. Best is wrong. If a hundred lawyers refuse to represent you, it is because (1) your case is difficult or impossible to prove given the available evidence and legal standards in play (even if you are legitimately aggrieved), (2) they don't think you can afford to do what needs to be done, (3) the case is winnable but the costs of litigating it don't justify the rewards available if you win (probably the mostly likely reason in the matter described), or (4) you're an asshole and they don't want to put up with you (the second most likely reason in the matter described). Often, even when the law provides for a remedy in a case, the cost-benefit ratio just isn't there and good lawyers refuse to take on cases where the client, for example, wants to spend $50,000 to litigate a $15,000 problem.\nOf course, out of 100 lawyers, if one isn't very thoughtful in how one tries to locate them, perhaps 40-60 simply don't have a relevant practice or are too busy to take on new work. I routinely decline new cases (dozens of times a year, at least) from prospective clients seeking representation in areas outside what I do (e.g. criminal law), or because there is only one of me and I'm currently working flat out on other cases. Small law firms have much less capacity to stretch to take on new work when they are busy than big ones do.\n", "topic": "law", "url": "https://law.stackexchange.com/questions/82155"}, {"image": "VQAonline_00023345.png", "question": "Stuck photos while inside an envelope", "context": "It seems the photos have been stuck to each other. I had kept them inside the envelope which was given when I purchased those photos. It's been about 5 months since I had last opened that envelope. And now today I need them and they are stuck to each other. \nWhat can I do to separate them?\n\n", "answer": "If they are gelatin-emulsion based, as used in traditional photography (or some coated ink-jet papers), hold the back side of the pack over a steaming kettle and let the steam slowly loosen the rear-most photo, one at a time. Just set them aside to dry separately afterwards.\nN.B. Ink jet photos might 'run' in water. Test this on one photo to see if it damages it. \n", "topic": "lifehacks", "url": "https://lifehacks.stackexchange.com/questions/14152"}, {"image": "VQAonline_00023307.png", "question": "How to cool a car down quickly", "context": "My car is black and I live in Australia. When I go to buy groceries it gets unbearably hot inside the car (especially in summer). \n\nIs there a quick way to cool down my car so that I don't have to blast the air-con or wait for 10-15 minutes?\n", "answer": "There are two hot things to deal with: SURFACES and AIR\nAIR has pretty much one best solution, exchange the air. Opening one door can take a long time if you're just waiting passively for the air to covect out. You can actually see the hot air if you look on a hot day for the air shimmer or (even cooler) watch the shadow of the door you've opened. If you have passengers opening the passenger side the smart thing to do is coordinate opening your door before they get in and leave it open as they enter, this gives air time to enter from the windward size and exit from the leeward. You can turnover the air in a car in a few seconds with even slight wind this way and if you're sly about it the passenger may never know. I do this a lot. If it's just you getting into the car it's a little tricky to open opposing doors. One possibility is to have a door and sunroof open to get crossflow but if it's a power sunroof opening it can take too long and put strain on your battery. If you are alone and have no convenient sunroof then you may have to resort to the old \"swing around a magazine\" approach or some other inelegant way to move the air although since you're alone in this case nobody should see :)\nSURFACES are harder to cool because they have both more thermal mass and lower thermal dissipation (air is an insulator but convection actually exchanges the air which increases cooling speed by factors of 100). My usual solution for surfaces is to pick a couple important ones and change the surface into something that doesn't gather or convey heat to the user as well. This is a good reason for wood steering wheels or certain types of steering wheel and seat covers. Choices all depend on your preferred looks and textures as well as which surfaces get hot enough to be worth fixing.\nThere are a lot of fine variations here (like maybe a small battery fan to move the air). I look forward to useful additions in the comments.\n", "topic": "lifehacks", "url": "https://lifehacks.stackexchange.com/questions/6849"}, {"image": "VQAonline_00023295.png", "question": "How to stop a trashbin from stinking", "context": "Okay, so we have bin collection once a week. For clarity sake, this is what our bin looks like:\n\nSo basically, occasionally the bag wears out and we loosen it so that it gets thrown out when the garbage collection comes, leaving the bin empty. The bin continues to smell even when there isn't the bag (or rubbish) inside of it. It's even clean inside the bin, which ends up in me having to wash the bin out.\nAre there any options to keep said bin \"smelling nice\" / \"not stinking\"? I'm thinking of a more permanent solution (removing the smell), not masking the smell. Would Baking Soda be the option of choice?\n", "answer": "Don't put food in the trash. Instead, flush down the toilet or use the disposal. Rinse out cans and containers before throwing in the trash.\nWash the bin out periodically. If you leave it sit, it will eventually air out, but it may help to wash out with a hose.\nI would remove the big trash bag from the external bin. It serves only to prevent the bin from airing out. Do not throw anything into the bin that is not already in a trash bag or grocery bag. A bag in the bin itself is unnecessary.\nWhen throwing small bags into the bin, insure they are tied.\nMake sure to have the bin at the curb on time for weekly pick up.\nI do not recommend baking soda or similar remedies. Once you stop whatever bad habits regarding the trash bin, putting an agent in the bin will not be necessary. Do things differently starting now and the smell will be gone in a few days.\n", "topic": "lifehacks", "url": "https://lifehacks.stackexchange.com/questions/5602"}, {"image": "VQAonline_00023272.png", "question": "Is there a way to erase pencil without a standard eraser?", "context": "There are times when I am away from home with my sketchpad and I realize I do not have my eraser with me. Are there any quick tricks to erase pencil marks on a paper without ruining the paper, or smearing the other pencil marks? \nI use a particular type of eraser known as a \"kneaded eraser\", not a generic pink pencil top eraser, or a handheld pencil eraser.\n\nI have tried using a gently moistened corner of my shirt to try and \"clean\" the pencil mark off of the paper, but that just smudged the pencil, and ruined the paper. I have tried dried chewing gum I was done with, but that too just seemed to smudge the pencil mark. On one occasion I tried silly putty and though it worked o.k., there were still faint lines and it wasn't erased completely.\nIs there something handy I could find that might help?\n", "answer": "You can use a rubber band. Rubber bands remove pencil matter as well as erasers do. You have to be careful what you erase and what rubber bands you use, though. \n\nYou may be able to use correction fluid (a.k.a. white out).\n\nBut as long as you have a rubber band you should be okay. Also, I have observed that many rubbery objects work for this task, just make sure they don't smear the paper before use. \nHow not to lose your eraser: Drill a hole through the top and hang it on a string around your neck. Alternatively, stick it the rings of your notebook or strap it to your pencil with a rubber band.\n\nAdditional Info\nGetting pencils with erasers should eliminate this problem, though. \nHow to make an eraser\nYahoo\n\nI lost my eraser..what else can I use? and How to not lose my eraser so often?(This link I found useless).\n\nAnd some things I haven't tried:\n\nFlip Flops and Dried Bread\n\nWhat is a good replacement for an eraser that works?\nFrom user Jane:\n\nThe side edge of a flip-flop (rubber sandal) makes a good eraser.\n\nFrom user Krazy Koala:\n\nDry bread gets pencil marks off wallpaper and an rubber band has been\n known to work.\n\n", "topic": "lifehacks", "url": "https://lifehacks.stackexchange.com/questions/422"}, {"image": "VQAonline_00023355.png", "question": "How to remove burnt grease on an oven shelf?", "context": "I have a wire oven shelf (rack) which has some burned-on grease:\n\nThe shelf is covered with chrome so I cannot put it in my oven when going through the pyrolysis cycle (apparently there would be toxic fumes) and normal washing does not help.\nI noticed that the stains can be scraped with a knife so I was wondering whether I could go ahead with a metal brush. I feel that this would endanger the chrome coating. I do not care that much about the coating itself, aesthetically-speaking, but rather for its protective and somehow anti-adhesive properties.\nIs there a standard way to clean such shelves?\n", "answer": "The simplest, most effective way to clean the rack without damaging the chrome plating is probably commercial oven cleaner in a spray can. Follow the instructions on the can and you'll probably get a very satisfactory result.\nMost any \"hacky\" equivalent to oven cleaner is too hazardous to recommend -- it would involve lye, which can blind you instantly or cause serious chemical burns on contact. These methods (oven cleaner or lye) work by converting the grease component of the grime on the rack to soap, which then dissolves in water. Oven cleaner (usually a spray foam), however, controls the lye it contains so as to be reasonably safe to use.\n", "topic": "lifehacks", "url": "https://lifehacks.stackexchange.com/questions/15689"}, {"image": "VQAonline_00023351.png", "question": "How to remove whitener ink from the clothes?", "context": "I just used whitener for marking the box, but due to force pressing it burst opened from the back and spilled fully over my clothes. Is there any best way to remove the spilled whitener?\nIt's the whitener used to erase words and not like whitener used for clothes or any other\n\n", "answer": "Most products of that type, in my experience, are more like a form of paint than ink. That is, they're a latex based pigment carried in a water based solution, rather than a dye solution. Most will come off with soap and water and scrubbing, especially before they're fully dry. This obviously works best with durable clothing items; if you spilled this on a silk suit, take it to a professional, because you're more likely to completely destroy it than clean it, but if it's blue jeans, khaki, or similar sturdy stuff, get some soap, warm water, and a brush and scrub away.\n", "topic": "lifehacks", "url": "https://lifehacks.stackexchange.com/questions/14889"}, {"image": "VQAonline_00023297.png", "question": "How to keep bread fresher for longer", "context": "How do you keep bread fresh (lasting to/beyond the actual expiration date quoted on the packaging) as the bread I purchase always goes stale/moldy within a couple of days of opening.\nI've heard that freezing the bread is a viable alternative, but I'd like to steer clear of, that as I don't want to have to take the bread out of the freezer the night before to make a sandwich the next day because it needs to defrost, not to mention the fact that the bread will be soggy the next morning anyway.\nI do try to keep it in its original packaging (as suggested in this article), yet it seems to go stale/moldy regardless of where or how it is stored.\nClarification\nThis is packaged bread that you can buy from the supermarket. I always buy sliced bread and it varies from white/wholemeal. The bread would look like this:\n\nIt could be the temperature, but I keep it in a bread bin and out of damp/moist places.\n", "answer": "It's all about location, location, location!\nThere are a couple of things that will aid the growth of mold and bacteria and it is best to try and prevent these conditions being met in order to prolong the freshness of the bread. The things that I can think of are:\n\nHeat - Don't allow the bread to be in a warm / hot location, room temperature is fine.\nLight - Try to keep bread in a dark place like in a bread bin or just in a cupboard away from both natural and artificial light.\nMoisture - Keep the bread away from any moisture and try to prevent moisture getting onto the bread, again this can be done by using a bread bin or a cupboard with only dried goods such as pasta. Also always redo the packaging in order to keep any air and moisture out.\n\nSo if getting a bread bin is an option then I would suggest that, and suggest that it be kept away from windows to avoid any sunlight heating it up and turning it into a bread sauna. Do not store the bread in a fridge as this will only increase the speed at which it will go stale but it would prevent mold - not really a good compromise but that is down to you. \n\nRefrigerating the bread speed up the crystallization of the starch molecules which is one of the main reasons for bread going stale and this happens much more quickly when just above freezing temperature as opposed to being at room temperature. See here for details. Also Sheldon in Big Bang Theory says it so it must be true.\n\nAs you've mentioned freezing helps a lot as it stops any mold full stop, you would have to thaw it but this can be done with a microwave in about 10 seconds or a toaster for a little bit longer (I'm not sure on the specific for that one but it isn't too long). You are correct in that the bread is slightly soggy once defrosted but if you are game for a toast sandwich then just leave it in the toaster to crisp up and it will give you a delicious crunchy sandwich!\nIf you fancy a change from white or 50/50 bread - try wholewheat bread, this takes a longer time to develop mold and bacteria (or at least it takes longer for you to notice it) and is also more healthy for you, but keep in mind you should still store the bread in a dark, dry place.\n", "topic": "lifehacks", "url": "https://lifehacks.stackexchange.com/questions/5610"}, {"image": "VQAonline_00023404.png", "question": "Influence of Polish and Czech on the phonology of German dialects", "context": "German has for more than 1000 years been in contact with West Slavic languages, notably Polish and Czech. This is highly likely to have led to borrowing or interference between these languages, in both directions. It seems that discussion of this is usually restricted to influence from German on Polish and Czech. For example, the Polish dialect spoken in the Pozna\u0144 region is reported to be heavily influenced by German. There is also a great number of German loanwords in Polish, such as malowa\u0107 from G. malen (both to paint).\nI'm interested in influence in the opposite direction, from Polish and/or Czech to German (standard or other dialects), particularly on pronunciation. There is a Euromosaic study alluding to some influence of Polish on Silesian and East Prussian dialects, but it doesn't give any details or sources.\nPossible influence\nThe rounded vowels /y/ and /\u00f8/ are realised as /i/ and /e/ in the Saxon dialect and used to be pronounced this way in Silesian and East Prussian dialects of German:\n\n'K\u00fche' as 'Kihe', 'S\u00fcden' as 'Siden'\n'H\u00f6hle' as 'Hehle'\n\nThis is, incidentally, also the way many learners of German with Polish as first language pronounce these vowels. These rounded vowels do not exist in Polish.\nThe Slavic influence on some German dialects might have occurred as early as the 10th c. as this is the time German eastward expansion into (not only) Slavic territory started. The relevant Wikipedia article states that old and new population frequently mixed, so there were many opportunities for contact between people.\nThe following Wikipedia map shows the time frame for different regions:\n\nQ: Can anybody confirm Polish influence on (East) German dialects, in general and in particular for these vowels?\nInfluence of Polish on varieties of German spoken in the Ruhr region has been documented, but this influence was more restricted in time. There was Polish migration to the Ruhr region in western Germany in the 19th c., and influence also seems to be restricted to some lexical items.\n", "answer": "Areal features are often under-appreciated, especially the more subtle structural and semantic ones, as opposed to the more superficial lexical and phonological ones.\nAnd the contact between Slavic and Germanic was certainly significant, and started before the written record, which therefore makes it difficult to fully understand.\nAnd there are more commonalities between Slavic and German phonology than chance alone would yield.\nBut the concrete example you give is provably not an example of Slavic influence on German.\n\n'K\u00fche' as 'Kihe', 'S\u00fcden' as 'Siden', 'H\u00f6hle' as 'Hehle'\n\nThis phenomenon, Entrundung, is widespread in the Southwest too, including Alsatian in France, far away from Slavic influence. Quoting Wikipedia: Im Deutschen gilt Entrundung der mittelhochdeutschen Umlaute \u201e\u00f6\u201c, \u201e\u00fc\u201c und \u201eeu/\u00e4u\u201c zu \u201ee\u201c, \u201ei\u201c und \u201eai\u201c in den meisten Dialekten des Oberdeutschen und des Mitteldeutschen.\nThat's also why Yiddish has it. The reason Polish and Czech have it for loanwords like Miller is because of local learned convention, perhaps based on the dialects with which they were in contact. Other Slavic languages like Russian and Bulgarian work differently, they tend to palatalise instead, so \u00fc and \u00f6 become ju and jo. Some Slavic languages take the entrundung approach for German and French loanwords but just drop the umlaut for Ottoman ones, which again suggests a learned convention. Also note that in no Slavic language is there entrundung of eu (to aj) like in German dialects or Yiddish. And this pattern is not unique to Slavic languages - in Armenian and Georgian it is like in Russian and Bulgarian.\nAs a rule, German dialects vary much more North to South than East to West, historically and today.\nConsider also that there were third groups, notably Baltic and Celtic, which were absorbed en masse into both Slavic and Germanic, which could explain some commonalities.\n", "topic": "linguistics", "url": "https://linguistics.stackexchange.com/questions/4477"}, {"image": "VQAonline_00023420.png", "question": "Recognize this script?", "context": "I've wondered about this script since I saw it years ago. I imagine it's an English cipher. Can anyone tell me?\n\n", "answer": "Sorry for digging up this old question but we finally have an answer.\nI reposted this question to puzzling stackexhange thinking they would be better equipped to solve it. Surely enough, within fifteen minutes of my posting, the user Deusovi recognized the script as the Elian script and deciphered the first few lines. Later, I spent some time to fully decipher it (except the first line):\n\npark nre renr\n the c programming langua#ge!\n i really want my pen!\n create a cetter [=better?] interface for reports\n hey mr flava! flava\n i reall#y should practice my elian s#cript more\n the d language is fun to use indeed!\n i really should practice my elian script\n mayby [=maybe?] # i should standerdiz ze [=standardize?] \n{ changes pens } \ntesting this pen\n it writes # mu#a [=much?] faster \n\n", "topic": "linguistics", "url": "https://linguistics.stackexchange.com/questions/12960"}, {"image": "VQAonline_00023474.png", "question": "Diacritic connecting c and t", "context": "What is the name and use of the extra bow on the letter c in Doctrine? Diacritic or calligraphic decoration, or misprint?\n\n\nEdit: With the name given in the comments, I found an existing good answer here:\nhttps://english.stackexchange.com/questions/25118/is-there-any-significance-in-little-curls-joining-the-st-and-ct-in-old-books\n", "answer": "It is not a diacritic, it is a ligature. These are probably more common in older typesetting.\nThe same thing can be done with the sequence as with .\n", "topic": "linguistics", "url": "https://linguistics.stackexchange.com/questions/37321"}, {"image": "VQAonline_00023481.png", "question": "What do these diagrams of vowels actually represent?", "context": "I've heard many times that spoken language is subjected to variations and we never make the exact same sound when we speak, even for the same word. If that's the case, how can you be so exact about the placement of certain vowels in diagrams like these, especially when talking about certain languages like Russian and Swedish which have too many vowels in a specific area that makes it very crowded and it seems like maintaining a distinction is pretty much impossible.\nI'm fully aware of the fact that in Swedish the primary distinction between some vowels phonemes is based on rounding, but there are still too many of them and not all can be distinguished primarily by rounding. Also, does the word primary here mean that they ARE in fact distinguished on other factors as well, which might include their positions in the vowel space, which would mean that native speakers do place them separately in their mouth when they make them?\n\n", "answer": "The \"how\" part is training on exemplars. First there needs to be some standard, and that was established by Daniel Jones with the cardinal vowel system. Essentially, Jones' students learned to produce the cardinal vowels, then taught their students. This provides reference points for a subset of the vowels in the IPA. The training tradition still exists in some institutions, and thanks to the miracle of the internet, one can find authoritative samples of the vowels (and consonants) performed by experts in the IPA, this chart from John Esling, and this chart with performances by Esling, House, Ladefoged and Wells (the Esling samples may be identical to his stand-alone collection). There are other such recordings, and classes that you can take.\nThese define the standard reference points for the IPA. If you hear Norwegian du you have a few choices as to which vowel it is closest to: credibly, [\u0289 y Y] but not [u] or [\u00e6]. If you listen to all of the reference samples, you will notice that the experts do not agree \u2013 this is a fundamental limit on the precision of the standard. Conventionally, \"u\" is assigned the value [\u0289] because (1) there is also [y] as in [dy], and (2) the vowel is auditorily closest to the standard [\u0289] for most expert performances. Each IPA symbol represents a range of values, and unfortunately ranges overlap a bit.\nThere is also a practice of micro-nudging vowel letters in the trapezoid, where the vowel [i] might be below the top line. This is the language-expert's judgment that the vowel is \"a but lower\", \"a bit further back\". Theoretically, this judgment can be aided by synthesizing vowels based on the formant values implied by this chart, and Keith Johnson did actually devise a \"click on the chart\" program which provided stimuli that aided people in making judgments as to where that vowel \"is\" (AFAIK it doesn't exist anymore for hardware reasons). One can generate comparative stimuli with Praat, which can give you a basis for saying \"further back\".\nThis is all about the phonetic values of vowels, and phonetic values are generally taken to be the primary determinant of vowel classification. However, phonological patterns also determine how people transcribe vowels. This is true to the point that people often disregard the phonetic facts in order to present a phonemic transcription, rather than a phonetic transcription. In Tigrinya, for example, the phoneme /\u028c/ is pronounced [\u028c \u025b \u00e6 a \u0275 \u0254] depending on context, but this is (almost) never indicated in transcriptions, because it is rule-governed and people typically transcribe only contrastive details.\nIn other words, transcriptions and vowel charts are usually not claims about pronunciation, they are claims about the phonological system, but those claim are influenced by ideas of what the phonetics is. A couple of examples of phonetic vagarity are that the phoneme /e/ in Norwegian is usually a diphthong on the order of [\u025b\u0259], and the phonetics vs. phonology of Russian \u044b is a matter of extreme controversy (IMO [\u0268] is phonetically wrong but might be okay phonologically).\nA chart like this might also represent mean formant values from a corpus in the language. Your particular chart looks more like a plot of computed formants, not a chart of auditory space.\n", "topic": "linguistics", "url": "https://linguistics.stackexchange.com/questions/39045"}, {"image": "VQAonline_00023438.png", "question": "Replacement of the letters in Japanese while compounding words", "context": "I've spent some time solving Fakepapershelfmaker NACLO problem, and later at solution I've read that some japanese letters do not require replacement while compounding. \nIn Japanese you should replace letters in the atomic words to give the compound word a specific meaning (for instance, initial h with b, initial k with g, and so on). As I've understood, this is a trick in order to exclude ambiguity.\n\nHowever, solution says that some letters do not require a replacement. I guess that it could lead to ambiguity of compound words. And my question is: do Japanese really has such letters? And, if yes, in which way the ambiguity in this case is excluded? \n", "answer": "This phenomenon is called rendaku, or \"sequential voicing\".\nMany phonemes in Japanese occur in voiced/unvoiced pairs. In kana writing, these are distinguished with a dakuten \"voice mark\" over the voiced version: for instance, \u304b /ka/, \u304c /ga/.\n(Side note: in a few cases this reflects historical rather than current pronunciation: /h/ voices to /b/ because it was originally /\u0278/, and /\u03b2/ isn't distinguished from /b/.)\nRendaku means that the first sound in the second part of a compound becomes voiced...most of the time. In reality, it's not quite this simple. There are a few additional rules:\n\nBranching constraints! This is what the NACLO problem is about.\nIf the sound is already voiced, nothing happens to it\nLyman's Law: rendaku doesn't happen if the second word contains any voiced obstruents\nRendaku also doesn't happen in \"A and B\" compounds: so yama \"mountain\" + kawa \"river\" could become yamakawa \"mountain and river\" or yamagawa \"river which is on a mountain\"\n\nSo as you can see, rendaku isn't completely free from ambiguity. Already-voiced sounds such as /n/, /r/, /j/ aren't affected by it at all, which is what the footnote in the problem is pointing out. But there are also specific words which completely ignore rendaku, or which cause it to happen inconsistently. A few roots become voiced when attached to nouns, but not to verbs. It doesn't help that this voicing isn't indicated at all in kanji writing.\nIn these cases, the ambiguity has to be resolved the same way as in English: context, world-knowledge, and familiarity with the language.\n", "topic": "linguistics", "url": "https://linguistics.stackexchange.com/questions/20192"}, {"image": "VQAonline_00023444.png", "question": "Is the V-T movement possible in English?", "context": "To clarify: V-T movement would be a type of movement of the V head (verb phrase) to I (or T) head (inflection phrase). Some languages, like French, allow for this movement. But during my syntax classes I've been told that such movement is illicit in English.\nHowever, in a book on syntax (A. Witkos 'Movement Rules. Foundations of GB syntax of English' p. 252) I came across this sentence:\n\nThere is a student outside the classroom\n\nit's bracketed derivation looked like this:\n\n[IP There is [VP t(is) [PP a student [P' outside the classroom]]]\n\n\nwhich shows an instance of such a movement. The 'is' trace sits in [head, VP] and then moves to [head, IP].\n", "answer": "Under the standard XP-Internal Subject Hypothesis, the derivation above is correct, and is does indeed exceptionally raise from V to T in cases like this. \nIt is true, though, that V-T movement (and further raising of finite main verbs into higher heads like Focus and C) was available in English until well after Shakespeare's time, and that, whereas in languages like Spanish, French, Italian or German it still remains generally available, in English it has since become restricted to a) auxiliaries (i.e., modals, aspectual have and be, passive be and 'dummy' do), b) the main verb be (in all its existential, identificational, or copular uses), and, to the extent the construction survives, in the speech of old-fashioned British English speakers, mainly, c) possessive uses of the main verb have, as in You haven't money, Have you money? (instead of the much more common You don't have money, Do you have money?). \nThe standard explanation for why V-T Raising has remained possible for just auxiliary verbs and main verb be used to be that none of them theta-marks its complements (and none projects a subject argument, either), which would leave them free to raise from the VP in order to check their inflectional features.\nThat may well be right for auxiliaries, certainly, copular uses of main verb be (e.g., She is brilliant, She is an engineer, She is in London, etc. in which the subject receives its theta-role from A, P or N), and for existential uses of be like the one you mention in your question, where be selects a PP complement but it is the P outside that theta-marks a student and the classroom. \nHowever, that 'explanation' gets much more dubious, or plainly unacceptable, for a) existential uses of be in which be is not followed by any theta-marker (as in, e.g., There is a cafeteria, unless some hidden/ellipted PP structure is assumed, of course), b) 'identificational' uses of main verb be, as in That tall man is my brother, where is must be a two-place predicate and theta-mark the two NPs/DPs, and c), to the extent the construction survives, possessive uses of the main verb have, which is also a two-place predicate and must theta-mark its two arguments, the subject as 'Possessor' and the object as`Theme', or whatever the right role-label is for the entity 'possessed'.\nThus, what used to be the standard explanation for the exceptional survival of V-T raising in the above-mentioned cases is far from satisfactory, and, to my knowledge, please correct me if I am mistaken, it has not been replaced with any other, but the facts are solid enough: finite auxiliaries, finite main verb be, and, marginally, finite main verb have can still raise to T/Infl and higher, whereas no other English verbs can.\n", "topic": "linguistics", "url": "https://linguistics.stackexchange.com/questions/22842"}, {"image": "VQAonline_00023432.png", "question": "Why must an AdvP have at least one AdvP as a Daughter Node?", "context": "Source: p 78, Syntax, A Generative Introduction (3 ed, 2012) by Andrew Carnie.\nThough I am only on Chapter 4 at the time of this post, I cannot wait until Chapter 6 to understand the following.\n\n\nHere is a common mistake to avoid: Notice that the AdvP rule specifies that its modifier is another AdvP: AdvP \u27f6 (AdvP) Adv. The rule does NOT say *AdvP \u27f6 (Adv) Adv, so you will never get trees of the form shown in (28) [see above]:\n\u2003\u2003You might find the tree in (27) a little confusing. There are two Advs and two AdvPs. In order to understand that tree a little better, let\u2019s introduce a new concept: heads. We\u2019ll spend much more time on heads in chapters 6 and 7, but here\u2019s a first pass: The head of a phrase is the word that gives the phrase its category.\n\nI reread pp 78-79, but still do not understand why 28 is wrong and only 27 is correct.\nWhat is the main idea here? How does the concept of Heads resolve?\n", "answer": "If AdvP immediately dominated two Advs, what would the head of the phrase be? There must be one distinct head, and in 28) this could be either of the Advs, there would be no way to clearly decide which node is the head of the phrase. Therefore, you specify which adverb should be the head of the AdvP (here it is quickly, because this is the \"main\" adverb that is again modified by another adverb) and the issue is resolved, since the AdvP containing the adverb very can in no way be the head of the adverb phrae.\n", "topic": "linguistics", "url": "https://linguistics.stackexchange.com/questions/18981"}, {"image": "VQAonline_00023520.png", "question": "Explain the 28 Feb 2022 Dilbert cartoon use of the phrase \"sticking the landing\"", "context": "Can someone explain today's Dilbert cartoon, in particular explain what \"sticking the landing\" means?\n\n", "answer": "'Stick the landing' is from gymnastics, when people pull off a tricky move and land neatly on both feet, properly balanced and with a flourish of the arms that communicates 'I did exactly what I intended to do'. It's like a whole-body 'QED'.\nFrom Wiktionary\n\nstick the landing (third-person singular simple present sticks the landing, present participle sticking the landing, simple past and past participle stuck the landing)\n(sports) To complete a gymnastic or other athletic routine involving leaps, vaults, somersaults, etc. by landing firmly, solidly, and flawlessly on one's feet.\n(aviation, astronautics) To execute a flawless landing (of an aircraft, rocket, or space capsule).\n(idiomatic, by extension) To complete a process in an impressive and conclusive manner.\n\nSo in context, the character in green (Who I am gratefully advised by @PatDobson is called \u2018Wally\u2019) wishes to spend the rest of his life as a coffee-swilling blob of organic matter.\nEDIT: I'm actively choosing not to address what the author may or may not have meant by referencing a DNA test. The test functions as the set up for the joke, and readers may bring whatever sub-text to the idea of DNA testing being used in the workplace according to their own world view without it affecting the joke.\n", "topic": "literature", "url": "https://literature.stackexchange.com/questions/21966"}, {"image": "VQAonline_00023518.png", "question": "Modern English versions of Wonders/Marvels of the East from the Nowell Codex", "context": "The famous epic poem Beowulf is known from a single manuscript, part of the Nowell Codex, which also contains other texts and fragments, including one called The Wonders of the East, sometimes also translated to modern English as The Marvels of the East, which describes fantastical beasts supposedly found in eastern regions of the world (presumably Asia).\nI'm fascinated by this apparent knowledge (or pure myth?) of Asian regions dating back to so long ago in western Europe, and I'd like to learn more about it. I tried to search online for an English translation of The Wonders of the East, as exists for Beowulf, but without success. Scans of the original manuscript are digitised online, but this is of course in Old English.\n\nHas The Wonders of the East been translated to modern English? More specifically, is there a translation which is public domain (obviously the original work is, but AFAIU [IANAL] translations are sometimes copyrighted separately) and preferably available online?\n", "answer": "I found the text on the site Rejected Scriptures. I also found the text in some other authoritative sources, and this seems to match up. (Note: leuuae means leagues.)\n(I'm not quite sure which pictures from the manuscript match up with which paragraph yet.)\n\n\nThe colony is at the beginning of the land Antimolima, which land is 500 in the tally of the lesser measurement, which are called stadia, and 368 of the greater, which are called leuuae. On that island there is a great multitude of sheep, and from there to Babylon it is 168 of the lesser measurement called stadia, and 115 in the greater measurement called leuuae.\n\nThe colony is mostly populated with merchants; there are rams born there as big as oxen, living right up to the city of the Medes. The name of that city is Archemedon. It is the biggest city after Babylon. To there from Archemedon is 300 of the lesser measurement, stadia, and 200 of the greater, called leuuae. There are great monuments there, which are the works which the mighty Alexander of Macedon had made. The land is in length and breadth 200 of the lesser measurement, stadia, and 133 and a half of the greater, called leuuae.\n\nAs you go towards the Red Sea there is a place called Lentibeisinea, where there are hens born like ours, red in color. If any one tries to take or touch them, they immediately burn up all his body. That is extraordinary magic.\n\nWild beasts are also born there. When these wild beasts hear a human voice, they run far away. The beasts have eight feet, and valkyrie-eyes, and two heads. If anyone tries to touch them, they set their bodies aflame. They are extraordinary beasts.\n\nHascellentia is the name of the land on the way to Babylon, that is in length and breadth nine of the lesser measurements called stadia. It is subject to the kingdom of the Medes, and that land is filled with all good things. This place contains serpents. The serpents have two heads, whose eyes shine at night as brightly as lanterns.\n\nIn one land there are born donkeys which have horns as big as oxen's. They are in that very great wasteland which is in the southern part of Babylonia. They retreat to the Red Sea, because of the multitude of snakes called Corsiae which are in those places. They have horns as big as ram's. If they strike or touch anyone, he immediately dies. In those lands there is an abundance of pepper. The snakes keep the pepper in their eagerness. In order to take the pepper people set fire to the place and then the snakes flee from the high ground into the earth; because of this the pepper is black. From Babylon to the city of Persia where the pepper grows is in the lesser measure which is called stadia 800 units. It is reckoned in the greater measure that is called leuuae six hundred and twenty-three and a half units. The place is barren because of the multitude of the snakes.\n\nAlso there are born there half-dogs who are called Conopenae. They have horses' manes and boars' tusks and dogs' heads and their breath is like a fiery flame. These lands are near the cities which are filled with all the worldly wealth: that is, in the south of Egypt.\n\nIn one land people are born who are six feet tall. They have beards to their knees, and hair to their heels. They are called Homodubii, that is 'doubtful ones', and they eat raw fish and live on them.\n\nThe river is named Capi in the same place, which is called Gorgoneus, that is 'valkyrie-like'. Ants are born there as big as dogs, which have feet like grasshoppers, and are of red and black color. The ants dig up gold from the ground from before night to the fifth hour of the day. People who are bold enough to take the gold bring with them male camels, and females with their young. They tie up the young before they cross the river. They load the gold onto the females, and mount them themselves, and leave the males there. Then the ants detect the males, and while the ants are occupied with the males, the men cross over the river with the females and the gold. They are so swift that one would think that they were flying.\n\nBetween these two rivers is a colony called Locotheo, which is situated between the Nile and the Brixontes. The Nile is the prince of foul rivers, and flows through Egypt. And they call the river Archoboleta, which means 'great water'. In these regions are born great multitudes of elephants.\n\nThere are people born there, who are, fifteen feet tall and have white bodies and two faces on a single head, feet and knees very red, and long noses and black hair. When they want to give birth, they travel in ships to India, and bring their young into the world there.\n\nThere is a land called Ciconia in Callia, where people are born of three colors, whose heads have manes like lions' heads, and they are twenty feet tall, and have mouths as big as fans. If they see or perceive anyone in those lands, or if anyone is following them, then they take flight and flee, and sweat blood. They are thought to be men.\n\nBeyond the River Brixontes, east from there, there are people born big and tall, who have feet and shanks twelve feet long, flanks with chests seven feet long. They are of a black colour, and are called Hostes. As certainly as they catch a person they devour him.\n\nThen there are on the Brixontes wild animals which are called Lertices. They have donkeys ears and sheep's wool and bird's feet.\n\nThen there is another island, south of the Brixontes, on which there are born men without heads who have their eyes and mouth in their chests. They are eight feet tall and eight feet wide.\n\nDragons are born there, who are one hundred and fifty feet long, and are as thick as great stone pillars. Because of the abundance of the dragons, no one can travel easily in that land.\n\nFrom this place there is another country on the south side of the ocean, which is reckoned in the lesser measurement known as stadia 323, and in the greater which is called leuuae 255. There are born there Homodubii, that is 'doubtful ones'. They have a human shape to the navel and below that the shape of a donkey, and they have long legs like birds, and a soft voice. If they see or perceive anyone in those lands, they run far off and flee.\n\nThen there is another place with barbarous people, and they have kings under them to the number of 110. They are the worst and most barbarous people, and there are two lakes there, one of the sun and the other of the moon. The suds lake is hot in the day and cold at night, and the moon's lake is hot at night and cold in the day. Their width is in the lesser measurement which is called stadia 200 units and in the greater called leuuae one hundred and thirty-three and a half.\n\nIn this place there are kinds of trees which are like laurel and olive. From these trees the most expensive oil, balsam, is wholly produced. The place is in the lesser measurement that is called stadia 151 and in the greater which is called leuuae fifty-one.\n\nThen there is an island in the Red Sea where there is a race of people we call Donestre, who have grown like soothsayers from the head to the navel, and the other part is human. And they know all human speech. When they see someone from a foreign country, they name him and his kinsmen with the names of acquaintances, and with lying words they beguile him and capture him, and after that eat him all up except for the head, and then sit and weep over the head.\n\nGoing east from there is a place where people are born who are in size fifteen feet tall and ten broad. They have large heads and ears like fans. They spread one ear beneath them at night, and they wrap themselves with the other. Their ears are very light and their bodies are as white as milk. And if they see or perceive anyone in those lands, they take their ears in their hands and go far and flee, so swiftly one might think that they flew.\n\nThen there is an island on which people are born whose eyes shine as brightly as if one had lit a great lantern on a dark night.\n\nThen there is an island, which is in length and breadth in the lesser measurement that is called stadia 360, and in the greater called leuuae 90. There was built in the days of Belus the king and Jove a temple made from wrought iron and brass. And in the same place there is east from there another temple, sacred to the sun, in which is ordained a fine and gentle priest, and he governs the halls and looks after them.\n\nThen there is a golden vineyard near the rising of the sun which has berries of 150 feet. On them, berries are produced like pearls or jewels.\n\nThere is another kingdom in the lands of Babylon where there is found the biggest mountain between the mountain of Media and of Armenia. It is the biggest and highest mountain of all. There are decent people there who have power and dominion over the Red Sea. Precious jewels are produced there.\n\nAround those places there are born women, who have beards down to their breasts, and have made clothes out of horse's hide. They are called great huntresses, and instead of dogs they breed tigers and leopards, that are the fiercest beasts. And they hunt for all the kinds of wild beasts which are born on the mountain.\n\nThen there are other women who have boar's tusks and hair down to their heels and ox-tails on their loins. Those women are thirteen feet tall and their bodies are of the whiteness of marble. And they have camel's feet and boar's teeth. Because of their uncleanness they were killed by Alexander the Great of Macedon. He killed them because he could not capture them alive, because they have offensive and disgusting bodies.\n\nBy the ocean is a breed of wild animals that is called Catini, and they are very beautiful animals. And there are people there who live on raw meat and honey.\n\nOn the left-hand side of the kingdom in which there are wild animals called Catini, there are hospitable people, kings who have subdued many tyrants. Their boundaries border on the Ocean, and from there, from the left-hand section, there are many kings.\n\nThis race of people live for many years, and they are generous people. If anyone visits them they give him a woman before they let him go. When Alexander of Macedon visited them, he was amazed at their humanity, and would not kill them or cause them any harm.\n\nThen there are kinds of tree from which the most precious stones are produced, and upon which they grow.\n\nThere is another race of people there of black color to look at, who are called Ethiopians.\n\nThen there is land in which very many vineyards grow, where there is a couch of ivory. It is 306 feet long.\n\nThen there is a mountain called Adamans. On that mountain is the kind of bird which is called a Gryphon. Those birds have four feet and the tail of a cow and the head of an eagle.\n\nIn the same place is another kind of bird called Phoenix. They have crests on their heads like peacocks, and they build their nests from the most precious spices, which are called cinnamon; and from its breath, after a thousand years, it kindles a flame, and then rises up young again from the ashes.\n\nThen there is another mountain where there are black people, and no one else can approach those people because the mountain is all aflame.\n\nHere it says how Mambres opened the magical books of his brother Iamnes, and to him were revealed the deep mysteries of his brother's idolatry. The soul of Iamnes answered him with these words: 'Brother, I am dead not unjustly, but rightly and justly am I dead, and God's judgment stands against me because I alone was wiser than all the other sorcerers, and I withstood the two brothers called Moses and Aaron, who performed those great portents and signs. For that reason am I dead, and for this am I brought to the middle kingdom of hell, where there is the great heat of eternal punishment, and where there is the pit of perpetual torment from which no one ever ascends. Now, my brother Mambres, take care that you do well to your children and your friends, because in hell there is nothing good, only misery and darkness; and after you are dead, then you will come to hell, and your dwelling-place will be among the dead, down in the ground, and your pit will be two cubits wide and four cubits long.'\n\n\n\n", "topic": "literature", "url": "https://literature.stackexchange.com/questions/19219"}, {"image": "VQAonline_00023508.png", "question": "What does Justice Scalia mean by \"Thoreauvian 'you-may-do-what-you-like-so-long-as-it-does-not-injure-someone-else' beau ideal\"?", "context": "I'm more interested in literature than law here. I think this means the same thing as \"your rights end where mine begin\"? Scalia is alluding to Henry David Thoreau, but my English is too unproficient to read Thoreau. Why does Scalia allude to Thoreau? What's \"beau ideal\"?\n\n\u00a0 An example of an opinion rejecting the evaluative portion of a policy argument is Justice Scalia's separate concurring opinion\nthe \"nude dancing\" case Barnes v. Glen Theatre.323 In finding that there was no First Amendment right to perform or view nude dancing, Scalia wrote, \"There is no basis for thinking that our society ever shared the Thoreauvian 'you-may-do-what-you-like-so-long-as-it-does-not-injure-someone-else' beau ideal\u2014much less for\nthinking that it was written into the Constitution.\"324\n\nWilson Huhn. The Five Types of Legal Argument (2 ed, 2014). Pg 140.\n\n", "answer": "Thoreau was a proponent of limited government. His famous essay Civil Disobedience begins:\n\nI heartily accept the motto, \"That government is best which governs least\"; and I should like to see it acted up to more rapidly and systematically. Carried out, it finally amounts to this, which also I believe- \"That government is best which governs not at all\"; and when men are prepared for it, that will be the kind of government which they will have.\n\nThoreau took the motto \"That government is best which governs least\" from an unsigned editorial in the first issue of The United States Magazine and Democratic Review (1837). This quote from the editorial explains Scalia's characterization of Thoreau's political philosophy:\n\nThe best government is that which governs least.... Government should have as little as possible to do with the general business and interests of the people. If it once undertake these functions as its rightful province of action, it is impossible to say to it, \"thus far shalt thou go, and no farther.\" (p. 6\u20137)\n\nIn other words, government should not regulate anything that it does not absolutely have to:\n\nIts domestic action should be confined to the administration of justice, for the protection of the natural equal rights of the citizen, and the preservation of social order. (p. 7)\n\nThis is a libertarian, laissez-faire ideology. Under such an ideology, viewing nude dancing would not be banned. If someone is willing to dance nude, and someone else willing to watch, it's not the government's business to step in and outlaw that.\nScalia argues that beautiful as this philosophy might be, it is not widely shared in American society. He also says it cannot be found in the Constitution and so is not fundamental to American government. He therefore says that regulating nude dancing is something that would fall under the government's remit.\nPersonally, of course, I find neither this alleged beau ideal nor Antonin Scalia's jurisprudence the slightest bit attractive, nor even intellectually honest, but that's neither here nor there.\n", "topic": "literature", "url": "https://literature.stackexchange.com/questions/16273"}, {"image": "VQAonline_00023489.png", "question": "How long have the cantillation marks been around?", "context": "Over the whole Jewish Bible, there are cantillation marks that denote how the text is supposed to be chanted. The sounds differ between places, such as between the Ashkenazi and the Sephardic communities. However, as far as I can tell, the marks are the same. How long have the current cantillation marks been around?\nHere's an example of some marks:\n\n", "answer": "Current consensus seems to be that cantillation marks as they are currently used originated around the 9th or 10th century CE, relatively recently in the history of Judaism. Cantillation itself has existed for a long while, but cantillation marks are a relatively recent means of codifying an otherwise opaque word-of-mouth (and potentially chironomical) system. There are very few facts about cantillation that are commonly held to be true, and there is a lot of debate about what the real history is.\nThis codification wasn't exactly early on in the history of Judaism - if it were, we'd expect to see more references to the marks specifically in the Talmud. Baruch Davidson, via Chabad notes that:\n\nThe use of the cantillation marks in current use dates to at least the 9th-10th century CE. This was the era of the Masoretes, meticulous scribes in Tiberias, Jerusalem and Babylon who worked to establish a precise common text, vowelization and cantillation for the Tanakh.\n\nThis is echoed by Cantor David Pincus:\n\nEventually, in the 9th century C.E. (in the \"Common Era\") a family of scholars in Tiberias codified a system of printed symbols which would indicate: 1) the grammatical phrases of each Torah verse, and 2) a series of notes for each sign.\n\nHowever, this doesn't preclude the possibility of cantillation marks predating this time - it only serves as a claim that our modern cantillation originated around that time. It does indeed appear that other forms of cantillation, both borrowed and manufactured, existed before the 9th century, and originated out of a practice called chironomy, where hand-gestures were used to indicate how a part should be sung. But it's plausible that information on the exact origins has been lost; the Jewish Encyclopedia states:\n\nAttempts have been made to reconstitute the oldest form of the cantillation by J. C. Speidel [et. al.]. ...their conclusions [outweigh] the probable.\n\nSo, there you have it. It would seem to have originated in the 9th or 10th century out of a desire to codify, and make a pass at standardizing, the way cant works. \nAs to the differences between Ashkenazi and Sephardic cant, well - cantillation isn't a part of the Torah itself, and doesn't necessarily have to stay fixed. Since there's no religious imperative to maintain universal cant in Judaism, it makes sense that different groups and regions would drift apart in cant at different times. (If you've ever been to services in differing temples, you'll know what I mean - many of the cants sound similar, and although frequently, are not always identical.)\n\nMore history for the curious: the origins of cantillation as a practice are not dated, and potentially aren't dateable at all. The Jewish Encyclopedia cites that:\n\nThe earliest reference to the definite modulation of the Scripture occurs in the Babylonian Talmud (Meg. 32a), where R. Johanan deprecates the indifference of such as \"read [the text] without tunefulness and repeat [the Mishnah] without song.\"\n\nSo it's known that as a practice it goes back at least that far. Chabad holds a slightly less reserved view:\n\nThe tradition of the ta'amim by which the Torah is to be sung, however, is as old as the Torah itself. It was taught to Moses together with the vowels, as it is integral to the correct understanding of the Torah.\n\nEither way, this establishes that the origins of cantillation itself seem disconnected from the origins of cantillation marks, which is tangential, but important.\n", "topic": "literature", "url": "https://literature.stackexchange.com/questions/1370"}, {"image": "VQAonline_00023490.png", "question": "Does V reference Stanislaw Lem's \"His Master's Voice\", and if so, why?", "context": "I've recently been reading and re-reading Alan Moore and David Lloyd's V for Vendetta. After having skimmed through it multiple times and settling a few questions I had about the continuity, only one question remains. In volume I, chapter 2, The Voice, on page 13 the following conversation occurs:\n\n\n... they eradicated some cultures more thoroughly than they did others.\n No Tamla and no Trojan. No Billie Holiday or Black Uhuru...\n Just His Master's Voice. Every hour. On the hour.\n We'll have to see what we can do about that...\n\nItalics authors', bold emphasis and choice of capitalisation mine\nV says \"his master's voice\", which is also the title of the novel by Stanislaw Lem.\nI can see that V is most likely talking about the order in which the citizens of post-war Britain live - the oppression, constant surveillance, fascism, etc. V's primary goal throughout the novel is to make people raise their heads and pave their own way with their own decisions, as opposed to their masters' voice.\nBut still I find the choice of words here peculiar. Having skimmed through Lem's novel's description on Wikipedia I can see that it is not exactly about the same things as V for Vendetta. There is a bit about \"criticizing Cold War military and political decision-making as corrupting the ethical conduct of scientists\", but as I said, this is not what V for Vendetta is about.\nSo here comes my question: is this quote a conscious reference to Stanislaw Lem's His Master's Voice? If so, how exactly are the two books linked? If not, what explains this choice of words (for instance, \"their master's voice\" would be more suitable, I think)?\n\n I would appreciate if spoilers from Lem's novels be tagged as such \n", "answer": "No, it has nothing to do with Lem's novel. It's a play on the famous trademark originated by the Gramophone Company (and later used by EMI) in the UK and used by Victor (and later RCA) in the USA. Take a look at the context:\n\nTamla and Trojan are both record labels known for releasing music by (respectively) African-American or African-British artists. (Tamla was the name used in the UK by Motown Records.)\nBillie Holiday was an African-American jazz & blues singer.\nBlack Uhuru is a Jamaican reggae group.\n\nEssentially, V is saying that music by black people has been eradicated from the national media, and replaced by state propaganda.\n", "topic": "literature", "url": "https://literature.stackexchange.com/questions/2122"}, {"image": "VQAonline_00023516.png", "question": "What do you call the \"message\" written before a poem?", "context": "From Re Judicial by Baticuling (Jes\u00fas Balmori) in El libro de mis vidas manile\u00f1as (1928)\n\nBefore the first stanza, the author begins with a little message: \"Antonio Manipula, juez ... y falsificaci\u00f3n.\" This provides the reader with some background on what the poem is about: some judge named Manipula condemned for embezzlement and fraud. The first two lines say \"these days, that who doesn't rob is a mule and those who don't embezzle are fools\", and goes on in how such crimes are \"ordinary\" (corrientes).\nWhat is the proper term for such a message?\nThis isn't limited to spanish-literature because I think that there are English poems with such a feature, although I can't find any.\n", "answer": "These are very rare in English (I don't know how common they are in Spanish); I've identified two poems that have them. A word fr them is headnotes.\nSome of these are arguments. From Wikipedia\n\nAn argument in literature is a brief summary, often in prose, of a poem or section of a poem or other work. It is often appended to the beginning of each chapter, book, or canto. They were common during the Renaissance as a way to orient a reader within a large work.\n\nFor example, Around the World in Eighty Days has an argument at the beginning of each chapter, the first one being:\n\nChapter I\nIN WHICH PHILEAS FOGG AND PASSEPARTOUT ACCEPT EACH OTHER, \nTHE ONE AS MASTER, THE OTHER AS MAN\n\nIf you need a name for a message at the beginning of a poem which is not an argument, you could use the word \"headnote\". The definition in Collins Dictionary\nis\n\na brief explanatory note prefacing a chapter, poem, story, legal report, etc.\n\nwhich seems to fit.\nThey are probably not \u201cepigraphs\u201d \u2014 The Poetry Foundation\ndefines epigraph as:\n\nA quotation from another literary work that is placed beneath the title at the beginning of a poem or section of a poem.\n\nThis is a poetry-centered definition. You can have epigraphs before chapters and stories as well. However, they generally have to be quotes from some other work.\nI also wouldn't call them prologues. There are some famous poems that have prologues:\nThe Canterbury Tales, Piers Plowman, Robert Browning's the Two Poets Of Croisic. All of these are actually parts of the poem, written in poetry and not prose.\nOne poem I found with a headnote is Coleridge's The Rime of the Ancient Mariner, the original version. It starts out as follows, with what Coleridge calls an argument:\n\nTHE RIME OF THE ANCYENT MARINERE \nARGUMENT.\nHow a Ship having passed the Line was driven by\nStorms to the cold Country towards the South Pole;\nand how from thence she made her course to the\ntropical Latitude of the Great Pacific Ocean; and of the\nstrange things that befell; and in what manner the\nAncyent Marinere came back to his own Country.\n\nThe other poem I found with a headnote is by Richard Wilbur. It starts:\n\nBallade for the Duke of Orl\u00e9ans\nwho offered a prize at Blois, circa 1457, for the best ballade employing the line \u201c Je meurs de soif aupr\u00e8s de la fontaine.\u201d\n\nThis poem, despite containing the line \u201cI die of thirst, here at the fountain-side\u201d is not a translation of one of the dozen or so Middle French poems we know containing that line, but a new ballade written by Wilbur.\n", "topic": "literature", "url": "https://literature.stackexchange.com/questions/18520"}, {"image": "VQAonline_00023503.png", "question": "What is meant by the word \"cumfarting\" in the 1742 English novel \"Joseph Andrews\"?", "context": "Yes I know it sounds like a superslang portmanteau of modern obscenities (and indeed it is), which is why I was surprised to see it in a 1742 book, and why I'm asking.\nToward the beginning of Book 4, Chapter 5 of The History of the Adventures of Joseph Andrews and of his Friend Mr. Abraham Adams is the following passage, with bolding added by me\n\n\"Why, there it is in peaper,\" answered the justice, showing him a deposition which, in the absence of his clerk, he had writ himself, of which we have with great difficulty procured an authentic copy; and here it follows verbatim et literatim:\u2014\n\n\n\nThe depusition of James Scout, layer, and Thomas Trotter, yeoman, taken before mee, one of his magesty's justasses of the piece for Zumersetshire.\n\n\n\n\n\"These deponants saith, and first Thomas Trotter for himself saith, that on the \u2014 of this instant October, being Sabbath-day, betwin the ours of 2 and 4 in the afternoon, he zeed Joseph Andrews and Francis Goodwill walk akross a certane felde belunging to layer Scout, and out of the path which ledes thru the said felde, and there he zede Joseph Andrews with a nife cut one hassel twig, of the value, as he believes, of three half-pence, or thereabouts; and he saith that the said Francis Goodwill was likewise walking on the grass out of the said path in the said felde, and did receive and karry in her hand the said twig, and so was cumfarting, eading, and abatting to the said Joseph therein. And the said James Scout for himself says that he verily believes the said twig to be his own proper twig,\" &c.\n\n\nGoogle Ngram shows virtually no use of the word other than, presumably, copies of this book itself (huge spike in 1742) and, after 1880, mostly few analyses of it.\n\nI found one dissertation by Przemys\u0142aw U\u015bci\u0144ski which suggests in a footnote the word cumfarting is part of \"a brilliant parody of tedious legal language\" (pg. 164).\nHelen Thompson suggests something similar, though more specific, in Fictional Matter, her book on chemistry and English novels. In fact she quotes the word specifically and individually (pg. 215):\n\nTypography concretizes\u2014and, in the eyes of the reader whose literacy enlists her in Fielding\u2019s game, rectifies\u2014an abuse of justice also enacted in words.\n\n\n\u201cCumfarting\u201d obviously invalidates the juridical text\u2019s claim to propri- ety. In this capacity, language in Joseph Andrews acts: by turning words into things, Fielding lends them qualities that override the prejudicial enactment of the law they would enable.\n\nHowever everyone seems to be skirting the question: What did cumfarting mean? Or, assuming it's a neologism, what might it have been imagined to mean by an 18th century reader?\nDid \"cum\" and \"fart\" have the same meanings in 1742 as today (to orgasm & to flatulate, respectively)? Did one but not the other, and it pairing just sounded funny?\nOr was it just a completely meaningless pun on \"comforting\" that only today sounds sexual/filthy? If so, what\u2019s the evidence of meaninglessness? Thompson and U\u015bci\u0144ski seem to argue it was a double entendre, but don\u2019t specify the second entendre.\n", "answer": "This is not an obscenity about \"climaxing\" or \"passing gas.\" It's a phonetic rendering of what was then spoken English. For instance, the sexual meaning of \"come\" is a modern usage dating back to the 1970s.\nThe bolded text, \"and so was cumfarting, eading, and abatting,\" reads in modern English, \"and so was comforting, aiding, and abetting...\" That might be a pleading in a court proceeding.\n", "topic": "literature", "url": "https://literature.stackexchange.com/questions/13592"}, {"image": "VQAonline_00023594.png", "question": "Punching with knuckles first correctly", "context": "It seems like when I punch and need to punch with knuckles first, I have to angle down, otherwise, it hits the bottom of fingers first and not the knuckles, when practicing against heavy bag.\nCheck the angle of pictures below,\nis this true for you all? Or how do you deal with the angle of knuckles and first contact? See how it hits the green line below.\n\nStock photos above (not of me, which show what I mean) :\n", "answer": "It is normal for the middle knuckles of your fingers to lead your first knuckles (top knuckles). This protrusion of the middle knuckles is exaggerated when wearing gloves. Your middle knuckles will be pushed back in towards your palm upon impact whilst your first knuckles will move very little, providing your wrist is firm and straight.\nDo not compensate by tilting your wrist, or you will be at far greater risk of wrist injury and much of the potential power of your strike may be lost by your wrist buckling upon impact.\nConcentrate on aligning the bones which run along the top of your hand - from the knuckles of your first and second fingers to your wrist - with the bones of your forearm. Focus on maintaining this alignment at the point of impact and on directing your power through the top knuckle of your second finger alone, or in combination with the top knuckle of your first finger. The sharper the tip of the spear, the more easily it will penetrate the body.\nRemember that in a fight, there are many target areas of a body which protrude, unlike a heavy bag's relatively flat surface. This, along with punch angle, means it is sometimes possible to land cleanly with your first knuckles alone, but it is otherwise normal for your middle knuckles to reach the target first.\n", "topic": "martialarts", "url": "https://martialarts.stackexchange.com/questions/13332"}, {"image": "VQAonline_00023531.png", "question": "Falling leaf ukemi", "context": "What is the Japanese terminology for a falling leaf break fall?\nFor those who do not know what it looks like. From standing, you would train it by putting both hands on the floor, kicking your legs upwards (almost to an hand stand), and lowering your arms as your legs go down in a see saw motion -- looking like a falling leaf. It is the ukemi that you would do when (for example) tori does oahitaoshi from kneeling. Or the ukemi from Hiki taoshi. In the drawing of hiki taoshi below, the falling leaf breakfall is what uke does in steps 7 to 10.\n\n", "answer": "Yes, I believe this is called, \"zempo ukemi\".\nNote that this is not \"zempo kaiten ukemi\". The \"kaiten\" part means rolling, and without rolling, you simply have \"zempo ukemi\".\nI didn't find much on the subject on the web. There is this page which you can send to Google Translate:\nhttp://escuelakuroobi.wordpress.com/2012/02/09/155/\nIt describes the Zempo Ukemi as a \"forward fall using forearms and thighs or toes to cushion the fall. It is also recommended turning the head to one side to avoid a blow in the face.\"\nI also found this article:\nhttp://kitakazebudo.blogspot.com/2010/04/forward-splat-fall.html\nIt seems the author of that article also sees this as a \"zempo ukemi\", but isn't completely sure about the name.\nIn Aikido circles, I see this often referred to in English as the \"soft\" version of the front breakfall.\nAnd in English, I see it often referred to as \"The Worm\" breakfall (probably named after the dance move by the same name). Oh, and I've also seen it referred to as \"The Wave\" breakfall.\nI think the reason for the naming confusion might be that this actually has no specific name in Japanese. There are many versions of \"mae ukemi\" (front breakfalls), each with a particular variation. Like you might see a traditional front breakfall, the kind without the rolling, but instead of falling down straight, you do a kind of split outwards with your legs and fall from the center of mass instead. We used to call that a \"parachute\" front breakfall, because you're straddling your legs out like a parachute. But in Japanese, this is still just called mae ukemi. They don't usually distinguish it from the regular version.\nAnd there are other types of mae ukemi as well. They're all named \"mae ukemi\". Like the Bujinkan ninjutsu guys have a variation of mae ukemi where you go down on one knee while pressing back up into the air with the other leg.\nThat's about all I can offer on this subject.\n", "topic": "martialarts", "url": "https://martialarts.stackexchange.com/questions/2933"}, {"image": "VQAonline_00023550.png", "question": "Name and meaning of stance where you stand with fists on hips?", "context": "After the first Kiai in Heian Sandan (source):\n\nBring left foot to the right, pivot left 180 degrees on the right\n foot, informal-attention stance (slowly), bring fists to hips, elbows\n to the sides, backs of fists face forward\n\nThe stance at the bottom-right corner of the image below, marked as the 11th move (image taken from here):\n\nWe didn't get to study this Kata's Bunkai yet in the class I attend.\nWhat is the meaning of this move and stance? To a novice like me it seems like a vulnerable position that I wouldn't expect to being used in combat. \n", "answer": "Your intuition is correct. It is a very vulnerable position. That should give you a clue about its bunkai.\nFor those that don't know, Bunkai is the Okinawan karate term used to refer to the practice of analyzing a kata for its self-defense applications.\nRight off the bat, you know this is not a block. And you can conclude that because it would be insane to keep your hands low and at the hips like that if you're expecting someone to punch you.\nAnother clue is that the form has you step forward and then turn around.\nWell, what these observations tell me is: 1) Someone is either holding onto you, or you are grabbing someone else, because you're not expecting a strike. And 2) That person is behind you after you turn around.\nLet's observe the bent elbows also. Why bend them like that?\nThe next thing you do from this position is to make a big step out while turning your body 90 degrees. You drop into a low horse stance. And you do this while keeping your elbows and arms locked in that original position.\nBut before that, did you notice that high knee? You actually bring that knee way up as high as your chest! What's that about?\nAt the end of this step forward, you twist your torso counter-clockwise and press forward with your right elbow a bit while remaining upright. You keep your arms in their initial position chambered at the hips throughout the movement.\nWhy would you not move your arms from the initial position to the next position?\nI think we're nearly done. We just have to add one more observation. Position 12 to position 13 follows up by doing a back-fist to head height. What's that about? It must mean our arm is now free. Was it free all along?\nOkay, let's get to the explanation.\nThis is what Judo refers to as an \"O-Goshi-Nage\" (hip throw), but there's a twist, which I'll get to in a moment.\nFirst, in the movement just before you turned around (movement 10 in the diagram), the position has your right arm straight out after a punch to the solar plexus. And your left fist is chambered at your left hip. It's a pretty typical \"karate\" stance. What does it mean?\nThe left fist chambered on the left hip is an indication that you've grabbed someone by their wrist or their sleeve and have pulled it back to your hip. Otherwise, it's ridiculous to keep your left hand that low. In a real fight, you would only do that if you had grabbed his wrist or sleeve like I said. You would never want to drop your hand to your hip in real life.\nThen you step forward with feet together and turn 180 degrees. While doing that, you're putting your arms in the weird elbows-out position with your fists both chambered at your hip. Why?\nWell imagine a guy is in front of you. You've just grabbed his right wrist with your left hand and yanked it to your hip. You took your other arm (right arm) and reached around behind his back and grabbed a hold of his belt or his clothing at his right hip. And you turned your back towards him as you did it, so that your back is now against his front.\nYou still have control over his right wrist. It's still being held securely at your left hip at this point.\nThis position now looks more or less exactly like the kata's position 11.\nNext, you bring your knee straight up high. Why?\nWell if you have control over his right arm by pulling it to your left hip as I've mentioned, his right elbow is now pointed down. That means you get an arm break for free here by simply bringing your knee straight up into his elbow. The leverage provides the strength to keep his arm straightened as you knee it. Snap!\nIncidentally, this explains why your feet are together as you turn around and why you're in a high stance. Usually you want a lower stance if you do a hip throw like you're about to do next. But if you're in a low stance, you can't break that arm of his as easily.\nAnd I've seen variations of this form where you don't just bring your knee straight up, you sweep it inwards also, as you bring it up. That could be more effective for breaking the elbow, since the elbow points slightly outward and not just perfectly downward.\nNext, you step out on a 90 degree angle into a low horse stance while maintaining the original arm positions. Why?\nThis is the hip throw (o-goshi-nage). You maintain control on his right wrist with your left hand while using your hip to throw him over your right side. You let go of him at his waist, though, after the throw. The low horse stance gives you the leverage. Taking a step while doing it provides some additional momentum.\nPosition 13 has you then do a back-fist high. We've seen this before in classical jujitsu, aikido, and judo. It's used as a distraction. It's a way of saying, \"You might want to begin this sequence by doing a quick back-fist to his face.\"\nWhile he's distracted by the back-fist, you can do stuff to him more easily, including that elbow break.\nBut I left out one detail: What is the scenario where you might find yourself able to use this?\nThere are actually many scenarios where it can apply. The basics are: Attacker is facing the defender. The attacker has his right hand forward in a position that can be grabbed easily by the defender. The attacker's left hand can be at his side, or it can be grabbing the defender's lapel.\nI have one particular scenario in mind that I think makes the most sense:\nImagine someone is threatening you with a knife in his right hand. He's pointing it at you in front of him. Maybe his right leg is forward, or maybe he's standing square with you. His left hand is in a guard position, perhaps. \nYou begin by doing a quick back-fist to the face. That causes him to be distracted, and his eyes will be closed and unable to see what you're about to do, if you're fast enough.\nNext, your left hand reaches up and grabs his right wrist or sleeve from the inside. Some people call this is a \"monkey grab\". You pull that wrist back to your left hip while twisting his wrist so that the under-side is pointing upwards, and his thumb is pointing outwards. This elongates his arm and points his elbow downwards. It also means that blade of the knife is pointing away from your body, which is important. Otherwise, he might use that knife to slash or stab at your abdomen.\nAnd then you complete the application. You step inwards to loop your right hand back behind him, then do the standing elbow break with your knee, and finish with a hip throw.\nThe elbow break is only important if your opponent has a weapon in his hand. It removes the threat of that weapon. And that's why I chose this scenario. Also, if he has a knife, twisting his wrist with a sudden jerk to your hip can cause the knife to go flying out of his hand.\nAn alternative to this bunkai is to go all the way back to the beginning. This time your left hand grabs his left hand instead of his right hand. And so you yank his left hand to your left hip, step forward, turn 180 degrees (your back towards him), and then press your right elbow against his elbow as you turn. You're side by side now. The pressure will cause an arm bar. Stepping out and at 90 degrees will extend that arm bar and throws him by it. It could also break that arm. The back-fist at the end can be used if the throw wasn't successful and it merely moved him.\nThat alternative bunkai feels good to me, but it lacks the high knee. It wouldn't make sense in this context. So this means that this bunkai is less likely to be the original intent of those movements in the kata in my opinion.\nBeing able to see this stuff requires cross-studying in classical jujitsu, in my opinion. Because, that's what karate kata is. It's classical jujitsu, encoded in a unique way meant to make it easier and more reliable to remember.\nHope that helps.\n", "topic": "martialarts", "url": "https://martialarts.stackexchange.com/questions/7008"}, {"image": "VQAonline_00023582.png", "question": "Punching: Which way to Rotate Knuckles?", "context": "Why do they teach person to hit with knuckles horizontal in especially MMA and Muay Thai? It seems this can cause wrist damage, and hyperextension over the years. In Krav Maga they say to rotate muscles at 45 degrees.\na) if you test your bare knuckles against wall, for most people\nindex/middle knuckles will naturally hit at 45. https://youtu.be/67fcgggDIDc?t=204 (there are certain exceptions, as everyone has different anatomical features)\nb) if you hit at horizontal, it will work. However, you may have to bend wrist down so your knuckles naturally hit (this can cause wrist jam or hyperextension)\nc) can also punch with 135 degrees of rotation, with index/middle knuckles down. However, this may cause wrist twist jam and inflexibility. https://youtu.be/fWeo0Od8xxM?t=117\nPunching uses index and middle knuckles, so not sure why people are going with options b and c. Sure rotating can cause more damage, but by how much? I actually get little more power and flexibility range, with only slight rotation. Any insight is helpful.\nResource: Complete Krav Maga: The Ultimate Guide to Over 250 Self-Defense and Combative Techniques, Darrin Levine\n\n", "answer": "There are lots of different ways to punch: There's perfectly vertical like kung-fu / wing-chun does it, also known as a \"half-twisting\" punch. There's the 3/4 twisting punch like your krav-maga and Okinawan karate tends to do. There's perfectly horizontal like Japanese karate and Taekwondo does it, also known as the \"full twisting\" punch. There's the corkscrew punch after that, which you're going to see most often in boxing.\nSo which is right? Which is better?\nThe answer is: It depends, all, none.\nThere are advantages and disadvantages to all of them. While the risk of breaking your finger bones or spraining your wrist is higher when punching hard objects like heads, no particular punch does better in this regard. That's provided you're doing them right. And to know if you're doing a punch wrong, you'll want to punch a heavy bag to see whether or not it would cause any problems in real life.\nThat's absolutely vital. If you don't train using a heavy bag or some kind of hard resistance, it won't matter if your form is technically perfect, you're just going to end up hurt. You need to condition your body to punch solid objects. And you can condition your body to punch in any of these ways without getting hurt.\nSo, let's just get that out of the way. None of these punches will end up hurting you any more or less than any of the others, so long as you're hitting a heavy bag and conditioning your body to punch with those punches.\nIf you do actually find that you get hurt when punching the bag even if you're going light, then it says that the punch you're doing can't be done without hurting you. So you learn not to do that punch. It's just that simple. This takes just a minute or two to figure out.\nWhat you'll notice with a half-twist punch (the vertical fist that kung-fu / wing-chun likes) is that it requires you keep your elbows in and pointed down. The force you generate is with your legs rather than just punching with the arm. There's very little biceps or triceps involved in the punch. And it feels like you're driving a wedge down the center line of the body. People that use it will just keep walking forward and use the force of stepping itself to attack. Since you're not swinging it from the outside, it lends itself well to short range fighting. Martial arts which use center line theory utilize this punch as their main punch.\nBut that half-twisting punch has some problems. First, it often relies on short power, which really depends on how skilled you are at it. Beginners will have very weak punches. It's debatable, but even experts at it tend to have weaker punches than western style boxing punches would be. But power is not really the goal with half-twisting punches. Or I'll say it's not the only goal. The main focus is to deliver quick, somewhat powerful strikes down the center line in order to attack your opponent's structure. And for that, it's fine.\nThe worst part about the half-twisting punch is that it leaves you wide open for punches to your face. Your elbow and shoulder are down. You can't deflect a hook punch to your head without taking the time to move first.\nA western style boxer has this figured out by utilizing the extra, downward twisting turn at the end of the strike. Ever notice that about boxing punches? Ever wonder why they do it? It seems \"wrong\" to most people.\nRamsey Dewey did a whole video about this, which you should see:\nhttps://www.youtube.com/watch?v=NLVj7orEGcs\nHis take is that you do that corkscrew turn at the end of the punch in order to raise both your elbow and your shoulder. That provides a shield to defend against punches to your face while you're performing the punch.\nIt has nothing to do with power, but rather, with defense. Or at least, it's debatable whether you can get any extra power or lose power by doing that extra little twist at the end.\nSo, power isn't the only goal of a good punch. It's just one of many goals. If your punch leaves you wide open to punches to your head, then your punch is kind of \"weak\" in a sense, regardless of how powerful it is.\nLet's get back to the 3/4-turn punch that you mentioned from krav-maga, and which is similar in some ways to that of Okinawan karate. Now that you understand how important the shoulder and elbow are in defense, do you see now how the 3/4-turn punch might leave you more vulnerable than the horizontal punch or the corkscrew punch would?\nIn terms of biomechanics, the 3/4-turn punch does cause the bones of your forearm and your first two knuckles to align. Again, it's not super important that those bones align in terms of avoiding damage to your wrists or the bones of your hand. Punching a heavy bag will condition you to punch in any way without hurting yourself. And if the argument is that you can avoid hurting yourself even if you're not conditioned by utilizing this 3/4-turn punch, the answer is no. You will get hurt no matter what, so long as you're not conditioning yourself to punch.\nOkinawan karate also adds a dipping motion to the first two knuckles that krav-maga does not. But you can only do that motion when you're doing a karate style lunge punch down the center of your opponent through his torso, rather than punching to the face. It's a very penetrating punch.\nNow here's what I think. It's my judgment that boxing has the best understanding and application of punching of pretty much any martial art or self-defense system. They take a very dynamic and realistic approach to it. They're not just looking at power. They're looking at the entire game. I think one year in a boxing gym, going 2 to 3 times a week would give you a better understanding of punching than krav-maga would give you in decades of training. Or at the very least, you would find it complements your krav-maga style and gives you a base you can use for performing other krav-maga skills. My opinion, anyway.\nWhat I hope you walk away with after reading this is that there's a lot more to punching than just biomechanical structure. Above all else, put it to the test. See how it does in the ring and on a heavy bag. And speaking of that heavy bag, you need to be hitting it hard in order to condition your body to punch for real, or else it won't matter how technically perfect your punch is.\nHope that helps.\n", "topic": "martialarts", "url": "https://martialarts.stackexchange.com/questions/12928"}, {"image": "VQAonline_00023547.png", "question": "Why do fencers point their back foot to the side in the en garde position?", "context": "\nI would expect that given fencing's linear nature, it would be more advantageous to angle the back foot more forward to push off. Why does the foot point off to the side in the basic stance?\n", "answer": "Mainly for versatility and stability. In fencing your actions moving backwards are just as important as your actions moving forwards. A lot of people think of it as advancing on your opponent or retreating from them, but the point of those things isn't to capture or cede strip, it is to open and close distance. \nWhen you're fencing, you know your range and you know where you're most effective. When you're new, you only know these things for perfect control and balance situations. As you get better and more experienced, you also start to get a sense of where those points are during motion and in situations of imbalance. At that point, what direction you're moving matters much less than whether the space between you and your opponent is opening, closing, or static. And your opponent is trying to do the same thing. So you're both trying to manage optimum distance at the same time. Micro changes in direction and pace are really really important. \nIf your foot is turned too far forwards, while it favors forward motion, it doesn't provide any stability in terms of quick changes of direction. If your foot is turned too far backwards, you just fall over.\nPosting guidelines says \"Share your research\" - I've been a fencer since 1985, competitive on the national level since 1990. I have taught beginners for most of that time. \n", "topic": "martialarts", "url": "https://martialarts.stackexchange.com/questions/6701"}, {"image": "VQAonline_00023537.png", "question": "To what extent does height give an advantage in any type of fight?", "context": "Last month Sylvester Stallone posted this picture in Twitter with the caption: Just in case you forgot how tall Ivan Drago is. The man was a walking eclipse.\n\nSo I was thinking to what extent does height give an advantage to an individual in Boxing, Kung Fu, Takewondo or any other martial arts? For example if a 6ft 5 inch Ivan Drago would fight against a 5ft 8 inch Rocky Balboa how much advantage does the extra 7 or 8 inch gives to Ivan Drago?\n", "answer": "Height gives a considerable advantage to striking martial arts. The first and most obvious advantage is that height means you can reach out further than your opponent, meaning you can hit him before he hits you.\nBut there are other advantages that you don't immediately consider:\n\nIf you have to punch upwards towards a taller opponent's head, you don't punch quite as hard as if you were punching straight. That's because you can't put the full force of your body behind the punch. You can still generate force with your legs though.\nA taller opponent can more easily kick you in the face without needing great flexibility.\nLonger arms and legs mean that your hooks and hooking kicks have a LOT more impact. It's science.\nbeing taller means that your opponent will have a hard time getting his leg above yours. You'll see especially in Taekwondo that kicks are blocked simply by lifting up your knee. It's very frustrating trying to kick someone in the face when they can lift their knee up to your chin.\n\nSome disadvantages to being taller than your opponent:\n\ngiven the same weight class, the shorter fighter will be stronger and therefore his punches will be harder given the same technique.\nthe shorter the arm or leg, the quicker the strike.\nif you're much taller, it will be difficult to \"work the body\" of your opponent.\n\n", "topic": "martialarts", "url": "https://martialarts.stackexchange.com/questions/5087"}, {"image": "VQAonline_00023551.png", "question": "What is the name of the side-turn defense in Capoeira?", "context": "It's the motion at the 3 second to 4 second mark here where the capoeiristra pivots so that their side faces their opponent and they're sitting back a bit. It's often done before a Queixada, particularly since it works well for a partner drill where capoeiristras trade kicks, using this defense to avoid the incoming kick while setting up for the next, but I've also seen it suggested as a defense against a B\u00ean\u00e7\u00e3o or Chapa (moving back from the attack while potentially moving to the side of it) or against a more lateral attack by turning your back to it, either catching the kick along the flat of the back to reduce its force (and because the back is not a valid target in the roda, thus negating the hit) or to facilitate ducking under it.\nThe description in the above-linked article is:\n\nFrom the base position of the ginga, with one leg pointed backward, shift the chest in an orthogonal line to the opponent, turning on the two feet; bend the legs at the knee, sitting on the pelvis (a straight leg can be an easy target, and it hurts bad).\n\nWhen stepping into the movement, I've often seen it described as Entrada. When I was studying in Philadelphia, Mestre Doutor had a name for it, which I think might have rhymed with \"Queixada\" because I remember him prompting the moves and I remember the two going together, which could match with it being Entrada other than that sounds backwards since you're retreating, not entering.\nAh, and I have found one pictorial reference. It's the first two steps, here depicted as the setup for a Queixada courtesy of this site:\n\n", "answer": "I was unable to get ahold of my instructor from Philadelphia, but I did get an answer from one of his senior assistants who said that they always just taught that pivot as the wind-up for the queixada and that there was no distinct name for it.\n\nYes, it's queixada. It's strange to read the technical description. [The] esquiva is part of the queixada. The textbook doesn't do it justice. I need to see it.\n\nI have since found a YouTube video which refers to this movement as quebrada (\"ravine\"), which matches the rhyming scheme I remember. What may have been throwing me off is that it involves a more bent position than the example images I provided above.\n", "topic": "martialarts", "url": "https://martialarts.stackexchange.com/questions/7065"}, {"image": "VQAonline_00023668.png", "question": "The format and values provided by Basis Set Exchange", "context": "In this question and answers, I found that Basis Set Exchange provides information about the orbitals (e.g., the orbital exponents and contraction coefficients). For example, the following example is orbital data of the oxygen:\n\nHowever, I cannot understand the details about these values and meanings; specifically I have three questions as follows.\n\nI understood that first 6 lines show 6 orbital exponent values and 6 contraction coefficient values in 6-31; however, each line of next 3 and 1 parts has \"one\" orbital exponent value and \"two\" contraction coefficient values. Why does one orbital have two coefficients?\n\nI believe that the oxygen orbital has 4 kinds of orbitals (i.e., 2s, 2px, 2py, and 2pz), but I cannot understand which value corresponds to 2s and which value corresponds to 2px.\n\n\nPostscript using a concrete example.\n1. They are Pople-type basis set. Namely, s and p have the same exponents but different contraction coefficients. Hence, one set of exponents, two sets of coefficients. (Maybe format Gaussian somehow looks clearer) 2. Px/Py/Pz have the same coef and exp.\nThank you for this comment from Lancashire3000. I add the following example for describing my confusing. I believe that \"31\" of 2px in the 6-31G basis set can be written (but omit the spherical harmonics) as:\n$\\phi_{2px} = \\sum_{i=1}^3 d_i \\exp(-\\zeta_i r^2) + d_4 \\exp(-\\zeta_4 r^2)$.\nSo, if px, py, pz have the same exp and coef, totally we have four exponents, $\\zeta_1, \\zeta_2, \\zeta_3, \\zeta_4$, and four coefficients, $d_1, d_2, d_3, d_4$. But the Basis Set Exchange file provides two contraction coefficients in each one; **that is, we seem to have $d_1, d'_1, d_2, d'_2, d_3, d'_3, d_4, d'_4$.\n$d$ is for the s orbital and $d'$ is for the px, py, pz orbitals?\n", "answer": "\n\"I understood that first 6 lines show 6 orbital exponent values and 6 contraction coefficient values in 6-31; however, each line of next 3 and 1 parts has \"one\" orbital exponent value and \"two\" contraction coefficient values. Why does one orbital have two coefficients?\"\n\nWhen it becomes confusing to figure out what's going on in a basis set, I find it useful to look at the same basis set in other formats.\nThe CFOUR / ACESII / MRCC format for that same basis set looks like this:\nO:6-31G(d,p)\n6-31G + polarization on all atoms\n\n 3\n 0 1 2\n 3 2 1\n 10 4 1\n\n0.5484671660D+04 0.8252349460D+03 0.1880469580D+03 0.5296450000D+02 0.1689757040D+02 \n0.1553961625D+02 0.5799635340D+01 0.3599933586D+01 0.1013761750D+01 0.2700058226D+00 \n\n0.1831074430D-02 0.0000000000D+00 0.00000000 \n0.1395017220D-01 0.0000000000D+00 0.00000000 \n0.6844507810D-01 0.0000000000D+00 0.00000000 \n0.2327143360D+00 0.0000000000D+00 0.00000000 \n0.4701928980D+00 0.0000000000D+00 0.00000000 \n0.0000000000D+00 -0.1107775495D+00 0.00000000 \n0.3585208530D+00 0.0000000000D+00 0.00000000 \n0.0000000000D+00 -0.1480262627D+00 0.00000000 \n0.0000000000D+00 0.1130767015D+01 0.00000000 \n0.0000000000D+00 0.0000000000D+00 0.1000000000D+01 \n\n0.1553961625D+02 0.3599933586D+01 0.1013761750D+01 0.2700058226D+00 \n\n0.7087426823D-01 0.00000000 \n0.3397528391D+00 0.00000000 \n0.7271585773D+00 0.00000000 \n0.0000000000D+00 0.1000000000D+01 \n\n0.8000000000D+00 \n\n1.0000000 \n\nYou can see here that all four of the P-type functions have the same exponents as the L-type functions in your version, and both versions show two \"contraction coefficients\" for each of these exponents, but in my version one contraction coefficient for each of these exponents is always zero, meaning that there's only ever one non-zero contraction coefficient for each exponent here.\n\n\"I believe that the oxygen orbital has 4 kinds of orbitals (i.e., 2s, 2px, 2py, and 2pz), but I cannot understand which value corresponds to 2s and which value corresponds to 2px.\"\n\nNotice that the first column says \"S\", \"L\", and \"D\". The \"L\" in your case represents \"SP\" which just means that there's an S-type function with the given parameters, and a P-type function with the same parameters. In my version of the basis set, I've explained how to find the S-type and P-type functions in my answer to your recent post here: The orbital exponent in Gaussian. Examine the following part:\n 0 1 2\n 3 2 1\n 10 4 1\n\nThis means that:\n\nFor S-type functions (L=0), we have 10 exponents and 3 contractions.\nFor P-type functions (L=1), we have 4 exponents and 2 contractions, and\nFor D-type functions (L=2), we have 1 exponent and 1 contraction.\n\nYou are then given the S-type exponents followed by the S-type contraction coefficients then the P-type exponents (0.1553961625D+02 0.3599933586D+01 0.1013761750D+01 0.2700058226D+00) followed by their contraction coefficients, etc. You can then match these four P-type exponents and their corresponding contraction coefficients with the ones in your version of the basis set.\n", "topic": "materials", "url": "https://materials.stackexchange.com/questions/8294"}, {"image": "VQAonline_00023639.png", "question": "What is the meaning of element-rich environment in calculating formation energy?", "context": "I want to use the formation energy formula used by the article in the picture below. The authors have used the term S or Sn-rich environment. What is the meaning of this? How it is related to the calculation of formation energy?\n\nReference paper : https://doi.org/10.1063/1.5022151\n", "answer": "How rich the environment is in $\\ce{Sn}$ vs $\\ce{S}$ affects the chemical potential $\\mu_\\ce{Sn}$ as described in Section IIIA of the linked paper:\n\n... $\\mu_\\ce{Sn}$ for the Sn-rich environment is equal to $E_\\ce{Sn}$, where $E_\\ce{Sn}$ is the total energy of Sn per atom in the diamond crystal structure.\n... $\\mu_\\ce{Sn}$ can be calculated for the S-rich\nenvironment as $\\mu_\\ce{Sn}= E_\\ce{SnS}-E_\\ce{S}$.\n\n(Note $E_\\ce{SnS}$ and $E_\\ce{S}$ in the quote above are the energy per formula unit and atom respectively).\nAs your image shows, the formation energy depends on $\\mu_\\ce{Sn}$, so it also depends on how rich the environment is in $\\ce{Sn}$ vs $\\ce{S}$.\nRichness here is just referring to how abundant a given element is in the surroundings. This won't change the simulations of the monolayer, but it does affect the formation energy. This is because in an Sn-rich environment, you are not just producing a lone Sn atom when you remove it from SnS. Rather, you are implicitly incorporating it into bulk metallic Sn, which has different energetics. For an S-rich environment, they assume the Sn is instead producing a site of SnS at the expense of a site from bulk S. So depending on the environment, the overall formation process is different.\nIf you aren't concerned about the environment effects, you could use the same definition for the chemical potential as was used for the doping transition metal (e.g. the energy of one isolated atom). They actually do this earlier in the linked paper, where they compute vacancy formation energies for SnS either using the environment affected chemical potentials or just the energy of a lone atom.\n", "topic": "materials", "url": "https://materials.stackexchange.com/questions/4730"}, {"image": "VQAonline_00023655.png", "question": "Define a geometry region using 'for' loop in MuMax3", "context": "I was trying to define regions using a 'for' loop.\nPlease find the regions:\ndefregion(0, xrange(-1024e-9, -964e-9))\ndefregion(1, xrange(-964e-9, -844e-9))\ndefregion(2, xrange(-844e-9, -784e-9))\ndefregion(3, xrange(-784e-9, -664e-9))\ndefregion(4, xrange(-664e-9, -604e-9))\ndefregion(5, xrange(-604e-9, -484e-9))\n...\n\nHere, the periodicity is w1=60e-9, and w2=120e-9.\nSo I was trying to define the regions using a loop like in the following way:\n l=1024e-9\n w1=60e-9\n w2=120e-9\n\n for i := 0; i <= 5; i+=2 {\n defregion(i, xrange(-l+i*w2, -l+(i+1)*w1));\n for j:=i+1; j<= 5; j+=1{\n defregion(j, xrange(-l+j*w1, -l+(j+1)*w2))\n }\n }\n\nIn this case, the software forms the geometry only with the w1 value. The w2 value is not taken.\nHowever, the required geometry has not been formed.\nCould anyone please let me know where I am wrong and how to solve this problem?\nI have tried with the following solution (got it from the first answer!)\nl=1024e-9\nw1=60e-9\nw2=120e-9\n\nfor i := 0; i <= 5; i+=2 {\n defregion(i, xrange(-l+i*w2, -l+(i+1)*w1));\n for j:=i+1; j<= 5; j+=1{\n defregion(j, xrange(-l+j*w1, -l+(j+1)*w2))\n }\n}\n\nAnd, the error looks like this:\n\n", "answer": "I have not used MuMax3 before, so I may be missing something, but I think you are overcomplicating things by trying to use a nested loop. It looks like you define a region in one loop and then redefine in another iteration, so its tough to reason out what values you will get.\nRunning the equivalent of your loops in Python, I get\n0 (-1.024e-06, -9.64e-07)\n1 (-9.64e-07, -7.84e-07)\n2 (-7.84e-07, -8.440000000000001e-07)\n3 (-8.440000000000001e-07, -5.440000000000001e-07)\n4 (-5.440000000000001e-07, -7.240000000000001e-07)\n5 (-7.240000000000001e-07, -3.040000000000002e-07)\n\nI believe you could instead use a single loop to get the result you want.\nl:=1024e-9\nw1:=60e-9\nw2:=120e-9\n\nstart:=0\nstop:=0\nswap:=0\nfor i := 0; i <= 5; i+=1 {\n stop=stop+w1\n defregion(i, xrange(-l+start, -l+stop));\n swap=w1\n w1=w2\n w2=swap\n start=start+w2\n}\n\nThe start and stop store how much you should add to the initial value to get the next interval. At each iteration, w1 and w2 are swapped so that the beginning and end of each region increase by the correct amount, which alternates.\nInitially, my answer used = everywhere rather than :=. It seems, based on the examples I could find on the MuMax3 page, that = is used to set options for the program or redefine existing variables while := is used for user created variable assignment.\n", "topic": "materials", "url": "https://materials.stackexchange.com/questions/6502"}, {"image": "VQAonline_00023678.png", "question": "Basics of numerical energy minimization techniques used in molecular dynamics?", "context": "The question below describes my plan to make a basic molecular dynamics calculation using a Python script rather than a canned, self-contained program.\nThere seems to be three parts:\n\na model of the honeycomb net and substrate atoms\nan expression for energy based on atomic positions\nan energy minimization procedure (the topic of this question)\n\nThe two procedures I can imagine implementing are\n\nMonte-carlo method jiggle the positions randomly using some pseudo-temperature parameter, keep the new positions if energy goes down and flip a coin about keeping the new positions if higher based on *how much higher) it is.\nKinematically using a damped differential equation and a standard ODE solver for all atomic positions.\n\nThese are general tools that I'm aware of and know how to implement in general and I can start with these no problem. But are these the numerical techniques that proper classical molecular dynamical simulations use, or are there different and/or better ways?\nReferences:\n\nDIY molecular dynamics for Xenes on crystal surfaces; where can I get applicable open-source force field parameters that I can use in my scripts?\n\n\nMy nascent DIY model, from here (click for larger)\n\n", "answer": "AFAIK most MD codes do something like conjugate gradients or BFGS for energy minimization. Your option 1 is Metropolis Monte Carlo, while option 2 sounds like the FIRE algorithm.\nHowever, why implement something yourself, when there is a multitude of solvers already available in e.g. scipy? See https://docs.scipy.org/doc/scipy/reference/generated/scipy.optimize.minimize.html, for example.\nYou could just interface to these routines and get a lot of functionality \"for free\"?\n", "topic": "materials", "url": "https://materials.stackexchange.com/questions/8760"}, {"image": "VQAonline_00023598.png", "question": "Fermi level change in surfaces with adsorbed molecules", "context": "In a DFT standard calculation of a solid 3D material (i.e. bulk metal), one of the properties that can be obtained is the Fermi level $E_F$. This feature is related with the required work to add an electron to this material. \nIn organic electronics, one possible utility of that value is to extract information about the charge-transfer properties of such material when it is contacted by a single molecule or even a molecular layer (organic semiconductor), thus obtaining the interfacial properties of the complete junction. As the orbitals of both entities tend to hybridize, the properties of the modified surface can be radically different from those of the initial isolated materials. This is particularly critical when either or both of the materials are magnetic, in which case it is called Spinterface.\n\nIn this context, some question arises. Which is the most adequate way of calculating a molecular surface on top of a metal? How to account on rearrangement of atomic positions, image-charge properties and orbital hybridization? and last but not least, How to model a descriptive molecular surface in which different molecules can be arranged in different positions?\n", "answer": "The work function is surface-dependent, and sensitive to surface reconstructions and passivation, amongst other things. The Fermi level of the infinite bulk material would be a rather crude proxy for the workfunction.\nYou also need to be aware that the Fermi level as computed in most periodic DFT codes is calculated with respect to an arbitrary pseudopotential-dependent reference energy, not a \"zero vacuum level\", so you would need to correct for this. Essentially, the DFT eigenenergies (and hence the Fermi energy) are computed with respect to the mean electrostatic potential in the simulation cell, and this includes a contribution from the local part of the pseudopotential. Adding a molecule to the surface in a periodic simulation is actually adding an infinite, periodic array of molecules and this changes the mean electrostatic potential (in addition to any actual physical or chemical phenomena).\n", "topic": "materials", "url": "https://materials.stackexchange.com/questions/428"}, {"image": "VQAonline_00023647.png", "question": "SCF Calculation not converging after two decimal places in Quantum ESPRESSO", "context": "I have been trying to do geometrical optimization for ZnSe Quantum Dots using Quantum ESPRESSO.\nThe convergence is very slow, so slow that even after 200 iterations I could not get convergence. I tried reducing mixing_beta to 0.3 like it says in the manual for QE(things got worse) and then I tried raising it to 0.8, this actually gave better results. Raising the cutoff energy also did not have much effect.\nThe weird thing is that once one decimal place is obtained it just keeps going a bit up and down not being able to converge the next digits easily. I think this could be solved by reducing the step size in the algorithm, is that somehow possible?\nConvergence graph(energy)\n\nInput file\n &CONTROL\n calculation = \"relax\"\n forc_conv_thr = 1.00000e-03\n max_seconds = 1.72800e+05\n nstep = 100\n outdir = \".\\outdir\"\n prefix = \"me\"\n pseudo_dir = \"C:\\Users\\aman\\.burai\\.pseudopot\"\n/\n\n&SYSTEM\n a = 2.50000e+01\n b = 2.50000e+01\n c = 2.50000e+01\n degauss = 1.00000e-02\n ecutrho = 2.49895e+02\n ecutwfc = 2.75032e+01\n ibrav = 8\n nat = 44\n ntyp = 2\n occupations = \"fixed\"\n smearing = \"gaussian\"\n/\n\n&ELECTRONS\n conv_thr = 1.00000e-06\n electron_maxstep = 200\n mixing_beta = 8.00000e-01\n startingpot = \"atomic\"\n startingwfc = \"atomic+random\"\n/\n\n&IONS\n ion_dynamics = \"bfgs\"\n/\n\n&CELL\n/\n\nK_POINTS {gamma}\n\nATOMIC_SPECIES\nZn 65.39000 Zn.pbe-van.UPF\nSe 78.96000 Se.pbe-n-rrkjus_psl.1.0.0.UPF\n\nATOMIC_POSITIONS {angstrom}\nZn 7.349204 5.983991 13.367822\nZn 11.402731 13.004906 13.367822\nZn 7.349204 5.983991 6.705404\nZn 11.402731 13.004906 6.705404\nZn 13.429495 11.834754 10.036613\nZn 9.375968 4.813839 10.036613\nZn 5.322440 9.494449 13.367822\nZn 9.375967 9.494448 13.367822\nZn 7.349204 13.004906 13.367822\nZn 5.322440 9.494449 6.705404\nZn 9.375967 9.494448 6.705404\nZn 7.349204 13.004906 6.705404\nZn 15.456259 8.324296 10.036613\nZn 11.402731 8.324296 10.036613\nZn 13.429494 4.813839 10.036613\nZn 13.429495 9.494449 13.367822\nZn 11.402731 5.983991 13.367822\nZn 13.429495 9.494449 6.705404\nZn 11.402731 5.983991 6.705404\nZn 5.322440 11.834754 10.036613\nZn 7.349204 8.324296 10.036613\nZn 9.375968 11.834754 10.036613\nSe 13.429495 11.834754 5.868711\nSe 9.375968 4.813839 5.868711\nSe 7.349204 5.983991 9.199920\nSe 11.402731 13.004906 9.199920\nSe 13.429495 11.834754 12.531128\nSe 9.375968 4.813839 12.531128\nSe 11.402731 8.324296 5.868711\nSe 9.375967 9.494448 15.862337\nSe 5.322440 9.494449 9.199920\nSe 9.375967 9.494448 9.199920\nSe 7.349204 13.004906 9.199920\nSe 15.456259 8.324296 12.531128\nSe 11.402731 8.324296 12.531128\nSe 13.429494 4.813839 12.531128\nSe 7.349204 8.324296 5.868711\nSe 9.375968 11.834754 5.868711\nSe 15.456259 5.983991 9.199920\nSe 13.429495 9.494449 9.199920\nSe 11.402731 5.983991 9.199920\nSe 5.322440 11.834754 12.531128\nSe 7.349204 8.324296 12.531128\nSe 9.375968 11.834754 12.531128\n\nAny other advice on convergence is also very much optimized, Thanks!\nP.S the structure is neutral and I don't think there is anything wrong with the structure.\n", "answer": "My suggestion is that you first get an SCF calculation to converge for this structure before you try relaxing, especially if you aren't sure what you're going to get. Then you can proceed to converging the numerical settings like the cutoff energies for your properties of interest.\nI was able to get an SCF calculation to converge using your input file by switching the occupations = \"smearing\" instead of \"fixed\". It seems like this may have been what you intended based on your choices for \"smearing\" and \"degauss\", which seem to be ignored if you have occupations = \"fixed\".\n", "topic": "materials", "url": "https://materials.stackexchange.com/questions/6209"}, {"image": "VQAonline_00023648.png", "question": "How to use wavefunctions/density to determine which orbitals lead to edge states?", "context": "I have a large matrix for a 1D zigzag edge model of an otherwise $3\\times 3$ tight-binding Hamiltonian (3 basis functions, each corresponding to an atomic orbital), involving the variable $k_x$. The large matrix contains blocks of this $3\\times 3$ Hamiltonian for each unit cell involved (I chose 100 unit cells).\nFor different values of $k_x$, I numerically calculate eigenvalues and eigenvectors, and use the eigenvalues to plot the band structure shown below:\n\nHow do I identify which of the three orbitals the edge states are coming from, by using the numerical wavefunctions or otherwise?\nWhat I've done so far: I used the following (HIGHLY INEFFICIENT) Python code to identify the indices of eigenvalues where the bands approximately cross. I was hoping to use this to find the eigenvectors corresponding to the point, and then identify which orbitals the edge states are coming from. My rationale is that the probability will suddenly jump to something else (or switch between bands) when the bands cross. I guess my biggest confusion is that I do not know which eigenvectors to choose, nor what they mean. That is, if my original matrix is tridiagonal, with $3\\times 3$ blocks making up the system, what bases do the resulting eigenvectors have? In the case of energy eigenstates in just momentum space, I understand that each eigenvector corresponds to an energy wavefunction. But now I have the added complication of several unit cells.\nfind_evec_with_energy = 1; # crossing occurs around here\ntol = 0.02\nrows,cols = np.asarray(energies).shape # for each iteration of kx, I append the eigenvalues into the list called 'energies'\n\nindices = []\nfor j in range(rows):\n for k in range(cols):\n if np.abs(np.asarray(energies)[j,k]-find_evec_with_energy)0$, there is a result which says that:\n$$\n|\\Gamma(ix)| \\ = \\ \\frac{\\sqrt{\\pi}}{\\sqrt{x\\sinh(\\pi x)}}\n$$\nSo this means the following:\n$$\n\\Gamma(ix) \\ = \\ \\frac{\\sqrt{\\pi}}{\\sqrt{x\\sinh(\\pi x)}} e^{i \\mathrm{Arg}\\left[ \\Gamma(ix) \\right]}\n$$\nIs there any way to determine $\\mathrm{Arg}\\left[ \\Gamma(ix) \\right]$? What properties does this function have? I've included a plot of the function, and there are a bunch of discontinuities for this function.\n\n", "answer": "$\\newcommand{\\bbx}[1]{\\,\\bbox[15px,border:1px groove navy]{\\displaystyle{#1}}\\,}\n \\newcommand{\\braces}[1]{\\left\\lbrace\\,{#1}\\,\\right\\rbrace}\n \\newcommand{\\bracks}[1]{\\left\\lbrack\\,{#1}\\,\\right\\rbrack}\n \\newcommand{\\dd}{\\mathrm{d}}\n \\newcommand{\\ds}[1]{\\displaystyle{#1}}\n \\newcommand{\\expo}[1]{\\,\\mathrm{e}^{#1}\\,}\n \\newcommand{\\ic}{\\mathrm{i}}\n \\newcommand{\\mc}[1]{\\mathcal{#1}}\n \\newcommand{\\mrm}[1]{\\mathrm{#1}}\n \\newcommand{\\pars}[1]{\\left(\\,{#1}\\,\\right)}\n \\newcommand{\\partiald}[3][]{\\frac{\\partial^{#1} #2}{\\partial #3^{#1}}}\n \\newcommand{\\root}[2][]{\\,\\sqrt[#1]{\\,{#2}\\,}\\,}\n \\newcommand{\\totald}[3][]{\\frac{\\mathrm{d}^{#1} #2}{\\mathrm{d} #3^{#1}}}\n \\newcommand{\\verts}[1]{\\left\\vert\\,{#1}\\,\\right\\vert}$\n\\begin{align}\n\\Gamma\\pars{\\ic x} & =\n{\\Gamma\\pars{1 + \\ic x} \\over \\ic x} =\n-\\,{\\ic \\over x}\\int_{0}^{\\infty}t^{\\ic x}\\expo{-t}\\dd t =\n-\\,{\\ic \\over x}\\int_{0}^{\\infty}\\expo{\\ic x\\ln\\pars{t}}\\expo{-t}\\dd t\n\\\\[5mm] & =\n-\\,{\\ic \\over x}\\int_{0}^{\\infty}\n\\bracks{\\cos\\pars{x\\ln\\pars{t}} + \\ic\\sin\\pars{x\\ln\\pars{t}}}\\expo{-t}\\dd t\n\\end{align}\n\n$$\n\\bbx{\\arg\\pars{\\Gamma\\pars{\\ic x}} =\n-\\arctan\\pars{\\ds{\\int_{0}^{\\infty}\n\\cos\\pars{x\\ln\\pars{t}}\\expo{-t}\\dd t} \\over\n\\ds{\\int_{0}^{\\infty}\n\\sin\\pars{x\\ln\\pars{t}}\\expo{-t}\\dd t}}}\n$$\n", "topic": "math", "url": "https://math.stackexchange.com/questions/2716308"}, {"image": "VQAonline_00035297.png", "question": "Expectation of Uniform Distributions", "context": "Given $X = \\operatorname{Uniform}(0, 4)$ distribution where the distribution is continuous. \nUsing the definition for $\\operatorname E(X)$\n\\begin{align}\n& \\operatorname E(X) \\\\[10pt]\n= {} & \\int_{-\\infty}^\\infty xP(X\\in dx) \\\\[10pt]\n= {} & \\int_{-\\infty}^\\infty xf(x)\\,dx \\\\[10pt]\n= {} & \\int_{-\\infty}^\\infty x\\frac 1 {4-0} \\, dx \\\\[10pt]\n= {} & \\int_{-\\infty}^\\infty \\frac 1 4 x \\, dx \\\\[10pt]\n= {} & \\text{undefined?}\n\\end{align}\nBut the textbook is like $E(X)=\\frac{a+b}{2} = \\frac{0+4}{2}$ ?\nDefinition of $E(X)$ for continous distributions:\n\n", "answer": "The density is $f(x) = \\begin{cases} \\dfrac 1 4 & \\text{if } 0 Q), with incoming edges\n representing known assertions (P) and outgoing edges representing\n deduced conclusions (Q). Thus Hallerberg, like Berger, Schacht, and\n Shields, makes the law of modus ponens the central feature of his\n format. Hallerberg's flow proofs lack the readability of Thorsen's\n and Ness's, largely because his diagrams consist solely of code labels\n ($S_n$, $G_k$, etc.) with a \"legend\" alongside the diagram decoding the\n symbols. The result might please a logician but is manifestly\n classroom \u2013unfriendly.\nTwo remarkable features of Hallerberg's\n article are that he makes no claim that this has worked well in the\n classroom (I believe his article is unique in not making this claim),\n and that he offers examples of non-geometric proofs as well. He also\n addresses using the flow proof with indirect proofs. \nFlow proofs\n are rediscovered once again in McMurray (1978). It is here that the\n name \"flow proof\" is first (?) used. McMurray's flow proofs consist\n of statements, joined by arrows; each arrow is labelled with a number,\n and under the flow proof the numbers are associated with \"reasons\". \n As in Hallerberg, McMurray stresses that the law of modus ponens\n (which he calls the \"law of detachment\") is made manifest through this\n representation. \nBasinger (1979) is a response to McMurray (1978). \n Basinger reports that he has also used flow proofs, with a few key\n superficial differences in notation. Interestingly, Basinger writes\n \"The flow proof format does seem to show graphically the 'flow of\n logic' in complicated proofs. But overemphasis of this proof format\n may be just as bad as over-emphasis on the traditional formats... It\n may be better to use several proof formats, with near-equal emphasis\n on each of them... And certainly the paragraph proof should not be\n left out.\"\nMy final paper in this section is Brandell (1994). Despite its title (\"Helping \n students write paragraph proofs in\n geometry\") the paper has much more to say about flow proofs than about\n paragraph proofs. In fact, Brandell's approach to teaching paragraph\n proofs seems to be: (1) First, make a flow proof. (2) Then, turn\n the flow proof into sentences and paragraphs. This approach seems\n amazingly backwards to me. The whole point of paragraph proof is that\n it's less rigorously formal, more like natural language. (Isn't it?) \n To teach paragraph proof as just a highly-condensed form of flow proof\n is just weird. Brandell's flow proofs have statements in boxes,\n linked by arrows, with the reasons labelling the arrows. \n Interestingly, he suggests as a learning tool that unnecessary\n statements could be included in the flow diagram; when the paragraph\n is finally written, those extraneous statements are jettisoned.\n\n", "topic": "matheducators", "url": "https://matheducators.stackexchange.com/questions/16989"}, {"image": "VQAonline_00043888.png", "question": "Is induction or recursion easier to understand?", "context": "This is not really a new question, more a revisiting of @vonbrand's \"Any suggestions on how to approach recursion and induction?\" In an introductory programming class this past year, I asked the students to create a Mondrian-like image\nvia recursion.\nHere is a typical submission:\n\n\u00a0 \u00a0 \u00a0 \u00a0\n\n\nThese 1st-yr college students had little difficulty understanding\nthe recursion necessary to produce such images. Most of them had\nlittle beyond US high-school mathematics\u2014maybe calculus, no discrete math, no proofs.\nAnd yet at the same time I taught more advanced students induction in the\ntheory of computation, and it was a mental hurdle for most of them.\nDespite the informative answers to \"Why are induction proofs so challenging for students?,\" I am still\nuncertain. The Mondrian experience has made me wonder if it might not be\nbetter pedagogically to introduce recursion first, nail that down thoroughly, and then teach\ninduction\u2014the apparently more difficult concept\u2014by connecting to \nand building upon their\nunderstanding of recursion. This may work even for students who have no programming experience, and are taught recursion \"theoretically.\"\n", "answer": "One thing that you have to keep in mind here, is that you don't need to understand recursion to implement it. There is a big difference between \"we were taught to do it like that, I implement it and it works\" and really understanding the concepts and why it works as it does. With induction, on the other hand, you are supposed to write down a complete, formal proof, so you need at least better understanding of the concepts, if possible you should understand completely why it works.\nRegarding your question if you should teach recursion or induction first, I would suggest this order:\n\nTeach recursion and let the students program recursive functions.\nAfter the students saw that it worked, focus on the theoretical concepts. For example give them a short recursive program and ask them to prove what it computes on a given input. Or go back to the problems programmed before and ask the students to prove that they really do what they are supposed to.\nShow the students that for such proofs, it is easier to build them up from the bottom, i.e. not showing \"on input $n$, we do this and that and use the function on $n-1$. Here, on input $n-1$, we do...\" but rather starting with $n=1$.\nLet them train induction and recursion parallel. For example assume that the students should show\n$$\\sum_{i=1}^n i = \\frac{n(n+1)}{2}.$$\nLet them write a recursive program, computing the left hand side (that should be a rather easy program...), then apply the methods learned above to show that this program actually computes the right hand side.\nGradually drop the programing tasks and focus more on the proofs.\n\nIn this way, the students don't just get induction as a strange, abstract concept, they have a practical component in learning it. They can learn that a proof is not something strange, it's just a formalized way to show something (in this case: \"my program does what I claim it does\"), show them that this formalism is not something to fear but is rather making it easier to show the correctness of their recursive programs and statements. In this way, you might actually be able to get them to understand that a formal proof is something good; this knowledge will be useful far beyond your course on induction.\nDisclaimer: I am assuming an optimal class. In practice, some of these things might not work so well due to time limits or unmotivated students.\n", "topic": "matheducators", "url": "https://matheducators.stackexchange.com/questions/12421"}, {"image": "VQAonline_00043875.png", "question": "Explanation for cutting a M\u00f6bius strip at one-third its width", "context": "Can anyone offer a concise, convincing explanation for why cutting\na M\u00f6bius strip along a line, not midway but rather one-third of the width of\nthe strip, and eventually joining back to itself, produces two linked, twisted\nloops, one long, one short:\n\n\n\n(Image from this web site.)\n\nI am not seeking a proof, but rather an explanation that could convince\nessentially anyone paying sufficient attention.\nEspecially the lack of symmetry in the result can be surprising.\nConsider it a teaching challenge. :-)\n", "answer": "The middle third is obtained by trimming the edges off the original m\u00f6bius loop. It is therefore simply a thinner m\u00f6bius loop (the short loop).\nThe outside thirds of the m\u00f6bius loop are obtained by cutting the loop in half and trimming 1/3 off the edge that was not originally the outside edge. It is the same as cutting the strip in half: as the outside edges are linked by a twist, the edge is a single loop twice as long as the original.\nImagining that a wire runs along the centre of the original m\u00f6bius loop, one can follow the path of the outside edge as you trace along the wire. The edge slowly twists around the inside loop, so that after following the wire for 2 rotations, the edge has made a complete loop around the centre of the M\u00f6bius strip, going through the middle of the wire loop. As the wire becomes the small M\u00f6bius strip, and edge becomes the long M\u00f6bius strip, the long strip loops itself once around the small strip. \n", "topic": "matheducators", "url": "https://matheducators.stackexchange.com/questions/7392"}, {"image": "VQAonline_00044467.png", "question": "Extending a map from $S^n\\to M^n$ to a nice map from $B^{n+1}\\to M^n$", "context": "Let $S^n$ and $B^{n+1}$ be the unit sphere and unit ball in $\\mathbb{R}^{n+1}$, and let $M^n$ be a contractible space of dimension $n$.\nIf necessary, assume that $M^n$ is a contractible simplicial $n$-complex.\nLet $\\sigma\\colon S^n\\to M^n$ be a continuous map.\nQuestion\nDoes there exist a continuous map $\\beta\\colon B^{n+1}\\to M^n$ such that for any $b\\in B^{n+1}$, the point $b\\in\\mathbb{R}^{n+1}$ lies in the convex hull of the preimage $\\sigma^{-1}(\\beta(b))$?\nEdit\nThe following stronger statement is false already if $n=1$.\nThere exists a continuous map $\\beta\\colon B^{n+1}\\to M^n$ and an involution $f$ on $S^n$ with $\\sigma=\\sigma\\circ f$, such that for any $b\\in B^{n+1}$ there exists some $x\\in S^n$ such that $\\sigma(x)=\\beta(b)$, and $b$ lies on the line segment between $x$ and $f(x)$ in $\\mathbb{R}^{n+1}$.\nThe figure below illustrates a counter example to this stronger claim.\n$Y$ is a star graph with three edges, and $\\sigma$ is indicated by the dashed lines. None of the points interior to the white triangle lie between any pair of points that map to the same point on $Y$.\n\n", "answer": "The answer is no, essentially since higher homotopy groups of spheres are nontrivial.\nFor example, in the $n=3$ case let $M=B^3$ and let $\\sigma\\colon S^3 \\to M$ be the Hopf map from $S^3$ to the boundary sphere $\\partial M$ of $M$. Viewing $S^3$ as the unit sphere in $\\mathbb{C}^2$, the group $U(1)$ of unit complex numbers acts on $S^3$ by scalar multiplication, and the preimage of each point in $\\partial M$ is a great circle in $S^3$ which is an orbit under the action of $U(1)$.\nNow suppose $\\beta\\colon B^4\\to M$ is any map satisfying the given conditions. Note that $\\sigma^{-1}(\\beta(b))$ will be empty if $\\beta(b)\\notin \\partial M$, so it must be the case that $\\beta(b)\\in \\partial M$ for all $b\\in B^4$, i.e. $\\beta$ maps $B^4$ to $S^2=\\partial M$. Note that each $\\sigma^{-1}(\\beta(b))$ is one of the circles described above, and the associated convex hull is a disk through the origin.\nNow, if $b$ is any point on the sphere $S^3\\subset B^4$, then $b$ lies in the convex hull of $\\sigma^{-1}(\\beta(b))$ if and only if $b$ lies on the circle $\\sigma^{-1}(\\beta(b))$, which occurs if and only if $\\beta(b)=\\sigma(b)$. Thus $\\beta$ restricts to $\\sigma$ on the boundary sphere $S^3$. But this is impossible since $\\beta$ maps $B^4$ into the $2$-sphere $\\partial M$ and the Hopf map is not nullhomotopic.\n", "topic": "mathoverflow.net", "url": "https://mathoverflow.net.stackexchange.com/questions/344039"}, {"image": "VQAonline_00044408.png", "question": "Is it a new method to construction of a conic, how can prove?", "context": "There are some methods to construct a conic, example: Based on Pascal theorem, Steiner construction, .....I propose a method to construct a conic as follows:\n\nLet $L_1, L_2$ be two parallel lines, let $A, B, C, D$ be four points in the plane. Let $E$ be a point lie on the line $L_1$, $F$ be the point lie on line $L_2$ such that $EF \\parallel AB$. Let circle $(E, ED)$ meets the circle $(F, FC)$ at two points $H$, $G$.\nMy question: I am looking for a proof that locus of $H, G$ is a conic section when $E$ be moved on line $L_1$.\n\n\nSee also:\n\nA chain of six circles associated with a conic\n\n", "answer": "Without loss of generality, let $L_1$ be the x-axis and $t$ be a parameter. Denote $C,D,E,F$ by $$C(x_1,y_1),D(x_2,y_2),E(t,0),F(t+a,b)$$ such that the vector $$ is in the same direction as $\\overrightarrow{AB}$ and $x_1,y_1,x_2,y_2,a,b$ are constants. Then the locus of $H$ (or $G$) satisfies $$\\frac{PE}{ED}=\\frac{PF}{FC}=1,$$ where $P(x,y)$ represents $H$ (or $G$). In terms of equations, one has $$(x-t)^2+y^2=(x_2-t)^2+y_2^2\\qquad (1)$$ and $$(x-t-a)^2+(y-b)^2=(x_1-t-a)^2+(y_1-b)^2\\qquad (2)$$ Then from (1), one gets $$t=\\frac{x^2+y^2-x_2^2-y_2^2}{2(x-x_2)}\\qquad (3)$$ Subtracting (2) from (1) gives $$(2x-2t-a)(a)+(2y-b)(b)=(x_1+x_2-2t-a)(x_2-x_1+a)+(y_2+y_1-b)(y_2-y_1+b)\\quad (4)$$ Now substituting (3) into (4) and clearing denominator, one gets a quadratic equation in $x$ and $y$, so it gives an equation for a conic.\n", "topic": "mathoverflow.net", "url": "https://mathoverflow.net.stackexchange.com/questions/316857"}, {"image": "VQAonline_00044253.png", "question": "Shape whose translated and scaled copies are closed under intersection", "context": "The translated and scaled copies of an equilateral triangle with fixed orientation are closed under intersection - the intersection is again an equilateral triangle with the same orientation.\n\nWhat other convex shapes in 2D have this property?\n", "answer": "I think triangles (degenerated ones included) are the only such convex shapes. The idea is: If $C$ is such a convex shape, let $p_i\\in\\partial C$ for $i=1,2,3$ be three smooth points. One can obtain an approximate triangle as an intersection of three large copies $C$, say $m(C-p_i)+p_i$ for large $m$. Since these intersections are similar to $C$, and converge to a triangle as $m\\to\\infty$, $C$ itself is a triangle.\n", "topic": "mathoverflow.net", "url": "https://mathoverflow.net.stackexchange.com/questions/232791"}, {"image": "VQAonline_00043975.png", "question": "Car movement - differential geometry interpretation.", "context": "I've posted this on Math Stack Exchange and I didn't get any answer in a couple of days, so I'll try and post it here too.\nThe problem presented below is from my differential geometry course. The initial reference is Nelson, Tensor Analysis 1967. The car is modelled as follows: \n\nDenote by $C(x,y)$ the center of the back wheel line, $\\theta$ the angle of the direction of the car with the horizontal direction, $\\phi$ the angle made by the front wheels with the direction of the car and $L$ the length of the car.\nThe possible movements of the car are denoted as follows:\n\nsteering: $S=\\displaystyle\\frac{\\partial}{\\partial \\phi}$;\ndrive: $D=\\displaystyle\\cos \\theta \\frac{\\partial}{\\partial x}+\\sin\\theta \\frac{\\partial}{\\partial y}+\\frac{\\tan \\phi}{L}\\frac{\\partial}{\\partial \\theta}$;\nrotation: $R=[S,D]=\\displaystyle\\frac{1}{L\\cos^2 \\phi}\\frac{\\partial }{\\partial \\theta}$;\ntranslation: $T=[R,D]=\\displaystyle\\frac{\\cos \\theta}{L\\cos^2 \\phi}\\frac{\\partial}{\\partial y}-\\frac{\\sin\\theta}{L\\cos^2\\phi}\\frac{\\partial}{\\partial x}$\n\nWhere $[X,Y]=XY-YX$ (I can't remember the English word now). All these transformations seem very logical. My question is:\n\nHow can we justify the mathematical interpretation made above, especially the part with the rotations and translations?\n\nThe interpretations are quite interesting: \n\nfrom the expression of $D$, when the car is shorter, you can change the orientation of the car very easily, but when it is longer, like a truck, you it is not that easy ( see the term with $\\frac{\\partial}{\\partial \\theta}$)\nthe rotation is faster for smaller cars, and for greater steering angle\ntranslation is easier for smaller cars.\n\n", "answer": "I'm not sure what you mean by \"justify\", but one way of looking at this is through the Chow theorem. \nYou've calculated that $R = [S, D]$. This is a statement about vector fields, which are infinitesimal flows. So if you steer by a small amount, followed by driving over a small amount, followed by turning the wheel back into the original position and driving backward, you end up performing a small rotation. You can interpret $T = [R, D]$ in the same way: if you want to parallel park the car (performing a translation), you turn ($R$), drive forward ($D$), turn back ($-R$) and drive backward ($-D$).\nA normal car has four degrees of freedom, the angles $\\theta$ and $\\phi$ on your figure and the coordinates $x, y$ of the center of mass. However, you as the driver can only be drive and steer the car, you can't just translate the car or rotate it around its axis. So you can only control $S$ and $D$. We say that this system is underactuated, meaning that you have less control inputs than there are degrees of freedom in the system.\nHowever, differential geometry comes to the rescue: if you consider the distribution spanned by $S$ and $D$, you'll find that it is totally nonholonomic, meaning that you can create any possible motion (any linear combination of $R$, $D$, $T$ and $S$) by a linear combination of $S$ and $D$ and their (iterated) commutators. Physically, this means that you can perform any motion with the car by driving and steering, although you will sometimes have to execute a nontrivial motion in $S$ and $D$ (look up \"control by sinusoids\") \nIf now you have a metric on the distribution spanned by $S$ and $D$ (telling you, for instance, how much it costs to steer resp. drive the car) you have an instance of what is called a sub-Riemannian geometry. (I hope this will entice Richard Montgomery to comment on this question).\nI can polish/extend this answer a great deal, so please feel free to ask follow-up questions.\n", "topic": "mathoverflow.net", "url": "https://mathoverflow.net.stackexchange.com/questions/66578"}, {"image": "VQAonline_00044335.png", "question": "Highest weight representations of Kac\u2014Moody algebras: what is inside the weight spaces?", "context": "Let $V(\\lambda)$ be the unique irreducible representation of a Kac\u2014Moody algebra $\\mathfrak{g}$ with the highest weight $\\lambda$. If $\\mathfrak{g}$ is not of finite type, then even for $\\lambda$ one of the fundamental weights the weights of $V(\\lambda)$ usually have multiplicities $\\geqslant2$.\n\nIs there a nice way to pick a basis in each of the weight spaces $V(\\lambda)_\\mu$?\n\nHere \"nice\" means that there is an explicit description of the action of the standard generators of $\\mathfrak{g}$, so one could actually write (infinite) matrices for the Lie algebra elements. I have checked some sources including Lie Algebras of Finite and Affine Type by R. Carter and Affine Lie Algebras, Weight Multiplicities and Branching Rules by Kass, Moody, Patera and Slansky, but none contains such details even for the smallest cases like $\\tilde{A}_1$. Here is a picture from the second:\n\nAs you can see, this diagram gives no hint to how exactly the generators act inside the weight spaces. I guess this can be reconstructed from the desription of $V(\\lambda)$ as a quotient of the Verma module, but this seems quite computationaly extensive, so I was wondering whether someone has already done it in some cases (say, for the untwisted affine KM algebras for some particular choice of $\\lambda$)?\n", "answer": "The key word you're looking for is Littelmann path model, which gives a nice geometric description for the basis of a weight space of a highest-weight representation of a symmetrisable Kac-Moody algebra (see e.g. http://people.bath.ac.uk/lpah20/GeomSemNP.pdf for an introduction), and gives a fairly explicit algebraic description of this basis (see http://www.mi.uni-koeln.de/~littelma/montreal.pdf Theorem 5.2), from which it is possible to calculate the action of the standard generators. \nWhile I don't know of a precise reference where this is written up, it is a good exercise to calculate these things for, say, $\\tilde A_1$ and $\\tilde A_2$. \n", "topic": "mathoverflow.net", "url": "https://mathoverflow.net.stackexchange.com/questions/279170"}, {"image": "VQAonline_00044175.png", "question": "Random Diophantine polynomials: Percent solvable?", "context": "Suppose one generates a random polynomial\nof degree $d$ with integer coefficients\nuniformly distributed within $[-c_\\max,c_\\max]$.\nFor example, for\n$d=8$, $|c_\\max|=100$, here is one random polynomial:\n$$\n-46 x^8-19 x^7+14 x^6-75\n x^5+94 x^4+18 x^3-48 x^2+29\n x-61=0\n\\;.\n$$\n\nQ. What is the probability that such \n a random $(d,c_\\max)$-polynomial has at least one\n integer solution?\n\nHere is a bit of data, based on $10^5$ polynomials for each $d=1,\\ldots,10$\nwith $|c_\\max|=100$:\n\n\u00a0\n\u00a0\n\u00a0\n\u00a0\n\u00a0\n\n\nI.e., For $d=1$, about $5.5$% have integer solutions,\nwhile for $d=10$, about $0.8$% have integer solutions.\nThe above displayed degree-$8$ example has the solution $x=-1$,\nand in addition these $7$ non-integer roots:\n$$\n-1.54767,\\\\ -0.0337862 \\pm 0.794431 i,\\\\ \n 0.196632 \\pm 1.19591 i,\\\\ \n 0.904467 \\pm 0.323333 i\n\\;.\n$$\nIf the constant coefficient is $0$, then of course $x=0$ is a solution.\nSo $\\frac{1}{2 c_\\max+1}$ is a lower bound,\nin the above chart, $\\frac{1}{201} \\approx 0.5$%.\n", "answer": "The case of degree $1$ has been handled by quid. This turns out to be an exceptional case, and for all degrees $d\\ge 2$ one can show that there is a constant $K(d)$ such that the number of degree $d$ polynomials with coefficient bounded by $C$ and having an integer root is \n $$ \n \\sim K(d) (2C+1)^d. \n $$ \n Moreover, for large $d$, the constant $K(d)$ is approximately equal to \n $$ \n 1+ 2 \\frac{\\sqrt{6}}{\\sqrt{\\pi d}}.\n $$ \n In this approximation, the term $1$ gives the contribution of polynomials having $0$ as a root, and the second term $\\sqrt{6/\\pi d}$ accounts for polynomials having a root at $1$ (and another similar contribution from those having a root at $-1$). For large $d$, the effect of having roots at integers larger than $1$ in size is substantially smaller. \nFor $d=10$ and $C=100$ this approximation is about $0.93\\%$ which is a little higher than your data, but for $d=9$ it is much closer (the approximation being $0.96\\%$). Perhaps the Monte-Carlo simulations haven't fully stabilized? \nClearly the number of polynomials having a root at $0$ is $(2C+1)^d$. Now suppose that $k\\ge 1$ is a positive integer, and consider polynomials having a root at $k$ (naturally the same holds for $-k$). Write $f(x)=a_dx^d+\\ldots+a_0 = (x-k) (b_{d-1}x^{d-1} + \\ldots +b_0)$. Note that $kb_0$ must lie in $[-C,C]$ giving us about $(2C+1)/k$ choices for $b_0$. Next if $b_0$ is fixed, then $-kb_1+b_0$ must lie in $[-C,C]$ giving us about $(2C+1)/k$ choices for $b_1$. Proceeding in this manner, we get at most $(2C+1)/k$ choices for each $b_j$, with the additional constraint that the final $b_{d-1}$ must also be constrained to be in $[-C,C]$. It follows that there are at most $(2C+1)^d/k^d$ possible polynomials having a root at $k$. This upper bound summed over $k$ converges for $d\\ge 2$ (but not for $d=1$), and shows that the proportion of polynomials having a large integer root is very small. Thus at any rate the number of polynomials having an integer root is at most $(1+2\\zeta(d)) (2C+1)^d$, and by similar reasoning we may see that the number of polynomials having two integer roots is at most $O((2C+1)^{d-1+\\epsilon})$. \nBy our work above, the number of polynomials having an integer root is essentially the sum of those polynomials having a root at $k$ over integers $|k|\\le K$ for some slowly growing $K$. Now for a given $k\\ge 1$, for the polynomial to have a root at $k$ means that given $a_1$, $\\ldots$, $a_d$ (all chosen in $[-C,C]$) we must have the sum $a_1k+a_2k^2+\\ldots +a_d k^d$ lying in $[-C,C]$ (which then uniquely determines $a_0$). But now we may write $a_j=Cx_j$, and then the $x_j$ behave like independent random variables chosen uniformly from $[-1,1]$ and then we are asking for the probability that $x_1 k+x_2k^2+\\ldots +x_d k^d$ also lies in $[-1,1]$. Clearly this probability must be some constant $K(d,k)$ (which by our earlier work is at most $1/k^d$), and therefore our claimed asymptotic holds with \n $$ \n K(d) = 1+ 2\\sum_{k=1}^{\\infty} K(d,k). \n $$ \nLastly we come to the approximation for $K(d)$ for large $d$. Note that the contribution of terms $k\\ge 2$ is $O(2^{-d})$ is extremely small. It remains to understand $K(d,1)$ -- the probability that the sum of $d$ independent random variables chosen uniformly from $[-1,1]$ also lies in $[-1,1]$. For large $d$, the sum of $d$ independent random variables is approximately normal with mean $0$, and variance $d/3$. From this our approximation follows. By using Parseval, one can also see that \n$$ \nK(d,1) = \\int_{-\\infty}^{\\infty} \\Big(\\frac{\\sin (\\pi x)}{\\pi x}\\Big)^{d+1} dx,\n$$ \nfrom which we can calculate, for example, that $K(10,1)=0.4109\\ldots$ which is pretty close to the approximation $0.437\\ldots$. \n", "topic": "mathoverflow.net", "url": "https://mathoverflow.net.stackexchange.com/questions/203981"}, {"image": "VQAonline_00044696.png", "question": "Special version of $\\Delta$-system Lemma for singular cardinals", "context": "In his article \"Remarks on cardinal invariants in topology\" (you can get the paper here: Where can I find the following S. Shelah's paper?), Saharon Shelah states the following claim: \n(A) is simply the classic version of the $\\Delta$-system Lemma. I have been trying to prove (B) and have not succeeded; also, I've looked in the usual set theory books to see if it's written somewhere, but I haven't had any luck here either. I can prove 2) and 3) by assuming that the cardinals $\\lambda_\\alpha$ are regular and uncountable (which is fairly reasonable since $\\lambda>cf(\\lambda)>\\omega$), but I have failed to argue that 1) is also possible.\nIn this case, any help is welcome: an argument for 1) or any reference would be great.\n", "answer": "I'll assume that you already have Clauses (B)(2)-(3). We need to thin down once more to get (1). WLOG, we can assume each $\\lambda_{\\alpha}$ is a successor cardinal strictly above $\\max(\\mu^+, \\sup_{\\beta < \\alpha} \\lambda_{\\beta})$. Put $\\mu = cf(\\lambda) > \\aleph_0$. First apply (A) to the family $\\{A_{\\alpha}: \\alpha < \\mu\\}$ to get $A$ and $X \\in [\\mu]^{\\mu}$ such that for every $\\alpha \\neq \\beta$ in $X$, $A_{\\alpha} \\cap A_{\\beta} = A$. WLOG, $X = \\mu$.\nFix $\\alpha < \\mu$. Put $W_{\\alpha} = \\bigcup \\{A_{\\beta}: \\beta < \\mu\\} \\cup \\bigcup \\{A_{\\beta, \\gamma}: \\beta < \\alpha, \\gamma < \\lambda_{\\beta}\\}$. Then $|W_{\\alpha}| < \\lambda_{\\alpha}$. Observe that for each $\\alpha < \\mu$, $\\{A_{\\alpha, \\beta} \\setminus A_{\\alpha}: \\beta < \\lambda_{\\alpha}\\}$ is a family of pairwise disjoint sets. So fewer than $\\lambda_{\\alpha}$ of these intersect $W_{\\alpha}$. Throw these away and check that (1) holds.\nEdit: I have added the union of the $A_{\\alpha}$'s to $W_{\\alpha}$. (1) should hold now.\n", "topic": "mathoverflow.net", "url": "https://mathoverflow.net.stackexchange.com/questions/419304"}, {"image": "VQAonline_00044786.png", "question": "Help setting up Bi-Xenon lights with h4 wiring", "context": "I am doing one of the oh-so controversial HID retrofits - I got myself I nice pair of projectors and bulbs (I am using cheep ballasts until I have the money for some nice OEM ones). The projectors I bought have solenoids to open up the high beam. I would like to use the lights with my Jeep's current light switch on the steering column. My halogen lights are h4 and the low beams are not on when the high beams are. If I want to wire a bi-xenon solenoid I will need to find some way of getting the light to stay on and have the solenoid open when the high beam is on.\nI have an idea that will work, but it involves a relay which means the lights will momentarily be off when I switch from low to high beam (as the relay changes from closed to open). Here is my drawing, will this be good enough, or will I need to make sure the voltage does not get cutoff when opening the solenoids?\n\n", "answer": "Cycling HID lamps the way you propose will reduce the lifespan of the bulbs, they aren't made to be turned back on shortly after being turned off. \nAlso, I'm having trouble making sense of your schematic - the oval labeled \"LIGHTS\" - is that supposed to represent the ballast? I don't know that I'd power that directly from your H4, seems like I'd want a new fused run from the battery through a relay powering the ballasts.\nI am not affiliated with them, but http://www.theretrofitsource.com/ sells kits that comes with a ballast controller module that can be interfaced directly to your factory H4 connector. I have one in my Subaru. It takes care of the things you appear to be overlooking - like keeping the ballast and bulbs powered while switching from low to high beams (on HID's all that should happen is the solenoid to move the shield should be powered), or implementing a time-delay so that the ballast does not fire, die, and fire again in rapid succession while turning the key from off through accessory to run, temporarily to start, and back to run.\n", "topic": "mechanics", "url": "https://mechanics.stackexchange.com/questions/4979"}, {"image": "VQAonline_00045425.png", "question": "1990 Suburban exhaust system replacement", "context": "I recently purchased a 1990 Suburban - and it has a Chevy 350. The exhaust merges and exits on the left side.\nI am unable to locate a replacement which is a direct fit. Also, almost all y-pipes online merge on the right.\nAnyone has any experience with this - or knows a cross reference with another model?\nUpdate - based on accepted answer - found the rest of the part numbers on the WALKER website\n\n", "answer": "If you looking under the \"V\" series Suburban (ie: V1500), the WALKER 40317 Y Pipe drops on the left side. It is probably what you'd need for a 4x4. If memory serves me correctly, the front drive shaft of the transfer case drops on the right side, so they had to route it down the left side. Usually, GM used the \"K\" designation for 4x4's, so the V is tossing me around a little bit. I would suggest you ensure this is the part you need for your truck before purchase.\nOn how to find other parts you might need ... you just have to look. Online searches are probably the best. Now that you might have the a better designation for your Suburban, you might be able to find it easier. Secondarily, you should consider just taking it to a shop and having them build one for you or find the parts for you. It'd be done right and you wouldn't have to worry about the outcome.\nIf you're still heck-bent on doing it yourself, a resource I use is rockauto.com (no affiliation). They have a very extensive parts list, which means you can pick and choose. If you find what you need there, you can take the part numbers and shop it wherever you need to to find the parts.\n", "topic": "mechanics", "url": "https://mechanics.stackexchange.com/questions/87740"}, {"image": "VQAonline_00044888.png", "question": "Trim repair on a 2003 Ford Focus", "context": "The door trim on the drivers side door of my 2003 Ford Focus has come loose at the front:\n\nI'm guessing that a proper repair would require the interior door panel to be removed, and this seems like a lot of work for such an old car. I would be happy with a \"bodge job\" repair if that could be guaranteed to last the few years the car has left to live. Can anyone suggest the best way to do this?\nI was thinking that double sided tape would do but the usual domestic double sided tape is too thin and I think it would come loose very quickly. Some sort of tape or other adhesive that is thicker and with more \"give\" would be needed. Can anyone suggest something like this that is easily obtainable?\n", "answer": "Get something like 3M Super Weatherstrip Adhesive or Permatex Super Weatherstrip Adhesive. Follow the directions on the product. You'll probably want to use some painter's tape (the blue stuff) to keep the trim in place while it cures. If it's a leading edge (appears to be so), make sure you have the tape covering the point of the trim so the wind won't try to screw up your job (if you are planning on using the vehicle while it is curing). \nYou'll want to leave the tape in place for probably 24 hours at least to ensure good bonding of the adhesive. The reason I suggest to use the blue tape is that it shouldn't leave a residue behind when you pull it off. Whatever you use will look ugly in the mean time, but have no fear, it's the end result you want.\nOh, one last thing ... ensure you clean the area where you want to apply the adhesive before you apply or your fix will not last. Pull up the trim the best you can and get all of the old glue off. It doesn't have to be perfect, but you should do the best you can on it. Make sure you have plenty of painted area to put the adhesive on. (EDIT NOTE: Leave the small piece of plastic on the door if it is firmly affixed as it would be a good point for adhesion between the trim and the door.)\n", "topic": "mechanics", "url": "https://mechanics.stackexchange.com/questions/18693"}, {"image": "VQAonline_00044972.png", "question": "06 Solstice LE5 DTCs", "context": "My love-to-hate-it relationship with the Solstice continues.\nI was having some issues with apparently-random warnings (ABS light, weird DTC, gas cap warning when the gas cap was not loose). The issues temporarily (for a few days) disappeared. Since an autocross last Sunday, I hadn't seen any weird behavior. (However, even though the ABS light was not on at the autocross, the brakes locked up a few times.)\nToday, my CEL came back on. I hooked up the scanner:\n0x7E8: P0171 - System Too Lean (Bank 1) (Pending, Current, History)\n\n0x7E8: P0420 - Catalyst System Efficiency Below Threshold (Bank 1) (Pending, Current)\n\n0x7E8: P0455 - Evaporative Emission System Leak Detected (gross leak/no flow) (Pending, Current, History)\n\n0x7E8: P0507 - Idle Air Control System RPM Higher Than Expected (Current)\n\n0x7E8: P0563 - System Voltage High (Pending, Current)\n\n0x7E8: P0621 - Generator Lamp Terminal Circuit\n\nI suspect a bad ECU. Even the alternator warning (P0563, P0621) appear to be harmless, since the battery is definitely getting charged. The car starts right up.\nOh, and I noticed yesterday that the tach doesn't really work for about the first 10 seconds after start. The needle just sort of bobs up and down about 5ish degrees. \nDefinitely some sort of electrical gremlin. :/ All the grounds I've checked have been fine, but there are a lot of them so it's possible I've missed one.\nUpdate\nI pulled the alternator yesterday, and saw this. Not sure how I missed this when I pulled it off of the old engine. It's worth noting that the previous motor threw a rod from the #1 cylinder. I think the rod grazed the alternator.\n\nLooks like there's a short in the coil. \n", "answer": "Oh wow, there's definitely something funky going on with the electrical system. \nWith regards to the P0563 and P0621, have you tried getting a live reading of the voltage from your scan tool or a multimeter when the car is running? \nMy advice - get a decent automotive electrician on the case. Your run of the mill mechanic isn't the right person for the job\n", "topic": "mechanics", "url": "https://mechanics.stackexchange.com/questions/27948"}, {"image": "VQAonline_00045408.png", "question": "How can I rectify Replacement filler cap?", "context": "Civic coupe vti 1999.\nI have just received my replacement filler cap off eBay and it does fit!\nHowever I noticed some white stuff around a bit the outer part of the rubber seal and thought it was dirt. Trying to remove it with some tissue the whole seal came off then I realised it\u2019s actually underlying glue which wasn\u2019t covered by the seal. The seal coming off so easily shows it was a poor manufacturing job I guess.\nAnyway im tired of this filler cap issue. Right now to remove the seal u really need to poke with your nails so I could just at least temporarily use car by manually put seal into place and every time I remove the cap just ensure the rubber is still in place as it\u2019s not moving easily even without glue.\nI could also dab a little bit of super glue(?) Where existing glue is.\nWhat do you recommenced?If the existing or super glue sticks out a little either side of the rubber will that matter as the original glue did will that matter in which case I\u2019ll need to clean and glue until I get it right?\nI imagine I could send it back but would rather avoid it. They may also argue I shouldn\u2019t have picked at the visible glue but of course I would how do I know of it is dirt that just needed picking out or glue. Plus the glue job was obviously poor to begin with.\nThanks\n", "answer": "Usually \"super glue\" type adhesives don't stick well to materials like that gasket is made of. It should stay put in the groove for it without any glue. If not, then it's not the right size or you stretched it out messing with it.\nGo to a local hardware or auto parts store and see if you can find a suitable o-ring that will fit properly.\nIf you have to, just put the o-ring on and tighten the cap as intended. Over time the o-ring will tend to stick to either the cap or the valve cover.\n", "topic": "mechanics", "url": "https://mechanics.stackexchange.com/questions/85534"}, {"image": "VQAonline_00045436.png", "question": "Honda Odyssey alignment results in camber out of spec? Normal?", "context": "I splurged on an alignment after getting new tires for my 2018 Odyssey. It was done by the my Honda dealership that has done all service on the vehicle, except the recently installed Costco tires.\nLooking at the report, it looks like Front Right Toe was out of spec before alignment, and in spec after. But then Rear Right Camber went from in spec to out of spec? (1.2 deg vs -1.0 deg to 0.0 deg?)\nIs this normal for an alignment result? I see the Toe on the same tire was almost out of spec before and is now in the middle of desired range. I was just surprised the alignment resulted in something going from being in spec to out of spec. Should I be concerned at all? Here's the alignment report:\n", "answer": "Most vehicles do not have a rear alignment adjustment. What you have is what you get. It is not uncommon for the rear alignment to go a little out of spec during an alignment procedure. Even so, looking at the report, the cross camber is within spec and the total toe became even better. You shouldn't have an issue with this alignment. The rear tires are pulled, which as the rear alignment isn't completely wonky, won't make much of a difference. If the toe was out of whack, you'd probably see a difference in fuel mileage as well as tracking issues.\nThe reason you might be seeing a little more camber that is there might be a little wear on the bushings which support the rear. Again, nothing to worry about.\nAs far as \"splurging\" for an alignment, it is my opinion you should ALWAYS get an alignment with fresh tires. You just laid down a fairly large chunk of change on those black donuts. Getting an alignment will help ensure they'll last longer and stay healthier throughout their lifetime. IOW: It is money well spent.\n", "topic": "mechanics", "url": "https://mechanics.stackexchange.com/questions/88917"}, {"image": "VQAonline_00044932.png", "question": "Why do diesel fuel lines have to be bled?", "context": "Background\nI've had a couple of situations with diesels. One in particular where I swapped a Ford F350 Powerstroke 7.8 transmission. When I was done. The truck wouldn't start. It was a painful troubleshooting situation for me because of my diesel ignorance. It turned out that the fuel lines needed to be bled.\nIt made no real sense to me but it was the solution. The fact that air was in the system at all didn't make sense to me.\nMy Question\nWhy do diesel fuel lines have to be bled?\nHere's a generic diesel fuel system image\n\n", "answer": "Not all Diesels are the same.\nMy former car, a 1994 VW Passat 1.9 TDI had only one fuel pump in the engine bay. This pump was responsible for several things:\n\nbring fuel from the fuel tank\nraise the pressure to the injection pressure (approximately 600 bar)\ndistribute pressured fuel to each injector in the correct order\n\nThere was no electric pump to \"push\", it was just the main pump which had to \"suck\" fuel from the tank and sucking is worse (in terms of efficiency) than pushing (no pun intended). Therefore whenever I had to change the fuel filter I had to carefully fill it up with fuel before installing it.\nNewer Diesels (be it common-rail or unit-injector system) usually have at least one electric fuel pump (my current common-rail TDI has two electric pumps and a mechanical high pressure pump) which aid this task a lot more.\nBut still they don't fix things completely. They still need to be activated using a diagnostic tool (for VW, TDIs there's a VCDS software pack along with an OBD-2 interface which can be used to prime the whole circuit).\nEither way, even if the pump is activated shortly using the ignition, this won't be enough to fill an empty fuel filter, or even push out a lot of air in the circuit. You will need to activate them using the diagnostic tool, or, some might connect them directly to a 12 V battery (I don't recommend).\nThe reasons for this necessity stand in a few things:\n\nDiesel fuel is a bit \"thicker\" than gasoline\nThe fuel filter is larger and having it emptied requires more effort to refill\nThe high pressure pump is usually lubricated with the Diesel fuel itself and it is much more sensitive to lack of lubrication (i.e. running dry)\n\n", "topic": "mechanics", "url": "https://mechanics.stackexchange.com/questions/24844"}, {"image": "VQAonline_00045799.png", "question": "As a young kid should I spend my time learning value investing or should I just chuck all my money into IVW/VOO", "context": "I have a genuine question, I am a noob at stocks. from what I have been reading value investors get higher returns than growth investors but then how come the growth etf IVW gets much higher returns than the value index fund VTV. I've looked at other value ETF's but they sucked even more. Is value investing worth the hype or are these ETF's just bad. If professionals can't do good value investing what chance do we have? Is this a skill worth spending my time in?\n\n", "answer": "\"Value investing\" is essentially looking for stocks that are cheaper than they \"should be\" by some measure. That ETF tracks an index of large cap value stocks,\nwhich may skew the results somewhat, since there may be more smaller \"value\" stocks that perform better than larger ones.\nAlso note that the two were largely in sync until mid-2018 and mid-2020, where large tech stocks (which are more \"growth\" than value) dominated from COVID lockdowns (there are many other factors, but that's a significant one).\nAnd note that this year, VTV has outperformed IVW - the value fund is only down 5% while the growth fund is down 18%. So it may be an anomaly that growth has outperformed over the period you're looking at.\nIf you're a \"noob at stocks\" then it's best to stick with very broad index funds until you understand more the risks associated with different styles of funds, and slowly move to individual stocks. There's no way to ensure that you pick the \"best fund\" or even the best strategy going forward.\nAs a young kid, you have plenty of time to make mistakes and learn from them. Don't try to \"get rich young\" (you might get lucky and do that, but more than likely you'll lose more than you win). Play the long game, save as much as you can, watch it grow, and as you grow you can set aside portions of your portfolio (say 10%) to experiment with individual stocks or other strategies.\n", "topic": "money", "url": "https://money.stackexchange.com/questions/152286"}, {"image": "VQAonline_00045585.png", "question": "I'm trying to setup my 401k with 5% match from my employer", "context": "I am newbie and I need help setting up my 401k. Basically, I just want to avail my employer's maximum match, and 5% is the minimum needed to reach it. But I'm not sure if I am doing it right. Here's the screenshot of my current setting:\n\n", "answer": "Based on the numbers on the screen, it looks like your income is in the range of $9,325 to $37,950 per year. This puts you in the 15% tax bracket. I personally would recommend you move the slider all the way to the right and put your entire contribution into the 401K Roth (so you'll have 0% pre tax and 100% post tax.) The reasoning here is that you'll pay the 15% tax now which is on the low side, but you'll never pay tax on the money's growth for the rest of your life, including when you take it out at retirement. If you do this your take home will be approximately $9 less per paycheck (or a little higher depending on your state income tax).\nNote that the company match will go into a traditional 401K so you'll likely see two separate accounts on your 401K statements.\nThe choice to go with traditional vs roth 401k is complicated and there are many variables. As a rule of thumb, the lower your current tax rate, the more likely you should be to lean towards the roth. Some exceptions might be if you are close to retirement age and you don't have much saved for retirement, or if you live in a state with a high income tax rate and you don't think you'll still live there when you retire, or if you are living paycheck to paycheck and the slight difference in take home pay would make a difference to you.\n", "topic": "money", "url": "https://money.stackexchange.com/questions/83267"}, {"image": "VQAonline_00045687.png", "question": "What is the strategy to gain profit from an option which has less volatility?", "context": "The following is the chart of Delta Airlines. The stock moved between the price of $52 and $62 for the last 6 months or so. Looking at this chart, someone who bought a PUT or CALL at the strike price $57 for $2 per option may not make much profit because the price has not moved much on either side.\nBeing less volatile is good, but what strategies can be used to make profit for such stocks that don't move much on either side?\n\n", "answer": "In your 6 month chart, Delta went from $56 to $63 to $56 to $60 to $52 to $57. That certainly presented a number of opportunities to double your option money or better if you had the magical ability to figure out when to be directionally long or when to be short as well as how to avoid being long or short during August/September when DAL range traded between $56 and $59. Good luck with that :->)\nTo answer your question, low risk neutral option strategies include calendar spreads, butterflies, iron condors, calendar straddles. \nThere are a number of other strategies that can be employed when you have a neutral outlook but they are by no means low risk: covered calls, covered puts, short calls, short puts, short strangles, short straddles.\nIn all cases, you're selling short term premium in the hope that price doesn't move against you and time decay works its magic.\n", "topic": "money", "url": "https://money.stackexchange.com/questions/117618"}, {"image": "VQAonline_00045554.png", "question": "Why is the highest quintile the only quintile whose wealth exceeds its income?", "context": "Based on this study, the highest quintile of the population is the only group whose wealth is greater than its income. Why might this be?\n\nIs it because they are more likely to inherit wealth from their parents?\nIs it because they invest more money so they generate more wealth faster?\n\nI am simply speculating without any proof, can anyone help?\n\n", "answer": "I think you came up with a worthy Masters/PhD research project, it is a great question.\nThis is in Australia so it is difficult for me to have complete perspective. However, I can speak about the US of A.\nTo your first point relatively few people inherit their wealth. According to a brief web search about 38% of billionaires, and 20% of millionaires inherited their wealth. The rest are self-made.\nAgain, in the US, income mobility is very common. Some act like high level earners are just born that way, but studies have shown that a great deal of income mobility exists. I personally know people that have grown up without indoor plumbing, and extremely poor but now earn in the top 5% of wage earners. \nQuid's points are valid. For example a Starbucks, new I-Phone, and a brake job on your car are somewhat catastrophic if your income is 50K/year, hurts if your income is 100K, and an inconvenience if you make 250K/year. These situations are normal and happen regularly. The first person may have to take a pay day loan to pay for these items, the second credit card interest, the third probably has the money in the bank. All of this exaggerates the effect of an \"emergency\" on one's net worth. \nTo me there is also a chicken-and-egg effect in wealth building and income. How does one build wealth? By investing wisely, planning ahead, budgeting, delaying gratification, finding opportunities, etc... Now if you take those same skills to your workplace isn't it likely you will receive more responsibility, promotions and raises? I believe so. And this too exaggerates the effect on one's net worth. If investing helps you to earn more, then you will have more to invest.\nTo me one of the untold stories of this graph is not just investing, but first building a stable financial base. Having a sufficient emergency fund, having enough and the right kind of insurance, keeping loans to a minimum. Without doing those things first investments might need to be withdrawn, often at an inopportune time, for emergency purposes.\nThanks for asking this!\n", "topic": "money", "url": "https://money.stackexchange.com/questions/70246"}, {"image": "VQAonline_00045718.png", "question": "What does it mean for the price of oil to be negative?", "context": "\nThe New York Times website currently has this graph, showing that the price of \"West Texas Intermediate, May contract\" is $-13.10. Does it literally mean that someone's offering $13.10 to take a barrel of oil off their hands, or is it a quirk of the financial market such as a trader trying desperately to make an upcoming event somebody else's problem?\n", "answer": "These are futures contracts that expire on 4/21, which means that if you hold a futures contract at the end of the trading day tomorrow (April 21st), then you are obligated to \"purchase\" 1,000 barrels (per contract) of oil for -$13 (effectively you get paid $13/barrel to take the oil).\nBut that also means that you will need the means to transport and store the oil. Which is why prices went negative - storage is full and there's no practical way for anyone to take the oil that's being produced. So speculators that were holding these futures contracts are pressured to sell them, otherwise they'll have to take ownership of the actual oil. But no one is looking to buy these contracts, so there is enormous downward pressure on price, which ultimately leads to negative prices. They are actually paying to get rid of these contracts.\n", "topic": "money", "url": "https://money.stackexchange.com/questions/124269"}, {"image": "VQAonline_00045583.png", "question": "How can Volume be greater than Open Interest?", "context": "\nNotice how at the strike price of 30 and 35 for Puts, you have volume at 22 and 7, but Open Interest at 0.\nIf volume is the number of trades made a day, and Open Interest is the number of contracts held by people, how on earth can Volume be greater than Open Interest? It doesn't make sense.\n", "answer": "Volume tells you how many trades occurred over the course of the day. Open interest is how many contracts are outstanding at that moment. So your question essentially asks how the open interest on an option can decrease---what can cause liquidation of an option? There are a few ways. These are the ones I could think of:\n\nOpen interest decreases when options are exercised. If this is not the settlement date, and if it's an American-style option, you may be looking at an early exercise situation.\nOpen interest can decrease via an offsetting trade. For example, if the option writer (creator) no longer wants to be on the hook for this option, they can basically purchase the option from the option holder and get out of their obligation. That is, if the option writer also owns the option, then the option disappears and is removed from open interest.\nOptions may disappear when the option writer violates their margin agreement and gets forced liquidation of their position.\n\nMy guess is that one of these situations (or something similar) occurred after some trades happened. My money is on #2.\n", "topic": "money", "url": "https://money.stackexchange.com/questions/81453"}, {"image": "VQAonline_00045691.png", "question": "How AAPL is keeping the EPS close to Analysts expectations?", "context": "The below chart shows that AAPL from 2017 beat the analysts expectations. I assume that the AAPL Financials are not public for Analysts to analyze, neither AAPL CFO knows how the Analysts figure out an expectation. But both of their results are a few points apart. How the Analysts figure the EPS out and how AAPL achieve a few points above the Analysts expectations of EPS?\n\n", "answer": "Most companies beat earnings estimates most of the time. So Apple isn't terribly unusual in this regard. \nWhy do companies generally beat earnings?\n\nExpectations management. Public companies regularly release guidance to analysts (both in terms of their own earnings target and in terms of qualitative and quantitative factors). As with any goal, a company's earnings target is generally something that they expect they will able to meet or exceed. If a company sees that analysts have set the target too high, they're likely to provide some sort of guidance (i.e. \"ongoing trade issues are creating some supply chain issues and may reduce sales\") that cause analysts to lower their targets. Then the company can beat the revised targets when they release earnings. Note that the company isn't lying, they're just being conservative in the guidance they provide. Regulators, for example, take a much harder look at a company that regularly misses guidance than one that regularly meets or exceeds their targets. If companies are excessively optimistic, it's usually because management is trying to keep the stock price up (so that management can dump shares at inflate prices) or so that they can borrow money from the bond market at lower rates (which potentially defrauds those buying the bonds).\nEarnings management. Accounting standards exist so that companies have less flexibility to \"massage\" earnings and that two similarly situated companies account for similar transactions in the same way. In practice, though, companies still have some amount of flexibility about how and when to recognize certain charges or how to structure certain transactions. Generally, companies use that flexibility to help ensure they beat earnings targets most of the time. If they're going to have a bad quarter, they usually use that opportunity to recognize a bunch of charges so that they have one really bad quarter that sets them up for a bunch of good quarters later.\nAnalysts, like companies, are conservative about earnings. Some of that is a result of the information analysts can acquire-- an Apple analyst may be able to get figures from some of the factories that supply iPhone screens and use that to estimate iPhone production for the year. Generally, they're going to be conservative when they extrapolate a final number from those estimates, however, to account for things that they don't know (i.e. has the market share of the factories they have numbers for shrunk from last quarter). Some is the result of market incentives-- most investors are going to be a lot more upset at an analyst whose predictions are generally a few pennies a share high than low. And some is the result of analysts getting talked down by companies when the company isn't sure they can meet the expectations.\n\n", "topic": "money", "url": "https://money.stackexchange.com/questions/118055"}, {"image": "VQAonline_00046406.png", "question": "Is \"The Architect\" in The Matrix actually a program?", "context": "We have confirmation that The Oracle is a program (in The Matrix Reloaded). So it's clear that some programs are allowed to interact with humans.\nBut I wonder whether The Architect is also a program? All the available evidence suggests so but is there any hard confirmation?\nThis also raises a related question: Are machines programming other machines as well as an artificial world for humans? And if so, then have machines reached a level of consciousness at least equivalent to (if not beyond) that of humans? And if so, are they in fact a totally independent life form? Could this be the actual future of the human race given the potential of AI?\nEdit: Also, who programmed the program who programmed the program and so on? Did this ability to program themselves spontaneously emerge or did humans somehow make this happen? Is there any evidence to support a theory on this?\n\n", "answer": "Yes, The Architect Is A Program\nFrom its article on the Matrix Wikia:\n\nThe Architect is a highly specialized, humorless program of the Machine world as well as the creator of the Matrix. As the chief administrator of the system, he is possibly a collective manifestation, or at the very least a virtual representation of the entire Machine mainframe.\n\nWhile his history is unclear, he appears to have been created by the AI behind the machine systems, and given the task of creating the Matrix - the human-like virtual world where the coppertops believe they live.\nAs for your second question (which should really be a separate question), humans were the ones who created the AI. It was this AI, who learned exponentially, that created the machines, waged war on, and eventually enslaved humanity. In order to keep its batteries happy, this AI created the Architect, who in turn created the Matrix, etc.\nNeo meets a representation of the controlling AI in the third film.\n", "topic": "movies", "url": "https://movies.stackexchange.com/questions/80336"}, {"image": "VQAonline_00046664.png", "question": "Okay, I'm gonna go eat this steak in the bathroom - why is it funny?", "context": "From Family Guy - Turban Cowboy [S11E15], why is Okay, I'm gonna go eat this steak in the bathroom funny?\nI imagine that I am missing a cultural refernce, but internet search does not help.\n\n", "answer": "I don't think it's about race, at least not directly. It could be implying and related to them eating at a cheap steak house with poorly trained chefs. Apparently eating improperly cooked steak is known to cause diarrhea, so the joke could very well be related to eating a cheap steak making the person eating it have to use the toilet immediately.\nIf there is a racial component to the joke, it could be that there's a stereotype that the demographic in question might be likely to order steak at low quality restaurants where they are likely to cook it incorrectly. I'm not familiar with such a stereotype, but it's a possibility.\nSee: https://www.livestrong.com/article/473861-steak-and-diarrhea/\nNote: This episode was removed from all streaming services after the Boston Marathon bombing because it contains a cutaway joke where Peter drives a van over some marathon runners to \"win\" the Boston Marathon. There is a transcript of the episode available, but I don't think linking to it is appropriate since it is probably not properly licensed.\n", "topic": "movies", "url": "https://movies.stackexchange.com/questions/112374"}, {"image": "VQAonline_00046440.png", "question": "How to they film walk on glass scenes in movie?", "context": "Like below image, walk on glass intro scene of mass hero is very common in Indian film.\n\nLike this kind of scenes, how to they use cameras?\n", "answer": "Whenever scene like this are needed to shoot, They usually uses a glass panel placed above the camera. \nActor walks on the glass panel. possible it could be a hardened glass. So actor walks on the glass, camera is located at below the glass, Not touching the glass. Then scene can be shot as normal one. No other special effects used. \n", "topic": "movies", "url": "https://movies.stackexchange.com/questions/85761"}, {"image": "VQAonline_00046295.png", "question": "Why was David's happiness so important to the Mecha descendants?", "context": "At the end of the movie Artificial Intelligence: A.I, the young protagonist David is found by the Mecha after 2000 years encased in ice.\nAccording to their representation of the blue fairie, there was no detail too small that David didn't store for them, yet they do everything in their power to fulfill his wishes - but why?\n\n", "answer": "David was of great historical interest to the future A.I.s, since he had actual memories of human beings, who are apparently long extinct by their time. Beyond that, he was actually built by humans, where they are undoubtedly built by other A.I.s. Studying him would help them understand their own past. They wanted to preserve him, and although they are not organic, seem to have a morality which prevents them from simply taking him apart or locking him in a cage.\n", "topic": "movies", "url": "https://movies.stackexchange.com/questions/68913"}, {"image": "VQAonline_00046356.png", "question": "What are these letters at the end of closing credits?", "context": "In 20th Century Women, I find some letters at the end of the closing credits.\n\nWhat are these? \n", "answer": "\nJM = Janet Meyer/Jan Mills, his mom\n KM = Most likely his oldest sister, but I can't find confirmation on her first name\n MA = Megan Ace, his older sister\n MJ = Miranda July, his wife\n HM = Hopper Mills, his son\n\n\nSource for names: http://www.newyorker.com/magazine/2017/01/09/mike-mills-anti-hollywood-family-films\n\n", "topic": "movies", "url": "https://movies.stackexchange.com/questions/73705"}, {"image": "VQAonline_00046008.png", "question": "Did Vin MoCap the Dance?", "context": "We all know Vin Diesel is famous for his silky moves...\n\nIs there anything to substantiate the claim that Vin Diesel was himself the model for the Motion Capture of Groot's already infamous pre-credit jive in Guardians of the Galaxy?\n", "answer": "I messaged Jesse Hildreth, the ILM animator who was responsible for that sublime little dancing Groot at the end and he told me that Vin Diesel definitely did not perform mo-cap for that shot, and in fact it was James Gunn who performed it for the reference footage!\nJesse was also responsible for most of Star-Lord's helmet shots, some close-ups on Rocket's hands and a couple of Milano shots - all cool stuff!\nUPDATE: Jesse just sent me a link with the following quote: \n\n\"I'm not motion-captured, but I'm motion-referenced and that is 100%\n me dancing...\"\n James Gunn\n\nRead more at http://www.comicbookmovie.com/fansites/MarvelFreshman/news/?a=105173#1ZQTzc6HS9QULsvb.99\n", "topic": "movies", "url": "https://movies.stackexchange.com/questions/23593"}, {"image": "VQAonline_00046566.png", "question": "Can't the Time Stone be touched with bare hands?", "context": "So throughout Infinity War and in the current Endgame, the Time Stone was never touched with bare hands, it seemed to be surrounded by a certain aura and gets transferred from person to person untouched. \nIs there a special reason behind it? \n\n", "answer": "There are no official sources for this, just some theories and speculations:\nOne speculation says it can be touched, but just like the power stone, you will suffer the effects of touching an infinity stone (Maybe it kills you, vaporizes you, etc).\nAnd that's why the Eye of Agamotto was created:\n\nThe Eye was created by Agamotto, using the Time Stone as a component to allow the user to safely wield it without directly touching it and suffering the adverse effects of touching an infinity stone. The container was created in the shape of an eye, hence its name. The mystical properties of the relic, as well as the spells and gestures necessary to use it\n\nOne other theory that I really like is that the stone represents Time, and time isn't really a physical thing that you can touch it.\n", "topic": "movies", "url": "https://movies.stackexchange.com/questions/101270"}, {"image": "VQAonline_00047403.png", "question": "Is there a typo in this exercise, or am I not counting correctly?", "context": "I have this set of right hand open string exercises for classical guitar. Many are in 4/4 but some are in 6/8 (see pic below). The notes are grouped in 6 but there is no tuple notation. Reading as straight 6/8 I expect 12 16th notes per measure. Lines 1 and 2 show 24 16th notes per measure. My question is am I reading it wrong or is there a missing bar in each line separating each into 2 groups of 12 16th notes. Thanks. \n\n", "answer": "It's incorrect. Either it's in 12/8, or there's a barline missing, or they should be demi-semiquavers. \n", "topic": "music", "url": "https://music.stackexchange.com/questions/72566"}, {"image": "VQAonline_00047554.png", "question": "How do you identify a key signature's name?", "context": "Normally, I'm really bad with key signatures. The thing is, I cannot memorize their names associated with their look. Is there an easy method/technique to recognizing the name of a key signature upon sight? For example, if I had a key signature that looked like this:\n\nHow would I find that key signature's name?\n", "answer": "The way I learned it, for flats you look at the second flat in the signature from the right. Here, it's A\u266d (B\u266d-E\u266d-A\u266d-D\u266d).\nFor sharps, take the last sharp and raise it by a half-step. If you had F\u266f, C\u266f, and G\u266f in the key signature, the last sharp is G\u266f. Up a half step from G\u266f is A.\nNo sharps is C major, one flat is F major. You absolutely need to know those two by sight.\nNote that this is for major keys. For minor, convert however you normally do from major. In this case, the minor key would be F minor.\nI highly recommend that you devote some effort to being able to recognise key signatures on sight. You don't want to spend 30 seconds figuring out that you're playing in D major during a sight-reading gig (Actually, this skill sort of comes with practice itself, but certainly don't neglect it).\nUser Dom found this site, and it has a specific exercise for this. I really like this site, and I think you'll find this useful, especially if you want to memorise them: (The Exercise)\nEDIT: If you need help with finding the minor key signatures, just find the major one (A\u266d), then find its sixth scale degree (A\u266d-B\u266d-C-D\u266d-E\u266d-F-G), et voil\u00e0, you know it's F minor. That, or memorization.\n", "topic": "music", "url": "https://music.stackexchange.com/questions/82415"}, {"image": "VQAonline_00047490.png", "question": "Are all modes relative to Ionian (major) in Roman numeral analysis?", "context": "I saw in this Wikipedia article that there are a lot of flats and sharps in the modal natural chords. And I was wondering if it's because they all refer to Ionian.\n\nSo in Aeolian the \"bIII\" chord is read \"take the 3rd chord from Ionian, and flatten its root; then use this flattened root and make a major chord from it since the roman numerals are capitalized\". Does that sound right?\nBorrowing chords from a parallel mode then is just a matter of picking chords from this table (I saw this video on modal interchange). So the resulting non-diatonic chord progression, for example I bVII IV bII, is still always in reference to Ionian (?). Meaning, bVII in this progression is read \"take the 7th chord from Ionian, flatten its root, use it to make a major chord\".\n", "answer": "Yes, you're correct! (One might say that it's actually relative to major instead of Ionian, but that's being pretty fussy.)\nThe only thing to watch out for is the accidentals that are \"hidden\" in some of the Roman numerals. Imagine you're in D and you want to add in the II chord that's borrowed from Lydian. There's no accidental with that II Roman numeral, so you might just think we have a regular supertonic chord in D. But remember that the ii chord is typically minor in major/Ionian, and this II chord is major, thus requiring a G\u266f. Another way to think of this is that Lydian has the raised fourth scale degree, which would again result in that G\u266f.\n", "topic": "music", "url": "https://music.stackexchange.com/questions/78844"}, {"image": "VQAonline_00046741.png", "question": "How do I know what pattern to strum based on chords alone?", "context": "I find a lot of music with only chords:\n\nThey don't tell me the pattern I should strum. How can I know when to strum up or strum down?\n", "answer": "For this, you can either go by ear by listening to the recorded song, or by feel (i.e. how do you WANT to play it?). As far as up and down strumming, a general rule of thumb is that - regardless of the actual strumming pattern - you should maintain a steady up/down rhythm. Then you will simply hit the strings when you want sound and not hit them when you don't. \nFor most basic strumming patterns, consider down-strumming every quarter note and up-strumming on the eighth notes in-between.\n", "topic": "music", "url": "https://music.stackexchange.com/questions/5459"}, {"image": "VQAonline_00047168.png", "question": "What is meant by setting semi-tone on Guitar Tuner?", "context": "On a guitar tuner there\u2019s a setting to set semi-tone. Options are Off, b, bb. What do these options mean? While I understand that semi-tone means interval of half-tone (one fret), I need some understanding of how b & bb are related in terms of Tuner setup.\n\nThanks.\n", "answer": "Tuners are generally set at A=440Hz. Concert pitch. With so many guitarists tuning down these days, the tuner can be reset so that the whole guitar is one semitone low,(b) as in Eb for top and bottom, or down to D, (bb),where the guitar is one tone lower than standard. \nSo, your tuner has 3 settings - standard, one semitone flat, or one tone flat.\n", "topic": "music", "url": "https://music.stackexchange.com/questions/51372"}, {"image": "VQAonline_00047413.png", "question": "Meaning of dots below note heads", "context": "I\u2019m learning music theory and I found a music sheet that I attach one part in a picture. If you see in the Treble Cleff, there are 8 eighth dotted notes and is a 4/4 signature. Like is a 4/4 is not supposed to be only the eighth notes alone without be dotted? Or why are dotted? It\u2019s supposed 2 eighth notes make 1 quarter note and 4 quarter notes fill the 4/4 signature no? But why are dotted, is not required the dot no? \n\n", "answer": "The notes in the treble clef are not dotted eighth notes, but just plain eighth notes. The dots beneath the note heads are articulation marks which mean that the notes should be played staccato. If the stems were down instead of up the dots would be probably be above the note heads in your example, but articulation marks may in general be placed either above or below the note with the goal of maximizing legibility.\n", "topic": "music", "url": "https://music.stackexchange.com/questions/73390"}, {"image": "VQAonline_00046870.png", "question": "How to determine which inversion a chord is?", "context": "Say I'm given the notes like this.\n\nHow do I determine whether it's already in root, 1st, 3rd, or 7th inversion?\n", "answer": "This is an F dominant seventh chord. The inversion is determined entirely by the lowest note, so here it would be first inversion.\nHow I identified the chord: I recognised F-A-C as an F Major chord. But even if we ignore that, when going up from A (A-C-E\u266d-F), C (C-E\u266d-F-A) or E\u266d (E\u266d-F-A-C) we get a major second every time. A chord, by definition, is a stack of thirds, so you have to reorder them until you get rid of the second. (Exception: incomplete chords skip thirds.)\nTo find out the inversion, we just have to list the notes in the order in which they are all a third apart: F-A-C-E\u266d. Then we just count these notes, starting at zero:\n\nF-A-C-E\u266d\n0 1 2 3\n\nAnd we see A maps to 1 for first inversion.\n", "topic": "music", "url": "https://music.stackexchange.com/questions/20372"}, {"image": "VQAonline_00048224.png", "question": "Help finding this older pop song", "context": "I am looking for a famous older pop song. This is the approximate melody of the chorus as sung by a woman, I think in spanish.\n\n", "answer": "I have found it : Porque te vas sung by Jeannette. Apparently, it is Spanish and from 1974.\n", "topic": "musicfans", "url": "https://musicfans.stackexchange.com/questions/7949"}, {"image": "VQAonline_00048225.png", "question": "Identify the music on Australian advertisement for Sidchrome spanners", "context": "Back in the 1980s or 90s, there was an advertisement on Australian TV for Sidchrome spanners. It showed a collection of spanners being made, starting with red hot metal being stamped out, then cleaned up, filed down, washed, and finally presented. At the end a voiceover said in a broad Australian accent \"Sidchrome. Ya canna hand a man a grander spanner.\"\nBehind all this there was quite a cheerful, flowing piece of classical music. I have never heard it anywhere else and I have no idea what it's called or who wrote it. Does anyone else remember this ad and know what the music was?\nI went looking on youtube and found a couple of old Sidchrome ads and lot of Australian ads from the period, but not this one. I will update if I can find a clip.\nUpdate: I wrote down the first couple of bars in musical notation. Take this with a grain of salt, because I can't read or write music; I sounded it out on a musical score site and took a screenshot. I couldn't make the tempo sound quite right, but I think the notes are correct relative to each other. I'll add more when I get a chance.\n\n", "answer": "I found it on youtube by accident. Not the advertisement, but the actual piece.\nIt's Bach's Violin Concerto in A Minor, BWV 1041. Here's a recording, with the musical score scrolling as it goes so you can see that my version was far from perfect.\nIt's a good piece. Go listen even if you never saw the ad.\n", "topic": "musicfans", "url": "https://musicfans.stackexchange.com/questions/8027"}, {"image": "VQAonline_00048242.png", "question": "What prejudices could neighbors have when they see a Rammstein flag on the house?", "context": "I've been listening to Rammstein since I was a teenager, and I've listened to all their songs an infinite number of times. Also the new band \"Lindemann\", inspires me.\nWhat prejudices could neighbors have if they see a Rammstein flag on the house?\n\n", "answer": "Having a quick look at the Controversies section of Wikipedia's article on the band suggests that neighbors might have any prejudice on seeing that flag. The band courts controversy actively, and their taste for using sarcasm and satire has caused many people to misconstrue the band's message.\nSomeone unfamiliar with the band, therefore, is more likely to think that the flag represents right-wing nationalistic views. Someone familiar with the band is more likely to see it as an endorsement of left-leaning anti-establishment views. Furthermore, those familiar with the band because of one of the many controversies they have sparked might associate the flag with that controversy.\nWhether any person harboring any of these prejudices views you more or less favorably as a result will, of course, depend on what that person thinks of nationalism, the establishment, or the particular controversy.\n", "topic": "musicfans", "url": "https://musicfans.stackexchange.com/questions/12083"}, {"image": "VQAonline_00048208.png", "question": "Who was the guy in the B-52's video \"Roam\"?", "context": "In the music video Roam by the B-52's there appears to be an African American B-52 with large hair (not really an Afro but similarly large).\nUpon seeing that I though \"Oh, cool, the B-52s were more diverse than I thought\" but then when I checked out the Wikipedia article to find out who he was I didn't see any African-American B-52 listed. \nWho was that guy?\n\n", "answer": "It Iooks like its Zachary 'Zack' Alford, a professional drummer known for his work in the Saturday Night Live Band, as well as with Bruce Springsteen and David Bowie.\nAccording to the biography on his website, zachalford.com, he played for the B52s in 1989. This was the year the B52s album 'Cosmic Thing' was produced, from which the single Roam was taken.\n", "topic": "musicfans", "url": "https://musicfans.stackexchange.com/questions/5476"}, {"image": "VQAonline_00048211.png", "question": "Did Jimi Hendrix's reverted Stratocaster influence his sound?", "context": "Jimi Hendrix was left-handed and at the time he started to play the guitar, left-handed ones may have been hard to find, so he strung his guitar upside down.\n\nThen, even he became rich enough to have access to any left-handed guitar he wanted, he kept playing on his reverted guitar. Was it just for personal visual style reasons or did it influence his playing, his sound on a way that he wanted to keep it?\nThe only thing I can see is with a reverted Strat, higher frets are harder to access.\n", "answer": "There are many ways it would have affected his sound.\nA few points about the interaction of pickups and strings changes.\n\nThe angle of the bridge pickup is backwards. This will cause his higher strings to be further away from the bridge where they pass over the pickup and his lower strings closer to the bridge where they pass over it. This will have caused his high strings to sound more mellow and his low strings to sound more sharp and biting.\nOn the headstock side, there is longer string travel on the bass strings and shorter string travel for the high strings between the nut and the tuners, as compared to a regularly Strat. The longer the strings travel the 'floppier' the string will sound. This one is more likely felt than heard but can definitely influence the player's technique.\nThe pole pieces which are staggered for certain strings sizes will be different and change the way individual strings sound. \n\nA couple of points about the upside down body contour.\n\nOn a normally worn Strat the upper horn, which is the longer one, will have the strap attached to it. Wearing it upside down will bring the the fretboard further away from one's body. This will make fretting open chords and notes lower down on the fretboard harder to get to and notes higher up will be easier to get to.\nThe cutaways of the two horns are not equal. The lower horn on a regularly worn Strat has a deeper cut into the body to allow easier access to upper frets. Wearing it upside down will make access to upper frets harder and the player will have to adjust his fingering technique to reach them. This needing to work hard for upper frets is something many players of Gibson Les Pauls say influences their playing.\n\nThese two points sound contradictory and one might think they cancel each other out, but in reality it doesn't work like that.\n", "topic": "musicfans", "url": "https://musicfans.stackexchange.com/questions/5842"}, {"image": "VQAonline_00048210.png", "question": "Sheet music puzzle from the Church of Higher Truth website", "context": "I got this piece of sheet music as part of an online game[?], called the Church of Higher Truth can you identify it for me? \nIt was presented as a puzzle to solve, along with this text: \n\n\"Name this song, and give me its meaning. This song is a prelude to the things to come, and it's important to know what to expect. Sing it, play it, and dream it.\" \n\nThe overarching context of the game (or is it not a game?) is that reality is not what we think, and those in power have been lying to us and we need to wake up.\n\n", "answer": "This is the opening to Richard Wagner's G\u00f6tterd\u00e4mmerung (\"Twilight of the Gods\"), the fourth and final music drama of Der Ring des Nibelungen.\nExcerpt #388 begins right at m. 1 of the Prelude to Act I; #389 begins at m. 17.\nIf this is a \"sign of things to come,\" it's probably a reference to the ominous outcome of the end of G\u00f6tterd\u00e4mmerung, where the gods are overthrown, the world burns, is immersed in water, and is reborn anew. It fits with your interpretation that \"those in power have been lying to us and we need to wake up\" and overthrow those in charge.\n(And, as an aside meant for The Church of Higher Truth: it's not a \"song.\" Songs have words, which these two excerpts do not.)\nEdit: If anyone is curious to hear the music for the end of the world, here you go.\n", "topic": "musicfans", "url": "https://musicfans.stackexchange.com/questions/5772"}, {"image": "VQAonline_00048232.png", "question": "Identifying an excerpt by Beethoven (Ddim7\u3161Abm/Eb\u3161Eb\u3161Abm)", "context": "While working on an assignment based on secondary leading-tone chords, I found this excerpt that I am not quite familiar with:\n\n\n\nI see a secondary leading-tone seventh chord to the dominant, then a cadential six-four, repeated Ebs which represent the V, and then the tonic chord. The key is Ab minor.\nCan someone identify this piece?\n", "answer": "It's the third movement of Beethoven's Piano Sonata no.31 in A flat (op.110) - III. Adagio ma non troppo.\n", "topic": "musicfans", "url": "https://musicfans.stackexchange.com/questions/9914"}, {"image": "VQAonline_00048256.png", "question": "The Enchantress and the blue rose tree", "context": "Does anyone know the title of this story and where it was translated from? \n\nSource: the Golden Bough\n", "answer": "Sir James George Frazer cited this story from the 1873 book Russian Folk-Tales by William Ralston Shedden-Ralston. Ralston, in turn, translated it from Ivan Khudyakov's 1861 collection Russian Fairy Tales, where it was numbered 109. Clearly the story is Russian in origin.\nRalston did not give the story a title, which probably indicates that neither did Khudyakov (but I can't read Russian, so someone else would have to confirm this). Traditional folk tales rarely come with names, or rather, only one widely accepted name. Hence many fairy tale collections use numbering instead. So we could reasonably say, there isn't a title.\nOn the other hand, in his The Golden Bouch, Frazier gave the paragrap containing the story the subtitle, \"Death in the blue-rose tree\". You could treat this as a title - it's as close to a name as there is for this story, in English at least.\n", "topic": "mythology", "url": "https://mythology.stackexchange.com/questions/3312"}, {"image": "VQAonline_00048261.png", "question": "What is that on Athena's chest?", "context": "\nIn this statue there is something what looks like a face on Athena's chest. I have seen it on some other statues of Athena. \nWhat is it? Is there a myth behind it? \n", "answer": "It's not clear from your picture, but the face is most probably a Gorgoneion, a recurring element of Athena's iconography:\n\nIn Ancient Greece, the Gorgoneion (Greek: \u0393\u03bf\u03c1\u03b3\u03cc\u03bd\u03b5\u03b9\u03bf\u03bd) was a special apotropaic amulet showing the Gorgon head, used most famously by the Olympian deities Athena and Zeus: both are said to have worn the gorgoneion as a protective pendant. It was assumed, among other godlike attributes, as a royal aegis to imply divine birth or protection, by rulers of the Hellenistic age, as shown, for instance, on the Alexander Mosaic and the Gonzaga Cameo.\nSource: Wikipedia contributors. (2018, June 16). Gorgoneion. In Wikipedia, The Free Encyclopedia. Retrieved 21:22, February 5, 2019, from https://en.wikipedia.org/w/index.php?title=Gorgoneion&oldid=846161112\n\nThe ever-so-helpful theoi.com has tons of information on the mythology of the Gorgons, if you wish to know more about them. We also happen to host some very interesting questions about Medusa and her sisters on this site.\n", "topic": "mythology", "url": "https://mythology.stackexchange.com/questions/5436"}, {"image": "VQAonline_00048264.png", "question": "What is the origin of a common Christmas tree quotation concerning an old Babylonish fable about an evergreen tree?", "context": "Searching for \"evergreen tree which sprang out of a dead tree stump\" gets nearly 3000 hits.\nThe Seed of the Woman and the Power of Darkness - Donbor Syiemlieh - Google Books is an example:\n\n\"An old Babylonish fable told of an evergreen tree which sprang out of a dead tree stump.\nThe old stump symbolized the dead Nimrod, the new evergreen tree symbolized that Nimrod had come to life again in Tammuz!\nAmong the Druids the oak was sacred, among the Egyptians it was the palm, and in Rome it was the fir, which was decorated with red berries\nduring the Saturnalia!\" (Walsh, Curiosities of Popular Customs, p. 242).\n\nObviously I didn't check all results, but those that I did check contain that same quotation, and almost all give the same attribution.\nThe problem is, while page 242 of that book does talk about Christmas trees, and the writing style is similar, it doesn't include the above quotation or anything like it:\n\nThe quotation doesn't appear elsewhere in that section or anywhere else in the book.\nSo what is its origin?\n", "answer": "This question has now been answered:\nquote identification \u2014 Literature Stack Exchange\nTL;DR:\n\n\u2026 those who attribute the quotation to Walsh are misattributing and/or misquoting.\nThe direct quotation originates with Ralph Woodrow, who paraphrases and cites, but does not quote, Walsh.\n\u2026 page 152 in chapter 20 of Woodrow's Babylon Mystery Religion.\n\n", "topic": "mythology", "url": "https://mythology.stackexchange.com/questions/7757"}, {"image": "VQAonline_00048245.png", "question": "How many Great Gods did the Sumerians have?", "context": "How many Great Gods did the Sumerians have?\nWas it twelve, as counted on Wikipedia, or it's just a coincidence? Does the number of Great Gods have any meaning?\nFor example in this article it's suggested:\n\nDepartment of Oriental Antiquities at the British Museum - describes a pantheon of 'twelve great gods' who despite having been given somewhat different names\n\nSee The Sumerian Pantheon (the family tree) below:\n\nDoes it indicate that the twelve was some special number? Similar to Greek pantheon (The Pantheon of Twelve)?\nThis source: \u201cAssembly of the Gods\u201d \u2013 12 Members reads:\n\nThere are 6 male deities and 6 female deities among the ruling twelve positions in the \u201cAssembly of the Gods\u201c.\n\n", "answer": "The source of the claim of twelve great gods comes from a book titled \"The Chaldean Account of Genesis\" by George Smith, written in 1876. You can see the list in Chapter 4, here:\nhttp://wisdomlib.org/mesopotamian/book/the-chaldean-account-of-genesis/d/doc2816.html \nI'm not really sure whether I buy the \"twelve great gods\" claim entirely, but nevermind that. I'll call it plausible.\n\nIn some remote age there appear to have been three great cities in the country, Erech, Eridu, and Nipur, and their divinities Anu, Hea, and Bel were considered the \"great gods\" of the country. Subsequent changes led to the decline of these cities, but their deities still retained their position at the head of the Babylonian system.\n\nSmith claims that gods became \"great\" as a function of the growth of their cities, so the list of great gods would grow naturally as cities rise and fall. Which leads, once again, to the number twelve is likely coincidental.\nMore recent resources have claimed a pantheon of 7 \"gods who decree\" and 50 \"great gods\" (from \"The Sumerians. Their History, Culture and Character\", by Samuel Noah Kramer, 1963)\n", "topic": "mythology", "url": "https://mythology.stackexchange.com/questions/206"}, {"image": "VQAonline_00048255.png", "question": "Why is Aether shown battling a lion-headed giant?", "context": "In this picture, the primordial god of the upper air, Aether, is depicted wrestling a lion-headed giant. \n\nIs there a myth/story for this anywhere?\n", "answer": "The lion-headed giant apparently is Leon, who, according to Ptolemy Hephaestion was slain by Herakles during the Gigantomachy:\n\nHerakles did not wear the skin of the Nemean lion, but that of a certain Leon (Lion), one of the Gigantes (Giants) killed by Herakles whom he had challenged to single combat.\nSource: Ptolemy Hephaestion, New History Book 5 (summary from Photius, Myriobiblon 190) (trans. Pearse) (Greek mythographer C1st to C2nd A.D.)\n\nUnfortunately, I do not think a tale of Leon battling Aether survive.\n", "topic": "mythology", "url": "https://mythology.stackexchange.com/questions/3128"}, {"image": "VQAonline_00048263.png", "question": "Who are the twelve gods of the underworld of the Hittites?", "context": "The west wall of Chamber B in the Yazilikaya sanctuary, near the Hittite capital Hattusa, depicts - according to Wikipedia - twelve gods of the underworld:\n\nWho are these twelve gods? Do they have names, and what is their function? How do they relate to other gods of the Hittite pantheon? Do they feature in any story?\n", "answer": "Thanks for the question. Very interesting search along the way to coming to the following info. \n\nhttps://www.heroicage.org/issues/15/malcor.php\nExcerpt:\nThe Twelve Companions\n\u00a718. As Macqueen (1986, 58) pointed out, \"The principle weapon\n employed [from Hittite chariots] was the stabbing spear,\" but the\n Hittite soldiers of the Anatolian hills wielded a slashing-sword,\n which was replaced by \"a long cutting-weapon with a straight blade by\n \"the end of the second millennium\" (Macqueen 1986, 59\u201360). The swords\n carried by the twelve runners in Chamber B resemble the short,\n crescent-shaped sword, decorated with animal heads which was carried\n by Hittite warriors in ceremonies rather than in battle (Macqueen\n 1986, 59\u201360). The Hittite axe born by Sarruma has \"ribbing round the\n shaft-hole [that] is a feature\" from, among other places, the northern\n Caucasus area (Macqueen 1986, 61), while the axe's blade \"is of a type\n which can be paralleled only in the Caucasus region\" (Macqueen 1986,\n 61).19 Although the gods carry the swords in this image, recall that\n in the texts the swords are embedded.\n\u00a719. The steppe war god himself was not the celestial Ram, but he was\n represented by the story of the ram, just as Jason can be represented\n by a reference to the Golden Fleece. This raises the possibility that\n he was not the sign of Aries but another sign that appeared in the sky\n along with\u2014or at the same time as\u2014Aries. The twelve figures\n associated with the sword god, then, may have nothing at all to do\n with Underworld deities. Instead, they may represent the twelve signs\n of the Zodiac, and the crossroads the Hittites associated with them\n might not be underground at all, but rather in the sky.\nConclusion\n\u00a720. The Hittite version of the Sword in the Stone story has several\n elements in common with the Arthurian variant. Both feature a sword in\n a graveyard. Both swords are associated with a king. The twelve\n runners in the Hittite variant parallel the Twelve Knights of the\n Round Table in Arthurian tradition. Also, the anvil of the Arthurian\n variant preserves the connection between the forging of iron and the\n story of the god who planted a sword in a stone. The tales are clearly\n part of the same tradition, yet, by placing the image of the sword god\n in conjunction with celestial deities at Yazilikaya, the Hittites\n retained an association that the Arthurian variant has lost: the tale\n of the Sword in the Stone had something to do with the stars.\n\n\nHistorical Dictionary of the Hittites By Charles Burney \n\n(Note: Any typos are mine.)\n\nThe clearest, most familiar manifestation of the Underworld in Hittite\n religion, or more accurately in the Hurrian cult, occurs in Chamber B\n of the Yazilikaya sanctuary outside the city of Hattusa, with the\n remarkable and indeed unique Sword-God, a god in human form emerging\n from the hilt of a dagger, its blade too shortened to be termed a\n sword. This has long been recognized as identifiable with Nergal, the\n god of the Underworld, one of the Mesopotamian deities imported by the\n Hurrians into Anatolia. From a magical ritual going far to explain the\n Sword-God of Chamber B at Yazilikaya come the words: \u201cHe makes them as\n swords and fixes them in the ground.\u201d In another text \u201cthe bronze\n swords of Nergal\u201d and \u201cthe twelve gods of the crossroads\u201d are\n mentioned together: hence the twelve running gods, not soldiers as\n once supposed, facing the Sword-God and likewise appearing in the\n larger Chamber A at Yazilikaya at the rear of the procession\u2026.\n\n", "topic": "mythology", "url": "https://mythology.stackexchange.com/questions/5894"}, {"image": "VQAonline_00048262.png", "question": "Why is rape featured so prominently in Greek mythology?", "context": "This question may be a little controversial so try to keep your answers as academically as possible. (take an example from @DukeZhou )\nIf on the subject general answers are also accepted\nI am familiar with the fact that this happened in pretty much every mythology, but Greek mythology seems to utilize it a lot. And I was wondering if there was a special reason for this behaviour apart from being the \"go-to deed\" for the writers of antiquity.\nWhile reading up on some Greek myths I started to notice a recurring pattern in the act of rape.\nIs there a reason for the writers in a considerable amount of Greek mythologies epics and poems to resort to rape?\nWhy is rape featured so prominently and what is the (if any) hidden meaning behind the rapes?\nThis is the list I put together as defined by Wikipedia:\nlist 1. list 2\n\nAethra (mother of Theseus); was raped by Poseidon\nAntiope; raped by Zeus.\nApemosyne; raped by Hermes\nCallisto; raped by Zeus.\nCassandra; raped by Ajax the Lesser, and later forced into\nconcubinage by \u1f08gam\u00e9mn\u014dn.\nChrysippus; raped by his tutor Laius.\nDemeter; according to an Arcadian myth, Demeter was\nbeing pursued by Poseidon and she changed into a horse to escape\nhim. Poseidon, however, transformed himself into a horse and,\nafter cornering Demeter, raped her, resulting in her giving\nbirth to Despoina, a maiden goddess, and Arion, a divine\nhorse.\nEuropa; raped by Zeus after he abducted her in the form of\na bull.(as seen on the Greek \u20ac2 coin)\n\nGanymede, son of Tros of Dardania; taken up to Mount\nOlympus by Zeus in the form of an eagle.\nLeda, raped by Zeus in the form of a swan.\nThe daughters of Leucippus, Phoebe and Hilaeira,\nwere abducted, raped and later married by Castor and Pollux. In\nreturn, Idas and Lynceus, nephews of Leucippus and\nrival suitors, killed Castor.\nMedusa; raped by Poseidon in Athena's temple.\nOdysseus; raped by Calypso on the island of Ogygia\nin his seven year stay.\nPersephone; raped by Hades and, in some versions, raped by\nher father, Zeus.\nPhilomela; raped by her brother-in-law Tereus.\n\n", "answer": "Preface: This subject is complicated by changing social standards. Forced marriage has been routinely practiced around the world into the contemporary era, depending on the society. (This is a major theme of Game of Thrones, as is the condition of women in pre-modern societies in general.) The Rape of the Sabine Women can provide some context. \n\nI see three main reasons, lurid appeal, ritual, and humanism. \nMythology and folklore serve many functions. They are form of entertainment (appeal), a commemoration of religious events (ritual), and, in the case of the Classical period in Greece, it is one of the vehicles for the birth of humanism. \n\nLurid Appeal \n\nThis rationale is easy. \"Sex sells\", and violence as well. You find everywhere from the very earliest myths to contemporary media. (Gilgamesh starts out with a prostitute and a fist fight. The Osiris myth involves murder, dismemberment, and necrophilia.) Rape combines the two (see: \"sexual violence\") and provides a titillation. \nStorytellers have to support themselves and to do that they need to hold the attention of an audience. One of the central rules of fiction is high stakes = high audience engagement. The more transgressive the better. (Kill Bill and The Girl With The Dragon Tattoo are contemporary exemplars, and both can be seen as forms of Philomel, which was a female-centered revenge story involving rape.) Rape is widely used in modern entertainment as a pretext for revenge, so this phenomenon is in no way restricted to Greek mythology. \n\nRitual\n\nBurkert, Eliade, and Frazer have written extensively on this subject. At a very high level, there is archeological evidence of human sacrifice in ancient Europe and other ancient cultures. Human sacrifice is a central element of several of Euripides' plays, but by the time of Euripides, these sacrifices of legend have been ritualized in the form of drama. (The Festivals of Dionysus were religious festivals, and the play The Bacchae is, at its core, a commemoration of the death of the god.)\nThere is an original religious function to mythology. It covers everything from the creation of the universe to the acts of the gods. In the case of the Greek gods, these acts are often lurid. The behavior of the gods is commemorated in stories, rituals (re-enactments), and in the acts of humans. Cronus swallows his own children, as do Tereus and Thyestes. Just as Cronus overthrows Ouranos, Zeus overthrows Cronus.\nPersephone, in the ancient context especially, is a coming-of-age story. Her abduction and forced marriage to Hades explain the cycle of life and the seasons. It's also symbolic of death and rebirth. [See: Kore & Persephone] \nPersephone's is a central myth, explaining winter, and the Kore is an archetype--in some sense, her story is the story of almost all women. Abductions and forced marriage in Greek myth may be reflections of her story.\nThere also the factor that Zeus is a serial rapist and so it's not surprising rape becomes a staple of Greek Mythology. A key difference is that what is permissible for the gods is not necessarily permissible for man--Zeus marries his own sister. So human rapists in Greek mythology are generally villains (Tereus and Thyestes) and, unlike the gods, typically come to bad ends, even in the case of the hero little Ajax.\nIn his Agamemnon, Aeschylus makes the king's concubinage of Cassandra Clytaemestra's main motivation for killing her husband (jealousy and fear of being supplanted). Although Aeschylus was quite sympathetic to Agamemnon, Euripides is significantly less so, citing Agamemnon's murder of her infant from her previous marriage, and sacrifice of their daughter, Iphigenia, at Aulis, as Clytaemestra's motivations, which leads to the next element: \n\nHumanism\n\nThe humanism wiki doesn't give the full story because the undermining of divine authority begins with works like the Iliad, which has many functions, including subversive. In this great poem the dignity of the gods is undermined while the dignity of man is elevated through Achilles and Hector. It also evokes tremendous pity, for Hector in particular--the poem, which begins with the \"rage of Achilles\", ends with \"the funeral of Hector, the breaker of horses.\"\nUnlike the storytellers, thinkers like Plato used mythology to explicate philosophical inquiry. (In the sense of the Iliad as a subversive text, Zeus may be said to not have escaped the prophecy on Thetis in being ultimately diminished by her son, ironically in a rare case where he forgoes having sex;)\nCatharsis (pity and dread) was a central element of Greek Drama. These plays rarely commemorate the travails of the gods, instead celebrating the ordeals of humans. In the Bacchae, it is Pentheus who serves as tanist for Dionysus. Aeschylus writes of the sufferings of Prometheus in service of mankind, but also writes of the trials, and vindication, of Orestes. Sophocles most famous heroine is Antigone, who dies in service of a principle. Euripides can be viewed as strongly feminist, through plays such as Hecuba, The Trojan Women, and Iphigenia at Aulis. (Humanism is rooted in the agency of humans, and feminism in the agency of women, both central elements of Greek Drama.) \nAn aspect of Shakespeare that makes the works such standouts is the creation of multi-dimensional female characters, whether villains or heroines. So with Euripides. \nLike The Rape of Lucrece, Ovid's account of Philomela evokes tremendous pity. Empathy is a key to seeing the other as human. Andromache is play about Hector's wife, and her life as slave, after the fall of Troy. It can argued that by creating strong, believable female characters, and in generating sympathy for their travails, authors like Euripides, Ovid and Shakespeare put a spotlight on the condition and plight of women. In an indirect sense, this supports the idea of women's rights.\n\nPolitical Uses\nThere can also be a political component, as in the myth of Lucrece, which casts her abuser, Tarquin, as a tyrant to be overthrown. This is the rationale for the establishment of the Roman Republic. Exploring the meaning of the Latin r\u0103p\u012do, rooted in abduction and seizure by force, women, as prizes of war, were \"booty\" in both senses of the word. The abduction of Helen is the pretext for the Trojan War, but, if there was such a war, it was surely fought over the trade route connecting Asia & Europe and the Euxine & Aegean. \n\nNotes:\nRape, in the modern conception, seems, in pre-modern times, only considered transgressive in regard to wives and daughters specifically in an extra-marital context--i.e. only a crime when involving \"free women\", or, women of status, as opposed to chattel, and only outside the bounds of marriage. \nModern statistics on rape can be controversial, but under-reporting is definitely an issue. Regardless of the precise numbers, sexual violence is still common, and rape is still used as an instrument of warfare. The prevalence of rape in stories and mythology may be a reflection of how often it occurs in the real world. \nI tried to keep this concise in that any one of these aspects could fill a dissertation, so I apologize if everything is not fully explicated. I can provide more details in comments. I also recognize this is a potentially charged subject and have attempted to approach it as academically as possible.\n", "topic": "mythology", "url": "https://mythology.stackexchange.com/questions/5461"}, {"image": "VQAonline_00048268.png", "question": "Is Linux Mint's version of Firefox really Free software? Is it compliant with the MPL?", "context": "Trying to keep this as short as possible... In a VM, to test a problem of someone else, I installed a fresh Linux Mint 17.3 and opened Firefox. The search bar (right of the address bar) didn't include Google as an option, as I'm used to. Proceeding to the option to add search providers, I finally found it, along with a \"reason\" it wasn't there by default.\n\nA little put off by that, but not too worried about it, (they need money too, right?) I found the link for Google and enabled it. Now it was available, and all was good. Except that it wasn't.\nNormally, as I type in the search box I get suggestions that lend themselves to what's been typed so far. Often this shortens my typing significantly. Now, it wasn't doing that. I switched back to Yahoo, and it work as normal. Back to Google, no such luck. Not unfamiliar with about:config I poked around, then dug around, and did some comparisons with settings between Yahoo and Google. No dice, can't find what triggers the difference in behavior.\n\"Ok,\" says I to self, I'm just go to the repository and drop the Mint custom version, and install the Mozilla original version. Into software manager and drill down to Firefox. No options to upgrade, downgrade, switch versions, or anything else, just \"Remove\" which, after doing that, only gives the option to \"Install.\" Shows what version will be installed, but no options to change it.\nI'm not a Mint user, so I don't know if there's other options for controlling software or not. I'm sure that a source install is likely to be available somehow. But.. Is that the point.\nAs I understand \"Free\" software, it's about choice, and not having someone, disto author in this case, dictate what I can use. The option to add Google seemed to fulfill that promise, until I found it only half-worked. Making it difficult, maybe prohibitively so, to replace the restrictive version with an unrestricted version was the icing on the cake.\nEdit\nSo, Linux Mint, Linux kernel and all, is, or is not, Free software? BTW: The source is available, so it's probably \"Open source\" at least.\nThe proper question relates only to the version of Firefox, firefox_51.0.1+linuxmint1+rosa, supplied in the repos of Mint, not to the distro itself, or to any other package in the disto, including the kernel. As such, the question should have been, \"Does the version of Firefox supplied with Linux Mint 17.3 comply with the MPL and is it still free within the FSF concept?\"\nNotes:\nIn re: the \"crippling\" of the software: I don't know how long I've been using Mozilla products, or Firefox in general, but as their product have evolved, and as I've moved from one distro to another, between OSs, and between platforms, I have become accustomed to features behaving consistently across them all.\nI'll not argue, here or in comments, as to whether or not that version is \"crippled.\" I do know that it is modified, and that it does not work the same as every other release of Firefox that I have available to me for testing. The modified source code is available, to anyone who wants it, and according to the accepted answer, modified or not it does comply with the FSF's four freedoms. That was the question, and that is the answer.\nI have been replying to comments that go beyond the scope of the question, and I probably shouldn't have been. By my rep, even across SE as a whole, you can see that I'm learning the Stack Exchange way. Comments about the real question will be replied to, while my crippled phrase, my improper scope to include the distro, and related debates won't be fueled by me. Further insights, or explanations that are germane to the OS topic are welcomed and encouraged. That's how others like me can learn.\n", "answer": "Software is free (as defined by the FSF) if it gives you the four freedoms:\n\n\nThe freedom to run the program as you wish, for any purpose (freedom 0).\nThe freedom to study how the program works, and change it so it does your computing as you wish (freedom 1). Access to the source code is a precondition for this.\nThe freedom to redistribute copies so you can help your neighbor (freedom 2).\nThe freedom to distribute copies of your modified versions to others (freedom 3). By doing this you can give the whole community a chance to benefit from your changes. Access to the source code is a precondition for this.\n\n\nIt could have bad usability, it could have bugs, it could spy on you, it could technically restrict browsing to certain sites or installation of certain software, it could intentionally delete your music \u2013 as long as it gives you the four freedoms, it qualifies as free software.\nThanks to these freedoms, you are allowed and enabled to change the things you don\u2019t like, and to share the changed software with others. Note that this doesn\u2019t mean that you need to be able to change it e.g. within the GUI, or that is has to be easy \u2013 it just has to be allowed (legally) and possible (source code provided).\n\nSo the issue you mention doesn\u2019t affect whether or not Linux Mint is free software.\nIs it free software? Well, this depends on what is meant with \"it\", as a GNU/Linux distribution consists of many different parts, and could come with separate but pre-installed software.\nIf we talk about the ISO that can be downloaded (so not taking any packages into account that can be installed from their repository): According to Linux Mint\u2019s FAQ Does Linux Mint include proprietary drivers?, it doesn\u2019t contain any proprietary software (but note that this applies to the current release; it seems that this was not the case for older releases). So yes, Linux Mint is free software.\n", "topic": "opensource", "url": "https://opensource.stackexchange.com/questions/5168"}, {"image": "VQAonline_00048265.png", "question": "Can I \"Override\" a licensing policy?", "context": "On many sites, especially Stack Overflow, I can't help but notice such amazing code snippets. However, they come with a \"catch\" designed by the author.\nWith user contributions being licensed with the Creative Commons ShareAlike 3.0 License, many users try and \"override\" this by stating how they license the work on their profile page. For example:\n\nAll contributions that I make to this site shall be licensed under the Apache 2.0 License\n\nThis surprises me, because it seems to violate the policy of licensing, if not the Terms of Use that are present as well.\n\n\nIs this acceptable? Are users allowed to essentially \"override\" and decide their own licenses, against what has been standardized by the organization?\n\n", "answer": "It depends. For Stack Exchange: yes.\nWhen you write something, you own the copyright, and have the right to do whatever you want with it (from a copyright perspective at least).\nWhen you post something to some site, you usually give a license to use that material on that site, through the terms and services, or you transfer the copyright to them.\nIf the terms are to transfer the copyright to them, you no longer own the copyright, and you can't grant additional licenses.\nIf you grant them a license, it's either an exclusive license, or a non-exclusive license. If it is an exclusive license, you can't grant anyone else a license anymore.\nIf it is a non-exclusive license, you may still dual license your content, and make it available under some other license as well.\nIn the case of Stack Exchange, the terms read\n\nYou agree that all Subscriber Content that You contribute to the\n Network is perpetually and irrevocably licensed to Stack Exchange\n under the Creative Commons Attribution Share Alike license.\n\nThis means it is a non-exclusive license, and you can release your content to others under a different license as well. This seems to be the most common scenario for other sites as well, but you should always double check.\nNote that this doesn't override the license. You are dual licensing your content. Per the terms and services, you have no choice but to license your work under CC-BY-SA. But you have no restriction to also license it under a different license of your choosing.\nThese licenses don't have to be compatible. Anyone can choose under which license they use your work, the standard StackExchange one (CC-BY-SA), or another you offer personally.\n", "topic": "opensource", "url": "https://opensource.stackexchange.com/questions/504"}, {"image": "VQAonline_00048270.png", "question": "How to handle a pattern of GitHub volunteers wanting to work on an issue then disappearing", "context": "Since I registered my project on Up For Grabs, I get a lot of novice volunteers, which is great as I like both the ideas of 1) improving the app 2) introducing novices to open source collaboration.\nBut I have a management+social+technical problem, illustrated by this example today:\n\nAs you can see, one after the other, 3 volunteers have taken the issue under their responsibility then become silent, with a month's interval between each.\n\nThey are volunteer novices, so the problem is not them. I fully expect that they might give up, and hold no grudge against them.\nOn the technical side, GitHub is not helping, as I can not assign an issue to someone who is not a member of the project. And I don't think making them a GitHub project member is a good idea at this step (or is it?). I created the \"assigned\" label to work around this problem, but it is not optimal as it does not say to whom the issue is assigned. I want to continue to use GitHub for issue tracking, though, as it has become a de facto standard.\nI hesitate between spending more time spoonfeeding newcomers, or spending less time so that I dedicate my time to other aspects of the project.\nBefore we had a policy of \"First pull request wins\", that lead to frustration as we had some real cases of novices posting nearly the same pull request at the same time for something non-trivial that they that obviously had dedicated a lot of time to. Not a pleasant situation, and a very traumatic first experience for some of them, which is something I really want to avoid.\n\nHow can I improve my handling of the situation? Be it via behavior, project guidelines, technical solutions, or anything else.\n", "answer": "EDIT: while the content of this answer may still be interesting, the limitation of GitHub that is discussed is no longer relevant, as this is now possible to assign an issue to anyone who comments it.\n\nOn the technical side, Github is not helping, as I can not assign an issue to someone who is not a member of the project. And I don't think making them a Github project member is a good idea at this step (or is it?). I created the \"assigned\" label to work around this problem, but it is not optimal as it does not say to whom the issue is assigned. I want to continue to use Github for issue tracking, though, as it has become a de facto standard.\n\nWhile agreeing completely with Philippe's answer, I will try to answer to the technical sub-question here: indeed GitHub does not make it easy to assign issues to new contributors but there are workarounds.\nIf your project is part of an organization (and only in this case), you may create a team in your organization (call it \"External\" or \"New contributors\" for instance) and give it \"Read access\" to your repository (see this screenshot of the repository settings). When your repository is already public, giving read access basically amounts to giving no privileges, but it will allow you assigning issues to people in the team (they have first to accept the invitation though).\nIf you use this to assign issues to newcomers, you should have a clear rule in your contributing guidelines that not communicating for a week (or any other predefined period) amounts to losing the issue.\n\nThe rest of this answer is drifting away from the original question but it might still be of interest to you: you can use \"Write access\" as the next level of privileges for people who have already contributed usefully a few times. If, at the same time, you protect your branches and activate the \"Review requested\" feature and restrict merging to a subset of contributors (possibly only administrators), then \"Write access\" will mean privilege to label / milestone / close / reopen / assign issues (and edit / delete comments), to mark a pull request as approved, but not to commit anything directly to a branch nor to merge pull requests. Then the next level of privilege can be permission to merge approved pull requests (leaving pushing on branches directly or merging unapproved pull requests to repo admins only).\n", "topic": "opensource", "url": "https://opensource.stackexchange.com/questions/6121"}, {"image": "VQAonline_00048281.png", "question": "Can code licensed under AGPL also be (re)licensed under LGPL by its contributor?", "context": "I have a puzzling question:\nSomebody contributed to a AGPL licensed project (he is not the owner). Now this code could also be applied to a different project licensed under LGPL. Could the author of this code part (he is listed in the license header of the AGPL code as (c) ) give permission to use that code in the other LGPL project? The contributor thinks that he can not because he signed some contributor agreement with the AGPL project but I wanted to ask this here and maybe someone has some insight...\nUpdate: I think I found the CLA, so here is the interesting part:\n\n", "answer": "What this author has signed in not a Contributor License Agreement, giving the project additional (non-exclusive) rights in addition to what they get under the AGPL, but a transfer of copyrights (CTA, Copyright Transfer Agreement).\nThis means that the AGPL project owns the copyrights on the contribution and the author has just as many rights as anyone else who obtained a copy1.\n\n1: This is not entirely true, depending on the view on moral rights in the author's jurisdiction, but that does not make a difference when it comes to re-using the code in a different project.\n", "topic": "opensource", "url": "https://opensource.stackexchange.com/questions/10136"}, {"image": "VQAonline_00048269.png", "question": "Mixing GPL v3 and Proprietary library", "context": "My application has two parts GUI and a backend algorithm module. I have used a library under GPLv3 to write my GUI. In the GUI we can create a workflow of different digital signal processing units so I thought to share this GUI to the community under GPL, but I wish to keep my some algorithm modules as proprietary, what are the possible approaches?\n\n", "answer": "That's easy. Make sure you don't compile your GUI and your algorithms together statically as a single executable. If you do that, you don't have any GPL obligation for your algorithms.\n\nRun your algorithms as a command-line tool and use inter-process communication in your GUI. This is a very common implementation in computer chess engines.\nRun your algorithms as an internal web server and use internal web connection\n\nExample from computer chess (use process communication):\n\nhttps://gamedev.stackexchange.com/questions/136434/how-can-i-use-gpl-software-like-stockfish-chess-engine-in-my-unity-game\nTo do this validly, you must make sure that the free and non-free programs communicate at arms length, that they are not combined in a way that would make them effectively a single program.\n\n", "topic": "opensource", "url": "https://opensource.stackexchange.com/questions/6044"}, {"image": "VQAonline_00048266.png", "question": "Interference of Software Patents with Free & Open Source Software", "context": "While visiting wikipedia page on Free and open-source software, I noticed the challenges from the template at bottom:\n\nIn which Digital rights management and Software patents seems considerable to me.\nSo, I want to know interference/interaction of Software patents with Free & Open Source Software\nIn other words how do we interact with it?\nNote: This question is split off from this to be specific on Software patents.\n", "answer": "Why is it a problem?\nPatents are dangerous because they can put an additional restriction on the use and distribution of software which is not mentioned in the license.\nA company can freely contribute to an open source project and license their work under an open source license, but in the meantime file a patent on some aspect of their contribution. Later they can then use that patent to collect license fees from distributors and users of the allegedly free software. All of this is completely legal because licenses usually only deal with copyright law, and patent law is a completely different beast, even though the outcome is usually the same. \nThis can also be done by companies which were not involved in the development at all but own a similar patent filed before the project was published. The patent doesn't even need to be enforceable. The threat alone that it might apply can be enough to stunt the adoption of free software, because patent lawsuits are expensive. Companies fear that they might lack the resources to fight a patent lawsuit, even when they could win with enough money.\nPatents can also be used to prevent someone from creating a free alternative for a proprietary product. A good example for this is the H.264 video encoding algorithm, also known as MPEG-4. The algorithm was developed as proprietary software, but how to implement it is no secret. An algorithm can not be copyrighted, only a specific implementation can. So as far as copyright is concerned, anyone could create a free implementation for everyone to use. But unfortunately there are various companies which hold several software patents which apply (or might apply) to this algorithm. This allows them to collect royalties from anyone implementing the algorithm, free or not.\nAdvocates of free software thus recommend to use different algorithms like OGG Vorbis which was specifically designed to circumvent any known patents on video encoding. But video encoding is such a minefield of patents that some people believe that there might still be some less known patents out there which could also be applied to Vorbis and fear that wide adoption of Vorbis could prompt the holders of these patents to surface and start to demand money.\nPatent law is an area which is not internationally homogenized. Different countries have different laws regarding what can be patented and which laws apply. Software patents in particular is a shaky area which is handled differently in different jurisdictions. This can lead to the situation that a certain piece of free software is only free in some parts of the world and not in others, violating the right to free distribution without discrimination of certain groups.\nHow can we deal with it?\n\nDemand patent licensing in licenses. The GPLv3 for example has a section on patents which says that any contributors must license any patents which apply to their work for free. However, this does not help against 3rd party patent claims.\nWork around the patents. When you have an open source project, do your research about which patents might apply to the area and try to not violate them. Release early, because after you released a product, any applications for new patents which apply to the product are invalid due to prior art.\nBust the patents. When prior art can be proven for a patent, it can be declared invalid. A good medium for crowdsourcing potential prior art is the stackexchange site Ask Patents which has a specific tag for prior art requests. The Electronic Frontier Foundation also has a patent busting project.\nBuild a defensive pool of patents. Just play the same game the patent trolls play. Register patents yourself, and then pledge that you will only enforce them against those who also try to press their own patent claims. This is the strategy many large companies use to protect themselves from frivolous patent claims. Unfortunately this only works against people who actually implement their patents in products and are not just in the business of filing lawsuits. Also, the strategy of fighting fire with fire might not sit well with the more ideological parts of the free software community.\nPolitical lobbying against software patents. Software patents have already be declared unenforceable in large parts of the world. But there are still countries which have not yet understood how bad they are for innovation and the international competitiveness of the countries IT economy. Support organizations which try to convince politicians to change the local patent laws for the better. When you live in such a country, try to convince your local politicians yourself and consider their opinions about software patents when casting your vote in the next election.\n\n", "topic": "opensource", "url": "https://opensource.stackexchange.com/questions/1088"}, {"image": "VQAonline_00048275.png", "question": "Can I use separate license for commercial use?", "context": "\n\nI would like to create a REST API in my server that internally use a GPL library. As far as I know the GPL license affects the 'distribution' not the 'usage'.\n\n-So whoever calls(uses) my REST API shouldn't be affected by the GPL license right ?\nAm I obliged to distribute the API code that uses the GPL library?\n\nSo must I Distribute Code server and make it under GPL?\n\n", "answer": "This boils down to \u201cwhat is a derivative work?\u201d and the answer is \u201cwe don't really know\u201d.\nHowever, there seems to be general industry consensus that a server program and a client program that behave as separate programs and just communicate over some IPC mechanism such as HTTP messages are separate works, not derivative works of each other. If that is the case, distributing the client cannot trigger any GPL requirements in the server code.\nThis means your understanding seems correct.\n\nAs long as you don't distribute/convey your server software you do not have to disclose the source code.\nYou are correct that the GPL does not trigger when merely running the software.\nException: if you include modified AGPL-covered software then that already triggers when users can interact with that software, i.e. when you publicly perform the software.\n\n", "topic": "opensource", "url": "https://opensource.stackexchange.com/questions/8047"}, {"image": "VQAonline_00048299.png", "question": "Why isn't $x_2+x_3+x_4\\le 2$ a cutting plane?", "context": "\nIn my textbook, to generate cutting planes, they tell you to proceed as follows:\nA procedure for generating cutting planes:\n\n\nSelect a ($\\le$) constraint that has only nonnegative coefficients.\nFind a group of variables such that\n\na) The constraint is violated when all variables in the group equals 1\n and the remaining variables equal to 0.\nb) But the constraint is satisfied if any one of the variables in the\n group is changed from 1 to 0.\n\nSuppose there are $K$ variables in the group add the following\n constraint as a new cutting plane:\n Sum of variables in the group $\\le \u2212 1$\n\n\nMy Question: Why isn't $x_2+x_3+x_4\\le 2$ a cutting plane? Since if we go by 2a) , and put $x_2=1 , x_3=1 , x_4=1$ and $x_1=0$ , then it violates the inequality? Since $11>7$\n", "answer": "It is a cutting plane, but it is implied by $x_2+x_4\\le 1$ and $x_3\\le 1$ so not very useful.\n", "topic": "or", "url": "https://or.stackexchange.com/questions/3267"}, {"image": "VQAonline_00048313.png", "question": "Can I apply decomposition methods for this scheduling problem", "context": "I have a centralized optimization problem for a residential area in the context of a smart grid and load flexibility. So let's say I have 10 buildings and each of them has an electric heating device. Further each building has a photovoltaic PV system that generates renewable energy. Now the goal of the centralized optimization problem is to schedule the heating activities of all buildings such that the overall usage of the total reneable energy (sum of all the PV generation of all buildings) is maximized.\nBasically every building has 24 decision variables (one for each hour): x_1, ..., x_24\nThe quantiy the (continious) power consumption of the electric heating device in each hour. Of course we also have some thermal constraints for the room temperature such that it is not possible to heat with full power during all hours. Moreover, each building has a PV generation that is given exogeneously meaning that this is a parameter: PV_1, ..., PV_24\nSo the goal of the centralized optimizer is to choose x_1, ..., x_24 for all 10 buildings such that the PV self consumption rate, is maximized. The total electrical power P_total for the residential area for one timeslot is just the sum of the x_1s, ..., x_24s for all buildings. The same is valid for the total PV generation PV_total of the residential area. Now we want to minimize the sum of PV_total - P_total over all 24 hours of the day (this maximizes the self-consumption rate of PV).\nI hope you have understood the basics of this problem (if not I can give you more information). My fundamental question now is, whether I can use one of the following decomposition methods to transform this centralized large optimization problem into distributed smaller problems that could be solved by the buildings themselves. Basically I would like to know if I can use one of these methods:\nADMM: Alternating methods of multipliers\nDual decomposition\nDantzig-Wolfe decomposition\nBenders decomposition\nI'd appreciate every comment.\nUpdate: Here is the complete optimization problem\nx_t_b and y_t_b are the decision variables for all timeslots t and all buildings b. The objective is to minimize the surplus power in the residential area. The surplus power is calculated by subtracting the total electrical deamand of the area from the total PV geneartion. But only the positive surplus power should be minimized. I use the Big-M approach to incorporate this into the model with the two big M parameters and an auxillary binary variable h^positive. I have two thermal storage systems (modelled by T^UFH and V^DHWuse) which have upper and lower limits. x_t_b is the decision variable for heating up the T^UFH_b and y_t_b the decision variables for heating up the V^DHWuse_b. Only on of them can be heated up at one time slot. To model this I use a binary auxilliary variable h^Aux for each building and each time slot. The total demand P^total comprises the electrical demand of the heat pumps and the inflexible demand P^Demand that is a parameter.\n", "answer": "Thank you for adding the formulation. Assuming $\\Delta t$ is a nonnegative coefficient, you can simplify the formulation by omitting $h_t^\\text{positive}$, $P_t^{\\text{Surplus}^-}$, and the associated constraints. Instead, you need only impose\n$$P_t^{\\text{Surplus}^+} \\ge P_t^\\text{PVtotal} - P_t^\\text{total} \\quad \\text{for all $t$} \\tag 1$$\nto get the desired behavior of minimizing the positive surplus.\nIt looks like you still need some constraints to enforce the relationships between $T$ and $V$ and the rest of the model. Currently, $T$ and $V$ appear by themselves.\nIf you substitute the expressions for $P_t^\\text{PVtotal}$ and $P_t^\\text{total}$ (or use implicit/defined variables) and treat $(1)$ as the linking/complicating constraints, the rest of the problem decomposes by building, so you might benefit from applying Dantzig-Wolfe decomposition with one block per building.\n\nUpdate to answer your additional questions:\nBy substitution, I meant replacing $(1)$ with\n$$P_t^{\\text{Surplus}^+} \\ge \\sum_{b=1}^B P_{t,b}^\\text{PV} - \\sum_{b=1}^B \\left((x_{t,b}+y_{t,b})P_b^\\text{HPmax}+P_{t,b}^\\text{Demand}\\right) \\quad \\text{for all $t$} \\tag{1$'$}$$\nTo treat $(1)$ or $(1')$ as linking constraints, some automated Dantzig-Wolfe implementations allow you to specify the block-angular structure by using constraint attributes. For example, see this Getting Started example from the SAS documentation.\nDantzig-Wolfe decomposition applies when you have a set of linking/complicating constraints whose omission makes the resulting problem much easier.\nIn contrast, Benders decomposition applies when you have a set of linking/complicating variables such that fixing the values of these variables makes the problem much easier. In principle, either decomposition algorithm can be used for any subset you choose, but the best performance is obtained when your problem has some structure where only a small percentage of constraints or variables are complicating. The Dantzig-Wolfe decomposition I suggested seems the most natural for your problem, and it matches your intuition of wanting to solve the buildings separately.\n", "topic": "or", "url": "https://or.stackexchange.com/questions/4395"}, {"image": "VQAonline_00048332.png", "question": "A variant of weighted perfect bipartite matching", "context": "I am currently working on weighted perfect bipartite matching, i.e., assignment problem.\nFormally speaking, it could be formulated as follows:\n$$\nmin \\sum\\limits_{i=1}^{N}\\sum_{j=1}^{N}c_{ij}x_{ij}\n$$\n, where\n$$\n\\sum\\limits_{j=1}^{N}x_{ij} = 1 (i=1...N),\\\\\n\\sum\\limits_{i=1}^{N}x_{ij} = 1 (j=1...N),\\\\\nx_{ij}\\in \\{0,1\\}\n$$\nHere, $c_{ij}\\in R$ is given as the cost matrix.\nBut now, my problem has an additional constraint on it : $c_{ij} < c_{ij'}$ if $j < j'$.\nThat is, each row of the cost matrix is monotonically increasing. For example, the following cost matrix:\n\n(By analogy with workers and jobs, it is like that jobs in right is more difficult so every workers needs more time to get it done.)\nThe classic Kuhn-Munkres algorithm can be used to solve assignment problem and it can be applied to this surely. But I wonder if any faster algorithm (or related research) exists. I have tried some greedy approaches but it doesn't seem to work.\nThanks!!\n", "answer": "I don't think the monotonicity will help much, for the following reason. Start with any arbitrary assignment problem. Add a constant amount $K$ to $c_{i2}$ for all $i$, with $K$ sufficiently large to make $c_{i2} > c_{i1}$ for all $i$. Since someone has to be assigned to sink 2, this effectively adds a constant amount $K$ to the objective function, and so any optimal solution to the modified problem will be optimal in the original problem (and vice versa). Now repeat, adding a (different) constant amount to each column of the cost matrix to make it bigger than the previous column, until the monotonicity condition is satisfied.\nWhat this shows is that every assignment problem is equivalent (in the sense of having exactly the same feasible region and exactly the same set of optimal solutions) to an assignment problem satisfying the monotonicity condition. If someone had found a way to exploit monotonicity, it would be used for all assignment problems. As far as I know, there is no such algorithm for the general assignment problem.\n", "topic": "or", "url": "https://or.stackexchange.com/questions/6945"}, {"image": "VQAonline_00048303.png", "question": "Transforming a Quadratic constraint to SOCP", "context": "I have a problem similar to Markowitz portfolio optimization that I would like to transform into second-order cone programming. I have a linear objective function with a quadratic constraint (assuming that I can take the square on both sides of the constraint to make it quadratic). \nAssume that $X$ is a vector of decision variables. The objective function and the constraint are below. How can I transform it into a second-order cone constraint? \\begin{align}\\min&\\quad c^\\top X\\\\\\text{s.t.}&\\quad\\sqrt{\\sum_{i,j}^n(W_i\\cdot X)^2+(W_j\\cdot X)^2+2\\rho_{i,j}(W_i\\cdot X)\\cdot(W_j\\cdot X)}\\le b\\end{align} where \n\n$W_i$ and $W_j$ are matrices of constant values of the same dimension as $c^\\top$\n$\\rho_{i,j}$ are correlation coefficients, the matrix generated by it can be assumed to be positive semi-definite\n$b$ is a constant.\n\nFor example, when $n=2$, the constraint is given by $$\\sqrt{(W_1\\cdot X)^2+(W_2\\cdot X)^2+2\\rho_{1,2}(W_1\\cdot X)\\cdot(W_2\\cdot X)}\\le b.$$\nI would like to understand this both when it is expressed in a more compact matrix form and also when written in the more simplified summation form (as in the question). \nFor example, \n", "answer": "Revamp of my answer given the example now provided.\nLet there be $n$ VaR factors. Let $R$ = $n$ by $n$ matrix of correlations (the 2nd matrix in your example) of the VaR factors.\nLet $W$ = $n$ by $1$ vector whose ith element is $W_i$.\nThe VaR portfolio constraint can be expressed as $$W^TRW \\le b^2.$$\nThis constraint can be rewritten in terms of $X$ as follows:\nLet $M$ be the 1st matrix in your example. Then it is the case that $$W = MX.$$ Using that, the VaR portfolio constraint can then be expressed as a convex quadratic inequality constraint in terms of $X$ (resulting in QCQP, or actually, QCLP) \n$$X^T(M^TRM)X \\le b^2.$$\nLet $F$ be the upper triangular Cholesky factor of $M^TRM$. I.e., $M^TRM = F^\\top F$. Then the quadratic inequality constraint can alternatively be expressed as a Second Order Cone Constraint\n$$\\|FX\\|_2 \\le b.$$\nThat is because $X^\\top M^TRMX = X^\\top F^\\top FX = (FX)^\\top(FX) = \\|FX\\|^2_2$.\nAlternatively, if $G$ is the upper triangular Choelsky factor of $R$, the Second Order Cone constraint can be expressed as $$\\|GMX\\|_2 \\le b.$$\n", "topic": "or", "url": "https://or.stackexchange.com/questions/3737"}, {"image": "VQAonline_00048300.png", "question": "How to formulate problems in the language of mathematical programming?", "context": "The question says it all. I am having difficulties formulating general problems (meaning no numbers just variables). When I read the solution, I understand but I can't figure how to formulate myself in new problems. I need some tips or what to look for, some guidance in general. By formulation I mean finding the objective to be minimized/maximized and the constraints. Here is an example with the solution (you can just look at the solution to understand what I mean):\nFormulate the problem of finding an optimal timetable for a school. The timetable is represented by variables $x_{i,j,k} \\in \\{0,1\\}$. The index $i\\in \\{1, ...,6\\}$ stands for the timeslot (with the duration of one lesson) on a day from the morning to the late afternoon. The index $j=\\{1, ...,5\\}$ stands for a weekday. The index $k\\in \\{1, ...,6\\}$ stands for the subject. The index $\\in \\{1,2,3\\}$ stands for the classroom.The decision variable $x_{i,j,k}$ is set equal to 1 if subject $k$ is given in timeslot $i$ on day $j$. Otherwise $x_{i,j,k}$ is set equal to 0. Formulate the mathematical program under the following constraints: (C1) Subject 1,2 and 3 are given 6 timeslots per week, subjects 4, 5 and 6 only 3 timeslots per week. (C2) At every time, a classroom can be used only for one subject. (C3) The same subject cannot be taught in parallel at the same time. (C4) the same subject should not be taught more than two times per day. Moreover, the total number of lessons taught in timeslots 1, 5 and 6 should be minimized.\n(The image has the solution)\n\n", "answer": "This is quite a broad question, and I'm not sure you'll get the answers you need here. To learn how to formulate mathematical optimization problems takes learning and practice, and there's no concrete answer that we can provide on a Q&A site like this. Your best bet is to take a class that teaches or uses optimization modeling, or to learn from a good book.\nHaving said that, there are some relevant Q&As on this site that might help you:\n\nWhat are good reference books for introduction to operations research?\nRecommended books/materials for practical applications of Operations Research in industry\nI've formulated my optimization model; now what?\n\nYou can also check out the questions tagged modeling. Good luck!\n", "topic": "or", "url": "https://or.stackexchange.com/questions/3438"}, {"image": "VQAonline_00048306.png", "question": "When using column generation, can I delete a node with negative reduced cost from my subproblem?", "context": "I am solving a minimization problem with a column generation procedure. The master problem is of the form\n$$\n\\min \\sum_{i\\in \\Omega}c_i \\lambda_i\n$$\nsubject to\n$$\n\\sum_{i\\in \\Omega \\mid v \\in i } \\lambda_i = 1 \\quad \\forall v \\in V\n$$\nColumns $\\lambda_i$ represent paths with nodes belonging to set $V$.\nLet $\\pi_v$ denote the dual variable associated with each constraint. In my subproblem, I am looking for columns that have smallest marginal cost :\n$$\n\\hat{c}_i = c_i - \\sum_{v\\in V, v \\in i} \\pi_v\n$$\nMy question is, if $\\pi_v <0$, can I safely delete node $v$ from the subproblem (as passing through $v$ will increase the global cost) ? Or do I have to keep it, in case passing through $v$ is the only way to access other nodes that together have a potential negative reduced cost ? \nMore generally, is there a strategy to delete useless nodes or edges from the subproblem to speed up computation ?\nIn this article by Francois Soumis et al (a VRP variant solved with column generation), edges and nodes are eliminated heuristically from the sunproblem: [p.15, section 4.3]\n\nThey refer the reader to another paper ([14]) for the details of these \"heuristics,\" but the paper is nowhere to be found. I am interested in such techniques as my problem is also a VRP variant (my subproblem also consists in finding some sort of shortest path).\n", "answer": "If your sub-problem is a shortest path problem on a complete graph, without resource constraints, you can delete vertices which don't decrease the reduced cost. Indeed, for any path containing such a vertex, removing the vertex gives another path which is both feasible for the sub-problem and which corresponds to a master problem column with smaller reduced cost.\nIf the graph is not complete, in general you cannot delete vertices because you might even disconnect the graph. Analogously, if you have resource constraints, removing a vertex might (in the extreme case) disallow the only resource-feasible paths. Whether you can do vertex removal under special circumstances is, I think, problem-dependent.\nFrom my experience, heuristic vertex or arc removal can give good results. Especially if your sub-problem has high complexity, say pseudo-polynomial or worse. Just make sure that you solve the sub-problem exactly (i.e., with no heuristics) at the end of the exploration of each node, to make sure that there are no more negative-reduced-cost columns.\n", "topic": "or", "url": "https://or.stackexchange.com/questions/3993"}, {"image": "VQAonline_00048312.png", "question": "Safety stock for lumpy demand", "context": "\nI have a product for which the monthly demand pattern for the last year looks like the following. The product is ordered only 22 days out of 366 days in batch sizes of 272 kgs. Since there are so many months with 0 demand, the demand distribution is heavily skewed. The mean and standard deviation come out to be 634 and 1167 (rounded off), respectively. I have been using this formula (which is the most widely used, I believe) for calculating the safety stock-\n$$SS = z_\\alpha\\sigma_{LTD} $$\nwhere-\n$$\\sigma^2_{LTD} = \\mu_L\\sigma_D^2$$\nTaking a lead time of 2.8 months and z = 2.33 (99% Service Level), I get SS = 4550 kgs. Now it doesn't make sense to hold 4550 kgs of inventory when the demand is so lumpy. I know that the above formula is best suited for demands that are approximately normally distributed and understandably results in really high SS values. This is a problem I have been facing with multiple products for which there are several months with very little demand or 0 demand. What is the best way to optimize the inventory for items with lumpy/erratic demand and calculate the safety stock and reorder points for the future? \n", "answer": "Just like any other demand distribution (e.g., this one), you want to set the base-stock level ($S$) equal to $F^{-1}(\\alpha)$, where $F(\\cdot)$ is the cdf of the lead-time demand distribution and $\\alpha$ is the desired service level; and then the safety stock is given by $SS = S - \\mu_{LTD}$. \n(In the case of normal demand, as in your question, $F^{-1}(\\alpha) = \\mu_{LTD} + z_\\alpha\\sigma_{LTD}$, so $SS = z_\\alpha\\sigma_{LTD}$, as you said.)\nSo, my recommendation is to fit a distribution $\\hat{F}$ to your demand data and then set $S = \\hat{F}^{-1}(\\alpha)$ and $SS = S - \\mu_{LTD}$. In your case, the demand distribution will have a point mass at 0. \nYou can set $\\mu_{LTD} = L \\mu_D$ and $\\sigma_{LTD}^2 = L\\sigma^2_D$ (where $\\mu_D$ and $\\sigma_D$ are the mean and SD of the demand per period and $L$ is the lead time). But you could also try to estimate these directly from the data, e.g., estimate the mean and SD of the demand over an $L$-period stretch.\nBy the way, there is some literature on this topic. A Google search for \"inventory optimization lumpy demand\" or something similar will turn up some hits that are probably more rigorous and general than my approach.\n", "topic": "or", "url": "https://or.stackexchange.com/questions/4267"}, {"image": "VQAonline_00048583.png", "question": "Water filters: is there always a tradeoff between longevity and safety", "context": "My research thus far has acquainted me with a kind of trade-off: one can try to minimize the amount of contamination (from bacteria/parasites) or one can try to minimize the amount of particulates that could clog the filter. I concede there are many factors to consider, but it essentially boils down to this: \nfaster moving water has less contamination risk from bacteria/viruses as it's constantly being diluted, however faster water tends kick up more sand/silt particulates which over time can pose a legitimate clog hazard to the filter.\nOther things to consider would be depth and temperature. If we go too deep we would encounter contamination from bacteria living on the nutrient rich bottom, but if we go too high, the higher temperatures enable bacteria/parasites to grow. Here is a cool picture illustrating this:\n\nSource\nQuestion\nIs there any way to have the best of both worlds; such that we both minimize the risk of contamination and clogging? I would imagine we'd have to put more effort in to it, so an assessment of whether or not it's worth the effort might be informative too.\n", "answer": "If the water is muddy or there are swimmers, then the usual solution is to prefilter before treating.\nA handkerchief is usually sufficient for filtering out all of the visible swimming bugs and sand/dirt although it won't filter out any of the things that will make you sick. Then you can treat/filter to kill the giardia/bacteria.\nOf course, the precautions necessary depend on the area you are in and how severe the risks are.\n", "topic": "outdoors", "url": "https://outdoors.stackexchange.com/questions/20644"}, {"image": "VQAonline_00048512.png", "question": "Why are common grackles standing on our bird feeders with their heads pointed straight up?", "context": "I live in Northeastern United States, where it's spring at the moment. We keep bird-feeders throughout the year, so those that over-winter always have something to eat. \nAs is usual at this time of year, a massive group of common grackles (Quiscalus quiscula) recently arrived, along with other birds of similar size and color, including starlings and red-winged blackbirds. There were hundreds of them, quite a magnificent sight. Some have already dispersed, especially the red-wings, but a large number will stay until the late fall migration. \nOver the last few weeks, we've been witnessing a behavior we don't remember having seen before. At least five grackles perch near each other on top of a grouping of bird feeders or, less often, on the ground. They stop eating and stare straight up into the sky. Some puff up their bodies and flap their wings. They screech at each other, but it doesn't sound like the usual grackle call we're used to hearing. \nAfter a minute or so, some fly off to join the groups scattered in the yard or trees, and the others will continue eating. When they all leave, it's only for a few minutes, and then it happens again. It could be the same group returning or a different group altogether, as we can't strictly identify them. We've seen it a few times over the course of a few hours on a number of different days. \nAt first we thought they were on the lookout for some kind of predator up high in the trees or in the sky. However, because they only see out of the sides of their eyes, if their heads are straight up they can't be watching the sky, or even the tall branches above them, so we've pretty much ruled out that theory. \nDoes anyone know what this behavior means? \nUnfortunately, they're quite skittish, so every time we try to take a picture, they fly off. They very much resemble the picture below. \n\nPicture Source\n", "answer": "Those grackles are engaged in a courting behavior, most frequently called \u201cbill tilt.\u201d It's also been called \u201chead up\u201d or \u201chead up threat.\u201d While establishing pairs for mating, males vie for the attention of the females. Usually as the birds arrive, a group of males will chase the females. Gradually the males drop off until there are fewer males vying for the attention of each female. Once they settle in at the feeding and nesting areas, the males begin the process of establishing dominance, thus increasing their chance of attracting the specific female who'll become his mate. The males won't do the head tilt unless at least one other male and one female is present. The females may also do the head tilt, but once the pair is established, they both discontinue it. In each group, the male who can tilt his head farthest back and hold it the longest is considered dominant and the other males respect that. \nAlthough the grackles may engage in some physical sparring behavior to establish dominance, much of that is avoided by this behavior. \nFrom page 208 of Stokes Backyard Bird Book: The Complete Guide to Attracting, Identifying and understanding the birds in your back yard.:\n\nWatch the common grackles at your bird feeder. Every once in a while they'll look up as if they were looking at the sky. (They are not really looking at the sky; when birds do that they tilt their head to one side.) This display, called \u201cbill tilt\u201d, is part of their language. It is usually done between two males or two females when they are competing for dominance over a mate or at a feeding site. The display is held for several seconds, and then the birds resume feeding. Bill tilting is especially likely to occur when new birds arrive on the scene. \n\nFrom Pennsylvania State University\n\nGroups of grackles foraging in a common feeding area frequently display dominance and aggression behaviors toward each other. One of these behaviors is the \u201cbill-tilt.\u201d Interacting birds tilt their heads upward and point their bills toward the sky. The more dominant the bird, the more upwardly tilted their heads! \n\nThe ruffling of feathers is also a courting or nesting behavior. \n\nMales put on a display for females by fanning their wings and tail, ruffling feathers and tilting their bills. Females quiver their wings in response. \n Source\n\nWorld-renowned ornithologist, Arthur Cleveland Bent, compiled a large group of studies and visual reports of grackles and their behavior dating back to the late 1800s. The following is a quote from Witmer Stone in 1937, published by Mr. Bent: \n\nAs early as March 5, I have seen evidence of mating and sometimes two males have been in pursuit of a single female, resting near her in the tree tops, where they adopted a curious posture with neck stretched up and bill held vertically. \nAs early as March 13, many of the Grackles are flying in pairs, the male just behind the female and at a slightly lower level. They are noisy, too, about the nest trees and there is a constant chorus of harsh alarm calls, 'chuck, chuck, chuck,' like the sound produced by drawing the side of the tongue away from the teeth, interspersed with an occasional long drawn 'seeek,' these calls being uttered by birds on the wing as well as those that are perching. Then at intervals from a perching male comes the explosive rasping \"song\" 'chu-seeeek' accompanied by the characteristic lifting of the shoulders, spreading of the wings and tail, and swelling up of the entire plumage. \n\nThe sounds are described and Illustrated in Peterson's Field Guide to Bird Sounds of North America\nInterestingly, the specific calls will be used between males and females or males and males while in a group. Much like the head tilt, the male uses it only in the presence of other males. Once he has found a mate, he doesn't use either again, and neither does she.\n", "topic": "outdoors", "url": "https://outdoors.stackexchange.com/questions/15838"}, {"image": "VQAonline_00048524.png", "question": "What is this tree with a spiral shaped trunk in Massachusetts?", "context": "On a recent hike at the Audubon Wildlife Sanctuary in Princeton, Massachusetts, my husband came across this tree. \nThe trunk is twisted in a spiral/curly fashion. It was in a forest of many other trees, but this was the only one that looked like that. \nWhat is the name of the tree? Is that just the way it grows, or an indication of something wrong with it? \n\n", "answer": "I suspect that tree grew with something else twined around it. That something else has meanwhile died or been removed. Bittersweet is a likely candidate for what might have wound around the tree earlier in its life. There is more information on bittersweet in this question.\nI can't tell exactly what type of tree it is without seeing the leaves. However, it is a hardwood. The leaves we can see appear to be maple, but it's hard to tell, and they don't look like they are from the tree in question.\n", "topic": "outdoors", "url": "https://outdoors.stackexchange.com/questions/16867"}, {"image": "VQAonline_00048612.png", "question": "Do golden color pine trees associate with the name of Fluffy?", "context": "I went to a nursery over the weekend and purchased a golden colored pine tree. The nursery was calling them Fluffy Trees (picture below). This tree name really makes me laugh and smile but I am having a hard time believing that is what it is actually called. In fact I went online and found that the name could be Wate's Golden Pine. Would someone be able to identify this tree and confirm if the word Fluffy is appropriately associated with this specie of tree?\n\n", "answer": "What you have there is almost certainly a variety of Chameacyparis lawsoniana. These trees are characterized by the flattened scale-like leaves in a fairly dense branching cluster. Wate's golden pine is a Pinus species, all of which have needles for leaves, like you might see on a classic christmas tree.\nAs to which variety it is - there are lots of them, some of them are yellow (it'll turn green in the middle, retaining yellow on the new foliage), so I can't tell from a photo, and for some it takes some aging to see the true colours and size come through.\nI have never heard of them called \"Fluffy\" trees before, though the foliage is described as fluffy by a few places, based on a quick google search.\n", "topic": "outdoors", "url": "https://outdoors.stackexchange.com/questions/25170"}, {"image": "VQAonline_00048423.png", "question": "is it a good idea to cover a tent with a tarp to protect for heavy rainfall?", "context": "Last summer, I went camping with my family for the first time. We bought a second hand tent, similar to the one on the picture below. We had a great time, but after a rainstorm that went on for 24 hours, the tent started leaking at the seams at the top, causing the rain to drip down at the inside and soaking our sleeping bags. More rain was coming for the next day, so we were forced to pack up our wet tent and return back home a few days earlier than planned.\nBack home, we discovered that the rubbery sealing of the seams had worn out, so we have repaired this, but it might not provide enough protection for heavy rainfall during a next camping trip.\nSo I was thinking to bring a large tarpaulin and use that to cover the tent at the top.\nIs this a good idea ? Or might this get dangerous if heavy wind gets under the tarp ? And should I tighten the tarp directly over the tent (like this) or keep some distance using cords (like this) ?\n\n", "answer": "First of all\nYou might need to look into getting a new impregnation for your second hand tent. This can be done either yourself using sprays or wash-in-products (and in your case a probably huge washing machine) or by giving the tent for re-impregnation to professionals, i.e. your local outdoor/tent supplier (this can be expensive though). In any case it might be a good idea to get their professional opinion on if and how you could re-impregnate the tent.\nAs for the tarp idea\nThis is totally feasible and also quite cheap. The main disadvantage is the added weight and packing space. But looking at the size of your tent it seems you're likely not carrying it for any significant distance, so the additional weight of a tarp (and the needed additional tent pegs/cords) shouldn't be much of a problem.\nI's also possible to just directly tighten the tarp over your tent. This will actually make the entire construction quite a bit more stable I'd guess, as there will be less area of attack for wind. Just make sure to tie the entire thing down well enough and you should have no worries.\nNote:\nAs correctly mentioned by @mattnz draping a tarp directly onto your fly sheet can lower its life expectancy, as friction between the two (cause by wind for example) can degrade the impregnation a bit. But seeing as your fly sheet is not currently rain-proof in the first place I think you'll still be better off with an additional tarp than without: that way at least you should have a working (read: rain-proof) construction for the time being.\n", "topic": "outdoors", "url": "https://outdoors.stackexchange.com/questions/8793"}, {"image": "VQAonline_00048632.png", "question": "What's inside this fishing reel?", "context": "I saw a teardown of a reel and noticed a part inside the spool:\n\nhttps://youtu.be/_NKVN9fRD4k?t=327\nDoes anyone know what the spring does?\n", "answer": "The metal tab is called the click tongue, and the spring holds the click tongue against a sprocket on the reel spindle. Together they are responsible for the clicking sound a reel makes when line is pulled off against the drag, for example, when fighting a large fish. If a fish removes line rapidly, the individual clicks sound more like a continuous whine, a coveted sound to anglers known as a screaming drag.\nThere are other mechanisms in use in some reels, but I've seen this on Penn reels before. This image search link shows a number of variations.\n", "topic": "outdoors", "url": "https://outdoors.stackexchange.com/questions/28057"}, {"image": "VQAonline_00048473.png", "question": "Are these tracks in the mud tortoise/turtle tracks?", "context": "I was having a nice evening walk near one of the nature reservations in Andover MA, close to Tewksbury, when I've found such a footprint. I'm really sorry for image quality, though. So it's a wet, bog-like, piece of ground, two lanes of deep small holes (probably the actual footprints) - the distance between them is like 25-30 centimeters (10-12 inches) - and the ground in the middle looks like something was dragged. In the top left corner of a picture the ground seems to be pushed below, and the hole is filled with water. \nI'm not from US, and have almost no idea about Massachusetts fauna, but to me it looks like a tortoise. It was lying on the ground, thinking it's slow thoughts - that's how the hole filled with water appeared - then it decided to do some action, got up and moved, dragging it's plastron upon the ground.\nThat's the best idea I have. But it should be pretty big tortoise, 10-12 inches wide. I wonder if there are any in MA. Does anyone recognize this footprint?\n\n", "answer": "This does look like some sort of turtle. Snapping turtles can easily get to the size you mention, and are fairly common. The picture doesn't give the impression the tracks are really 10 inches apart. They seem narrower than that, which would open the possibility for a number of other turtles. Box turtles get to a reasonable size, and then there are a list of other more rare possibilities.\n", "topic": "outdoors", "url": "https://outdoors.stackexchange.com/questions/13067"}, {"image": "VQAonline_00048642.png", "question": "How can my toddler learn to stay under the blanket at night?", "context": "My 2yo son slept in a sleeping bag (see image below) until around 1\u00bd years old, then he didn't like that anymore so we switched to using a blanket. He moves around so much while sleeping that the blanket usually isn't covering him at all. This hasn't been a problem so far during summer, but now the cold season is coming and we think he'll get too cold at night. He's already rather cold in the mornings -- with cold hands and cold feet -- so the current mode of operation won't do.\nWe're not tucking him in \"English style\" with the blanket going under the mattress, but average \"European style\" where it's just draped over him. He's very active and dislikes being restrained, so \"English style\" is not an option.\nI've learned to stay underneath my blanket during sleep, but I have no recollection how I learned it -- can we somehow \"teach\" him, or does it only come naturally with age? How can we keep him under the blanket? \nWhat we're already trying:\n\nWe are re-covering him several times during the evening until we go to bed ourselves, and sometimes again during the night if we're up for any reason. \nWe are dressing him in warmer pajamas and socks to keep him from getting cold at night, but we can't comfortably dress him warm enough in winter. If we dress him warmly (so that he wouldn't become too cold without the blanket) then the blanket makes him too warm and he wakes in a sweat, so we avoid that.\nWe are trying to find a sleeping bag that fits his current size (92cm) but seem to be rare in these sizes; apparently they're only popular for infants but not so for toddlers. Even if we find one, there's a risk that he'll reject it, just like he ended up doing half a year ago.\n\n\n", "answer": "Until she was 5, my daughter would sleep on top of her blanket with her pillow on her feet. She knew about sleeping under the blankets (having seen us doing it), but just wasn't interested. \nDress them warmly and let them figure it out on their own.\n", "topic": "parenting", "url": "https://parenting.stackexchange.com/questions/3098"}, {"image": "VQAonline_00048652.png", "question": "How long does breast milk last after taken from the fridge?", "context": "There's a lot of information available about how long pumped breast milk can be kept for, generally citing different temperature ranges.\nFrom the CDC (though the same reference is also cited by the Canadian Pediatric Society): \n\nIt's cited that milk thawed from the freezer should be used within 24 hours in the fridge, or 1 hour at room temperature.\nThe Mayo Clinic has an article saying \"Other studies have shown that refrigeration beyond two days might reduce the bacteria-killing properties of breast milk\", but no references to where this comes from or any further information.\nMy wife is exclusively pumping, and so there is a lot of milk cycling in and out of the fridge. She is always concerned about using milk taken out of the fridge (generally +/- 1 day old) within an hour or two. We often talk milk out (and let it warm up to room temperature), only to have baby fall back asleep. Is there a basis to how long fridge-temperature milk (stored at 3 degrees C for a day or two) can be out of the fridge for before it has to be used/tossed?\n", "answer": "The Short Answer\nBreast milk can be safely consumed if it has been at room temperature for no more than 6 to 8 hours total, which includes all of the time the milk spends above refrigerated temperature. So, if you heat it up to room temperature, then put it back in the fridge after half a period of time, you to account for: \n\nThe time it takes to heat up the milk\nThe time the milk spends outside of the fridge\nThe time it takes for the milk to cool back down, once back inside the fridge. Depending on the initial temp, the bottle size, and the fridge temp, this could take another 30+ minutes.\n\nThe longer the breast milk stays above refrigerated temperatures, the less time it will be viable in the fridge. Guidelines say that milk in the fridge can last 5-8 days, but that time will be considerably less if the milk isn't properly placed in the refrigerator immediately after pumping. Conversely, the longer the milk stays in the fridge, the less time it will be viable at room temperature. Continue reading for more explanation and details.\nThe Long Answer\nIn food service, temperatures that are ideal for breeding micro-organisms are called the Danger Zone or something similar. Different foods have different amounts of time they can spend in the danger zone based on the types of bacteria/organisms that grow on/in them. \nOnce a food has been taken into the danger zone, you also shorten its refrigerated expiration date. While fresh breast milk that's immediately put in the fridge may last 5-8 days, a bottle that's spent 2 hours at room temp may only last 3-4 days instead.\nI don't think you'll find any hard data about how many more days a bottle will last in the fridge based on how long it spent at room temperature. That would have to be some very controlled experiments that, in the end, wouldn't serve much usefulness. \nTo answer your question, we must make some inferences with the data we do have.\nFor instance, we also know that if you freeze the expressed milk at all, then after you thaw it out it must be consumed within 24 hours or thrown out. \nSo, we know:\n\nKeeping milk at higher temperatures decreases how long it will last at lower temperatures\nKeeping milk at lower temperatures does not necessarily preserve the initial maximum safe time at higher temperatures\n\nPutting that together, and some general assumptions:\nThe longer the milk has been in the fridge, the sooner it will go bad. If it's only been refrigerator for a day, then it should still be okay for about 6 hours. If it's been in there for 5 days, I'd only give it 2 hours.\nYour wife shouldn't have to worry about using day-old milk within two hours. I'd be surprised if you didn't get at least four hours. However, if the room/milk temperature is greater than 77 F/25 C, then it will last significantly less than the noted 6 hours.\nOther factors to keep in mind that will affect how long milk last:\n\nThe cleanliness of the nipple area before/during pumping\nHow well sanitized the storage containers are if they're reusable\nHow well sanitized and maintained the pieces of the pump that contact the milk are\nWhether or not you store it in the back of the fridge (because the front meets warmer air every time the fridge door is open)\nHow much you warm up the milk, if \"serving\" it above room temperature\n\nPersonal Experience\nWe never had problems with refrigerated milk spoiling within 1-2 hours. When my son was under 1 year old we'd often leave a bottle in his crib with him at night. The ones he didn't finish would still be okay 4-6 hours later. We usually didn't give it to him again, but if he woke up and decided to finish the bottle before we had a chance to take it there were never any problems. Rarely, if ever, did the milk stay in the fridge for longer than 2 days.\nHowever, we did have times when milk stored in an insulated cooler bag with ice packs would spoil much faster than we expected, which made some of our longer car trips surprisingly more difficult.\n", "topic": "parenting", "url": "https://parenting.stackexchange.com/questions/18263"}, {"image": "VQAonline_00048649.png", "question": "When is a child ready to start playing with bricks?", "context": "Just out of curiosity: at what age is a child ready to play with its first bricks? The light, large, colored wooden ones, like these below?\n\nBy playing I mean actually creating something of of them: a structure of sorts, animals, corridors, or whatever the imagination suggests.\n", "answer": "When they can grasp them is the short answer.\nThere are various stages of play but as a first starting point as soon as they have the coordination to pick them up one by one to begin exploring colour, shape texture and (of course) taste!\nCreating structures requires the ability to do imaginative play and is a considerably later developmental stage so I wouldn't expect to see that until much later around 3 years or so based on articles from the LEGO\u00ae DUPLO\u00ae page.\n", "topic": "parenting", "url": "https://parenting.stackexchange.com/questions/10882"}, {"image": "VQAonline_00048650.png", "question": "food feeders can be used for baby lead weaning?", "context": "Can food feeders be used as first steps to baby lead weaning (6-7 m.o baby)? Or there is really no connection?\nIf not, is there any benefit in food feeders over pur\u00e9ed foods? \nI mean food feeders like Kidsme Feeder or Munchkin Fresh food feeder\n\n\nThis award-winning product allows your baby to enjoy lots of delicious foods without the choking risk. Simply put a piece of fruit, vegetables or meat into the mesh bag and snap shut. Your baby can chew, suck and enjoy all the whole food goodness and taste, with only the tiniest, digestible pieces coming through. Your baby gets great flavor and you get great peace of mind.\n\n", "answer": "I'm not an expert (we did BLW but not in a terribly organized manner), but I would tend to say, not very much. Baby-led weaning, from what I know, is mostly about teaching the baby to learn how to eat; and while feeders are nice to use for teething (which is what we used them for), they don't really teach the baby how to eat very well. \nTo the extent that they do help with baby led weaning, they probably are most useful early on when the baby is first learning how to move the gums/jaw to chew as an alternative to soft foods. Once that has been learned, they don't really offer very much in terms of help in that regard. In addition, it would be better to learn to chew on bananas or other soft foods so that the baby can get used to swallowing chewed food.\n", "topic": "parenting", "url": "https://parenting.stackexchange.com/questions/12069"}, {"image": "VQAonline_00048656.png", "question": "Toddler's head falling back in car seat", "context": "My 2.5 year old is in a forward-facing Maxi Cosi Pria 85 carseat. The problem is, if he falls asleep while we're driving somewhere, his head falls back. Every site that mentions bad sleeping positions in the carseat fails to mention what to do for heads that fall BACK. Even if I stop and reposition him, his head goes right back to where it was. His face is parallel to the roof of the car. I've also tried keeping it to the most reclined position for forward-facing but that does not help.\nI would like to know if this is safe and if not, what can be done about it. \nEDIT: I added a picture to show what I'm talking about. I tried adjusting the back part so each picture shows it in a different position. It happens in both positions. The strap is tight and the chest clip is in proper position.\n", "answer": "Without seeing it's hard to say anything for sure, but based on how every car seat I have encountered is constructed I am inclined to ask whether your boy's belt straps are sufficiently tight. \nFor his head to be that far back the seat would either need to\n\nNot have sufficient padding/bracing behind his head, which is a problem in and of itself\nBe letting his chest move forward and away from the back of the seat so that his head can move back that far\nHave a lot of padding around his torso that doesn't exist behind his head.\n\nThe last could be possible if you're strapping him in while still wearing a very puffy coat (but in August?) but the recommendations are that you don't leave kids in thick coats when strapping them down anyway, precisely because then you can't tighten the straps sufficiently tight.\nMake sure he's in tight enough and that the chest strap/buckle is high enough up on his chest to keep it firmly back against the seat.\n", "topic": "parenting", "url": "https://parenting.stackexchange.com/questions/26687"}, {"image": "VQAonline_00048658.png", "question": "Is it ok to play a computer game with a 3yo", "context": "My 3yo daughter has recently noticed me playing Craft the World, a game which is (mostly) about dwarves working and building their home, with hardly any violence. She wanted to know what it's about, I started explaining, and after a while we've started playing together.\n\nShe really enjoyed building dwarves their home, furnishing it, dressing dwarves and giving them tools to work better, etc. etc.\nSo far, over the last week or so, we've played 3 or 4 times for 15 minutes max each time. I'm doing most of the clicking but decisions are made together. I encouraged her to use the mouse, but with little success.\nI'm having doubts about this. While I'm against giving a child a phone or tablet to get rid of them, I didn't really feel bad playing this game with her. I had the opportunity to talk about some things she never heard about before, explain with a good example that one has to work to have something.\nShould I let her play with me at all? Do you think that this activity, as I have described it, is acceptable for a 3 yo?\nfour-years-later edit, 2020.08.22\nOur daughter did not develop an unhealthy attachment to computer games. She is now 7 and is allowed to play age-appropriate nintendo DS, tablet or PS4 games, but on an ask-first basis. She does ask a few days a week and plays 30-45 min at a time. Whenever we ask her to stop gaming, she does it without protest - we do agree to \"let me finish it\" or \"one more minute\" and she does not abuse the permission. I'll try to write in a few years, because we apply the same strategy to our now 3-yo son.\n", "answer": "The primary reason \"screen time\" is considered bad for children is that, even when somewhat interactive as games are, it does not stimulate their brain as much as other things do. Kids need all sorts of different activities - physical ones to help develop coordination and strength, social ones to help develop social awareness and social skills, they need to talk to people and hear them talk in order to develop vocabulary and comprehension.\nPlaying a computer game with you, in a highly interactive way, is to me comparable to reading with a child. If it is a way for you to give her attention and to give you a chance to bond, then it's a perfectly fine option in my opinion, so long as it is reasonably limited; it shouldn't be the main thing she does, but a half hour or so won't be a problem. It's a great way to help her learn problem solving skills, and if she is moving the mouse she's also developing her fine motor control (as opposed to a tablet which tends towards more simple, large areas to select).\nThe thing to be wary about is the game turning into the only thing she wants to do. You'll want to have a set amount of time that it can be played per day, and stick to that. You'll probably also want to limit yourself in the same fashion, and do so obviously, so that the child understands this is something you need to learn to do, as opposed to this being a hypocritical \"do what I say not what I do\" type of thing. If you're playing the game 4 hours a day and she's only allowed 1, it'll be hard for her to understand why.\nI play Pok\u00e9mon with my five year old, for example, and while he enjoys it quite a lot, it can be a struggle to limit the time effectively. I only play when he's playing (or when he's asleep); and the asleep-time I limit sufficiently so that we are at roughly the same stage in the game (he often plays when I don't). This has worked fairly well, and I don't think it's had a negative effect on him; but it has led to him wanting to play constantly, and needing to keep hard limits on his play, so be aware of that.\nAll in all, though, I think gaming as a social activity is a great thing for even younger children, as long as it's in moderation and age-appropriate of course. Getting your undivided attention, and getting to learn things with you, is a great thing for your daughter.\n", "topic": "parenting", "url": "https://parenting.stackexchange.com/questions/28264"}, {"image": "VQAonline_00048661.png", "question": "The teacher erased 3 pages of my 5 year old's completed homework", "context": "I have a 5-year-old smart child. He does his homework as neat as he can. Yesterday, he stayed up for an hour doing 3 pages of homework as good as he can and today when checking his copy-book, I found that his teacher had erased all the 3 pages and wrote a note on each page (do your homework neatly). \nIs this the right way of checking a small kid's homework, and if not, how am I supposed to act?\n\nI attached one of the pages.\n\nIn response to comments: The teacher replied that he has to write on the lines not off lines. I went to the school and tried to meet her, but they said it's not allowed during school time, so I had a meeting with the coordinator and the school principal who both insisted that this is the right way of correcting the homework and said that it is better than marking all the homework with red and crossing it out for him. I certainly believe that erasing a kid's homework will cause a negative impact which already happened, he cried when he found it erased and asked \"why do I have to do it all again?\"\n", "answer": "This teacher is carefully producing a future math-hater. I've worked with enough high school students to know.\nI've also worked with enough people in personal counseling to know that whether you care to face it or not, this sort of work is NOT accidental. This is a deliberate, even if unconscious, effort to sabotage your child's learning.\nAt that age the MOST important asset your child has school-wise is a love of learning. Nothing can substitute for that. A teacher who will destroy a love of learning in pursuit of a perfectly written piece of homework\u2014entirely invalidating the careful hours your son put in to do a good job\u2014is valuing form over content to the point of insanity. One could also say it is valuing mechanics over purpose; whatever you call it, it is totally counter-productive.\nPersonally, I value my son highly, and I respect his rights and dignity as a person. I would remove him immediately from the influence and power of this teacher.\n\nWhen I was very young (about 8 years old), my parents sent me to a new school. It was supposed to be an excellent school, with an excellent curriculum, etc., etc. I only went for a semester, and then (to my relief) I went to another, smaller school where I enjoyed myself much more and where, even though the curriculum wasn't as jammed with \"high value subjects,\" I learned a lot more a lot faster. And had more fun. I subsequently finished 4th, 5th and 6th grades in the same year, and went on to graduate High School (12 grades) by the age of 13.\nYears later, talking to my mother, I found out why she had taken me out of that prestigious new school after only one semester. It was because she found out I didn't like going there. That was the only reason.\n(Thank you, mom.)\n\nYou know your child. You care more for your child than anyone else on the planet, and you know what is best for your child. Don't let anyone tell you otherwise. If you don't like the approach used by that school, don't let your child be subjected to it.\n", "topic": "parenting", "url": "https://parenting.stackexchange.com/questions/30013"}, {"image": "VQAonline_00048772.png", "question": "Is this kind of puppy's stool normal?", "context": "Is it normal that 13 days old puppy's stool is liquid and with foam (little bubbles) ? \nToday I notice that one of the 4 puppies made that kind of stool and at the same time the puppy screamed loudly like if someone were hurting the puppy. So the puppy felt pain while doing it.\nWhy?\nShould I take the puppy to the vet? Or is something passenger?\n\n \n\n", "answer": "I never had contact with such young puppy as the fastest you can get one is 8(or 7) weeks old, but if it's stool is different from the rest of puppies, I would contact vet, just because of how young the dog is. \nBut for older dog, that kind of stool is not normal. My puppy got that kind of runny stool after 3rd dose of vaccines, but we gave him overcooked rice, with a little bit of coal, and next day, overcooked rice,chicken and carrot, and it went back to normal. Though he was 12 weeks old. And I'm not sure what kind of food very young puppies can eat.\n", "topic": "pets", "url": "https://pets.stackexchange.com/questions/20911"}, {"image": "VQAonline_00048800.png", "question": "Can cats identifiy specific tones/frequencies?", "context": "I have a cat who lost her eyesight due to blood-pressure complications when she was about 6 years old (she is 8 now). She adapted very well to living without eyesight, to the point where it would almost seem as if she wasn't blind at all.\nShe obviously uses her hearing and sense of smell to navigate to food, water and toilet successfully, but she is currently sick with a flu and has lost her sense of smell, and to my understanding some loss of her hearing too (blockages from flu). Because of this, she is having some difficulty finding things.\nMy question is - can cats identify specific tones or frequencies generated by something like an Arduino & piezo buzzer, and then associate that tone to something, such as food?\nI'm looking to create beacon-like devices which emit a specific frequency for a certain duration that a blind cat can potentially use as a reverse-echolocator, and just wanted to validate whether this is probably a waste of time, or something worth exploring?\n\nUpdate (1/28/2020):\nThank you for all your suggestions. Unfortunately before I could complete a beacon POC, she had to be put to sleep due to a number of pre-existing issues that having flu/not eating catalyzed.\nIf you're reading this, and you have a blind cat, please ensure that they can wayfind without their sense of smell should they get sick.\n\nRIP Katja, 2011-2020.\n", "answer": "Short answer: yes.\nWhat you intend to do is Classical Conditioning and works just like clicker training. Your cat will probably need a few days to learn that a certain frequency means food, but it will surely learn.\nLong answer:\nI really like your idea, but I think you need to adapt it to the physiology and needs of a cat. Cats hear better than humans, especially high frequencies. A piezo buzzer emitting a high beep in the \"normal\" loudness for a human might be uncomfortably loud for a cat. Please keep in mind that cats hunt rodents by listening to their faint sounds in the grass and even underground.\nAs a rule of thump, I would try the following configuration:\n\nMake the piezo buzzer as quiet as possible\nChoose a frequency you can hear\nIf you cannot make it very quiet, go for a lower frequency and emit the sound for only 100 - 200 milliseconds\nA simple piece of soft foam or thick fabric wrapped around the piezo buzzer can help reducing the loudness as well.\nIf the sound beacon is close to the sleeping place of your cat, either tune the loudness down during late night or deactivate it if you cannot tune it very quiet.\n\nStart with only 1 beacon and see how your cat reacts. If it seems to work and you want to add another beacon for another place, choose a frequency you can distinguish from the first one. Your cat will be able to distinguish it as well.\nSince you simply want to mark certain locations with the sounds, it will be enough to place the beacons in the intended location and let your cat get used to them. Hiding them between walls or behind furniture might confuse your cat because of echos.\n", "topic": "pets", "url": "https://pets.stackexchange.com/questions/26883"}, {"image": "VQAonline_00048795.png", "question": "Cat looks wounded, what is wrong with it?", "context": "\nI found this stray cat outside my home with yellowish liquid running down its eyes. I'm not sure if it's pus from an infection or a bad wound from a fight. Tried to offer it some food but it didn't seem interested.\nDoes anybody have an idea what's afflicting it? I'd like to know if it is serious and what can I do immediately to help it. Thanks\n", "answer": "This is definitely serious, looks like it could be a major infection and there are definite signs of fleas. Bring it to the vet if you can. \n", "topic": "pets", "url": "https://pets.stackexchange.com/questions/26372"}, {"image": "VQAonline_00048788.png", "question": "Found this skink in my tomato plant bucket. Is he trapped? Or could he leave if he wanted?", "context": "I've been growing some tomatoes in a 5 gallon bucket. About 4 days ago, I found this skink sitting there in the dirt. .\nSince he's been there, he's been happily just burrowing around in the dirt and (probably) eating some bugs. He's totally welcome to stay as long as he wants, I think he's pretty neat. My concern is, I read skinks aren't that great at climbing, unlike geckos. If he wanted to leave the bucket, would he be able to? Or do I need to catch him and set him free?\n", "answer": "I think it needs some help getting out of there,maybe you can put some thing in there for it to climb on.\nThey do an important job eating bugs and snails protecting your tomato plant(s).\nTry not to use any pestecides where it lives.\nIt is best if you try to avoid catching him it can have a negative effect on him some of them are known to drop their tail if they get scared.I do not know the exact type you have there https://en.wikipedia.org/wiki/Skink\n", "topic": "pets", "url": "https://pets.stackexchange.com/questions/24273"}, {"image": "VQAonline_00048692.png", "question": "Can I locate my cats' feeding maze close to their litter-box?", "context": "I have two cats. We feed them wet food daily (in the kitchen far from the litter-box,) and we provide them free access to dry kibble all day long.\nThe litter-box and the feeding zone are located in the \"no-dog\" part of the house and are in the same room.\nWe've never had any (apparent) problem with that setup: both cats are using the litter-box with no issue, and both cats are eating a few times a day.\nI know this is not the ideal situation, but we've never had any problem, and I'm trying to find a balance between the theoretical ideal setup (2+1 litter-boxes, feeding in another room, etc.). Should I worry about that?\nI'm also planning to change their bowl with a feeding maze (see picture). Would it be acceptable for them to \"play\" close to their litterbox? Should I just try and, if they use it, conclude that it is OK with them?\n\n", "answer": "Many cats are very fussy about their litter box and food proximity, bearing in mind that their sense of smell is substantially better than ours, but not all cats are that particularly bothered by it. So, short answer is \"yes\" you can do that if the cat(s) tolerate it. Some things to consider:\n\nKeep the boxes clean, scooped daily (or more frequent) and washed weekly.\nKeep the boxes in good shape, scratched/old boxes are more likely to hold odors in them.\nKeep an eye on the feeding habits. If they start to show signs of food avoidance, look to separate the food from the litter boxes. Mind you, if it's just one cat, then this may be an indication of something else.\nKeep an eye on litter usage. If they start going outside of the boxes, away from the food, then this a sign that they don't like this arrangement.\n\nAnecdotally, I had a cat many years ago where the food and litter box were side by side and he never fussed about it, always ate his food, always used the box. On the other hand, our current cats won't tolerate that, at all, so we have the boxes and food in different rooms. It's going to depend on the cat.\n", "topic": "pets", "url": "https://pets.stackexchange.com/questions/3037"}, {"image": "VQAonline_00048798.png", "question": "Something is wrong with my cockatiel - she is missing feathers", "context": "I have two cockatiels and four budgies. Since I got my second cockatiel, I noticed that she doesn't have feathers under her crest. I thought that the reason for it is that she is really submissive and lets herself to be pecked all the time. This was far from true, because now she got worse (see picture). She is 4 years old.\nPlease help me, I need some group knowledge about this topic, because I have never even encountered such issue with my parrots so far, so I really don't know what to do next.\n\nThe bald spot is reddened by her blood - others pecked the feathers out. But I noticed black base of quill (maybe blood in it?). \n", "answer": "There seem to be basically 2 possibilities:\n\nAnother bird pecks her and plucks her feathers\nShe scratches or plucks herself, which can indicate anything from stress or boredom to malnutrition or parasites.\n\nThe first thing to do is getting the bird examined by a vet to rule out any skin condition, malnutrition, allergy, fungal infection or parasites.\nThen you need to observe what or who caused the wound. If she plucks herself and the behavior started before you adopted her, she might have been very stressed by her former living conditions. Since it's gotten worse since you adopted her, the change in her environment could have intensified her plucking. Since self-plucking is unfortunately quite common in birds, you can find many resources online, like this video, this article on PetMD or this article on petco and many more.\nIf another birds plucks her, you'll need to separate them for a while. But this honestly sounds and looks a lot like self-plucking.\n", "topic": "pets", "url": "https://pets.stackexchange.com/questions/26806"}, {"image": "VQAonline_00048761.png", "question": "Hostile hamster - Biting hands whilst holding", "context": "I recently got a Syrian hamster at the age of seven weeks. Now, at the age of three months, he is not curiously nibbling me, but actually biting until I am bleeding (three times already). What am I supposed to do?\nBackground story: We had a rather unpleasant first day: he desperately attempted to escape his cardboard box we got him in (see also the opening scene of Jurassic Park). Later that night I tried to reconcile and have him on my hand. Unfortunately he jumped of and we had a chase. Short story, lot of poo, lot of screaming and a trauma for the both of us.\n\nWeeks passed and things got better. I gave him food out of my hand, gave him treats after being in the big ball and all, and I could even pet him while he was eating. This was the moment where I thought I could try to hold him (with one hand covering him a bit), so I could have him in my hands out of the cage. But this is the point where it all changed. He became very vicious when I held him and bit me very hard. From that point on, I couldn't even put my hand in the cage without the hamster biting me.\nIs there a chance that he stops this behavior, or is the possibility to bond gone?\n", "answer": "I wouldn't say the possibility of bond is gone. I hate to say this, because I want to be as positive as possible, but when I worked at a pet store there were 2 things we for sure ALWAYS recommended when taking a hamster home: 1) don't hold him on the first night, so he/she doesn't associate you with the stress or become more stressed and 2) don't hand feed him because he'll associate your hand with food, nibbling and biting, and may expect the food- as he would expect food in his bowl- since that's where he's seen food before. BUT, with all that's said I think you can try a fresh start.\nMaybe give him a week with little to no handling, give him a break, in a sense. Move things around in his cage so he has a change in scenery and routine, so he doesn't expect his environment to always be the same. Next time you do handle him, maybe try sitting on the floor with him with your legs forming a border and let him explore you and your scent without your hands as part of the equation (except to take him out and put him down) but try to let him know that you two can co-exist without food in hand and so he's free to move around. Anytime you hold him or interact, don't push- simply let it go if he squirms, it's a sign he's had enough and will likely bite if you fight it, just gently place him back in his cage. After testing the floor theory maybe let him on your lap and explore other parts of you- one of my hammies I had, loved to just chill in my hoodie pocket or in the hood. A dark hiding place makes them feel safe.\nWhen you change his bedding and give him fresh food and water, do it with him outside of the cage, in a separate box and in between feedings remove the bowl and fill it outside of the cage and place it back instead of putting your hand in and the food coming from you.\nAfter some time of doing this regularly, from time to time just rest your hand in his cage and see how he takes it. If he runs from your hand, don't chase him, just leave it still so it's non-threatening, and encourage him to explore without picking him up, after a few times of doing this you can pick him up by keeping your hand flat and gently lifting up when he gets on your hand. Keep these visits brief and eventually you'll be able to make each time a little longer until he's used to you taking him out and NOT getting food until he's back in the cage.\nAlways wash your hands well before handling him, and before it becomes a habit try Keeping a bottle of hand sanitizer near the cage as a reminder to clean off smells and residue from your day. They have a great sense of smell and you want to keep smells from food, other environments and other people off your hands so he doesn't smell you and get freaked out by a threatening or Yummy smell.\nGood luck and update us on how things go in the coming weeks.\nAlso, there will be more bites in the future, when he does, put him back in the cage and wait until the next day to try again, to give him time to calm down and disassociate.\n", "topic": "pets", "url": "https://pets.stackexchange.com/questions/18707"}, {"image": "VQAonline_00048886.png", "question": "Authors who think Culture is power?", "context": "The USA is leading country in military expenditure in the world. To match what the USA spends you have to add up the budget of the next 11 countries on the list combined. ref here \nThe USA is not just the most militaristic nation on earth but also the first world cultural superpower. The whole world is flooded American films, music, news, fashion, architecture, foods, etc. Even American English is the international language nowadays. In most mainstream literature and art this country is generally portrayed as the hero country of the world givers of freedom and capitalistic prosperity. It's rare to find cases in which the Americans are portrayed as villains not even in non-American films. \nIn history American culture (mainly Hollywood) has tried to offer an unrealistic vision of the world through art and propaganda which antagonises those considered enemies or apathetic to their Anglo-capitalistic agenda whose distortion of reality still remains up until today. ex. (Native Indians, African slavery, Socialism, Communism, Spanish-American war, The Japanese during WWII, the Vietnam war, The Russians, The Iraqis, The Iranians, etc.)\nThe relationship between culture and military power is nothing new. The Romans took a great deal of culture from the Greeks and the Roman Empire influenced culturally not just their colonies but neighbouring areas. \nAre there any books, films authors etc who ponder on how culture is power and vice-versa?\n\n\nSid Meier's Civilization\u00ae is an strategy game about world domination. In this game you can also achieve a cultural victory\n\n", "answer": "America's Danger of Imperial Overstretch Bloomberg News. https://www.bloomberg.com/opinion/articles/2016-07-13/america-s-in-danger-of-imperial-overstretch\nBooks by the American historian Chalmers Johnson. http://americanempireproject.com/authors/chalmers-johnson/\n", "topic": "philosophy", "url": "https://philosophy.stackexchange.com/questions/65208"}, {"image": "VQAonline_00048870.png", "question": "Can inductive arguments be made in first order logic and, if not, why not?", "context": "After reading a question by rus9384 Why is faulty generalization called an informal fallacy? I wondered whether induction can be part of any argument in first order logic (FOL). \nrus9384 symbolized an example of an inductive argument: \u2203x: F(x) \u2234 \u2200x: F(x)\nGiven the rules for existential elimination and universal introduction, I don't think this argument can even get started, but I may be wrong.\nClearly, we can go in the opposite direction. Here is a proof of going from a universally quantified sentence to an existentially quantified one: \u2200x: F(x) \u2234 \u2203x: F(x)\n\nI am not concerned with whether induction is a fallacy, formal or informal, but to what extent induction arguments can be symbolized or made at all in FOL. \nQUESTION: Can inductive arguments be made in FOL and, if not, why not? If they can, an example of their use would be helpful.\n\nReference\nKevin Klement's JavaScript/PHP Fitch-style natural deduction proof editor and checker http://proofs.openlogicproject.org/\n", "answer": "You can formalize inductive logic, but it is usually though to require the introduction of an ambient Bayesian probability theory. The reason first-order logic alone doesn't work is because first-order logic examines whether an argument is valid or invalid. An argument is valid if and only if the truth of the premises guarantees the truth of the conclusion. By contrast, an inductive argument provides probable support for the conclusion. To evaluate inductive arguments you need a way to evaluate the degree of support a set of premises provide for the conclusion. First-order logic makes no such provision. Hence to formalize inductive logic probability theory is used. Intuitively, what you want to know is how probable a state of affairs P is given the premises. \nAs a corollary, the argument form you cite (\u2203x: F(x) \u2234 \u2200x: F(x)) does not really capture the form of inductive reasoning. First, the existential quantification requires only one entity that is F to be satisfied. Almost no inductive arguments have that form, and usually involve a large number of observations. In fact, in the sciences, inductive arguments typically make testable hypotheses relative to background hypotheses from other scientific domains (This, by the way, is another reason first-order logic cannot really \"formalize\" inductive reasoning adequately). Hence the form you list here is at best a highly idealized way of thinking about inductive reasoning that is so divorced from inductive reasoning in practice as to be useless. Secondly, inductive arguments most often have the form of predictions about what our future observations will likely be, and don't require a universal quantification over a domain. For example, if you argue, \"I've tasted thousands of lemons and every single one has been sour. Therefore, the very next one I taste will likely be sour.\" This is a perfectly cogent inductive argument that does not require a universal quantification over a domain. \nCheck out the following page if you want to know more: https://plato.stanford.edu/entries/logic-inductive/\n", "topic": "philosophy", "url": "https://philosophy.stackexchange.com/questions/54112"}, {"image": "VQAonline_00048884.png", "question": "What are the conditions for RAA?", "context": "My textbook states that:\n\nIn this case, however, what about situations where we can get Q ^ ~Q (sorry, unfamiliar with this formatting) without depending on P? For instance, the proof of EFQ:\n1 (1) P A \n2 (2) ~P A \n3 (3) ~Q A \n1,2 (4) P ^ ~P 1^2 \n1,2 (5) ~~Q 3,4 RAA \n1,2 (6) Q 5 DN \n\nWhy isn't it a problem that line 5 does not depend on line 3, when it is the negation of line 3 that we are proving? If we take the above definition of RAA to be true, don't we need 3 to be a dependency of P ^ ~P?\n", "answer": "It only seems like a problem if we think the set \u0393 is consistent, in which case we feel like it's P which \"makes\" the premises inconsistent, and so must be used in the proof. But if \u0393 is already inconsistent, then we don't need to use P at all. Think about it in cases:\nIf \u0393 is consistent, and \u0393, P \u22a2 Q \u2227 \u00acQ, then {\u0393, P} is inconsistent, so that \u0393 \u22a2 \u00acP\nIf, on the other hand, \u0393 is already inconsistent (as in your example proof), then \u0393 \u22a2 \u00acP anyways (since inconsistent sets of statements can prove anything).\n", "topic": "philosophy", "url": "https://philosophy.stackexchange.com/questions/63571"}, {"image": "VQAonline_00048836.png", "question": "Logical Consequence and Tarski's World", "context": "I found this question in an old exam paper. I have no idea what is expected from a question like this.\nConsider the Tarski's world below. This world illustrates that \u00ac\u2203xTet(x) is NOT a logical consequence of the premises below. Explain why/\n1. \u2200x\u2200y [LeftOf(x, y] \u2192 Larger(x, y)]\n2. \u2200x\u2200y [Smaller(y, x) \u2192 (Cube(x) \u2227 Dodec(y))]\n\n\nThis is worth 6 marks.\nI think the obvious things are that you cannot infer \u00ac\u2203xTet(x) from the premisses.\nMaybe I just don't really understand the question\n", "answer": "In order to show that the conclusion is not a logical consequence of the premises, we have to find a counter-example, i.e. a \"world\" were the premsies are ture and the conclusion is not.\nAfter having checked that the two premises are satisfied in the \"world\" depicted, we can conlude that \u00ac\u2203xTet(x) it is not true, simply because there are two tetrahedra.\n", "topic": "philosophy", "url": "https://philosophy.stackexchange.com/questions/17857"}, {"image": "VQAonline_00048858.png", "question": "Why does negation reverse order of strength?", "context": "\n1. It is the negation that does the trick. Think of a \"condition\" as a restriction on the class of things that satisfy it, the stronger the restriction the narrower the class. Normally NC are weaker than SC, but [2.] negation always reverses the order of strength: [End of 2.]\n \u00ac{a weaker condition} is always stronger than \u00ac{a stronger condition}.\n\nI already saw these pictures, but the following (created by me) depict 1: P \u2286 Q \u27fa QC \u2286 PC.\nBut I still sense that I can delve more into 1: why is 2 true intuitively? \n\n", "answer": "Consider splitting a set in two based, one subset of which the elements satisfy the predicate, and one subset of which the elements don't satisfy the predicate. If the predicate is stronger, the Yes-set will be smaller (\"the stronger the restriction, the narrower the class\"). Therefore, the No-set will be bigger. And since it works both ways, this means that the negation of the predicate is now weaker.\nThis does not only work for predicates that become stronger or weaker. If we have a strong predicate, that means the Yes-set is small. Since the No-set is the complement of the Yes-set, it has to be big, and therefore the negation of the original predicate (that is, the predicate of the No-set) has to be weak.\n\nThe same, mathematically.\nSuppose we have a set X and a predicate P. Then Y = {x \u2208 X \u2223 P x} and N = {x \u2208 X | \u00acP x} partition X, that is, Y \u222a N = X and Y \u2229 N = \u2205.\nSince \"the stronger the restriction, the narrower the class\", Y will be smaller when P is stronger. And since N = X \u2216 Y, this means that N will be greater, therefore, \u00acP will be weaker. A similar argument can be made when P gets weaker: Y will be greater, so N smaller, therefore, \u00acP will be stronger.\n", "topic": "philosophy", "url": "https://philosophy.stackexchange.com/questions/36995"}, {"image": "VQAonline_00048877.png", "question": "What is the justification for closing a whole branch in a modal truth tree diagram if there is a contradiction in only one world?", "context": "I am watching \"Modal logic 1.2 - truth trees for system K\" presented through Kane B's channel.\nThe standard propositional truth tree diagrams are not a problem. Where I am having trouble is opening worlds in the tree diagram and closing branches with these opened worlds. I don't see why I am justified in closing a whole branch if some worlds in that branch do not have a contradiction in them.\nAt about 14:00 in the video the presenter states, \"If we derive a contradiction in either of these worlds the whole branch closes.\" \nShouldn't we close all worlds prior to closing the whole branch? \nHere is a screenshot of the tree:\n\nI don't see why stacking the worlds justifies the step. Creating world \"w2\" was not permitted without the stacking.\n", "answer": "Up to the branch in w0, we have only propositional logic rules.\nThen we go on on the right branch first : \u25a1\u00acp.\nWe apply the modal rule for \u25ca with \u25cap: we need a new world w1 such that w1 \u22a9 p.\nBut from w0 \u22a9 \u25a1\u00acp it follows that w1 \u22a9 \u00acp and we have the contradiction closing the right branch.\nNow for the left one with \u25a1q; we apply again the modal rule for \u25ca to \u25ca\u00acq.\nThis step needs a new world w2 such that w2 \u22a9 \u00acq.\nBut from w0 \u22a9 \u25a1q it follows that w2 \u22a9 q and we have the contradiction closing also this branch.\n\"Worlds\" behave exactly as the \"single universe\" of classical logic : they cannot tolerate contradictions.\n\nThe basic rule for Kripke semantics for Modal logic is :\n\nv(\u25a1A,w)=T iff for every world w\u2032 in W we have : v(A,w\u2032)=T.\n\nThe rule states that \u25a1A is true (at a world w) exactly when A is true in all possible worlds. \nFrom it, we have the corresponding one for \u25ca :\n\nv(\u25caA,w)=T iff for some world w\u2032 in W we have : v(A,w\u2032)=T.\n\nThe rule insures that \u25caA is true just in case A is true in some possible world.\n", "topic": "philosophy", "url": "https://philosophy.stackexchange.com/questions/57135"}, {"image": "VQAonline_00048834.png", "question": "Who does Wil Wheaton represent in \"Big Bang Theory\"?", "context": "Following the aesthetics-challenge my first question on this site:\nIn several episodes of \"The Big Bang Theory\" Wil Wheaton appears. \n\nIn the credits it is stated that he plays \"himself\". But I have problems with that statement. It seems to me that there are two different entities: The A-ctor on the set of Big Bang Theory and the C-haracter he portrays. They must be different entities because they have different properties: A has no friend named Sheldon Cooper. C was never actor in a show named \"The Big Bang Theory\". So following Leibniz A and C cannot be the same entity.\nBut there are very strange consequences of this. For one the character he plays is said to be Wil Wheaton. If someone asks me, which character is played by Liam Neeson in \"Schindler's List\" I would answer \"He plays Oskar Schindler, a historic person of the third Reich\". Accordingly if someone asks me which character is played by Wil Wheaton in \"The Big Bang Theory\" I'd answer \"He plays Wil Wheaton, an actor known from Star Trek\". I would not answer \"He plays a character very familiar to Wil Wheaton\". If a character in a show is introduced as Mr. So-and-so he is Mr. So-and-so, not a character very similar to Mr. So-and-so. If we need to drop this \"rule\", we were also forced to say: \"In Skyfall Daniel Craig plays a character similar to James Bond.\" That sounds most weird to me. And it feels wrong. \nAlso with this approach we create a whole new set of entities: To every C there is also an A then, and that's not exactly ontological parsimony.\nAre there any theories dealing with cameo-problem? And if so: How is it dealt with?\n", "answer": "I generally take characters on shows to be a different instance of the same person, that is, the name doesn't matter, but in the context of the show the actor is that same actor but in the context of the parallel universe developed for the show.\nLet's use object orientated because that is freakishly easy notation for this problem.\n(Consensus Reality).(Wil Wheaton) portrays (The Big Bang Theory).(Wil Wheaton).\nSo if someone asked me what character Wil Wheaton plays in the The Big Bang Theory, I would say \"He plays Wil Wheaton\" which is the verbal representation of the statement above in OO notation because context is reasonably clear from the discussion.\nFor example, my name is Calvin. Moreover, the protagonist of my perception of reality is Calvin. So I have two Calvins.\n(Consensus Reality).Calvin and (Calvin's Perception).Calvin.\nIn Consensus Reality, Calvin is portrayed by the same Calvin as in Calvin's Perception, so I would say that the credits of Consensus Reality could list Calvin as played by himself.\nHowever, there are certain differences. For example, (Calvin's Perception).Calvin was not born, but just slowly emerged into consciousness over time (due to the nature of memory). However, this difference can be reconciled by viewing these different Calvins as different instances of the same Calvin across multiple abstractions of reality.\nOr at least, that makes sense to me.\n", "topic": "philosophy", "url": "https://philosophy.stackexchange.com/questions/15551"}, {"image": "VQAonline_00049686.png", "question": "What does the Gamma histogram represents?", "context": "I do not understand what does the Gamma Adjustment histogram represents, as it differs a lot from the regular brightness histogram. Would very much appreciate an explanation :)\nHere taken from Canon's Digital Photo Professional:\n\n", "answer": "I found the answer I was looking for here:\nhttps://www.cambridgeincolour.com/tutorials/histograms2.htm\nBasically, it says that the RGB histogram displays the summation of the three Red, Blue, Green histograms, while the Gamma histogram displays the luminosity histogram, which gives different weight to different colors, as the human-eye is more sensitive to green than red.\n", "topic": "photo", "url": "https://photo.stackexchange.com/questions/108068"}, {"image": "VQAonline_00049404.png", "question": "How can I photograph bright lights properly?", "context": "I built a toy light saber, and I'd really like to get a good picture of it. This is what it looks like when I try to take a picture:\n\nThe blade is not actually that color. With my own eyes, it looks red, but through cell phones and digital cameras, it always looks bright orange. Is there a good way to correct the way lights look through a camera?\n", "answer": "There are two issues at play here.\nThe first is exposure. If your camera completely blows out (oversaturated) the light from the \"blade\" of the model because the rest of the scene is much darker then it doesn't matter what color the \"blade\" is, it will show up white in the photo. Why? Because even though there may be more red than blue or green light in the \"blade\", if there's enough blue and green in there for them to be completely saturated on your camera's sensor at the exposure selected by the camera, the (R, G, B) values on a scale of 0-255 will be (255, 255, 255). Even though there is ten times as much red as there is green or blue, the camera can only count the red up to 255, not 2550! When viewing the image this area will appear white, since all three color channels have the same numerical value. In your case it appears the red and green channel may both be oversaturated but the blue channel is not, thus you get orange as a result.\nThe second is white balance. If your camera is set to \"auto\" white balance it tries to determine what color temperature and tint values to use when interpreting the data from the sensor based on the brightest parts in the picture that aren't oversaturated. If the brightest non-saturated part of your picture is not actually white, then the camera will fail to set white balance correctly.\nSo, what can you do to correct this? If possible with the camera you are using dial in some negative exposure compensation or use manual exposure to reduce the overall exposure. When the light from the \"blade\" is no longer blown out it will appear more red if that is the actual color of the light from the \"blade\". You can also attempt to adjust the white balance to match the lighting conditions in the room. Since you haven't told us what type of light is illuminating the \"blade\" from the inside, you might need to experiment until you find a preset WB setting that works best.\n", "topic": "photo", "url": "https://photo.stackexchange.com/questions/71780"}, {"image": "VQAonline_00048994.png", "question": "Can a cheap DX battery damage my camera?", "context": "I would like to purchase a spare battery for my new Canon EOS 550D\\Rebel T2i for a trip abroad. \nI've found this battery on DealExtreme. It costs 6.90$ (with shippment), a small fraction from the street price.\nWhat are the chances this battery will damage my camera?\n\n", "answer": "The chances are high. Nikon started putting holograms on their batteries so they would be recognized as originals because some fakes burst into flames after some use.\nMost importantly, is why are you even considering this? You paid good money for a nice camera and you want to risk it by saving some money on a cheap battery? If the one you have is not enough, even one more will give you at least another 400-shots per charge unless you use the flash more than you should.\n", "topic": "photo", "url": "https://photo.stackexchange.com/questions/15277"}, {"image": "VQAonline_00049038.png", "question": "Are there any techniques to aid/improve reverse macro photography", "context": "I recently started playing with reverse macro with my canon 50mm f1.8. A couple things I noticed immediately is:\n\nShallow depth of field\nNo control over aperture\ndifficult to shoot handheld\n\nI believe these are the common problems with this type of shooting\nAre there any techniques that I can use to improve/help with Reverse Macro shooting besides buying a Macro lens as they are kind of expensive.\nHere is what I've tried so far\n\nps. I have read Are there macro focusing techniques for handheld shots? but focusing isn't really a problem.\n", "answer": "I've faced similar problems and also was able to overcome a few of them. A few techniques that has proved to be efficient for me are:\n\nFocus changes even if you move a millimeter. I tried so hard to keep my body as stiff as possible but that did NOT work. So, I just let it go. I start by looking at the blur image through the viewfinder, slowly move forward and start shooting as soon as the image looks clearer, keep shooting and moving all the way upto the point the image becomes blurry again. Do NOT move using your whole body/waist, instead try inhaling and exhaling slowly.\nI always shoot in continuous mode which enables me to increase the chance of getting a better focused shot.\nIS wont work, nor will aperture control. So, you'll need to set the aperture before detaching the lens. I usually use higher than normal ISO to be able to use a higher shutter speed, thus remove any chance of shake blur.\nNot sure if you're using a reverse mount ring, if not, get one asap. It helps a lot and you can concentrate on things other than keeping your lens safe from falling.\nDOF stacking is another technique you can apply. Its true I wasn't able to plan and execute DOF stacking yet, but if you take a lot of pictures of the same subject, chances are you'll find a couple or two which you can use to stack and gain greater DOF.\nUse of external flash is also very helpful and let you use smaller aperture as well as higher shutter speed while keeping the ISO low. Generally you'd like to use a diffuser along with it. Use this link to get an idea about how to make your own DIY lighting setup for better macro shooting.\n\n", "topic": "photo", "url": "https://photo.stackexchange.com/questions/20937"}, {"image": "VQAonline_00048955.png", "question": "Dust-like speck visible every few pictures \u2014 is it dust, or worse?", "context": "I seem to have to dust-like speck on my Sony Cybershot S950's lens, but it appears randomly every few pictures. Could something be going bad internally? \n\n", "answer": "That sure looks like sensor dust, for which I'd recommend seeing the answers in the Best way to clean a DSLR sensor thread. The spot may appear \"randomly\" because it only appears when you're using a narrow aperture.\nThat said, it seems you don't have a dSLR, thus no interchangeable lenses which are what usually causes the situation where dust can get inside the camera body. I suppose it's possible that dust got inside the camera through some other way although I think that's a fairly rare occurrence.\nIf the camera is still under warranty, it's worth a call to the manufacturer.\n", "topic": "photo", "url": "https://photo.stackexchange.com/questions/11215"}, {"image": "VQAonline_00049452.png", "question": "What is this red dot on the T6 camera body?", "context": "What is this red dot next to the viewfinder and live view button on the Canon EOS Rebel T6 (1300D)? I have been unable to find a reference to it online or in the basic or full instruction manuals.\n\n", "answer": "\nWhat is this red dot next to the viewfinder and live view button on the Canon EOS Rebel T6 (1300D)?\n\nA red dot is a common symbol for the record function, and as on other EOS bodies that button serves the dual purpose of starting Live View mode and starting and stopping movie recording. The icon on the button itself is the Live View icon, and I think the red dot next to the button just reminds you of its alternate function in movie mode.\nYou'll see a similar red dot on the display when the camera is recording a movie.\n", "topic": "photo", "url": "https://photo.stackexchange.com/questions/77875"}, {"image": "VQAonline_00049315.png", "question": "What could I do to take a better picture of this outdoor fountain at night?", "context": "I was practicing with long exposure on this water fountain and here is what I got.\nI would like to know what mistakes do you see I have made and how to improve such a shot? In this location if it were you what would you have done? So I can learn something new to improve.\nI had a 24-70mm lens to use on a Nikon D610.\n\n", "answer": "You wanted to use the effect of long exposures.\nWhat effects of long exposure are visible in your image?\nLights are blown out.\nFor the street lamp on the right, it seems to be ok.\nBut the neon sign is rather ugly. The name of the restaurant is hard to read.\nSo the long exposure doesn't work on lights close to you.\nThe fountain becomes a closed curtain, which is interesting.\nThere aren't many parts that are affected by long exposure and on some parts it even has an unpleasant effect.\nWhat other things are in the image that could be interesting, beautiful, ...?\nThere's a statue on the fountain which looks good with the highlights on the dark material.\nThere's a symmetry to the basin of the fountain, but with the neon lights and the bright ugly building in the back, it will be hard to incorporate it into the image.\nYou can blur out the background, but it will still be visible.\nWhat other image could be taken at this fountain that uses effects of long exposure?\nGet closer. Most of the surroundings should not be part of the image\n as they tend to look ugly under these circumstances. Getting closer to the fountain will show less of those surroundings. Place the camera on the edge of the basin. Pointing up, framing the statue and filling the bottom of the image with the curtain of water. The background should be mostly night sky, which is nice and neutral.\nIf the angles work out, you can try and get the street light in your shot as well, acting as a wrong sun in the background, giving the statue a nice side/back light.\nEdit:\nWhat mode&settings can be used for the fountain shot?\nS is good as it let's you choose a shutter speed (a long one), right? Well, yes, it adjusts the aperture (and maybe even iso) so the exposure is \"right\". But the problem is that a \"right\" exposure for a camera means a grey one (not too dark, not too bright). Say there's a lot of black night sky in your image. The camera will try to find the average grey exposure and as a result it will probably over expose the rest of the image.\nThis will also brighten up the sky and it will look mushy from all the surrounding lights from the city.\nIn your case, a \"right\" exposure could be an image consisting of a lot of pure black and that's not what the camera is looking for.\nIn manual mode, you are in control of all the settings. This however requires a bit of knowledge about what those settings do. You say \"S prevented me from picking f2.8\". This implies that you want to use f2.8 on this shot. This won't work, because f2.8 means the aperture is wide open (probably as wide as your lens can go), which means a lot of light can go through the lens. With that much light, it won't take long before enough light reached the sensor for a good exposure, but you want a long one. That's why the automatic S mode picked f22, which means the aperture is closed and only a bit of light goes through the lens. So the camera takes quite a while untile enough light is captured -> this allows for a long shutter speed.\nIf this is new to you, it is a great opportunity to learn.\nGo ahead and use Aperture priority mode and dial in the f2.8 that you apparently want to use and see what your camera picks as a shutter speed, it will not be a long exposure.\nNow dial in f22 and see how much the shutter speed changes.\nBoth: Long shutter speeds and small f-numbers will make the image brighter, you need something to balance. that's what the S mode did by dialing in f22.\nIf you want to use f2.8 anyway, you have to darken the image by some other means. Often an ND filter is used to reduce the amount of light, but this is a story for another day.\nThis is a good stationary subject to experiment with all this. Set up the tripod. Keep two values constant (aperture and iso, for example) and play with the other one. Don't be surprised to see all white or all black images, just keep going until something shows up. You will get a feel for what these values do.\nGood luck and have fun\n", "topic": "photo", "url": "https://photo.stackexchange.com/questions/59364"}, {"image": "VQAonline_00050524.png", "question": "Why does a perpendicular force not change speed?", "context": "I have been taught that a perpendicular force acting on an object will only change the direction of its velocity, not its magnitude. The explanation that was provided to me is that because there no force in the direction of its velocity, there must be no work done and thus change in the object's kinetic energy. However, I have also studied the vectorial form of the equations of motion \n$$\\vec v_f = \\vec v_i + \\vec a\\cdot t$$ \nSo I assume that the velocity after a constant force has acted on it for a period of time is simply the vector sum of $\\vec v_i $ and $\\vec a \\cdot t$, which I just assume is the acceleration vector (determined by the force) scaled up by a factor of the time the force acts.Diagrammatically, I took the sum as\n\nIt is obviously clear from the diagram that the new speed (which is the length of the $\\vec v_f$ vector) is bigger than the previous speed. \nThis can be confirmed by Pythagoras's theorem, the magnitude of the $\\vec v_f$ vector is \n$$ |\\vec v_f|=\\sqrt{|\\vec v_i|^2 + |\\vec a|^2 \\cdot t^2} $$\nwhich is very clearly not equal to $|\\vec v_i| $\nSo, I conclude that the speed of the object has changed even though the force acting on it was perpendicular to its velocity. So my question is, what have I missed?\nIs my assumption that the final velocity is the vector sum of the initial velocity and the product of acceleration and time incorrect?\nOr have I made a mistake somewhere else?\nThank you in advance for your help.\n", "answer": "In drawing your triangle, you assume that the object has already moved in the direction of your force (maybe just a little bit). In doing so, you forget that the force then stops being perpendicular. \nWhat you have (re)discovered is simply that a force which is constant throughout space cannot remain perpendicular to the motion of a free particle if it lasts a finite amount of time: the particle will simply start to move in the direction of the force, picking up velocity as it starts moving in that direction.\nHowever, if $\\vec{F}$ is not of the same magnitude and direction everywhere, one can produce a force that remains perpendicular. The canonical example is that of a body exerting gravity. The direction of the force is always towards the particle (i.e. not the same everywhere). It also decreases with distance, but that's really not an essential point here. Now, if a test particle at a distance $d$ moves past the other body with the right velocity, this force will always remain perpendicular. Can you guess what kind of shape the trajectory of the test particle will trace out? Hint: think about the symmetry of the problem.\n", "topic": "physics", "url": "https://physics.stackexchange.com/questions/113102"}, {"image": "VQAonline_00053450.png", "question": "Derivation of operator version of the classical wave equation", "context": "I have the following summarised derivation for the operator version of the classical wave equation for massless and material particle.\n\n\nMy question is about the statement:\n\n\nHowever, a problem is that in the probabilistic interpretation of its solution as representing a single particle.\nDifficulty comes from the two possible signs of the square root operations: $E=\u00b1\\sqrt{\\left(pc\\right)^2+\\left(mc^2\\right)^2}$.\nThis can be shown to give rise to antiparticles which must be included for self-consistency.\n\n\nWouldn't the same problem occur for massless particles as well as it is still to the $E^2$?\n", "answer": "There is no unique choice of $\\hat{E}$ in the first place.\nYou are correct in that there is a flaw in the argument from the very beginning as follows:\nWe know the solution for monochromatic electromagnetic plane waves as $\\sim \\exp(\\pm i(\\omega t - kx))$, not only $\\sim \\exp(- i(\\omega t - kx))$! Furthermore, from the photoelectric effect we know that $E = \\hbar \\omega$. That means that we might try to define the energy operator as $\\hat{E} = \\pm i \\hbar \\partial_t$.\nHowever, there is a problem with both of these signs from the very beginning, since we deal with real waves in electromagnetism. By choosing one fixed sign for $\\hat{E}$ (and thus also $\\hat{p}$), a real plane wave $\\sim \\exp(i(\\omega t - kx)) + \\exp(-i(\\omega t - kx))$ would be interpreted as a superposition of particles with both positive and negative energies. Consequently, for any sign choice of $\\hat{E}=\\pm i \\hbar \\partial_t$, every real wave has zero energy!!! That is obviously unacceptable, since we derived the form of the operator from observations about real waves (the photoelectric effect).\nBut notice the following. If we apply $\\hat{E}^2 = -\\hbar^2 \\partial_t^2$, it does give the right answer for a real wave. In other words, we seem to know what $\\hat{E}^2$ should be, but we have no idea what $\\hat{E}$ should be from the very beginning.\nThat is, the excerpt in the OP is misleading in omitting that the choice for the unsquared energy operator is simply unknown at the outset, be it for massive or massless particles.\n\nNegative mass-energy currents - the historical misunderstanding\nThis being said, the real problem with massive particles as compared to massless ones was the following. For electromagnetism, it was kind of obvious that the $T^{00}$ component of the EM stress-energy tensor\n\\begin{align}\nT^{00} = \\frac{1}{\\mu_0} \\left(F^{0\\kappa}F^0{}_\\kappa + F^{\\mu\\nu}F_{\\mu\\nu} \\right) \\propto E^2 + B^2\n\\end{align}\nis somehow proportional to the total energy density of the photons and it automatically stays positive definite. This also has a nice conservation law $T^{0\\mu}{}_{,\\mu} = 0$, i.e., $T^{00}$ is the zeroth component of a conserved current.\nHowever, people were used to the complex Schr\u00f6dinger equation for massive particles. In fact, they knew that the complex nature of this equation is what makes it possible to express the dynamics of, e.g., an electron in the Hydrogen atom. However, if you try to find a conserved current for the equation $(\\hbar^2 \\Box + m^2)\\phi =0$ that reduces to the conserved matter-density current, you would only find a current that exhibited both positive and negative matter densities!\nNow in retrospect we know that the conserved \"matter-density current\" of the Schr\u00f6dinger equation is in fact the non-relativistic limit of a charge-density current. Since in the non-relativistic limit, the energies of motion are such that a single electron always stays a single electron, the charge per mass $m_e/e$ always stays the same. So we were able to multiply the charge current with $m_e/e$ and simply use it as an equivalent of a density current. However, in the relativistic case the electron can in principle be superposed with positrons, which have opposite charge $-e$. By multiplying the conserved charge current by $m_e/e$, we then got seemingly negative mass (or probability-of-occurence) densities. There is, however, a stress-energy tensor of the scalar field $T^{\\mu\\nu}$ which does have a (classically) positive-definite $T^{00}$ with the meaning of mass-energy density and that has been ignored because it would not correspond as easily to the Schr\u00f6dinger-equation current.\nSo that is the whole deal with massive particles.\n(P.S.: I kind of skip the part where electrons have spin and their Schr\u00f6dinger equation is the non-relativistic limit of the Dirac equation. But the gist is the same.)\n", "topic": "physics", "url": "https://physics.stackexchange.com/questions/607101"}, {"image": "VQAonline_00049982.png", "question": "Cyclist's electrical tingling under power lines", "context": "It's been happening to me for years. I finally decided to ask users who are better with \"practical physics\" when I was told that my experience \u2013 that I am going to describe momentarily \u2013 prove that I am a diviner, a psychic, a \"sensibil\" as we call it. The right explanation clearly needs some electrodynamics although it's \"everyday electrodynamics\" and theoretical physicists are not trained to quickly answer such questions although each of us has probably solved many exercises that depend on the same principles.\n\nWhen I am biking under the power lines \u2013 which probably have a high voltage in them \u2013 I feel a clear tingling shocks near my buttocks and related parts of the body for a second or so when I am under a critical point of the power lines. It is a strong feeling, not a marginal one: it feels like a dozen of ants that are stinging me at the same moment. It seems almost clear that some currents are running through my skins at 50 Hz. I would like to know the estimate (and calculation or justification) of the voltage, currents etc. that are going through my skin and some comparison with the shock one gets when he touches the power outlet.\nNow, \n\nmy bike that makes this effect particularly strong is a mountain bike, Merida;\nthe speed is about 20 km/h and the velocity is perpendicular to the direction of the current in the power line;\nthe seat has a hole in it and there is some metal \u2013 probably a conducting one \u2013 just a few centimeters away from the center of my buttocks. It's plausible that I am in touch with the metal \u2013 or near touch;\nmy skin is kind of sweating during these events and the liquid isn't pure water so it's probably much more conductive than pure water;\nthe temperature was 22 \u00b0C today, the humidity around 35%, clear skies, 10 km/h wind;\nthe power lines may be between 22 kV and 1 MV and at 50 Hz, the altitude is tens of meters but I don't really know exactly.\n\nWhat kind of approximation for the electromagnetic waves are relevant? What is the strength? How high currents one needs? \nDoes one need some amplification from interference etc. (special places) to make the effect detectable? (I only remember experiencing this effect at two places around Pilsen; the most frequent place where I feel it is near Druztov\u00e1, Greater Pilsen, Czechia.) \nIs the motion of the wheels or even its frequency important? Is there some resonance?\nDoes the hole in the seat and the metal play any role? Just if you think that I am crazy, other people are experience the effect (although with different body parts), see e.g. here and here. This PDF file seems to suggest that the metals and electromagnetic induction is essential for the effect but the presentation looks neither particularly comprehensive nor impartial enough.\nAn extra blog discussion on this topic is here:\n\nhttp://motls.blogspot.com/2012/05/electric-shocks-under-high-voltage.html\n\n", "answer": "First, Field strength. \nThis calculation is strictly an electric potential calculation; radiation and induction are safely ignored at 50Hz.*\nFor a 200kV transmission line 20m above ground, the max electric field at ground level is about 1.2 kV/m.** This number is reduced from the naive 200kV/20m=10 kV/m calculation by two effects:\n1) The ~1/r variation in the electric field (reduction to 3 kV/m). I used the method of images to calculate this field, with a 10 cm conductor diameter to keep the peak field below the 1MV/m breakdown field.\n2) Cancellation from the other two power lines in this 3-phase system, which are at +/-120 degree electrical phases with respect to the first, and are physically offset in a horizontal line per the photo. I estimated 7m spacings between adjacent lines. The maximum E-field actually occurs roughly twice as far out as the outermost line; the field under the center conductor is lower.\nNext, Can you feel it? \n1) The human body circuit model for electrostatic discharge is 100pF+1.5kohm; that's a gross simplification but better than nothing. If one imagined a 2m high network, the applied voltage results in a 50Hz current of about 70uA ($C \\omega V$). Very small.\n2) There will be an AC voltage difference between the (insulated) human and (insulated) bicycle. A 1m vertical separation between their centers of gravity would yield roughly 1200V. This voltage is rather small compared to some car-door-type static discharges, but it would still be sufficient to break down a short air gap (but not a couple cm), and would repeat at 100Hz. I imagine it would be noticeable in a sensitive part of the anatomy.\nIf the transmission voltage is actually 400 kV, all the field strengths and voltages would of course double.\n\n(*) In response to a comment, here's an estimate of the neglected induction and radiation effects, courtesy of Maxwell 4 and 3:\nInduction: Suppose a power line is carrying a healthy 1000A AC current (f=50 Hz). Then by Ampere's law, there is a circumferential AC magnetic field; at the wire-to-ground distance of 20 meters that field's amplitude is $10 \\mu T$. (Compare with the earth's DC field of approximately 0.5 gauss, or $50 \\mu T$.) \nThe flux of this magnetic field through a $1 m^2$ area loop (with normal parallel to the ground and perpendicular to the wire) is $\\Phi = 10 \\mu Wb$ AC. Then from Faraday's law, the voltage around the loop is $d \\Phi /dt = 2 \\pi f \\Phi = 3 mV$ (millivolts). So much for induction.\nOne can also estimate the magnetic field resulting from the $1200 V/m$ ground-level AC electric field, which has an electric flux density $D =\\epsilon_0 E = 10.6 nC/m^2$ and a displacement current density $\\partial D / \\partial t = 2 \\pi f D = 3.3 \\mu A/m^2$. The flux of this field through a $1 m$ square loop (parallel to the ground) is $3.3 \\mu A$, so the average magnetic field around the square is $0.8 \\mu A/m$, for a ridiculously small magnetic flux density of $1 pT$. \n\n(**) 1 Sep 2014 update. Dmytry very astutely points out in a comment that there will be local electric field intensification effects from conductive irregularities in the otherwise flat ground surface, such as our cyclist (who, being somewhat sweaty, will have a conductive surface). The same principle applies to lightning rods.\nFor the proverbial spherical cyclist, the local field will be increased by a factor of 3, independent of the sphere's size, as long as it's much less than the distance to the power line. It turns out that it doesn't matter whether the sphere is grounded or insulated, since its total charge remains 0. \nFor more elongated shapes the intensification can be much higher: for a grounded prolate spheroid with 10:1 dimensions, the multiplication factor is 50. This intensification of course enhances any sensation one might feel. \n", "topic": "physics", "url": "https://physics.stackexchange.com/questions/29016"}, {"image": "VQAonline_00050690.png", "question": "Why don't we consider both the forces while calculating the magnitude of stress in an elastic body?", "context": "Consider a wire being stretched from two ends with equal forces. We know that both of these forces collectively participate in elongating the wire; had there been one force the wire would have accelerated in the direction of force. Why can't then the stress be calculated using the two forces (knowing that the vector resultant of the two forces would come out to be zero)?\n\n", "answer": "Tricky question. Basically you would think the total force is 0 on any plane intersecting the cylinder at right angles, hence there would be no pressure, right? Well, the first point (0 net force) is correct, second is not.\nImagine being physically pulled by two equally strong friends in opposite directions. Total force is 0 so you remain standing where you are, but you will feel a stress (pressure) in your body from your pulling friends.\nIt helps to think of the situation from a different perspective: One force is trying to pull out (extend) the cylinder, and is therefore providing a pressure/stress on the cylinder, of magnitude P=F/A. But the cylinder is not moving, so there must be an opposite force of equal magnitude, which maintains the cohesion of the cylinder. At some level of external force this cohesion is no longer strong enough, so the external force will rip the cylinder apart.\nYou can think of the cohesive force as a sort of reaction (or supportive, passive) force, and the pulling as an \"active\" force.\n", "topic": "physics", "url": "https://physics.stackexchange.com/questions/138828"}, {"image": "VQAonline_00050407.png", "question": "How does a canvas water bag cool water?", "context": "I was reading about this water bottle by Botl that behaves like a canvas water bags to keep water cool. I found out that this idea is an old idea and cars would drive with water bags in front as shown below.\n \nHere is what I found out about them: it seeps water through the bag and evaporates, causing the water to cool inside. Here is where I need some help:\n\nWhy does the water seep through the bag? Is it because the water molecules have a Maxwell speed distribution and only the fastest molecules seep out?\nWhat mechanism is occurring for heat being removed from the water to evaporate the water on the bag\u2019s surface? Doesn't radiation also contribute here?\nWhy does moving increases the cooling effect faster?\n\n", "answer": "As far as I understand the idea is very simple:\nas the bottle is covered in a film of water (due to a leakage of the container), this can evaporate. Due to the Boltzman distribution this will also happen well below $100\u00b0C$. The process of evaporating though takes energy from the system (called latent heat). This cools down the bottle.\nIf the surrounding air is \"filled with water\" it is harder for the liquid to evaporate and thus slowing the process, so always bringing fresh air to the system helps.\nThis is basically the concept of sweating as well, where you can immediately feel the effect of wind blowing on your moist skin.\nHope this helps and I didn't miss something.\n", "topic": "physics", "url": "https://physics.stackexchange.com/questions/97959"}, {"image": "VQAonline_00051983.png", "question": "A question regarding Newton's Third Law for torques", "context": "I was trying to prove why, if two objects are rotating around some axis, and are interacting, then the sum of the torques produced on both objects via the mutual interactions will be zero. However, I am having a confusion as to how the forces are being applied. I will use a diagram to make thinks clear.\nConsider the following diagram:\n\nHere , there are two particles, $A$ and $B$, which are rotating about an axis, which is the center of this circle. Let $B$ exert a force on $A$, which I have denoted by a vector. The torque produced on $A$ is thus magnitude of the force times the perpendicular distance from the axis, which again I have shown. By Newton's third law, an equal and opposite force acts on $B$, which I have denoted by a vector, equal in magnitude but opposite in direction. Now here is my confusion: If Newton's law says that there is an equal and opposite force along the same line of action, then the torques cancel each other. But, if that is the case, then $B$ isn't lying on the line of action, so how would it experience the force? If I draw an equal and opposite vector through $B$, then the torques don't cancel each other. \nNote that the force vector on $A$ is any general force, not necessarily lying on the line segment joining $A$ and $B$. What is wrong here?\n", "answer": "You're right; the torques only cancel out if and only if the forces are applied along the line connecting the particles. Here's the proof: Let $\\vec{F}_{AB}$ be the force on $A$ due to $B$, and define $\\vec{F}_{BA}$ similarly. The torque on $A$ due to $B$ is then \n$$\n\\vec{\\tau}_{AB} = \\vec{r}_A \\times \\vec{F}_{AB}\n$$\nand the torque on $B$ due to $A$ is then \n$$\n\\vec{\\tau}_{BA} = \\vec{r}_B \\times \\vec{F}_{BA}.\n$$\nThe net torque is then\n$$\n\\vec{\\tau}_\\text{net} = \\vec{r}_A \\times \\vec{F}_{AB} + \\vec{r}_B \\times \\vec{F}_{BA}.\n$$\nBut by Newton's Third Law, $\\vec{F}_{BA} = - \\vec{F}_{AB}$, which allows us to simplify this to\n$$\n\\vec{\\tau}_\\text{net} = (\\vec{r}_A - \\vec{r}_B) \\times \\vec{F}_{AB}.\n$$\nNote that $\\vec{r}_A - \\vec{r}_B$ is a vector pointing from $B$ to $A$; note also the cross product of two non-zero vectors vanishes if and only if they are parallel. We conclude that $\\vec{\\tau}_\\text{net} = 0$ if and only if the force between the two particles acts along the line connecting them.\nWhat this means is that in your diagram, if $A$ and $B$ are really exerting forces on each other in this way, then the net torque they exert on each other will not in fact be zero. The net torque will only be zero if the particles interact with each other via a central force, which is a fancy way of saying that the force is parallel to the line connecting the particles.\n", "topic": "physics", "url": "https://physics.stackexchange.com/questions/354520"}, {"image": "VQAonline_00051812.png", "question": "Would like to experiment with a safe sized leyden jar", "context": "I would like to experiment with leyden jars, but do not want to mess with anything big enough to kill me should I get zapped.\nWhat are some good guidelines as far as size when making homemade Leyden Jars?\nHere is a picture of one I created, could it carry a lethal charge?\n\nThanks!\nBen\n", "answer": "The safety considerations for Leyden jars are the same as for any capacitor; these depend on how you intend to use the device and how much energy you put into it. Leyden jars have small capacitances, but they can withstand very high voltages, which means that they can store a great deal of energy, e.g. if you charge them with a Van de Graaff generator, for example.\nEstimate, or better, measure the device's capacitance with an LCR meter or impedance meter. \nYou should measure the capacitance and check that it agrees with the estimate. A good estimate will be gotten from the formula (which you can derive from the integral form of Gauss's law):\n$$C = \\frac{\\epsilon\\,A}{d}$$\nwhere $A$ is the surface area covered by the foil on one side of the jar, $\\epsilon$ the low frequency dielectric constant of the glass (can be up to 10, considerably more than the square of the optical frequency dielectric constant) and $d$ the thickness of the glass (distance between the two foils). For 1mm thick glass, you'll get a figure of the order of $10^{-8}{\\rm F}$ - about ten nanofarad. \nThe glass dielectric has a very high breakdown or dielectric strength; typically this is ten to thirty MV per meter, meaning that the device can withstand ten to twenty thousand volts across its electrodes. At that degree of charging, you have stored $\\frac{1}{2}\\,C\\,V^2$ or of the order of one to ten joules. You're unlikely to get to this, but you could approach it if you charged the device with a Van de Graaff generator and this is approaching the realm of defibrillator level energies.\nIn short it is unlikely to be lethal, but it could be if misused. You need to plan what you want to do with it. You could try measuring the voltage across it in its charged state in a typical experiment with a high impedance voltmeter, and work out the energy stored in it with the formula $\\frac{1}{2}\\,C\\,V^2$ with $C$ set to ten nanofarads. If you come up with a figure of more than a few millijoules, then you're in danger territory. \nFor example, with $A=0.05{\\rm m^2}$ (wine or 1 liter softdrink bottle size), $d=1{\\rm mm}$ and a relative permeability of glass, I get a capacitance of $4.4{\\rm nF}$. To limit this device's energy to $0.001{\\rm J}$, therefore, you need to limit the charging voltage to about $660{\\rm V}$. In this state it would still deliver quite a whack to your hand. I would check these figures with a capacitance measurement for your actual device; some plastics can have rather large permeabilities.\n", "topic": "physics", "url": "https://physics.stackexchange.com/questions/320472"}, {"image": "VQAonline_00054215.png", "question": "Can separate roles in DSDM Atern be handled by the same person?", "context": "In the DSDM Atern project delivery framework roles fit into the following three categories:\n\nProject Level\nSolution Development Team, and\nOther\n\nThe diagram below shows how an Atern team is structure:\n\nFor example, in the Solution Development Team we have the roles of the Business Ambassador and the Business Analyst(s).\nI appreciate that the Business Ambassador's role is more focused on the day-to-day communication between the project and the business whereas the Business Analyst role facilitates the communication between the developers (technical roles) and other business roles.\nThere are many organisations that insist on assigning these roles to two different people. However, in smaller organisations where the resources are limited in all categories -i.e., there is no enough people to fulfil every business role; could the same person be assigned to both roles?\nWhat are the risks or limitations of merging these two roles?\nAre there any (other) roles that lend themselves to being covered by the same people?\nAre there any that definitely shouldn't be covered by the same person? \n", "answer": "Actually, the Business Sponsor and the Business Visionary could be the same person, in some organisations.\nDSDM states that a role can be shared between more than one person, or one person may have more than one role.\nExample 1- you could have several people fulflling Business Advisor role, each with their own area of specialism.\nExample 2 - on a small simple project, one person could fulfil the Project Manager and Team Leader role.\nWe would normally only expect a single Business Visionary (rather than several Visionaries, which could add confusion) - i.e. having one Visionary is \"normal\", but having more than one could happen in exceptional circumstances\nWe expect only one Business Sponsor. (I have never ever had a project with more than one Sponsor in 18 years of doing DSDM!!)\nThere is no rule that says a Business Analyst and a Business Ambassador cannot be the same person. It is simply that this is an unlikely combination, since a Business Ambassador, with all their business experience, is less likely to have the analysis skills. But a good analyst / programmer might well fulfil the Business Analyst role AND the Solution Developer role (in an organisation where there are no Business Analysts)\nHope this helps\n", "topic": "pm", "url": "https://pm.stackexchange.com/questions/6021"}, {"image": "VQAonline_00054223.png", "question": "Getting estimated working time from MS Project", "context": "Our company has just recently started using MS Project Professional 2010 and we plan to eventually have a Project Server installation. I've created a few projects so far, but still claim to be a complete novice.\nA customer has just requested a quote for some work, so I've created a project plan for the proposed work, however I'd like MS Project to tell me how much working time I've allocated in the plan I've created.\nThe only information MS Project seems to offer is the difference between the start and end date of the project (9.38 days), but what I want is the time my staff will be working on the project, which shouldn't include lag time or times outside the calendar that I've created. I've calculated this as being 36 hours from the non-summary tasks in the example below (apologies for having to blur the task names).\nWhilst this example is only tiny, I want to know how to get MS Project to calculate this for me in future on much bigger projects. Can anyone point me in the right direction or tell me what I'm doing wrong?\n\n\nUpdate: I've found that if I allocate a new 'test' staff resource rate of \u00a31/hour for simplicity, I can see a cost of \u00a336, which matches the 36h I'm expecting to see.\n", "answer": "There are two easy ways of doing this:\n\nGo to the \"Resource Usage\" view. Go you get there differs slightly depending on which version of Project you are using, but look for a Views menu or on Project 2010, look at the leftmost button on the Task tab (by default) for your Views selection. This will give you the actual time allocated for each resource in the plan broken down by task and totalled at a resource level.\nShow the \"Work\" column in the Gantt Chart (right-click the column headings on the left side of the Gantt Chart, select \"Insert Colum\", add 'Work'). This will show you the actual amount of work specified for each task irrespective of the type and nature of the resources assigned to that task.\n\nWhich method you use depends on what you are comfortable with and what kind of breakdown you want to get out of it.\n", "topic": "pm", "url": "https://pm.stackexchange.com/questions/10209"}, {"image": "VQAonline_00054226.png", "question": "How to arrange tasks around a fixed milestone?", "context": "i am tasked to take over a project that has recently taken off the ground. However, there has been no project plan and we need to create one now. Due to circumstances, I can only fix the next milestone in time and need to arrange tasks around that milestone.\nThis is fine for future tasks, but current tasks don't align so that they culminate into the milestone, even if they are assigned as predecessors to the milestone. If I assign the the previous tasks to the milestone as predecessors, the milestone is moved.\n\nHow can I arrange all the tasks around the milestone?\n", "answer": "The problem looks to be that you are scheduling the very first task from \"today\"... If you link them nose-to-tail the entire project will schedule from \"today\" and the milestone must move to accommodate that. \nIf you set the start date of the first task sufficiently far in the past that there is enough time to fit all the first tasks in before the milestone then all will be well. This might mean you have to set the Project Start Date in the past too.\nOnce you have done all that:\n\nFix the start of the milestone by making it a \"Must start on...\" or \"Start no later than...\"\nMake the milestone a dependant of the last task in the first section (i.e. make the final previous task a predecessor of the milestone).\nMake all the prior tasks \"Finish as late as possible\"\n\nHope that helps.\n", "topic": "pm", "url": "https://pm.stackexchange.com/questions/14258"}, {"image": "VQAonline_00054238.png", "question": "Why in JIRA my field Resolution is labeled as Unresolved, when status is Resolved?", "context": "Question is pretty simple, I've a Kanban project, with a personal workflow that provides different statuses, as Resolved.\nWhen I drag and drop to this last one through the lanes, or when I set it manually to Resolved status, the Resolution fields is still Unresolved, and one of my widget - Created VS Resolved chart - is not working properly.\nAny idea?\nThanks in advance\n\n", "answer": "In JIRA, an issue is only considered resolved if it has a value in the RESOLUTION field.\nThe status of the issue doesn't actually matter. For example, it could have a status of RESOLVED or CLOSED but would still be considered as unresolved by JIRA if the RESOLUTION field was empty.\nSo, the trick to resolving issues is to ensure that you have setup your workflow correctly. Make sure that the final transition in your workflow includes the resolve issue screen.\nThis screen is where you set the RESOLUTION. For example, you might have a RESOLUTION of \"done\", \"duplicate\", \"won't fix\", etc.\n", "topic": "pm", "url": "https://pm.stackexchange.com/questions/22987"}, {"image": "VQAonline_00054245.png", "question": "Start and End node in Critical Path Calculation", "context": "I have to calculate the critical path for a project, where activity A and C can start in parallel. Please see the image below\n\nSince I had two activities that could start in parallel (A and C), I also used the dummy start node. Similarly, since my project had 2 nodes at the end (B and D), I added the dummy end node. Please tell me if my approach is correct.\n", "answer": "\nYes, adding start and finish milestones to the network is correct. (Calling them \"dummy nodes\" is unconventional, as that term is usually reserved for activity-on-arrow diagrams.)\nYou might want to double-check your calculations. Conventional terminology starts the project on Day 1, not Day 0. In addition, your start and finish days are not consistent with the durations shown. \n\n", "topic": "pm", "url": "https://pm.stackexchange.com/questions/25710"}, {"image": "VQAonline_00054241.png", "question": "What is part of the Customer Relations team in Scrum@Scale", "context": "The Scrum@Scale guide documents a Customer Relations team as a separate entity, but does not specify what it entails.\n\nCustomer Relations, Legal / Compliance, and People Operations are\n included here since they are necessary parts of organizations and will\n exist as independent Scrum teams on their own, which all of the others\n may rely upon.\nhttps://www.scrumatscale.com/wp-content/uploads/Scrum@Scale-Guide.pdf @Page 16\n\n\n\nWhat are the common roles/responsibilities of a CR team in Scrum@Scale? e.g. would CR also contain a sales like activity?\nWhy is this team not near the pentagon they relate to? As an independent Support/Implementation/Sales/Customer team, with a PO which feeds its local MetaScrum.\n\nI guess my question really is why is it specially named and not part of the normal scaling organisation? It makes me feel you can handle it with a small team 3-9 people and does not need to scale ever.\n", "answer": "Customer relations typically includes everything needed to maintain relationships with existing customers and reach new customers. It tends to be focused on communication - answering questions, resolving issues or complaints, and similar types of work. It may includes sales and marketing, as well as social media and others. The exact definition varies by organization, but the Scrum @ Scale guide is showing how common functions integrate with the product development Scrum Teams at the Executive Meta Scrum / EAT level.\nIn Scrum @ Scale, Customer Relations (along with Legal / Compliance and HR / People Ops) are their own Scrum Teams. If you are scaling your product development teams with Scrum @ Scale, then you will likely also scale these other teams with Scrum @ Scale. All of the different team types (Product Development, Customer Relations, Legal / Compliance, and HR / People Ops) all converge at the Executive Meta Scrum and EAT levels.\nWithin these teams, you would continue to have a Scrum Master, a Product Owner (who wouldn't be managing a Product, but the Services provided by these organizations - perhaps it should be called a Service Owner), and a Development Team that carries out the work. If necessary, they would have Scrum of Scrums and Meta Scrum and Scrum of Scrum of Scrums and so on. However, all aspects of the organization come together at the Executive Meta Scrum and EAT.\n", "topic": "pm", "url": "https://pm.stackexchange.com/questions/24526"}, {"image": "VQAonline_00054222.png", "question": "How can I visually represent interdependant resources?", "context": "This one may be tricky to explain, but here's the use case. Our organization aims to reduce the amount of \"time-stealing\" by other staff. We want to eventually see this visual indicator show less intersects, with each resource keeping as much of their available working time free from interactions with other people as possible. For example:\n\nPerson A's total available working time as a circle. \nPerson B who requires some of that person's time per month, so both\ncircles intersect the amount of time they interact.\nPerson C then requires a big slice of Person A and B's time.\nPerson D requires only a little of Person A and B's time.\n\nHow could I simply and visually represent such interconnected resource usage? I really can't think of how to visualize this, so am open to any ideas, even 3D if required.\nEDIT:\nThank you for the feedback so far. \nI could agree to not calling it \"waste\", but perhaps \"shared time\". Let's assume my organization is full of really capable people, who if left alone, are super-producers (ignoring reality for a while).\nI played with this visualisation, which is getting closer to what I'm hoping for:\n\nAs shown, each person has their own colour (the outer line colour. The amount of time shared with another person is always of equal amount on each person's bar, represented by a fill of the shared person's colour. \nEveryone has a finite amount of time, so over-sharing can be quickly spotted by those who's boxes are filled up more and the most time demanding people can be seen by their color being in most people's boxes.\nThis could just as easily be pie-charts, I guess. For our company's purposes, we can limit the total amount of pie or bar charts to around 15, which would still give an at-a-glance feel for the information.\n", "answer": "Your Question, Rephrased\nYour question, as phrased, seems non-trivial. You seem to be asking how to represent process dependencies within a single infographic. There are various charts and graphs one might use to show dependencies, but if you dig a bit deeper, that's not the real underlying information that you're trying to represent here.\nThe information that you seem to need is one of either:\n\nConsumption of available time by \"waste\" attributable to other people.\nSome kind of organizational capacity chart that breaks out waste and available capacity.\n\nPossible Representations\nThe number of possible graphical representations of any given data set are largely limited by your imagination. What is \"best\" is not answerable. However, I can certainly share some insight into what data you might need to track to represent your informational objective.\nIf you're looking to break out consumption, pie or donut charts are an obvious choice. Trying to pack it all into one pie chart will probably lead to information overload if you are charting more than a few people, but it's technically possible.\nConsider an exploded pie chart based on the following metrics:\nResource | Available Man-Hours | Waste by A | Waste by B | Waste by C | Waste by D\nPerson A 40 8 18 2 5 \nPerson B 35 6 2 1 7\nPerson C 38 0 6 7 7\nPerson D 53 1 1 1 7\n\nThe idea is that the whole pie represents the total team capacity for a given interval, while each exploded slice represents the percentage of total team man-hours attributable to each person. Each exploded slice is then sub-divided into color-coded slices attributable to waste by some other person in the data set.\nIf you want to show total productive/wasted hours, rather than percentages of a whole, then a stacked bar chart might be a better option. Again, the choice of format is only bounded by your communications intent.\nWhy You May Not Want to Do This\nProcesses are performed by people. It is worth considering whether interruptions or \"time-stealing\" are truly wasteful interactions, or whether they represent undocumented processes and procedures within your organization. Mapping undocumented processes that operate pervasively throughout an organization is a non-trivial exercise, but there are certainly practices such as value-stream mapping that may make the task manageable.\nEven if you decide that the interactions truly are wasteful, you need to consider the process overhead of tracking all that information with any accuracy or for any given granularity. At one organization, I once determined that the requirement for team members to accurately track time allocations in 15-minute intervals consumed almost 50% of team capacity. Is that worth it for your use case?\nYou might cut down on this overhead by replacing accurate and continuous journaling with guesstimates, or by assigning values based on simplistic calculations like Waste = Wall Time - Ideal Hours to Complete. However, the Garbage In, Garbage Out principle would certainly apply.\nGenerally, if you want to track waste, the metrics are more meaningful at the process or project levels rather than at the individual level. However, organizational circumstances may certainly vary.\n", "topic": "pm", "url": "https://pm.stackexchange.com/questions/8656"}, {"image": "VQAonline_00054278.png", "question": "Confusion about two pair rule in poker, can someone help me who will win", "context": "\nHello guys please help here. i think it is a draw what do you say?\n", "answer": "A poker hand is built with 5 cards (and no more than 5). There is no limit to how many of those cards come from your hole cards, and how many of those come from the table (community cards)\nFor instance, if the board comes 23456, and your hole cards are A9, you will build your hand using all 5 community cards, getting a straight.\nIn the situation your present, both players would get a QQJJ8 as their hand (the pair of 3's is irrelevant since any 5-card combination cointaining threes would be worse than QQJJ8). The pot is therefore split\n", "topic": "poker", "url": "https://poker.stackexchange.com/questions/10603"}, {"image": "VQAonline_00054267.png", "question": "C Net Adjusted, What does it mean? Should it be low or high?", "context": "I am still pretty fuzzy on this concept of C Net Adjusted. The answers I read are vague and I'm just looking for a simple answer.\nIf C Net Adjusted is lower than C Net Won (Actual), am I doing better than I should, or worse?\nAs far as my understanding, if C Net Adjusted is lower than C Net Won, then you are running good. And this should be a marginal difference.\nIf my assumption is correct, am I also correct in assuming the stats from this run of tourny's (see image) are indicitive of a rigged poker site due to the extremely large margin between C Net Won and C Net Adjusted ? \nThanks for any help!\n9 Player $5 + $0.50 NLTH (Some are regular and some are turbo.. so 10 mins blinds and 5 min blinds)\n\n", "answer": "It seems the \"Adjusted\" description relates to a calculation involving ICM. \nRipped direct from the PT4 forums, posted by the mod WhiteRider: (See the whole thread here)\n\n\"My C\" means \"My Currency\", and implies a monetary stat.\n\"My C Net Adjusted\" is your expected winnings based on the difference\n in ICM values between your actual chip stack at the end of a\n hand and the all-in equity adjusted chip stack you \"should have had\"\n at the end of the hand; for all all-in hands in a tournament. This\n calculation uses every all-in hand in a STT and final table hands from\n MTTs where we know the complete payout structure.\n\nWith regard to your graph, this means that eventually the two lines will converge. In the short term, it's an indication of how above, or below average, your luck is. \nThis is as comprehensive an explanation of what it means as you'll find I believe. Direct from the developer. \n", "topic": "poker", "url": "https://poker.stackexchange.com/questions/6057"}, {"image": "VQAonline_00054270.png", "question": "Why did i win a portion of pot in this hand?", "context": "I am the player with the hand Q\u2660T\u22c4. After the turn, I was thinking that anyone with a 4 would win the full pot. I am surprised that I won 286 of the pot. Can someone please explain how this works? I searched online, but couldn't find any explanation for this.\n\n", "answer": "TL;DR A side pot is a pot when certain players have no participation due to lacking the required amount of chips. It is the way to accept bets when players have different chips amounts. This situation can only occur when involved active players are all-in or under special circumstances in the table rules or online poker services.\nDisclaimer: With such screenshot we are unable to see the whole hand history. The only possibility for you to win a pot like that is that the pot you won is a side pot. A side pot is a pot in which a player who went all-in could not participate due to not having enough chips to call an all-in bet. Take this example:\n\nPlayer A bets 100 chips.\nPlayer B calls 100 chips.\nPlayer C wants to participate but can only call with their all-in, which is 70 chips only.\n\nIn this case, players A B C make a main pot from the lower amount of each bet: 70, 70, 70 (210 total pot). The remaining players make an additional pot for the remaining quantities: players A B make a side pot (a pot which is not the main, where every active player has equal participation) of 30 30 (60 total pot).\nIn the case player C wins, will only win 210 chips, and players A B will dispute their hands for the A B pot: who will win the remaining 60.\nThis case could also arise:\n\nPlayer A bets 100 chips.\nPlayer B calls all-in with 90 chips.\nPlayer C calls all-in with 60 chips.\n\nPots: A B C (60 60 60 = 180 chips), Sidepot A B (30 30 = 60 chips), Sidepot A (10 chips, returned immediately to A after the hand resolves).\nEither case, you seem to be the player A (notice how C is all-in).\n", "topic": "poker", "url": "https://poker.stackexchange.com/questions/6999"}, {"image": "VQAonline_00054279.png", "question": "What hand is this?", "context": "I've been starting to learn poker recently (as a reference, I tend to use this list of hands). I was watching a tournament and this hand showed up.\nOne player has a 99, the other a JJ. I understand the JJ wins, but why does the dealer raise the K, Q & 7?\nThanks]2\n", "answer": "A poker hand is 5 cards, so the dealer is showing that the winning hand is JJKQ7. If, instead, the winning hand had been AhKh for a flush, the dealer instead would have raised the Qh 4h and 7h.\n", "topic": "poker", "url": "https://poker.stackexchange.com/questions/10623"}, {"image": "VQAonline_00054271.png", "question": "Advanced Poker Calculator", "context": "Does anyone know of an advanced poker calculator for Windows?\nOr for Universal Windows Platform (runs on Windows and smart phone)? \nI found this for Mac Poker Cruncher. I am looking for a tool exactly like that but for Windows. I know you can use tools to host a Mac application in Windows. I am looking for a native Windows application.\nNot talking about basic statistics on you versus a known hand or random hand. A tool where you can put in a range for your opponent and get an equity.\nTurns out there is Equilab and Flopzilla. Toby answered in Chat. \nI decided to write one. Will not have the features of Flopzilla but I want to write a phone application and this seems like good application for practice. Clearly it is not done. \n\n", "answer": "Toby Booth answered in chat\nEquilab and Flopzilla\n", "topic": "poker", "url": "https://poker.stackexchange.com/questions/8112"}, {"image": "VQAonline_00054275.png", "question": "How to count the number of starting hands in Texas Hold 'em", "context": "I'm revisiting the excellent book by Tony Guerrera, Killer Poker by the Numbers. In the first chapter he goes over how to count the number of possible starting hands but leaves, as an exercise for the reader, counting the number of starting hands that are unpaired and unsuited. He does show the probability of being dealt 2 unsuited unpaired cards as 1/110.5.\nMy calculation seems to be wrong and I can't figure out where I'm missing.\nIf you select any one of 52 cards, then there remains 36 cards left that are not of that suit or that rank. And there are 52*51 permutations of 2-card hands available.\nThis gives me 52*36/52*51 = 1872 / 2652 = 70.5% \nI'm obviously way off here but I can't tell what I'm doing wrong.\n", "answer": "Probability of a pair is 3/51, because the first card dealt does not matter. Any card can pair up. The second card needs to be of the same rank as the first card. There are 3 cards left that satisfy this condition, but because one card is already dealt there are 51 cards left. Therefore the answer is 52/52 * 3/51 which is just 3/51.\nProbability of no pair is 48/51. Similarly the first card does not matter, but the second card needs to not pair, which are all remaining card minus three, which is 48/51.\nProbability of a suited hand is 12/51, since there are 13 cards of each suit in a deck of cards. The first card you get does not matter, but the second card needs to be of the same suit, which are 12 cards.\nProbability of a non suited hand is 39/51. Same as the previous example, but now you need to essentially dodge 12 cards from the remaining 51 cards. 51 - 12 = 39.\nProbability of a non suited non pair hand is 36/51, which indeed rougly equals 70.5%. Just like the previous example, but now you also need to not pair. Since these three cards are all different suits than the original cards we are not counting any cards twice, just substract three more.\nNow we understand this logic we can tackle your problem. So we need to get dealt J9o. This means our first card has to be a jack or a nine, which are 8 cards total. Since there are 52 cards in a deck odds of this happening is 8/52. Now after that we need to hit the other card. There are 3 cards left that will get the job done, because suits need to be different. Odds of this happening therefore is 3/51. To calculate the odds of both these events occuring we need to multiply both numbers. We get 8/52 * 3/51 = 24/2652. We can simplify this to 1/110.5.\n", "topic": "poker", "url": "https://poker.stackexchange.com/questions/9520"}, {"image": "VQAonline_00054264.png", "question": "What is top N% of hands in PokerStove?", "context": "In PokerStove FAQ;\n\nThe slider interface for setting the top N% of hands orders the hands\n according to their preflop all-in equity versus three random hands.\n This rather arbitrary selection was picked because it balances the\n value of high cards with the value of drawing cards. It is not an\n absolute ordering, and depending on the specific situation you may\n want to edit that range of hands when doing equity calculations.\n\nSo, what is exactly N% ? Is it equal to VPIP/PFR ? How can I calculate this statistic healthy?\n\n", "answer": "To answer the question, \"What is exactly N% of top hands?\", there really is no correct answer as it depends on the stage of play (i.e. Preflop, Postflop) that's currently occuring. Specifically, it isn't VPIP/PFR.\nMaking use of these programs and hand-rankings is a matter of hand reading skill. The hand-rankings are a generalization of how a player should enumerate the plausible combinations in their opponents range according to the situation at hand.\nJust as the Pokerstove FAQ mentions, hand-ranking systems are arbitrary because the nature of the game will rarely, if ever, present players with situations that conform to linear orderings of hand equity match-ups. To put it another way that's well known, \"it depends\"!\nLooking a little deeper, there have been a number of methods used to rank hand match-ups according to differing scenarios, preflop and postflop.\nPokerstove uses, as they mention, an arbitrary measure of All-In equity vs. Three random hands. What is commonly called hot-and-cold equity, as it relates to showdown equities without any extra raises or folds involved, from any street onward. The full equity breakdown is found in a zip file through this page here: Holdem Preflop Match-Ups.\nOther hand-ranking systems such as Sklansky-Karlson (aka. Sklansky-Chubukov), the inception of which is here: The Sklansky-Karlson All In No Limit Holdem Rankings I believe relate to a limited game of SB vs. BB dynamics and specifically push-fold ranges, more suited to tournament scenarios.\nPokerProTools uses an \"evolutionary computer simulation, whereby the set of \"good hands\" is gradually refined, and hands are ranked against other \"good hands\"\". It's based on a scientific papers findings (The Challenge of Poker, Billings et al.). I've skimmed, and read over this paper a few times and, as you'd expect, it's a dense scientific piece of writing but still carries lots of insight in plain English about determining relative hand strength. This is where I found it some time ago: Towards a Meaningful Ordering of Hands\nI'm sure there are more rankings systems than these, as I can recall a few being mentioned on Poker.SE already, but these are probably the most referenced versions.\nI came to realise that good observation of your opponents tendencies should cause you to create detailed individualised hand-rankings based on your opponents play, and that these systems are good as starting points, but won't ever be definitive examinations of what your opponents are doing in real time.\n", "topic": "poker", "url": "https://poker.stackexchange.com/questions/993"}, {"image": "VQAonline_00054445.png", "question": "How much \"Russian money\" have Russian oligarchs injected into the UK's economy and contributed to UK politicians?", "context": "The following compares statements by two very disparate British commentators, Christopher Steele and the fictional reporter Johnathan Pie.\nSteele\nThe notes below Sky News' April 1, 2022 Beth Rigby Interviews... : Ex British spy says he can't see a way back for Putin say that \"Christopher Steele ran the Russia desk at MI6\". After about 08:13 Sky News' political editor Beth Rigby asks him about prominent Russians active in London's political and business scene:\n\nRigby: And on the issue of how the west has acted to Putin, I want to ask you a bit about the relationship between the UK and Russia. And about Oligarchs and money - Russian money in London.\nIn the UK for years, you were monitoring Russia and the misdeeds of its spies like murdering people in the UK. And yet you also saw politicians and businessmen; they'd go into parties and holidaying with oligarchs. Do you think that the establishment here let themselves get seduced?\nSteele: Absolutely. I think there were two things going on there. One is an element of naivety, because many of the Russians who come to London are very urbane, very convincing, very sort-of westernized. They don't represent the \"real Russia\" if you like. There is a dark side to Russia which not many people understand. Sadly I've been studying it for the last 20 years.\nBut I also think in addition to that, Russian money -- a wall of money that hit London; it wasn't just Russian but it was Russian money first I think -- sort-of set up a lobby, a group of interests who serviced this elite, who... the lawyers, the estate agents, the PR firms, and so on. And it became a pretty important part of our economy. And I suspect, also in terms of investment and so on, particularly after Brexit and what have you when we were looking for other forms of investment in the country.\nThis golden visa scheme which is now so controversial and in retrospect was such a bad mistake, grows out of that problem.\n\nand in response to Rigby's questions about Evgeny Lebedev being \"nominated for a life peerage by UK prime minister Boris Johnson\" and and about other prominent Russian business people operating in London:\n\nSteele: ...this is such a controversial appointment, and the story out there is that it was against the advice of the security services, so I think it's a pretty exceptional case. And there was a debate in Parliament the other day and I think probably the outcome of that should be more transparency.\nRigby: You have some experience, would you think that that has credibility; that the security services would have been anxious about this appointment?\nSteele: I think any sort-of family member of a former KGB officer or a KGB agent -- and actually Lebedev is not the only one, there are quite a few former KGB officers who are quite prominent in Britain and in business here who need to be very closely looked at. And certainly it's surprising to me that the've managed to operate in the country in the way that they have.\n\nPie\nFrom the New York Times; March 11, 2022 Opinion video Jonathan Pie: How Putin Weaponized London\u2019s Greed | NYT Opinion after 00:26:\n\nPie: I don't really know what to say. For the first time in my career, there are no words or explitives that can do this situation justice.\n\nand that not being very likely, he goes on to say\n\nHere in the UK, we don't have quite the same problem. Only five percent of our gas comes from Russia. Here we're reliant on something far more precious -- Russian money.\n\nFor 20 years, Putin has been using illicit money to fund an oligarchy of billionaires to keep his corrupt regime in power. And for 20 years, those Kremlin-approved oligarchs have been coming here to \"invest\" their ill-gotten gains via a loophole that allows people to buy property in the UK anonymously.\n[...] Welcome to Londongrad, Moscoow-on-Thaimes. Eighty-seven thousand homes in Britain were bought through off-shore companies by anonymous millionaires. At least 1.5 billion1 pounds' worth of those are owned by Russians with direct links to the Kremlin.\n[...] We have let it happen for years, because it's because it's how the UK makes its money. It's the only industry we have left.\n[...] Speaking of Boris Johnson, he's up to his (ears) in it as well. Since he became prime minister, Tories have accepted 2 million pounds in Russian donations. I know in American terms, 2 million is the political donation equivalent of tipping the bellboy, but here we're much more strict. We put a limit on the amount of dirty, illegal, kleptocratic money that our main political parties can take, which has an upside for Putin because it means British politicians are incredibly cheap to buy.\nIn 2020, 14 members of Johnson's government received Russian cash. The Tory party chairman even runs a luxury concierge service that helps oligarchs buy property in London.\n\nQuestion:\nHow much \"Russian money\" have Russian oligarchs injected into the UK's economy and contributed to UK politicians? Pie offers some bits and pieces but at this point in time surely some authoritative source has compiled at least a rough enumeration.\n\n1\"billion pounds\" Somebody help me here; is that one thousand million or one million million? update: comment suggest one thousand million.\n", "answer": "It's very difficult to put an absolute figure on this. Firstly, it's important to differentiate between Russian investment in the UK & Russian donations to politicians. In the first case, Walker's character appears to be referencing a February 2022 report by anti-corruption organisation Transparency International, which presents the \u00a31.5 billion figure, based on figures from June 2021:\n\n\u00a31.5 billion worth of property was bought by Russians accused of corruption or links to the Kremlin.\nNearly \u00a3430 million worth (28.3% by value) is in the City of Westminster, while \u00a3283 million (18.8%) is in Kensington and Chelsea. A more detailed breakdown can be found below.\nAnalysis of how this \u00a31.5 billion is owned reveals \u00a3830 million worth (55% by value) is held by companies in Britain\u2019s Overseas Territories and Crown Dependencies. The secrecy provided by these offshore financial centres is often used by those seeking to hide their ownership of assets.\n\nIt's not clear that this represents the whole picture, however. The House of Commons Library's April 1st 2022 report, Countering Russian influence in the UK, notes that putting an exact figure on the amount of Russian investment in the UK is impossible due to \"the difficulty in tracing the beneficial (ultimate) owners of some assets\", but estimates that it totals \"tens of billions\". Vladimir Ashurkov of the Anti-Corruption Foundation told the Foreign Affairs Select Committee in 2018 (Question 98) that \"there have been some estimates of money coming from Russia to Britain over the last 20 years\u2014100 billion is probably a good number. Most of that money is legitimate; I think only a fraction of that is the proceeds of corruption and crime.\"\nThis is borne out by the number of investor visas issued to Russian applicants - the Tier 1 investment visa granted permission to stay in the UK for up to 5 years in return for a minimum investment of \u00a32 million until it was ended in February 2022. The Commons Library report gives a figure of over 2,500 of these visas having been issued to Russian applicants, representing at least \u00a35 billion of investment. The report by the Intelligence and Security Committee on Russia gave the following explanation for the Government's promotion of the scheme:\n\nThe UK welcomed Russian money, and few questions \u2013 if any \u2013 were asked about the provenance of this considerable wealth. It appears that the UK Government at the time held the belief (more perhaps in hope than expectation) that developing links with major Russian companies would promote good governance by encouraging ethical and transparent practices, and the adoption of a law-based commercial environment.\n\nAs far as direct donations to politicians and political parties go, however, it's possible to be a little more precise. The figure that has been reported in the last few weeks, ~\u00a32.3 million in donations to the Conservative Party since Boris Johnson became leader, is based on calculations made on figures published by the Electoral Commission - predominantly donations made by Lubov Chernukhin, Alexander Temerko, and Mohamed Amersi, of \u00a3700,000, \u00a3357,000 and \u00a3258,000 respectively. On the other hand, the Labour party has received more than \u00a31 million from financier Martin Taylor, whose Nevsky Fund has major investments in Russia.\nUnder UK law, all donations must be reported to the Electoral Commission and all donations to candidates over the sum of \u00a350 must come from an individual on the UK electoral register - essentially, if you can vote for a party, you are eligible to donate to it. The argument that has been put forward by the recipients of these donations is that if a voter is on the electoral register, they can hardly be described as a 'Russian oligarch'. It's almost impossible, however, to definitively determine the ultimate source of money donated, even if the last stop is a permissible source. The result is often spurious but eye-catching headline figures obtained by journalists scouring the register for donations made by British individuals with Russian-sounding names, and it remains incredibly difficult to accurately trace the initial source of any donated wealth.\n", "topic": "politics", "url": "https://politics.stackexchange.com/questions/72297"}, {"image": "VQAonline_00054389.png", "question": "Lawful for some English public hospitals to stock more kinds of medications than others?", "context": "Left to right \u2013 Guy's, Cambridgeshire, Salisbury, Oxford. \n\n\nHow's it legal for NHS trusts to have different formularies? \nAssume you live in Cambridge, but your migraine can be only treated with a triptan on Oxford's formulary, but not Cambridgeshire's, like Eletriptan. What do you do? Must you go to a NHS hospital in Oxford? \n\nI don't want to cram too many pics in the collage, but Southampton has Almotriptan, Sumatriptan and Zolmitriptan, but Herefordshire just the latter two. \n", "answer": "There is no legislative requirement for individual hospitals, primary care facilities or secondary care facilities to provide the same treatments or specific level of care across NHS England, but there are legislative requirements to reduce the inequalities in healthcare across the NHS.\nThe Health and Social Care Act 2012 introduced general legislation requiring the NHS to reduce the effects of healthcare inequality, but it does not go into specifics such as requiring all hospitals to fund the same treatments.\nAs a result, patients have a legal right to choose where to receive treatment in secondary care facilities (ie, hospitals) once they have a referral, but no specific right to choose any GP they wish (GP surgeries have specific catchment areas, outside of which they can refuse you as a patient - hospitals in England have no such limitations and can only refuse you if they do not provide the treatment you are seeking).\nPart of the point of splitting up NHS England into various Clinical Commissioning Groups (the descendant of Primary Care Trusts, which ceased to exist in 2013) is that healthcare requirements are not the same across the country, and some areas need more funding in certain healthcare areas than others. Requiring the same level of care for all healthcare issues across the board all over the country would mean some areas are underfunded for some outcomes and overfunded for others.\nIn short, you need a referral to a specialist from your GP, and then you can choose to go to a hospital in a CCG which funds the treatment you are seeking. This may not always work, as the treating CCG can request the referring CCG to fund the work and thus you may be rejected anyway. You also have the option of funding the drugs yourself.\n", "topic": "politics", "url": "https://politics.stackexchange.com/questions/47864"}, {"image": "VQAonline_00054422.png", "question": "Today, 6 Jan 2021, Twitter deleted three tweets by Donald Trump. What did they say?", "context": "What did the three tweets say?\n\n", "answer": "All of Donald Trump's deleted tweets and retweets can be found here on factba.se, which is a site that tracks what Trump and other political figures say or tweet.\nYou can see screenshots of the original tweets on that site. Here are the transcripts of the regular tweets:\n\nMike Pence didn't have the courage to do what should have been done to protect our Country and our Constitution, giving States a chance to certify a corrected set of facts, not the fraudulent or inaccurate ones which they were asked to previously certify. USA demands the truth!\n\n\nThese are the things and events that happen when a sacred landlside election victory is so unceremoniously & viciously stripped away from great patriots who have been badly & unfairly treated for so long. Go home with love & in peace. Remember this day forever!\n\nAnd the transcript of the video he posted:\n\nI know your pain. I know you\u2019re hurt. We had an election that was stolen from us. It was a landslide election, and everyone knows it, especially the other side, but you have to go home now. We have to have peace. We have to have law and order. We have to respect our great people in law and order. We don\u2019t want anybody hurt. It\u2019s a very tough period of time. There\u2019s never been a time like this where such a thing happened, where they could take it away from all of us, from me, from you, from our country. This was a fraudulent election, but we can\u2019t play into the hands of these people. We have to have peace. So go home. We love you. You\u2019re very special. You\u2019ve seen what happens. You see the way others are treated that are so bad and so evil. I know how you feel. But go home and go home at peace.\n\nTwitter's TwitterSafety account stated that they removed these tweets for repeated violations of their policies that went above their usual public-interest policy which allowed other violations to go undeleted. His account is locked until the tweets are deleted, with a minimum 12-hour suspension, and it may be suspended if he continues to break their policies.\n", "topic": "politics", "url": "https://politics.stackexchange.com/questions/61468"}, {"image": "VQAonline_00054347.png", "question": "Can a President mandate Upvotes?", "context": "This image is from when Barack Obama (the then serving president) thanked Reddit for helping protect the open internet.\n\n\nThanks Redditors! Wish I could upvote every one of you for helping\n keep the internet open and free!\n\nMy question, if this was taken at face-value, and the President really wanted to force a private company to help him apply a blanket Upvote to everybody on Reddit, why could/can't he, and what would he have to do to accomplish this?\nI'm pretty sure this isn't allowed by Reddit's policies given it's a form of \"vote cheating\" (albeit the rules are oddly shaky for a blanket Upvote).\nI know realistically nothing about the extent of power a President has over the functions of a private company, so I was hoping someone could break down the various methods the President could go about giving an Upvote to every user on Reddit.\nI would prefer answers that avoided simply asking nicely (given not every President has had the same rapport with the company behind Reddit so lets assume they don't want this to happen) and instead focused on the steps and barriers the President would face before being provided with a hopefully enormous glorious green button. Or, if such an mandate is actually legally impossible given the checks and balances I've heard about.\n", "answer": "A president has the power, under the general terms of Article 2, to instruct the various federal agencies how to act, and how to spend the money that they already have. Any such order is then subject to scrutiny by Congress and the SCOTUS.\nThe ability of the President to instruct a private company is limited. In 1952, with a steel strike threatened, President Truman signed an order renationalising the steel mills. This was overturned by the Supreme Court (Youngstown Sheet & Tube Co. v. Sawyer), who ruled that even though the President had legitimate national security concerns (steel was essential for the Korean war effort), he lacked the authority to do so. The court decided that there are limits to what the constitution calls \"executive power\", and that seizing property is beyond that limit.\nIn this case of upvoting all redditors, the president is not attempting to take complete control of a private company, but he is attempting to exert authority over a company to act in a way other than what they see as their best business practice. And the indication of Youngstown Sheet & Tube Co. v. Sawyer is that the President lacks this power.\nHowever, the president would be free to create a Department of Reddit Upvoting and get federal employees to log onto Reddit and start upvoting everyone. That would be the President directing the executive how to act. Congress would be able to review this and prevent it by refusing to fund such a department, or simply overruling the order. \nThe President also has various tools that he could use to put pressure on a private company. He can make business difficult for a company while \"asking nicely\": \"Give a upvote to everyone and I won't send the IRS round to audit your books every Monday\" but probably less crude. The president has a lot of soft power so a bit of carrot and stick might well be the best strategy.\n", "topic": "politics", "url": "https://politics.stackexchange.com/questions/26027"}, {"image": "VQAonline_00054306.png", "question": "Can a country withdraw from a free trade agreement (NAFTA)?", "context": "The North American Free Trade Agreement (NAFTA) was established in 1994, with support from the national governments of Canada, the United States, and Mexico.\nIf one of these governments suddenly wanted to, could they withdraw from NAFTA? Or could they force a member of NAFTA, perhaps Mexico, to leave?\n\n", "answer": "Each free trade agreement sets the terms, so the answer will vary by FTA. In the case of NAFTA, the terms of withdrawal are dictated in Article 2205. Specifically:\n\nArticle 2205: Withdrawal\nA Party may withdraw from this Agreement six months after it provides written notice of withdrawal to the other Parties. If a Party withdraws, the Agreement shall remain in force for the remaining Parties.\n\nThe 6 month written notification is fairly standard for most international agreements, free trade or otherwise.\nArticle 2205 also answers whether or not a country can force another out: No.\nIf one country were to leave NAFTA, the others would still have the agreement, at least nominally, but if that country were the U.S., Mexico and Canada would have a hard time making NAFTA worthwhile.\nThe U.S. could, in theory, agree to leave NAFTA at the same time as Canada, and then immediately resign an identical treaty with Canada, which would have the effect of kicking Mexico out. The realities of politics in both countries make such a course of action unlikely, as once the treaty is open for renegotiation, then interest groups in both countries would want to seek better terms than they got 20 years ago.\n", "topic": "politics", "url": "https://politics.stackexchange.com/questions/10739"}, {"image": "VQAonline_00054386.png", "question": "In CBC's poll tracker, why do the probabilities of winning not add up to 100%?", "context": "In CBC's poll tracker at: https://newsinteractives.cbc.ca/elections/poll-tracker/canada/, probabilities of winning the 2019 Canadian federal election are provided. \n\nCurrently, these are 11% Liberal majority, 39% Liberal minority, 42% Conservative minority, and 6% Conservative majority. By my count, these probabilities add up to 98%. Shouldn't this be 100%?\n", "answer": "I don't see any explanation on their Full Methodology Info page, but when percentages don't add up to 100%, the usual answer is that it's due to rounding, so 11% might actually be 11.4%, which was rounded down to 11. Among 4 options, rounding could explain almost all of the missing 2%.\nAnother possibility, is that these 4 options don't represent all possibilities. For example, it's possible that the Liberals and Conservatives get equal numbers of seats, but neither has a majority (due to seats gained by other parties)\n", "topic": "politics", "url": "https://politics.stackexchange.com/questions/46667"}, {"image": "VQAonline_00054351.png", "question": "How do I interpret this table of the number of UK bills for the session 2016-2017?", "context": "Does the following table mean that 6 of 33 bills that were passed in that year, originated in the House of Lords?\n\nSource.\n", "answer": "Yes.\nAs noted here:\n\nBills are introduced in either the House of Commons or House of Lords for examination, discussion and amendment.\n\nAlthough most government bills are introduced in the House of Commons (HoC), parliamentary time is a scarce resource, and for reasons of scheduling bills are sometimes introduced in the House of Lords (HoL) instead. The bill must still be debated in and passed by the HoC at a later stage.\nFor example, scrolling down past the first page of the document in the OP, we can see one of the 6 bills introduced in the Lords was the Bus Services Bill, which eventually became the Bus Services Act 2017.\nThe government must be able to command a majority in the HoC; but it also has a presence in the HoL, through members of the HoL who are affiliated to the governing party or parties. The Leader of the Lords is a Cabinet minister specifically responsible for managing government business in the HoL. (He or she has a counterpart in the HoC, who surprisingly enough is known as the Leader of the Commons.)\n", "topic": "politics", "url": "https://politics.stackexchange.com/questions/26398"}, {"image": "VQAonline_00054482.png", "question": "\"N\u00f3s ossos que aqui estamos pelos vossos esperamos\" - n\u00e3o deveria ter uma v\u00edrgula?", "context": "Na entrada da Capela dos Ossos, em Portugal, h\u00e1 a seguinte frase:\n\nNos ossos qve aqvi estamos pelos vossos esperamos\n\n\nPara dar mais contexto \u00e0 mensagem, aqui est\u00e1 uma breve explica\u00e7\u00e3o encontrada na Wikip\u00e9dia:\n\nFoi constru\u00edda no s\u00e9culo XVII por iniciativa de tr\u00eas monges franciscanos que, dentro do esp\u00edrito da altura (contra-reforma religiosa, de acordo com as normativas do Conc\u00edlio de Trento), pretendeu transmitir a mensagem da transitoriedade da vida, tal como se depreende do c\u00e9lebre aviso \u00e0 entrada: \"N\u00f3s ossos que aqui estamos pelos vossos esperamos\".\n\nLer essa frase me parece estranho, sinto a falta de uma v\u00edrgula (ou duas). O portugu\u00eas mudou bastante do s\u00e9culo XVII para c\u00e1, e por isso estou em d\u00favida se isso \u00e9 apenas uma varia\u00e7\u00e3o da l\u00edngua ao longo do tempo. Imagino que a frase pudesse ser escrita das seguintes maneiras:\n\nN\u00f3s ossos que aqui estamos, pelos vossos esperamos.\nN\u00f3s, ossos que aqui estamos, pelos vossos esperamos.\n\nRealmente faltam v\u00edrgulas na frase original? Se sim, ambas op\u00e7\u00f5es acima seriam corretas? E, como b\u00f4nus, essa necessidade da v\u00edrgula \u00e9 realmente recente ou n\u00e3o se encontra na entrada por outros motivos?\n", "answer": "\nRealmente faltam v\u00edrgulas na frase original? Se sim, ambas op\u00e7\u00f5es acima seriam corretas?\n\nSe considerarmos\n\n\"ossos que aqui estamos\" como um aposto explicativo, deve estar separado por pontua\u00e7\u00e3o do restante da frase, portanto op\u00e7\u00e3o 2 \u00e9 a correta.\n\n\"n\u00f3s ossos\" como o sujeito e \"que aqui estamos\" como uma ora\u00e7\u00e3o subordinada adjetiva explicativa, deve estar separada por v\u00edrgulas do restante da frase, portanto a op\u00e7\u00e3o 2 \u00e9 a correta.\n\n\"n\u00f3s ossos\" como o sujeito e \"que aqui estamos\" como uma ora\u00e7\u00e3o subordinada adjetiva restritiva, n\u00e3o pode ser separada por v\u00edrgulas do restante da frase, portanto est\u00e1 correta a frase original.\n\n\nAs tr\u00eas interpreta\u00e7\u00f5es s\u00e3o aceit\u00e1veis a meu ver, ent\u00e3o concluo que, haja vista a \u00faltima interpreta\u00e7\u00e3o, n\u00e3o deveria ter uma v\u00edrgula obrigat\u00f2riamente, embora a op\u00e7\u00e3o 2 tamb\u00e9m seja aceit\u00e1vel.\n", "topic": "portuguese", "url": "https://portuguese.stackexchange.com/questions/8953"}, {"image": "VQAonline_00054472.png", "question": "Qual \u00e9 o nome dumas sali\u00eancias transversais nas lombadas de alguns livros de capa de couro?", "context": "Muitos livros antigos encadernados a couro t\u00eam umas coisas salientes atravessadas nas lombadas, que eu assinalei na figura abaixo. Aquilo \u00e9 os fios que unem as folhas que passam por baixo daquilo. Mas qual \u00e9 o nome daquilo? \n\n", "answer": "Um termo que encontrei foi nervura:\n\nSegundo a Infopedia: sali\u00eancia transversal na lombada dos livros encadernados\nNeste artigo \u00e9 mencionando, na p\u00e1gina 4 do PDF, v\u00e1rios significados de \"nervura\", entre eles: Na tipografia, \"nervura\" se refere \u00e0 sali\u00eancia transversal das lombadas dos livros encadernados.\n\nNo livro intitulado \"Livro\" (Michel Melot), h\u00e1 tamb\u00e9m uma refer\u00eancia ao termo nervo:\n\nNo passado, as encaderna\u00e7\u00f5es dos livros eram mantidas por um sistema complicado de fios cosidos sobre os cadernos, o que os deixava salientes no dorso. Estas sali\u00eancias dos nervos marcavam linhas horizontais paralelas, deixando-as bem vis\u00edveis sob a cobertura do couro. As t\u00e9cnicas de encaderna\u00e7\u00e3o h\u00e1 muito tornaram desnecess\u00e1rias estas sali\u00eancias, as quais, entretanto, continuam marcadas por aquilo que os encadernadores chamam de \"falsos nervos\".\n\n", "topic": "portuguese", "url": "https://portuguese.stackexchange.com/questions/4708"}, {"image": "VQAonline_00054483.png", "question": "\"Cada empresa tem o valor que merece ser negociado\" - isto est\u00e1 correto?", "context": "Estou lendo um livro e achei a frase:\n\nCada empresa tem o valor que merece ser negociado.\n\nEu escreveria:\n\nCada empresa tem o valor que merece ser negociada.\n\nem concord\u00e2ncia com \"cada empresa\".\nFiz uma pesquisa r\u00e1pida no Google, e n\u00e3o achei nada. Apesar de adorar portugu\u00eas, sempre erro algo, aqui, ali!\nEstaria o livro correto?\n==\nCompleto\n\nFonte: https://www.amazon.com.br/Valuation-Precificar-A%C3%A7%C3%B5es-Alexandre-P%C3%B3voa/dp/8535253750\n", "answer": "A frase est\u00e1 gramaticalmente correta. Se faz sentido no contexto, \u00e9 que eu n\u00e3o posso dizer, porque n\u00e3o tenho o contexto. Uma maneira intuitiva de desmontar a frase \u00e9 assim:\n\nCada empresa tem o valor.\nQual valor?\nO valor que merece ser negociado.\n\nO constituinte que merece ser negociado \u00e9 uma ora\u00e7\u00e3o subordinada adjetiva restritiva que modifica o nome valor, e o valor que merece ser negociado \u00e9 o complemento direto de tem.\nA alternativa que tu prop\u00f5es \u2015 \u201ccada empresa tem o valor que merece ser negociada\u201d \u2015 \u00e9 que \u00e9 muito estranha; por duas raz\u00f5es. Nesse caso, a empresa \u00e9 que merece ser negociada, mas a ordem dos termos \u00e9 estranh\u00edssima, porque a ora\u00e7\u00e3o subordinada (a negrito) fica separada do nome que modifica; o normal seria:\n\nCada empresa que merece ser negociada tem o valor\n\nDepois, a empresa tem o valor? Qual valor? Na frase original, n\u00f3s sabemos que \u00e9 o valor que merece ser negociado.\nH\u00e1 casos em que a tua quest\u00e3o realmente se p\u00f5e. Por exemplo, as duas frases seguintes est\u00e3o corretas:\n\n(a) Comprei um banco de madeira muito rara.\n(b) Comprei um banco de madeira muito raro.\n\nEm (a), a madeira \u00e9 que \u00e9 rara (\u00e0 parte disso, o banco pode nem ter interesse; podemos querer desmont\u00e1-lo e aproveitar a madeira para outra coisa); em (b) o banco, que at\u00e9 pode ser de madeira barata, \u00e9 que \u00e9 raro. Repara que, ao contr\u00e1rio da tua alternativa, muito raro/a fica em qualquer dos casos junto ao nome que modifica, s\u00f3 madeira em (a), banco de madeira em (b).\n", "topic": "portuguese", "url": "https://portuguese.stackexchange.com/questions/9331"}, {"image": "VQAonline_00054470.png", "question": "Reg\u00eancia do verbo lembrar - comentando um post na rede social Linkedin", "context": "Vi a frase a seguir na rede social Linkedin.\n\nQuando fui fazer um coment\u00e1rio e fiquei com d\u00favida na reg\u00eancia do verbo lembrar.\nExemplo:\n\nI) Tem raz\u00e3o, \u00e9 bom lembrar-se dessa liberdade :). Para nos fazer esquec\u00ea-la existem muitas pessoas.\nII) Tem raz\u00e3o, \u00e9 bom lembrar dessa liberdade :). Para nos fazer esquec\u00ea-la existem muitas pessoas.\nIII) Tem raz\u00e3o, \u00e9 bom nos lembrar dessa liberdade :). Para nos fazer esquec\u00ea-la existem muitas pessoas.\nIV) Tem raz\u00e3o, \u00e9 bom lembrar essa liberdade :). Para nos fazer esquec\u00ea-la existem muitas pessoas.\n\nQual seria a melhor op\u00e7\u00e3o? Por qu\u00ea?\n", "answer": "Lembrar tem muitas reg\u00eancias poss\u00edveis. As reg\u00eancias ideais dependem do que queres dizer exatamente. Apresento aqui as que me parecem relevantes para a tua pergunta.\n1. \u2018manter na mem\u00f3ria, pensar em\u2019\u2015lembrar algo:\n\n(IV) \u00e9 bom lembrar essa liberdade\n\u00e9 bom lembrarmos essa liberdade\n\nPortanto aqui estarias dizer, por exemplo, que as pessoas que mant\u00eam na sua mente a liberdade tiram da\u00ed prazer, ou que \u00e9 bom para a sociedade que n\u00f3s mantenhamos a liberdade nas nossas mentes, ou at\u00e9 que \u00e9 um dever nosso manter a liberdade nas nossa mentes.\nEncontramos esta reg\u00eancia para este sentido em v\u00e1rios dicion\u00e1rios:\n\nlembrar bons momentos/a inf\u00e2ncia (Aulete 1)\nLembrou o tempo em que vivia no Rio (Michaelis 2)\nlembravam a figura do Jo\u00e3o com saudade (Houaiss, Lisboa, 2003)\n\n2. Parecido com o anterior, mas com mais \u00eanfase em \u2018n\u00e3o esquecer\u2019 ou em \u2018trazer \u00e0 ideia, \u00e0 aten\u00e7\u00e3o consciente\u2019\u2015lembrar-se de algo:\n\n\u00e9 bom nos lembrarmos dessa liberdade / \u00e9 bom lembrarmo-nos dessa liberdade\n\n\u00c9 tamb\u00e9m poss\u00edvel \u00e9 bom lembrar-se dessa liberdade para dar um conselho a algu\u00e9m:\n\n(I) \u00e9 bom [voc\u00ea/o senhor] lembrar-se dessa liberdade\n\nExemplos com esta reg\u00eancia e sentido:\n\nj\u00e1 n\u00e3o se lembrava de onde tinha posto a chave (Aulete 1)\nComo vai a senhora, dona Alice? Lembra-se de mim? (Michaelis 2)\n\nEu encontro com este sentido no Google Books lembrar de algo (II, \u201c\u00e9 bom lembrar dessa liberdade\u201d). A mim soa-me estranho, porque \u00e9 uma reg\u00eancia que n\u00e3o se ouve em Portugal. Tamb\u00e9m n\u00e3o a encontro nos dicion\u00e1rios. Poder\u00e1 ser uma reg\u00eancia que seja aceite apenas coloquialmente no Brasil.\n3. \u2018chamar a aten\u00e7\u00e3o a algu\u00e9m para\u2019\u2015lembrar a algu\u00e9m algo ou lembrar algu\u00e9m de algo:\n\n\u00e9 bom [voc\u00ea] nos lembrar essa liberdade\n(III) \u00e9 bom [voc\u00ea] nos lembrar dessa liberdade\n\nLembrar algu\u00e9m e lembrar a algu\u00e9m ficam iguais quando o algu\u00e9m somos \u201cn\u00f3s\u201d. Aqui estarias a dizer que foi bom que a pessoa que postou a mensagem nos tivesse lembrado a/da liberdade. A minha primeira impress\u00e3o foi que n\u00e3o era este o sentido que pretendias, pois a frase n\u00e3o me soava muito bem sem explicitar o sujeito que lembra. Mas vendo melhor o contexto, provavelmente at\u00e9 \u00e9 mesmo isto que queres.\nJ\u00e1 temos uma pergunta sobre a reg\u00eancia nesta ace\u00e7\u00e3o: lembrei ao Jo\u00e3o o seu dever / lembrei o Jo\u00e3o do seu dever. Exemplos das duas reg\u00eancias neste sentido:\n\nlembrar a algu\u00e9m algo\nO rapaz lembrou ao pai o trato que haviam feito (Michaelis 1)\nlembrou ao marido que a conta de luz tinha que ser paga (Aulete 4)\nlembrar algu\u00e9m de algo\nLembrou o empregado de suas obriga\u00e7\u00f5es (Aulete 3)\nLembrei-o do documento que tinha que assinar (Aulete 4)\n\n\nEu foquei aqui apenas as ace\u00e7\u00f5es que me parecem relevantes para os teus exemplos e contexto. Lembrar pode ser usado com v\u00e1rias outras conota\u00e7\u00f5es. Nesta pergunta sobre lembrar vs recordar encontras uma discuss\u00e3o mais abrangente, mas naturalmente n\u00e3o dirigida \u00e0 tua pergunta em particular.\n", "topic": "portuguese", "url": "https://portuguese.stackexchange.com/questions/4391"}, {"image": "VQAonline_00054474.png", "question": "What does \"CONCOC\" stand for?", "context": "I wanted to check if \"Concoc\" was already a word or a brand name for anything, and it seems it is used in Brazillian videos, but I can't find it in a Portuguese dictionary.\nDoes CONCOC mean anything?\nI don't speak any Portuguese so I can't deduce the meaning from context.\n\n", "answer": "Concoc is a brand in Brazil. They are schools around the country. Usually, they give scholarship, so, these videos are tips to the test for scholarship.\nHere part of a news about it:\n\nO aul\u00e3o CONCOC ocorre no Cinesystem do Shopping Iguatemi, em\n Florian\u00f3polis, das 8h \u00e0s 12h. Das 14h \u00e0s 17h, do mesmo dia, ser\u00e1\n aplicada uma prova de conhecimentos gerais, na sede do COC, para os\n alunos concorrerem a bolsas de estudo na unidade. Todos os\n participantes da prova ganhar\u00e3o descontos na mensalidade do ensino\n m\u00e9dio.\n\nIn a quick and free translate: A class preparation for scholarship happened.\nSource: AO VIVO: assista ao Aul\u00e3o CONCOC em parceria com DC neste s\u00e1bado\n", "topic": "portuguese", "url": "https://portuguese.stackexchange.com/questions/5673"}, {"image": "VQAonline_00054455.png", "question": "Como se chama a parte da lata que serve para a abrir?", "context": "\u201cTodas\u201d as latas de refrigerante t\u00eam uma certa parte, no topo, que server para empurrar a aba, e abrir a lata. Como se chama essa parte?\n\n", "answer": "Em Portugal o nome dado a essa parte da lata \u00e9 anilha de lata. \n", "topic": "portuguese", "url": "https://portuguese.stackexchange.com/questions/277"}, {"image": "VQAonline_00054475.png", "question": "Qual \u00e9 a diferen\u00e7a entre \u00abs\u00edmbolo\u00bb e \u00abemblema\u00bb?", "context": "Estava a falar sobre futebol e disse:\n\n...j\u00e1 para n\u00e3o falar do s\u00edmbolo do Benfica1, que monocrom\u00e1tico fica\n feio.\n\nE algu\u00e9m me veio corrigir:\n\nS\u00edmbolo n\u00e3o, emblema. Os s\u00edmbolos s\u00e3o Coluna, Eus\u00e9bio, Bento,\n Chalana,...\n\nPelo que pesquisei parece que s\u00e3o sin\u00f3nimos, mas na priberam n\u00e3o faz nenhuma refer\u00eancia a isso. \nSer\u00e1 um regionalismo? S\u00e3o sin\u00f3nimos ou s\u00e3o diferentes?\n\n1 - Imagem do Simbolo/Emblema do SLBenfica\n\n", "answer": "\"S\u00edmbolo\" \u00e9 um termo mais amplo - por exemplo, n\u00e3o apenas o emblema, mas tamb\u00e9m seu uniforme \u00e9 um s\u00edmbolo do clube.\n\"Emblema\" \u00e9 o s\u00edmbolo oficial do time, sua ins\u00edgnia, divisa, marca, distintivo, mas apenas um dos s\u00edmbolos do clube, como fica claro em sua p\u00e1gina:\n\nO Emblema do Benfica \u00e9, sem d\u00favida alguma, a sua imagem de marca. Tal como a maior parte das tradi\u00e7\u00f5es e s\u00edmbolos do Clube, o emblema foi elaborado entre 13 de dezembro de 1903, quando surgiu a ideia de criar o Clube e a data da sua funda\u00e7\u00e3o, a 28 de fevereiro de 1904. \n\nNesse sentido, \"s\u00edmbolo\" n\u00e3o \u00e9 incorreto, mas \"emblema\" de fato pode ser prefer\u00edvel por ser mais espec\u00edfico.\n", "topic": "portuguese", "url": "https://portuguese.stackexchange.com/questions/6012"}, {"image": "VQAonline_00055659.png", "question": "Rosetta Stone puzzle", "context": "The answer to this puzzle is one word.\n\n", "answer": "The first line shows\n\n lots of threes.\n\nThe second says\n\n $\\left(\\frac{2838}{43\\cdot\\sqrt{36}}\\right)^3-\\frac{23}{360}\\cdot\\left(\\frac{238}{34}\\right)!$\n\nin\n\n $\\left(\\frac{\\textrm{Greek}}{\\textrm{Khmer}\\cdot\\sqrt{\\textrm{Armenian}}}\\right)^?-\\frac{\\textrm{modern Western}}{\\textrm{Chinese}}\\cdot\\left(\\frac{\\textrm{Hebrew}}{\\textrm{Indian}}\\right)!$ numerals\n\nyielding\n\n 1009\n\nwhich\n\n in Roman numerals, which we haven't used yet, gives MIX.\n\nConfessions:\n\n I don't know what the ? is, but it's clearly 3 (1) just from looking at it and (2) because it appears on the first line, which is all threes. I'm not sure whether \"Armenian\" is right. The thing I've labelled as Khmer seems like the first figure is Khmer and the second is Thai. I am not an expert on any of this stuff.\n\n", "topic": "puzzling", "url": "https://puzzling.stackexchange.com/questions/72638"}, {"image": "VQAonline_00055967.png", "question": "The Tripan Balance", "context": "\nThe tripan balance is used for comparing different weights. For example, when the pans carries 7 balls,5 balls and one is empty. The heaviest tilt to lowest level while the empty pan is at the highest level. In case of the same number of balls on two pans, both pans tilts to same level.\nTwelve solid balls looks identical but two of them is either lighter or heavier (say 1% different to normal balls). Trying the tripan balance, how much checkings will it takes to find out the odd balls?\n", "answer": "\n It's easy to do in 3 weighings. Just put 4 balls on each plate to start. If all three plates are at different heights, then you know the top plate has the light ball and the bottom plate has the heavy ball. Take the 4 in the top plate and put 3 of them individually into a plate. If one plate is light, there is the light ball. If not, it's the one you left out. Do the same for the heavy ball, and you're done in 3 weighings (4x4x4, 1x1x1, 1x1x1).\n\nOtherwise, \n\n the plates will be at the same height after the first weighing. This means that the heavy and light ball are together. Note which group each ball was in (mark them with 3 colors, for example), and rearrange them into 4 groups of 3, with each new group containing one ball from each of the original groups. Now put 3 of the new groups on plates. We know that the light and heavy ball are split up, so the plates cannot be at the same height. If one plate is light, then the light ball is there and the heavy ball is on the table. If one plate is heavy, then the heavy ball is there and the light ball is on the table. If the plates are all different heights, then you know where the light and heavy ball are.\n\nIn any event,\n\n we now know which group of 3 has the light ball, and which has the heavy ball. Pick one of those groups (say the heavy group) and put each of its balls on a plate. This will give you the heavy one, but also the light one - it's just the ball in the light group that has the same marking as the heavy ball. 3 weighings total (4x4x4, 3x3x3, 1x1x1).\n\n", "topic": "puzzling", "url": "https://puzzling.stackexchange.com/questions/87174"}, {"image": "VQAonline_00056742.png", "question": "Do your best! (I can't really say no pressure)", "context": "FROM:\n(-)(- -)(> \u2022 -)(- \u2022 - \u2022)(- - - - -)(- - - - - -)(> \u2022 - - - - -)(- - - - - - - -)(- - - - - - - - -)(> \u2022 - - - - - - - \u2022)\n 6 4 2 1 3 5 \n\n \n\nTO:\n\n\nWhat's the answer to this puzzle?\n\nNote: The brackets are just there to seperate them from each other... they're not needed once you realize what this puzzle is about.\n", "answer": "The upper diagram\n\n is, once line breaks are introduced in the obvious places, an upper portion of Pascal's triangle. Each \"\u2022\" represents one number from the triangle; each \">\u2022\" represents two numbers, the first always being a 1, concatenated to form a 2-digit number in base 10. Reading these in order of the numbers below them, we get PASCAL.\n\nThe lower diagram is\n\n is simply a triangle, so to get FROM the upper TO the lower we need to remove PASCAL. So perhaps the title shouldn't say \"no pressure\" after all. (The pascal is the SI unit of pressure.)\n\nIt's not quite clear to me whether\n\n \"the answer\" is meant to be PASCAL or NO PASCAL or even NO PRESSURE. :-)\n\n", "topic": "puzzling", "url": "https://puzzling.stackexchange.com/questions/118068"}, {"image": "VQAonline_00056366.png", "question": "4x4 grid equations", "context": "Can you place all numbers from 1 to 16 into cells, such that the following 8 equations hold? Note that the operator \"/\" only works for non-remainder division, i.e. you can have \"8 / 4\" but not \"8 / 3\". As usual multiplication and division are performed before addition and subtraction. Good luck!\n\n", "answer": "I think the answer is\n\n$$\\begin{array}{ccccccc}9 & + & 16 & \\div & 8 & = & 11 \\\\+&&-&&-&&+\\\\6 & \\div & 3 & + & 12 & = & 14 \\\\\\times&&\\times&&\\div&&\\div\\\\1 & + & 2 & + & 4 & = & 7 \\\\=&&=&&=&&=\\\\15 & - & 10 & \\div & 5 & = & 13\\end{array}$$\n\n(Partial) Explanation:\n\nLet's label the grid as follows:\n$$\\begin{array}{ccccccc}A & + & B & \\div & C & = & D \\\\+&&-&&-&&+\\\\E & \\div & F & + & G & = & H \\\\\\times&&\\times&&\\div&&\\div\\\\I & + & J & + & K & = & L \\\\=&&=&&=&&=\\\\M & - & N & \\div & O & = & P\\end{array}$$\nObserve that there are five divisions, none of which share any number. Since all numbers are distinct, the ratios must be at least 2, and therefore the divisors cannot exceed 8. The positions for divisors are $C, F, K, L, O$. Also since $I+J+K=L$, it follows that all of $I,J,K$ are less than $L$, and therefore they cannot exceed 8 either. So far, we've identified seven cells that cannot exceed 8: $C, F, I, J, K, L, O$. Also, if you're familiar with Kakuro, you can notice that 1 must be one of $I,J,K$ from $I+J+K=L \\le 8$.\n\n\nNow consider the two prime numbers 11 and 13. They cannot be a part of division (neither dividend nor divisor): They cannot divide anything else and they cannot be divided by anything else other than 1. The only place where 1 can divide something is $G \\div K$, but then $C$ becomes too high. Therefore, 11 and 13 must be placed in $ADPM$.\n\n\nAlso note that $A \\rightarrow D \\rightarrow P \\rightarrow M$ forms a strictly increasing sequence with gaps of at least 2, since the quotients are at least 2 and\n$$\\begin{align}A + B \\div C &= D \\\\D + H \\div L &= P \\\\P + N \\div O &= M\\end{align}$$\nSo 11 and 13 must be adjacent in the sequence. If they're assigned to $AD$, $M$ gets too large, so the only options are $D=11, P=13$ or $P=11, M=13$.\n\n\nAt this point, combined with the information that $HL$ can only be one of $(12,6),(14,7),(16,8)$, I started brute-forcing the cells, and I was lucky that I guessed the 3rd row and 4th column correctly early on.\n\nAs a bonus, a computer-based proof that the solution is unique (Python + Z3Py):\n\nimport z3\n\nsolver = z3.SolverFor('QF_FD')\nvarlist = [z3.Int(var) for var in 'abcdefghijklmnop']\nfor var in varlist:\n solver += 0 < var\n solver += var <= 16\nsolver += z3.Distinct(varlist)\n\na,b,c,d,e,f,g,h,i,j,k,l,m,n,o,p = varlist\n\nsolver += b == c * (d - a)\nsolver += e == f * (h - g)\nsolver += l == i + (j + k)\nsolver += n == o * (m - p)\nsolver += m == a + (e * i)\nsolver += n == b - (f * j)\nsolver += g == k * (c - o)\nsolver += h == l * (p - d)\nassert solver.check() != z3.unsat\nmodel = solver.model()\nfor var in varlist:\n print(model.eval(var), end=' ')\nprint()\nsolver += z3.Or(*[var != model.eval(var) for var in varlist])\nassert solver.check() == z3.unsat\nprint('No more solutions')\nOutput:\n9 16 8 11 6 3 12 14 1 2 4 7 15 10 5 13\nNo more solutions\n\n", "topic": "puzzling", "url": "https://puzzling.stackexchange.com/questions/102658"}, {"image": "VQAonline_00056479.png", "question": "A circle touches two sides of a triangle and two of its medians", "context": "A circle touches two sides of a triangle and two of its medians. Prove that the triangle is isosceles.\n\nThis problem came from the Mathematical Digest issue 62 (Jan 1986) which in turn cited a Russian mag called KVANT (meaning \"Quantum\").\n", "answer": "\n As the triangles ADC and BEC have the same incircle and their areas are equal (half that of ABC), so are their perimeters\n DC+DA+AC = EC+EB+BC\n or, subtracting from both sides CD+CE+DA+EB\n AE-EB = BD-DA.\n This means that D and E lie on the same pair of hyperbolas with foci A and B. Since they also have the same distance to the base AB (half that of C) the triangle must be isosceles by symmetry.\n\n", "topic": "puzzling", "url": "https://puzzling.stackexchange.com/questions/107801"}, {"image": "VQAonline_00055702.png", "question": "A Chess Lock Puzzle?", "context": "I was recently talking to one of my friends who played chess (much better than me, I'll admit). He said he had a different kind of chess problem, one that may be a little easier, or that may be a little bit harder than usual. He said it was a chess lock puzzle.\nHe then grabbed two(!) chess sets and set up the following position: \n\nHe challenged me to find the least number of moves to capture the black king, using the following rules: \n\n\nWhite gets an infinite number of moves. \nWhite may only capture undefended pieces, and the black king (checkmate isn't a thing, just capture the black king)\nRooks are the only keys that work. You must capture the black king with a rook. \n\n\nHe said his best was to do it in 17 moves. Can you do it quicker?\n", "answer": "I think there's a quicker way to do it.\n\n Rb2 Nb1 Rd2 Bd1 Rf2 B3e2 Rf4 Bgf3 Rg4 Rxg7 Rg8 Rxa8\n\nfor 12 moves.\n", "topic": "puzzling", "url": "https://puzzling.stackexchange.com/questions/74522"}, {"image": "VQAonline_00054876.png", "question": "Puzzling Pieces of Pumpkin Pi", "context": "\nYou bake a nice, delicious-looking pumpkin $\\pi$, cut it into 6 slices and serve it on your table. As a fan of numbers, you label each slice with numbers from 1 to 6.\nFeeling very pleased and satisfied, you go back into your kitchen to grab a glass of water. Once you get back to your table, instead of finding your pumpkin $\\pi$, you find a piece of paper with a pumpkin $\\pi$ drawn on it, with the numbers you'd labelled it with replaced by letters (as shown above).\nOh no - that's not good news.\nYou look at the back of the paper and find a message that reads:\n\n\"Har har... want to get your pumkpin $\\pi$ back? Not until you tell me the numbers labelled on your $\\pi$.\"\n\nBummer! Having a terrible memory, you have absolutely no idea how you labelled your $\\pi$.\nYou continue reading the message:\n\n\"Don't worry! Just in case you forgot, I left a liiiiiiitle bit of a clue on 6 separate pieces of paper. But mind you... I'm not always honest...\"\n\nSurprisingly, you find 6 pieces of paper under your table:\n\n\"1---- is adjacent to 4; is adjacent to 3\"\n\"2---- is adjacent to 4; is opposite 6\"\n\"3---- is opposite 4; is adjacent to 5\"\n\"4---- is adjacent to 6; is not adjacent to 5\"\n\"5---- is adjacent to 3; is not adjacent to 6\"\n\"6---- is not opposite 3; is not adjacent to 1\"\n\nEach piece of paper contains 2 pieces of information about how your numbers are labelled. You also know that on every paper, one piece of information is true while the other is false.\nCan you deduce the numbers labelled on your pumpkin $\\pi$?\n(Note: Please don't answer YES.)\n(Note 2: If 2 slices are next to each other, then they are adjacent. e.g. A and B. If 2 slices have 2 other slices between them, then they are opposite to each other. e.g. A and D.)\n", "answer": "From (5) we know that 5 is either adjacent to 3 and 6, or adjacent to neither of them. Assume it is adjacent to both:\n\n3 5 6 _ _ _\n\nFrom (6), since 6 is not opposite of 3, it must be adjacent to 1:\n\n3 5 6 1 _ _\n\nFrom (1), since 1 is not adjacent to 3, it must be adjacent to 4, and then the last remaining spot must be filled by 2:\n\n3 5 6 1 4 2\n\nBut this makes both of the claims in (2) true, which is a contradiction. So our original assumption is false, and we conclude that 5 is not adjacent to 3 or 6. From (3), this means that 3 is opposite of 4, and the only way that works is if 4 is adjacent to 5:\n\n5 4 _ _ 3 _\n\nFrom (4) we know that 4 is adjacent to 6:\n\n5 4 6 _ 3 _ \n\nFrom (6) we know that 6 is adjacent to 1, and the last remaining spot is filled by 2:\n\n5 4 6 1 3 2\n\nThis satisfies all the conditions of the problem.\n", "topic": "puzzling", "url": "https://puzzling.stackexchange.com/questions/31145"}, {"image": "VQAonline_00056973.png", "question": "Computing T-Bill Yield across leap year boundary", "context": "Consider this T-Bill (912796TE9) that was purchased on 2019-10-30 and matures on 2020-02-06:\n\nI'm trying to work through some of the basics of the yield calculation. \nThe days until maturity is 99. (2020-02-06 minus 2019-10-30). That's easy enough.\nThe price is 99.563575. That's equal to 100 - ((discount) * (99/360)). Again, pretty straightforward.\nWhere I'm running into trouble is the yield. According to this page we should compute:\nyield = ((100-price)/price) * (365/daysLeft)\n\nBy my calculations, that generates a yield of around 1.616095%. But as you can see from the screenshot, the actual yield is 1.620522%. So I'm off.\nNow it turns out, if I plug 366 into the equation instead of 365, I get the correct result. Why is that? Presumably it has something to do with 2020 being a leap year. But a good fraction of the holding period takes place in 2019 which is not a leap year. What's the rule on this? If any fraction of the holding period until maturity touches a leap year then 366 shall be used?\n", "answer": "Due to the leap year 366 days need to be used here to match UST conventions (which is ACT/ACT). In this case it doesn't matter whether your interest period extends to only 1 day after the 29th of February or, e.g., 200. In fact if you look at the daycount description of the bill it says:\n\n\"the day count basis for price and yield calculations is 365 depending on the number of actual days in year counting forward from the Original Issue Date. The basis will usually be 365 but if the year following the issue date includes February 29th then it's 366\".\n\n", "topic": "quant", "url": "https://quant.stackexchange.com/questions/49469"}, {"image": "VQAonline_00056882.png", "question": "EUR/CHF fx rate drop on the 15th of January 2015", "context": "I was playing around with some fx rates and what caught my attention was the sudden drop of the fx rate EUR/CHF at the 15th of January 2015 4:00 - 5:00 am EST. What really happened that day? Was there any prior information directed towards traders and market makers in the context of qualitative risk management to avoid heavy looses?\nIs there any paper that describes the situation and how biased the volatility forecasts can be from the outliers pollution ?\n\n", "answer": "What happened was totally unexpected end of peg against the euro @ 1.2CHF regime that Swiss central bank aborted. See some articles about it. As far as I know nobody in the markets knew, there was no indication whatsoever..\nIn terms of management, I'm afraid lots of people got heavy losses, particularly banks (Austria, Poland, Hungary) - lots of Swiss loans that got shockingly higher instalments overnight (by 20%) caused serious stress to portfolios of banks. \nSee for instance this good article. or google around this, to get more info - you can set google to look for content only at the beginning of 2015.\nRegarding academia, there does not seem to be anything particularly interesting: \nhttps://scholar.google.com/scholar?as_ylo=2015&q=swiss+franc+unpegging&hl=cs&as_sdt=0,5\nRegarding volatility forecasting, this would depend on your model, so hard to give a specific answer to that..\n", "topic": "quant", "url": "https://quant.stackexchange.com/questions/29694"}, {"image": "VQAonline_00056855.png", "question": "Linear combination of Payoffs using Black-Scholes", "context": "Write the payoffs in Figure 3.8 as linear combination of call options and derive a closed form formula for the Black-Scholes price, the Delta, and the Gamma of them. All the Greeks of the option are also linear combination of these call option Greeks. For instance, $$\\Delta(t,S) = \\Phi(d_1(\\tau,K_1,S)) - \\Phi(d_1(\\tau,K_2,S)) - \\Phi(d_1(\\tau,K_3,S)) + \\Phi(d_1(\\tau,K_4,S))$$\nPartial Solution: For the strangle we have a pay off of $$(K - S_T)_{+} + (S_T - K)_{+}$$ Therefore the closed form solution of B-S price of option is $$V(\\tau,S) = P(\\tau,K,S) + C(\\tau,K,S)$$ and the delta of the position is $$\\Delta(\\tau,S) = -\\Phi(-d_1(\\tau,K,S)) + \\Phi(d_1(\\tau,K,S))$$ Finally our gamma for this position is $$\\Gamma(\\tau,S) = \\frac{\\Phi'(d_1(\\tau,K,S)) + \\Phi'(d_1(\\tau,K,S))}{S\\sigma \\sqrt{\\tau}}$$\nI guess my professor made a mistake in regards to the B-S closed form price: for the strange it is $$V(\\tau,S) = (-S_0\\Phi(-d_1) + e^{-rT}K\\Phi(-d_2)) + (S_0\\Phi(d_1) - e^{-rT}K\\Phi(d_2))$$ and similar for the straddle\nwhere $\\tau = T - t$ not sure why we use $\\tau$ any explanation of that would be great. \n", "answer": "To express such payoff in mathematical form, it is better to use indicator functions. I assume that the bottom of graphs (i.e., the vertex for the left one and the bottom segment for the right side one) represents zero. \nFor the left-hand one, the payoff is given by\n\\begin{align*}\n(K-S_T)\\pmb{1}_{S_T \\le K} + (S_T-K)\\pmb{1}_{S_T \\ge K} = (K-S_T)^+ + (S_T-K)^+,\n\\end{align*}\nthat is, a straddle that involves both a European call and put with the same strike price and maturity date.\nFor the right-hand one, the payoff is given by\n\\begin{align*}\n(K_1-S_T)\\pmb{1}_{S_T \\le K_1} + (S_T-K_2)\\pmb{1}_{S_T \\ge K_2} = (K_1-S_T)^+ + (S_T-K_2)^+,\\tag{1}\n\\end{align*}\nthat is, a strangle that involves both a European call and put with the same maturity date, but different strikes.\nFor valuation, as an example, let's consider (1). According to the Black-Scholes' formula, the value of Payoff (1) is given by\n\\begin{align*}\nV=\\Big[K_1 e^{-rT} \\Phi(-d_2^1) - S_0 \\Phi(-d_1^1)\\Big] + \\Big[S_0 \\Phi(d_1^2) - K_2 e^{-rT} \\Phi(d_2^2)\\Big],\n\\end{align*}\nwhere the first term is the value of the put option payoff $(K_1-S_T)^+$ and the second is the value of the call option payoff $(S_T-K_2)^+$. Here,\n\\begin{align*}\nd_1^1 &= \\frac{\\ln \\frac{S_0}{K_1} + (r+\\frac{1}{2}\\sigma^2)T}{\\sigma \\sqrt{T}},\\\\\nd_2^1 &= d_1^1 - \\sigma \\sqrt{T},\\\\\nd_1^2 &= \\frac{\\ln \\frac{S_0}{K_2} + (r+\\frac{1}{2}\\sigma^2)T}{\\sigma \\sqrt{T}},\\\\\nd_2^2 &= d_1^2 - \\sigma \\sqrt{T}.\\\\\n\\end{align*}\nThe delta hedge ratio is the sum of deltas of the first put option and the second call options, that is,\n\\begin{align*}\n\\frac{\\partial V}{\\partial S_0} &= -\\Phi(-d_1^1) + \\Phi(d_1^2)\\\\\n&=\\Phi(d_1^1) + \\Phi(d_1^2) - 1,\n\\end{align*}\nand the gamma hedge ratio is the sum of gammas of the first put option and the second call options, that is,\n\\begin{align*}\n\\frac{\\partial^2 V}{\\partial S_0^2} &= \\frac{\\Phi'(d_1^1)}{S_0\\sigma \\sqrt{T}}+ \\frac{\\Phi'(d_1^2)}{S_0\\sigma \\sqrt{T}}\\\\\n&=\\frac{\\Phi'(d_1^1) + \\Phi'(d_1^2)}{S_0\\sigma \\sqrt{T}}.\n\\end{align*}\n", "topic": "quant", "url": "https://quant.stackexchange.com/questions/24927"}, {"image": "VQAonline_00056937.png", "question": "Struggling with tau in Black-Litterman", "context": "According to the omega formula in B-L tau is used in the Omega estimation to determine the degree of uncertainty given to views of the investor:\n\nSo, if tau is given a low value then the inverse of omega will be large and therefore suppose a lot of uncertainty in the investors view and so giving more importance to implicit returns in contrast to investors.\nIn short, and based on this assumption (don't hesitate in correct it) tau can be used to calibrate the importance I give to investors views in opposition to implicit returns and vice versa.\nSupposing the above statement is correct, according to \nThomas M. Idzorek paper on B-L model\nregarding page 15 he comments that \n\n\"When the covariance matrix\n of the error term (\n \u03a9\n ) is calculated using this method, \n the actual value \n of the scalar (\n \u03c4\n ) becomes irrelevant because only the ratio \n \u03c4\n \u03c9\n /\n enters \n the model. For example, changing the assumed value of the scalar (\n \u03c4\n ) from 0.025 to 15 \n dramatically changes the value \n of the diagonal elements of \n \u03a9\n , but the new Combined \n Return Vector (\n ]\n [\n R\n E\n ) is unaffected. \" \n\nI have made some calculations assigning tau a 0.025 and a 1 and the new Combined Return Vector is unaffected\nThen I understand that there is no way I can assign investors views a degree of uncertainty because it does not matter the value I assign to tau, that the return vector will be the same. \nTherefore, my questions are: \nAre my statements above correct?\nIf so , whats the point of the existence of tau?\nAre there alternative methods to determine investors uncertainty / weights given to implicit or investor returns ?\nThank you in advance for taking your time reading my post.\n", "answer": "Your statement about the properties of $\\tau$ is correct. $\\tau$ is a measure of uncertainty. I think the problem you are having is because in most practical situations nobody really knows what values should be used for $\\tau$ and/or $\\Omega$. \nThere is plenty of practical advice out there, and some of it is very confusing! For example, Jay Walters has written an entire paper on $\\tau$ (see https://papers.ssrn.com/sol3/papers.cfm?abstract_id=1701467). That paper discusses the origin of $\\tau$ as a measure of an investors uncertainty in their prior returns. Unfortunately, this does not translate into practical advice and it refers to different papers giving different values for $\\tau$ (0.05 in one paper and 1.0 in another). \nNobody really knows what $\\Omega$ should be either because that would require them to have an estimate of the uncertainty of the views. Most human-generated forecasts don\u2019t come with an associated covariance matrix. Hence, most people use an $\\Omega$ that is derived from their estimate of the variance matrix. So, for example, Idzorek\u2019s formula for $\\Omega$ assumes that the forecasts are independent when they are not likely to be in practice.\nIdzorek\u2019s formula has rolled $\\tau$ in to the estimate of $\\Omega$. That inclusion means that the effect of $\\tau$ is cancelled out in the Combined Return Vector. Hence, as you have noticed, you cannot use $\\tau$ as a single scalar to alter the weight of the prior in the final Combined Return Vector if you use Idzorek\u2019s formula. Including $\\tau$ has the advantage of giving one single result, but that is not necessarily what you need. \nA simple modification to Idzorek\u2019s formula would be to set $\\Omega = diag(P\\Sigma P\u2019)$. The Combined Return Vector would then change as $\\tau$ changes.\nAnother popular choice for $\\Omega$ is given in chapter 9 of Meucci\u2019s Risk And Asset Allocation (and also discussed at Black-Litterman, how to choose the uncertainty in the views $\\Omega$ for smooth transitions form prior to posterior). Meucci suggests that people use:\n$\\Omega = (\\frac{1}{c} -1) P\\Sigma P\u2019$\n", "topic": "quant", "url": "https://quant.stackexchange.com/questions/40820"}, {"image": "VQAonline_00057044.png", "question": "conditional expectation formula of default in CVA", "context": "Here is the formula of CVA in page 74 in book Modern Derivatives Pricing and Credit Exposure Analysis.\n\nHere $t_0 = tt_i}X(t_{i-1})|\\mathcal{F}_t] = E^Q\\Big[E^Q[\\mathbb{1}_{\\tau>t_i}]X(t_{i-1})|\\mathcal{F}_t\\Big]$$\nIt hints that\n\nThis is possible for the expectations containing $X(t_i)$ and $X(t_{i\u22121})$ since these are both $\\mathcal{F}(t_i)$-measurable; apply the tower law of conditional expectations.\n\nDoes that mean\n$$E^Q[\\mathbb{1}_{\\tau>t_i}X(t_{i-1})|\\mathcal{F}_t] = E^Q\\Big[E^Q[\\mathbb{1}_{\\tau>t_i}X(t_{i-1})|\\mathcal{F}_{t_{i-1}}]|\\mathcal{F}_t\\Big]=E^Q\\Big[E^Q[\\mathbb{1}_{\\tau>t_i}|\\mathcal{F}_{t_{i-1}}]X(t_{i-1})|\\mathcal{F}_t\\Big].$$\nBut how to convert $E^Q[\\mathbb{1}_{\\tau>t_i}|\\mathcal{F}_{t_{i-1}}]$ to $E^Q[\\mathbb{1}_{\\tau>t_i}]?$\n", "answer": "Note that, for any $u > 0$,\n\\begin{align*}\nE^Q(1_{\\tau > u} \\mid \\mathscr{F}_u) = e^{-\\int_0^u \\lambda(s)ds}.\n\\end{align*}\nFor example, given $\\lambda$, we can define the default time $\\tau$ as\n\\begin{align*}\n\\tau = \\inf\\left\\{t \\in \\mathbb{R}_+: e^{-\\int_0^t \\lambda_s ds} \\le \\xi \\right\\}, \n\\end{align*}\nwhere $\\xi$ is independent of $\\mathscr{F}_{\\infty}$ and is uniformly distributed over $(0, 1)$.\nThen\n\\begin{align*}\nE^Q\\left(1_{\\tau>t_i} X(t_{i-1}) \\mid \\mathscr{F}_t \\right) &= E^Q\\left(X(t_{i-1})E^Q(1_{\\tau>t_i} \\mid \\mathscr{F}_{t_i}\\big) \\mid \\mathscr{F}_t \\right)\\\\\n&=E^Q\\left(X(t_{i-1}) e^{-\\int_0^{t_i} \\lambda(s)ds} \\mid \\mathscr{F}_t \\right).\n\\end{align*}\nSimilarly,\n\\begin{align*}\nE^Q\\left(1_{\\tau>t_i} X(t_i) \\mid \\mathscr{F}_t \\right) &= E^Q\\left(X(t_i)E^Q(1_{\\tau>t_i} \\mid \\mathscr{F}_{t_i}\\big) \\mid \\mathscr{F}_t \\right)\\\\\n&=E^Q\\left(X(t_i) e^{-\\int_0^{t_i} \\lambda(s)ds} \\mid \\mathscr{F}_t \\right).\n\\end{align*}\n", "topic": "quant", "url": "https://quant.stackexchange.com/questions/68425"}, {"image": "VQAonline_00056843.png", "question": "Why is there a stong intraday-correlation between spot and vol?", "context": "\nFig.1 shows an intraday scatterplot of the DAX future against its volatility index VDAX on 6-Jan-2016. \nThe data suggest a strong negative correlation between the two. \nThere are various models available that \"describe\" this effect: For example stochastic vol-models such as the Heston model. However these models only describe but do not explain the effect. \nSince prices are the result of trading and market-making, a plausible explanation could be that market participants are buying put-option when the market goes down in order to protect their (long) position thereby driving the vol up; and re-selling them when the market goes up (driving the vol down). \nHowever it is easy to convince oneself that the correlation persists at time-scales that correspond to vol moves that make it difficult to trade out of the bid-offer spread of the option (at the ODAX-exchange). If it was only to reduce the downside risk (in a down move) it would be much cheaper to reduce ones long-position temporarily instead. Clearly options exhibit \"gamma\", but (as mentioned above) the cost of gamma seems too high given the bid-offer spread of the options.\nSo I do not really understand which market factors cause this intraday phenomenon of negative correlation between vol and spot moves.\nCan anyone suggest an answer?\n", "answer": "This effect is coming from the supply and demand in the options markets. Many portfolio managers want (or need) to buy out of the money put options, and many are willing to sell out of the money call options (thereby funding the purchase of put options). Now, when the market goes down, dealers find themselves short vol and they need to buy options to cover (hence vol goes up). Likewise, when the market goes up, dealers find themselves long vol and they need to sell (thus vol goes down). That's the effect you observe. The timeframe for this effect is almost continuous - many dealers recalculate their positions multiple times intraday.\nYes, there is a skew whereby out of the money puts are more expensive than ATMs and out of the money calls are cheaper. You can think of this as compensation for the expected hedging loss if you are using a simple Black Scholes model. However, many dealers would use a model that anticipates the vol change for the given market move. This model would price the puts and calls closer to the market skew. It would also reduce the losses from flattening the vol position after a market move. As for the question about bid-offers, it is an issue, but in practice a dealer won't just lift and hit in the market. They will rather work to buy and sell close to mid market, which they can probably do using their network of clients.\n", "topic": "quant", "url": "https://quant.stackexchange.com/questions/22614"}, {"image": "VQAonline_00056898.png", "question": "Relationship between hike in interest rate and drop in treasuries yields", "context": "How does a hike in interest rate results in drop in yields of treasuries?\nMy understanding was that when interest rate rises, investors would sell-off bonds, which would result in increase in yields. \n\n", "answer": "What happened was that the Fed indeed hiked by 25bp (as expected) but also conveyed the message that future hikes would be quite gradual (this was not expected). The market therefore altered its expected path of Fed funds downwards and hence the yield of Treasuties downwards. It's all about what happened versus previous expectations. \n", "topic": "quant", "url": "https://quant.stackexchange.com/questions/33052"}, {"image": "VQAonline_00057134.png", "question": "Do Arcane Caster Levels Count as Cleric Levels for Turning/Rebuking Checks?", "context": "If I am a Wizard, with no levels of Cleric, and I gain a domain power (through a prestige class1) that Turns X-Creatures as a Cleric Turns Undead; does my arcane caster level equal Cleric levels for that purpose?\nExample (Arachnomancer)\n1st Level: Poison Save Bonus, Handle Spiders, Spider Domain \n\nSpider Domain: Rebuke or command spiders as an evil cleric rebukes or commands undead. You can use this abilities a number of times per day equal to 3 + your Cha modifier.\n\nCleric level seems important due to the following table:\n\nIf I have no \"cleric level\" is the Rebuke or Command Spiders essentially doomed as ineffective?\n\n1Earthshaker (Dragon Magazine) gets Earth Domain, which grants Rebuking/Commanding Earth Creatures.\n", "answer": "\nThe prestige class arachnomancer (Drow of the Underdark 65-8) says\n\nYou gain access to the Spider domain and the granted power associated with the domain (rebuke spiders as an evil cleric rebukes undead, a number of times per day equal to 3 + your Cha modifier). (65)\n\nHence a level 1 arachnomancer rebukes spiders like a level 1 cleric rebukes undead\u2014no matter what other abilities the arachnomancer possesses\u2014, and that's terrible.\nThe prestige class earthshaker (Dragon #314 57-9) says that the creature\n\ngains the Earth domain's granted power, substituting his earthshaker levels for cleric levels. (58)\n\nAnd, yes, that's also terrible, but it's worse because while expected entry into the prestige class arachnomancer is at about character level 5, expected entry into the prestige class earthshaker is at about character level 9. Your granted power isn't just bringing almost nothing to the table, it's actually insulting.\n\nExtra domains in general\nAlthough earthshaker predates Complete Divine, that version of the arachnomancer was released long after Complete Divine.1 Both, however, seem to follow similarly the Complete Divine's vague rules on Extra Domains which, in part, says\n\nIf a noncleric enters a prestige class that allows access to a domain, the character still gains access to the domain. She can use the granted power bestowed by the domain normally. (20)\n\nComplete Divine, of course, doesn't actually say what normally means here, but, apparently, based on the anecdotal evidence from the above two classes, the game takes normally to mean here that the creature treats its levels in the prestige class as cleric levels, combining with nothing unless the text indicates otherwise. Which, let me say again, is terrible.\nNegotiate with the DM\nI suggest asking the DM to allow the arachnomancer to rebuke spiders like an evil cleric of twice the creature's arachnomancer level. While that's still terrible, it does sort incentivize staying in arachnomancer, because it (ahem) bugs some DMs when players have a bunch of a different classes on their character sheets. Creatures likely enter earthshaker too late for this quick-and-dirty fix to do any good, and thrice (maximum 20th) earthshaker level might be overreaching, but it couldn't hurt to ask.\n\n1 The original arachnomancer was, confusingly enough, introduced in Underdark, an early but still 3.5 product.\n", "topic": "rpg", "url": "https://rpg.stackexchange.com/questions/73968"}, {"image": "VQAonline_00057136.png", "question": "What is the correct term for the die with multiples of ten written on it?", "context": "What is the correct term for this die? It is a standard die in any 7 dice set.\n\nIt has multiples of ten from 10 to 00 written on it (10, 20, 30, 40, 50, 60, 70, 80, 90, (1)00).\nI seem to think that it is called a 'percentile die', is this correct?\n", "answer": "The two ten-sided dice used together to generate a number in the range 1-100 (or 0-99) are percentile dice (plural). The same term is applies to a pair of twenty-sided dice, each marked 0-9 twice, used for the same purpose.\nBack in the Before Time, dice sets didn't include a die marked with double-digits; you would just roll different-colored dice, having declared one of them the \"tens\". I think that sometimes the specific die you are asking about is called the tens die; Chessex calls it a \"Tens 10\" on their website.\n", "topic": "rpg", "url": "https://rpg.stackexchange.com/questions/75705"}, {"image": "VQAonline_00057200.png", "question": "Who is pictured on the cover of Dragonlance Adventures?", "context": "Who are the characters depicted on the cover of the Dragonlance Adventures hardcover?\nThe painting was done by Jeff Easley, and I'm pretty sure it's depicting Tanis taking the crown from Emperor Ariakas. But who is the undead guy in the back? I think it might be Lord Soth, but I don't think I've ever seen him depicted without his trademark bucket helmet.\n\n", "answer": "You correctly identified the characters depicted in the image: indeed they are\n\nLord Ariakas (the dying man): the commander of the Dragon Army\nTanis Half-Elven (the standing one with a sword and a crown): one of the protagonists of Dragonlance Saga\nLord Soth (the undead): a Death Knight, whose is an ally of Kitiara, one of the Dragon Highlords\n\nThis drawing refers to the moment where Tanis went to pledge allegiance to Ariakan, but instead of putting his sword at the Lord Commander's feet the half elf struck a fatal blow (with the aid of Raistlin Majere). The Crown of Power felt off from Ariakan's head, and since \"Whoever holds the Crown, rules!\" Tanis went to catch the crown. As soon as Kitiara had understood what was going on, she ordered Lord Soth to catch the crown for herself:\n\nThere was uneasy silence in the Hall of Audience as Ariakas\u2019s body tumbled to the floor. The Crown of Power fell from his head with a clatter and lay within a tangle of blood and thick, black hair.\nWho would claim it?\nThere was a piercing scream. Kitiara called out a name, called to someone.\nTanis could not understand. He didn\u2019t care anyway. He stretched out his hand for the Crown.\nSuddenly a figure in black armor materialized before him.\nLord Soth!\n\nThe above quotes are taken from Dragons of Spring Dawning, Book 3, Chapter 9, Horns of Doom.\n", "topic": "rpg", "url": "https://rpg.stackexchange.com/questions/140411"}, {"image": "VQAonline_00057214.png", "question": "Can characters escape from Death House through this method?", "context": "I will be DMing the Death House adventure very soon. Because I have players that tend to exploit every little loophole, I am wondering the following:\n\n Once the party refused to sacrifice someone and have to escape, would there be an easy escape route by climbing up through the trap door opening in the basement, then going through the wall near the exit and then getting out through the front gate?\n\nTo illustrate what I mean, here is a map with the escape route I'm thinking of:\n\n \n\n As written, once you re-enter the house, all doors will become scythes and all windows will become bricks and inner walls will become brittle and turn into rat swarms. But, there is nothing written about the front gate and whether the wall next to it, is considered an outer wall.\n\nIn someone's notes, I read that the only valid escape would be\n\n from the balcony.\n\nIs that really true?\nI mean, of course it would be up to me to decide and most likely it would be too easy to just shortcut in the first floor, but I think it still could be argued nonetheless and I want to be sure I didn't overlook anything.\n", "answer": "That's a valid route.\nBut it's still not easy. You've got to get from location 38 to location 32 in the basement (assuming you even know location 32 exists). Once you're there it's clear from the text that the trapdoor is intended to allow movement from the basement into the house proper.\nThen when you're up in the house you're in a poisonous-smoke filled room whose walls will burst with rats when you try to hack through them. It's a shortcut, but no cake-walk.\nWelcome to Barovia.\n", "topic": "rpg", "url": "https://rpg.stackexchange.com/questions/149128"}, {"image": "VQAonline_00057217.png", "question": "Is this attack type chart correct?", "context": "I am wondering if this chart showing the types of attacks in 5e-dnd and what things they can count towards is correct:\n\n(source1)\nThe chart works by listing on the y-axis different ways of making an attack and then along the x-axis it lists different phrases features throughout the books use. If something on the y-axis satisfies that condition on the x-axis their box is green, if not, the box is red. Several have descriptions explaining their content as I felt it might be necessary/helpful. The two Yellow boxes are yellow because the answer is conditional (not simply yes or no).\nI have chosen to avoid talking about spells in this chart mostly because they would take up quite a few columns/rows, and also because they do not seem to cause people nearly as much confusion as the other types of attacks. \nFor the purposes of this I would like three things to be assumed. This is so I can know if the main part of the chart is correct and not the sections that have been put up to debate previously on this Stack:\n\nThat using a weapon to make an improvised attack removes all of its properties including melee/ranged. Thus throwing a bow at somebody would not be considered an attack with a ranged weapon, because the bow has lost that property, similarly throwing a mace at somebody would not be considered an attack with a melee weapon. \nFrom the same question as above, that improvised attacks transform whatever is being used to make them into weapons for the purpose of that attack. Thus using a table-leg or shield in an improvised attack would momentarily make each of them count as a weapon.\nThat you cannot use a weapon for a shove or grapple so they do not qualify as attacks with weapons. (A question where the answer was iffy)\n\n1 - Note that the source may change after this posting as it is a Google Doc. For the sake of this question, use the image I have embedded here.\n\n", "answer": "Yes\nThis table appears to be correct, although the presence of rows 8 and 9 are not necessary since the data is covered by the previous rows.\nFor your third assumption, I would like to clarify that you wouldn't have to drop your weapon to make a shove attack. You could still use the weapon to push someone for example, but it would not qualify as a \"weapon attack\" at all - e.g. the Great Weapon Master feat would not apply. This is because grapple specifically states that you must have at least one free hand, whereas the shove action says nothing on the matter.\n\nUsing at least one free hand, you try to seize the target by making a grapple check instead of an attack roll [...]\n\n", "topic": "rpg", "url": "https://rpg.stackexchange.com/questions/149651"}, {"image": "VQAonline_00057212.png", "question": "Can a flying character can use Ant Haul to carry another character?", "context": "One of my player is a level 8 Oracle of Flame and she has the revelation Wings of Fire.\nShe is not strong enough to lift someone while flying. She would like to. She plays a Peri Aasimar with a strong angelic savior behavior.\n\nAs an Oracle, she could learn Ant Haul.\n\nThe target\u2019s carrying capacity triples. This does not affect the creature\u2019s actual Strength in any way, merely the amount of material it can carry while benefiting from this spell. It also has no effect on encumbrance due to armor. If the creature wears armor it still takes the normal penalties for doing so regardless of how much weight the spell allows it to carry.\n\nAs DM, I plan to allow it. Now, I would like to know if it not a big deal if I consider that the amount of material includes a living being.\n", "answer": "Yes, they can\nAnt Haul says that \"the target\u2019s carrying capacity triples\", meaning that her carrying capacity, assuming that if it's triple is enough to carry all their gear plus the carried creature and their gear too, shouldn't be affected by the fact that she is flying, other than reducing her speed and increasing her armor check penalty.\nIf three times her carrying capacity is still not enough to put her under Heavy Load, then she cannot freely move while carrying that weight, and should probably fall (GM Fiat) or drag the creature at 5 feet per round (see Lifting and Dragging). If she has 10 Str for instance, her maximum load is 100 lbs (~45Kg), or 300 lbs (~136Kg) under Ant Haul, which should be enough to carry someone else.\n", "topic": "rpg", "url": "https://rpg.stackexchange.com/questions/148760"}, {"image": "VQAonline_00057207.png", "question": "Is this d15 verifiably fair?", "context": "Following off of a question asking for how to make a d150 in which this d15 was mentioned in an answer, I was wondering if this d15 is fair (gives an equal distribution of results over its range). Is it?\n\n\nFifteen-sided d15 die. This design is based on spacing points as equally as possible on a sphere and then cutting planar slices perpendicular to those directions. [...] Note that this design is not isohedral.\n\n", "answer": "Based on the description, this die is most likely not perfectly fair\nThe important parts of the description are:\n\nThis design is based on spacing points as equally as possible on a sphere and then cutting planar slices perpendicular to those directions. [...] Note that this design is not isohedral.\n\nIsohedral means that all faces of the die are identical. The fact that this d15 is not isohedral means that the different faces have different shapes. Furthermore, the method of construction described in the first sentence will not result in a fair die. Note that the points are spaced \"as equally as possible\". It is not possible to space 15 points out on a spherical surface exactly equally; this is only possible for 4, 6, 8, 12, and 20, which is why those dice are the only completely symmetrical ones. Hence, when the 15 faces are cut, some of them will be more distant from their neighbors than others, which gives those faces a higher chance of being rolled. In theory, it might be possible to counteract this by varying the sizes of the faces, but this would be a difficult optimization problem since shrinking one face not only reduces the probability of rolling that face, but it also makes the neighboring faces more likely to be rolled and affects the overall center of gravity of the die, which in turn changes the probabilities of every face. Realistically, the only way to get this right would be through empirical testing and iteration. That is, building some dice, testing them, making adjustments, and repeating until the dice is empirically determined to be fair to within detectable limits.\nGiven the amount of work that would be required to design a fair die, there's no way they would do all the work to to make a fair die and then fail to mention that in the description. From that, we can conclude that the die is almost certainly not fair.\nIn fact, there are other dice where the process of making them as fair as possible is described in detail, such as the d30. The fact that no such description is provided for the d15 or any of the other non-isohedral dice is further evidence that they are no more fair than the non-uniform arrangement of faces would imply.\nHow fair do you need it to be?\nHowever, it's probably pretty close to fair. And if the designers are smart, they will have placed the numbers on the die in such a way as to minimize the unfairness by ensuring that the most and least likely faces are spread equally throughout the range of the die instead of clustered together in a small range. Given that, it's likely fair enough for many purposes. For example, if the arrangement of faces has any kind of symmetry to it, they could use that to make the distribution of rolls symmetric, such that the average roll is exactly 8 (i.e. the average of all faces), and that the probability of rolling over 8 is the same as the probability of rolling below 8. But if you need something truly and precisely uniform, this isn't it.\n", "topic": "rpg", "url": "https://rpg.stackexchange.com/questions/147221"}, {"image": "VQAonline_00057329.png", "question": "\"\u0410\u0432\u0442\u043e\u0440\u0441\u0442\u0432\u043e \u0441\u0442\u0430\u0442\u0435\u0439 \u043f\u0440\u0438\u043d\u0430\u0434\u043b\u0435\u0436\u0438\u0442 \u0432\u0435\u0434\u0443\u0449\u0438\u043c \u0441\u043f\u0435\u0446\u0438\u0430\u043b\u0438\u0441\u0442\u0430\u043c, \u043a\u0430\u0436\u0434\u043e\u0433\u043e \u0432 \u0441\u0432\u043e\u0451\u043c \u0432\u043e\u043f\u0440\u043e\u0441\u0435\" - \u043c\u043e\u0436\u043d\u043e?", "context": "\n\n\u0423\u043d\u0438\u043a\u0430\u043b\u044c\u043d\u043e\u0435 \u0438\u0437\u0434\u0430\u043d\u0438\u0435, \u0447\u044c\u044f \u0434\u0430\u043b\u044c\u043d\u0435\u0439\u0448\u0430\u044f \u0441\u0443\u0434\u044c\u0431\u0430 \u043f\u043e\u043a\u0430 \u043d\u0435\u0438\u0437\u0432\u0435\u0441\u0442\u043d\u0430. \u041f\u0435\u0440\u0432\u044b\u0439 \u0442\u043e\u043c\n \u0432\u044b\u0448\u0435\u043b \u0432 2000 \u0433\u043e\u0434\u0443. \u201c\u0421 \u0441\u0430\u043c\u043e\u0433\u043e \u043d\u0430\u0447\u0430\u043b\u0430 \u044d\u043d\u0446\u0438\u043a\u043b\u043e\u043f\u0435\u0434\u0438\u044f \u043f\u043b\u0430\u043d\u0438\u0440\u043e\u0432\u0430\u043b\u0430\u0441\u044c \u043a\u0430\u043a\n \u0438\u0437\u0434\u0430\u043d\u0438\u0435, \u0434\u0430\u043b\u0435\u043a\u043e \u0432\u044b\u0445\u043e\u0434\u044f\u0449\u0435\u0435 \u0437\u0430 \u0440\u0430\u043c\u043a\u0438 \u043f\u0440\u0430\u0432\u043e\u0441\u043b\u0430\u0432\u043d\u043e\u0433\u043e \u043c\u0438\u0440\u0430: \u0432 \u043d\u0451\u043c \u0434\u043e\u043b\u0436\u043d\u044b\n \u0431\u044b\u043b\u0438 \u0431\u044b\u0442\u044c \u043f\u0440\u0435\u0434\u0441\u0442\u0430\u0432\u043b\u0435\u043d\u044b \u043e\u0441\u043d\u043e\u0432\u043d\u044b\u0435 \u0441\u0432\u0435\u0434\u0435\u043d\u0438\u044f \u043f\u043e \u0432\u0441\u0435\u043c \u0445\u0440\u0438\u0441\u0442\u0438\u0430\u043d\u0441\u043a\u0438\u043c\n \u043a\u043e\u043d\u0444\u0435\u0441\u0441\u0438\u044f\u043c \u0438 \u0438\u043d\u044b\u043c \u0432\u0435\u0440\u043e\u0438\u0441\u043f\u043e\u0432\u0435\u0434\u0430\u043d\u0438\u044f\u043c, \u0437\u043d\u0430\u0447\u0438\u0442\u0435\u043b\u044c\u043d\u044b\u0435 \u043c\u0430\u0442\u0435\u0440\u0438\u0430\u043b\u044b \u0438\u0437 \u043e\u0431\u043b\u0430\u0441\u0442\u0438\n \u0444\u0438\u043b\u043e\u0441\u043e\u0444\u0438\u0438, \u043c\u043e\u0440\u0430\u043b\u0438, \u044d\u0442\u0438\u043a\u0438, \u0438\u0441\u043a\u0443\u0441\u0441\u0442\u0432\u0430, \u043c\u0443\u0437\u044b\u043a\u0438\u201d. \u0410\u0432\u0442\u043e\u0440\u0441\u0442\u0432\u043e \u0441\u0442\u0430\u0442\u0435\u0439\n \u043f\u0440\u0438\u043d\u0430\u0434\u043b\u0435\u0436\u0438\u0442 \u0432\u0435\u0434\u0443\u0449\u0438\u043c \u0441\u043f\u0435\u0446\u0438\u0430\u043b\u0438\u0441\u0442\u0430\u043c, \u043a\u0430\u0436\u0434\u043e\u0433\u043e \u0432 \u0441\u0432\u043e\u0451\u043c \u0432\u043e\u043f\u0440\u043e\u0441\u0435. \u0412 \u0440\u0430\u0431\u043e\u0442\u0435\n \u043f\u043e \u0441\u043e\u0437\u0434\u0430\u043d\u0438\u044e \u044d\u043d\u0446\u0438\u043a\u043b\u043e\u043f\u0435\u0434\u0438\u0438 \u0443\u0447\u0430\u0441\u0442\u0432\u0443\u044e\u0442: \u041c\u043e\u0441\u043a\u043e\u0432\u0441\u043a\u0430\u044f \u0434\u0443\u0445\u043e\u0432\u043d\u0430\u044f \u0430\u043a\u0430\u0434\u0435\u043c\u0438\u044f,\n \u0438\u043d\u0441\u0442\u0438\u0442\u0443\u0442\u044b \u0420\u043e\u0441\u0441\u0438\u0439\u0441\u043a\u043e\u0439 \u0430\u043a\u0430\u0434\u0435\u043c\u0438\u0438 \u043d\u0430\u0443\u043a, \u041c\u043e\u0441\u043a\u043e\u0432\u0441\u043a\u0438\u0439, \u0421\u0430\u043d\u043a\u0442-\u041f\u0435\u0442\u0435\u0440\u0431\u0443\u0440\u0433\u0441\u043a\u0438\u0439 \u0438\n \u0440\u044f\u0434 \u0434\u0440\u0443\u0433\u0438\u0445 \u043a\u0440\u0443\u043f\u043d\u0435\u0439\u0448\u0438\u0445 \u0440\u0435\u0433\u0438\u043e\u043d\u0430\u043b\u044c\u043d\u044b\u0445 \u0443\u043d\u0438\u0432\u0435\u0440\u0441\u0438\u0442\u0435\u0442\u043e\u0432, \u0441\u0438\u043d\u043e\u0434\u0430\u043b\u044c\u043d\u044b\u0435 \u043a\u043e\u043c\u0438\u0441\u0441\u0438\u0438\n \u0438 \u043e\u0442\u0434\u0435\u043b\u044b \u041c\u043e\u0441\u043a\u043e\u0432\u0441\u043a\u043e\u0439 \u041f\u0430\u0442\u0440\u0438\u0430\u0440\u0445\u0438\u0438, \u0420\u0410\u041d, \u0430 \u0442\u0430\u043a\u0436\u0435 \u043d\u0430\u0443\u0447\u043d\u044b\u0435 \u0446\u0435\u043d\u0442\u0440\u044b \u0421\u0428\u0410,\n \u0413\u0440\u0435\u0446\u0438\u0438 \u0438 \u0418\u0442\u0430\u043b\u0438\u0438. \u0412 \u043d\u0430\u0441\u0442\u043e\u044f\u0449\u0435\u0435 \u0432\u0440\u0435\u043c\u044f \u0432\u044b\u043f\u0443\u0449\u0435\u043d\u043e 40 \u0442\u043e\u043c\u043e\u0432 (\u0441 \u0443\u0447\u0451\u0442\u043e\u043c\n \u043d\u0435\u0430\u043b\u0444\u0430\u0432\u0438\u0442\u043d\u043e\u0433\u043e \u0442\u043e\u043c\u0430 \u2014 41-\u0433\u043e). \u0412 \u0447\u0430\u0441\u0442\u043d\u043e\u0441\u0442\u0438: \"\u041f\u0440\u0430\u0432\u043e\u0441\u043b\u0430\u0432\u043d\u0430\u044f \u044d\u043d\u0446\u0438\u043a\u043b\u043e\u043f\u0435\u0434\u0438\u044f. \u0422\u043e\u043c\n XXVIII\". \u0412\u0441\u0451 \u043e\u0431 \u0438\u0441\u043a\u0443\u0441\u0441\u0442\u0432\u0435 \u0418\u0442\u0430\u043b\u0438\u0438. \u041a\u043e\u0440\u043e\u0442\u043a\u043e \u0438 \u0441 \u043f\u043e\u0441\u043b\u0435\u0434\u043d\u0438\u043c\u0438 \u0434\u0430\u0442\u0438\u0440\u043e\u0432\u043a\u0430\u043c\u0438.\n\n\u041f\u043e\u0436\u0430\u043b\u0443\u0439\u0441\u0442\u0430, \u0443\u0432\u0438\u0434\u0438\u0442\u0435 \u043e\u0448\u0438\u0431\u043a\u0438 - \u0441\u0432\u0438\u0441\u0442\u043d\u0438\u0442\u0435!\n", "answer": "\u0410\u0432\u0442\u043e\u0440\u0441\u0442\u0432\u043e \u0441\u0442\u0430\u0442\u0435\u0439 \u043f\u0440\u0438\u043d\u0430\u0434\u043b\u0435\u0436\u0438\u0442 \u0432\u0435\u0434\u0443\u0449\u0438\u043c \u0441\u043f\u0435\u0446\u0438\u0430\u043b\u0438\u0441\u0442\u0430\u043c, (\u0441\u043f\u0435\u0446\u0438\u0430\u043b\u0438\u0437\u0430\u0446\u0438\u044f) \u043a\u0430\u0436\u0434\u043e\u0433\u043e \u0432 \u0441\u0432\u043e\u0435\u0439 \u043e\u0431\u043b\u0430\u0441\u0442\u0438. \n\u0418\u043b\u0438: \u0410\u0432\u0442\u043e\u0440\u0441\u0442\u0432\u043e \u0441\u0442\u0430\u0442\u0435\u0439 \u043f\u0440\u0438\u043d\u0430\u0434\u043b\u0435\u0436\u0438\u0442 \u0432\u0435\u0434\u0443\u0449\u0438\u043c \u0441\u043f\u0435\u0446\u0438\u0430\u043b\u0438\u0441\u0442\u0430\u043c \u2014 \u043a\u0430\u0436\u0434\u043e\u0433\u043e \u0432 \u0441\u0432\u043e\u0435\u0439 \u043e\u0431\u043b\u0430\u0441\u0442\u0438.\n\u0422\u0438\u0440\u0435 \u0447\u0451\u0442\u043a\u043e \u043e\u0431\u043e\u0437\u043d\u0430\u0447\u0430\u0435\u0442 \u043f\u0440\u0438\u0441\u043e\u0435\u0434\u0438\u043d\u0435\u043d\u0438\u0435, \u043d\u043e \u043d\u0435 \u0441\u043c\u043e\u0442\u0440\u0438\u0442\u0441\u044f \u0432 \u044d\u0442\u043e\u043c \u0442\u0435\u043a\u0441\u0442\u0435. \u0412 \u0442\u043e \u0436\u0435 \u0432\u0440\u0435\u043c\u044f \u0447\u0438\u0442\u0430\u0442\u044c \u043f\u0440\u0435\u0434\u043b\u043e\u0436\u0435\u043d\u0438\u0435 (\u043f\u0440\u0438 \u043f\u043e\u0441\u0442\u0430\u043d\u043e\u0432\u043a\u0435 \u0437\u0430\u043f\u044f\u0442\u043e\u0439 \u0438\u043b\u0438 \u0442\u0438\u0440\u0435) \u043d\u0430\u0434\u043e \u0441 \u043f\u043e\u0434\u0447\u0435\u0440\u043a\u043d\u0443\u0442\u043e\u0439 \u043f\u0430\u0443\u0437\u043e\u0439.\n\u041f\u043e\u044f\u0441\u043d\u0435\u043d\u0438\u0435\n1) \u041e\u0431\u043e\u0441\u043e\u0431\u043b\u0435\u043d\u043d\u044b\u0439 \u043e\u0431\u043e\u0440\u043e\u0442 \u043f\u043e\u0445\u043e\u0436 \u043d\u0430 \u043f\u0440\u0438\u0441\u043e\u0435\u0434\u0438\u043d\u0438\u0442\u0435\u043b\u044c\u043d\u0443\u044e \u043a\u043e\u043d\u0441\u0442\u0440\u0443\u043a\u0446\u0438\u044e, \u0442\u043e\u0433\u0434\u0430 \u0441\u043e\u0433\u043b\u0430\u0441\u043e\u0432\u0430\u043d\u0438\u0435 \u0432 \u043f\u0430\u0434\u0435\u0436\u0435 \u043d\u0435 \u0442\u0440\u0435\u0431\u0443\u0435\u0442\u0441\u044f. \u0410 \u043f\u0440\u0438 \u0441\u043e\u0433\u043b\u0430\u0441\u043e\u0432\u0430\u043d\u0438\u0438 \u0442\u0435\u0440\u044f\u0435\u0442\u0441\u044f \u0441\u043c\u044b\u0441\u043b, \u0442\u043e\u0433\u0434\u0430 \u043e\u0431\u043e\u0440\u043e\u0442 \u0434\u0435\u0439\u0441\u0442\u0432\u0438\u0442\u0435\u043b\u044c\u043d\u043e \u043b\u0443\u0447\u0448\u0435 \u0443\u0431\u0440\u0430\u0442\u044c. \u041e\u0441\u0442\u0430\u0432\u0438\u0442\u044c \u2014 \"\u0432\u0435\u0434\u0443\u0449\u0438\u043c \u0441\u043f\u0435\u0446\u0438\u0430\u043b\u0438\u0441\u0442\u0430\u043c \u0432 \u0441\u043e\u043e\u0442\u0432\u0435\u0442\u0441\u0442\u0432\u0443\u044e\u0449\u0438\u0445 \u0432\u043e\u043f\u0440\u043e\u0441\u0430\u0445/\u043e\u0431\u043b\u0430\u0441\u0442\u044f\u0445\".\n2) \u0421\u043e\u0447\u0435\u0442\u0430\u043d\u0438\u0435 \"\u0441\u043f\u0435\u0446\u0438\u0430\u043b\u0438\u0441\u0442 \u0432 \u0441\u0432\u043e\u0435\u043c \u0432\u043e\u043f\u0440\u043e\u0441\u0435\" \u043a\u0430\u043a-\u0442\u043e \u043c\u0430\u043b\u043e \u0443\u043f\u043e\u0442\u0440\u0435\u0431\u043b\u044f\u0435\u0442\u0441\u044f, \u043e\u0431\u044b\u0447\u043d\u043e \u2014 \u0432 \u0441\u0432\u043e\u0435\u0439 \u043e\u0431\u043b\u0430\u0441\u0442\u0438, \u0432 \u0441\u0432\u043e\u0435\u043c \u0434\u0435\u043b\u0435, \u0432 \u043e\u0431\u043b\u0430\u0441\u0442\u0438 \u0447\u0435\u0433\u043e-\u0442\u043e.\n", "topic": "rus", "url": "https://rus.stackexchange.com/questions/436519"}, {"image": "VQAonline_00057320.png", "question": "\u041a\u0430\u0436\u0434\u044b\u0439 \u043c\u0435\u0441\u044f\u0446 Facebook \u0438\u0441\u043f\u043e\u043b\u044c\u0437\u0443\u0435\u0442 (\u0438\u043b\u0438 \"\u0438\u0441\u043f\u043e\u043b\u044c\u0437\u0443\u044e\u0442\") 1,8 \u043c\u0438\u043b\u043b\u0438\u0430\u0440\u0434\u0430 \u0447\u0435\u043b\u043e\u0432\u0435\u043a", "context": "\u0412 \u043e\u0434\u043d\u043e\u043c \u0444\u0435\u0439\u0441\u0431\u0443\u0447\u043d\u043e\u043c \u0441\u043e\u043e\u0431\u0449\u0435\u0441\u0442\u0432\u0435 \u043b\u044e\u0434\u0435\u0439 \u043f\u043e\u0437\u0430\u0431\u0430\u0432\u0438\u043b\u043e \u0442\u0430\u043a\u043e\u0435 \u0441\u043e\u043e\u0431\u0449\u0435\u043d\u0438\u0435:\n\n\n\n\u041c\u043e\u0436\u043d\u043e \u043f\u0440\u0435\u0434\u0441\u0442\u0430\u0432\u0438\u0442\u044c, \u0447\u0442\u043e \u044d\u0442\u043e \u043f\u043b\u0430\u0442\u0444\u043e\u0440\u043c\u0430 Facebook \u0438\u0441\u043f\u043e\u043b\u044c\u0437\u0443\u0435\u0442 \u043b\u044e\u0434\u0435\u0439, \u0430 \u043d\u0435 \u043e\u043d\u0438 \u0438\u0441\u043f\u043e\u043b\u044c\u0437\u0443\u044e\u0442 \u043f\u043b\u0430\u0442\u0444\u043e\u0440\u043c\u0443 Facebook. \n\u042f \u0437\u0430\u0433\u043b\u044f\u043d\u0443\u043b \u043d\u0430 \u0441\u0430\u0439\u0442 gramota.ru:\n\n\u0421\u043e\u0433\u043b\u0430\u0441\u043e\u0432\u0430\u043d\u0438\u0435 \u0441\u043a\u0430\u0437\u0443\u0435\u043c\u043e\u0433\u043e \u0441\u043e \u0441\u043b\u043e\u0432\u0430\u043c\u0438 \u00ab\u0442\u044b\u0441\u044f\u0447\u0430, \u043c\u0438\u043b\u043b\u0438\u043e\u043d, \u043c\u0438\u043b\u043b\u0438\u0430\u0440\u0434\u00bb\n\u0421\u043a\u0430\u0437\u0443\u0435\u043c\u043e\u0435 \u043f\u0440\u0438 \u0441\u043b\u043e\u0432\u0430\u0445 \u0442\u044b\u0441\u044f\u0447\u0430, \u043c\u0438\u043b\u043b\u0438\u043e\u043d, \u043c\u0438\u043b\u043b\u0438\u0430\u0440\u0434 \u043e\u0431\u044b\u0447\u043d\u043e \u043f\u0440\u0438\u043d\u0438\u043c\u0430\u0435\u0442 \u0444\u043e\u0440\u043c\u0443 \u043f\u043e\u0434\u043b\u0435\u0436\u0430\u0449\u0435\u0433\u043e (\u0435\u0434\u0438\u043d\u0441\u0442\u0432\u0435\u043d\u043d\u043e\u0435 \u0447\u0438\u0441\u043b\u043e, \u0436\u0435\u043d\u0441\u043a\u0438\u0439 \u0438\u043b\u0438 \u043c\u0443\u0436\u0441\u043a\u043e\u0439 \u0440\u043e\u0434). \u0422\u044b\u0441\u044f\u0447\u0430 \u0447\u0435\u043b\u043e\u0432\u0435\u043a \u043a\u0430\u0436\u0434\u044b\u0439 \u0433\u043e\u0434 \u043e\u0442\u0434\u044b\u0445\u0430\u0435\u0442 \u0432 \u044d\u0442\u043e\u043c \u0441\u0430\u043d\u0430\u0442\u043e\u0440\u0438\u0438. \u041c\u0438\u043b\u043b\u0438\u043e\u043d \u0442\u044e\u043b\u044c\u043f\u0430\u043d\u043e\u0432 \u043f\u043e\u0441\u0430\u0436\u0435\u043d \u043d\u0430 \u043a\u043b\u0443\u043c\u0431\u0435.\n\n\u041d\u043e, \u0441 \u0434\u0440\u0443\u0433\u043e\u0439 \u0441\u0442\u043e\u0440\u043e\u043d\u044b:\n\n\u0421\u043e\u0433\u043b\u0430\u0441\u043e\u0432\u0430\u043d\u0438\u0435 \u043f\u043e \u0441\u043c\u044b\u0441\u043b\u0443 \u043f\u0440\u0435\u0434\u043f\u043e\u0447\u0442\u0438\u0442\u0435\u043b\u044c\u043d\u043e:\n3) \u0435\u0441\u043b\u0438 \u043d\u0443\u0436\u043d\u043e \u043f\u043e\u0434\u0447\u0435\u0440\u043a\u043d\u0443\u0442\u044c \u0440\u0430\u0437\u0434\u0435\u043b\u044c\u043d\u043e\u0441\u0442\u044c \u0434\u0435\u0439\u0441\u0442\u0432\u0438\u0439 \u043a\u0430\u0436\u0434\u043e\u0433\u043e \u0434\u0435\u0439\u0441\u0442\u0432\u0443\u044e\u0449\u0435\u0433\u043e \u043b\u0438\u0446\u0430, \u043d\u0430\u0437\u044b\u0432\u0430\u0435\u043c\u043e\u0433\u043e \u043f\u043e\u0434\u043b\u0435\u0436\u0430\u0449\u0438\u043c, \u0430 \u0442\u0430\u043a\u0436\u0435 \u043f\u043e\u0434\u0447\u0435\u0440\u043a\u043d\u0443\u0442\u044c \u0430\u043a\u0442\u0438\u0432\u043d\u043e\u0441\u0442\u044c \u0434\u0435\u0439\u0441\u0442\u0432\u0443\u044e\u0449\u0438\u0445 \u043b\u0438\u0446: \u0420\u044f\u0434 \u0441\u043e\u0442\u0440\u0443\u0434\u043d\u0438\u043a\u043e\u0432 \u043d\u0430\u0448\u0435\u0439 \u043e\u0440\u0433\u0430\u043d\u0438\u0437\u0430\u0446\u0438\u0438 \u0432\u044b\u0441\u0442\u0443\u043f\u0438\u043b\u0438 \u0441 \u0438\u043d\u0438\u0446\u0438\u0430\u0442\u0438\u0432\u043e\u0439; \u0441\u0440.: \u0412 \u043f\u0440\u043e\u0448\u043b\u043e\u043c \u0433\u043e\u0434\u0443 \u0431\u044b\u043b\u043e \u043f\u043e\u0441\u0442\u0440\u043e\u0435\u043d\u043e \u043c\u043d\u043e\u0436\u0435\u0441\u0442\u0432\u043e \u0434\u043e\u0440\u043e\u0433.\n\n\u041d\u0430\u0441\u043a\u043e\u043b\u044c\u043a\u043e \u044f \u043f\u043e\u043d\u044f\u043b, \"\u0438\u0441\u043f\u043e\u043b\u044c\u0437\u0443\u0435\u0442\" \u043d\u0435 \u044f\u0432\u043b\u044f\u0435\u0442\u0441\u044f \u043e\u0448\u0438\u0431\u043a\u043e\u0439, \u043d\u043e \u0432\u0441\u0451-\u0442\u0430\u043a\u0438 \u043f\u0440\u0435\u0434\u043f\u043e\u0447\u0442\u0438\u0442\u0435\u043b\u044c\u043d\u0435\u0435 \u043f\u0438\u0441\u0430\u0442\u044c \"\u0438\u0441\u043f\u043e\u043b\u044c\u0437\u0443\u044e\u0442\"? \n", "answer": "\u041a\u0430\u043a \u0443\u0436\u0435 \u0431\u044b\u043b\u043e \u0441\u043a\u0430\u0437\u0430\u043d\u043e, \u043d\u0430\u043f\u0438\u0441\u0430\u043d\u043d\u043e\u0435 \u043d\u0435 \u044f\u0432\u043b\u044f\u0435\u0442\u0441\u044f \u043e\u0448\u0438\u0431\u043a\u043e\u0439. \u0411\u043e\u044e\u0441\u044c, \u043d\u0435\u0442 \u043e\u0431\u044a\u0435\u043a\u0442\u0438\u0432\u043d\u043e\u0433\u043e \u043e\u0442\u0432\u0435\u0442\u0430 \u043d\u0430 \u0432\u043e\u043f\u0440\u043e\u0441, \u0447\u0442\u043e \u043f\u0440\u0435\u0434\u043f\u043e\u0447\u0442\u0438\u0442\u0435\u043b\u044c\u043d\u043e \u0432 \u0434\u0430\u043d\u043d\u043e\u043c \u0441\u043b\u0443\u0447\u0430\u0435. \n\u0410\u0440\u0433\u0443\u043c\u0435\u043d\u0442\u044b \u0432 \u043f\u043e\u043b\u044c\u0437\u0443 \u0435\u0434\u0438\u043d\u0441\u0442\u0432\u0435\u043d\u043d\u043e\u0433\u043e \u0447\u0438\u0441\u043b\u0430: \u0436\u0435\u043b\u0430\u043d\u0438\u0435 \u043e\u0431\u0440\u0430\u0442\u0438\u0442\u044c \u0432\u043d\u0438\u043c\u0430\u043d\u0438\u0435 \u043d\u0430 \u043a\u043e\u043b\u0438\u0447\u0435\u0441\u0442\u0432\u043e, \u0430 \u043d\u0435 \u043d\u0430 \u0430\u043a\u0442\u0438\u0432\u043d\u043e\u0441\u0442\u044c \u0434\u0435\u0439\u0441\u0442\u0432\u0443\u044e\u0449\u0438\u0445 \u043b\u0438\u0446, \u043d\u0430\u043b\u0438\u0447\u0438\u0435 \u0441\u043b\u043e\u0432\u0430 \"\u0431\u043e\u043b\u0435\u0435\" (\u043e\u0431\u043e\u0437\u043d\u0430\u0447\u0435\u043d\u0438\u0435 \u043f\u0440\u0438\u0431\u043b\u0438\u0437\u0438\u0442\u0435\u043b\u044c\u043d\u043e\u0433\u043e \u043a\u043e\u043b\u0438\u0447\u0435\u0441\u0442\u0432\u0430). \u0412 \u043f\u043e\u043b\u044c\u0437\u0443 \u043c\u043d\u043e\u0436\u0435\u0441\u0442\u0432\u0435\u043d\u043d\u043e\u0433\u043e \u0447\u0438\u0441\u043b\u0430 - \u0442\u043e\u043b\u044c\u043a\u043e \u0443\u0441\u0442\u0440\u0430\u043d\u0435\u043d\u0438\u0435 \u0434\u0432\u0443\u0441\u043c\u044b\u0441\u043b\u0435\u043d\u043d\u043e\u0441\u0442\u0438 (\u043a\u0442\u043e \u043a\u043e\u0433\u043e \u0438\u0441\u043f\u043e\u043b\u044c\u0437\u0443\u0435\u0442), \u043d\u043e, \u043d\u0430 \u043c\u043e\u0439 \u0432\u0437\u0433\u043b\u044f\u0434, \u044d\u0442\u043e\u0442 \u0430\u0440\u0433\u0443\u043c\u0435\u043d\u0442 \u0432\u0430\u0436\u043d\u0435\u0435 \u043e\u0441\u0442\u0430\u043b\u044c\u043d\u044b\u0445.\n\u0410 \u0435\u0449\u0435 \u0437\u0430\u043c\u0435\u0447\u0443, \u0447\u0442\u043e \u0434\u0432\u0443\u0441\u043c\u044b\u0441\u043b\u0435\u043d\u043d\u043e\u0441\u0442\u044c \u043c\u043e\u0436\u043d\u043e \u0443\u0441\u0442\u0440\u0430\u043d\u0438\u0442\u044c, \u0443\u043f\u043e\u0442\u0440\u0435\u0431\u0438\u0432 \u043f\u0440\u044f\u043c\u043e\u0439 \u043f\u043e\u0440\u044f\u0434\u043e\u043a \u0441\u043b\u043e\u0432: \"\u041a\u0430\u0436\u0434\u044b\u0439 \u043c\u0435\u0441\u044f\u0446 \u0431\u043e\u043b\u0435\u0435 1.8 \u043c\u0438\u043b\u043b\u0438\u0430\u0440\u0434\u0430 \u0447\u0435\u043b\u043e\u0432\u0435\u043a \u0438\u0441\u043f\u043e\u043b\u044c\u0437\u0443\u0435\u0442 Facebook\". \u0418 \u0432\u043e\u043b\u043a\u0438 \u0441\u044b\u0442\u044b, \u0438 \u043e\u0432\u0446\u044b \u0446\u0435\u043b\u044b.\n\n\u0414\u043e\u043f\u043e\u043b\u043d\u0435\u043d\u0438\u0435\n\u0410 \u0432\u043e\u043e\u0431\u0449\u0435-\u0442\u043e \u0424\u0435\u0439\u0441\u0431\u0443\u043a \u043d\u0435 \u0438\u0441\u043f\u043e\u043b\u044c\u0437\u0443\u044e\u0442, \u0438\u043c \u043f\u043e\u043b\u044c\u0437\u0443\u044e\u0442\u0441\u044f. \u0422\u0430\u043a \u0447\u0442\u043e \u0440\u0435\u043a\u043b\u0430\u043c\u0443 \u044d\u0442\u0443 \u043d\u0443\u0436\u043d\u043e \u043c\u0435\u043d\u044f\u0442\u044c.\n", "topic": "rus", "url": "https://rus.stackexchange.com/questions/431813"}, {"image": "VQAonline_00057339.png", "question": "\u041a\u0430\u043a \u0441\u043a\u043b\u043e\u043d\u044f\u0435\u0442\u0441\u044f \u0438 \u043a\u0430\u043a\u043e\u0433\u043e \u0440\u043e\u0434\u0430 \u0441\u043b\u043e\u0432\u043e \"\u043d\u043e\u0434(\u0430)\"?", "context": "\u041d\u043e\u0434\u0430 - \u044d\u0442\u043e \u0437\u0430\u0438\u043c\u0441\u0442\u0432\u043e\u0432\u0430\u043d\u0438\u0435 \u043e\u0442 \u0441\u043b\u043e\u0432\u0430 node (\u0443\u0437\u0435\u043b). \u0418\u0437 \u0432\u0438\u043a\u0438\u0441\u043b\u043e\u0432\u0430\u0440\u044f, \u0443\u0437\u043d\u0430\u043b, \u0447\u0442\u043e \u044d\u0442\u043e \u0441\u043b\u043e\u0432\u043e \u0436\u0435\u043d\u0441\u043a\u043e\u0433\u043e \u0440\u043e\u0434\u0430 (\u0432\u0440\u043e\u0434\u0435 \u043a\u0430\u043a). \u0418\u0441\u043f\u043e\u043b\u044c\u0437\u0443\u0435\u0442\u0441\u044f \u0432 \u043a\u043e\u043c\u043f\u044c\u044e\u0442\u0435\u0440\u043d\u044b\u0445 \u043f\u0440\u0438\u043b\u043e\u0436\u0435\u043d\u0438\u044f\u0445 \u0434\u043b\u044f \u043e\u0431\u043e\u0437\u043d\u0430\u0447\u0435\u043d\u0438\u044f \u0431\u043b\u043e\u043a\u0430 \u043f\u0440\u043e\u0433\u0440\u0430\u043c\u043c\u044b, \u043a\u043e\u0442\u043e\u0440\u044b\u0439 \u0441\u043e\u0435\u0434\u0438\u043d\u044f\u0435\u0442\u0441\u044f \u0441 \u0434\u0440\u0443\u0433\u0438\u043c\u0438 \u043d\u043e\u0434\u0430\u043c\u0438 \u0438 \u043e\u0431\u0440\u0430\u0437\u0443\u0435\u0442 \u0441\u043b\u043e\u0436\u043d\u0443\u044e \u0441\u0442\u0440\u0443\u043a\u0442\u0443\u0440\u0443:\n\n\u0422\u0430\u043a\u0436\u0435 \u0438\u0441\u043f\u043e\u043b\u044c\u0437\u0443\u0435\u0442\u0441\u044f \u0434\u043b\u044f \u043e\u0431\u043e\u0437\u043d\u0430\u0447\u0435\u043d\u0438\u044f \u043a\u043e\u043c\u043f\u044c\u044e\u0442\u0435\u0440\u0430 \u0432 \u0441\u043f\u0435\u0446\u0438\u0444\u0438\u0447\u0435\u0441\u043a\u0438\u0445 \u0441\u0435\u0442\u044f\u0445\n\u041a\u043e\u0435-\u0447\u0442\u043e \u043e\u0447\u0435\u0432\u0438\u0434\u043d\u043e \u0438 \u0438\u0441\u043f\u043e\u043b\u044c\u0437\u0443\u0435\u0442\u0441\u044f \u043f\u043e\u0432\u0441\u0435\u043c\u0435\u0441\u0442\u043d\u043e:\n\n\u041d\u043e\u0434\u043e\u0432\u044b\u0439 \u0440\u0435\u0434\u0430\u043a\u0442\u043e\u0440\n\u041d\u043e\u0434\u044b\n\u0413\u0440\u0443\u043f\u043f\u0430 \u043d\u043e\u0434\u043e\u0432\n\n\u041a\u043e\u0435-\u0447\u0442\u043e \u043d\u0435\u043e\u0447\u0435\u0432\u0438\u0434\u043d\u043e, \u0438 \u043f\u0440\u0438\u043c\u0435\u0440\u043e\u0432 \u0438\u0441\u043f\u043e\u043b\u044c\u0437\u043e\u0432\u0430\u043d\u0438\u044f \u043d\u0435 \u043d\u0430\u0439\u0442\u0438. \u041d\u0430\u043f\u0440\u0438\u043c\u0435\u0440, \u043a\u0430\u043a\u0438\u043c \u043e\u0431\u0440\u0430\u0437\u043e\u043c \u0441\u043a\u043b\u043e\u043d\u044f\u0442\u044c \u044d\u0442\u043e \u0441\u043b\u043e\u0432\u043e, \u043e\u0441\u043e\u0431\u0435\u043d\u043d\u043e \u0432\u043e \u0444\u0440\u0430\u0437\u0430\u0445:\n\n\u0418\u0441\u043f\u043e\u043b\u044c\u0437\u043e\u0432\u0430\u0442\u044c \u0430\u043a\u0442\u0438\u0432\u043d\u0443\u044e \u043d\u043e\u0434(\u0443)\n\u0412\u044b\u0434\u0435\u043b\u0435\u043d\u043d\u044b\u0445 \u043d\u043e\u0434(\u043e\u0432)\n\u0426\u0432\u0435\u0442 \u043d\u043e\u0434\u044b(?)\n\u0414\u043e\u0431\u0430\u0432\u0438\u0442\u044c \u043a \u043d\u043e\u0434\u0435(\u0443)\n\n\u0422\u043e\u0447\u043d\u043e \u043b\u0438 \u044d\u0442\u043e \u0441\u043b\u043e\u0432\u043e \u0436\u0435\u043d\u0441\u043a\u043e\u0433\u043e \u0440\u043e\u0434\u0430?\n\u0415\u0441\u0442\u044c \u0432\u0430\u0440\u0438\u0430\u043d\u0442\u044b \u0438\u0441\u043f\u043e\u043b\u044c\u0437\u043e\u0432\u0430\u043d\u0438\u044f \u043a\u0430\u043a \u043c\u0443\u0436\u0441\u043a\u043e\u0433\u043e \u0440\u043e\u0434\u0430:\n\n\u0418\u0441\u043f\u0440\u0430\u0432\u043b\u044f\u0435\u043c \u044d\u0442\u043e \u043d\u0435\u0434\u043e\u0440\u0430\u0437\u0443\u043c\u0435\u043d\u0438\u0435 \u0438\u0441\u043f\u043e\u043b\u044c\u0437\u0443\u044f \u043d\u043e\u0434 Hue/Saturation/Value.\n\nhttps://pikabu.ru/story/vozmozhnosti_sovremennoy_kompyuternoy_grafiki_4291152\n\u0442\u0430\u043a \u0438 \u0436\u0435\u043d\u0441\u043a\u043e\u0433\u043e:\n\n\u041d\u043e\u0434\u0430 Copy. \u041d\u0430\u0441\u043b\u0435\u0434\u043e\u0432\u0430\u043d\u0438\u0435 \u0442\u0440\u0430\u043d\u0441\u0444\u043e\u0440\u043c\u0430\u0446\u0438\u0439 \u0438 \u0430\u0442\u0440\u0438\u0431\u0443\u0442\u043e\u0432. \u0422\u0435\u0445\u043d\u0438\u043a\u0430 Copy\nStamping.\n\nhttp://vfxlaboratory.ru/kurs/houdini-artist-101\n", "answer": "\u0412 \u0441\u0444\u0435\u0440\u0435 IT \u0440\u0430\u0431\u043e\u0442\u0430\u044e \u0443\u0436\u0435 \u043f\u043e\u0447\u0442\u0438 30 \u043b\u0435\u0442. \u041f\u043e \u0440\u0430\u0431\u043e\u0447\u0438\u043c \u0432\u043e\u043f\u0440\u043e\u0441\u0430\u043c \u043e\u0431\u0449\u0430\u044e\u0441\u044c \u0441 \u043e\u0433\u0440\u043e\u043c\u043d\u044b\u043c \u043a\u043e\u043b\u0438\u0447\u0435\u0441\u0442\u0432\u043e\u043c \u043a\u043e\u043b\u043b\u0435\u0433 \u0438 \u0432\u0441\u0435\u0433\u0434\u0430 \u0441\u043b\u043e\u0432\u043e \"\u043d\u043e\u0434\u0430\" \u0431\u044b\u043b\u043e \u0436\u0435\u043d\u0441\u043a\u043e\u0433\u043e \u0440\u043e\u0434\u0430. \u0418\u043d\u043e\u0433\u0434\u0430 \u0434\u043e\u0432\u043e\u0434\u0438\u043b\u043e\u0441\u044c \u0441\u043b\u044b\u0448\u0430\u0442\u044c \u043f\u0440\u0438\u043c\u0435\u043d\u0435\u043d\u0438\u0435 \u044d\u0442\u043e\u0433\u043e \u0441\u043b\u043e\u0432\u0430 \u0432 \u043c\u0443\u0436\u0441\u043a\u043e\u043c \u0440\u043e\u0434\u0435 \u043e\u0442 \u043d\u0430\u0447\u0438\u043d\u0430\u044e\u0449\u0438\u0445 IT-\u0448\u043d\u0438\u043a\u043e\u0432, \u043d\u043e \u043e\u0431\u044b\u0447\u043d\u043e \u0441\u0442\u0430\u0440\u0448\u0438\u0435 \u0442\u043e\u0432\u0430\u0440\u0438\u0449\u0438 \u0440\u0430\u043d\u043e \u0438\u043b\u0438 \u043f\u043e\u0437\u0434\u043d\u043e \u043e\u0431\u044a\u044f\u0441\u043d\u044f\u043b\u0438 \u0438\u043c, \u043a\u0430\u043a \u0434\u043e\u043b\u0436\u043d\u043e \u0431\u044b\u0442\u044c \u043f\u0440\u0430\u0432\u0438\u043b\u044c\u043d\u043e, \u0438 \u043e\u043d\u0438 \u0438\u0441\u043f\u0440\u0430\u0432\u043b\u044f\u043b\u0438\u0441\u044c ;)\n\u0418 \u043a\u0441\u0442\u0430\u0442\u0438, \u044d\u0442\u043e \u043d\u0435 \u0435\u0434\u0438\u043d\u0441\u0442\u0432\u0435\u043d\u043d\u044b\u0439 \u043f\u0430\u0440\u0430\u0434\u043e\u043a\u0441, \u0434\u043e\u0441\u0442\u0430\u0432\u0448\u0438\u0439\u0441\u044f \u043d\u0430\u043c \u0441\u043e \u0432\u0440\u0435\u043c\u0435\u043d \u0424\u0418\u0414\u041e. \u0414\u0432\u0430 \u043d\u0430\u0438\u0431\u043e\u043b\u0435\u0435 \u0437\u043d\u0430\u0447\u0438\u043c\u044b\u0445 \u0442\u0435\u0440\u043c\u0438\u043d\u0430 \u0442\u0430\u043c \u0438\u043c\u0435\u044e\u0442 \u0441\u043e\u0432\u0435\u0440\u0448\u0435\u043d\u043d\u043e \u043d\u0435\u043b\u043e\u0433\u0438\u0447\u043d\u044b\u0439 \u0440\u043e\u0434. \u041a\u0440\u043e\u043c\u0435 \u043d\u043e\u0434\u044b, \u043a\u043e\u0442\u043e\u0440\u0430\u044f \u0432 \u043e\u0440\u0438\u0433\u0438\u043d\u0430\u043b\u0435 \"node\" (\u0443\u0437\u0435\u043b), \u0438 \u043f\u043e\u044d\u0442\u043e\u043c\u0443, \u043f\u043e \u0438\u0434\u0435\u0435, \u0434\u043e\u043b\u0436\u043d\u0430 \u0431\u044b\u043b\u0430 \u0431\u044b \u0431\u044b\u0442\u044c \u043c\u0443\u0436\u0441\u043a\u043e\u0433\u043e \u0440\u043e\u0434\u0430, \u0435\u0441\u0442\u044c \u0435\u0449\u0435 \u043f\u043e\u0438\u043d\u0442, \u043a\u043e\u0442\u043e\u0440\u044b\u0439 \u0438\u043c\u0435\u0435\u0442 \u043c\u0443\u0436\u0441\u043a\u043e\u0439 \u0440\u043e\u0434, \u0445\u043e\u0442\u044f \u0432 \u043e\u0440\u0438\u0433\u0438\u043d\u0430\u043b\u0435 \u043e\u043d point (\"\u0442\u043e\u0447\u043a\u0430\"), \u0438 \u0434\u043e\u043b\u0436\u0435\u043d \u0431\u044b\u043b \u0431\u044b \u0438\u043c\u0435\u0442\u044c \u0436\u0435\u043d\u0441\u043a\u0438\u0439 \u0440\u043e\u0434.\n\u0427\u0442\u043e \u0436\u0435 \u043a\u0430\u0441\u0430\u0435\u0442\u0441\u044f \u043f\u0440\u0438\u043c\u0435\u0440\u043e\u0432 \u043d\u0435\u043f\u0440\u0430\u0432\u0438\u043b\u044c\u043d\u043e\u0433\u043e \u043f\u0440\u0438\u043c\u0435\u043d\u0435\u043d\u0438\u044f \u0440\u043e\u0434\u0430 \u044d\u0442\u0438\u0445 \u0441\u043b\u043e\u0432 \u043d\u0430 \u043d\u0435\u043a\u043e\u0442\u043e\u0440\u044b\u0445 \u0440\u0435\u0441\u0443\u0440\u0441\u0430\u0445 - \u0442\u043e \u0442\u0443\u0442, \u043f\u043e-\u0432\u0438\u0434\u0438\u043c\u043e\u043c\u0443, \u0432\u0438\u043d\u043e\u0432\u0430\u0442\u043e \u0441\u043b\u0438\u0448\u043a\u043e\u043c \u0431\u044b\u0441\u0442\u0440\u043e\u0435 \u0440\u0430\u0441\u0448\u0438\u0440\u0435\u043d\u0438\u0435 \u043a\u0440\u0443\u0433\u0430 \u044d\u043d\u0442\u0443\u0437\u0438\u0430\u0441\u0442\u043e\u0432, \u043a\u043e\u0442\u043e\u0440\u044b\u0435 \u043d\u0435 \u0441\u0440\u0430\u0437\u0443 \u0443\u0441\u043f\u0435\u0432\u0430\u044e\u0442 \u043e\u0437\u043d\u0430\u043a\u043e\u043c\u0438\u0442\u044c\u0441\u044f \u0441 \u0442\u0440\u0430\u0434\u0438\u0446\u0438\u043e\u043d\u043d\u043e\u0439 \u0442\u0435\u0440\u043c\u0438\u043d\u043e\u043b\u043e\u0433\u0438\u0435\u0439. \u041d\u043e \u043e\u043d\u0438 \u0442\u043e\u0436\u0435 \u0438\u0441\u043f\u0440\u0430\u0432\u044f\u0442\u0441\u044f, \u043f\u043e\u0432\u0435\u0440\u044c\u0442\u0435 :)\n", "topic": "rus", "url": "https://rus.stackexchange.com/questions/447681"}, {"image": "VQAonline_00057351.png", "question": "\u0421\u043b\u043e\u0432\u0430\u0440\u0438 \u043d\u0435 \u0445\u043e\u0442\u044f\u0442 \u0432\u043e\u0434\u043d\u044b\u0445 \u0447\u0430\u0441\u043e\u0432, \u0442\u043e\u043b\u044c\u043a\u043e \u0432\u043e\u0434\u044f\u043d\u044b\u0435, - \u043a\u0430\u043a \u0431\u044b\u0442\u044c?", "context": "\n\u0423 \u043c\u0435\u043d\u044f:\n\n\u0412 \u0441\u0430\u0434\u0430\u0445 \u043d\u0430 \u0432\u0438\u043b\u043b\u0435 \u0411\u043e\u0440\u0433\u0435\u0437\u0435 \u0435\u0441\u0442\u044c \u0432\u043e\u0434\u043d\u044b\u0435 \u0447\u0430\u0441\u044b, \u0447\u0443\u0442\u044c \u043b\u0438 \u043d\u0435 \u0431\u0430\u0440\u043e\u0447\u043d\u044b\u0435.\n \u041d\u0430\u0434\u043e \u043f\u043e\u0432\u043d\u0438\u043c\u0430\u0442\u0435\u043b\u044c\u043d\u0435\u0435 \u043f\u043e\u0447\u0438\u0442\u0430\u0442\u044c \u043f\u0440\u043e \u0438\u0445 \u0443\u0441\u0442\u0440\u043e\u0439\u0441\u0442\u0432\u043e.\n\n", "answer": "\u041e\u0431\u0449\u0435\u043f\u0440\u0438\u043d\u044f\u0442\u043e\u0435 \u043d\u0430\u0437\u0432\u0430\u043d\u0438\u0435 \u2014 \u0432\u043e\u0434\u044f\u043d\u044b\u0435 \u0447\u0430\u0441\u044b, \u0447\u0442\u043e \u0441\u043e\u043e\u0442\u0432\u0435\u0442\u0441\u0442\u0432\u0443\u0435\u0442 \u043e\u0434\u043d\u043e\u043c\u0443 \u0438\u0437 \u0437\u043d\u0430\u0447\u0435\u043d\u0438\u0439 \u0441\u0443\u0444\u0444\u0438\u043a\u0441\u0430 \u042f\u041d \u0432 \u0441\u043b\u043e\u0432\u0430\u0440\u0435 \u0415\u0444\u0440\u0435\u043c\u043e\u0432\u043e\u0439: \u0440\u0430\u0431\u043e\u0442\u0430\u044e\u0449\u0438\u0439 \u043d\u0430 \u0442\u043e\u043c, \u0447\u0442\u043e \u043d\u0430\u0437\u0432\u0430\u043d\u043e \u043c\u043e\u0442\u0438\u0432\u0438\u0440\u0443\u044e\u0449\u0438\u043c \u0441\u043b\u043e\u0432\u043e\u043c (\u0432\u0435\u0442\u0440\u044f\u043d\u043e\u0439, \u0432\u043e\u0434\u044f\u043d\u043e\u0439, \u0442\u043e\u0440\u0444\u044f\u043d\u043e\u0439).\n\u0418\u043d\u043e\u0433\u0434\u0430 \u0432\u0441\u0442\u0440\u0435\u0447\u0430\u0435\u0442\u0441\u044f \u043d\u0430\u0437\u0432\u0430\u043d\u0438\u0435 \u0432\u043e\u0434\u043d\u044b\u0435 \u0447\u0430\u0441\u044b, \u043d\u0430\u043f\u0440\u0438\u043c\u0435\u0440: \u041c\u0430\u043b\u0435\u043d\u044c\u043a\u0438\u0439 \u0433\u0435\u043d\u0438\u0439 / \u041d\u0430\u0443\u0447\u043d\u044b\u0439 \u043d\u0430\u0431\u043e\u0440. \u0412\u043e\u0434\u043d\u044b\u0435 \u0447\u0430\u0441\u044b. https://www.wildberries.ru/catalog/3108786/detail.aspx\n\u041d\u043e \u0432 \u044d\u0442\u043e\u043c \u0441\u043b\u0443\u0447\u0430\u0435 \u0432\u043e\u0434\u0430 \u043d\u0435 \u044f\u0432\u043b\u044f\u0435\u0442\u0441\u044f \u043d\u0435\u043f\u043e\u0441\u0440\u0435\u0434\u0441\u0442\u0432\u0435\u043d\u043d\u043e \u0440\u0430\u0431\u043e\u0447\u0435\u0439 \u0436\u0438\u0434\u043a\u043e\u0441\u0442\u044c\u044e: \u043d\u0430 \u0432\u043e\u0434\u0435 \u0440\u0430\u0431\u043e\u0442\u0430\u044e\u0442 \u0431\u0430\u0442\u0430\u0440\u0435\u0439\u043a\u0438, \u0430 \u0447\u0430\u0441\u044b \u043f\u043e\u043b\u0443\u0447\u0430\u044e\u0442\u0441\u044f \u044d\u043b\u0435\u043a\u0442\u0440\u0438\u0447\u0435\u0441\u043a\u0438\u0435. \u0417\u0434\u0435\u0441\u044c \u0438\u0441\u043f\u043e\u043b\u044c\u0437\u0443\u0435\u0442\u0441\u044f \u0441\u0443\u0444\u0444\u0438\u043a\u0441 \u043e\u0442\u043d\u043e\u0441\u0438\u0442\u0435\u043b\u044c\u043d\u043e\u0433\u043e \u043f\u0440\u0438\u043b\u0430\u0433\u0430\u0442\u0435\u043b\u044c\u043d\u043e\u0433\u043e \u041d \u0441 \u043e\u0431\u0449\u0438\u043c \u0437\u043d\u0430\u0447\u0435\u043d\u0438\u0435\u043c.\n\u041f\u043e\u044d\u0442\u043e\u043c\u0443 \u0432\u043e\u0434\u043d\u044b\u0435 \u0447\u0430\u0441\u044b \u2014 \u044d\u0442\u043e \u0441\u043a\u043e\u0440\u0435\u0435 \u0430\u0432\u0442\u043e\u0440\u0441\u043a\u043e\u0435 \u043d\u0430\u0437\u0432\u0430\u043d\u0438\u0435 (\u0432\u043e\u0437\u043c\u043e\u0436\u043d\u043e, \u0434\u043e\u043f\u0443\u0441\u043a\u0430\u0435\u0442\u0441\u044f \u043a\u0430\u043a \u0443\u043f\u0440\u043e\u0449\u0435\u043d\u043d\u044b\u0439 \u0432\u0430\u0440\u0438\u0430\u043d\u0442).\n", "topic": "rus", "url": "https://rus.stackexchange.com/questions/450608"}, {"image": "VQAonline_00057338.png", "question": "\u041a\u043b\u044e\u0447\u0438 \u043e\u0442 \u043e\u0431\u0435\u0438\u0445 \u0432\u043e\u0440\u043e\u0442?", "context": "\u041d\u0430 \u0410\u0432\u0438\u0442\u043e \u043f\u0440\u043e\u0434\u0430\u044e\u0442 \u0433\u0430\u0440\u0430\u0436 \u0441 \u0442\u0430\u043a\u0438\u043c \u043e\u043f\u0438\u0441\u0430\u043d\u0438\u0435\u043c: \n\n\u0415\u0441\u0442\u044c \u043a\u043b\u044e\u0447\u0438 \u043e\u0442 \u043e\u0431\u0435\u0438\u0445 \u0432\u043e\u0440\u043e\u0442 \u0438\u043b\u0438 \u043e\u0442 \u043e\u0431\u043e\u0438\u0445 \u0432\u043e\u0440\u043e\u0442? \u0418\u043b\u0438 \u0441\u043b\u0435\u0434\u0443\u0435\u0442 \u043f\u0438\u0441\u0430\u0442\u044c \u0442\u0430\u043a: \u0438\u043c\u0435\u044e\u0442\u0441\u044f \u043a\u043b\u044e\u0447\u0438 \u043e\u0442 \u043e\u0431\u0435\u0438\u0445 \u0441\u0442\u0432\u043e\u0440\u043e\u043a \u0432\u043e\u0440\u043e\u0442? \n", "answer": "\n\u0415\u0441\u0442\u044c \u043a\u043b\u044e\u0447\u0438 \u043e\u0442 \u043e\u0431\u0435\u0438\u0445 \u0432\u043e\u0440\u043e\u0442 \u0438\u043b\u0438 \u043e\u0442 \u043e\u0431\u043e\u0438\u0445 \u0432\u043e\u0440\u043e\u0442?\n\n\u041e\u0431\u0430 \u0432\u0430\u0440\u0438\u0430\u043d\u0442\u0430 \u043d\u0435\u043d\u043e\u0440\u043c\u0430\u0442\u0438\u0432\u043d\u044b. \n\n\u0418\u043b\u0438 \u0441\u043b\u0435\u0434\u0443\u0435\u0442 \u043f\u0438\u0441\u0430\u0442\u044c \u0442\u0430\u043a: \u0438\u043c\u0435\u044e\u0442\u0441\u044f \u043a\u043b\u044e\u0447\u0438 \u043e\u0442 \u043e\u0431\u0435\u0438\u0445 \u0441\u0442\u0432\u043e\u0440\u043e\u043a \u0432\u043e\u0440\u043e\u0442?\n\n\u0414\u0430, \u0432 \u0441\u043b\u0443\u0447\u0430\u0435, \u0435\u0441\u043b\u0438 \u0440\u0435\u0447\u044c \u0438\u0434\u0435\u0442 \u0438\u043c\u0435\u043d\u043d\u043e \u043e \"\u0441\u0442\u0432\u043e\u0440\u043a\u0430\u0445\". \u041d\u043e \u0432\u043e\u043e\u0431\u0449\u0435-\u0442\u043e \u0443 \u0432\u043e\u0440\u043e\u0442 \u043d\u0435 \u0441\u0442\u0432\u043e\u0440\u043a\u0438, \u0430 \u0441\u0442\u0432\u043e\u0440\u044b. \u041f\u043e\u044d\u0442\u043e\u043c\u0443 \u043b\u0443\u0447\u0448\u0435 \u0431\u0443\u0434\u0435\u0442 \"\u0418\u043c\u0435\u044e\u0442\u0441\u044f \u043a\u043b\u044e\u0447\u0438 \u043e\u0442 \u043e\u0431\u043e\u0438\u0445 \u0441\u0442\u0432\u043e\u0440\u043e\u0432 \u0432\u043e\u0440\u043e\u0442\".\n\u0414\u0440\u0443\u0433\u043e\u0439 \u0432\u043e\u043f\u0440\u043e\u0441, \u0435\u0441\u043b\u0438 \u043c\u044b \u0438\u043c\u0435\u0435\u043c \u0434\u0432\u043e\u0435 \u0432\u043e\u0440\u043e\u0442. \n\u0412\u0440\u043e\u0434\u0435 \u0431\u044b, \u043a\u043e\u043b\u044c \u0441\u043a\u043e\u0440\u043e \u0432\u043e\u0440\u043e\u0442 \u0434\u0432\u043e\u0435, \u0442\u043e \u0438 \u043e\u0431\u043e\u0438\u0445 (\u043e\u0431\u0435 \u0444\u043e\u0440\u043c\u044b - \u043c. \u0440.). \u041d\u043e \u0432\u043e\u0442 \u0442\u0443\u0442 \u0432\u043e\u0437\u043d\u0438\u043a\u0430\u044e\u0442 \u0441\u043b\u043e\u0436\u043d\u043e\u0441\u0442\u0438.\n\u0412\u043e-\u043f\u0435\u0440\u0432\u044b\u0445, \u0441\u0430\u043c\u0430 \u0442\u0430\u043a\u0430\u044f \u0444\u043e\u0440\u043c\u0430 \"\u0434\u0432\u043e\u0435 \u0432\u043e\u0440\u043e\u0442\" \u043d\u0435\u0441\u0432\u043e\u0431\u043e\u0434\u043d\u0430, \u0445\u043e\u0442\u044f \u0432 \u0441\u043e\u0432\u0440\u0435\u043c\u0435\u043d\u043d\u043e\u043c \u044f\u0437\u044b\u043a\u0435 \u043f\u0440\u0438\u0437\u043d\u0430\u0435\u0442\u0441\u044f \u043f\u043e\u0447\u0442\u0438 \u0432\u0441\u0435\u043c\u0438. \n\n\u0412 \u043d\u0435\u043a\u043e\u0442\u043e\u0440\u044b\u0445 \u0441\u043b\u0443\u0447\u0430\u044f\u0445 \u0434\u043b\u044f \u043e\u0431\u043e\u0437\u043d\u0430\u0447\u0435\u043d\u0438\u044f \u043a\u043e\u043b\u0438\u0447\u0435\u0441\u0442\u0432\u0430 \u0438\u0441\u043f\u043e\u043b\u044c\u0437\u0443\u044e\u0442\u0441\u044f, \u043d\u0430\u0440\u044f\u0434\u0443 \u0441\n \u043a\u043e\u043b\u0438\u0447\u0435\u0441\u0442\u0432\u0435\u043d\u043d\u044b\u043c\u0438 \u0447\u0438\u0441\u043b\u0438\u0442\u0435\u043b\u044c\u043d\u044b\u043c\u0438, \u0442\u0430\u043a\u0436\u0435 \u0447\u0438\u0441\u043b\u0438\u0442\u0435\u043b\u044c\u043d\u044b\u0435 \u0441\u043e\u0431\u0438\u0440\u0430\u0442\u0435\u043b\u044c\u043d\u044b\u0435 (\u0434\u0432\u043e\u0435,\n \u0442\u0440\u043e\u0435, \u0447\u0435\u0442\u0432\u0435\u0440\u043e, \u043f\u044f\u0442\u0435\u0440\u043e, \u0448\u0435\u0441\u0442\u0435\u0440\u043e, \u0441\u0435\u043c\u0435\u0440\u043e, \u0432\u043e\u0441\u044c\u043c\u0435\u0440\u043e, \u0434\u0435\u0432\u044f\u0442\u0435\u0440\u043e). \u0414\u0432\u0443\u043c\u044f\n \u043f\u043e\u0441\u043b\u0435\u0434\u043d\u0438\u043c\u0438 \u0447\u0438\u0441\u043b\u0438\u0442\u0435\u043b\u044c\u043d\u044b\u043c\u0438 \u043c\u044b \u043f\u0440\u0430\u043a\u0442\u0438\u0447\u0435\u0441\u043a\u0438 \u043d\u0435 \u043f\u043e\u043b\u044c\u0437\u0443\u0435\u043c\u0441\u044f. \u0421\u043e\u0431\u0438\u0440\u0430\u0442\u0435\u043b\u044c\u043d\u044b\u0435\n \u0447\u0438\u0441\u043b\u0438\u0442\u0435\u043b\u044c\u043d\u044b\u0435 \u0438\u0441\u043f\u043e\u043b\u044c\u0437\u0443\u044e\u0442\u0441\u044f \u0438\u0441\u043f\u043e\u043b\u044c\u0437\u0443\u044e\u0442\u0441\u044f \u043e\u0433\u0440\u0430\u043d\u0438\u0447\u0435\u043d\u043d\u043e, \u0430 \u0438\u043c\u0435\u043d\u043d\u043e:\n ...\n \u0431) \u0441 \u0441\u0443\u0449\u0435\u0441\u0442\u0432\u0438\u0442\u0435\u043b\u044c\u043d\u044b\u043c\u0438, \u0438\u043c\u0435\u044e\u0449\u0438\u043c\u0438 \u0444\u043e\u0440\u043c\u044b \u0442\u043e\u043b\u044c\u043a\u043e \u043c\u043d\u043e\u0436\u0435\u0441\u0442\u0432\u0435\u043d\u043d\u043e\u0433\u043e \u0447\u0438\u0441\u043b\u0430: \u0434\u0432\u043e\u0435 \u043d\u043e\u0436\u043d\u0438\u0446,\n \u0442\u0440\u043e\u0435 \u0441\u0443\u0442\u043e\u043a;\n\nhttp://gramma.ru/RUS/?id=2.13\n\u0410 \u0432\u043e-\u0432\u0442\u043e\u0440\u044b\u0445, \u0432 \u043e\u0442\u043b\u0438\u0447\u0438\u0435 \u043e\u0442 \"\u0434\u0432\u043e\u0435\" \u0444\u043e\u0440\u043c\u0430 \"\u043e\u0431\u0430\"/\"\u043e\u0431\u0435\" \u0432 \u0438\u043c\u0435\u043d\u0438\u0442\u0435\u043b\u044c\u043d\u043e\u043c \u043f\u0430\u0434\u0435\u0436\u0435 \u0441\u043e\u0432\u0435\u0440\u0448\u0435\u043d\u043e \u043d\u0435 \u0432\u044f\u0436\u0435\u0442\u0441\u044f \u0441 \u043c\u043d\u043e\u0436\u0435\u0441\u0442\u0432\u0435\u043d\u043d\u044b\u043c \u0447\u0438\u0441\u043b\u043e\u043c \u0434\u0430\u0436\u0435 \u0432 \u043f\u0440\u043e\u0441\u0442\u043e\u0440\u0435\u0447\u0438\u0438.\n\n\u041f\u0440\u0438 \u0443\u043f\u043e\u0442\u0440\u0435\u0431\u043b\u0435\u043d\u0438\u0438 \u0441\u043e\u0431\u0438\u0440\u0430\u0442\u0435\u043b\u044c\u043d\u043e\u0433\u043e \u0447\u0438\u0441\u043b\u0438\u0442\u0435\u043b\u044c\u043d\u043e\u0433\u043e \u043e\u0431\u0430 - \u043e\u0431\u0435 \u0432 \u0441\u043e\u0447\u0435\u0442\u0430\u043d\u0438\u044f\u0445 \u0441\n \u0441\u0443\u0449\u0435\u0441\u0442\u0432\u0438\u0442\u0435\u043b\u044c\u043d\u044b\u043c\u0438 \u0436\u0435\u043d\u0441\u043a\u043e\u0433\u043e \u0440\u043e\u0434\u0430 \u0432 \u043a\u043e\u0441\u0432\u0435\u043d\u043d\u044b\u0445 \u043f\u0430\u0434\u0435\u0436\u0430\u0445 \u0438\u0441\u043f\u043e\u043b\u044c\u0437\u0443\u044e\u0442\u0441\u044f \u0444\u043e\u0440\u043c\u044b\n \u043e\u0431\u0435\u0438\u0445, \u043e\u0431\u0435\u0438\u043c\u0438, \u043e\u0431\u0435\u0438\u043c \u0438 \u0442. \u0434. \u042d\u0442\u0438 \u0447\u0438\u0441\u043b\u0438\u0442\u0435\u043b\u044c\u043d\u044b\u0435 \u043d\u0435 \u0443\u043f\u043e\u0442\u0440\u0435\u0431\u043b\u044f\u044e\u0442\u0441\u044f \u0441\n \u0441\u0443\u0449\u0435\u0441\u0442\u0432\u0438\u0442\u0435\u043b\u044c\u043d\u044b\u043c\u0438, \u043d\u0435 \u0438\u043c\u0435\u044e\u0449\u0438\u043c\u0438 \u0444\u043e\u0440\u043c\u044b \u0435\u0434\u0438\u043d\u0441\u0442\u0432\u0435\u043d\u043d\u043e\u0433\u043e \u0447\u0438\u0441\u043b\u0430, \u0442\u0430\u043a \u043a\u0430\u043a \u0443 \u043d\u0438\u0445\n \u043e\u0442\u0441\u0443\u0442\u0441\u0442\u0432\u0443\u0435\u0442 \u043a\u0430\u0442\u0435\u0433\u043e\u0440\u0438\u044f \u0440\u043e\u0434\u0430. \u041d\u0435\u043b\u044c\u0437\u044f \u0441\u043a\u0430\u0437\u0430\u0442\u044c \u0443 \u043e\u0431\u043e\u0438\u0445 \u0432\u043e\u0440\u043e\u0442 (\u043d\u0435\u0442 \u0444\u043e\u0440\u043c\u044b\n \u0438\u043c\u0435\u043d\u0438\u0442\u0435\u043b\u044c\u043d\u043e\u0433\u043e \u043f\u0430\u0434\u0435\u0436\u0430: \u043e\u0431\u0430 - \u043e\u0431\u0435 \u0432\u043e\u0440\u043e\u0442\u0430), \u043c\u043e\u0436\u043d\u043e: \u0443 \u0442\u0435\u0445 \u0438 \u0443 \u0434\u0440\u0443\u0433\u0438\u0445\n \u0432\u043e\u0440\u043e\u0442.\n\n(\u0442\u0430\u043c \u0436\u0435, \u043a\u0443\u0440\u0441\u0438\u0432 \u043c\u043e\u0439 - b-s)\n\u0412\u0438\u0434\u0438\u043c\u043e, \u0438 \u043a\u043e\u0441\u0432\u0435\u043d\u043d\u044b\u0435 \u043f\u0430\u0434\u0435\u0436\u0438 (\"\u043e\u0431\u043e\u0438\u0445\"/\"\u043e\u0431\u0435\u0438\u0445\") \u0442\u0443\u0442 \u043d\u0435\u043f\u0440\u0438\u043c\u0435\u043d\u0438\u043c\u044b, \u0445\u043e\u0442\u044f \u043e\u043d\u0438 \u0438 \u043c\u0435\u043d\u044c\u0448\u0435 \u0440\u0435\u0436\u0443\u0442 \u0441\u043b\u0443\u0445, \u0447\u0435\u043c \u0438\u043c\u0435\u043d\u0438\u0442\u0435\u043b\u044c\u043d\u044b\u0439.\n\u0422\u0430\u043a \u0447\u0442\u043e \u0444\u0440\u0430\u0437\u0443 \u0432 \u043b\u044e\u0431\u043e\u043c \u0441\u043b\u0443\u0447\u0430\u0435 \u043d\u0430\u0434\u043e \u043f\u0440\u0430\u0432\u0438\u0442\u044c. \n\u0418\u043b\u0438 \"\u043e\u0442 \u043e\u0431\u043e\u0438\u0445 \u0441\u0442\u0432\u043e\u0440\u043e\u0432\" \u0438\u043b\u0438 \"\u043e\u0442 \u0442\u0435\u0445 \u0438 (\u043e\u0442) \u0434\u0440\u0443\u0433\u0438\u0445 \u0432\u043e\u0440\u043e\u0442\".\n", "topic": "rus", "url": "https://rus.stackexchange.com/questions/444752"}, {"image": "VQAonline_00057354.png", "question": "\u041a\u0440\u0435\u0441\u043b\u043e \u0438\u043b\u0438 \u0441\u0442\u0443\u043b?", "context": "\n\u0427\u0442\u043e \u044d\u0442\u043e, \u043a\u043e\u043c\u043f\u044c\u044e\u0442\u0435\u0440\u043d\u044b\u0439 \u0441\u0442\u0443\u043b \u0438\u043b\u0438 \u043a\u043e\u043c\u043f\u044c\u044e\u0442\u0435\u0440\u043d\u043e\u0435 \u043a\u0440\u0435\u0441\u043b\u043e? \u041e\u0442\u0447\u0435\u0433\u043e \u0441\u0445\u043e\u0434\u043d\u044b\u0435 \u043f\u043e \u0444\u043e\u0440\u043c\u0435 \u043f\u0440\u0435\u0434\u043c\u0435\u0442\u044b \u043c\u0435\u0431\u0435\u043b\u0438 \u043c\u043e\u0433\u0443\u0442 \u043e\u0431\u043e\u0437\u043d\u0430\u0447\u0430\u0442\u044c\u0441\u044f \u0440\u0430\u0437\u043b\u0438\u0447\u043d\u043e? \u041f\u0440\u043e\u0432\u0435\u0434\u0438\u0442\u0435, \u043f\u043e\u0436\u0430\u043b\u0443\u0439\u0441\u0442\u0430, \u0433\u0440\u0430\u043d\u0438\u0446\u0443 \u043c\u0435\u0436\u0434\u0443 \u043f\u043e\u043d\u044f\u0442\u0438\u044f\u043c\u0438 \"\u041a\u0440\u0435\u0441\u043b\u043e\" \u0438 \"\u0421\u0442\u0443\u043b\". \u041d\u0430 \u043a\u0430\u043a\u043e\u043c \u0441\u0443\u0449\u0435\u0441\u0442\u0432\u0435\u043d\u043d\u043e\u043c \u043e\u0441\u043d\u043e\u0432\u0430\u043d\u0438\u0438 \u043c\u044b \u0438\u043c\u0435\u0435\u043c \u043f\u0440\u0430\u0432\u043e \u0438\u0445 \u0440\u0430\u0437\u0434\u0435\u043b\u044f\u0442\u044c? \n", "answer": "\u0412 \u043d\u0430\u0441\u0442\u043e\u044f\u0449\u0435\u0435 \u0432\u0440\u0435\u043c\u044f \u0432\u044b\u043f\u0443\u0441\u043a\u0430\u0435\u0442\u0441\u044f \u0431\u043e\u043b\u044c\u0448\u043e\u0435 \u043a\u043e\u043b\u0438\u0447\u0435\u0441\u0442\u0432\u043e \u043f\u0440\u0435\u0434\u043c\u0435\u0442\u043e\u0432 \u043c\u0435\u0431\u0435\u043b\u0438 \u0434\u043b\u044f \u0441\u0438\u0434\u0435\u043d\u0438\u044f. \u041a \u043d\u0438\u043c \u043e\u0442\u043d\u043e\u0441\u044f\u0442\u0441\u044f \u0441\u0442\u0443\u043b\u044c\u044f, \u043a\u0440\u0435\u0441\u043b\u0430, \u043f\u043e\u043b\u0443\u043a\u0440\u0435\u0441\u043b\u0430. \n\n\u041e\u0441\u043d\u043e\u0432\u043d\u043e\u0439 \u043f\u0440\u0438\u0437\u043d\u0430\u043a \u0434\u043b\u044f \u0440\u0430\u0437\u043b\u0438\u0447\u0435\u043d\u0438\u044f \u2014 \u0432\u044b\u0441\u043e\u0442\u0430 \u0441\u0438\u0434\u0435\u043d\u044c\u044f.\n\u0424\u0443\u043d\u043a\u0446\u0438\u044f \u0441\u0442\u0443\u043b\u0430 \u2014 \u043f\u0440\u0438\u0434\u0430\u0442\u044c \u0442\u0435\u043b\u0443 \u0447\u0435\u043b\u043e\u0432\u0435\u043a\u0430 \u0443\u0434\u043e\u0431\u043d\u043e\u0435 \u043f\u043e\u043b\u043e\u0436\u0435\u043d\u0438\u0435 \u0437\u0430 \u043e\u0431\u0435\u0434\u0435\u043d\u043d\u044b\u043c \u0438\u043b\u0438 \u0440\u0430\u0431\u043e\u0447\u0438\u043c \u0441\u0442\u043e\u043b\u043e\u043c, \u043f\u043e\u044d\u0442\u043e\u043c\u0443 \u0441\u0438\u0434\u0435\u043d\u044c\u0435 \u043d\u0430\u0445\u043e\u0434\u0438\u0442\u0441\u044f \u043d\u0430 \u0440\u0430\u0441\u0441\u0442\u043e\u044f\u043d\u0438\u0438 45\u201450 \u0441\u043c \u043e\u0442 \u043f\u043e\u0432\u0435\u0440\u0445\u043d\u043e\u0441\u0442\u0438 \u043f\u043e\u043b\u0430.\n\u0412 \u0441\u043b\u0443\u0447\u0430\u0435 \u043a\u0440\u0435\u0441\u043b\u0430 \u044d\u0442\u043e\u0442 \u043f\u0430\u0440\u0430\u043c\u0435\u0442\u0440 \u043d\u0438\u0436\u0435 \u0438 \u0441\u043e\u0441\u0442\u0430\u0432\u043b\u044f\u0435\u0442 40\u201442 \u0441\u043c. \n\u0421\u0442\u0443\u043b \u0438\u043c\u0435\u0435\u0442 \u043d\u0435\u0433\u043b\u0443\u0431\u043e\u043a\u043e\u0435 (50\u201455 \u0441\u043c) \u0438 \u0443\u0437\u043a\u043e\u0435 (43 \u0441\u043c) \u0441\u0438\u0434\u0435\u043d\u044c\u0435, \u0441\u043f\u0438\u043d\u043a\u0430 \u043e\u0431\u044b\u0447\u043d\u043e \u0437\u0430\u0444\u0438\u043a\u0441\u0438\u0440\u043e\u0432\u0430\u043d\u0430 \u0432 \u043e\u0434\u043d\u043e\u043c \u043f\u043e\u043b\u043e\u0436\u0435\u043d\u0438\u0438. \u0415\u0435 \u0432\u044b\u0441\u043e\u0442\u0430 \u043d\u0430\u0445\u043e\u0434\u0438\u0442\u0441\u044f \u0432 \u043f\u0440\u0435\u0434\u0435\u043b\u0430\u0445 \u043e\u0442 40 \u0434\u043e 45 \u0441\u043c.\n\u041d\u043e \u0441\u0443\u0449\u0435\u0441\u0442\u0432\u0443\u044e\u0442 \u043c\u043e\u0434\u0435\u043b\u0438 \u0434\u043b\u044f \u0440\u0430\u0431\u043e\u0442\u044b, \u043a\u043e\u0442\u043e\u0440\u044b\u0435 \u043f\u0440\u0435\u0434\u043e\u0441\u0442\u0430\u0432\u043b\u044f\u044e\u0442 \u0432\u043e\u0437\u043c\u043e\u0436\u043d\u043e\u0441\u0442\u044c \u0440\u0435\u0433\u0443\u043b\u0438\u0440\u043e\u0432\u0430\u0442\u044c \u0443\u0433\u043e\u043b \u043d\u0430\u043a\u043b\u043e\u043d\u0430 \u0441\u043f\u0438\u043d\u043a\u0438, \u0432\u044b\u0441\u043e\u0442\u0443, \u0430 \u0442\u0430\u043a\u0436\u0435 \u0448\u0438\u0440\u0438\u043d\u0443 \u0438 \u0433\u043b\u0443\u0431\u0438\u043d\u0443 \u0441\u0438\u0434\u0435\u043d\u044c\u044f \u2014 \u044d\u0442\u043e \u043f\u043e\u0437\u0432\u043e\u043b\u044f\u0435\u0442 \u0441\u043e\u0437\u0434\u0430\u0442\u044c \u043a\u043e\u043c\u0444\u043e\u0440\u0442\u043d\u044b\u0435 \u0443\u0441\u043b\u043e\u0432\u0438\u044f. \u0422\u0430\u043a\u043e\u0439 \u0432\u0438\u0434 \u043c\u0435\u0431\u0435\u043b\u0438 \u043d\u0430\u0437\u044b\u0432\u0430\u044e\u0442 \u043e\u0444\u0438\u0441\u043d\u044b\u043c\u0438 \u043a\u0440\u0435\u0441\u043b\u0430\u043c\u0438, \u043d\u043e \u043f\u043e \u0441\u0432\u043e\u0435\u0439 \u0441\u0443\u0442\u0438 \u043e\u043d\u0438 \u0432\u0441\u0435 \u0440\u0430\u0432\u043d\u043e \u0441\u0447\u0438\u0442\u0430\u044e\u0442\u0441\u044f \u0441\u0442\u0443\u043b\u044c\u044f\u043c\u0438, \u043d\u0435\u0441\u043c\u043e\u0442\u0440\u044f \u043d\u0430 \u0440\u0430\u0437\u043b\u0438\u0447\u043d\u044b\u0435 \u0444\u0443\u043d\u043a\u0446\u0438\u0438 \u0438 \u0434\u043e\u043f\u043e\u043b\u043d\u0438\u0442\u0435\u043b\u044c\u043d\u044b\u0435 \u043a\u043e\u043d\u0441\u0442\u0440\u0443\u043a\u0446\u0438\u043e\u043d\u043d\u044b\u0435 \u044d\u043b\u0435\u043c\u0435\u043d\u0442\u044b (\u043f\u043e\u0434\u043b\u043e\u043a\u043e\u0442\u043d\u0438\u043a\u0438, \u043f\u043e\u0434\u0433\u043e\u043b\u043e\u0432\u043d\u0438\u043a, \u043f\u043e\u0434\u043a\u043e\u043b\u0435\u043d\u043d\u0438\u043a\u0438).\n\u041a\u0440\u0435\u0441\u043b\u043e \u043e\u0431\u043b\u0430\u0434\u0430\u0435\u0442 \u0431\u043e\u043b\u044c\u0448\u0438\u043c\u0438 \u0440\u0430\u0437\u043c\u0435\u0440\u0430\u043c\u0438. \u0413\u043b\u0443\u0431\u0438\u043d\u0430 \u0438\u0437\u0434\u0435\u043b\u0438\u044f, \u043f\u0440\u0435\u0434\u043d\u0430\u0437\u043d\u0430\u0447\u0435\u043d\u043d\u043e\u0433\u043e \u0434\u043b\u044f \u043e\u0442\u0434\u044b\u0445\u0430, \u0441\u043e\u0441\u0442\u0430\u0432\u043b\u044f\u0435\u0442 \u043e\u0442 45 \u0434\u043e 60 \u0441\u043c, \u0430 \u0448\u0438\u0440\u0438\u043d\u0430 \u043e\u0442 48 \u0434\u043e 60 \u0441\u043c.\n\u0412 \u0441\u0442\u0430\u0446\u0438\u043e\u043d\u0430\u0440\u043d\u044b\u0445 \u043c\u043e\u0434\u0435\u043b\u044f\u0445 \u043d\u0430\u043a\u043b\u043e\u043d \u0441\u043f\u0438\u043d\u043a\u0438 \u043d\u0430\u0445\u043e\u0434\u0438\u0442\u0441\u044f \u0432 \u0438\u043d\u0442\u0435\u0440\u0432\u0430\u043b\u0435 \u043e\u0442 110 \u0434\u043e 125\u00b0. \u0421\u0443\u0449\u0435\u0441\u0442\u0432\u0443\u044e\u0442 \u043a\u043e\u043d\u0441\u0442\u0440\u0443\u043a\u0446\u0438\u0438, \u043f\u043e\u0437\u0432\u043e\u043b\u044f\u044e\u0449\u0438\u0435 \u0440\u0435\u0433\u0443\u043b\u0438\u0440\u043e\u0432\u0430\u0442\u044c \u044d\u0442\u043e \u0437\u043d\u0430\u0447\u0435\u043d\u0438\u0435, \u0447\u0442\u043e \u0434\u0430\u0435\u0442 \u0432\u043e\u0437\u043c\u043e\u0436\u043d\u043e\u0441\u0442\u044c \u043f\u0440\u0438\u043d\u044f\u0442\u044c \u0440\u0430\u0441\u0441\u043b\u0430\u0431\u043b\u044f\u044e\u0449\u0443\u044e \u043f\u043e\u0437\u0443. \n\u041c\u043d\u043e\u0433\u0438\u0435 \u0441\u0447\u0438\u0442\u0430\u044e\u0442, \u0447\u0442\u043e \u0440\u0430\u0437\u043d\u0438\u0446\u0430 \u043c\u0435\u0436\u0434\u0443 \u0441\u0442\u0443\u043b\u043e\u043c \u0438 \u043a\u0440\u0435\u0441\u043b\u043e\u043c \u0437\u0430\u043a\u043b\u044e\u0447\u0430\u0435\u0442\u0441\u044f \u0432 \u043d\u0430\u043b\u0438\u0447\u0438\u0438 \u043f\u043e\u0434\u043b\u043e\u043a\u043e\u0442\u043d\u0438\u043a\u043e\u0432. \u0414\u0438\u0437\u0430\u0439\u043d\u0435\u0440\u044b \u043d\u0430\u0437\u044b\u0432\u0430\u044e\u0442 \u0441\u0442\u0443\u043b \u0441 \u043f\u043e\u0434\u043b\u043e\u043a\u043e\u0442\u043d\u0438\u043a\u0430\u043c\u0438 \u043f\u043e\u043b\u0443\u043a\u0440\u0435\u0441\u043b\u043e\u043c, \u0445\u043e\u0442\u044f \u0441\u0443\u0442\u044c \u043f\u0440\u0435\u0434\u043c\u0435\u0442\u0430 \u043e\u0442 \u044d\u0442\u043e\u0433\u043e \u043d\u0435 \u043c\u0435\u043d\u044f\u0435\u0442\u0441\u044f. \u0415\u0441\u0442\u044c \u043c\u043e\u0434\u0435\u043b\u0438 \u043a\u0440\u0435\u0441\u0435\u043b, \u0432 \u043a\u043e\u0442\u043e\u0440\u044b\u0445 \u043f\u043e\u0434\u043b\u043e\u043a\u043e\u0442\u043d\u0438\u043a\u043e\u0432 \u0432\u043e\u043e\u0431\u0449\u0435 \u043d\u0435\u0442. \n\u0416\u0435\u0441\u0442\u043a\u043e\u0441\u0442\u044c \u2014 \u043d\u0435 \u043f\u043e\u043a\u0430\u0437\u0430\u0442\u0435\u043b\u044c. \u0421\u043e\u0432\u0440\u0435\u043c\u0435\u043d\u043d\u044b\u0435 \u043a\u0440\u0435\u0441\u043b\u0430 \u0432\u044b\u043f\u043e\u043b\u043d\u044f\u044e\u0442\u0441\u044f \u0438\u0437 \u043d\u0430\u0442\u0443\u0440\u0430\u043b\u044c\u043d\u043e\u0433\u043e \u0434\u0435\u0440\u0435\u0432\u0430, \u043c\u0435\u0442\u0430\u043b\u043b\u0430, \u0441\u0442\u0435\u043a\u043b\u0430, \u043f\u043b\u0430\u0441\u0442\u0438\u043a\u0430 \u0438 \u043c\u043e\u0433\u0443\u0442 \u043d\u0435 \u0438\u043c\u0435\u0442\u044c \u043c\u044f\u0433\u043a\u043e\u0439 \u043e\u0431\u0438\u0432\u043a\u0438. \u0414\u043b\u044f \u0441\u043e\u0437\u0434\u0430\u043d\u0438\u044f \u043d\u0435\u043e\u0431\u0445\u043e\u0434\u0438\u043c\u043e\u0433\u043e \u043a\u043e\u043c\u0444\u043e\u0440\u0442\u0430 \u0438\u0445 \u0434\u043e\u043f\u043e\u043b\u043d\u044f\u044e\u0442 \u043f\u043e\u0434\u0443\u0448\u043a\u0430\u043c\u0438 \u0438\u043b\u0438 \u0441\u044a\u0435\u043c\u043d\u044b\u043c\u0438 \u043f\u043e\u043a\u0440\u044b\u0442\u0438\u044f\u043c\u0438. \n\n\u0418\u0442\u0430\u043a, \u0435\u0441\u043b\u0438 \u043f\u0440\u0435\u0434\u043c\u0435\u0442 \u043c\u0435\u0431\u0435\u043b\u0438 \u0434\u0430\u0435\u0442 \u0432\u043e\u0437\u043c\u043e\u0436\u043d\u043e\u0441\u0442\u044c \u0443\u0434\u043e\u0431\u043d\u043e \u0440\u0430\u0441\u043f\u043e\u043b\u043e\u0436\u0438\u0442\u044c\u0441\u044f \u0437\u0430 \u0441\u0442\u043e\u043b\u043e\u043c \u0432\u043e \u0432\u0440\u0435\u043c\u044f \u0440\u0430\u0431\u043e\u0442\u044b \u0438\u043b\u0438 \u043f\u0440\u0438\u0435\u043c\u0430 \u043f\u0438\u0449\u0438, \u0442\u043e \u044d\u0442\u043e \u043f\u0440\u0438\u0432\u044b\u0447\u043d\u044b\u0439 \u0441\u0442\u0443\u043b. (\u041f\u043e\u0434\u043b\u043e\u043a\u043e\u0442\u043d\u0438\u043a\u0438 \u0438 \u043c\u044f\u0433\u043a\u043e\u0441\u0442\u044c \u0441\u0438\u0434\u0435\u043d\u044c\u044f \u043d\u0435 \u044f\u0432\u043b\u044f\u044e\u0442\u0441\u044f \u043e\u043f\u0440\u0435\u0434\u0435\u043b\u044f\u044e\u0449\u0438\u043c\u0438 \u0445\u0430\u0440\u0430\u043a\u0442\u0435\u0440\u0438\u0441\u0442\u0438\u043a\u0430\u043c\u0438.) \u0410 \u0432\u043e\u0442 \u043a\u0440\u0435\u0441\u043b\u043e \u0434\u0430\u0435\u0442 \u0432\u043e\u0437\u043c\u043e\u0436\u043d\u043e\u0441\u0442\u044c \u0440\u0430\u0441\u0441\u043b\u0430\u0431\u0438\u0442\u044c\u0441\u044f \u043f\u043e\u0441\u043b\u0435 \u0442\u0440\u0443\u0434\u043e\u0432\u043e\u0433\u043e \u0434\u043d\u044f, \u0437\u0430\u043d\u044f\u0442\u044c \u0441\u0430\u043c\u0443\u044e \u0443\u0434\u043e\u0431\u043d\u0443\u044e \u043f\u043e\u0437\u0443 \u0438 \u0441\u043e\u0432\u0435\u0440\u0448\u0435\u043d\u043d\u043e \u043d\u0435 \u0440\u0430\u0441\u043f\u043e\u043b\u0430\u0433\u0430\u0435\u0442 \u043a \u0442\u0440\u0443\u0434\u043e\u0432\u044b\u043c \u043f\u043e\u0434\u0432\u0438\u0433\u0430\u043c. \n\u0427\u0435\u043c \u043e\u0442\u043b\u0438\u0447\u0430\u0435\u0442\u0441\u044f \u0441\u0442\u0443\u043b \u043e\u0442 \u043a\u0440\u0435\u0441\u043b\u0430?\n", "topic": "rus", "url": "https://rus.stackexchange.com/questions/452785"}, {"image": "VQAonline_00057357.png", "question": "\u041c\u0430\u0442\u0443\u0448\u043a\u0430-\u0437\u0435\u043c\u043b\u044f (-\u043f\u0440\u0438\u0440\u043e\u0434\u0430, -\u0420\u043e\u0441\u0441\u0438\u044f, -\u0412\u043e\u043b\u0433\u0430), \u043d\u043e \u043c\u0430\u0442\u0443\u0448\u043a\u0430 \u0433\u043e\u0441\u0443\u0434\u0430\u0440\u044b\u043d\u044f (\u0438\u0433\u0443\u043c\u0435\u043d\u044c\u044f, \u043d\u0430\u0441\u0442\u043e\u044f\u0442\u0435\u043b\u044c\u043d\u0438\u0446\u0430, \u0446\u0430\u0440\u0438\u0446\u0430), - \u043f\u043e\u0447\u0435\u043c\u0443?", "context": "\u041f\u043e\u0447\u0435\u043c\u0443 \"\u043c\u0430\u0442\u0443\u0448\u043a\u0430\" \u0442\u043e \u0441 \u0434\u0435\u0444\u0438\u0441\u043e\u043c, \u0442\u043e \u0431\u0435\u0437? \u0415\u0441\u0442\u044c \u043b\u0438 \u043f\u0440\u0430\u0432\u0438\u043b\u043e?\n\u0421\u043b\u043e\u0432\u0430\u0440\u044c \u0441\u0435\u0431\u0435 \u043f\u0440\u043e\u0442\u0438\u0432\u043e\u0440\u0435\u0447\u0438\u0442 (\u0441\u043c. \u043c\u0430\u0442\u0443\u0448\u043a\u0430-\u0420\u0443\u0441\u044c - \u043c\u0430\u0442\u0443\u0448\u043a\u0430 \u0420\u043e\u0441\u0441\u0438\u044f):\n\n", "answer": "\u0421\u043b\u043e\u0432\u043e \u043c\u0430\u0442\u0443\u0448\u043a\u0430 \u043c\u043e\u0436\u0435\u0442 \u0438\u0441\u043f\u043e\u043b\u044c\u0437\u043e\u0432\u0430\u0442\u044c\u0441\u044f \u043a\u0430\u043a \u043e\u0431\u0440\u0430\u0449\u0435\u043d\u0438\u0435 \u0438 \u043a\u0430\u043a \u043f\u0440\u0438\u043b\u043e\u0436\u0435\u043d\u0438\u0435. \u0422\u0430\u043a\u0436\u0435 \u044d\u0442\u043e \u0441\u043b\u043e\u0432\u043e \u043c\u043e\u0436\u0435\u0442 \u043e\u0442\u043d\u043e\u0441\u0438\u0442\u044c\u0441\u044f \u043a \u043d\u0430\u0440\u0438\u0446\u0430\u0442\u0435\u043b\u044c\u043d\u043e\u043c\u0443 \u0438 \u0441\u043e\u0431\u0441\u0442\u0432\u0435\u043d\u043d\u043e\u043c\u0443 \u0438\u043c\u0435\u043d\u0438. \u0412\u0441\u0435 \u044d\u0442\u0438 \u0432\u0430\u0440\u0438\u0430\u043d\u0442\u044b \u043d\u0430\u0434\u043e \u0440\u0430\u0437\u043b\u0438\u0447\u0430\u0442\u044c.\n\u041f\u043e \u043e\u0431\u0449\u0438\u043c \u043f\u0440\u0430\u0432\u0438\u043b\u0430\u043c \u0420\u043e\u0437\u0435\u043d\u0442\u0430\u043b\u044f \u043e\u0431\u0440\u0430\u0449\u0435\u043d\u0438\u0435 \u043f\u0438\u0448\u0435\u0442\u0441\u044f \u0440\u0430\u0437\u0434\u0435\u043b\u044c\u043d\u043e (\u0441\u0440\u0430\u0432\u043d\u0438\u0442\u044c: \u0442\u043e\u0432\u0430\u0440\u0438\u0449, \u0433\u0440\u0430\u0436\u0434\u0430\u043d\u0438\u043d, \u0433\u043e\u0441\u043f\u043e\u0436\u0430 \u0438 \u0442.\u0434.).\n\u041d\u0430\u043f\u0438\u0441\u0430\u043d\u0438\u0435 \u043f\u0440\u0438\u043b\u043e\u0436\u0435\u043d\u0438\u044f \u0437\u0430\u0432\u0438\u0441\u0438\u0442 \u043e\u0442 \u043f\u043e\u0437\u0438\u0446\u0438\u0438 \u0434\u043b\u044f \u0438\u043c\u0435\u043d \u0441\u043e\u0431\u0441\u0442\u0432\u0435\u043d\u043d\u044b\u0445: \u0440\u0430\u0437\u0434\u0435\u043b\u044c\u043d\u043e \u043f\u0435\u0440\u0435\u0434 \u0438\u043c\u0435\u043d\u0435\u043c \u0441\u043e\u0431\u0441\u0442\u0432\u0435\u043d\u043d\u044b\u043c \u0438 \u0447\u0435\u0440\u0435\u0437 \u0434\u0435\u0444\u0438\u0441 \u043f\u043e\u0441\u043b\u0435 \u043d\u0435\u0433\u043e.\n\u041f\u0440\u0438\u043b\u043e\u0436\u0435\u043d\u0438\u044f, \u043e\u0442\u043d\u0435\u0441\u0435\u043d\u043d\u044b\u0435 \u043a \u043d\u0430\u0440\u0438\u0446\u0430\u0442\u0435\u043b\u044c\u043d\u043e\u043c\u0443 \u0441\u0443\u0449\u0435\u0441\u0442\u0432\u0438\u0442\u0435\u043b\u044c\u043d\u043e\u043c\u0443, \u043f\u0438\u0448\u0443\u0442\u0441\u044f \u0447\u0435\u0440\u0435\u0437 \u0434\u0435\u0444\u0438\u0441 \u0432 \u043b\u044e\u0431\u043e\u0439 \u043f\u043e\u0437\u0438\u0446\u0438\u0438, \u043d\u043e \u0440\u0430\u0437\u0434\u0435\u043b\u044c\u043d\u043e\u0435 \u043d\u0430\u043f\u0438\u0441\u0430\u043d\u0438\u0435 \u043a\u0430\u043a \u0438\u0441\u043a\u043b\u044e\u0447\u0435\u043d\u0438\u0435 \u0434\u043b\u044f \u043f\u0440\u0438\u043b\u043e\u0436\u0435\u043d\u0438\u0439 \u0441 \u043a\u0430\u0447\u0435\u0441\u0442\u0432\u0435\u043d\u043d\u044b\u043c (\u043e\u043f\u0440\u0435\u0434\u0435\u043b\u0438\u0442\u0435\u043b\u044c\u043d\u044b\u043c) \u0437\u043d\u0430\u0447\u0435\u043d\u0438\u0435\u043c (\u043a\u0440\u0430\u0441\u0430\u0432\u0438\u0446\u0430 \u043d\u0435\u0432\u0435\u0441\u0442\u0430), \u0432 \u041f\u0410\u0421 \u044d\u0442\u043e\u0433\u043e \u0438\u0441\u043a\u043b\u044e\u0447\u0435\u043d\u0438\u044f \u043d\u0435\u0442.\n\u0418 \u0447\u0442\u043e \u043c\u044b \u0442\u043e\u0433\u0434\u0430 \u0438\u043c\u0435\u0435\u043c:\n\u043c\u0430\u0442\u0443\u0448\u043a\u0430 \u0420\u043e\u0441\u0441\u0438\u044f, \u043c\u0430\u0442\u0443\u0448\u043a\u0430 \u0420\u0443\u0441\u044c, \u0420\u043e\u0441\u0441\u0438\u044f-\u043c\u0430\u0442\u0443\u0448\u043a\u0430, \u0420\u0443\u0441\u044c-\u043c\u0430\u0442\u0443\u0448\u043a\u0430, \u0412\u043e\u043b\u0433\u0430-\u043c\u0430\u0442\u0443\u0448\u043a\u0430;\n\u043b\u0435\u043d\u044c-\u043c\u0430\u0442\u0443\u0448\u043a\u0430, \u043c\u0430\u0442\u0443\u0448\u043a\u0430-\u043f\u0440\u0438\u0440\u043e\u0434\u0430, \u0437\u0435\u043c\u043b\u044f-\u043c\u0430\u0442\u0443\u0448\u043a\u0430, \u043c\u0430\u0442\u0443\u0448\u043a\u0430-\u0437\u0435\u043c\u043b\u044f, \u0434\u0435\u0440\u0435\u0432\u043d\u044f-\u043c\u0430\u0442\u0443\u0448\u043a\u0430;\n\u043c\u0430\u0442\u0443\u0448\u043a\u0430 \u0446\u0430\u0440\u0438\u0446\u0430, \u043c\u0430\u0442\u0443\u0448\u043a\u0430 \u0438\u0433\u0443\u043c\u0435\u043d\u044c\u044f (\u043e\u0431\u0440\u0430\u0449\u0435\u043d\u0438\u044f), \u0443 \u043c\u0430\u0442\u0443\u0448\u043a\u0438 \u0446\u0430\u0440\u0438\u0446\u044b (\u043e\u043f\u0440\u0435\u0434\u0435\u043b\u0438\u0442\u0435\u043b\u044c\u043d\u043e\u0435 \u0437\u043d\u0430\u0447\u0435\u043d\u0438\u0435 \u043f\u0440\u0438\u043b\u043e\u0436\u0435\u043d\u0438\u044f);\n\u0420\u043e\u0437\u0435\u043d\u0442\u0430\u043b\u044c \u0434\u043e\u043f\u0443\u0441\u043a\u0430\u0435\u0442 \u043d\u0430\u043f\u0438\u0441\u0430\u043d\u0438\u0435 \u043c\u0430\u0442\u0443\u0448\u043a\u0430-\u0420\u0443\u0441\u044c \u043a\u0430\u043a \u0443\u0441\u0442\u043e\u0439\u0447\u0438\u0432\u043e\u0435. http://old-rozental.ru/punctuatio.php?sid=111#pp111\n\u041f\u0440\u0438 \u043d\u0430\u043b\u0438\u0447\u0438\u0438 \u043e\u043f\u0440\u0435\u0434\u0435\u043b\u0435\u043d\u0438\u044f \u0432\u043e\u0437\u043c\u043e\u0436\u043d\u043e \u0434\u0435\u0444\u0438\u0441\u043d\u043e\u0435 \u043d\u0430\u043f\u0438\u0441\u0430\u043d\u0438\u0435: \u043d\u0430\u0448\u0430 \u043c\u0430\u0442\u0443\u0448\u043a\u0430-\u0420\u043e\u0441\u0441\u0438\u044f.\n\u041f\u0440\u0438\u043c\u0435\u0440\u044b:\n\u041e\u0445, \u0442\u044b \u0420\u0443\u0441\u044c-\u043c\u0430\u0442\u0443\u0448\u043a\u0430\u2026 \u0418 \u0432\u043f\u0440\u044f\u043c\u044c \u2015 \u0443\u043c\u043e\u043c \u0442\u0435\u0431\u044f \u043f\u043e\u043d\u044f\u0442\u044c \u043d\u0435\u0432\u043e\u0437\u043c\u043e\u0436\u043d\u043e. [\u00ab\u041a\u0440\u0438\u043c\u0438\u043d\u0430\u043b\u044c\u043d\u0430\u044f \u0445\u0440\u043e\u043d\u0438\u043a\u0430\u00bb, 2003.07.24]\n\u041b\u044e\u0434\u0438 \u0431\u044b\u043b\u0438 \u041d\u0435\u044f\u0441\u043d\u044b \u0441\u0430\u043c\u0438\u043c \u0441\u0435\u0431\u0435. \u0410\u0445 \u0442\u044b, \u043c\u0430\u0442\u0443\u0448\u043a\u0430 \u0420\u043e\u0441\u0441\u0438\u044f, \u0447\u0442\u043e \u0442\u044b \u0434\u0435\u043b\u0430\u0435\u0448\u044c \u0441\u043e \u043c\u043d\u043e\u0439? \u0422\u043e \u043b\u0438 \u0432\u0441\u0435 \u0432\u043e\u043a\u0440\u0443\u0433 \u0441\u043c\u0443\u0440\u043d\u044b\u0435? \u0422\u043e \u043b\u0438 \u044f \u043e\u0434\u0438\u043d \u0441\u043c\u0443\u0440\u043d\u043e\u0439? [\u0412\u0430\u0441\u0438\u043b\u0438\u0439 \u0410\u043a\u0441\u0435\u043d\u043e\u0432. \u0422\u0430\u0438\u043d\u0441\u0442\u0432\u0435\u043d\u043d\u0430\u044f \u0441\u0442\u0440\u0430\u0441\u0442\u044c (2007)]\n\u0410 \u043f\u0440\u0438\u0440\u043e\u0441\u0442 \u043d\u0430\u0441\u0435\u043b\u0435\u043d\u0438\u044f \u0432 \u043f\u043e\u043b\u0442\u043e\u0440\u0430 \u0440\u0430\u0437\u0430 \u0443\u0432\u0435\u043b\u0438\u0447\u0438\u043b\u0441\u044f. \u041e\u043f\u044f\u0442\u044c \u043e\u0442\u0434\u0443\u0432\u0430\u0435\u0442\u0441\u044f \u0434\u0435\u0440\u0435\u0432\u043d\u044f-\u043c\u0430\u0442\u0443\u0448\u043a\u0430. \u041d\u0435 \u0445\u043e\u0442\u0438\u043c \u043c\u044b \u0440\u043e\u0436\u0430\u0442\u044c \u0432 \u0433\u043e\u0440\u043e\u0434\u0435, \u0438 \u0432\u0441\u0451 \u0442\u0443\u0442. [\u0412\u0430\u0441\u0438\u043b\u0438\u0439 \u0428\u0443\u043a\u0448\u0438\u043d. \u041f\u0435\u0447\u043a\u0438-\u043b\u0430\u0432\u043e\u0447\u043a\u0438 (1970-1972)]\n\u041e\u043d \u0443\u043b\u044b\u0431\u043d\u0443\u043b\u0441\u044f. \u2015 \u041f\u043e\u0434\u043e\u0436\u0434\u0438\u0442\u0435, \u0432\u043e\u0439\u043d\u0430 \u043a\u043e\u043d\u0447\u0438\u0442\u0441\u044f \u043f\u043e\u0431\u0435\u0434\u043e\u0439, \u0438 \u0442\u043e\u0433\u0434\u0430 \u0438\u043d\u0442\u0435\u0440\u043d\u0430\u0446\u0438\u043e\u043d\u0430\u043b\u0438\u0441\u0442\u044b \u043e\u0431\u044a\u044f\u0432\u044f\u0442: \" \u041d\u0430\u0448\u0430 \u043c\u0430\u0442\u0443\u0448\u043a\u0430-\u0420\u043e\u0441\u0441\u0438\u044f \u0432\u0441\u0435\u043c\u0443 \u0441\u0432\u0435\u0442\u0443 \u0433\u043e\u043b\u043e\u0432\u0430\". [\u0412\u0430\u0441\u0438\u043b\u0438\u0439 \u0413\u0440\u043e\u0441\u0441\u043c\u0430\u043d. \u0416\u0438\u0437\u043d\u044c \u0438 \u0441\u0443\u0434\u044c\u0431\u0430, \u0447. 1 (1960)]\n\u0421\u0430\u043c\u0438 \u043f\u043e\u0432\u0435\u0440\u0438\u043b\u0438 \u0432 \u0441\u0432\u043e\u0439 \u0442\u0438\u0442\u0430\u043d\u0438\u0437\u043c. \u0422\u044b \u0436\u0435 \u0443\u0433\u043e\u0434\u043b\u0438\u0432\u0430, \u0442\u044b \u0438 \u0437\u0430\u0432\u0438\u0441\u0442\u043b\u0438\u0432\u0430, \u043c\u0430\u0442\u0443\u0448\u043a\u0430 \u0420\u0443\u0441\u044c. \u0421\u043b\u043e\u0436\u043d\u044b\u0439 \u0445\u0430\u0440\u0430\u043a\u0442\u0435\u0440. \u0412 \u0442\u0430\u043a\u043e\u0439 \u0434\u0438\u0445\u043e\u0442\u043e\u043c\u0438\u0438 \u0432\u0441\u044f \u0435\u0435 \u0441\u043b\u0430\u0441\u0442\u044c. [\u041b\u0435\u043e\u043d\u0438\u0434 \u0417\u043e\u0440\u0438\u043d. \u0413\u043b\u0430\u0441 \u043d\u0430\u0440\u043e\u0434\u0430 (2007-2008) // \u00ab\u0417\u043d\u0430\u043c\u044f\u00bb, 2008]\n", "topic": "rus", "url": "https://rus.stackexchange.com/questions/455807"}, {"image": "VQAonline_00057436.png", "question": "What is the difference between \"\u041a\u0443\u0434\u0430 \u0442\u044b \u0438\u0434\u0451\u0448\u044c?\" and \"\u041a\u0443\u0434\u0430 \u0442\u044b \u0441\u043e\u0431\u0440\u0430\u043b\u0441\u044f?\"", "context": "I recently learned that\n\n\u041a\u0443\u0434\u0430 \u0442\u044b \u0441\u043e\u0431\u0440\u0430\u043b\u0441\u044f?\n\nusually gets translated as\n\nWhere are you going?\n\nOr, at least it does on Reverso.\nThis raised at least a couple of questions for me. The first is the title of this post \u2014 What is the difference between\n\nK\u0443\u0434\u0430 \u0442\u044b \u0438\u0434\u0451\u0448\u044c?\n\nand\n\n\u041a\u0443\u0434\u0430 \u0442\u044b \u0441\u043e\u0431\u0440\u0430\u043b\u0441\u044f?\n\nThe second thing that piqued my curiosity is the fact that \"\u0441\u043e\u0431\u0440\u0430\u043b\u0441\u044f\" is past tense perfective. If one makes the decision to use \u0441\u043e\u0431\u0440\u0430\u0442\u044c\u0441\u044f, why not use a future tense form of it? But, as you can see from the Google Ngram below, \"\u041a\u0443\u0434\u0430 \u0442\u044b \u0441\u043e\u0431\u0435\u0440\u0451\u0448\u044c\u0441\u044f?\" doesn't even make the cut:\n\nI am also curious about the frequency of past tense perfective verbs being translated as present tense verbs in English. Does this happen with great frequency? Are there certain types of verbs that tend to do this? Is \"\u0441\u043e\u0431\u0438\u0440\u0430\u0442\u044c\u0441\u044f\" one of them, and, if so, does this group of verbs have a category with a name? And, finally, is there a particular name for when a verb is past tense in the original language, but present tense in the translated language?\nIf you want more context, this whole inquiry came about after a learning session in which one of my Russian tutors on iTalki is helping me understand the dialog in this short film:\n\u041d\u043e\u0447\u044c\nThe phrase in question is found just shortly after minute 1:20, and conjugated for a female obviously (\u0441\u043e\u0431\u0440\u0430\u043b\u0430\u0441\u044c vs. \u0441\u043e\u0431\u0440\u0430\u043b\u0441\u044f).\nI didn't realize I had so many questions about this phrase until after the session ended, but that's okay because somebody else might be curious about this, too.\n", "answer": "This depends on context \u2014 particularly whether the question is posed while the person addressed is preparing to leave, or in the process of actually leaving.\n\nWhile preparing to leave (e.g. ironing their clothes):\n\nK\u0443\u0434\u0430 \u0442\u044b \u0438\u0434\u0451\u0448\u044c? - Appropriate;\n\u041a\u0443\u0434\u0430 \u0442\u044b \u0441\u043e\u0431\u0440\u0430\u043b\u0441\u044f? - Not appropriate because preparations to leave are not yet concluded;\n\u041a\u0443\u0434\u0430 \u0442\u044b \u0441\u043e\u0431\u0438\u0440\u0430\u0435\u0448\u044c\u0441\u044f? (Where are you preparing to go?) - Appropriate, because the question is about an ongoing process;\n\n\nAfter preparations are concluded, but before starting off (e.g. the person addressed has put on new clothes, but hasn't walked out the door):\n\nK\u0443\u0434\u0430 \u0442\u044b \u0438\u0434\u0451\u0448\u044c? - Appropriate;\n\u041a\u0443\u0434\u0430 \u0442\u044b \u0441\u043e\u0431\u0440\u0430\u043b\u0441\u044f? - Appropriate. It may imply disapproval \u2014 particularly if there were no visible preparations.\n\n\nWhen someone is in the process of leaving (e.g. on the street):\n\nK\u0443\u0434\u0430 \u0442\u044b \u0438\u0434\u0451\u0448\u044c? - Appropriate;\n\u041a\u0443\u0434\u0430 \u0442\u044b \u0441\u043e\u0431\u0440\u0430\u043b\u0441\u044f? - Appropriate; The crux here is that someone's appearance and/or behavior is unusual and suggests some special occasion.\n\n\n\n", "topic": "russian", "url": "https://russian.stackexchange.com/questions/25003"}, {"image": "VQAonline_00057387.png", "question": "Meaning of the word \u0441\u0430\u0444\u0438\u0440(\u044a) in this passage?", "context": "I want to find out what the word \u0441\u0430\u0444\u0438\u0440\u044a means in this passage from an old textbook. I cannot find the word in any dictionary. I think it might mean \"ambassador\" but am not sure.\n\n", "answer": "\u0421\u0410\u0424\u0418\u0420 \u041c\u043e\u0440\u0438\u0446 \u0413\u043e\u0442\u043b\u0438\u0431 (Moritz Gottlieb Saphir, 1795-1858) - \u043d\u0435\u043c\u0435\u0446\u043a\u0438\u0439 \u044e\u043c\u043e\u0440\u0438\u0441\u0442 \u0438 \u0436\u0443\u0440\u043d\u0430\u043b\u0438\u0441\u0442. \u0412\u043f\u0435\u0440\u0432\u044b\u0435 \u043e\u0431\u0440\u0430\u0442\u0438\u043b \u043d\u0430 \u0441\u0435\u0431\u044f \u0432\u043d\u0438\u043c\u0430\u043d\u0438\u0435 \u0441\u0431\u043e\u0440\u043d\u0438\u043a\u043e\u043c \u0441\u0442\u0438\u0445\u043e\u0432 \u00abPoetische Erstlinge\u00bb (1821). \u0418\u0437\u0434\u0430\u0432\u0430\u043b \u0442\u0435\u0430\u0442\u0440\u0430\u043b\u044c\u043d\u044b\u0435 \u0438 \u044e\u043c\u043e\u0440\u0438\u0441\u0442\u0438\u0447\u0435\u0441\u043a\u0438\u0435 \u0436\u0443\u0440\u043d\u0430\u043b\u044b, \u043f\u043e\u043b\u044c\u0437\u043e\u0432\u0430\u0432\u0448\u0438\u0435\u0441\u044f \u0431\u043e\u043b\u044c\u0448\u0438\u043c \u0443\u0441\u043f\u0435\u0445\u043e\u043c. \u0412 \u0411\u0435\u0440\u043b\u0438\u043d\u0435 \u0438\u0437\u0434\u0430\u043d \u0431\u044b\u043b \u043f\u0440\u043e\u0442\u0438\u0432 \u0421. \u043f\u0430\u043c\u0444\u043b\u0435\u0442, \u043d\u0430 \u043a-\u0440\u044b\u0439 \u043e\u043d \u043e\u0442\u0432\u0435\u0442\u0438\u043b \u0431\u0440\u043e\u0448\u044e\u0440\u043e\u0439, \u0432\u044b\u0434\u0435\u0440\u0436\u0430\u0432\u0448\u0435\u0439 4 \u0438\u0437\u0434\u0430\u043d\u0438\u044f \u0437\u0430 \u043d\u0435\u0434\u0435\u043b\u044e: \u00abDer getodtete und dennoch lebende Saphir\u00bb (\u0423\u0431\u0438\u0442\u044b\u0439, \u043d\u043e \u0442\u0435\u043c \u043d\u0435 \u043c\u0435\u043d\u0435\u0435 \u0436\u0438\u0432\u043e\u0439 \u0421\u0430\u0444\u0438\u0440, 1828). \u041b\u0438\u0442\u0435\u0440\u0430\u0442\u0443\u0440\u043d\u0430\u044f \u043f\u0440\u043e\u0438\u0437\u0432\u043e\u0434\u0438\u0442\u0435\u043b\u044c\u043d\u043e\u0441\u0442\u044c \u0421. \u0432\u0435\u0441\u044c\u043c\u0430 \u0432\u0435\u043b\u0438\u043a\u0430. \u0412 1884 \u0431\u044b\u043b\u043e \u0438\u0437\u0434\u0430\u043d\u043e \u0441\u043e\u0431\u0440\u0430\u043d\u0438\u0435 \u0435\u0433\u043e \u0438\u0437\u0431\u0440\u0430\u043d\u043d\u044b\u0445 \u0441\u043e\u0447\u0438\u043d\u0435\u043d\u0438\u0439 \u0432 12\u0442\u0442., \u0432 1890 - \u0441\u043e\u0447\u0438\u043d\u0435\u043d\u0438\u044f \u0432 26\u0442\u0442. \u041d\u0430\u0438\u0431\u043e\u043b\u044c\u0448\u0438\u043c \u0443\u0441\u043f\u0435\u0445\u043e\u043c \u043f\u043e\u043b\u044c\u0437\u043e\u0432\u0430\u043b\u0438\u0441\u044c \u00abFliegendes Album fur Ernst, Scherz, Humor...\u00bb (\u041b\u0435\u0442\u0443\u0447\u0438\u0439 \u0430\u043b\u044c\u0431\u043e\u043c \u0441\u0435\u0440\u044c\u0435\u0437\u043d\u043e\u0433\u043e, \u0448\u0443\u0442\u043e\u043a, \u044e\u043c\u043e\u0440\u0430 \u0438 \u0436\u0438\u0437\u043d\u0435\u0440\u0430\u0434\u043e\u0441\u0442\u043d\u043e\u0441\u0442\u0438, 1846), \u00abConversations-Lexikon fur Geistwitz und Humor\u00bb \u0421\u043b\u043e\u0432\u0430\u0440\u044c \u043e\u0441\u0442\u0440\u043e\u0443\u043c\u0438\u044f \u0438 \u044e\u043c\u043e\u0440\u0430, 1851-1852, 2 Aufl., 1860, 3 Aufl., 1893), \u0430 \u0442\u0430\u043a\u0436\u0435 \u00abHumoristische Abende\u00bb (\u042e\u043c\u043e\u0440\u0438\u0441\u0442\u0438\u0447\u0435\u0441\u043a\u0438\u0435 \u0432\u0435\u0447\u0435\u0440\u0430, 1830), \u00abDumme Briefe\u00bb (\u0413\u043b\u0443\u043f\u044b\u0435 \u043f\u0438\u0441\u044c\u043c\u0430, 1834) \u0438 \u0434\u0440.\n", "topic": "russian", "url": "https://russian.stackexchange.com/questions/12179"}, {"image": "VQAonline_00057388.png", "question": "\u041a\u0430\u043a \u043d\u0430\u0437\u044b\u0432\u0430\u0435\u0442\u0441\u044f \u0443\u0434\u043b\u0438\u043d\u0438\u0442\u0435\u043b\u044c, \u0441\u043d\u0430\u0431\u0436\u0451\u043d\u043d\u044b\u0439 \u0434\u0432\u0443\u043c\u044f \u0438\u043b\u0438 \u0431\u043e\u043b\u0435\u0435 \u0440\u043e\u0437\u0435\u0442\u043a\u0430\u043c\u0438?", "context": "\u041a\u0430\u043a \u043d\u0430\u0437\u044b\u0432\u0430\u0435\u0442\u0441\u044f \u0443\u0434\u043b\u0438\u043d\u0438\u0442\u0435\u043b\u044c, \u0441\u043d\u0430\u0431\u0436\u0451\u043d\u043d\u044b\u0439 \u0434\u0432\u0443\u043c\u044f \u0438\u043b\u0438 \u0431\u043e\u043b\u0435\u0435 \u0440\u043e\u0437\u0435\u0442\u043a\u0430\u043c\u0438?\n\n", "answer": "\u0414\u043e\u043f\u043e\u043b\u043d\u044e:\n\u041d\u0430 \u0431\u043e\u043b\u044c\u0448\u0435\u0439 \u0447\u0430\u0441\u0442\u0438 \u0442\u0435\u0440\u0440\u0438\u0442\u043e\u0440\u0438\u0438 \u0420\u043e\u0441\u0441\u0438\u0438 \u044d\u0442\u0443 \u0448\u0442\u0443\u043a\u0443 \u043d\u0430\u0437\u044b\u0432\u0430\u044e\u0442 \"\u0443\u0434\u043b\u0438\u043d\u0438\u0442\u0435\u043b\u044c\".\n\u041f\u0440\u0438 \u044d\u0442\u043e\u043c \u0441\u0440\u0435\u0434\u0438 \u043f\u043e\u0436\u0438\u043b\u043e\u0433\u043e \u043f\u043e\u043a\u043e\u043b\u0435\u043d\u0438\u044f, \u0430 \u0442\u0430\u043a \u0436\u0435 \u0432 \u0434\u0435\u0440\u0435\u0432\u043d\u044f\u0445 \u043c\u043e\u0436\u043d\u043e \u0432\u0441\u0442\u0440\u0435\u0442\u0438\u0442\u044c \u043d\u0430\u0437\u0432\u0430\u043d\u0438\u0435 \"\u043f\u0435\u0440\u0435\u043d\u043e\u0441\u043a\u0430\", \u043a\u043e\u0442\u043e\u0440\u043e\u0435 \u043c\u043e\u0436\u043d\u043e \u043e\u0442\u043d\u0435\u0441\u0442\u0438 \u043a \u043c\u0435\u0441\u0442\u043d\u043e\u043c\u0443 \u0434\u0438\u0430\u043b\u0435\u043a\u0442\u0443.\n\u0422\u0430\u043a \u0436\u0435, \u043f\u043e\u0436\u0438\u043b\u044b\u0435 \u043b\u044e\u0434\u0438 \u0447\u0430\u0441\u0442\u043e \u043d\u0430\u0437\u044b\u0432\u0430\u044e\u0442 \u0438\u0445 \"\u0442\u0440\u043e\u0439\u043d\u0438\u043a\" \u043d\u0435\u0437\u0430\u0432\u0438\u0441\u0438\u043c\u043e \u043e\u0442 \u0442\u043e\u0433\u043e, \u0441\u043a\u043e\u043b\u044c\u043a\u043e \u0432 \u043d\u0438\u0445 \u0440\u043e\u0437\u0435\u0442\u043e\u043a.\n\u0415\u0441\u043b\u0438 \u0432 \u043f\u0440\u0438\u0431\u043e\u0440\u0435 \u0438\u043c\u0435\u0435\u0442\u0441\u044f \u0437\u0430\u0449\u0438\u0442\u0430 \u043e\u0442 \u043f\u0435\u0440\u0435\u043f\u0430\u0434\u043e\u0432 \u043d\u0430\u043f\u0440\u044f\u0436\u0435\u043d\u0438\u044f - \u044d\u0442\u043e \"\u0441\u0435\u0442\u0435\u0432\u043e\u0439 \u0444\u0438\u043b\u044c\u0442\u0440\".\n\u041e\u0434\u043d\u0430\u043a\u043e, \u0431\u043e\u043b\u044c\u0448\u043e\u0435 \u043a\u043e\u043b\u0438\u0447\u0435\u0441\u0442\u0432\u043e \u043b\u044e\u0434\u0435\u0439 \u043d\u0430\u0437\u044b\u0432\u0430\u0435\u0442 \u0438\u0445 \"\u043f\u0438\u043b\u043e\u0442\", \u0442.\u043a. \u043d\u0430 \u043c\u043e\u043c\u0435\u043d\u0442 \u043f\u043e\u044f\u0432\u043b\u0435\u043d\u0438\u044f \u0441\u0435\u0442\u0435\u0432\u044b\u0445 \u0444\u0438\u043b\u044c\u0442\u0440\u043e\u0432 \u0432 \u0420\u043e\u0441\u0441\u0438\u0438 \u0441\u0430\u043c\u043e\u0439 \u0440\u0430\u0441\u043f\u0440\u043e\u0441\u0442\u0440\u0430\u043d\u0435\u043d\u043d\u043e\u0439 \u0444\u0438\u0440\u043c\u043e\u0439, \u0432\u044b\u043f\u0443\u0441\u043a\u0430\u044e\u0449\u0435\u0439 \u0438\u0445, \u0431\u044b\u043b\u0430 \u0444\u0438\u0440\u043c\u0430 \"PILOT\", \u0430 \u0442.\u043a. \u043e\u0441\u043e\u0431\u0435\u043d\u043d\u043e \u0441\u0442\u0430\u0440\u0448\u0435\u0435 \u043f\u043e\u043a\u043e\u043b\u0435\u043d\u0438\u0435 \u043d\u0435 \u0437\u043d\u0430\u043b\u043e \u043f\u0440\u0430\u0432\u0438\u043b\u044c\u043d\u043e\u0433\u043e \u043d\u0430\u0437\u0432\u0430\u043d\u0438\u044f - \u043f\u0440\u043e\u0441\u0442\u043e \u0447\u0438\u0442\u0430\u043b\u0438 \u0435\u0434\u0438\u043d\u0441\u0442\u0432\u0435\u043d\u043d\u043e\u0435 \u0441\u043b\u043e\u0432\u043e, \u043d\u0430\u043f\u0438\u0441\u0430\u043d\u043d\u043e\u0435 \u043d\u0430 \u0441\u0435\u0442\u0435\u0432\u043e\u043c \u0444\u0438\u043b\u044c\u0442\u0440\u0435. \u041d\u0430\u0437\u0432\u0430\u043d\u0438\u0435 \u043f\u0440\u0438\u0436\u0438\u043b\u043e\u0441\u044c, \u043d\u043e \u0441\u0435\u0439\u0447\u0430\u0441 \u0438\u0441\u043f\u043e\u043b\u044c\u0437\u0443\u0435\u0442\u0441\u044f \u0447\u0430\u0449\u0435 \u0432\u0437\u0440\u043e\u0441\u043b\u044b\u043c\u0438 \u043b\u044e\u0434\u044c\u043c\u0438.\n\u041f\u043e\u0434\u043e\u0431\u043d\u043e\u0435 (\u041a\u0430\u043a \u0441 PILOT) \u0442\u0430\u043a \u0436\u0435 \u0431\u044b\u043b\u043e \u0441 \u043a\u043e\u043f\u0438\u0440\u043e\u0432\u0430\u043b\u044c\u043d\u044b\u043c\u0438 \u0430\u043f\u043f\u0430\u0440\u0430\u0442 \u0444\u0438\u0440\u043c\u044b Xerox - \u044d\u0442\u043e \u043d\u0430\u0441\u0442\u043e\u043b\u044c\u043a\u043e \u043f\u0440\u0438\u0436\u0438\u043b\u043e\u0441\u044c, \u0447\u0442\u043e \u0441\u0435\u0439\u0447\u0430\u0441 \u043f\u0440\u0430\u043a\u0442\u0438\u0447\u0435\u0441\u043a\u0438 \u043b\u044e\u0431\u043e\u0439 \u0440\u0443\u0441\u0441\u043a\u043e\u0433\u043e\u0432\u043e\u0440\u044f\u0449\u0438\u0439 \u0447\u0435\u043b\u043e\u0432\u0435\u043a \u043d\u0430\u0437\u043e\u0432\u0435\u0442 \u043b\u044e\u0431\u043e\u0439 \u043a\u043e\u043f\u0438\u0440\u043e\u0432\u0430\u043b\u044c\u043d\u044b\u0439 \u0430\u043f\u043f\u0430\u0440\u0430\u0442 \"\u043a\u0441\u0435\u0440\u043e\u043a\u0441\", \u043d\u0435\u0437\u0430\u0432\u0438\u0441\u0438\u043c\u043e \u043e\u0442 \u0444\u0438\u0440\u043c\u044b-\u043f\u0440\u043e\u0438\u0437\u0432\u043e\u0434\u0438\u0442\u0435\u043b\u044f.\n", "topic": "russian", "url": "https://russian.stackexchange.com/questions/12671"}, {"image": "VQAonline_00057396.png", "question": "One Crayz but nice to underestand on KitKat Choclate?", "context": "anyone can help me to understand what is the translation of words in a box that I draw a circle on it in the following KitKat Picture into English? (what is the words top of 5% in English)? it's so strange for me to read it because my native is Spanish. \n\nany idea?\n", "answer": "The answer is simple;\n\u043d\u0430 \u043f\u043e\u0440\u0446\u0438\u044e 23,5 \u0433 = per portion of 23,5 g / por cada raci\u00f3n de 23,5 g;\n\u043a\u043a\u0430\u043b = kcal (a unit of energetical value of food).\nTrivia: in Ukranian (another East-Slavic language) the words kit-kat could be understood as 'cat the executioner' (gato el verdugo).\n", "topic": "russian", "url": "https://russian.stackexchange.com/questions/14310"}, {"image": "VQAonline_00057405.png", "question": "What is \"spiral binding machine\" called in Russian language?", "context": "What is \"spiral binding machine\" called in Russian language? \n*n.b. it is also known as \"comb binding machine\". \nI've been searching googling and I didn't find answer. My purpose is to find the translation in order to find one in Ukrainian web-store which sells it. \n\n", "answer": "There are 3 options:\n\n\u041f\u0435\u0440\u0435\u043f\u043b\u0435\u0442\u043d\u0430\u044f \u043c\u0430\u0448\u0438\u043d\u0430\n\u0431\u0440\u043e\u0448\u044e\u0440\u043e\u0432\u0449\u0438\u043a\n\u043f\u0435\u0440\u0435\u043f\u043b\u0435\u0442\u0447\u0438\u043a\n\n", "topic": "russian", "url": "https://russian.stackexchange.com/questions/15133"}, {"image": "VQAonline_00057380.png", "question": "English Conditionals to Russian?", "context": "Does anyone want to try to translate this table into Russian? \nIn English there are 4 conditionals (Type 0, 1, 2, 3, 4).\nI have a good idea of how these translate, but I would like to see what the great members on this stack will come up with. \n\n", "answer": "I'm not a professional linguist, so I can't provide any reference to the textbooks on this matter, but as a native speaker, my translation is below (the cells in the table correspond to the table from the question):\n+----+---------------------------------------+----------------------+\n| 0 | \u0435\u0441\u043b\u0438 \u044f \u043f\u044c\u044e \u043a\u043e\u0444\u0435 \u0432\u0435\u0447\u0435\u0440\u043e\u043c | \u044f \u043f\u043b\u043e\u0445\u043e \u0441\u043f\u043b\u044e |\n| | \u0435\u0441\u043b\u0438 \u044f \u0432\u044b\u043f\u044c\u044e \u043a\u043e\u0444\u0435 \u0432\u0435\u0447\u0435\u0440\u043e\u043c | \u044f \u043f\u043b\u043e\u0445\u043e \u0441\u043f\u043b\u044e |\n| | \u0435\u0441\u043b\u0438 \u044f \u0432\u044b\u043f\u0438\u0432\u0430\u044e \u043a\u043e\u0444\u0435 \u0432\u0435\u0447\u0435\u0440\u043e\u043c | \u044f \u043f\u043b\u043e\u0445\u043e \u0441\u043f\u043b\u044e |\n| | \u0435\u0441\u043b\u0438 \u044f \u0432\u044b\u043f\u0438\u043b \u043a\u043e\u0444\u0435 \u0432\u0435\u0447\u0435\u0440\u043e\u043c | \u044f \u043f\u043b\u043e\u0445\u043e \u0441\u043f\u043b\u044e |\n+----+---------------------------------------+----------------------+\n| 1 | \u0435\u0441\u043b\u0438 \u044f \u0432\u044b\u043f\u044c\u044e \u043a\u043e\u0444\u0435 \u0441\u0435\u0433\u043e\u0434\u043d\u044f \u0432\u0435\u0447\u0435\u0440\u043e\u043c | \u044f \u0431\u0443\u0434\u0443 \u043f\u043b\u043e\u0445\u043e \u0441\u043f\u0430\u0442\u044c |\n+----+---------------------------------------+----------------------+\n| 2 | \u0435\u0441\u043b\u0438 \u044f \u0431\u044b \u0432\u044b\u043f\u0438\u043b \u043a\u043e\u0444\u0435 \u0441\u0435\u0433\u043e\u0434\u043d\u044f \u0432\u0435\u0447\u0435\u0440\u043e\u043c | \u044f \u0431\u044b \u043f\u043b\u043e\u0445\u043e \u0441\u043f\u0430\u043b |\n+----+---------------------------------------+----------------------+\n| 3 | \u0435\u0441\u043b\u0438 \u044f \u0431\u044b \u0432\u044b\u043f\u0438\u043b \u043a\u043e\u0444\u0435 \u0432 \u043f\u0440\u043e\u0448\u043b\u044b\u0439 \u0432\u0435\u0447\u0435\u0440 | \u044f \u0431\u044b \u043f\u043b\u043e\u0445\u043e \u0441\u043f\u0430\u043b |\n+----+---------------------------------------+----------------------+\n\nAs far as I can see from this table, with imaginary situations one always uses the particle \"\u0431\u044b\" with a verb in the past.\nAlso imaginary situations in the future and present differ from ones in the past only by a verb in the if clause, that is only by the tense of a verb in this clause.\n", "topic": "russian", "url": "https://russian.stackexchange.com/questions/9506"}, {"image": "VQAonline_00057401.png", "question": "Meaning of \"\u0432\u044b\u0434\u0430\u0432\u0438\u0442\u044c \u043d\u0430 \u043c\u043e\u0440\u043e\u0437\"?", "context": "I have seen somewhere they use it not in terms you could think you would do with tooth paste.. or in Siberia, related to a physiological process..\nNo, the example is:\n\n\"\u0410 \u043a\u043e\u043d\u043a\u0443\u0440\u0435\u043d\u0442\u043e\u0432 \u0432\u044b\u0434\u0430\u0432\u044f\u0442 \u043d\u0430 \u043c\u043e\u0440\u043e\u0437\"?\n\nA Russian mafia way of doing business?\n\nhttp://fritzmorgen.livejournal.com/1065051.html#cutid1\nI must admit there is just one place so far where it has been used, so metaphor or a typo?\n", "answer": "Rather than a typo, I would call it a mixed metaphor - squeezing used as a metaphor for applying whatever tactics to reduce your competitors' ability to compete, and the frost part implying that these competitors won't like the situation they'll end up in. \n", "topic": "russian", "url": "https://russian.stackexchange.com/questions/14720"}, {"image": "VQAonline_00057608.png", "question": "Scaling/Performance of Matlab's svds function (Lanczos bidiagonalization)", "context": "I have a simple Matlab script which aims to compute $k$ singular values of a matrix $A$. $A$ is a random dense square matrix of size $5000\\times5000$, with 100 of its singular values constrained to be 0 (though that last detail does not seem to matter for my question).\nI'm doing this in Matlab via [Uk, Sk, Vk] = svds(A, k);. According to the documentation, svds uses Lanczos bidiagonalization to compute these values. I looked at the function definition (edit svds) and do not see any relevant branching, e.g. using different algorithms under the hood based on different conditions. However, when I increase $k$, I get very curious scaling/performance:\n\nThe docs mention\n\nIncreasing k can sometimes improve performance, especially when the matrix has repeated singular values.\n\nBut I interpret this to mean performance would be improved per $k$, rather than some huge reduction in total overall runtime.\nIs this a known behavior of Lanczos bidiagonalization (an algorithm I'm not very familiar with)? Or does anyone have any speculations as to why the performance of svds is like this?\nEdit: Here is a minimal version of my script so others can try to reproduce:\nresults = [];\nA = rand(5000, 5000);\n[U, S, V] = svd(A);\ndS = diag(S);\ndS(4900:5000) = 0;\nA = U*diag(dS)*V;\nb = rand(5000, 1);\nfor k = 100 : 100 : 4500\n tic\n [Uk, Sk, Vk] = svds(A,k);\n Ahat = Vk*diag(1./diag(Sk))*Uk';\n test = Ahat * b;\n time_k = toc\n results = [results; k time_k];\nend\nplot(results(:,1), results(:,2))\n\n", "answer": "I was able to reproduce your initial result via the snippet. However, by adding some more options to your svd call:\n[Uk, Sk, Vk] = svds(A,k,'largest','display',true);\nwe see that the algorithm indeed changes to a dense one (@ThijsSteel).\nFor 300 singular values:\n=== Singular value decomposition A*v = sigma*u, A'*u = sigma*v ===\n\nComputing 300 largest singular values of 5000-by-5000 matrix A.\n\nParameters:\n Maximum number of iterations: 100\n Tolerance: 1e-10\n Subspace Dimension: 900\n\nFind largest singular values for A*v = sigma*u, A'*u = sigma * v.\n\n--- Start of Lanczos bidiagonalization method ---\nIteration 1: 144 of 300 singular values converged. Smallest non-converged residual 1.4e-09 (tolerance 1.0e-10).\nIteration 2: 300 of 300 singular values converged.\n---\nTo check if singular value multiplicities were missed, restart the method, looking for k+1 singular values.\n---\nIteration 3: 301 of 301 singular values converged.\n---\nNo additional multiple singular values found. Successful return.\n---\n\ntime_k =\n\n 55.5512\n\nFor 2300 singular values:\n=== Singular value decomposition A*v = sigma*u, A'*u = sigma*v ===\n\nComputing 2300 largest singular values of 5000-by-5000 matrix A.\n\nParameters:\n Maximum number of iterations: 100\n Tolerance: 1e-10\n Subspace Dimension: 6900\n\nCompute SVDS by calling SVD, because the subspace dimension is equal to the minimum matrix size.\n\ntime_k =\n\n 45.7203\n\n", "topic": "scicomp", "url": "https://scicomp.stackexchange.com/questions/35670"}, {"image": "VQAonline_00057617.png", "question": "How to demonstrate the order of convergence of FTBS method for solving a hyperbolic PDE", "context": "consider the Purely hyperbolic model problem\n$$u_t+au_x=0$$\n$$u(-1,t)=u(1,t) \\text{ (periodic boundary)}$$\n$$u(x,0)=f(x)$$\nwith $f(y)=\\sin(2\\pi y)$. Furthermore the exact solution is given by $u(x,t)=f(x-at)$. I have implemented Forward\u2013time backward\u2013space (see page 1: http://www.etakl.net/notes_etc/numerical/schemes.pdf ) on a uniform grid in time and space and the figure demonstrates my results with certain values of dt, dx and a.\nMY QUIESTION: How can I construct a test/example to demonstrate the order of convergence? In theory the order of convergance should be 1 in time and 1 in space, right?\n\nI am using the FDM book by LeVeque and this problem is from chapter 10.\nI am using Python and I will be happy to share my code if needed. Thanks\n", "answer": "You could refine your discretization and then compare the logarithm of the error ($\\log |e|$) with the logarithm of the size ($\\log h$) of your elements. Using a linear regression you could obtain an approximation of the order of convergence. Keep in mind that this order of convergence is asymptotic when $h \\rightarrow 0$, so, for \"large\" $h$ you could see a different behavior.\n", "topic": "scicomp", "url": "https://scicomp.stackexchange.com/questions/37318"}, {"image": "VQAonline_00057584.png", "question": "Finding curves where function goes to zero in two dimensions", "context": "Suppose $f(x,y)$ is a complex function of two real arguments with roots* that are not discrete points but lie in curves. (Is there are term for this characteristic?) An example is shown below: the black curves show all the points in the $(x, y)$ plane where $f(x, y) = 0$. What is the best way to find these curves numerically, within a rectangular region?\nThe obvious solution is to consider 1-D slices along the $x$- or $y$-axis, and use standard 1-D root-finding algorithms to find the discrete roots along these slices. These points can then be joined up appropriately to form the curves. However, I wonder if there is a more efficient strategy, taking into account 2-D information.\nAnswers can assume some of the properties shown in the example plot. The curves do not terminate within the rectangular region, they do not intersect, and they always have a negative gradient.\n*Definition of root: a point $(x_r, y_r)$ such that $f(x_r, y_r) = 0$.\n\n", "answer": "This is a technique called continuation. It typically works by using Newton's method to find one root, then you take steps along the root curve by picking a nearby point as the initial guess for the next Newton step. This is usually done to solve problems of the form $$F_\\lambda(x_\n\\lambda)=0$$ where $\\lambda$ is some parameter, but your problem can be recast into this form. \nIf done correctly, this should get what you need provided that 2 root curves don't intersect each other and the root curves don't suddenly stop. An easy method to code yourself provided you have a good Newton solver is pseudo-arclength continuation that essentially tries to progress along the root curve via an estimation of the arclength along the curve, and enforces this condition by adding an extra equation to the Newton problem. In this formulation, the derivatives can be approximated by a forward difference and the \"forward\" values of $u$ and $\\lambda$ are the unknowns for the new system.\n", "topic": "scicomp", "url": "https://scicomp.stackexchange.com/questions/33290"}, {"image": "VQAonline_00057594.png", "question": "Estimation of viscosity from critical properties", "context": "\nThe above graph represents reduced viscosity as a function of reduced temperature for several values of the reduced pressure. \n\nI am writing a code which will estimate the viscosity, in the following steps :\n\nCalculating critical viscosity($\\mu_c$) by using the formula, \n\n$$ \\mu_c = 7.70M^{0.5}p_c^{2/3}T_c^{-1/6} $$\n\nCalculating reduced temperature and pressure as,\n\n$$ \nT_r = \\frac{T}{T_c} \\\\ \np_r = \\frac{p}{p_c} $$\nNow, from the graph at the top, estimating $\\mu_r$ by using $T_r$ and $p_r$ values calculated from the previous step.\n\nFinally, calculating the predicted value of $\\mu$ as,\n\n$$ \\mu = \\mu_r\\mu_c $$\n(this value of $\\mu$ is unusually a good agreement with the measured value)\n\nQuestion: How do I feed/extract the data/equation of the plot (top) which is experimentally generated so that I can also plot/test it in my code?\nP.S - All other parameters $T_c$ , $p_c$ , $T$ , $p$, $M$ will be input by the user\n\nREFERED : O. A. Uyehara and K. M. Wastson, Nat. Petroleum News, Tech. Section, 36, 764(Oct. 4, 1944); revised | Transport Phenomena, 2nd edition, R. Byron Bird, Warren E. Stewart, Edwin N. Lightfoot\n", "answer": "You can use Plot Digitizer and extract the data points in your image graph as a xml file and then you can parse it by using Python. It's pretty straightforward. You need to import the image of your graph into the software. Then calibrate the X and Y axes by specifying the $x_{min}$, $x_{max}$, $y_{min}$, and $y_{max}$ which are the min and max of X and Y axes. Note that if X or Y axis is in logarithmic scale, you can specify it in the software during calibration. Finally you just manually touch the points in your graph and software would save the X and Y of points for you and you can easily import it as xml file and you can parse this xml file in Python or even in Microsoft Excel or LibreOffice.\n", "topic": "scicomp", "url": "https://scicomp.stackexchange.com/questions/34024"}, {"image": "VQAonline_00057591.png", "question": "Defining Current Density in a FEM model (MATLAB)", "context": "I'm attempting to solve the Poisson equation in 3D for a magnetic vector potential in the presence of a current source. To validate my code, I'm initially looking to reproduce the model described in the following pair of papers:\n\n\nN. Demerdash, T. Nehl and F. Fouad, \"Finite element formulation and analysis of three dimensional magnetic field problems,\" in IEEE Transactions on Magnetics, vol. 16, no. 5, pp. 1092-1094, September 1980.\n doi: 10.1109/TMAG.1980.1060817\n URL: http://ieeexplore.ieee.org/stamp/stamp.jsp?tp=&arnumber=1060817&isnumber=22843\nN. A. Demerdash, F. A. Fouad, T. W. Nehl and O. A. Mohammed, \"Three Dimensional Finite Element Vector Potential Formulation of Magnetic Fields in Electrical Apparatus,\" in IEEE Transactions on Power Apparatus and Systems, vol. PAS-100, no. 8, pp. 4104-4111, Aug. 1981. doi: 10.1109/TPAS.1981.317005. URL: http://ieeexplore.ieee.org/stamp/stamp.jsp?tp=&arnumber=4111101&isnumber=4111054\n\n\nThe paper gives the following figure to describe the coil which carries the current: \nAlongside the following parameters:\n\n20 Amps of current. \nWire is of type AWG #16 with 861 turns \nWinding Height (Z axis): 8.89cm \nOuter Boundary Cross Section: 15.24 cm x 15.24 cm\nInner Boundary Cross Section: 10.42 cm x 10.42 cm\n\nI also know that the current density should be homogeneous in a specific element. In this model Jz = 0 throughout and I know J = N*I / A, where A is the cross sectional area, N the number of turns and I the current.\n", "answer": "Defining current density in this system can be done by considering the average current density within the winding region, $J_{0}$ (then subdividing this into elements). For winding height h, Inner cross section $L_{1}$, Outer cross section $L_{2}$ with N turns carrying current I:\n$ J_{0} = \\frac{IN}{h(L_{2} - L_{1})/2} $.\nI note this expression does not incorporate the specific wire properties which are also specified in the paper - this maybe to allow physical reconstruction of the system rather than for the modelling side. \n", "topic": "scicomp", "url": "https://scicomp.stackexchange.com/questions/33835"}, {"image": "VQAonline_00057446.png", "question": "Finding which triangles points are in", "context": "Suppose I have a 2D mesh consisting of nonoverlapping triangles $\\{T_k\\}_{k=1}^N$, and a set of points $\\{p_i\\}_{i=1}^M \\subset \\cup_{k=1}^N T_K$. What is the best way to determine which triangle each of the points lies in? \nFor example, in the following image we have $p_1 \\in T_2$, $p_2 \\in T_4$, $p_3 \\in T_2$, so I would like a function $f$ that returns the list $f(p_1,p_2,p_3)=[2,4,2]$.\n \nMatlab has the function pointlocation which does what I want for Delaunay meshes, but it fails for general meshes.\nMy first (dumb) thought is, for all nodes $p_i$, loop through all the triangles to find out which triangle $p_i$ is in. However, this is is extremely inefficient - you might have to loop through every triangle for every point, so it could take $O(N \\cdot M)$ work.\nMy next thought is, for all points $p_i$, find the nearest mesh node via nearest-neighbor search, then look through triangles attached to that nearest node. In this case, the work would be $O(a\\cdot M\\cdot log(N))$, where $a$ is the maximum number of triangles attached to any node in the mesh. There are a couple solvable but annoying issues with this approach,\n\nIt requires implementing an efficient nearest-neighbor search (or finding a library that has it), which could be a nontrivial task. \nIt requires storing a list of which triangles are attached to each node, which my code is currently not set up for - right now there is just a list of node coordinates and a list of elements.\n\nAltogether it seems inelegant, and I think there should be a better way. This must be a problem that arises a lot, so I was wondering if anyone could recommend the best way to approach finding what triangles the nodes are in, either theoretically or in terms of available libraries. \nThanks!\n", "answer": "The usual randomized edge hopping method should work. Basically, start with any triangle of the mesh, then determine which of the edges the target point lies on the opposite side of. That is, determine which of the edges, when extended out to a line, separate the point from the interior of the triangle. When there are two possibilities, choose one at random, and consider the triangle that is adjacent to that shared edge, and repeat. The randomization should make this method converge with probability 1 for Delaunay triangulations, and I can't think of a reason it would not work for arbitrary triangulations.\nEdit: I should add that edge hopping should be $O(\\log N)$ with a reasonable constant for a single point, so it would be $O(M \\log N)$ for $M$ points. However, if you sort your points by locality (like using a Hilbert curve ordering first), you can initialize each new query with the triangle of the previous query, to further reduce the runtime (I'm not a CS theorist so I can't tell you what the big-O would be there).\nEdit2: Found this PDF describing such a \"walking\" scheme that is guaranteed to terminate, and reviews the more naive approaches.\nAnother alternative to using quadtrees is using a Triangulation Hierarchy. See\nOlivier Devillers. Improved incremental randomized Delaunay triangulation. In Proc. 14th Annu. ACM Sympos. Comput. Geom., pages 106-115, 1998.\nIt works best for Delaunay triangulations, but can also work for non-Delaunay.\nBasically whatever you do to speed up point location will require the construction of an auxiliary data structure. In the case of quadtrees or some other spatial subdivision, you need to build the subdivision tree. In the case of edge-hopping, you need to build the triangle adjacent topological structure. The triangulation hierarchy also requires building a tree of coarser triangulations.\n", "topic": "scicomp", "url": "https://scicomp.stackexchange.com/questions/2624"}, {"image": "VQAonline_00057609.png", "question": "Solving the heat diffusion equation with source term", "context": "I am trying to solve the 1-D heat equation numerically with a variable source term. The system is basically a tank containing styrene in which it polymerizes to liberate heat. I have assumed that the cross-section of the tank is much smaller compared to the length so any variations in the y and z directions can be neglected. I am starting with $T_0 = 313.15 \\text{K}$. The domain space I am looking at is $0 \\leq x\\leq 5\\text{m}$ and $t \\geq 0$. The equations are as follows. I write the heat equation as-\n$$\\frac{\\partial T}{\\partial t} = \\frac{k}{\\rho C_p}\\frac{\\partial^2 T}{\\partial x^2} + \\frac{1}{\\rho C_p}\\left(\\frac{\\rho\\Delta H}{MW}\\frac{\\partial m}{\\partial t}\\right)$$\nThe $\\frac{\\partial m}{\\partial t}$ is basically the rate of reaction expressed in terms of the mole fraction $m$. The rate of reaction is-\n$$\\frac{\\partial m}{\\partial t} = -A\\left(\\frac{\\rho}{MW}\\right)^{\\frac{3}{2}}m^{\\frac{5}{2}}$$\nHere:\n$$k = 0.03 \\hspace{8pt} \\text{[cal/(s-m-K)]}$$\n$$A = A_0\\exp(A_1m_p + A_2m_p^2 + A_3m_p^3)$$\nwhere:\n$$m_p = 1-m$$\n$$A_0 = 1.964\\times10^5\\exp\\left(\\frac{-10,040}{T}\\right)$$ $$A_1 = 2.57-5.05\\times10^{-3}T$$ $$A_2 = 9.56-1.76\\times10^{-2}T$$ $$A_3 = -3.03+7.85\\times10^{-3}T$$\nThe rest of the values are constants and they are-\n$$\\rho = 906 \\hspace{6pt} \\text {[g/L]}$$ $$MW = 104.15 \\hspace{6pt} \\text{[g/mol]}$$ $$C_p = 0.4365 \\hspace{6pt} \\text{[cal/g-K]}$$ $$\\Delta H = -17,300 \\hspace{6pt}\\text{[cal/mole]}$$\nThe initial conditions are-\n$$T = T_0$$ $$m = 1$$\nThe boundary conditions are-\n$@ x= 0$ $$ \\frac{\\partial T}{\\partial x} = 0, \\hspace{5pt} \\frac{\\partial m}{\\partial x} = 0$$\n$@ x= 5m$ $$ \\frac{\\partial T}{\\partial x} = 0, \\hspace{5pt} \\frac{\\partial m}{\\partial x} = 0$$\nI am assuming that there is no flux at either boundary. I used the 'pdepe' command in MATLAB to solve this system of coupled PDEs. Here's the code I wrote-\nclc\nclear all\nglobal rho Cp MW dH k\ntspan = 60*(1:500:30000);\nx = 0:0.25:5;\nm = 0;\nsol = pdepe(m, @pdefun, @pdeic, @pdebc, x, tspan);\n\nu1 = sol(:,:,1);\nu2 = sol(:,:,2);\nsurf(x,tspan,u1)\ntitle('u_1(x,t)')\nxlabel('Distance x')\nylabel('Time t')\n\nfunction [c,f,s] = pdefun(x,t,u,dudx)\n rho = 906;\n Cp = 0.4365;\n MW = 104.15;\n dH = -17800;\n k = 0.03;\n \n y2 = 1-u(2);\n A0 = 1.964*(10^5)*exp(-10040/u(1));\n A1 = 2.57-5.05*u(1)*(10^(-3));\n A2 = 9.56-1.76*u(1)*(10^(-2));\n A3 = -3.03+7.85*u(1)*(10^(-3));\n A = A0*exp(A1*(y2) + A2*(y2^2) + A3*(y2^3));\n \n F = -A*((rho/MW)^(3/2))*((u(2))^(5/2));\n \n c = [1; 1];\n f = [k*rho/Cp; 0].*dudx;\n s = [(dH/(MW*Cp))*F; F];\nend\n\nfunction u0 = pdeic(x)\n u0 = [313.15; 1];\nend\n\nfunction [pl,ql,pr,qr] = pdebc(xl,ul,xr,ur,t)\n pl = [0; 0];\n ql = [1; 1];\n pr = [0; 0];\n qr = [1; 1];\nend\n\nThe problem is that on drawing the surface plot of the temperature [u(1)], I don't see any variation with the length of the tank. This is what the surface plot looks like-\n\nAnother funny thing is that even if I change the function 'f' in 'pdefun' to garbage values like f = [65675; 767].*dudx (these numbers are meaningless, I just typed out something randomly), I get the same plot regardless of the function f. I even cleared all variables before running the code but the same thing happens. I don't understand what's going on here. Any help is appreciated.\n", "answer": "You are starting from a uniform temperature and you have insulated\nboundary conditions; so there is no heat conduction occurring.\nLikewise, your initial mole fraction is also constant in $x$ so that\nthe heat input is uniform along the length. So the fact that the\ntemperature and mole fraction don't change as a function of $x$ is\nexactly what you should expect.\nAnother way of looking at this is: As you have defined it, this problem\ncould be modeled as two coupled ODE with time as the only independent variable.\n", "topic": "scicomp", "url": "https://scicomp.stackexchange.com/questions/35729"}, {"image": "VQAonline_00058068.png", "question": "Why were his vocal cords removed in Marvel's Agent Carter?", "context": "In the pilot episode of Marvel's Agent Carter, a man with his vocal cords removed was shown with a scar on his neck. Why were the vocal cords removed and why was the scar kept in such a way?\n\n", "answer": "Since the show is still in its infancy, this question is impossible to answer at the moment. However, if I were to speculate, it may just be a way of showing how ruthless both Leviathan AND its operatives are. The group is so dedicated to keeping its secrets that it will remove the vocal cords of its own operatives. AND the operatives are so loyal to the ideals of Leviathan that they're willing to sacrifice their own vocal cords.\nI may be wrong, given that a certain character without vocal cords seems to have changed his mind at the last moment, but that may also be the reason that Mr. Green-Suit was after him to begin with. As it is, we'll just have to wait and see.\nUPDATE: As of the show's finale, we now have an answer to this question:\n\n The operatives in question are survivors of the Battle of Finow, and were thus exposed to the \"Midnight Oil\" gas accidentally invented by Howard Stark.\n\n", "topic": "scifi", "url": "https://scifi.stackexchange.com/questions/78835"}, {"image": "VQAonline_00057974.png", "question": "What is the significance of the Red Comet?", "context": "For a few episodes in the show, in Season 2, there was a heavy emphasis on this Red Comet that was falling from the sky. All of a sudden it was just sort of... not a thing anymore?\n\nWhat is the comet's importance and where did it suddenly disappear off to?\nOr was it really just a comet?\nNote: I have not yet read the books so if it's explained in there, I'd still like to know about it.\n", "answer": "I believe The Red Comet is what is mentioned in the prophecy of Azor Ahai\n\nThere will come a day after a long summer when the stars bleed and the cold breath of darkness falls heavy on the world. In this dread hour a warrior shall draw from the fire a burning sword. And that sword shall be Lightbringer, the Red Sword of Heroes, and he who clasps it shall be Azor Ahai come again, and the darkness shall flee before him.\n\nThe red comet -- or the bleeding star, if you will -- is the fulfilment of part of that prophecy, and it appears when Dany hatches her dragon eggs in Drogo's funeral pyre. \nThere are many theories and much speculation on this, and it is also related to the Prince that was promised, something that Rhaegar Targaryen was preoccupied with investigating.\nIt is also said\n\nWhen the red star bleeds and the darkness gathers, Azor Ahai shall be born again amidst smoke and salt to wake dragons out of stone.\n\nWe can note that Daenerys has done all of these things. Her final sacrifice is also mirrored in the story of Nissa Nissa, the wife of Azor Ahai, who had to be sacrificed to make the legendary sword Lightbringer. (Yes, Melisandre tried to create a magic sword for Stannis, which she claimed was Lightbringer) Daenerys burned her husband Drogo, and Mirri Maaz Duur.\nShe woke the dragons from stone eggs. She was reborn in the fire (smoke). Lightbringer is her dragons.\nOh, and also, (Spoiler AFFC)\n\nMaester Aemon became convinced of this after meeting Xhondo:Daenerys is the one, born amidst salt and smoke. The dragons prove it.\nXhondo's captain is Quhuru Mo, who met Daenerys and her dragons in Qarth (A Clash of Kings)\n\nQ.E.D.\n", "topic": "scifi", "url": "https://scifi.stackexchange.com/questions/53590"}, {"image": "VQAonline_00058877.png", "question": "Animation film with giant sphere-like bad \"robot\"", "context": "Asking community to help identify anime (animation film) from my childhood.\nBy the style of that animated cartoon it seems like it was from 70s or 80s, and it had Japanese picture style to it. I saw it approx. in 1994 on some local TV channel that does not exist any more. I think it called \"KS video\".\nAll I remember is that there was a boy/man hiding in some buildings or ruined sky scrapers in partially destroyed city. It was dark. He was hiding from a giant hovering robot-like thing, that looked like a giant (as tall as buildings) metal sphere with a hood-like \"head\" and no legs or arms. This robot was slowly hovering, scanning with creepy noise for any movement, and firing a destructive beam out of his \"head\". Under that hood there was no face but something glowing red or orange. If I remember correctly there was 2 of that \"robots\" slowly destroying the city.\nUnfortunately I don't remember any more details, I have seen this on TV long time ago, but I remember that it was cool and scary animation cartoon.\nI have created a rough image of how this robot looked like:\n\nClick image to enlarge.\n", "answer": "I finally found it, it seems that someone here also was looking for this cartoon. I found it in suggestion section here: Anime style cartoon - 4 characters on a planet with giant oval floating robots looking for them - 1989 or before\nCartoon is called Birth aka Planet Busters aka The World of the Talisman(1984).\n\nAqualoid was a prosperous planet, but an attack from a mysterious life force, the Inorganics, transforms it into a post-apocalyptic shell of its former self. When Nam finds a mysterious sword, he is suddenly the object of a planet-wide chase. With the Inorganics closing in, will Nam and his friends discover the secret of the sword and save their world? Or will they destroy Aqualoid in favor of a new Birth?\n\n", "topic": "scifi", "url": "https://scifi.stackexchange.com/questions/227322"}, {"image": "VQAonline_00058061.png", "question": "Unknown feature on Middle-earth's map?", "context": "On Christopher Tolkien's map of Middle-earth, one feature has always caught my eye, and to my knowledge has never been explained: The mass in the middle of the Sea of Rh\u00fbn:\n\nHas this ever been talked or written about? Is it an island? Is it a rocky outcrop? Are they supposed to be trees? Sand? Marshes? Whatever it is, it appears to be very deliberate.\nI've also noticed that many fan-made maps just ignore this feature altogether.\nWhat is this feature in the middle of the Sea of Rh\u00fbn?\n", "answer": "As far as I remember, there isn't any name for it. It is an unnamed island.\nThe LotR Wikia says the same:\n\nThe sea also contains an unnamed island that is about thirty miles along the southeastern and northwestern coasts and twenty upon the northeastern and southwestern coasts. \n\nAlso, it must have been wooded. (thainsbook).\nThere has been a similar discussion on TheTolkienForum. A user contacted The Tolkien Society with the question, and here is the answer:\n\nHello. Sorry about the delay. A staw poll revealed that in a very\n early version of the map, the Sea of Rhun had a large island in the\n middle. (Unfeasibly large, given the scale of the Sea, which is very\n big.) By the time the map hit publication, this had vanished and the\n dots appeared.\nThose who have looked carefully at the various maps point out that\n there are similar dotty effects at the Mouths of Sirion and near Balar\n (Silmarillion), and references to \"sands\" in the Sil. The dots in the\n Sea of Rhun (on scale) could quite easily represent a constellation of\n small islands, but are more likely to mean sands, swamp or sand bars.\n (Actually small islands would make better geography on that scale.)\n There appears to be no actual reference to what is going on there.\nThere is a discussion of this also at TheOneRing.net which adds some\n details worth having. I don't however understand Quickbeam's reference\n to CRT \"finding later manuscripts etc.\" as CRT drew up this map in\n co-operation with JRR while JRR was still alive and well and writing\n the books - there have AFAIK been no revisions by CRT to these maps on\n the basis of stuff he found after his father's death, when he started\n editing the unpublished mss. Indeed, it would be against his whole\n attitude of respect for his father's decisions to start \"editing\" LotR\n material subsequent to publication. I have asked QB what he means, but\n have not had a reply. As this was some time ago and you are waiting,\n this is my conclusion!\nI would say \"be careful of taking any craft with deep draft through\n these waters. You may get stuck.\"\nHelen\n\nWhich, TBH, doesn't add much more than that there is no name to the island.\nThe discussion Helen references may be worth quoting for posterity:\n\nI\u2019ve been wondering about this for years, and now have found a few\n bits of evidence in the History of Middle-earth Volume VII that gives\n us a better clue.\nMost all of the maps we see in today\u2019s editions of LOTR come from the\n hand of Christopher Tolkien; redrafted from his father\u2019s originals.\n Professor Tolkien altered and added to his maps as his work on the\n story progressed. His original working map (the \"First Map\") was a\n conglomeration of sections and glued bits of paper that he fussed with\n for many years: \"It represents an evolution, rather than a fixed state\n of the geography.\" There was a later version called the \"1943 map\"\n that Christopher created.\nComparing these two maps, Christopher points out a well-forested\n island his father drew in the middle of the Sea of Rh\u00fbnaer, which\n would later be called the Sea of Rh\u00fbn. \"The island in the Sea is\n coloured green on the First Map, and on the 1943 map is marked as\n wooded.\" That\u2019s your island right there, but in later maps it seems to\n have vanished, replaced with an array of dots.\nI don\u2019t know exactly why Christopher changed it, but I suspect that\n later manuscripts and maps he found showed the removal of the island.\n Perhaps the discrepancy caused him to leave the dots to indicate what\n was a \"questionable area.\"\n\nThis seems to be the clearest answer, if still vague.\n", "topic": "scifi", "url": "https://scifi.stackexchange.com/questions/76511"}, {"image": "VQAonline_00058046.png", "question": "Why was the episode \"Self-Help\" almost pulled?", "context": "Michael Cudlitz (Abraham) posted on Twitter that Self-Help (S05E05) was almost pulled by the network.\n\nThere have been a number of articles guessing what censors may have objected to, but they all note that they're just guesses.\nWhy did censors almost pull the episode?\n", "answer": "First, we should doubt the source, since the actors' own Twitter account can't be completely trusted (why would you pull a whole episode instead of just the controversial scene? what if it was just a promotional resource?)\nPrior to the release of the episode some sources echoed Cudlitz's account and wondered if it could be violence, sex, cannibalism or the F-word.\nEven after the release of the episode, it was unclear if or what was wrong with it. What could be \"too much\" in a show about zombies?\nSo my guess is, as this source says, that\n\nCudlitz was probably just exaggerating for the sake of a good tease\n\nThere is no official explanation from AMC, all websites you could check about the issue can't give you a plausible reason, even after the release of the episode. There is a source which stated (prior to the release) that\n\n(The) episode of The Walking Dead would have a VERY difficult getting past the censors if they follow along with the comic book series.\n\nBut that is just more guessing. The comic event they would be referring, and that supposely might be too much for the censors would be\n\n A flashback, in which Abraham\u2019s family is hiding out at a grocery store when he decides to go on a supply run. When he returns, he finds that his wife and daughter have both been brutally raped and the assailants have forced his son to watch the whole thing.\n\nBut something like that probably would have been addressed by scriptwriters or executives from AMC long before a \"censor\" would have anything to say about the matter. \n", "topic": "scifi", "url": "https://scifi.stackexchange.com/questions/72541"}, {"image": "VQAonline_00057838.png", "question": "Why is Wolverine's hand missing?", "context": "In the picture below it appears that (this version of) Wolverine has only got one hand and one set of claws, the other hand seems to be missing. \nWhat happened to his missing hand and claws? How did he lose them and how does his body react to the missing piece of Adamantium?\n\n", "answer": "It is an alternate version of Wolverine from the Age of Apocalypse storylines. He is called Weapon X in that universe. The Age of Apocalypse continuum briefly replaced the current Earth-616 reality but was later retconned and redubbed Earth-295.\n\nIn the \"Age of Apocalypse\" story arc, the Wolverine/Logan character is again a member of the X-Men, this time using the code-name Weapon X rather than Wolverine. In this reality the events which saw the character's Adamantium ripped out have never occurred, and it is Magneto that helps him to control his feral rages. Most significantly, Weapon X is missing a hand, cut off by Cyclops (who in turn is missing an eye thanks to Weapon X). The claws on this hand still exist, however, which appear by piercing through the metal stump covering the break. Consistent with the main characterisation, this version is also shown to be a loner, his back story presenting him as an unwilling recruit to the X-Men. He and Jean Grey are lovers in this reality.\n\n", "topic": "scifi", "url": "https://scifi.stackexchange.com/questions/23365"}, {"image": "VQAonline_00058250.png", "question": "Any official evidence that Rey is based on early \"Star Wars\" concept art?", "context": "In The Force Awakens, it would seem a lot of the \"new\" designs for characters and objects are, in fact, highly informed by concept art that was generated during the early stages of development on the original Star Wars. \nIn particular, Rey seems to be very similar to the concept art produced by Ralph McQuarrie during a period of time when the character that would become Luke Skywalker was going to be female: \n\nFound at: www.StarWarsUnderworld.com\nThe article cited in the link above merely speculates that these designs were used to influence the character, but I'm looking for concrete evidence that the production team involved in designing The Force Awakens purposefully used McQuarrie designs, and the Female Luke design in particular, in their concept stage. Was this explicitly mentioned anywhere in any articles or interviews?\n", "answer": "Based upon the information in The Art of The Force Awakens, I'm going to say \"not likely\". \nThe book makes it clear that a number of elements, particularly locations, were based on old art by Ralph McQuarrie, but his name is never mentioned in regards to Rey's designs in any way. \nThe only character designs mentioned that were influenced by McQuarrie's work are Lor San Tekka (his robes are influenced by a sketch of Boba Fett) and for a \"Yoda like character\" that existed in the earliest stages of the concept art phase (much of this was done before a script had even been turned in, and was just the artists kicking around ideas).\nGlyn Dillon seems to have done a fair bit of work on Rey's (known as Kira during pre-production) costume designs. Dillon makes specific mention of the goggles, but does not mention McQuarrie:\n\nThe idea was that we wouldn't be able to see who Kira was; we wouldn't know if she were a man, a woman, an alien, or something else entirely. And I thought it would be great fun to have her fashion her own goggles out of scavenged stormtrooper lenses. (page 161)\n\nIf McQuarrie's old art was an influence, and it's definitely possible since the artists were combing through the archives, it was likely something subconscious amongst the artists.\n", "topic": "scifi", "url": "https://scifi.stackexchange.com/questions/111103"}, {"image": "VQAonline_00059396.png", "question": "Do vaginal tightening creams work?", "context": "From a billboard in Ulaanbaatar, Mongolia (population 1.4 million)\n\n\nVagina Tight\n \u0437\u0430\u043b\u0443\u0443\u0436\u0443\u0443\u043b\u0430\u0445 \u0446\u043e\u0433\u0446 \u044d\u043c\u0447\u0438\u043b\u0433\u044d\u044d [Google translate from Mongolian into English: comprehensive rejuvenation treatment]\n\n(I think it is talking about creams - I might be wrong and it's talking about surgery, but the following is definitely talking about creams)\n'Feel tight and wanted again!' The disturbing 'vaginal shrinking' creams that promise to make you feel like a virgin again from the Daily Mail gives more information on vaginal tightening creams. It says that there are several of them, and that some people say they work, and some people say they don't work\n\n'Recapture your vagina's youth so you can feel tight and wanted again'\n - the slogan accompanying one of several creams currently on the market which claim to 'shrink' and 'tighten' your lady bits.\n...\n18 Again, with its acid pink branding, isn't the only cream in its\n category. None of them appear to be FDA approved, but this is not a\n legal requirement for companies selling such products.\n...\nTellingly, the product reviews on Amazon are extremely polarized, as\n with all of the other similar creams; with ratings of either five\n stars or one. 'You do become tight and stay tight. Worth the money,'\n one review reads. 'I tried it and not only did it not work whatsoever,\n but it also caused me to get a bladder infection and a yeast\n infection,' reads another.\n\nSome pages talk about whether tightening creams work or not, but they don't seem that reliable. For example Do Vagina Tightening Creams Really Work? and Here Are the Real and Ridiculous Reviews for Vaginal Shrinking Cream. One product's site has a page titled Clinical Trials, which talks about a \"Prospective, Randomized, Double-Blind, Placebo controlled Pilot Trial of the efficacy and safety of PK24\u00ae for women with vaginal wall relaxation\", but the page doesn't indicate how the control group fared.\nCan you tighten your vagina with one of these creams?\n", "answer": "The answer is \"probably not\". An in-depth medical review here has some exceedingly good points about why. To summarize the main factors: vaginal tightness is mainly controlled by deeper layers of muscle in the pelvic floor, which most topical creams will not easily penetrate to. There is also little to no evidence that the ingredients in these creams increase blood flow to the targeted muscles, or any other effects that may aid in muscle tone improvement. \nMany of the ingredients do have astringent properties, which will cause a sensation of tightness for the user, similar to mouth pucker. They are also likely to mess up the microbiome of the user, leading to additional discomfort. Since generally vaginal tightness can be inversely correlated with relaxation and arousal, being uncomfortable physically or emotionally could certainly make the orifice feel tighter, so that may be part of the explanation.\nAnother part of the issue is that many of these products' usage guidelines state that after application you also perform vaginal strengthening exercises. The cream itself is likely not contributing to the positive improvements reported by some, and instead the muscle strengthening is the actual helpful factor. \n", "topic": "skeptics", "url": "https://skeptics.stackexchange.com/questions/37388"}, {"image": "VQAonline_00059339.png", "question": "Did Einstein say this about his marriage?", "context": "I found a picture with a quote on facebook today quoting Einstein.\n\nThe quote states:\n\nWhen we first got married, we made a pact. It was this:In our life together, it was decided I would make all of the big decisions and my wife would make all of the little decisions. For fifty years, we have held true to that agreement. I believe that is the reason for the success in our marriage. However, the strange thing is that in fifty years, there hasn't been one big decision.\n\nIs there any truth to that he actually said this? (Given that he did divorce once)\n", "answer": "Very unlikely. This is sometimes attributed to Einstein on his 50th Wedding anniversary\nEinstein was married twice:\n\nThe first marriage was to Mileva Mari\u0107, from January 1903 until their divorce on 14 February 1919 - i.e. 16 years of marriage.\nThe second marriage was to Elsa L\u00f6wenthal on 2 June 1919 until her death in December 1936 - i.e. 17 years of marriage.\n\n[I used Wikipedia as a source, as I don't think these dates are in dispute.]\nHe never had a 50th wedding anniversary, and he never had an opportunity to say \"in fifty years, there hasn't been one big decision.\"\nThis doesn't prove the quote isn't a distorted version of something he did say.\n", "topic": "skeptics", "url": "https://skeptics.stackexchange.com/questions/34322"}, {"image": "VQAonline_00059456.png", "question": "Did New York Times post this Hitler apology stating Jews baited the Nazis by trying to fight back on June 15, 1934?", "context": "Post from Revolution News reads,\n\n\n\"By hating Hitler and trying to fight back, Jews are only increasing the severity of his policies against them.\n\"If Jews throughout the world try to instill into the minds of Hitler and his supporters recognition of the ideals for which the race stands, and if Jews appeal to the German sense of justice and the German national conscience, I am sure the problem will be solved more effectively and earlier than otherwise.\"\n\n", "answer": "As the source of the image says, it\u2019s from the June 15, 1934 issue of The New York Times. Here\u2019s a screenshot of the article, \"URGES GOOD WILL BY JEWS FOR NAZIS\" by Henry J. Cadbury the Professor of Biblical Literature of Bryn Mawr College. Here is the article in full, with the quote from the question being under \"Urges Appeal to Justice\":\n\nURGES GOOD WILL BY JEWS FOR NAZIS\nProf. Cadbury of Society of Friends Says It Will Gain More Than Will Hate\n\nDECRIES BOYCOTT AS 'WAR'\n\nDr. Goldenson Tells Session of Rabbis a Minority Must Not Use Tactics of Foes\n\nSpecial to The New York Times\nWernersville, PA., June 14 \u2014 Good will, not hate or reprisals, will end, or offset, the evils of the Hitler government\u2019s persecution of Jews, Professor Henry J. Cadbury, Professor of Biblical Literature at Bryn Mawr College, told the Central Conference of American Rabbis as it opened its convention here today.\nThe message as president of the conference was delivered tonight by the Rev. Dr. Samuel H. Goldenson, rabbi of Temple Emanu-El, New York.\nProfessor Cadbury is chairman of the service committee, American Society of Friends.\n\u201cOppression of Jews in German by Hitler and his Nazi forces can be ended not by hate that their victims may display, or by attempts to fight back,\u201d he said, \u201cbut by efforts to cultivate good will.\u201d\n\u201cYou can prove to your oppressors that their objectives and methods are not only wrong, but unavailing in the face of the world\u2019s protests and universal disapproval of the injustices the Hitler program entails.\nUrges Appeal to Justice\n\u201cBy hating Hitler and trying to fight back, Jews are only increasing the severity of his policies against them.\u201d\n\u201cIf Jews throughout the world try to instill into the minds of Hitler and his supporters recognition of the ideals for which the race stands, and if Jews appeal to the German sense of Justice and German national conscience, I am sure the problem will be solved more effectively and earlier than otherwise.\u201d\nProfessor Cadbury declared \u201cit is the duty of Christians the world over to help right the injustices wrought by the Nazis.\u201d\n\u201cEvery God-believing Christian, in any nation,\u201d he said, \u201cshould join in trying to atone for the wrongdoings done Jewish people, if only for the sake of their own Christian beliefs and Christian doctrines of universal goodwill and brotherhood.\u201d\nThe boycott against German, he asserted, is not an effective means of meeting the evil.\n\u201cBoycotts are simply war without bloodshed,\u201d he said, \u201cand war in any form is not they way to right the wrongs being inflicted on the Jewish people.\u201d\nGoldenson Prescribes Remedy.\nHitler and what he symbolizes are spiritually contemporary with the Inquisition, although he uses modern means to \u201cdistill his poison,\u201d Dr. Goldenson said in his message.\nBut he warned that the remedy against Hitlerism and the protection for minority groups lay not in a mere denunciation but \u201cin the improvement of social conditions and relations wherever we live.\u201d\nAs a foundation for this task he urged support for a program among Jews looking to \u201cthe revitalization of religious faith and reconsecration to ethical and spiritual ideals.\u201d\nDr. Goldenson pointed out that \u201cmen like Hitler unfortunately are always present in the world, at any rate potentially,\u201d adding:\n\u201cIt is only when their fellows are greatly disturbed, defeated and frustrated that they become a prey to demagogues and false messiahs.\u201d\nHe described the dangers inherent in the adoption by minorities of \u201cthe methods that seem to have made their enemies successful.\u201d\n\u201cShould illiberalism develop in America,\u201d he added, \u201cit would most certainly show itself the denial of the humane and democratic principles upon which our government is founded an in the substitution of ideas not unlike those that are used to explain and to justify the German persecutions and atrocities.\u201d\n\u201cRealizing then, as everyone must, that in the spread of intolerance we Jews are always the first victims, it behooves us to be especially watchful of our own conduct and not commit the folly of believing that similar illiberalism may not develop even among ourselves.\u201d\n\u201cEverywhere there is a cry for Jewish solidarity. It is unquestionably important that especially in these threatening times we should think and act together. But solidarity will not serve us, as it does not serve the world at large, unless it is the result of critical thinking and of clearly conceived and completely justified objectives.\u201d\n\u201cWhat is not quite so easy to understand is that Jewish likemindedness has always been derived from an adequate appreciation of the meaning of our history and the content of our literature. Unless we are informed upon our heritage, no Jewish standards can be developed, and when no such standards are developed, Jewish solidarity is certain to become nothing more than brute gregariousness in the hours when clouds gather and storms threaten.\u201d\n\u201cWe have already seen the signs of havoc wrought by uninformed and uncritical solidarity in our midst. In many cities our people have been called together by self-appointed leaders for the assertion of Jewish rights and for the promotion of special programs and objectives. the appeal has almost invariably been made in the name of Jewish solidarity.\u201d\n\u201cSuch an appeal, we should be reminded, is not unlike that of the superpatriots who frequently go so far as to insinuate that, if one does not heed their call, it is a certain sign of disloyalty to one\u2019s people and to one\u2019s country.\u201d\nAid Program Should Be United\nOne of the tasks which should not be clouded by differences, Dr. Goldenson added, is helping German Jews to reach a haven wherever one is offered.\nJews, Dr. Goldenson said, \u201cshould wish to give the world more than successful careers.\u201d\n\u201cOur bankers, commercial and industrial leaders, movie magnates, doctors, lawyers, scientists, and artists,\u201d he said, \u201cshould in their respective fields so conduct their affairs as to body forth some added benefaction to their neighbors.\u201d\nStressing the need for the development of spiritual values, he said he \u201cwould suggest that all our religious forces unite upon a program looking for the revival of Jewish interest and learning and for the stimulation of Jewish loyalties and idealism.\u201d\n\u201cThe Synagogue Council fo America, representing the three religious groups in our midst, may well make this program its primary business,\u201d he proceeded.\n\u201cIn recent years our people have become specialist in the art of conducting campaigns. We have managed many successful drives for relief, philanthropy and general social welfare. The time has come to use our expert knowledge as campaigners for the most vital need of our people and our day, the revitalization of religious faith and the reconsecration to ethical and spiritual ideals.\u201d\n\u201cWhy may not the week of Chanukah be used for such a campaign?\u201d\nIn line with this program, Dr. Goldenson urged also the establishment of a weekly paper \u201cwhose sole purpose should be the dissemination of the knowledge of Judaism.\u201d\nHe stressed the importance of revising the liturgy.\nCalls for Adult Education\nRabbi Abraham J. Feldman of Hartford, Conn., said that adults as well as children need education. As chairman of the educational committee he said religious education alone is not sufficient.\n\u201cParents as well as their children must be urged to educate themselves, broaden their vision, and enlarge their objectives in life,\u201d he declared.\nSpeaking of adult Jewish education, Dr. Leon Fram of Detroit urged through study of Jewish history and exhorted Jewish parents to set an example to their children in seeking higher education and in developing Jewish cultural aims and ideals.\nSuch a course, he said, will increase respect for Jewish schools. Twenty-five large schools in various cities are now giving advanced instruction to over 5,000 adults he reported.\nRabbi Morton M. Berman, director of Jewish education of the Free Synagogues of America, declared Judaism is something more than a religion.\n\u201cAcceptance of this view ought to bring about a revolution in studies and in methods in Jewish schools,\u201d he proceeded.\n\u201cPalestine is the centre of Jewish civilization, not merely the remote ancestral home of the Jew. Jewish children should be taught more about Jews, their history and their religious and social tenets.\u201d\n\n(If you prefer to look at images of the full original article, the quote starts at the bottom left of this first image and continues at the top left of the second.)\n", "topic": "skeptics", "url": "https://skeptics.stackexchange.com/questions/41580"}, {"image": "VQAonline_00059424.png", "question": "Did Morgan Freeman say \"Oppression is being told you can't say certain words because you will offend someone else\"?", "context": "\nA friend sent me an image of Morgan Freeman with the following phrase as subtitle, assuring that the man himself said it:\n\n\"Oppression is not hearing a word that offends you. Oppression is being told you can't say certain words because you will offend someone else\"\n\nBut when I googled it, I could only find posts related to this tweet, so I'm assuming this woman said it, not Morgan Freeman. But when I said that to my friend, he insisted Mr. Freeman said it first.\nSo, does anyone know for a fact if Morgan Freeman said this phrase?\n", "answer": "Going through all of Morgan Freeman's quotes in the website BrainyQuote, he didn't say this, but this was said by Blaire White (random YouTuber).\nsource: All of Morgan Freeman's Quotes\n", "topic": "skeptics", "url": "https://skeptics.stackexchange.com/questions/39070"}, {"image": "VQAonline_00059406.png", "question": "Has California built only 1 University since 1980?", "context": "Kambree Kawahine Koa posted this image:\n\nHas California built only 1 University since 1980?\n", "answer": "No, this Huff Post article* which repeats the claim was corrected to say that it considers only University of California system:\n\nCorrection: [...] Language has also been amended to indicate that the article is considering the University of California system, and not the California State University system.\n\nThe University of California system indeed opened only one new campus since 1980, UC Merced, which started working in 2003-2004:\n\nFaculty members began to arrive in 2003 with graduate students in tow and began setting up research laboratories and programs at UC Merced's ancillary research facility on the former Castle Air Force Base, biding their time until buildings were ready on campus. The first graduate courses began in fall 2004.\n\nBut, the California State University program opened another 3 universities in that time:\n\nCSUSM:\n\n\n1989: Governor George Deukmejian signs Senate Bill 365 into law. The bill officially reconstitutes the SDSU satellite campus as California State University San Marcos \u2014 the first comprehensive U.S. university to be founded in two decades, and the 20th campus in the CSU system. Bill W. Stacy is named the university\u2019s first president and is tasked with recruiting 12 founding faculty members.\n\nCSUMB:\n\n\nFounded in 1994 on the former site of Fort Ord by educators and community leaders, Cal State Monterey Bay faculty and staff build on that legacy as we explore innovative ways to meet the needs of a new generation of students while simultaneously powering the Monterey County economy.\n\nCSUCI:\n\n\nEstablished in 2002, we\u2019re the youngest of 23 campuses in the CSU family.\n\n\nDuring that time at 7 new community colleges were opened in California:\n\nClovis Community College:\n\nClovis Community College was originally opened as a center in 2007, with the 80,000 square foot Academic Center One complex\n\nFolsom Lake College\n\nFolsom Lake College was established in 2004 as the fourth campus of the Los Rios Community College District\n\nIrvine Valley College:\n\nIrvine Valley College, established in 1985 in Irvine\n\nMoreno Valley College:\n\nThe Moreno Valley and Norco Campuses, opened in March 1991, have grown rapidly and were granted initial accreditation in January 2010\n\nNorco College:\n\nNorco College opened to students in 1991\n\nSantiago Canyon College:\n\nSantiago Canyon College (SCC), established in 1985\n\nWest Hills College Lemoore:\n\nWest Hills College Lemoore was established in 2002\n\n\nThere are some community colleges which were opened before 1980, but became part of the California Community Colleges system after 1980.\nIn addition, according to the List of colleges and universities in California wikipedia page 37 private institutions opened since 1980 (of those two have been since closed and one is unaccredited).\n* - The original claim seems to come from Business Insider, but appears without any correction or clarification, see here (dated Jun. 13, 2012) and here (dated Apr. 12, 2012) \n", "topic": "skeptics", "url": "https://skeptics.stackexchange.com/questions/37834"}, {"image": "VQAonline_00059481.png", "question": "Was the quantity of Fentanyl imported in 2018 enough to kill every American?", "context": "ABC reporter Meridith McGraw, who covers the White House, recently tweeted:\n\nInbox: For a payment of $20.20, the Trump campaign says they will send a \u201cfaux brick\u201d to Pelosi and Schumer\u2019s offices \u201cwith facts about the need for border security and an appeal to \u201cTEXT \u2018WALL\u2019 to 88022.\u201d\n\n\nThe brick reads:\n\nTEXT \"WALL\" TO 88022\n2018: FENTANYL AT THE BORDER\nENOUGH ENTERED U.S, TO KILL EVERY AMERICAN\n\nThis tweet was notable enough to be picked up by The Week,\nDid enough Fentanyl get (illegally?) imported to the US in 2018 to kill every (US) American?\n", "answer": "According to Fentanyl seizures at US-Mexico border up 750 percent:\n\nThe amount of fentanyl seized by border agents has skyrocketed in recent years, from two pounds in fiscal year 2013 to approximately 1,485 pounds from Oct. 1, 2016, to Sept. 30, 2017 \n\nMatthew Barden of the Drug Enforcement Administration is cited as saying: \n\n300 pounds of fentanyl is lethal enough to kill about 100 million people\n\nSo enough fentanyl was seized by border agents in FY2017 to kill every American. Plus, obviously not all the fentanyl is interdicted. As long as 2018 amounts are at least as much as FY2017, the claim is defensible. \n", "topic": "skeptics", "url": "https://skeptics.stackexchange.com/questions/43246"}, {"image": "VQAonline_00059223.png", "question": "Is this a way to survive a lightning strike?", "context": "\nsource\nText in the image (clockwise):\n\n\nCrouch down low like a baseball catcher. Get as low as you can. The nearer\n you are to the ground, the less likely you are to be struck by lightning. But never lie down!\nIf your hair begins to stand on end or your skin starts to tingle, a lightning strike is imminent. Immediately get into the crouching position. Lightning may strike without this warning, however.\nPlace hands over ears to minimize hearing loss from the loud clap of thunder that will boom very close to you.\nThe only thing touching the ground should be the balls of your feet. Lightning can hit the ground first, and then enter your body. The more you minimize your contact with the ground, the less chance of electricity entering your body.\nTouch the heels of your feet together. If electricity from a ground strike enters through your feet, this increases the chances of the electricity going in one foot and out the other, rather than into the rest of your body.\nDon't touch any possible conductors.\n\n\n", "answer": "It's a bit of a stretch. FEMA describes most of that advice as a last resort: if you feel a tingle, and you are in the open, then assume that position, meant to minimize damage.\nOn the other hand, FEMA and all other sources that I've seen clearly spell out that you need to find shelter, and that's the best way to ensure your safety.\n\nIf you feel your hair stand on end (which implies that lightning is about to strike), squat low to the ground on\nthe balls of your feet. Place your hands over your ears and your head between your knees. Make yourself the\nsmallest target possible and minimize your contact with the ground. DO NOT lie flat on the ground.\n\nReady.gov agrees:\n\nDuring Thunderstorms and Lightning\nIf thunderstorm and lightning are occurring in your area, you should:\n[...]\n\nAvoid hilltops, open fields, the beach or a boat on the water.\nTake shelter in a sturdy building. Avoid isolated sheds or other small structures in open areas.\n\n\nA good summary on how to survive a lightning storm is provided by [Live Science]\n\nHow to survive a lightning strike\nThe best way to survive, of course, is to avoid a lightning strike.\nThe Federal Emergency Management Agency (FEMA) recommends people\nfollow the 30/30 rule: If, after seeing lightning, you can't count to\n30 before hearing thunder, get inside a building immediately (because\nthe lightning storm is close). And don't go outside until 30 minutes\nafter the last clap of thunder.\n[...]\nIf you're caught outdoors in a forest during a lightning storm, seek\nshelter in a low area under a dense growth of small trees. Avoid tall\ntrees, since lightning tends to strike the tallest object in an area.\nIf you're in an open area, go to a low area, like a valley or a ravine\n(but be alert for flash floods). If you're in a boat in open water,\nget to land as quickly as possible.\nAnd if you feel your hair stand on end, that means lightning is about\nto strike. As a last resort, immediately squat down on the balls of\nyour feet, cover your ears with your hands (to minimize hearing loss)\nand put your head between your knees.\n\n(my emphasis)\nHow to Survive a Lightning Strike\n", "topic": "skeptics", "url": "https://skeptics.stackexchange.com/questions/22430"}, {"image": "VQAonline_00059705.png", "question": "An\u00e1lisis sint\u00e1ctico de una oraci\u00f3n muy compleja (con un \u00e1rbol)", "context": "Estoy tratando de analizar las oraciones en la frase :\n\nme temo que el agua fresca no sea la mejor soluci\u00f3n para curar la quemadura que se hizo con la cacerola llena de aceite hirviendo\n\nPero despu\u00e9s de unos d\u00edas, mi mente se volvi\u00f3 en una tostada... Estoy muy inseguro sobre la calidad de este an\u00e1lisis.\n\nSi algui\u00e9n tiene el coraje de ayudarme dici\u00e9ndome lo que es falso y lo que es correcto, usted ser\u00e1 un shah.\nCordialmente.\n", "answer": "Tu an\u00e1lisis parece bastante correcto. Lo mejor en estos casos es ir descomponiendo el problema en partes m\u00e1s peque\u00f1as, con la vieja t\u00e1ctica de \"divide y vencer\u00e1s\". As\u00ed que vamos all\u00e1.\n\nMe temo que el agua fresca no sea la mejor soluci\u00f3n para curar la quemadura que se hizo con la cacerola llena de aceite hirviendo.\n\nLa primera frase es:\n\nMe temo X.\n\nAqu\u00ed se usa el verbo temer, que es transitivo y usado m\u00e1s como pronominal, de ah\u00ed el me inicial. As\u00ed que X es el complemento directo de la oraci\u00f3n. Descompongamos X:\n\nEl agua fresca no es Y.\n\nCambio el verbo a indicativo para mejor comprensi\u00f3n de la frase. Aqu\u00ed la estructura es sencilla: oraci\u00f3n copulativa tipo \"X [no] es Y\", donde Y es el atributo de X. Descompongamos Y:\n\nLa mejor soluci\u00f3n para Z.\n\nAqu\u00ed tenemos un sintagma nominal la mejor soluci\u00f3n, seguida de Z que, como bien has dicho, en este caso es una oraci\u00f3n subordinada sustantiva con el verbo en infinitivo (equivaldr\u00eda a \"la mejor soluci\u00f3n para eso\"), introducida por para en su acepci\u00f3n 4. Descomponemos Z:\n\n[...] cura la quemadura W.\n\nCambio el verbo y supongo un sujeto elidido. Analizando, aqu\u00ed W ser\u00eda un complemento adjetival de quemadura, que a su vez ser\u00eda el complemento directo de curar, verbo transitivo. Descomponemos W:\n\nSe la hizo con la cacerola llena de aceite hirviendo.\n\nLa preposici\u00f3n con denota el medio, modo o instrumento que sirve para hacer algo, por lo que efectivamente introduce un complemento circunstancial de modo. \u00bfC\u00f3mo se hizo la quemadura? Con la cacerola de aceite hirviendo. A partir de aqu\u00ed ya no tenemos m\u00e1s frases subordinadas. Simplemente hirviendo es el adjetivo de aceite, llena es adjetivo de cacerola, y de aceite hirviendo es un sintagma preposicional que complementa al adjetivo llena.\n", "topic": "spanish", "url": "https://spanish.stackexchange.com/questions/35708"}, {"image": "VQAonline_00059683.png", "question": "\u00bfDe d\u00f3nde viene la expresi\u00f3n \"dar sopas con honda\"?", "context": "Es curioso que yo siempre he pensado que la expresi\u00f3n era \"dar sopa con ondas\", creo que desde que le\u00ed este c\u00f3mic de Ast\u00e9rix:\n\nSin embargo, descubro en el diccionario que la expresi\u00f3n es, en realidad:\n\ndar sopas con honda alguien o algo a otra persona o cosa\n\nloc. verb. coloq. Mostrar una superioridad abrumadora sobre ella.\n\n\nBuscando en Google parece, en todo caso, que la forma \"sopa con ondas\" tiene m\u00e1s popularidad que la original \"sopas con honda\". As\u00ed pues, \u00bfcu\u00e1l es la forma correcta? \u00bfDe d\u00f3nde viene?\n", "answer": "Desde luego, ni lo de tirar pedazos de pan empapados en un l\u00edquido usando un arma ni lo de crear ondulaciones en un caldo parece tener mucho sentido. Sin embargo, todo cobra m\u00e1s sentido si nos fijamos en esta acepci\u00f3n:\n\nsopa de arroyo\n\nf. coloq. desus. Piedra suelta o guijarro.\n\n\nEsta acepci\u00f3n, hoy en desuso, estaba vigente hace algunos siglos, como atestigu\u00f3 el Autoridades en 1739:\n\nSOPA DE ARROYO. En estilo festivo llaman \u00e0 la piedra suelta, \u00f2 guijarro; porque en los arroyos suelen estar unas veces en seco, y otras mojadas, conforme crece, \u00f2 mengua el agua de ellos.\n\nSe recoge su uso en la segunda parte del Quijote:\n\nAsaz de locura ser\u00eda intentar tal empresa: considere vuesa merced, se\u00f1or m\u00edo, que para sopa de arroyo y tente bonete no hay arma defensiva en el mundo, sino es embutirse y encerrarse en una campana de bronce [...].\n\nSe entiende pues el origen: aquel que sea capaz de darle con un guijarro y una honda al oponente es el que demostrar\u00e1 mayor pericia.\nEn todo caso, la expresi\u00f3n parece relativamente reciente en comparaci\u00f3n. El primer caso que encuentro es en la hemeroteca de la BNE:\n\nY luego, al final, vendr\u00e1 la lucha con el traidor, que aunque le d\u00e9 sopas con honda, tendr\u00e1 que dejarse vencer, porque si no el director se cabrea.\nMuchas gracias (Madrid). 13/10/1928, n.\u00ba 244, p\u00e1gina 8.\n\nEn la misma hemeroteca es en 1979 cuando aparece el primer caso de la versi\u00f3n \"sopa con ondas\". Curiosamente, la versi\u00f3n \"sopas con onda\" aparece en la misma publicaci\u00f3n que la citada anteriormente:\n\nCon el Extraordinario de Primavera, vamos a dar sopas con onda a la Vie Parisienne.\nMuchas gracias (Madrid). 28/3/1925, n.\u00ba 61, p\u00e1gina 15.\n\nPor lo cual se ve que la confusi\u00f3n honda/onda existi\u00f3 desde el primer momento.\n", "topic": "spanish", "url": "https://spanish.stackexchange.com/questions/30622"}, {"image": "VQAonline_00059604.png", "question": "Understanding a cartoon", "context": "I was recently shown the Spanish cartoon below and I'm having trouble understanding it. When I translate it directly, I get:\n\nI am turning myself off!\n I am turning myself off!\n I don't have a connection!\n\nI understand that \"me apago\" is an usual use of apagar, which can be inferred as both I am dimming myself (extinguishing myself) and as I am turning myself off. However, I just don't understand the comical side of the cartoon/what is the bigger picture. \n\n", "answer": "I believe the humor behind this particular cartoon is that this man can no longer think by himself and is depending upon an Internet connection for him to perform the most basic brain functions such as \"staying on\".\nWhile \"me apago\" could be correctly translated as \"I am turning myself off\" in this particular context is more like \"I am powering off\". When you turn on an electrical device you are \"powering it on\", when you turn it off you are \"powering it off\". As mentioned by Jdamian, this would be akin to \"I am shutting down\" which was what I was looking for when writing this answer.\n\"No tengo conexi\u00f3n\" is frequently used to express \"I have no Internet connection\" or \"I have no phone service\".\nSo, I am assuming in this context he (or his brain) is expressing \"I am powering off (I am shutting down), I have no internet connection\". He is expressing this in an anxious manner, so this cartoon might be trying to make fun of how nowadays people can't do anything unless they are connected to the Internet because they can't think for themselves. It's obviously an exaggeration, of course.\nIf there is more to this cartoon (like other panels, a caption, a tool-tip, etc.) please share it, it might help to understand its meaning since the punchline could be hidden there.\nUpdate: Thanks to fedorqui, it is clear that there is no hidden punchline for this cartoon. I had suspected the author was making fun of younger generations for their inability to do anything without an Internet connection but I refrained from adding such conclusion since there was no proof that the author would be of an older generation (I didn't know it was El Roto); however, it appears my suspicions were not far off.\n", "topic": "spanish", "url": "https://spanish.stackexchange.com/questions/19064"}, {"image": "VQAonline_00059546.png", "question": "\u00bfQu\u00e9 palabra se usa para describir cuando un pago cubre hasta cierta fecha?", "context": "Soy el autor de una aplicaci\u00f3n de computadora para administrar Casas de Empe\u00f1os en Puerto Rico. Cuando un cliente debe tres meses de intereses, podr\u00eda perder la prenda, pero si solo paga dos meses de intereses, el cliente se queda atrasado un mes para poner los intereses al d\u00eda. \nEjemplo: El cliente empe\u00f1\u00f3 una prenda el 1-feb-2012. Tres meses despu\u00e9s, en 1-may-2012, el cliente solo pag\u00f3 2 meses de intereses y esta pago hasta 1-abr-2012. Yo estoy definiendo esta fecha como \"Fecha de Vigencia\". \u00bfHay alguna palabra que describa mejor esta fecha?\nHe aqu\u00ed un ejemplo de pantalla de transacciones:\n\n", "answer": "Yo trabajo en un equipo de Payroll para US/Canada en Accenture y solemos tratar con este tipo de formas todo el d\u00eda. Creo que lo m\u00e1s adecuado deber\u00eda de ser no la mejor traducci\u00f3n literal, sino la mejor expresi\u00f3n de la idea para que al usuario le quede claro y el programa sea intuitivo. Yo buscar\u00eda soluciones m\u00e1s cortas y simples como: \n\n\u00daltimo pago\n\u00daltimo inter\u00e9s\nInteres pago en:\nInteres pagado:\nPago efectivo desde:\n\u00daltimo pago entrado\n\u00daltima entrada\n\u00daltimo plazo pagado\n\u00daltimo check in\netc. Combinaciones hay miles.\n\nEn realidad sacar\u00eda las palabras que se dan por sobrentendidas: Si est\u00e1s en una pantalla de -plazos-, se sobrentiende que \"\u00faltimo pago\" es del plazo. Y as\u00ed sucesivamente: si est\u00e1s en una pantalla de \"alquiler\", \"\u00faltimo pago\" es del alquiler, o \"\u00faltimo alquiler\", al ser fecha se sobrentiende que es el \u00faltimo pago.\nEs m\u00e1s, te recomiendo usar abreviaturas para palabras comunes en gente que se dedica a finanzas, o ni siquiera:\n\n\u00dalt. pago\n\u00damo. pago\n\n", "topic": "spanish", "url": "https://spanish.stackexchange.com/questions/2474"}, {"image": "VQAonline_00059714.png", "question": "\u00bfCu\u00e1l es el mejor t\u00e9rmino para traducir la palabra \"nice\" en un contexto de Matem\u00e1ticas?", "context": "\u00a1Hola!\nEstos d\u00edas comenc\u00e9 a trabajar con mi tesina y me encontr\u00e9 con un t\u00e9rmino para el cual no acabo de encontrar una buena traducci\u00f3n al espa\u00f1ol sin que se pierda la seriedad del trabajo o que quede muy fuera de lugar. El t\u00e9rmino aparece en la siguiente oraci\u00f3n\n\nA maximal locally finite cellular family of open sets is called nice.\n\nDicha oraci\u00f3n viene en la siguiente imagen que a su vez es un extracto del art\u00edculo que se puede encontrar aqu\u00ed\n\nEn pocos t\u00e9rminos, dice que cuando una familia de de abiertos es maximal, localmente finita y celular es llamada nice. Hasta ahora, en la redacci\u00f3n de mi trabajo me he referido a estas familias como familias nice pero obviamente queda muy fuera de lugar. Intent\u00e9 con familia \u00f3ptima, buena familia, pero siento que es mala la traducci\u00f3n porque a veces se usan precisamente las palabras \u00f3ptima y buena como adjetivos en este contexto y solo causar\u00eda confusi\u00f3n. \u00bfHay alg\u00fan t\u00e9rmino que sea mejor que pudieran sugerirme?\nGracias por leerme.\n", "answer": "En esta tesis doctoral en espa\u00f1ol la autora usa el t\u00e9rmino nice 312 veces sin despeinarse.\nInventarte un t\u00e9rmino que no sea del agrado de alg\u00fan miembro del tribunal puede ser desastroso. Ya llegar\u00e1 el momento en el que puedas sentar c\u00e1tedra. Si ni t\u00fa ni tu director de tesina hab\u00e9is visto una traducci\u00f3n ya establecida, yo preguntar\u00eda a otros profesores e indagar\u00eda por foros de matem\u00e1ticas. Y si no encuentras ninguna publicaci\u00f3n en la que te puedas apoyar, mejor lo dejas sin traducir.\nEso s\u00ed, yo lo escribir\u00eda en cursiva todas las veces.\n", "topic": "spanish", "url": "https://spanish.stackexchange.com/questions/37835"}, {"image": "VQAonline_00059629.png", "question": "Uso y origen de la expresi\u00f3n \"pon t\u00fa\"", "context": "En esta respuesta que da @aparente001, leo la expresi\u00f3n \"pon t\u00fa\" en esta frase:\n\nPon t\u00fa: imagina, vas a construir una casa con m\u00e9todos tradicionales.\n\nY en una b\u00fasqueda r\u00e1pida me aparecen otros ejemplos como este:\n\nPon t\u00fa que eres parte de la corte rusa y tu familia es invitada a cenar al Palacio de los zares...\n\ny como este otro:\n\nPor el contexto creo que significa \"sup\u00f3n\" o \"imagina\". \u00bfEstoy en lo cierto? \u00bfD\u00f3nde se usa? \u00bfQu\u00e9 significa exactamente? \u00bfC\u00f3mo se origin\u00f3?\n", "answer": "\u00c9sta es una frase que invita al oyente a imaginar o suponer una situaci\u00f3n, o da por cierta una suposici\u00f3n a modo de hip\u00f3tesis; introduce una hip\u00f3tesis no de la manera sint\u00e1ctica habitual (Suponiendo que\u2026 Supongamos que\u2026 Imaginemos que\u2026 Vamos a postular que\u2026) sino por medio de un imperativo. No s\u00e9 qu\u00e9 nombre tiene esta clase de discurso, pero me recuerda mucho a esas traducciones de la Biblia donde se emplea el imperativo \u00a1mira! casi como puntuaci\u00f3n.\nSeg\u00fan leo en este foro, en Chile se dice ponte t\u00fa tambi\u00e9n con el significado de \"mira, he aqu\u00ed que\", pero por lo dem\u00e1s el significado es el mismo que el notado en la pregunta. En el diario La Tercera encuentro por ejemplo:\n\n\"No digo que sea una causa, pero ponte que se perdi\u00f3 una caja con un par de millones justificables all\u00ed.\"\n\nEn Espa\u00f1a (o por lo menos en Murcia) se usa el ponle:\n\n\"Ponle que te da un perrengue. Bebes agua sudando, o te comes un esp\u00e1rrago de bote en mal estado, y sufres una embolia.\"\n\nEn Argentina decimos ponele (o pon\u00e9le con la tilde ortogr\u00e1fica que ya no es estrictamente necesaria). Se emplea muchas veces sola esta palabra, sin subordinada, con el sentido de \"Supongamos que es as\u00ed\". Hay un ponele de esta clase en Un tal Lucas, de Julio Cort\u00e1zar (1979). Despu\u00e9s de una larga perorata de Lucas, el di\u00e1logo sigue as\u00ed:\n\n\u2014Ponele \u2014dice alguien\u2014, pero frente a la coyuntura hist\u00f3rica el escritor y el artista que no sean pura Torredemarfil tienen el deber, o\u00edme bien, el deber de proyectar su mensaje en un nivel de m\u00e1xima recepci\u00f3n.\n\nSe dice tambi\u00e9n pongamos (como en la canci\u00f3n de Joaqu\u00edn Sabina, Pongamos que hablo de Madrid) y hasta pong\u00e1mosle, como en el tango Se dice de m\u00ed:\n\nSi fea soy / pong\u00e1mosle / que de eso a\u00fan / no me enter\u00e9.\nEn el amor / yo s\u00f3lo s\u00e9 / que a m\u00e1s de un gil / dej\u00e9 de a pie.\n\nEn todos los casos hay una oscilaci\u00f3n entre los dos usos:\n\nEl hipot\u00e9tico: \"supongamos que\", \"imaginemos que\".\nEl concesivo: \"aceptando que\", \"dando por cierto que\".\n\nHay un par de apariciones del ponle t\u00fa en Calder\u00f3n de la Barca (aqu\u00ed y aqu\u00ed), que me suenan a algo as\u00ed como un OK, una simple muletilla verbal, uso que no es muy diferente al uso concesivo que actualmente le damos.\nLamentablemente no s\u00e9 si pueda responder de d\u00f3nde viene la expresi\u00f3n. En algunas formas suena como un abreviatura de la forma correspondiente del verbo suponer, pero el pronombre le no va bien con esa teor\u00eda. \n", "topic": "spanish", "url": "https://spanish.stackexchange.com/questions/22225"}, {"image": "VQAonline_00059582.png", "question": "\u00bf\"Hater\" en castellano?", "context": "En la jerga de internet, es muy com\u00fan usar el anglicismo \"hater\":\n\n\u00daltimamente Twitter est\u00e1 lleno de haters: cada vez que alguien\n publica algo sobre el partido X, lo critican sin misericordia.\n\n\"Hater\" se traduce como persona que odia, pero la connotaci\u00f3n es de un odio ligero, muy visceral pero circunscrito al \u00e1mbito de lo escrito, sin llegar a la violencia: Alguien puede ser un hater de un lenguaje de programaci\u00f3n, un jugador de f\u00fatbol o vete a saber qu\u00e9.\nMe pregunto si hay alguna palabra en castellana que quiera decir lo mismo. Se me ocurri\u00f3 decir odioso, porque los haters lo son, pero esto se referir\u00eda a lo que se odia, y no a la persona.\nY ya que estamos, \u00bfc\u00f3mo traducir\u00edais \"haters gonna hate\"? \u00bf\"Los que odian van a odiar\"?\n\n", "answer": "No creo que haya un sin\u00f3nimo exacto (como casi con todo) pero se me ocurren varias traducciones para escoger, dependiendo del contexto. No s\u00e9 si entiendo bien lo del panda: \u00bfse refiere a \"disfruta, aunque siempre hay alguien a quien le va a caer mal\" o \"ya ver\u00e1s como incluso esto le cae mal a alguien\"?\n\nUn buen t\u00e9rmino ser\u00eda \"atrabiliario\", que suele aplicarse a la cr\u00edtica, pero que es de poco uso en la lengua cotidiana. Se podr\u00eda decir \"sus comentarios sobre este tema son atrabiliarios\". Para usarlo en el ejemplo que das no funciona muy bien. \nCritic\u00f3n: la traducci\u00f3n podr\u00eda ser \"los criticones lo van a criticar\" o \"los criticones critican\" o incluso \"siempre va a haber criticones que critiquen\".\nAguafiestas: \"los aguafiestas te aguar\u00e1n (o intentar\u00e1n aguarte) la fiesta\".\nCol\u00e9rico: \"los col\u00e9ricos se encolerizar\u00e1n\".\nSa\u00f1udo: no es un t\u00e9rmino muy de uso informal pero se puede decir \"lo sa\u00f1udos se ensa\u00f1ar\u00e1n\".\nRabioso: \"los rabiosos se enrabiar\u00e1n\".\n\n", "topic": "spanish", "url": "https://spanish.stackexchange.com/questions/17159"}, {"image": "VQAonline_00059787.png", "question": "Can a back row setter reach into the plane to play the ball?", "context": "A first hit is over passed towards the net and passes partially through the plane of the net. The back row setter is tall and jumps to pull the ball back with a closed fist. Is this a legal hit? Or does the moment the ball touches the plane mean that the back row setter makes an illegal attack on the ball even though he is pulling the ball back for a third hit?\nThis is the ball position where the back row setter makes contact.\n\n", "answer": "(Edited per amended question. TL;DR: the setter made a fault of \u201creaching beyond the net\u201d, signalled by the referee by a hand placed above the net, palm facing downwards.)\n\nThere are two issues when a back-row player sets near/above the net:\n\nWhether the hit itself was legal: Did the setter touch the ball within his own playing space? Was it a proper hit? (Was the ball not held/caught/thrown)? According to your image, the setter touched the ball beyond the net, above the opponent\u2019s court. In that case, the setter made a fault of \u201creaching beyond the net\u201d, signalled by the referee by a hand placed above the net, palm facing downwards.\nWhen the set is imperfect and the setter actually plays the ball towards the opponent\u2019s court, it counts as an attack hit. And back-row players cannot complete an attack hit from the front zone when the ball is completely above the top of the net. This means that if the setter hits the ball completely above the top of the net towards the opponent\u2019s court, it is an \u201cattack hit fault\u201d if the attack hit is completed, which means either the ball crosses the vertical plane of the net completely, or the opponent touches the ball.\n\nBut if the back-row setter touches the ball within his own playing area (or, theoretically, above the free zone, if the ball is overflying the net partially or completely outside the antennae), and plays it \u201cbackwards\u201d into his own half of the court or parallel to the net or his teammate touches the ball prior to the ball crossing the net completely and prior to any blocking opponent touching the ball, it is valid play.\nRelevant rules (see FIVB Official Volleyball Rules 2015-2016):\n\n9: Each team must play within its own playing area and space\n11.4.1 [Player\u2019s faults at the net] A player touches the ball or an opponent in the opponent's space before or during the opponent\u2019s attack hit.\n13.1.3: An attack hit is completed at the moment the ball completely crosses the vertical plane of the net or is touched by an opponent.\n13.2.3: A back-row player may also complete an attack hit from the front zone, if at the moment of the contact a part of the ball is lower than the top of the net.\n13.3.3: [Faults of the attack hit] A back-row player completes an attack hit from the front zone, if at the moment of the hit the ball is entirely higher than the top of the net.\n\nCf. also cases 3.10.2 and 3.30 in the Volleyball Casebook:\n\n3.10.2\nTeam A\u2019s setter hit the ball above the net so that at the moment of the hit his/her fingers were in the opponent\u2019s space. After the set the ball flew parallel to the net toward an attacker. The blocker of team B touched the ball in team\u2019s A space, so that the team A player could not execute the attack hit. How should the 1st referee decide?\nAccording to Rule 9, each team must play the ball within its own playing area and space (except in the case of Rule 10.1.2). Therefore, since the setter has hit the ball in the opponent\u2019s space, the setter committed a fault. The blocker also committed a fault by touching the ball in the opponent\u2019s space before the attack hit. However, only the first fault is penalized.\n3.30\nA back row setter jumped from within the attack zone and set the ball while it was completely above the height of the net, directing it towards an attacker. Before the attacker could contact the ball, it penetrated the vertical plane of the net where it was blocked by the opponent\u2019s setter. Was there a fault?\nYes. The set became an illegal attack hit by a back row player when the attack hit was completed (in this case by contacting the opponent\u2019s block). The rally should have been won by the opponents.\n\n", "topic": "sports", "url": "https://sports.stackexchange.com/questions/12116"}, {"image": "VQAonline_00059816.png", "question": "What does the superscript used in tennis scores mean?", "context": "In the score below, what do the 7 and 4 superscripts mean in the first set?\n\n", "answer": "When a set reaches 6-6, a tiebreaker is used to determine the winner of the thirteenth game and hence the set. The first player serves once, then the players alternate serving twice consecutively, scoring one point per point won, until one player has both\n\nseven points or more, and\na lead on the opponent of two points or more\n\nThe superscript numbers record the final score in the tiebreaker. The first set went to 6-6, where a tiebreaker was played to a score of 7-4 in Carreno Busta's favour, winning him the set 7-6.\nCompare with other matches in the competition on the same day.\n\nDolgopolov wins first set versus Schwartzman, tiebreaker 7-1.\nSeppi wins second set versus Karlovic, tiebreaker 7-4.\nKarlovic wins third set versus Seppi, tiebreaker 7-3.\nEdmund wins first set versus Basilashvili, tiebreaker 7-0.\nKyrgios wins first set and third set and fourth set versus Tsonga, tiebreakers 7-5 and 8-6 and 7-5.\nCilic wins first and third sets versus Harrison, tiebreakers 7-4 and 7-4.\n\n", "topic": "sports", "url": "https://sports.stackexchange.com/questions/17636"}, {"image": "VQAonline_00059747.png", "question": "Two dismissals off a single ball", "context": "\nIs it possible to dismiss both batsmen off the same delivery (i.e., by a catch and a run out)?\nIf the ball touched the lower end of the bat, edged up and touched the bat again, will it be given out? This is assuming that the batsman does not intend to hit it twice.\nIf the ball hits the bat on the other side of the bat, does it count as a dismissal?\n\n \n", "answer": "\nIs it possible to dismiss both the batsmen in ground(ie,by a catch and a run out)?\n\nNo. It is not possible to dismiss both batsmen on the same delivery. Only one batsman can be legally dismissed. In the event of both being dismissed on the same delivery, the batsman dismissed first goes back to the pavilion while the other continues with his/her innings.\n\nIs \"Hit the ball twice\" out?\n\nAccording to the Laws of Cricket (Law 34 - as stated below), if done intentionally, then yes, it is out.\n\nLaw 34 (Hit the ball twice)\n 1. Out Hit the ball twice \n\nThe striker is out Hit the ball twice if, while the ball is in play, it strikes any part of his person or is struck by his bat and, before the ball has been touched by a fielder, he willfully strikes it again with his bat or person, other than a hand not holding the bat, except for the sole purpose of guarding his wicket. See 3 below and Laws 33 (Handled the ball) and 37 (Obstructing the field). \nFor the purpose of this Law, 'struck' or 'strike' shall include contact with the person of the striker.\n\n\nHowever, as with any potential dismissal, the batsman can still continue his/her innings in the following cases:\n\nThe opposition team does not appeal (Law 27.1).\nThe appeal is withdrawn by the opposition captain and the umpire consents to the withdrawal, before the batsman leaves the field of play (Law 27.8).\n\n\nIs it considered a dismissal if the ball hits the other side of the bat?\n\nNo. As of the current laws, this does not constitute a dismissal. However, the way the modern bat is designed, the batsmen do not get any advantage by using the other side of the bat. So, it does not make sense for the batsmen to use the other side of the bat.\n", "topic": "sports", "url": "https://sports.stackexchange.com/questions/1722"}, {"image": "VQAonline_00059833.png", "question": "Fair Play award and team performances in IPL", "context": "In IPL fair play award seem to resemble points table except few outliers.\nDoes fair play award has to do anything with who won the match also?\n\n", "answer": "No, this is obviously not the case.\nOf the 58 final points table positions after pool play in the IPL seasons 2012-2018, only twelve (12) matched the fair play rank for that team, and only three of these were the award winners.\nIf final points table positions and fair play award ranks are plotted with a linear regression model, there is almost zero dependence on the points table position (moving one place up or down has an effect of moving about ~0.11 places in the fair play ranking), with the starting position being almost exactly the mean final points table position (the fair play ranks are almost uniformly random rather than being biased to one side), and the data has a correlation coefficient of just 0.0131 (the points table position has as close to zero effect as it probably could!).\nIf the distance between fair play rank and final points table position is checked instead, the mean absolute difference is 2.48 ranks with an approximately Poisson distribution (a vast majority of ranks are not the same as positions, with many being very different). The actual differences have approximately a normal distribution with a standard deviation of 2.02 (which, given the range of ~7.3, again suggests that fair play ranks are almost perfectly random and separated from table positions).\nThis should also make perfect sense, since most teams will most often receive most of the fair play points they are eligible for, barring unlikely and extreme events, and teams may show high sportsmanship despite losing or conversely demonstrate poor spirit despite playing very well.\n", "topic": "sports", "url": "https://sports.stackexchange.com/questions/21335"}, {"image": "VQAonline_00059740.png", "question": "What are those blue stripes or patches on Balotelli's back?", "context": "I've seen a lot of players with those blue stripes or patches like Balotelli wore in the Italy - Germany Euro 2012 semi-final:\n\nWhat are those stripes / patches/ tapes?\n", "answer": "It is Kinesio Tape, which is an adhesive-tape that is applied to problematic muscles.\nProblematic as in, the muscles are weak after returning from injury (you can support them using Kinesio tape), or you can apply it to aching muscles to sooth the pain. It's application can also increase blood flow to the muscles, and speed up the recovery process.\nSpecifically to Mario, no-one knows the exact reason he's using it.\nHere's an article which explains the science behind it (beyond me :P).\n", "topic": "sports", "url": "https://sports.stackexchange.com/questions/1043"}, {"image": "VQAonline_00059767.png", "question": "US Assault class gear detail in Battlefield 4", "context": "I am an airsoft player and would like to reproduce the US Assault class in Battlefield 4 as seen below.\n\nFor the base layer, I'm assuming any sand balaclava, sand gloves and multicam pants would do. I don't know if Dark Earth combat shirts with sand sleeves exist.\nFor the head part, A sand Mich 2000 or similar, sand goggles, and a Peltor Comtac.\nI have absolutely no idea of what type of chest rig/plate carrier is used. Also the belt is some sort of harness ( I don't know the exact word ).\nIf anyone could help me, a name or a link to similar equipment would be nice.\n", "answer": "From top to bottom: \n\nOPS-CORE: FAST Ballistic high cut (XP) Helmet: Tan + LUX LINER H-NAPE\nPrinceton Tec Charge MPLS\nMS2000 Strobe\nPeltor Comtac 3\nXGO Phase 4 Neck Gaiter in Coyote Brown\nESS Profile NVG Military Goggles black with gray Lenses \nLBT 6094 in Coyote Brown\nVIP IR LED Strobe Light \nSAFARILAND 6378USN ALS\u00ae Low Signature Holster Multicam\nNew Eagle Industries USMC MOLLE Coyote FB 9MM Mag Pouch \nTwo red chem lights\nBlackhawk S.T.R.I.K.E 40MM Grenade Molle Pouch in Coyote Tan\nLBT-6159 Modular Double M4/M14/MP5 Mag Pouch in Coyote Brown\nDouble M67 Pouch in Coyote (NSN 8465-01-558-5185)\nCombat First Response Pouch (Condor)\nTactical Tailor 40mm Belt 12rd in Foliage\nYATES 313 Lightweight Assault Harness in Terra\nCRYE Combat Pant AC Multicam + AirFlex\u2122 Combat Knee Pads in Khaki\nCRYE G3 Combat Shirt\u2122 Khaki 400\nTACOPS\u00ae M9 Assault Medical Backpack in coyote + IR reflective reversed Velcro Flag + three red and one white chem light\nFR FAST ROPE GLOVE COYOTE (Oakley)\nLowa Elite Desert Ws\n\nAll I didn't find is the waist pouch. \nI'm building the same loadout and I hope You have success.\n", "topic": "sports", "url": "https://sports.stackexchange.com/questions/4786"}, {"image": "VQAonline_00059827.png", "question": "When did jerseys get dots on them?", "context": "I thought jerseys have last names (or nothing) on the back of them. However, I recently saw dots on a jersey.\n\nWhen did jerseys get dots on them? What do they represent? It sorta looks like scoreboard dots.\n", "answer": "I'm pretty sure that's Braille, the writing system for visually impaired people.\nThe letters spell B-U-N-D-Y, so it's probably Dylan Bundy, who plays with #37 for the Orioles.\n", "topic": "sports", "url": "https://sports.stackexchange.com/questions/20231"}, {"image": "VQAonline_00062046.png", "question": "Scale of variable importance in randomForest, party & gbm", "context": "I've computed some variable measures using the packages, gbm, randomForest and party. I develop binary classification models predicting survival in cancer patients. Although the gbm package, randomForest package and party package is well described in both CRAN files, vignettes and publications, they don't mention much about the x-axis scale or implications, other than that gbm normalizes the scale to 100. I obtained these results from random forest computed with party package (cforest function):\n\nSo the x-axes ranges from 0 to 0.03, which seems strange to an amateur like me. I guess these scales are arbitrary in a sense, but should I be worried? Does the scale matter at al?\n", "answer": "For the variable importance in the party package help(\"varimp\", package = \"party\") has a short explanation.\n\nValue: A vector of 'mean decrease in accuracy' importance scores.\n\nMore details and the exact formulas can be found in Strobl et al. (2008). \"Conditional Variable Importance for Random Forests.\" BMC Bioinformatics, 9(307). doi:10.1186/1471-2105-9-307\nHowever, the values of these importances may also change with the type of permutation (conditional vs. unconditional), the evaluation set (in-bag vs. out-of-bag), etc. And sometimes the importances are rescaled as you point out. So it is not always possible to exactly compare the numeric values between different packages/functions.\n", "topic": "stats", "url": "https://stats.stackexchange.com/questions/329995"}, {"image": "VQAonline_00061674.png", "question": "What is an analytical expression for so called value function $V^{\\pi}$ in reinforcement learning?", "context": "One thing that struck me as odd when going through literature on reinforcement learning is how \"theoretical\" everything is. I expected the results in this field to be things that you can immediately punch into Python or MATLAB and get your hands dirty, but I am seeing most things in reinforcement learning is not expressed are an analytical fashion (and poorly defined - but maybe I am early on in the book).\nFor instance, this so called value function\n\nhttps://webdocs.cs.ualberta.ca/~sutton/book/ebook/node34.html\nBut what is $E_\\pi$? I cannot punch $E_\\pi$ into MATLAB, it is not a function that I am familiar with, so I must get rid of it immediately to arrive at an analytical expression that I can evaluate.\nIf $\\pi(s,a)$ is a probability distribution as I suspect it is, then using the expression of an expectation for a discrete random value, https://en.wikipedia.org/wiki/Expected_value, $V^\\pi(s)$ should be expressed as:\n$$V^\\pi(s) = \\sum\\limits_{a = 1}^n \\pi(s,a)\\left(\\sum\\limits_{k =0}^\\infty \\gamma^k r_{t+k+1}\\right)$$\nCan someone check if I have the expression for the value function at state $s$ correctly?\n", "answer": "This is answered in the Sutton page you link to, emphasis mine:\n\nRecall that a policy, $\\pi$, is a mapping from each state, $s$, and action, $a$, to the probability of taking action $a$ when in state $s$. Informally, the value of a state $s$ under a policy $\\pi$, denoted $V^{\\pi}(s)$, is the expected return when starting in $s$ and following $\\pi$ thereafter.\n\nExpressing the expectation isn't that simple, because subsequent rewards depend on subsequent transitions from state to state. This is why Sutton writes, \"A fundamental property of value functions used throughout reinforcement learning and dynamic programming is that they satisfy particular recursive relationships.\" \nCheck equation 3.10 on that same page for an expression that expresses this relationship.\n", "topic": "stats", "url": "https://stats.stackexchange.com/questions/269265"}, {"image": "VQAonline_00061494.png", "question": "ANN regression, linear function approximation", "context": "I have built a regular ANN\u2013BP setup with one unit on input and output layer and 4 nodes in hidden with sigmoid. Giving it a simple task to approximate linear f(n) = n with n in range 0-100. \nPROBLEM: Regardless of number of layers, units in hidden layer or whether or not I am using bias in node values it learns to approximate f(n) = Average(dataset) like so:\n\nCode is written in JavaScript as a proof of concept. I have defined three classes: Net, Layer and Connection, where Layer is an array of input, bias and output values, Connection is a 2D array of weights and delta weights. Here is the Layer code where all important calculations happen:\nAnn.Layer = function(nId, oNet, oConfig, bUseBias, aInitBiases) {\nvar _oThis = this;\n\nvar _initialize = function() {\n _oThis.id = nId;\n _oThis.length = oConfig.nodes;\n _oThis.outputs = new Array(oConfig.nodes);\n _oThis.inputs = new Array(oConfig.nodes);\n _oThis.gradients = new Array(oConfig.nodes);\n _oThis.biases = new Array(oConfig.nodes);\n\n _oThis.outputs.fill(0);\n _oThis.inputs.fill(0);\n _oThis.biases.fill(0);\n\n if (bUseBias) {\n for (var n=0; n0; n--) {\n if (n === nLast) {\n this.layers[n].calculateGradients(aTarget);\n } else {\n this.layers[n].calculateGradients();\n }\n }\n\n for (n=nLast; n>0; n--) {\n this.layers[n].updateInputWeights();\n this.layers[n].updateInputBiases();\n }\n}\n\nConnection code is rather simple:\nAnn.Connection = function(oNet, oConfig, aInitWeights) {\nvar _oThis = this;\n\nvar _initialize = function() {\n var nX, nY, nIn, nOut;\n\n _oThis.from = oNet.layers[oConfig.from];\n _oThis.to = oNet.layers[oConfig.to];\n\n nIn = _oThis.from.length;\n nOut = _oThis.to.length;\n\n _oThis.weights = new Array(nIn);\n _oThis.deltaWeights = new Array(nIn);\n\n for (nX=0; nX let's go with a mix of coal+gas as the worst case scenario.\n\nIs there an easy way how I can calculate the amount of extra diesel-fuel the ferry will consume when 2 tons of extra load are added on board?\nTime savings are not a factor; environmental gains are more important to me. I'm interested in the overall environmental impact (CO2 + toxic air pollution both local and global). And I'm looking for the most accurate answer possible.\nMotivational image:\n\n", "answer": "Take the ferry!\nThe short ferry trip likely results in lower emissions of CO2 compared to the long detour, if the EV is charged from a grid that uses fossil fuels (coal/gas).\n\nI think there are actually three different change scenarios worth considering here.\n\nMarginal: You change your route, but nobody else does. (This is looking at the problem from an individual \"carbon footprint\" perspective).\nGroup: a significant number of people all change their route.\nSystem: the ferry is shut down entirely.\n\nMarginal Scenario (one more car on the ferry)\nFrom my reading I've learned there are several factors that significantly impact fuel usage and emissions on passenger/vehicle ferry transports (eg. trip frequency, speed, prevailing winds, vessel size, fuel type) but number of vehicles on board is not a significant factor. (disclosure: I am not a marine engineer.)\nAlthough I think your assumption about ship's fuel use (below) is intuitively correct, it's such a small factor that it doesn't make a significant difference and cannot be reasonably measured.\n\nHowever this ferry will carry +2 tons of cargo (car+me) which will consume more fuel.\n\nAlthough it's tempting to use a measure of fuel use like litres/passenger-km to estimate the impact of a single trip, the reality is that the actual carbon emissions and air pollution don't really change in any significant measurable way whether you board the ship or not.\nTherefore, you can assume approximately zero emissions for the diesel ferry. On the other hand, natural gas peaking plants are designed to scale power output up or down based on power demand from the grid, so using electricity to charge your EV likely does result in a very slight marginal increase to carbon emissions and air pollution.\nWinner: Diesel Ferry\nGroup Scenario (many more cars on the ferry)\nVessel speed and number of return trips have a very significant impact on fuel usage. If many more people decide to take the ferry, the ferry may need to perform more round trips and/or operate at a higher sailing speed. (Although sailing speed probably doesn't matter much for a very short-haul trip like the river crossing you're asking about.)\nIf the ferry needs to make an additional round trip to handle increased demand, then we can distribute the fuel usage across the group using a measure like litres/passenger-km. Luckily there's a study we can refer to: Comparison of Ferry Boat and Highway Bridge Energy Use.\n\nExaminations of three of the systems found that the passenger-MPG of the ferries ranged from 2.61 to 14.00 (1.11 to 5.95 km/L) [0.90 L/km to 0.16 L/km]\n\nOn a per-passenger basis, your 0.16 km river crossing would use between 0.0256 - 0.14 litres of diesel fuel. Diesel engines produce 2.7 kg of CO2 per litre of diesel fuel consumed.1 So a single trip by diesel ferry would result in around 0.07 - 0.38 kg of CO2 emissions.\nElectric passenger vehicles typically use around 20 kWh/100 km, so your 15 km drive would use around 3 kWh. Natural gas peaking plants emit 0.91 pounds2 (0.41 kg) of CO2 per kWh produced, so the 15 km drive would result in 1.23 kg CO2 emissions.\nWinner: Diesel Ferry\nSystem\n[Maybe I'll add this in the future. This answer is already quite long and I have to stop for now.]\n", "topic": "sustainability", "url": "https://sustainability.stackexchange.com/questions/11808"}, {"image": "VQAonline_00063856.png", "question": "Why are my LED light bulbs so short lived?", "context": "I thought the idea of LED is, that the light bulb has a very long life and uses very little electricity.\nI have not invested in full LED lighting but tried the LED bulbs, you screw in your normal sockets and I have to replace them pretty frequently.\nThey also tend to lose a lot of their performance over time.\nSpecifically, I have had something like that in my house:\n\nBut I have also tried a lot of other bulbs, some of which were LED. I always have the same problems with all new school bulbs:\n\nWhen you switch the light on, it takes time to reach its full brightness (this problem seems to occur less in recent times)\nThe light gradually loses its brightness, which is maddening and forces you to replace the bulb, before its broken. The maddening part is, that it is never the same level of brightness, you are either on a continuous progress to less bright, or make a big jump in brightness when you replace the bulb.\nThey cost a ton of money. I could live with that if they'd perform better.\nThey never seem to achieve the warm light of a 100W old school bulb, even if you buy the expensive stuff.\n\nSo, what am I doing wrong here? Do I have wrong expectations or am I buying the wrong equipment?\n", "answer": "Consumer tests have shown that there are good and bad LED lights; indeed some of them do not reach their advertized life span.\nI also have some bad experiences with LEDs dying long before their advertised end-of life, but that's anecdotes, not evidence.\nYou can imagine that actually testing these is hard, it takes a long time!\nA major Dutch consumer organization has been testing LEDs since at least 2013 and they report:\n\nOur latest extra long life test shows that 75% of the LED lamps from a sample reach 20,000 burning hours. At the end of 2014, we reported that 80% of the lamps from that sample achieved 15,000 burning hours.\n\nSo, 20% die before 15000 hours, 25% before 20000 hours.\nA detailed 2015 breakdown of brands and models is here.\nI would add: buy from regular well-known brands and avoid cheap non-name products.\nAs for diminishing intensity they state:\n\nOur test shows that there is often some deterioration of the amount of light, but usually it remains limited. If a lamp emits 80% of the initial quantity after 15,000 hours, we consider it as: still working well.\n\nSome tips they give:\n\nPreferably do not use a LED lamp in a closed luminaire. It won't be able to get rid of its warmth and will soon be broken.\n\nSo this may be something specifically to watch out for. Also:\n\nBecause there can be a lower quality lights in a batch (for any brand), our advice remains: keep the proof of purchase and the box of LED lights.\n\nWhether that last thing helps you depends on the consumer laws in your country. \nNote also that irregular voltages and spikes can have an impact:\n\n... these high-tech lights have very sensitive electronics in them that can be damaged permanently by a spike in electricity. \n\nFinally, LED lights often do not only show a guaranteed number of hours on the packaging, but also a number of on/off switches. Excessive (automatic?) switching may reduce their life time.\n", "topic": "sustainability", "url": "https://sustainability.stackexchange.com/questions/6990"}, {"image": "VQAonline_00063840.png", "question": "Aquaponic system doesn't grow the plants", "context": "I have created aquaponic system and tried to plant tomatoes, cucumbers and some other herbs. They sprout without problem but after couple of days they dried out. I tried it 2 times with the same result.\nThe water circulates 15 min every hour. I'm not using siphon setup but water flow one. At the bottom I have a gravel to filter it and at the top I use clay pellets in which I seeded the seeds.\nThe white drain pipe goes up to where the clay pellets and the gravel meets. So the seeds are not constantly in the water but absorb the moisture from the pellets. Any idea how to make it work? I'm planning to give up and just to plant some flowers meant for ponds.\n\n", "answer": "(This is only meant as an extended comment. I hope someone with aquaponics experience will be able to answer your question. I have only seen much bigger aquaponics set-ups, outside.)\nI unfortunately don't have any experience with aquaponics, but it looks like your system is inside. My experience with tomatoes is that they need lots of warmth and light. My tomatoes I grow at 30 \u00baC outside in the sun. On top of that I've been told that tomatoes get sweeter with less water, so I only water them once a day. (I think that cucumbers (like melons), also need light and warmth, and depending on your herbs, they, too might need light and warmth.)\nSo, I think that a problem with your setup might be not enough sunlight. (Seeds do sprout without sunlight, but as soon as they produce leaves, they need enough light to grow larger.)\n", "topic": "sustainability", "url": "https://sustainability.stackexchange.com/questions/4529"}, {"image": "VQAonline_00063851.png", "question": "How to clean a reusable plastic coffee cup, such as KeepCup?", "context": "I've owned a KeepCup for about a year and though I clean it after use it really is starting to stink.\nI've tried scrubbing it with a scouring pad and detergent and leaving it to soak overnight, but it still smells bad.\nThe cup looks like the one below. All pieces can be disassembled to make cleaning easy. It's like the smell is in the plastic it's self.\n\n", "answer": "Wash it in a dishwasher once every month or two. I own a Contigo mug, and once had similar problems with the plastic/rubber lid. I found some tips on their website, but ultimately found that if I washed the mug and lid in the dishwasher about once a month, the smell and staining would go away. I don't have a dishwasher, but find that I visit someone who does often enough to keep the smell at bay.\nKeepCup also includes cleaning tips on their website, which you may want to try first if you don't have access to a dishwasher.\nAlternatively, KeepCup offers a one year warranty, so if you've owned the cup for less than a year you may want to try contacting them.\n", "topic": "sustainability", "url": "https://sustainability.stackexchange.com/questions/5812"}, {"image": "VQAonline_00063852.png", "question": "Why does cheese have such a high carbon footprint?", "context": "\n(Source of information here)\nAs you can see, cheese has the third highest carbon footprint. Only beef and lamb have a higher footprint.\nSeveral other sources (such as this) corroborate this.\nWhy is it so high? You would expect it to be due to the milk production, but milk itself it one of the lowest on the chart.\n", "answer": "TL;DR\nThe chart is misleading since it compares carbon imprint by mass instead of a measure of how much a human needs to survive.\nArgument\nThe chart you link contains false comparisons. They are comparing mass of foods against each other. However, you don't eat for mass, you eat for calories (or protein or nutriment, or whatever). A better comparison would be to multiply each 1 kg of food by the calories in that food. \nI got my numbers from here, with data sourced from USDA. The numbers in my chart below are kilograms carbon per 1000 calories:\nLamb 20.85\nBeef 13.78\nTurkey 5.83\nBroccoli 5.71\nTuna 5.26\nSalmon 5.15\nCheese 4.47\nPork 4.45\nYogurt 3.49\nChicken 3.37\nMilk 3.17\nEggs 3.06\nRice 2.08\nPotatoes 1.46\nBeans 1.40\nTomato 1.39\nTofu 1.38\nLentils 0.78\nPeanut Butter 0.42\nNuts 0.39\n\nI used the cooked option of each food when available. For the numbers with some options, I used whole milk, pinto beans, plain lowfat yogurt, almonds for the nuts, and part-skim mozarella cheese. \nMy first conclusion, is that Cheese doesn't have such a high footprint after all. It is just a little higher than Yogurt, which is just a little higher than Milk, which is about even with eggs. All the animal products dominate the top of the list.\nMy other conclusions are that this is kind of a garbage comparison anyways. Broccoli doesn't provide calories, but it has more vitamin K than everything else on the chart combined. The peanut butter and nuts are really calorie efficient, but so what? If you try to live on peanut butter instead of rice you'll die of heart disease because you are getting 70% of your calories from fat and three times the recommended saturated fat dosage.\n", "topic": "sustainability", "url": "https://sustainability.stackexchange.com/questions/5883"}, {"image": "VQAonline_00063862.png", "question": "Does 1 container ship really pollute as much as 50,000,000 cars?", "context": "I remember seeing many news articles and posts on social media saying that a single container ship pollutes as much as 50 million cars. Is this true? How could that be possible?\n\n\nhttps://tech.slashdot.org/story/10/11/23/1618229/one-giant-cargo-ship-pollutes-as-much-as-50m-cars\nhttps://newatlas.com/shipping-pollution/11526/\nhttps://www.greencarreports.com/news/1020063_pollution-perspective-one-giant-cargo-ship-emits-as-much-as-50-million-cars\nhttps://go.enfos.com/blog/2015/06/23/behemoths-of-emission-how-a-container-ship-can-out-pollute-50-million-cars\n\n", "answer": "It depends on how you define \"pollution\".\nCargo ships use some of the dirtiest fuels available: bunker fuel is basically what's left over after you refine all the good stuff out of crude oil. It's got all sorts of contaminants that something like gasoline or jet fuel doesn't have. On top of that, there are basically no regulations on emissions by cargo ships. This results in very high emissions of nitrogen and sulfur oxides, while cars emit almost none -- a ratio of 50 million to 1 is not unreasonable.\nCarbon dioxide emissions are a different story. Cargo ships are the most fuel-efficient way of moving things from one place to another, while passenger cars are the least. Per ton of cargo moved, cargo ships are between a hundred and a thousand times more efficient than cars, depending on how heavily-loaded the car is.\n", "topic": "sustainability", "url": "https://sustainability.stackexchange.com/questions/10757"}, {"image": "VQAonline_00063866.png", "question": "What is the difference between energy-from-waste (EFW) and cogeneration?", "context": "What is the difference between energy-from-waste (EFW) and cogeneration, and why is the latter more efficient (let's compare electricity to electricity)? Aren't both essentially incineration?\nSee below from pages 9 and 12 (respectively) from \"The Environmental Impact of Paper Waste Recycling: A Comparative Study\" (emphasis added):\n\n\nIn fact, due to the energy requirements of paper production, combustion of\nunusable fibre can be used even more effectively when integrated into milling. Because a significant portion of the energy required for paper production is thermal, combustion of waste paper can be used directly to fuel the process. Wood fibre cogeneration plants, which generate both electricity and useable thermal energy, obtain efficiencies of 25-30% for electrical generation (mean of 27.5%), and up [to] 75% overall when heat is recovered (FAO 1990). So if the average thermal content of paper is assumed to be 17.3 GJ/T (Morris 1996), a maximum of 2.60 GJ/T could be recovered as electricity through EFW, but 4.76 GJ/T could be recovered through cogeneration, plus an additional 8.23 GJ/T in thermal energy (see Appendices 2, 3). This represents an improvement of 2.16 GJ/T in electrical production, and 10.4 GJ/T overall.\n\n", "answer": "Cogeneration is a more specialized process with specific fuel requirements which is used to produce electric and heat energy. Energy from waste is a general process whereby any mix of waste is burned to produce electric energy only. This is explained further in the paper. From page seven:\n\nIt is important to note the efficiency level of EFW generation. Because these\nplants operate by burning all household wastes: organics, plastics, glass, aluminum, etc. as well as paper, it is difficult to optimize plant design for generation due to the variability of the combustibles (Ekvall and Finnveden 2000). As a result of this, and the normal inefficiencies involved in energy conversion, burning garbage to produce\nelectricity is able to capture only 15% of the waste\u2019s intrinsic energy (Morris 1996),\nsignificantly lower than other fuels (such as coal, which averages approximately 38%\nefficiency) (Taylor et al. 2008). This could be improved if further sorting were carried\nout in the waste-fuel stream, with specific incinerators for different fuels, but this would largely undermine the simplicity advantage conferred by such a catch-all disposal\nmethod.\n\nNote that when cogeneration is introduced in the quote included in the question it is referred to as wood fibre cogeneration, meaning that the authors are looking at one specific type of cogeneration, and comparing it to a broad array of EFW processes.\n", "topic": "sustainability", "url": "https://sustainability.stackexchange.com/questions/11991"}, {"image": "VQAonline_00064050.png", "question": "Where can I find a list of cities with public bicycles available in Europe?", "context": "Whenever I visit a city I find traveling by bicycle very convenient. It's very flexible and you get to see the city in a very friendly way.\nI was wondering if there is a website or other resource that lists cities in Europe where public bikes are accessible also to tourists.\nNote: I am not referring to companies renting bikes. I am really referring to public systems like exist in Paris or Copenhagen where everyone can use bikes.\nIf the list contains the requirements to use the bicycles that would be a big plus. Eg.: In Paris, if I remember right, you need a Credit Card.\n\n", "answer": "Wikipedia is your friend with an entry on Bicycle sharing systems.\n", "topic": "travel", "url": "https://travel.stackexchange.com/questions/30406"}, {"image": "VQAonline_00064490.png", "question": "In the Netherlands, does the green stripe in the middle of a road always allow to drive 100 km/h?", "context": "In the Netherlands, there are roads with a green stripe in the middle. Here an example from Google Street View: \n\nBoth times I was driving on such a road there was an autoweg (expressway) sign at the beginning of that section allowing to drive at 100 km/h. \nDoes this green stripe always mark an autoweg, e.g. can I be sure that if there is a green stripe, then it is allowed to drive 100 km/h?\n", "answer": "Apparently, normally yes, if there is also a continuous line on the outsides of the roadway.\nThe green area is called a Optische middenberm, which means optical central verge. When I learned driving I was taught it had no other function, but that appears to be untrue (either I remember wrongly, or I was misinformed, or it has changed; I obtained my license in January 2007 and moved out of the country 8 months later). According to Rijkswaterstaat, which is the official road authority in The Netherlands:\n\nDe groene kleur tussen twee doorgetrokken strepen geeft aan welke snelheid er gereden mag worden. Bij een dubbele doorgetrokken streep met een groen vlak ertussen, is de maximumsnelheid 100 kilometer per uur, mits [sic] anders aangegeven.\n\nWhich means:\n\nThe green colour between two continuous lines indicates what speed is permitted. In case there is a double continuous line with a green area in between, the maximum speed is 100 kilometres per hour, if [sic] otherwise indicated.\n\nNB: the use of mits (if) is incorrect, and should be tenzij (unless).\nIt goes on to note:\n\nOp wegen waar 100 gereden mag worden is de lijn aan de buitenkant van de rijbaan doorgetrokken, op wegen waar maximaal 80 gereden mag worden is de buitenste lijn onderbroken.\n\nwhich means:\n\nOn roads where driving 100 [km/h] is permitted, the line at the outside of the roadway is continuous, on roads where the speed limit is 80 km/h the outer line is interrupted.\n\nSo: yes, the green area between two continuous white lines does mean the speed limit is 100 km/h, if there are also continuous lines on the outside. This applies if nothing else is indicated, any locally posted speed limit (permanent or temporary) is still binding (naturally).\nNote that it is called a autoweg and not a snelweg. A snelweg is a motorway/freeway with at least two lanes in each direction, fully segregated roadways, speed limits up to 120 km/h or recently 130 km/h, no at-grade intersections, and some other limitations not applying to autoweg. And just as a reminder: both autoweg and snelweg can have locally posted speed limits, either permanently or temporarily, that the posted speed limit remains the legal limit, regardless of what lines are drawn on the road.\n(NB: I deleted my previous, incorrect answer to this question)\n", "topic": "travel", "url": "https://travel.stackexchange.com/questions/123560"}, {"image": "VQAonline_00063877.png", "question": "Name this church in Moscow, please?", "context": "I'm trying to type up my notes of Moscow, and can't seem to find the name of this church I took a photo of. After using google image search, however, I refound a photo of it at this location so have a latitude and longitude there, but no name. Can anyone help with this treasure hunt? :)\n\n", "answer": "Here's my guess...\nTrinity Church, in Serebryaniki\nSource: http://www.panoramio.com/photo/6837647 (translation)\n", "topic": "travel", "url": "https://travel.stackexchange.com/questions/1435"}, {"image": "VQAonline_00064686.png", "question": "UK Visitor visa to visit my girlfriend refused under V4.2 (a) (c) & (e)", "context": "\nHello,\nI'm new here (and to the rules) and apologise for any errors in posting or asking questions which may not fit. In any case please let me know of anything wrong.\nI have read almost all the previous great answers on TSE, especially from Gayot Fow (RIP), regarding my situation i.e UK visa refused under V4.2 (a),(c) & (e). These answers helped me understand some flaws in my original application & I am almost ready with my second application (with about 6 pages of explainer cover letter & additional financial proofs).\nQuestions below are asked particularly in context of my application(s) but may be generalised by others. I would also love your suggestions.\n1. My refusal letter (from mid-september) only states -\n\n\u201cAny future UK visa applications you make will be considered on their\nindividual merits, however you are likely to be refused unless the\ncircumstances of your application change\u201c\n\nI saw other letters of refusal here mention\n\n\u2018unless additional evidence is provided\u2019\n\n\n\u2018unless your circumstances change significantly\u2019\n\nQuestion Set 1 -\n\nWhat does this really mean in my case?\nIs there a general 'severity estimate\u2019 based on the different 'languages' used above?\nWill they even consider my additional evidence & explanations?\nHow bad is my refusal vis-\u00e0-vis reapplication relevance? Is it even worth making a re-application?\n\n\n2. There was an error in calculating the value of my fixed deposits by the ECO. The deposit statement I had provided did not mention any totals and thus ECO computed the amounts from Page 2 in their refusal letter which is only a third of my actual deposits, while they completely ignored Page 1. I read here that 4.2(e) is mostly an icing on top of more severe 4.2 (a) & (c)\nQuestion Set 2 -\n\nWill apprising them about this oversight in calculation (which I must bring up) improve my chances even slightly?\nWhat is the best way to communicate this in my new application?\n\n\n3. Seems they mainly refused me on 4.2 (a) & (c), I assume it was because of my profile - a 30+yo single Indian male, freelance consultant, possible to work & earn remotely in the UK, asking for 4 months to visit his long-term girlfriend. I can see how I fit the refusal profile from UKVI \u2019s point of view.\nI\u2019ve now reduced my visit duration to 2 months (already lost 2 months due to refusal) in my new application, and attaching confirmed return tickets (which I also did last time) but I guess it won\u2019t make a difference. We\u2019ve literally been apart 13 months and I honestly wish to spend 60 days with her. Historically we\u2019ve only managed to have 3 week long vacations together, but want to spend more time than that when it is finally possible. Is it too much to humanly ask?\nQuestion Set 3 -\n\nWhat are my other options to show ties to the country? I had already attached property details (owned jointly with my mother) to show my ties to land & family, I can explain a bit more but I can\u2019t think of any other positive evidence to show. What do I do?\n\nI have a small bank loan on my apartment, which I didn't mention explicitly, but my expenses reported during application and bank statement show these monthly instalment debits. Do I disclose this in my new application in case it adds value to my ties or acts positively?\n\nHow do I prove I am a genuine visitor if I gave them pretty much all the evidence I have already. I can give evidence of my additional income, income being genuinely derived from my profession (tax statements from the government) and the deposits resulting from the income, but that only answers the V4.2(e); points 4.2 (a) & (c) still remain.\n\n\nI\u2019m just stressed that in my opinion I can give them all genuine reasons (and some new financial evidence) but I\u2019m afraid I\u2019ll fall short to show my intent of being a genuine visitor. Any pointers to improve my application outcome will be super-welcome.\nThanks in advance.\nEDIT (for gratitude): Thank you everyone for your help with the answers & comments, specifically @Traveller & @DJClayworth. It has given me a lot of information & other person's perspective to improve my next application. For me, both answers are equally great & sensible, however in order to mark one as 'accepted' I'll just go by the upvotes. Cheers!\n", "answer": "Lets deal with question set 1 first. All of the statements in the refusal are aiming to tell you \"You are welcome to apply again, but unless you fix the problems the next application will be rejected too.\" You can't just repeat the application and hope for a different result. It's standard statements that are always made. Some people think it's like a lottery, and if they just submit the same application again they might win this time.\nFor the second question set, the calculation error is not a big issue. It's not worth trying to get the decision overturned because of it. You can point it out in your next application, in notes.\nNow for the questions you didn't ask. There are two serious issues with your application:\n\nThe officer can't find evidence to back up your statements about income\nThere are unexplained deposits in your accounts.\n\n(The unexplained deposits may be from your income but the officer can't verify that).\nYou need to find better evidence that the deposits in your accounts actually do come from your work. For example, if your invoices show Customer X paid you a certain amount for some services, then there should be a deposit to your account for that amount from Customer X. Also any substantial deposits need to have an explanation. Any amounts you received that are unexplained will be treated with suspicion. People sometimes try to \"pad\" their bank balances by borrowing money, and any unexplained deposit may look like that to the officer.\nYour comments about \"the profile\" are reasonable. You do indeed look very like the kind of person who would fail to leave the UK after your visit. You have, on paper, a huge incentive to do that - a girlfriend in the UK, a job you can probably do remotely, no other ties to your home country. (Joint ownership of property is of little value - you could continue to own it and receive income from the UK.)\nBut the really important question you haven't answered for them is \"How can you afford to take four months off work?\" Remember that you are not allowed to work, even remotely, while in the UK. Taking that time off, on top of the cost of flying to the UK, would wipe out almost all of your savings. Most people can't afford to have no income for four months. And if you can live without income for four months (for example because your girlfriend is earning enough money for both of you), then maybe you can live without income for twelve months, in which case you have another incentive to overstay your visa.\n", "topic": "travel", "url": "https://travel.stackexchange.com/questions/169226"}, {"image": "VQAonline_00063938.png", "question": "What cityscape is this picture of?", "context": "I have come across this cityscape on the web and I wonder what city it is. Does anyone know?\n\n", "answer": "Definitely Hong Kong, with Victoria Peak in the background.\nMost likely taken from somewhere around North Point - based on the angle I'd say from the top of one of the high-rise buildings in that area.\nThe original photo is posted on Flickr, although without any details of exactly where it was taken. The same photographer has some other fairly impressive similar photos as well!\n", "topic": "travel", "url": "https://travel.stackexchange.com/questions/6979"}, {"image": "VQAonline_00064488.png", "question": "How to wash hands if tap water is too hot?", "context": "A few days ago I settled in the UK and noticed that hot tap water was not specific to Heathrow Airport. The only option in some toilets (e.g. in restaurants) is \"tooooo hot\" water. I cannot touch so hot water for even one second, otherwise it is painful.\n\nHow can I wash my hands if hot water is the only option?\n", "answer": "As a British person I can say in this instance I just splash my hands, or move them in and out of the water stream very quickly. I would do in the winter when it is very cold, too, even when both taps are present, because splashing quickly with too hot water is often preferable to ice cold water.\nAlso try using a different sink, because water usually cools in the pipes and/or takes a while to heat up, so you can often get a few seconds of cooler water if nobody has used the sink for a few minutes.\n", "topic": "travel", "url": "https://travel.stackexchange.com/questions/123392"}, {"image": "VQAonline_00063893.png", "question": "Where can you go lake ice skating in Southern Sweden?", "context": "Winter is coming, and if the Swedish Meteorologic and Hydrologic Institute (SMHI) is to be believed, it is going to be even colder than last year. \nFor reference, last year in Sweden was one of the coldest on record -- lake V\u00e4ttern froze in a radius of 30 km or so around J\u00f6nk\u00f6ping, and I've seen speed skaters crossing the waters from Bankeryd to Huskvarna -- a good 5-10 km run. I tried skating myself, for the first time in my life, and that.. didn't end too well, but it was tons of fun.\n\nImage source - my private collection\nThis year, I'm much more prepared, and since it's so much more fun than a regular ice rink, I was wondering which lakes in southern Sweden (everything in a radius of 300 km around J\u00f6nk\u00f6ping) freeze routinely and are used for ice skating in the winter. \nSince my friends and I are fairly new to this, I'd like to stick to more frequented lakes, to be able to get assistance if needed.\n", "answer": "The site skridsko.net is crowd sourcing information about skating in all regions of sweden. You can search there when it starts to get cold enough. I live in the Stockholm area and here municipalities provide with skating lanes on a couple of lakes and also have information on their home pages about the avilability and ice quality. The lake Norrviken north of Stockholm is a good choice for newcomers to the game. You can choose between 3.5, 7, 10.5 and 14 kms tracks. The very best - of course - is when there hasn't been any snow yet and you can skate all over the lake but that is very seldom. \nI would guess that v\u00e4ttern - a very deep lake - is less likely to freeze over compared to other lakes. Many winters there is no skating at all in southern sweden but the last 2 ones have been exceptional both for skiing and skating.\n", "topic": "travel", "url": "https://travel.stackexchange.com/questions/2998"}, {"image": "VQAonline_00064742.png", "question": "How to pronounce `\u0449`?", "context": "I've heard \u0449 to be pronounced in Ukrainian exactly like it pronounced in modern standard Russian (as a single phoneme) or like /\u0283t\u0361\u0283/ (shch) - this screenshot is some kind of evidence that it also can be pronounced like /\u0283t/\n\nMy question would be - is there a single acceptable form of pronouncing \u0449. If yes, which one? If no - how acceptable are alternatives (like acceptable in colloquial speech, not acceptable at all etc.)\n", "answer": "No, a commercial software can't serve an evidence of pronunciation.\nEspecially, if it is about toponyms, which are usually transliterated, not transcribed (see the difference between the two).\nIn Standard Ukrainian, \u00ab\u0449\u00bb is pronounced as a consonant cluster, unlike the standard Russian:\n\nUA: [\u0283 t\u0361\u0283]\nRU: [\u0255\u02d0]\n\nIn Eastern Ukraine, where many people speak surzhyk sociolect, if it easy to hear the [\u0255\u02d0] instead. Assuming the number of speakers, someone may consider it a dialect already.\nMore phonetic differences are listed here.\nAnd no, \u00ab\u0449\u00bb is not the same thing as \u00ab\u0448\u0442\u00bb [\u0283t] (however, there is some historic relation). There's even a pair of words to see the difference:\n\n\u0449\u0443\u043a\u0430 \u2014 a pike fish\n\u0448\u0442\u0443\u043a\u0430 \u2014 a thing\n\nAs usual, adding dialectal words/pronunciation may add some emotional flavor to one's speech. It can be a double-edged weapon however, so a language learner should probably strive to keep the Standard language unless they master the language well enough.\n", "topic": "ukrainian", "url": "https://ukrainian.stackexchange.com/questions/2677"}, {"image": "VQAonline_00064741.png", "question": "\u042f\u043a \u043f\u0440\u0430\u0432\u0438\u043b\u044c\u043d\u043e \"\u0431\u043b\u043e\u043a\u043e\u0432\u0438\u0439\" \u0447\u0438 \"\u0431\u043b\u043e\u0447\u043d\u0438\u0439\"", "context": "e2u \u0432\u0456\u0434\u0434\u0430\u0454 \u043f\u0435\u0440\u0435\u0432\u0430\u0433\u0443 \u0431\u043b\u043e\u043a\u043e\u0432\u0438\u0439 \u043f\u0435\u0440\u0435\u0434 \u0431\u043b\u043e\u0447\u043d\u0438\u0439\n\n\nblock = [bl\u0252k] 1. \u0431\u043b\u043e\u043a; \u0433\u0440\u0443\u0301\u043f\u0430; \u0432\u0443\u0301\u0437\u043e\u043b || \u0431\u043b\u043e\u0301\u043a\u043e\u0432\u0438\u0439 || \u0440\u043e\u0437\u0431\u0438\u0432\u0430\u0301\u0442\u0438/\u0440\u043e\u0437\u0431\u0438\u0301\u0442\u0438 \u043d\u0430 \u0431\u043b\u043e\u0301\u043a\u0438, \u043e\u0431\u2019\u0454\u0301\u0434\u043d\u0443\u0432\u0430\u0442\u0438/\u043e\u0431\u2019\u0454\u0434\u043d\u0430\u0301\u0442\u0438 \u0432 \u0431\u043b\u043e\u0301\u043a\u0438,\n \u0437\u0432\u043e\u0301\u0434\u0438\u0442\u0438/\u0437\u0432\u0435\u0301\u0441\u0442\u0438\u0301 \u0434\u043e \u0431\u043b\u043e\u0301\u043a\u043e\u0432\u043e\u0457 \u0444\u043e\u0301\u0440\u043c\u0438 // in ~s \u0433\u0440\u0443\u0301\u043f\u0430\u043c\u0438; \u0431\u043b\u043e\u0301\u043a\u0430\u043c\u0438\n 2. \u0431\u043b\u043e\u043a\u0443\u0432\u0430\u0301\u0442\u0438/\u0437\u0430\u0431\u043b\u043e\u043a\u0443\u0432\u0430\u0301\u0442\u0438\n\n\u0410\u043b\u0435 \u0432 \u0456\u043d\u0442\u0435\u0440\u043d\u0435\u0442\u0456 \u043c\u043e\u0436\u043d\u0430 \u0431\u0430\u0447\u0438\u0442\u0438 \u043e\u0431\u0438\u0434\u0432\u0430 \u0432\u0430\u0440\u0456\u0430\u043d\u0442\u0438. \u042f\u043a \u043f\u0440\u0430\u0432\u0438\u043b\u044c\u043d\u043e?\n", "answer": "\u0421\u043b\u043e\u0432\u043d\u0438\u043a \u0423\u041c\u0406\u0424 \n\n\u0431\u043b\u043e\u043a\u043e\u0432\u0438\u0439 (\u0432\u0456\u0434: \u0431\u043b\u043e\u043a \u2014 \u0443\u0433\u043e\u0434\u0430)\n\u0431\u043b\u043e\u0447\u043d\u0438\u0439 (\u0432\u0456\u0434: \u0431\u043b\u043e\u043a \u2014 \u043c\u0435\u0445\u0430\u043d\u0456\u0437\u043c)\n\n\n\u041e\u0440\u0444\u043e\u0433\u0440\u0430\u0444\u0456\u0447\u043d\u0438\u0439 \u0441\u043b\u043e\u0432\u043d\u0438\u043a \u0443\u043a\u0440\u0430\u0457\u043d\u0441\u044c\u043a\u043e\u0457 \u043c\u043e\u0432\u0438\n\n\u0431\u043b\u043e\u043a\u043e\u0432\u0438\u0439 (\u0432\u0456\u0434: \u0431\u043b\u043e\u043a \u2014 \u043c\u0435\u0445\u0430\u043d\u0456\u0437\u043c, \u0443\u0433\u043e\u0434\u0430)\n\u0431\u043b\u043e\u0447\u043d\u0438\u0439 (\u0432\u0456\u0434: \u0431\u043b\u043e\u043a \u2014 \u043c\u0435\u0445\u0430\u043d\u0456\u0437\u043c, \u043f\u043b\u0438\u0442\u0430)\n\n\n\u0422\u043b\u0443\u043c\u0430\u0447\u043d\u0438\u0439 \u0441\u043b\u043e\u0432\u043d\u0438\u043a \u0443\u043a\u0440\u0430\u0457\u043d\u0441\u044c\u043a\u043e\u0457 \u043c\u043e\u0432\u0438 \u0443 20 \u0442\u043e\u043c\u0430\u0445\n\n\u0431\u043b\u043e\u043a\u043e\u0432\u0438\u0439\n\n\n\u041f\u0440\u0438\u043a\u043c. \u0434\u043e \u0431\u043b\u043e\u043a1. \u0411\u043b\u043e\u043a\u043e\u0432\u0438\u0439 \u0441\u043f\u043e\u0441\u0456\u0431 \u0441\u043a\u0438\u0440\u0442\u0443\u0432\u0430\u043d\u043d\u044f.\n\u0441\u043f\u0435\u0446. \u041f\u0440\u0438\u043a\u043c. \u0434\u043e \u0431\u043b\u043e\u043a2. \u0411\u043b\u043e\u043a\u043e\u0432\u0430 \u0431\u0443\u0434\u043e\u0432\u0430.\n\n\u043f\u043e\u043b\u0456\u0442. \u041f\u0440\u0438\u043a\u043c. \u0434\u043e \u0431\u043b\u043e\u043a3. \u0412\u0438\u0433\u043b\u044f\u0434\u0430\u043b\u043e \u043d\u0430 \u0442\u0435, \u0449\u043e \u0432\u0441\u0456\u0445 \u0432\u043b\u0430\u0448\u0442\u043e\u0432\u0443\u0454 \u0431\u043b\u043e\u043a\u043e\u0432\u0438\u0439 \u0441\u0442\u0430\u0442\u0443\u0441. \u0417\u0440\u0435\u0448\u0442\u043e\u044e, \u043f\u043e\u043b\u0456\u0442\u0438\u043a\u0438 \u0434\u0456\u0439\u0448\u043b\u0438 \u0432\u0438\u0441\u043d\u043e\u0432\u043a\u0443, \u0449\u043e \u043f\u0440\u0456\u043e\u0440\u0438\u0442\u0435\u0442\u043e\u043c \u0454 \u043f\u0435\u0440\u0435\u043c\u043e\u0433\u0430 \u043d\u0430 \u043f\u0440\u0435\u0437\u0438\u0434\u0435\u043d\u0442\u0441\u044c\u043a\u0438\u0445 \u0432\u0438\u0431\u043e\u0440\u0430\u0445 (\u0437 \u0433\u0430\u0437.).\n\u041f\u0440\u0438\u043a\u043c. \u0434\u043e \u0431\u043b\u043e\u043a3. \u0411\u043b\u043e\u043a\u043e\u0432\u0443 \u0441\u0442\u0440\u0443\u043a\u0442\u0443\u0440\u0443 \u0442\u0435\u043b\u0435\u0432\u0456\u0437\u0456\u0439\u043d\u043e\u0457 \u043f\u0440\u043e\u0433\u0440\u0430\u043c\u0438 \u0431\u0443\u043b\u043e \u0437\u043c\u0456\u043d\u0435\u043d\u043e (\u0437 \u0433\u0430\u0437.).\n\n\n\u0431\u043b\u043e\u0447\u043d\u0438\u0439\n\n\n\u041f\u0440\u0438\u043a\u043c. \u0434\u043e \u0431\u043b\u043e\u043a1. \u0410\u0432\u0442\u043e\u043d\u0430\u0432\u0430\u043d\u0442\u0430\u0436\u0443\u0432\u0430\u0447, \u043e\u0431\u043b\u0430\u0434\u043d\u0430\u043d\u0438\u0439 \u0431\u043b\u043e\u0447\u043d\u043e\u044e \u0441\u0442\u0440\u0456\u043b\u043e\u044e, \u0437\u0430\u0441\u0442\u043e\u0441\u043e\u0432\u0443\u0454\u0442\u044c\u0441\u044f \u0432 \u043f\u043e\u0440\u0442\u0430\u0445, \u0434\u043e\u043a\u0430\u0445 \u0456 \u043d\u0430 \u0431\u0443\u0434\u0456\u0432\u0435\u043b\u044c\u043d\u0438\u0445 \u043c\u0430\u0439\u0434\u0430\u043d\u0447\u0438\u043a\u0430\u0445 (\u0437 \u043d\u0430\u0443\u043a.-\u043f\u043e\u043f\u0443\u043b. \u043b\u0456\u0442.).\n\u0441\u043f\u0435\u0446. \u041f\u0440\u0438\u043a\u043c. \u0434\u043e \u0431\u043b\u043e\u043a2. \u0411\u043b\u043e\u0447\u043d\u0438\u0439 \u043c\u0430\u0442\u0435\u0440\u0456\u0430\u043b; \uff0f \u042f\u043a\u0438\u0439 \u0441\u043a\u043b\u0430\u0434\u0430\u0454\u0442\u044c\u0441\u044f \u0437\u0456 \u0441\u0442\u0430\u043d\u0434\u0430\u0440\u0442\u043d\u0438\u0445 \u0431\u043b\u043e\u043a\u0456\u0432. \u0411\u043b\u043e\u0447\u043d\u0456 \u0442\u0435\u043f\u043b\u0438\u0446\u0456 \u2014 \u0446\u0435 \u043e\u0431\u02bc\u0454\u0434\u043d\u0430\u043d\u043d\u044f \u0434\u0432\u043e\u0441\u0445\u0438\u043b\u0438\u0445 \u0442\u0435\u043f\u043b\u0438\u0446\u044c, \u0443 \u044f\u043a\u0438\u0445 \u043d\u0435\u043c\u0430\u0454 \u043f\u0440\u043e\u043c\u0456\u0436\u043a\u0456\u0432 \u043c\u0456\u0436 \u0441\u0435\u043a\u0446\u0456\u044f\u043c\u0438, \u0430 \u0431\u043e\u043a\u043e\u0432\u0456 \u043f\u0440\u043e\u0441\u0442\u0456\u043d\u043a\u0438 \u0437\u0430\u043c\u0456\u043d\u0435\u043d\u0456 \u0441\u0442\u043e\u044f\u043a\u0430\u043c\u0438 (\u0437 \u043d\u0430\u0443\u043a. \u043b\u0456\u0442.).\n\n\u0431\u043b\u043e\u043a\n\n\n-\u0430 (\u0440\u043e\u0434\u043e\u0432\u0438\u0439 \u0432\u0456\u0434\u043c\u0456\u043d\u043e\u043a) \u041f\u0440\u043e\u0441\u0442\u0438\u0439 \u043c\u0435\u0445\u0430\u043d\u0456\u0437\u043c \u0434\u043b\u044f \u043f\u0456\u0434\u043d\u044f\u0442\u0442\u044f \u0432\u0430\u0436\u043a\u0438\u0445 \u043f\u0440\u0435\u0434\u043c\u0435\u0442\u0456\u0432, \u0449\u043e \u043c\u0430\u0454 \u0444\u043e\u0440\u043c\u0443 \u043a\u043e\u043b\u0435\u0441\u0430 (\u043d\u0430 \u043e\u0441\u0456) \u0437 \u0436\u043e\u043b\u043e\u0431\u043e\u043c, \u0447\u0435\u0440\u0435\u0437 \u044f\u043a\u0435 \u043f\u0435\u0440\u0435\u043a\u0438\u043d\u0443\u0442\u043e \u043b\u0430\u043d\u0446\u044e\u0433, \u043a\u0430\u043d\u0430\u0442 \u0456 \u0442. \u0456\u043d. \u041c\u0456\u043d\u0438 \u043b\u0435\u0436\u0430\u0442\u044c \u043d\u0430 \u043f\u0430\u043b\u0443\u0431\u0456 \u0439 \u0443 \u0442\u0440\u044e\u043c\u0456, .. \u0431\u043b\u043e\u043a\u0430\u043c\u0438 \u043e\u043f\u0443\u0441\u043a\u0430\u044e\u0442\u044c \u0457\u0445 \u0443 \u043c\u043e\u0440\u0441\u044c\u043a\u0443 \u0434\u043e\u0440\u043e\u0433\u0443 (\u042e. \u042f\u043d\u043e\u0432\u0441\u044c\u043a\u0438\u0439); \u0412\u0456\u0442\u0435\u0440 \u0433\u0443\u043b\u044f\u0454 \u0443 \u0432\u0456\u0442\u0430\u0445 \u043a\u0430\u0448\u0442\u0430\u043d\u0456\u0432... \u041d\u0456\u0436\u043d\u043e \u043f\u043e\u0441\u0432\u0438\u0441\u0442\u0443\u0454 \u0431\u043b\u043e\u043a\u0430\u043c\u0438 \u043a\u0440\u0430\u043d\u0456\u0432 (\u041c. \u0413\u0456\u0440\u043d\u0438\u043a); \u0421\u043b\u0443\u0433\u0438 \u0432\u0432\u043e\u0437\u0438\u043b\u0438 \u0437\u0430\u043c\u043e\u0440\u0441\u044c\u043a\u0456 \u0444\u0440\u0443\u043a\u0442\u0438 \u0432 \u0432\u0430\u0437\u0430\u0445 \u0437 \u0447\u0438\u0441\u0442\u043e\u0433\u043e \u0437\u043e\u043b\u043e\u0442\u0430 \u0456 \u0442\u0430\u043a\u0438\u0445 \u0432\u0430\u0436\u043a\u0438\u0445, \u0449\u043e \u043f\u0456\u0434\u0456\u0439\u043c\u0430\u0442\u0438 \u0457\u0445 \u043d\u0430 \u0441\u0442\u043e\u043b\u0438 \u0434\u043e\u0432\u043e\u0434\u0438\u043b\u043e\u0441\u044c \u043d\u0430 \u043e\u0431\u0448\u0438\u0442\u0438\u0445 \u043f\u043e\u0437\u043e\u043b\u043e\u0447\u0435\u043d\u043e\u044e \u0448\u043a\u0456\u0440\u043e\u044e \u0432\u0456\u0440\u044c\u043e\u0432\u043a\u0430\u0445, \u043f\u0435\u0440\u0435\u043a\u0438\u043d\u0443\u0442\u0438\u0445 \u0447\u0435\u0440\u0435\u0437 \u0431\u043b\u043e\u043a\u0438, \u0445\u0438\u0442\u0440\u043e \u0441\u0445\u043e\u0432\u0430\u043d\u0456 \u043f\u0456\u0434 \u0441\u0442\u0435\u043b\u0435\u044e (\u041f. \u0417\u0430\u0433\u0440\u0435\u0431\u0435\u043b\u044c\u043d\u0438\u0439).\n-\u0430 (\u0440\u043e\u0434\u043e\u0432\u0438\u0439 \u0432\u0456\u0434\u043c\u0456\u043d\u043e\u043a)\n\n\u0442\u0435\u0445\u043d. \u041e\u043a\u0440\u0435\u043c\u0430, \u043d\u0435\u0437\u0430\u043b\u0435\u0436\u043d\u0430 \u0447\u0430\u0441\u0442\u0438\u043d\u0430 \u0441\u043f\u043e\u0440\u0443\u0434\u0438, \u043c\u0430\u0448\u0438\u043d\u0438 \u0456 \u0442. \u0456\u043d., \u0449\u043e \u0441\u043a\u043b\u0430\u0434\u0430\u0454\u0442\u044c\u0441\u044f \u0442\u0430\u043a\u043e\u0436 \u0437 \u043e\u043a\u0440\u0435\u043c\u0438\u0445 \u0435\u043b\u0435\u043c\u0435\u043d\u0442\u0456\u0432 \u0430\u0431\u043e \u0434\u0435\u0442\u0430\u043b\u0435\u0439. \u201e\u041f\u0440\u0438\u043a\u0430\u0440\u043f\u0430\u0442\u043b\u0456\u0441\u201c \u2014 \u043e\u0434\u0438\u043d \u0437 \u043f\u0440\u043e\u0432\u0456\u0434\u043d\u0438\u0445 \u043f\u043e\u0441\u0442\u0430\u0447\u0430\u043b\u044c\u043d\u0438\u043a\u0456\u0432 \u043d\u0430 \u0440\u0438\u043d\u043e\u043a \u0423\u043a\u0440\u0430\u0457\u043d\u0438 \u043f\u0438\u043b\u043e\u043c\u0430\u0442\u0435\u0440\u0456\u0430\u043b\u0456\u0432, \u0434\u0432\u0435\u0440\u043d\u0438\u0445 \u0442\u0430 \u0432\u0456\u043a\u043e\u043d\u043d\u0438\u0445 \u0431\u043b\u043e\u043a\u0456\u0432 (\u0456\u0437 \u0436\u0443\u0440\u043d.).\n\u0431\u0443\u0434. \u041f\u0440\u0438\u0440\u043e\u0434\u043d\u0438\u0439 \u0430\u0431\u043e \u0448\u0442\u0443\u0447\u043d\u0438\u0439 \u0431\u0443\u0434\u0456\u0432\u0435\u043b\u044c\u043d\u0438\u0439 \u043a\u0430\u043c\u0456\u043d\u044c, \u043f\u043e\u0440\u0456\u0437\u0430\u043d\u0438\u0439 \u043d\u0430 \u043f\u043b\u0438\u0442\u0438. \u0413\u0440\u0443\u0431\u0435\u0437\u043d\u0456 \u0439\u043e\u0433\u043e [\u0445\u0440\u0430\u043c\u0443] \u0441\u0442\u0456\u043d\u0438, \u043c\u0443\u0440\u043e\u0432\u0430\u043d\u0456 \u0437 \u043a\u0430\u043c\u02bc\u044f\u043d\u0438\u0445 \u0431\u043b\u043e\u043a\u0456\u0432, \u0431\u0443\u043b\u0438 .. \u043f\u043e\u0431\u0456\u043b\u0435\u043d\u0456 \u0432\u0430\u043f\u043d\u043e\u043c (\u0420. \u0424\u0435\u0434\u043e\u0440\u0456\u0432); \u0423\u0447\u0435\u043d\u0456 \u0440\u0456\u0437\u043d\u0438\u0445 \u043a\u0440\u0430\u0457\u043d \u0434\u0430\u0432\u043d\u043e \u043b\u0430\u043c\u0430\u044e\u0442\u044c \u0433\u043e\u043b\u043e\u0432\u0443 \u043d\u0430\u0434 \u0441\u0435\u043a\u0440\u0435\u0442\u043e\u043c \u0425\u0435\u043c\u0456\u0443\u043d\u0430: \u044f\u043a \u0432\u0456\u043d \u0443\u043c\u0443\u0434\u0440\u0438\u0432\u0441\u044f \u0437\u043c\u043e\u043d\u0442\u0443\u0432\u0430\u0442\u0438 \u0433\u0456\u0433\u0430\u043d\u0442\u0441\u044c\u043a\u0456 \u043a\u0430\u043c\u02bc\u044f\u043d\u0456 \u0431\u043b\u043e\u043a\u0438 \u043f\u0456\u0440\u0430\u043c\u0456\u0434\u0438 \u0425\u0435\u043e\u043f\u0441\u0430 (\u0437 \u043d\u0430\u0443\u043a. \u043b\u0456\u0442.); \u0417 \u0440\u0456\u0437\u043d\u0438\u0445 \u0433\u043b\u0438\u043d \u0432\u0438\u0433\u043e\u0442\u043e\u0432\u043b\u044f\u044e\u0442\u044c \u043a\u0435\u0440\u0430\u043c\u0456\u0447\u043d\u0456 \u0431\u043b\u043e\u043a\u0438 \u0434\u043b\u044f \u0441\u0442\u0456\u043d, \u043f\u0430\u043d\u0435\u043b\u0435\u0439 \u0442\u0430 \u043f\u0435\u0440\u0435\u043a\u0440\u0438\u0442\u0442\u0456\u0432 (\u0437 \u043d\u0430\u0443\u043a.-\u043f\u043e\u043f\u0443\u043b. \u043b\u0456\u0442.).\n\u0423\u043f\u0430\u043a\u0443\u0432\u0430\u043d\u043d\u044f, \u0449\u043e \u043c\u0456\u0441\u0442\u0438\u0442\u044c \u043a\u0456\u043b\u044c\u043a\u0430 (\u0431\u0430\u0433\u0430\u0442\u043e) \u043e\u0434\u043d\u0430\u043a\u043e\u0432\u0438\u0445 \u043f\u0440\u0435\u0434\u043c\u0435\u0442\u0456\u0432. \u042f\u043a\u0431\u0438 \u0437\u043b\u043e\u0447\u0438\u043d\u0435\u0446\u044c \u043d\u0430\u0432\u0456\u0434\u0430\u0432\u0441\u044f \u0432\u0447\u043e\u0440\u0430 \u0447\u0438 \u043d\u0430\u0441\u0442\u0443\u043f\u043d\u043e\u0433\u043e \u0434\u043d\u044f, \u0442\u043e \u0439\u043e\u0433\u043e \u0437\u0434\u043e\u0431\u0438\u0447\u0447\u044e \u0431\u0443\u043b\u043e \u0431 \u043d\u0435 \u0431\u0456\u043b\u044c\u0448\u0435 \u0434\u0435\u0441\u044f\u0442\u043a\u0430 \u0431\u043b\u043e\u043a\u0456\u0432 \u0441\u0438\u0433\u0430\u0440\u0435\u0442 (\u0437 \u0433\u0430\u0437.).\n\n-\u0443 (\u0440\u043e\u0434\u043e\u0432\u0438\u0439 \u0432\u0456\u0434\u043c\u0456\u043d\u043e\u043a)\n\n\u043f\u043e\u043b\u0456\u0442. \u041e\u0431\u02bc\u0454\u0434\u043d\u0430\u043d\u043d\u044f \u0434\u0435\u0440\u0436\u0430\u0432, \u043e\u0440\u0433\u0430\u043d\u0456\u0437\u0430\u0446\u0456\u0439, \u043f\u0430\u0440\u0442\u0456\u0439 \u0456 \u0442. \u0456\u043d. \u041d\u0410\u0422\u041e \u2014 \u0432\u0456\u0439\u0441\u044c\u043a\u043e\u0432\u043e-\u043f\u043e\u043b\u0456\u0442\u0438\u0447\u043d\u0438\u0439 \u0431\u043b\u043e\u043a, \u0434\u043e \u044f\u043a\u043e\u0433\u043e \u043d\u0430 \u0447\u0430\u0441 \u0441\u0442\u0432\u043e\u0440\u0435\u043d\u043d\u044f \u0432\u0445\u043e\u0434\u0438\u043b\u0438 \u0421\u0428\u0410, \u0412\u0435\u043b\u0438\u043a\u043e\u0431\u0440\u0438\u0442\u0430\u043d\u0456\u044f, \u0424\u0440\u0430\u043d\u0446\u0456\u044f, \u0406\u0442\u0430\u043b\u0456\u044f, \u041a\u0430\u043d\u0430\u0434\u0430 \u0442\u0430 \u0456\u043d\u0448\u0456 \u043a\u0440\u0430\u0457\u043d\u0438 (\u0437 \u043d\u0430\u0432\u0447. \u043b\u0456\u0442.); \u0427\u0438\u043c\u0430\u043b\u043e \u043d\u0430\u0440\u043e\u0434\u043d\u0438\u0445 \u0434\u0435\u043f\u0443\u0442\u0430\u0442\u0456\u0432 \u0437\u0430\u043f\u0435\u0432\u043d\u044f\u044e\u0442\u044c, \u043c\u043e\u0432\u043b\u044f\u0432, \u043f\u0440\u043e\u0433\u043e\u043b\u043e\u0441\u0443\u0439\u0442\u0435 \u0437\u0430 \u043c\u0435\u043d\u0435 \u0447\u0438 \u043c\u0456\u0439 \u0432\u0438\u0431\u043e\u0440\u0447\u0438\u0439 \u0431\u043b\u043e\u043a, \u0456 \u043f\u0438\u0442\u0430\u043d\u043d\u044f \u0456\u0437 \u0432\u043a\u043b\u0430\u0434\u0430\u043c\u0438 \u041e\u0449\u0430\u0434\u0431\u0430\u043d\u043a\u0443 \u0421\u0420\u0421\u0420 \u0431\u0443\u0434\u0435 \u0437\u043d\u044f\u0442\u043e (\u0437 \u0433\u0430\u0437.).\n\u0421\u0435\u0440\u0456\u044f \u043e\u0434\u043d\u043e\u0442\u0438\u043f\u043d\u0438\u0445 \u0442\u0435\u043b\u0435- \u0430\u0431\u043e \u0440\u0430\u0434\u0456\u043e\u043f\u0440\u043e\u0433\u0440\u0430\u043c, \u043f\u0435\u0440\u0435\u0434\u0430\u043d\u0438\u0445 \u043e\u0434\u043d\u0430 \u0437\u0430 \u043e\u0434\u043d\u043e\u044e, \u0430\u0431\u043e \u043a\u0456\u043b\u044c\u043a\u0430 \u0434\u0440\u0443\u043a\u043e\u0432\u0430\u043d\u0438\u0445 \u043c\u0430\u0442\u0435\u0440\u0456\u0430\u043b\u0456\u0432 \u043d\u0430 \u043e\u0434\u043d\u0443 \u0442\u0435\u043c\u0443. \u041e\u0434\u043d\u0430 \u0437 \u0447\u0430\u0441\u0442\u0438\u043d \u043d\u043e\u0432\u043e\u0457 \u0442\u0435\u043b\u0435\u043f\u0440\u043e\u0433\u0440\u0430\u043c\u0438 \u2014 \u0449\u043e\u0434\u0435\u043d\u043d\u0438\u0439 \u0431\u043b\u043e\u043a \u0441\u043f\u043e\u0440\u0442\u0438\u0432\u043d\u0438\u0445 \u043d\u043e\u0432\u0438\u043d (\u0437 \u0433\u0430\u0437.).\n\u041b\u043e\u0433\u0456\u0447\u043d\u043e \u0437\u0430\u0432\u0435\u0440\u0448\u0435\u043d\u0430 \u0447\u0430\u0441\u0442\u0438\u043d\u0430 \u043f\u0435\u0432\u043d\u043e\u0457 \u043f\u0440\u043e\u0433\u0440\u0430\u043c\u0438 \u0430\u0431\u043e \u043f\u0435\u0440\u0435\u043b\u0456\u043a\u0443 \u0434\u0456\u0439. \u041f\u043e\u0441\u0456\u0431\u043d\u0438\u043a \u0441\u043a\u043b\u0430\u0434\u0430\u0454\u0442\u044c\u0441\u044f \u0437 \u0447\u043e\u0442\u0438\u0440\u044c\u043e\u0445 \u0431\u043b\u043e\u043a\u0456\u0432, \u0434\u043e \u044f\u043a\u0438\u0445 \u0443\u0432\u0456\u0439\u0448\u043b\u0438 \u0456\u043d\u0442\u0435\u0440\u0430\u043a\u0442\u0438\u0432\u043d\u0456 \u0432\u043f\u0440\u0430\u0432\u0438, \u0430\u043f\u0440\u043e\u0431\u043e\u0432\u0430\u043d\u0456 \u043f\u0456\u0434 \u0447\u0430\u0441 \u0440\u0435\u0430\u043b\u0456\u0437\u0430\u0446\u0456\u0457 \u043f\u0440\u043e\u0435\u043a\u0442\u0443 (\u0437 \u043d\u0430\u0432\u0447. \u043b\u0456\u0442.).\n\n\n\n\n\u041d\u043e\u0440\u043c\u0438 \u0443\u043a\u0440\u0430\u0457\u043d\u0441\u044c\u043a\u043e\u0457 \u043b\u0456\u0442\u0435\u0440\u0430\u0442\u0443\u0440\u043d\u043e\u0457 \u043c\u043e\u0432\u0438 (\u041e\u043b\u0435\u043a\u0441\u0430 \u0421\u0438\u043d\u044f\u0432\u0441\u044c\u043a\u0438\u0439, 1941)\n\n-\u043d\u0438\u0439, -\u043d\u0456\u0439 \u2014 \u043d\u0430\u0439\u0448\u0438\u0440\u0448\u0435 \u0432\u0436\u0438\u0432\u0430\u043d\u0438\u0439 \u043d\u0430\u0440\u043e\u0441\u0442\u043e\u043a \u043f\u0440\u0438 \u0440\u0456\u0437\u043d\u0438\u0445 \u043f\u043d\u044f\u0445 \u0456 \u0437 \u0440\u0456\u0437\u043d\u043e\u043c\u0430\u043d\u0456\u0442\u043d\u0438\u043c \u0437\u043d\u0430\u0447\u0456\u043d\u043d\u044f\u043c:\n\n\u0430) \u043f\u0440\u0438 \u0456\u043c\u0435\u043d\u043d\u0438\u043a\u043e\u0432\u0438\u0445 \u043f\u043d\u044f\u0445 \u043d\u0430 \u043e\u0437\u043d\u0430\u0447\u0435\u043d\u043d\u044f \u043c\u0430\u0442\u0435\u0440\u0456\u044f\u043b\u0443, \u0434\u0430\u043b\u0456 \u0432\u0456\u0434\u043d\u043e\u0448\u0435\u043d\u043d\u044f, \u0491\u0430\u0442\u0443\u043d\u043a\u0443, \u0432\u043b\u0430\u0441\u0442\u0438\u0432\u043e\u0441\u0442\u0438 (\u0430\u043b\u0435 \u0432 \u043c\u0435\u043d\u0448\u0456\u0439 \u043c\u0456\u0440\u0456, \u044f\u043a -\u0430\u0441\u0442\u0438\u0439, -\u0438\u0441\u0442\u0438\u0439): \u043c\u043e\u043b\u043e\u0447\u043d\u0438\u0439, \u043f\u0448\u0435\u043d\u0438\u0447\u043d\u0438\u0439, \u0437\u0430\u043b\u0456\u0437\u043d\u0438\u0439, \u043a\u0430\u043c\u0456\u043d\u043d\u0438\u0439, \u043c\u0456\u0434\u043d\u0438\u0439, \u0440\u043e\u0437\u0443\u043c\u043d\u0438\u0439, \u0431\u0435\u0437\u0434\u043e\u0433\u0430\u043d\u043d\u0438\u0439, \u0441\u0440\u0456\u0431\u043d\u0438\u0439, \u0436\u0438\u0442\u043d\u0456\u0439, \u043e\u043a\u0441\u0430\u043c\u0438\u0442\u043d\u0438\u0439, \u0431\u0443\u0434\u0456\u0432\u0435\u043b\u044c\u043d\u0438\u0439, \u043d\u0430\u0440\u043e\u0434\u043d\u0456\u0439, \u0431\u0430\u0440\u0432\u043d\u0438\u0439 (\u043e\u0442\u0436\u0435 \u043d\u0430\u0431\u043b\u0438\u0436\u0430\u0454\u0442\u044c\u0441\u044f \u0437\u043d\u0430\u0447\u0456\u043d\u043d\u044f\u043c, \u0430 \u0456\u043d\u043e\u0434\u0456 \u0439 \u0441\u043f\u0430\u0434\u0430\u0454\u0442\u044c\u0441\u044f \u0437 -\u043e\u0432\u0438\u0439, -\u044f\u043d\u0438\u0439).\n\u0412\u0456\u0434 \u0434\u0435\u044f\u043a\u0438\u0445 \u0456\u0437 \u0442\u0430\u043a\u0438\u0445 \u043f\u0440\u0438\u043a\u043c\u0435\u0442\u043d\u0438\u043a\u0456\u0432 \u0432\u0456\u0434\u043e\u043a\u0440\u0435\u043c\u043b\u044e\u0454\u0442\u044c\u0441\u044f \u043a\u0430\u0442\u0435\u0433\u043e\u0440\u0456\u044f \u201e\u0431\u0430\u0433\u0430\u0442\u0438\u0439 \u043d\u0430 \u0449\u043e\u0441\u044c\u201c, \u0437\u0432\u0438\u0447\u0430\u0439\u043d\u043e \u0437 \u043f\u0435\u0440\u0435\u0445\u043e\u0434\u043e\u043c \u043d\u0430\u0433\u043e\u043b\u043e\u0441\u0443 \u043d\u0430 -\u043d\u0438\u0439:\n\u0440\u0438\u0431\u043d\u0438\u0439 (\u043b\u043e\u0432\u0435\u0446\u044c) \u2014 \u0440\u0438\u0431\u043d\u0430 (\u0440\u0456\u0447\u043a\u0430)\n\u0445\u043b\u0456\u0431\u043d\u0438\u0439 (\u043a\u0432\u0430\u0441) \u2014 \u0445\u043b\u0456\u0431\u043d\u0438\u0439 (\u043a\u0440\u0430\u0439)\n(\u043b\u0456\u0441\u043e\u0432\u0438\u0439) \u2014 \u043b\u0456\u0441\u043d\u0438\u0439 (\u043a\u0440\u0430\u0439)\n(\u0441\u043d\u0456\u0433\u043e\u0432\u0438\u0439) \u2014 \u0441\u043d\u0456\u0436\u043d\u0430 (\u0437\u0438\u043c\u0430)\n(\u043c\u0435\u0434\u043e\u0432\u0438\u0439) \u2014 \u043c\u0435\u0434\u043d\u0438\u0439 (\u0440\u0456\u043a)\n(\u043b\u044e\u0434\u043e\u0432\u0438\u0439) \u2014 \u043b\u044e\u0434\u043d\u0438\u0439 (\u0431\u0430\u0437\u0430\u0440)\n(\u0433\u043e\u0440\u043e\u0432\u0438\u0439 \u0456 \u0433\u0456\u0440\u043d\u0438\u0439) \u2014 \u0433\u0456\u0440\u043d\u0430 (\u043a\u0440\u0430\u0457\u043d\u0430)\n(\u0445\u043c\u0430\u0440\u043e\u0432\u0438\u0439 \u0456 \u0445\u043c\u0430\u0440\u043d\u0438\u0439) \u2014 \u0445\u043c\u0430\u0440\u043d\u0438\u0439 (\u0434\u0435\u043d\u044c)\n \u2014 \u0437\u0432\u0456\u0440\u043d\u0438\u0439 (\u043b\u0456\u0441)\n\u0417\u043e\u043a\u0440\u0435\u043c\u0430 -\u043d\u0438\u0439 \u0447\u0430\u0441\u0442\u043e \u0431\u0443\u0432\u0430\u0454 \u0432 \u0441\u043b\u043e\u0432\u0430\u0445 \u0447\u0443\u0436\u043e\u043c\u043e\u0432\u043d\u043e\u0433\u043e \u043f\u043e\u0445\u043e\u0434\u0436\u0435\u043d\u043d\u044f, \u043f\u0440\u0438 \u0442\u0456\u043c \u0456\u043d\u043e\u0434\u0456 \u0437 \u0447\u0443\u0436\u043e\u043c\u043e\u0432\u043d\u0438\u043c\u0438 \u043f\u0435\u0440\u0435\u0434 \u043d\u0438\u043c \u043d\u0430\u0440\u043e\u0441\u0442\u043a\u0430\u043c\u0438 -\u0438\u0432-, -\u0430\u0442\u0438\u0432- \u0442\u043e\u0449\u043e: \u0430\u043a\u0442\u0438\u0432\u043d\u0438\u0439, \u043e\u043f\u0435\u0440\u0430\u0442\u0438\u0432\u043d\u0438\u0439, \u0456\u043b\u044e\u0441\u0442\u0440\u0430\u0442\u0438\u0432\u043d\u0438\u0439.\n\u0431) \u043f\u0440\u0438 \u0434\u0456\u0454\u0441\u043b\u0456\u0432\u043d\u0438\u0445 \u043f\u043d\u044f\u0445 \u043d\u0430 \u043e\u0437\u043d\u0430\u0447\u0435\u043d\u043d\u044f \u043f\u0440\u0438\u043a\u043c\u0435\u0442\u043d\u0438\u043a\u0456\u0432, \u0456\u043d\u043e\u0434\u0456 \u0431\u043b\u0438\u0437\u044c\u043a\u0438\u0445 \u0437\u043d\u0430\u0447\u0456\u043d\u043d\u044f\u043c \u0434\u043e \u0437\u0430\u0433\u0443\u0431\u043b\u0435\u043d\u0438\u0445 \u0434\u0456\u0454\u043f\u0440\u0438\u043a\u043c\u0435\u0442\u043d\u0438\u043a\u0456\u0432 \u043f\u0435\u0440\u0435\u0454\u043c\u043d\u0438\u0445 \u0442\u0435\u043f\u0435\u0440\u0456\u0448\u043d\u044c\u043e\u0433\u043e \u0447\u0430\u0441\u0443, \u0430 \u043f\u043e\u0447\u0430\u0441\u0442\u0438 \u0439 \u043c\u0438\u043d\u0443\u043b\u043e\u0433\u043e: \u043f\u0440\u043e\u0445\u0456\u0434\u043d\u0438\u0439, \u043d\u0430\u0441\u0442\u0443\u043f\u043d\u0438\u0439, \u0434\u043e\u0441\u0442\u0443\u043f\u043d\u0438\u0439, \u043f\u043e\u0436\u0438\u0301\u0432\u043d\u0438\u0439, \u043f\u0440\u0438\u0434\u0430\u0442\u043d\u0438\u0439, \u0447\u0443\u0442\u043d\u0438\u0439, \u043d\u0435\u043f\u043e\u0445\u0438\u0442\u043d\u0438\u0439, \u043d\u0435\u0437\u0430\u0431\u0443\u0442\u043d\u0456\u0439, \u043d\u0430\u0441\u0442\u0443\u043f\u043d\u0438\u0439, \u043d\u0435\u043f\u0440\u043e\u0431\u0443\u0434\u043d\u0438\u0439 \u0456 \u0442. \u0456\u043d. \u2014 \u0434\u0438\u0432. \u00a7 89.3 \u0434\u0440\u0456\u0431\u043d. \u0448\u0440\u0438\u0444\u0442.\n\u0432) \u043f\u0440\u0438 \u043f\u0440\u0438\u0441\u043b\u0456\u0432\u043d\u0438\u043a\u043e\u0432\u0438\u0445 \u043f\u043d\u044f\u0445 \u0447\u0430\u0441\u0442\u043e \u0437 \u0448 \u043f\u0435\u0440\u0435\u0434 -\u043d\u0456\u0439: \u0433\u043e\u0440\u0456\u0448\u043d\u0456\u0439, \u0434\u043e\u043b\u0456\u0448\u043d\u0456\u0439, \u043a\u043e\u043b\u0438\u0448\u043d\u0456\u0439, \u0442\u0435\u043f\u0435\u0440\u0456\u0448\u043d\u0456\u0439, \u0442\u0443\u0442\u0435\u0448\u043d\u0456\u0439, \u0434\u043e\u043c\u0430\u0448\u043d\u0456\u0439, \u0441\u043f\u0440\u0430\u0432\u0434\u0435\u0448\u043d\u0456\u0439 \u2014 \u0434\u0438\u0432. \u00a7 54.\n\u0433) \u0432 \u0434\u0456\u0454\u043f\u0440\u0438\u043a\u043c\u0435\u0442\u043d\u0438\u043a\u0430\u0445 \u043f\u0435\u0440\u0435\u0454\u043c\u043d\u0438\u0445 \u043c\u0438\u043d\u0443\u043b\u043e\u0433\u043e \u0447\u0430\u0441\u0443: \u0434\u0430\u043d\u0438\u0439, \u043a\u0430\u0440\u0430\u043d\u0438\u0439, \u0443\u043b\u044e\u0431\u043b\u0435\u043d\u0438\u0439, \u0437\u0430\u0441\u0432\u043e\u0454\u043d\u0438\u0439... \u2014 \u0434\u0438\u0432. \u00a7 89.\n\n-\u043e\u0432\u0438\u0439 (-\u0439\u043e\u0432\u0438\u0439), -\u0435\u0432\u0438\u0439 (-\u0454\u0432\u0438\u0439) \u2014 \u0434\u043e\u0441\u0438\u0442\u044c \u043f\u043e\u0448\u0438\u0440\u0435\u043d\u0438\u0439 \u043d\u0430\u0440\u043e\u0441\u0442\u043e\u043a \u043d\u0430 \u043e\u0437\u043d\u0430\u0447\u0435\u043d\u043d\u044f \u0432\u0456\u0434\u043d\u043e\u0441\u043d\u0438\u0445 \u043f\u0440\u0438\u043a\u043c\u0435\u0442\u043d\u0438\u043a\u0456\u0432, \u043e\u0442\u0436\u0435 \u0442\u0456\u043b\u044c\u043a\u0438 \u043f\u0440\u0438 \u0456\u043c\u0435\u043d\u043d\u0438\u043a\u043e\u0432\u0438\u0445 \u043f\u043d\u044f\u0445:\n\n\u0430) \u043d\u0430 \u043f\u0438\u0442\u0430\u043d\u043d\u044f \u201e\u0437 \u0447\u043e\u0433\u043e?\u201c, \u201e\u044f\u043a\u0438\u0439?\u201c, \u0440\u0456\u0434\u043a\u043e \u201e\u0447\u0438\u0439?\u201c: \u0434\u0443\u0431\u043e\u0432\u0438\u0439, \u0441\u043e\u0431\u043e\u043b\u0435\u0432\u0438\u0439, \u0457\u0436\u0430\u043a\u043e\u0432\u0438\u0439, \u0432\u043e\u043b\u043e\u0432\u0438\u0439, \u0432\u0435\u0440\u0431\u043e\u0432\u0438\u0439, \u043f\u0430\u043f\u0435\u0440\u043e\u0432\u0438\u0439, \u043b\u0438\u0301\u0441\u043e\u0432\u0438\u0439, \u043e\u0440\u043b\u043e\u0432\u0438\u0439, \u043b\u043e\u0439\u043e\u0432\u0438\u0439, \u0441\u043c\u0443\u0448\u0435\u0432\u0438\u0439, \u0433\u0440\u0443\u0448\u0435\u0432\u0438\u0439, \u0431\u0430\u0454\u0432\u0438\u0439, \u043a\u0440\u0438\u0446\u0435\u0432\u0438\u0439, \u043e\u0432\u043e\u0447\u0435\u0432\u0438\u0439 (\u043f\u043e\u0440\u0456\u0432\u043d. -\u0438\u043d\u0438\u0439, -\u044f\u0447\u0438\u0439, -\u044f\u043d\u0438\u0439).\n\u0431) \u043d\u0430 \u043f\u0438\u0442\u0430\u043d\u043d\u044f \u201e\u044f\u043a\u0438\u0439?\u201c \u201e\u0434\u043e \u0447\u043e\u0433\u043e \u0432\u0456\u0434\u043d\u043e\u0441\u043d\u0438\u0439?\u201c (\u043f\u0440\u0438\u043a\u043c\u0435\u0442\u043d\u0438\u043a\u0438 \u0432\u0456\u0434\u043d\u043e\u0448\u0435\u043d\u043d\u044f): \u043d\u0430\u0443\u043a\u043e\u0432\u0438\u0439, \u0441\u043b\u0443\u0436\u0431\u043e\u0432\u0438\u0439, \u0440\u043e\u043a\u043e\u0432\u0438\u0439, \u0448\u043b\u044f\u0445\u043e\u0432\u0438\u0439, \u0432\u0435\u0440\u0441\u0442\u0432\u043e\u0432\u0438\u0439, \u0437\u0430\u0433\u0430\u0434\u043a\u043e\u0432\u0438\u0439, \u0432\u0438\u043f\u0430\u0434\u043a\u043e\u0432\u0438\u0439, \u0441\u0442\u0435\u043f\u043e\u0432\u0438\u0439, \u0434\u043e\u0449\u043e\u0432\u0438\u0439, \u0431\u0430\u0433\u0430\u0436\u0435\u0432\u0438\u0439...\n\u0412\u0437\u0430\u0433\u0430\u043b\u0456 \u043d\u0430\u0440\u043e\u0441\u0442\u043e\u043a -\u043e\u0432\u0438\u0439 \u0443 \u0437\u043d\u0430\u0447\u0456\u043d\u043d\u0456 \u0456 \u0430) \u0456 \u0431) \u0447\u0430\u0441\u0442\u043e \u0447\u0435\u0440\u0433\u0443\u0454\u0442\u044c\u0441\u044f \u0437 \u0456\u043d\u0448\u0438\u043c\u0438 \u043f\u0440\u0438\u043a\u043c\u0435\u0442\u043d\u0438\u043a\u043e\u0432\u0438\u043c\u0438 \u043d\u0430\u0440\u043e\u0441\u0442\u043a\u0430\u043c\u0438, \u0437\u043e\u043a\u0440\u0435\u043c\u0430 \u0437 -\u043d\u0438\u0439, \u043f\u0440\u0438 \u0442\u0456\u043c \u0447\u0430\u0441\u0442\u043e \u0439 \u0437\u043d\u0430\u0447\u0456\u043d\u043d\u044f \u0441\u043b\u043e\u0432\u0430 \u0437\u043c\u0456\u043d\u044f\u0454\u0442\u044c\u0441\u044f \u0437 \u0437\u043c\u0456\u043d\u043e\u044e \u043d\u0430\u0440\u043e\u0441\u0442\u043a\u0430, \u0430\u043b\u0435 \u043d\u0435 \u0442\u0430\u043a, \u0449\u043e\u0431 \u043c\u043e\u0436\u043d\u0430 \u0431\u0443\u043b\u043e \u0432\u0438\u0437\u043d\u0430\u0447\u0438\u0442\u0438 \u0432\u0456\u0434\u043c\u0456\u043d\u043d\u0456\u0441\u0442\u044c -\u043e\u0432\u0438\u0439 \u0441\u0443\u043f\u0440\u043e\u0442\u0438 \u0456\u043d\u0448\u0438\u0445, \u043d\u0430\u043f\u0440.: \u0440\u043e\u0437\u0443\u043c\u043e\u0432\u0438\u0439 (\u043f\u0440\u0438\u043a\u043c\u0435\u0442\u043d\u0438\u043a \u0432\u0456\u0434\u043d\u043e\u0441\u043d\u0438\u0439) \u2014 \u0440\u043e\u0437\u0443\u043c\u043d\u0438\u0439 (\u0456\u043d\u0448\u0435 \u0437\u043d\u0430\u0447\u0456\u043d\u043d\u044f, \u043c\u0456\u0436 \u0456\u043d\u0448\u0438\u043c, \u0437 \u0441\u0442\u0443\u043f\u0435\u043d\u044e\u0432\u0430\u043d\u043d\u044f\u043c \u2014 \u0440\u043e\u0437\u0443\u043c\u043d\u0456\u0448\u0438\u0439), \u0432\u0430\u0433\u043e\u0432\u0438\u0439 \u2014 \u0432\u0430\u0436\u043d\u0438\u0439, \u0430\u043b\u0435 \u0432 \u0440\u043e\u043a\u043e\u0432\u0438\u0439 \u2014 \u0440\u0456\u0447\u043d\u0438\u0439, \u043e\u043a\u0441\u0430\u043c\u0438\u0442\u043e\u0432\u0438\u0439 \u2014 \u043e\u043a\u0441\u0430\u043c\u0438\u0442\u043d\u0438\u0439 \u0432\u0436\u0435 \u043d\u0435\u043c\u0430\u0454 \u0442\u0430\u043a\u043e\u0457 \u0432\u0456\u0434\u043c\u0456\u043d\u043d\u043e\u0441\u0442\u0438.\n\n\u041c\u043e\u044f \u0434\u0443\u043c\u043a\u0430\n\u0420\u0435\u0437\u0443\u043b\u044c\u0442\u0430\u0442\u0438 \u0432\u043a\u0430\u0437\u0443\u044e\u0442\u044c, \u0449\u043e \u0432\u0441\u0435 \u0437\u0430\u043b\u0435\u0436\u0438\u0442\u044c \u0432\u0456\u0434 \u043a\u043e\u043d\u0442\u0435\u043a\u0441\u0442\u0443: \n\n\u0411\u043b\u043e\u0447\u043d\u0438\u0439 \u2014 \u043c\u043e\u0436\u0435 \u0437\u0430\u043c\u0456\u043d\u044e\u0432\u0430\u0442\u0438 \u0441\u043b\u043e\u0432\u043e \u0441\u043a\u043b\u0430\u0434\u043d\u0438\u0439.\n\u0411\u043b\u043e\u043a\u043e\u0432\u0438\u0439 \u2014 \u0432\u043a\u043b\u044e\u0447\u0430\u0454 \u0437\u043d\u0430\u0447\u0435\u043d\u043d\u044f \u201e\u0431\u043b\u043e\u0447\u043d\u0438\u0439\u201c (\u0432\u043a\u0430\u0437\u0443\u0454 \u044f\u043a\u0430 \u0454 \u201e\u0441\u0438\u0441\u0442\u0435\u043c\u0430\u201c) \u0439 \u043c\u0430\u0454 \u0432\u0456\u0434\u043d\u043e\u0448\u0435\u043d\u043d\u044f \u0434\u043e \u0432\u0441\u0456\u0445 \u201e\u0431\u043b\u043e\u043a\u0456\u0432\u201c, \u0447\u043e\u0433\u043e \u043d\u0435 \u0441\u043a\u0430\u0437\u0430\u0442\u0438 \u043f\u0440\u043e \u201e\u0431\u043b\u043e\u0447\u043d\u0438\u0439\u201c.\n\n\u0410\u043b\u0435 \u044f \u0432\u0432\u0430\u0436\u0430\u044e, \u0449\u043e \u043f\u0440\u0430\u0432\u0438\u043b\u044c\u043d\u043e \u0439 \u0431\u0456\u043b\u044c\u0448 \u0454 \u043b\u043e\u0433\u0456\u0447\u043d\u0438\u043c \u2014 \u0431\u043b\u043e\u043a\u043e\u0432\u0438\u0439.\n\n\u041f\u043e\u0434\u0456\u0431\u043d\u0435\n\nHow to choose between the changing consonants in the root?\n\n", "topic": "ukrainian", "url": "https://ukrainian.stackexchange.com/questions/2585"}, {"image": "VQAonline_00064759.png", "question": "\"\u041f\u0456\u0446\u0415\u0440\u0456\u044f\" \u0447\u0438 \"\u041f\u0456\u0446\u0435\u0440\u0406\u044f\"?", "context": "\u042f\u043a \u043f\u0440\u0430\u0432\u0438\u043b\u044c\u043d\u043e \u0441\u0442\u0430\u0432\u0438\u0442\u0438 \u043d\u0430\u0433\u043e\u043b\u043e\u0441 \u0443 \u0441\u043b\u043e\u0432\u0456 \u043f\u0456\u0446\u0435\u0440\u0456\u044f\n\"\u043f\u0456\u0446\u0415\u0440\u0456\u044f\" \u0447\u0438 \"\u043f\u0456\u0446\u0435\u0440\u0406\u044f\"?\n\u041f\u043e\u0441\u0438\u043b\u0430\u043d\u043d\u044f \u043d\u0430 \u043e\u0431\u0433\u043e\u0432\u043e\u0440\u0435\u043d\u043d\u044f:\nhttps://www.facebook.com/654887197880807/posts/2305225662846944/\n\u041c\u0430\u044e \u043f\u0435\u0432\u043d\u0456 \u0441\u0443\u043c\u043d\u0456\u0432\u0438 \u0449\u043e\u0434\u043e \u043d\u0430\u0432\u0435\u0434\u0435\u043d\u043e\u0433\u043e \u0432\u0430\u0440\u0456\u0430\u043d\u0442\u0430 \u043d\u0430 \u0437\u043e\u0431\u0440\u0430\u0436\u0435\u043d\u043d\u0456.\n\n", "answer": "\u041e\u0440\u0444\u043e\u0433\u0440\u0430\u0444\u0456\u0447\u043d\u0438\u0439 \u0441\u043b\u043e\u0432\u043d\u0438\u043a \u0443\u043a\u0440\u0430\u0457\u043d\u0441\u044c\u043a\u043e\u0457 \u043c\u043e\u0432\u0438\n\n\u043f\u0456\u0446\u0435\u0301\u0440\u0456\u044f \u0456\u043c\u0435\u043d\u043d\u0438\u043a \u0436\u0456\u043d\u043e\u0447\u043e\u0433\u043e \u0440\u043e\u0434\u0443\n\n\u0412\u0422\u0421\u0421\u0423\u041c\n\n\u043f\u0456\u0446\u0435\u0301\u0440\u0456\u044f\n-\u0457, \u0436. \u0417\u0430\u043a\u0443\u0441\u043e\u0447\u043d\u0430, \u0432 \u044f\u043a\u0456\u0439 \u0433\u043e\u0442\u0443\u044e\u0442\u044c \u0442\u0430 \u043f\u043e\u0434\u0430\u044e\u0442\u044c \u043f\u0456\u0446\u0443.\n\n\u0412\u0456\u043b\u044c\u043d\u0438\u0439 \u0442\u043b\u0443\u043c\u0430\u0447\u043d\u0438\u0439 \u0441\u043b\u043e\u0432\u043d\u0438\u043a\n\n\u043f\u0456\u0446\u0435\u0301\u0440\u0456\u0301\u044f -\u0457, \u0436\u0456\u043d. \u0417\u0430\u043a\u043b\u0430\u0434 \u0445\u0430\u0440\u0447\u0443\u0432\u0430\u043d\u043d\u044f, \u0443 \u044f\u043a\u043e\u043c\u0443 \u0433\u043e\u043b\u043e\u0432\u043d\u0438\u043c \u0447\u0438\u043d\u043e\u043c \u0433\u043e\u0442\u0443\u044e\u0442\u044c\n\u043f\u0456\u0446\u0443.\n\n\u041f\u0440\u0430\u0432\u0438\u043b\u044c\u043d\u043e \u043d\u0430\u0433\u043e\u043b\u043e\u0441 \u043f\u0456\u0446\u0415\u0440\u0456\u044f\n", "topic": "ukrainian", "url": "https://ukrainian.stackexchange.com/questions/5495"}, {"image": "VQAonline_00064758.png", "question": "\u041a\u0430\u043f\u0443\u0441\u0442\u0430 \u0446\u0432\u0456\u0442\u043d\u0430 \u0447\u0438 \u043a\u043e\u043b\u044c\u043e\u0440\u043e\u0432\u0430?", "context": "\u041d\u0430\u0442\u0440\u0430\u043f\u0438\u043b\u0430 \u043d\u0430 \u0440\u0435\u0446\u0435\u043f\u0442\u0438 \u041e\u0431\u0441\u043c\u0430\u0436\u0435\u043d\u0430 \u043a\u043e\u043b\u044c\u043e\u0440\u043e\u0432\u0430 \u043a\u0430\u043f\u0443\u0441\u0442\u0430 \u0437 \u044f\u0439\u0446\u044f\u043c\u0438 \u0442\u0430 \u0426\u0432\u0456\u0442\u043d\u0430 \u043a\u0430\u043f\u0443\u0441\u0442\u0430 \u0443 \u0432\u0435\u0440\u0448\u043a\u043e\u0432\u043e\u043c\u0443 \u0441\u043e\u0443\u0441\u0456 \n\u0426\u0456\u043a\u0430\u0432\u0438\u0442\u044c, \u044f\u043a \u043f\u0440\u0430\u0432\u0438\u043b\u044c\u043d\u043e \u043d\u0430\u0437\u0432\u0430\u0442\u0438 \u0432\u0438\u0434 \u043a\u0430\u043f\u0443\u0441\u0442\u0438, \u0437\u043e\u0431\u0440\u0430\u0436\u0435\u043d\u0438\u0439 \u043d\u0430 \u0444\u043e\u0442\u043e - \u0446\u0432\u0456\u0442\u043d\u0430 \u0447\u0438 \u043a\u043e\u043b\u044c\u043e\u0440\u043e\u0432\u0430? \n\n\u0412\u0456\u043a\u0456\u043f\u0435\u0434\u0456\u0457 \u0441\u0442\u0430\u0442\u0442\u044f \u041a\u0430\u043f\u0443\u0441\u0442\u0430 \u0446\u0432\u0456\u0442\u043d\u0430 \n\n\u041a\u0443\u0447\u0435\u0440\u044f\u0432\u0430 \u043a\u0430\u043f\u0443\u0441\u0442\u0430, \u043a\u0432\u0456\u0442\u043d\u0430 \u043a\u0430\u043f\u0443\u0441\u0442\u0430 \u0430\u0431\u043e \u0446\u0432\u0456\u0442\u043d\u0430 \u043a\u0430\u043f\u0443\u0441\u0442\u0430 (Brassica oleracea\n var. botrytis)\n\n\u0412 Google \u043f\u043e\u0448\u0443\u043a \u0432\u0438\u0434\u0430\u0454 \u043e\u0431\u0438\u0434\u0432\u0430 \u0432\u0430\u0440\u0456\u0430\u043d\u0442\u0438. \n", "answer": "\u042f\u043a \u043f\u0440\u0430\u0432\u0438\u043b\u044c\u043d\u043e \u0432 \u043a\u043e\u043c\u0435\u043d\u0442\u0430\u0440\u0456 \u043a\u0430\u0436\u0435 Yola, \u043a\u0430\u043f\u0443\u0441\u0442\u0430 \u0446\u0432\u0456\u0442\u043d\u0430, \u0431\u043e \u0446\u0432\u0456\u0442\u0435. (\u041a\u043e\u043b\u044c\u043e\u0440\u043e\u0432\u0430 \u2014 \u0446\u0435 \u0441\u043f\u0440\u043e\u0431\u0430 \u043f\u0435\u0440\u0435\u043a\u043b\u0430\u0441\u0442\u0438 \u0437 \u0440\u043e\u0441\u0456\u0439\u0441\u044c\u043a\u043e\u0457 \u043f\u0440\u0438\u043a\u043c\u0435\u0442\u043d\u0438\u043a \u0446\u0432\u0435\u0442\u043d\u0430\u044f, \u044f\u043a\u0438\u0439 \u043c\u043e\u0436\u0435 \u043f\u043e\u0437\u043d\u0430\u0447\u0430\u0442\u0438 \u0456 \u00ab\u043a\u043e\u043b\u044c\u043e\u0440\u043e\u0432\u0430\u00bb, \u0456 \u00ab\u0446\u0432\u0456\u0442\u043d\u0430\u00bb, \u0442\u043e\u043c\u0443 \u0443 \u0432\u0438\u043f\u0430\u0434\u043a\u0443 \u043a\u0430\u043f\u0443\u0441\u0442\u0438 \u0439\u043e\u0433\u043e \u0447\u0430\u0441\u0442\u043e \u043d\u0435\u043f\u0440\u0430\u0432\u0438\u043b\u044c\u043d\u043e \u0456\u043d\u0442\u0435\u0440\u043f\u0440\u0435\u0442\u0443\u044e\u0442\u044c.)\n\u00ab\u0421\u043b\u043e\u0432\u043d\u0438\u043a \u0443\u043a\u0440\u0430\u0457\u043d\u0441\u044c\u043a\u043e\u0457 \u043c\u043e\u0432\u0438\u00bb \u0432 20 \u0442\u043e\u043c\u0430\u0445:\n\n\u041a\u0410\u041f\u0423\u0301\u0421\u0422\u0410, \u0438, \u0436[\u0456\u043d\u043e\u0447\u0438\u0439 \u0440\u0456\u0434]. <\u2026>\n\u25b3 (7) \u0426\u0432\u0456\u0442\u043d\u0430\u0301 \u043a\u0430\u043f\u0443\u0301\u0441\u0442\u0430 \u2014 \u0441\u043e\u0440\u0442 \u043a\u0430\u043f\u0443\u0441\u0442\u0438, \u0441\u0443\u0446\u0432\u0456\u0442\u0442\u044f \u044f\u043a\u043e\u0457 \u0441\u043a\u043b\u0430\u0434\u0430\u0454\u0442\u044c\u0441\u044f \u0437 \u0443\u043a\u043e\u0440\u043e\u0447\u0435\u043d\u0438\u0445 \u0442\u0430 \u043f\u043e\u0442\u043e\u0432\u0449\u0435\u043d\u0438\u0445 \u043a\u0432\u0456\u0442\u043a\u043e\u043d\u043e\u0441\u043d\u0438\u0445 \u043f\u0430\u0433\u043e\u043d\u0456\u0432, \u0449\u043e \u0449\u0456\u043b\u044c\u043d\u043e \u043f\u0440\u0438\u043b\u044f\u0433\u0430\u044e\u0442\u044c \u043e\u0434\u0438\u043d \u0434\u043e \u043e\u0434\u043d\u043e\u0433\u043e. \u0426\u0432\u0456\u0442\u043d\u0456\u0439 \u043a\u0430\u043f\u0443\u0441\u0442\u0456 \u043d\u0430\u043b\u0435\u0436\u0438\u0442\u044c \u043f\u0435\u0440\u0448\u0456\u0441\u0442\u044c \u0437\u0430 \u0432\u043c\u0456\u0441\u0442\u043e\u043c \u043d\u0430\u0439\u0432\u0430\u0436\u043b\u0438\u0432\u0456\u0448\u0438\u0445 \u0432\u0456\u0442\u0430\u043c\u0456\u043d\u0456\u0432 \u0442\u0430 \u043c\u0456\u043d\u0435\u0440\u0430\u043b\u044c\u043d\u0438\u0445 \u0440\u0435\u0447\u043e\u0432\u0438\u043d \u043d\u0435 \u043b\u0438\u0448\u0435 \u0441\u0435\u0440\u0435\u0434 \u0456\u043d\u0448\u0438\u0445 \u0432\u0438\u0434\u0456\u0432 \u043a\u0430\u043f\u0443\u0441\u0442\u0438, \u0430 \u0439 \u0441\u0435\u0440\u0435\u0434 \u043e\u0432\u043e\u0447\u0456\u0432 \u0432\u0437\u0430\u0433\u0430\u043b\u0456 (\u0456\u0437 \u0436\u0443\u0440\u043d.); \u041f\u043e\u043a\u0430\u0437\u0430\u043b\u0430 [\u041c\u0430\u0440\u0438\u043d\u0430],\n \u0434\u0435 \u043a\u0430\u0440\u0442\u043e\u043f\u043b\u044f \u0437 \u043c\u043e\u0440\u043a\u0432\u043e\u044e, \u0434\u0435 \u0431\u0430\u043d\u044f\u043a\u0438, \u043f\u043e\u044f\u0441\u043d\u0438\u043b\u0430, \u044f\u043a \u0432\u043e\u043d\u0430 \u0433\u043e\u0442\u0443\u0454 \u0441\u0443\u043f \u0437 \u0446\u0432\u0456\u0442\u043d\u043e\u0457 \u043a\u0430\u043f\u0443\u0441\u0442\u0438 (\u0413.\u00a0\u0412\u0434\u043e\u0432\u0438\u0447\u0435\u043d\u043a\u043e).\n\n\u0412\u0456\u043a\u0456\u043f\u0435\u0434\u0456\u044f \u0432 \u0441\u0442\u0430\u0442\u0442\u0456 \u00ab\u041a\u0430\u043f\u0443\u0441\u0442\u0430 \u0446\u0432\u0456\u0442\u043d\u0430\u00bb \u0442\u0430\u043a\u043e\u0436 \u043d\u0430\u0432\u043e\u0434\u0438\u0442\u044c \u0432\u0430\u0440\u0456\u0430\u043d\u0442\u0438 \u043a\u0432\u0456\u0442\u043d\u0430 \u043a\u0430\u043f\u0443\u0441\u0442\u0430 \u0439 \u043a\u0443\u0447\u0435\u0440\u044f\u0432\u0430 \u043a\u0430\u043f\u0443\u0441\u0442\u0430, \u0430\u043b\u0435 \u0431\u0435\u0437 \u0434\u0436\u0435\u0440\u0435\u043b.\n", "topic": "ukrainian", "url": "https://ukrainian.stackexchange.com/questions/5486"}, {"image": "VQAonline_00064747.png", "question": "\u0427\u0438 \u043c\u043e\u0436\u043d\u0430 \u0433\u0430\u0434\u0430\u0442\u0438 \u043d\u0430 \u0406\u0432\u0430\u043d\u0430 \u041a\u0443\u043f\u0430\u043b\u0430?", "context": "\u041d\u0430 \u041c\u043e\u0432\u0430 \u2014 \u0414\u041d\u041a \u043d\u0430\u0446\u0456\u0457\n\u0437\u0456\u0442\u043a\u043d\u0443\u043b\u0430\u0441\u044c \u0437 \u0442\u0430\u043a\u043e\u044e \u043a\u0430\u0440\u0442\u0438\u043d\u043a\u043e\u044e\n\n\u0412 \u0434\u0430\u043d\u043e\u043c\u0443 \u0432\u0438\u043f\u0430\u0434\u043a\u0443 \u043f\u043e\u043a\u0430\u0437\u0430\u043d\u043e, \u0449\u043e \u0433\u0430\u0434\u0430\u0442\u0438 \u043d\u0435 \u043c\u043e\u0436\u0435 \u0431\u0443\u0442\u0438 \u0441\u0438\u043d\u043e\u043d\u0456\u043c\u043e\u043c \u0441\u043b\u043e\u0432\u0430 \u0432\u043e\u0440\u043e\u0436\u0438\u0442\u0438. \u0410\u043b\u0435 \u0432 \u0421\u0423\u041c-11 \u0441\u043b\u043e\u0432\u043e \u0433\u0430\u0434\u0430\u0442\u0438 \u0442\u0430 \u0432\u043e\u0440\u043e\u0436\u0438\u0442\u0438 \u0441\u0438\u043d\u043e\u043d\u0456\u043c\u0438. \u0422\u043e \u0436 \u0447\u0438 \u043f\u0440\u0430\u0432\u0438\u043b\u044c\u043d\u043e \u0433\u0430\u0434\u0430\u0442\u0438 \u043d\u0430 \u0406\u0432\u0430\u043d\u0430 \u041a\u0443\u043f\u0430\u043b\u0430?\n", "answer": "\u0412 \u0410\u043a\u0430\u0434\u0435\u043c\u0456\u0447\u043d\u043e\u043c\u0443 \u0442\u043b\u0443\u043c\u0430\u0447\u043d\u043e\u043c\u0443 \u0441\u043b\u043e\u0432\u043d\u0438\u043a\u0443 (1970\u20141980) \u043d\u0430\u0432\u0435\u0434\u0435\u043d\u043e \u0442\u0430\u043a\u0456 \u0437\u043d\u0430\u0447\u0435\u043d\u043d\u044f \u0434\u0456\u0454\u0441\u043b\u043e\u0432\u0430 \u0433\u0430\u0434\u0430\u0442\u0438:\n\n\u0413\u0410\u0414\u0410\u0422\u0418 1, \u0430\u044e, \u0430\u0454\u0448, \u043d\u0435\u0434\u043e\u043a.\n\n\u043f\u0440\u043e \u043a\u043e\u0433\u043e \u2014 \u0449\u043e \u0456 \u0431\u0435\u0437 \u0434\u043e\u0434\u0430\u0442\u043a\u0430. \u0414\u0443\u043c\u0430\u0442\u0438, \u0440\u043e\u0437\u043c\u0456\u0440\u043a\u043e\u0432\u0443\u0432\u0430\u0442\u0438. // \u041c\u0440\u0456\u044f\u0442\u0438.\n\u0413\u0430\u0434\u0430\u0442\u0438 \u0434\u0443\u043c\u0443 (\u0434\u0443\u043c\u043a\u0443, \u0433\u0430\u0434\u043a\u0443) \u2014 \u0442\u0435 \u0441\u0430\u043c\u0435, \u0449\u043e \u0414\u0443\u043c\u0430\u0442\u0438 \u0434\u0443\u043c\u0443\n(\u0434\u0443\u043c\u0438, \u0434\u0443\u043c\u043a\u0443) (\u0434\u0438\u0432. \u0434\u0443\u043c\u0430\u0442\u0438).\n\n\u043f\u0435\u0440\u0435\u0432. \u0437 \u0441\u043f\u043e\u043b. \u0449\u043e. \u041c\u0430\u0442\u0438 \u0434\u0443\u043c\u043a\u0443, \u043c\u0456\u0440\u043a\u0443\u0432\u0430\u043d\u043d\u044f \u0437 \u044f\u043a\u043e\u0433\u043e-\u043d\u0435\u0431\u0443\u0434\u044c \u043f\u0440\u0438\u0432\u043e\u0434\u0443; \u0432\u0432\u0430\u0436\u0430\u0442\u0438.\n// \u041f\u0435\u0440\u0435\u0434\u0431\u0430\u0447\u0430\u0442\u0438, \u043f\u0440\u0438\u043f\u0443\u0441\u043a\u0430\u0442\u0438.\n\n\u0437 \u0456\u043d\u0444\u0456\u043d. \u041c\u0430\u0442\u0438 \u043d\u0430\u043c\u0456\u0440 \u0449\u043e-\u043d\u0435\u0431\u0443\u0434\u044c \u0440\u043e\u0431\u0438\u0442\u0438. // \u0421\u043f\u043e\u0434\u0456\u0432\u0430\u0442\u0438\u0441\u044f.\n\n\n\u0413\u0410\u0414\u0410\u0422\u0418 2, \u0430\u044e, \u0430\u0454\u0448, \u043d\u0435\u0434\u043e\u043a., \u0437\u0430\u0441\u0442. \u0412\u043e\u0440\u043e\u0436\u0438\u0442\u0438.\n\n\u0421\u0435\u0440\u0435\u0434 \u0440\u043e\u0441\u0456\u0439\u0441\u044c\u043a\u043e-\u0443\u043a\u0440\u0430\u0457\u043d\u0441\u044c\u043a\u0438\u0445 \u0441\u043b\u043e\u0432\u043d\u0438\u043a\u0456\u0432 \u0437\u043d\u0430\u0445\u043e\u0434\u0438\u043c\u043e \u0433\u0430\u0434\u0430\u0442\u0438 \u044f\u043a \u0432\u0456\u0434\u043f\u043e\u0432\u0456\u0434\u043d\u0438\u043a \u0440\u043e\u0441\u0456\u0439\u0441\u044c\u043a\u043e\u043c\u0443 \u0433\u0430\u0434\u0430\u0442\u044c \u043b\u0438\u0448\u0435 \u0443 \u0421\u043b\u043e\u0432\u0430\u0440\u0456 \u0440\u043e\u0441\u0438\u0439\u0441\u044c\u043a\u043e-\u0443\u043a\u0440\u0430\u0457\u043d\u0441\u044c\u043a\u043e\u043c\u0443 1893\u20131898\u0440\u0440. (\u041c.\u0423\u043c\u0430\u043d\u0435\u0446\u044c, \u0410.\u0421\u043f\u0456\u043b\u043a\u0430.)\n\u041b\u043e\u0433\u0456\u0447\u043d\u0438\u043c \u0432\u0438\u0441\u043d\u043e\u0432\u043a\u043e\u043c \u0454 \u0442\u0430\u043a\u0438\u0439: \u0434\u0456\u0454\u0441\u043b\u043e\u0432\u043e \u0433\u0430\u0434\u0430\u0442\u0438 \u0443 \u0437\u043d\u0430\u0447\u0435\u043d\u043d\u0456 \u0432\u043e\u0440\u043e\u0436\u0438\u0442\u0438 \u0434\u043b\u044f \u0443\u043a\u0440\u0430\u0457\u043d\u0441\u044c\u043a\u043e\u0457 \u043c\u043e\u0432\u0438 \u0454 \u0437\u0430\u0441\u0442\u0430\u0440\u0456\u043b\u0438\u043c, \u0442\u043e\u043c\u0443 \u0456 \u043d\u0435 \u0440\u0435\u043a\u043e\u043c\u0435\u043d\u0434\u0443\u0454\u0442\u044c\u0441\u044f \u0434\u043b\u044f \u0432\u0436\u0438\u0432\u0430\u043d\u043d\u044f.\n", "topic": "ukrainian", "url": "https://ukrainian.stackexchange.com/questions/3440"}, {"image": "VQAonline_00064755.png", "question": "\u0428\u0430\u043c\u043f\u0456\u043d\u044c\u0439\u043e\u043d \u0447\u0438 \u043f\u0435\u0447\u0435\u0440\u0438\u0446\u044f?", "context": "\u0421\u0423\u041c-11 \u043c\u0456\u0441\u0442\u0438\u0442\u044c \u043e\u0431\u0438\u0434\u0432\u0430 \u0441\u043b\u043e\u0432\u0430\n\n\u0428\u0430\u043c\u043f\u0456\u043d\u044c\u0439\u043e\u043d \u0422\u0435 \u0441\u0430\u043c\u0435, \u0449\u043e \u043f\u0435\u0447\u0435\u0440\u0438\u0446\u044f 1.\n\u041f\u0435\u0447\u0435\u0440\u0438\u0446\u044f 1. \u0407\u0441\u0442\u0456\u0432\u043d\u0438\u0439 \u0433\u0440\u0438\u0431 \u0440\u043e\u0434\u0438\u043d\u0438 \u043f\u043b\u0430\u0441\u0442\u0438\u043d\u043d\u0438\u043a\u043e\u0432\u0438\u0445 \u0456\u0437 \u0441\u0456\u0440\u043e\u044e \u0430\u0431\u043e\n \u0431\u0456\u043b\u0443\u0432\u0430\u0442\u043e\u044e \u0448\u0430\u043f\u0438\u043d\u043a\u043e\u044e \u043d\u0430 \u0442\u043e\u043d\u043a\u0456\u0439 \u0431\u0456\u043b\u0456\u0439 \u043d\u0456\u0436\u0446\u0456, \u044f\u043a\u0438\u0439 \u0440\u043e\u0441\u0442\u0435 \u043d\u0430 \u043f\u043e\u043b\u044f\u0445, \u043b\u0443\u043a\u0430\u0445,\n \u0433\u043e\u0440\u043e\u0434\u0430\u0445, \u0443 \u0441\u0430\u0434\u043a\u0430\u0445 \u0456 \u0442. \u0456\u043d.; \u0448\u0430\u043c\u043f\u0456\u043d\u044c\u0439\u043e\u043d.\n\n\u041d\u0430 \u0441\u0430\u0439\u0442\u0456 \u041c\u043e\u0432\u0430 - \u0414\u041d\u041a \u043d\u0430\u0446\u0456\u0457, \u043d\u0430\u0433\u043e\u043b\u043e\u0448\u0443\u044e\u0442\u044c, \u0449\u043e \u043f\u0440\u0430\u0432\u0438\u043b\u044c\u043d\u043e \u043b\u0438\u0448\u0435 \u043f\u0435\u0447\u0435\u0440\u0438\u0446\u044f.\n \n\u0427\u0438 \u043f\u043e\u0442\u0440\u0456\u0431\u043d\u043e \u0432\u0456\u0434\u043c\u043e\u0432\u0438\u0442\u0438\u0441\u044c \u0432\u0456\u0434 \u0432\u0438\u043a\u043e\u0440\u0438\u0441\u0442\u0430\u043d\u043d\u044f \u0441\u043b\u043e\u0432\u0430 \"\u0448\u0430\u043c\u043f\u0456\u043d\u044c\u0439\u043e\u043d\" \u0434\u043b\u044f \u043e\u0437\u043d\u0430\u0447\u0435\u043d\u043d\u044f \u0433\u0440\u0438\u0431\u0430? \u0427\u0438 \u043c\u043e\u0436\u043d\u0430 \u0432\u0438\u043a\u043e\u0440\u0438\u0441\u0442\u043e\u0432\u0443\u0432\u0430\u0442\u0438 \u043e\u0431\u0438\u0434\u0432\u0430 \u0441\u043b\u043e\u0432\u0430, \u044f\u043a \u0432\u0437\u0430\u0454\u043c\u043e\u0437\u0430\u043c\u0456\u043d\u0456?\n", "answer": "\"\u0428\u0430\u043c\u043f\u0456\u043d\u044c\u0439\u043e\u043d\" - \u0446\u0435 \u0437\u0430\u043f\u043e\u0437\u0438\u0447\u0435\u043d\u0435 \u0441\u043b\u043e\u0432\u043e (\u0432\u0456\u0434 \u0444\u0430\u043d\u0446. \"champignon\", \u0449\u043e \u043e\u0437\u043d\u0430\u0447\u0430\u0454 \u0433\u0440\u0438\u0431 (\u0434\u0438\u0432. \u0412\u0456\u043a\u0456\u043f\u0435\u0434\u0456\u044e).\n\u0410 \u043e\u0442 \u0441\u043b\u043e\u0432\u043e \"\u043f\u0435\u0447\u0435\u0440\u0438\u0446\u044f\" (\u0415\u0421\u0423\u041c, \u0441\u0442 364 \u041f\u0414\u0424) \u0437\u0430 \u043f\u043e\u0445\u043e\u0434\u0436\u0435\u043d\u043d\u044f\u043c \u0431\u043b\u0438\u0436\u0447\u0435 \u0434\u043e \u043d\u0430\u0448\u043e\u0457 \u043c\u043e\u0432\u0438. \u0425\u043e\u0447 \u043d\u0435\u043c\u0430\u0454 \u043e\u0434\u043d\u043e\u0437\u043d\u0430\u0447\u043d\u043e\u0457 \u0432\u0435\u0440\u0441\u0456\u0457 \u044f\u043a \u0432\u043e\u043d\u043e \u0437'\u044f\u0432\u0438\u043b\u043e\u0441\u044f (\u0447\u0438 \u0442\u043e \u0432\u0456\u0434 \u0441\u043b\u043e\u0432\u0430 \"\u043f\u0435\u0447\u0435\u0440\u0430\", \u0447\u0438 \u0442\u043e \u0432\u0456\u0434 \u0441\u043b\u043e\u0432\u0430 \"\u043f\u0435\u043a\u0442\u0438\" \u0447\u0438 \u043d\u0430\u0432\u0456\u0442\u044c \u0432\u0456\u0434 \u0441\u043b\u043e\u0432\u043e\u0441\u043f\u043e\u043b\u0443\u0447\u0435\u043d\u043d\u044f \"\u043f\u0435\u0447\u0435\u043d\u044f \u0442\u0443\u0440\u043e\u0432\u0430\").\n\u041f\u043e\u0434\u0438\u0432\u0456\u0442\u044c\u0441\u044f \u043d\u0430\u0437\u0432\u0443 \u0440\u043e\u0437\u0434\u0456\u043b\u0443 \u041c\u043e\u0432\u0430 \u0414\u041d\u041a \u043d\u0430\u0446\u0456\u0457 - \"\u0410\u043d\u0442\u0438\u0441\u0443\u0440\u0436\u0438\u043a\". \u041e\u0442\u0436\u0435, \u044f\u043a\u0449\u043e \u0432\u0438 \u0445\u043e\u0447\u0435\u0442\u0435 \u0443\u043d\u0438\u043a\u0430\u0442\u0438 \u0441\u0443\u0440\u0436\u0438\u043a\u0430, \u0442\u043e \u0432\u0430\u043c \u0441\u043b\u0456\u0434 \u0437\u0430\u043c\u0456\u043d\u0438\u0442\u0438 \u0441\u043b\u043e\u0432\u043e \"\u0448\u0430\u043c\u043f\u0456\u043d\u044c\u0439\u043e\u043d\" \u043d\u0430 \"\u043f\u0435\u0447\u0435\u0440\u0438\u0446\u044f\". \u041e\u0434\u043d\u0430\u043a \u0441\u0430\u043c\u0435 \u0432\u0436\u0438\u0432\u0430\u043d\u043d\u044f \u0441\u043b\u043e\u0432\u0430 \"\u0448\u0430\u043c\u043f\u0456\u043d\u044c\u0439\u043e\u043d\" \u043d\u0435 \u0431\u0443\u0434\u0435 \u043f\u043e\u043c\u0438\u043b\u043a\u043e\u044e, \u0431\u043e \u0432\u043e\u043d\u043e \u0432\u0441\u0435 \u0436 \u0432\u043e\u043d\u043e \u0454 \u0432 \u0421\u0423\u041c\u0456 \u0456 \u0431\u0456\u043b\u044f \u043d\u044c\u043e\u0433\u043e \u043d\u0435\u043c\u0430\u0454 \u043d\u0456\u044f\u043a\u0438\u0445 \u043f\u043e\u0437\u043d\u0430\u0447\u043e\u043a, \u044f\u043a\u0456 \u0431 \u043d\u0435 \u0434\u043e\u0437\u0432\u043e\u043b\u044f\u043b\u0438 \u0439\u043e\u0433\u043e \u0432\u0436\u0438\u0432\u0430\u0442\u0438.\n", "topic": "ukrainian", "url": "https://ukrainian.stackexchange.com/questions/5299"}, {"image": "VQAonline_00064736.png", "question": "\u042f\u043a \u043f\u0435\u0440\u0435\u043a\u043b\u0430\u0441\u0442\u0438 \u0437 \u0430\u043d\u0433\u043b\u0456\u0439\u0441\u044c\u043a\u043e\u0457 - viewing frustum?", "context": "\u0417\u0430\u0440\u0430\u0437 \u0437\u0430\u0439\u043c\u0430\u044e\u0441\u044f \u043f\u0435\u0440\u0435\u043a\u043b\u0430\u0434\u043e\u043c \u0441\u0430\u0439\u0442\u0443 \u0437 \u043f\u0440\u043e\u0433\u0440\u0430\u043c\u0443\u0432\u0430\u043d\u043d\u044f \u043a\u043e\u043c\u043f'\u044e\u0442\u0435\u0440\u043d\u043e\u0457 \u0433\u0440\u0430\u0444\u0456\u043a\u0438. \u041f\u043b\u0430\u043d\u0443\u044e \u0437\u0440\u043e\u0431\u0438\u0442\u0438 gitbook \u043d\u0430 \u043e\u0441\u043d\u043e\u0432\u0456 \u043f\u0435\u0440\u0435\u043a\u043b\u0430\u0434\u0443. \u0410\u0434\u0440\u0435\u0441\u0430 \u0441\u0442\u043e\u0440\u0456\u043d\u043a\u0438. \n\u0421\u043b\u043e\u0432\u043e\u0441\u043f\u043e\u043b\u0443\u0447\u0435\u043d\u043d\u044f viewing frustum \u0441\u0442\u043e\u0441\u0443\u0454\u0442\u044c\u0441\u044f \u043f\u0456\u0440\u0430\u043c\u0456\u0434\u0438, \u0430\u043b\u0435 \u044f \u043d\u0435 \u0437\u043d\u0430\u044e \u044f\u043a \u0442\u043e\u0447\u043d\u043e \u0446\u0435 \u043f\u0435\u0440\u0435\u043a\u043b\u0430\u0441\u0442\u0438.\n\u0411\u0443\u0434\u0443 \u0440\u0430\u0434\u0438\u0439 \u043f\u043e\u043c\u0456\u0447\u0456)\n\n\u041e\u043f\u0438\u0441 \u0434\u043e \u0440\u0438\u0441\u0443\u043d\u043a\u0443:\n\n\u042f\u043a\u0449\u043e \u0432\u0438 \u0437\u0432'\u044f\u0436\u0435\u0442\u0435 \u043a\u0443\u0442\u0438 \u043a\u0430\u043d\u0432\u0438 \u0437 \u043e\u043a\u043e\u043c, \u044f\u043a\u0435 \u0437\u0430 \u0437\u0430\u043c\u043e\u0432\u0447\u0430\u043d\u043d\u044f\u043c \u0432\u0438\u0440\u0456\u0432\u043d\u044f\u043d\u0435 \u0456\u0437 \u043d\u0430\u0448\u043e\u044e \u0434\u0435\u043a\u0430\u0440\u0442\u043e\u0432\u043e\u044e \u0441\u0438\u0441\u0442\u0435\u043c\u043e\u044e \u043a\u043e\u043e\u0440\u0434\u0438\u043d\u0430\u0442, \u0456 \u043f\u0440\u043e\u0434\u043e\u0432\u0436\u0438\u0442\u0435 \u043b\u0456\u043d\u0456\u0457 \u0434\u0430\u043b\u0456 \u0432\u0433\u043b\u0438\u0431 \u0441\u0446\u0435\u043d\u0438, \u0432\u0438 \u043e\u0442\u0440\u0438\u043c\u0430\u0454\u0442\u0435 \u043f\u0435\u0432\u043d\u043e\u0433\u043e \u0440\u043e\u0434\u0443 \u043f\u0456\u0440\u0430\u043c\u0456\u0434\u0443, \u044f\u043a\u0443 \u043c\u0438 \u0437\u0432\u0435\u043c\u043e viewing frustum. \u0411\u0443\u0434\u044c-\u044f\u043a\u0438\u0439 \u043e\u0431'\u0454\u043a\u0442 \u0432\u0441\u0435\u0440\u0435\u0434\u0438\u043d\u0456 frustum (\u0430\u0431\u043e \u0442\u0430\u043a\u0438\u0439, \u0449\u043e \u043f\u0435\u0440\u0435\u0442\u0438\u043d\u0430\u0454 \u0439\u043e\u0433\u043e) \u0432\u0432\u0430\u0436\u0430\u0454\u0442\u044c\u0441\u044f \u0432\u0438\u0434\u0438\u043c\u0438\u043c \u0456 \u0437'\u044f\u0432\u0438\u0442\u044c\u0441\u044f \u043d\u0430 \u0437\u043e\u0431\u0440\u0430\u0436\u0435\u043d\u0456.\n\n", "answer": "\u042f\u043a\u0449\u043e \u0434\u043e\u0441\u043b\u0456\u0432\u043d\u043e \u2014 \u0437\u0440\u0456\u0437\u0430\u043d\u0438\u0439 \u043a\u043e\u043d\u0443\u0441 (\u0430\u0431\u043e \u0437\u0440\u0456\u0437\u0430\u043d\u0430 \u043f\u0456\u0440\u0430\u043c\u0456\u0434\u0430) \u0437\u043e\u0440\u0443 (/\u043e\u0433\u043b\u044f\u0434\u0443/\u043f\u043e\u0433\u043b\u044f\u0434\u0443/\u0431\u0430\u0447\u0435\u043d\u043d\u044f/\u0442\u043e\u0449\u043e).\n\u042f\u043a\u0449\u043e \u043f\u0440\u0438\u0431\u043b\u0438\u0437\u043d\u043e \u0437\u0430 \u0441\u0435\u043d\u0441\u043e\u043c \u0456 \u0442\u043e\u0447\u043d\u0456\u0441\u0442\u044c \u043d\u0435 \u0442\u0430\u043a\u0430 \u0432\u0430\u0436\u043b\u0438\u0432\u0430 \u2014 \u043c\u043e\u0436\u0435, \u043f\u043e\u043b\u0435 \u0437\u043e\u0440\u0443?\nUpdate 1: \u0404 \u0442\u0435\u0440\u043c\u0456\u043d \u043a\u043e\u043d\u0443\u0441 \u0437\u043e\u0440\u0443 (\u0440\u043e\u0441. \u043a\u043e\u043d\u0443\u0441 \u0437\u0440\u0435\u043d\u0438\u044f) \u2014 \u0430\u043b\u0435 \u0447\u0438 \u0442\u0435 \u0432\u0456\u043d \u043e\u0437\u043d\u0430\u0447\u0430\u0454, \u043d\u0435\u0432\u0456\u0434\u043e\u043c\u043e.\nUpdate 2: \u0420\u043e\u0441\u0456\u0439\u0441\u044c\u043a\u043e\u044e \u043f\u0435\u0440\u0435\u043a\u043b\u0430\u0434\u0430\u044e\u0442\u044c \u044f\u043a \u043f\u0438\u0440\u0430\u043c\u0438\u0434\u0430 \u0432\u0438\u0434\u0438\u043c\u043e\u0441\u0442\u0438 (\u0456\u043d\u043e\u0434\u0456 \u043a\u043e\u043d\u0443\u0441 \u0432\u0438\u0434\u0438\u043c\u043e\u0441\u0442\u0438) \u2014 \u043d\u0430\u043f\u0440\u0438\u043a\u043b\u0430\u0434: \u00ab\u0438\u0441\u043f\u043e\u043b\u044c\u0437\u043e\u0432\u0430\u043d\u0438\u0435 \u043d\u0430\u043a\u043b\u043e\u043d\u043d\u043e\u0439 \u043f\u0438\u0440\u0430\u043c\u0438\u0434\u044b \u0432\u0438\u0434\u0438\u043c\u043e\u0441\u0442\u0438 (Oblique Frustum)\u00bb.\nUpdate 3: \u0423\u043a\u0440\u0430\u0457\u043d\u0441\u044c\u043a\u043e\u044e \u0442\u0435\u0436 \u043a\u0430\u0436\u0443\u0442\u044c \u043f\u0456\u0440\u0430\u043c\u0456\u0434\u0430 \u0432\u0438\u0434\u0438\u043c\u043e\u0441\u0442\u0456 \u0447\u0438 \u043a\u043e\u043d\u0443\u0441 \u0432\u0438\u0434\u0438\u043c\u043e\u0441\u0442\u0456 \u2014 \u043c\u043e\u0436\u043b\u0438\u0432\u043e, \u0449\u043e \u0446\u0435 \u043f\u043e\u0448\u0438\u0440\u0435\u043d\u0438\u0439/\u0437\u0430\u0433\u0430\u043b\u044c\u043d\u043e\u043f\u0440\u0438\u0439\u043d\u044f\u0442\u0438\u0439 \u0442\u0435\u0440\u043c\u0456\u043d.\n\u041f\u0440\u0438\u0447\u043e\u043c\u0443 \u043a\u043e\u043d\u0443\u0441 \u0432\u0438\u0434\u0438\u043c\u043e\u0441\u0442\u0456 \u2014 \u043d\u0430\u0432\u0456\u0442\u044c \u0447\u0430\u0441\u0442\u0456\u0448\u0435 \u0437\u0430 \u043f\u0456\u0440\u0430\u043c\u0456\u0434\u0443 \u0432\u0438\u0434\u0438\u043c\u043e\u0441\u0442\u0456.\n\u041f\u043e\u0441\u0456\u0431\u043d\u0438\u043a \u00ab\u0413\u0435\u043e\u043c\u0435\u0442\u0440\u0438\u0447\u043d\u0435 \u043c\u043e\u0434\u0435\u043b\u044e\u0432\u0430\u043d\u043d\u044f \u0456 \u043a\u043e\u043c\u043f\u2019\u044e\u0442\u0435\u0440\u043d\u0430 \u0433\u0440\u0430\u0444\u0456\u043a\u0430: \u0432\u0438\u043a\u043e\u0440\u0438\u0441\u0442\u0430\u043d\u043d\u044f \u0431\u0456\u0431\u043b\u0456\u043e\u0442\u0435\u043a\u0438 OpenGL\u00bb (\u0410.\u00a0\u0410.\u00a0\u041b\u044f\u0449\u0435\u043d\u043a\u043e, \u0412.\u00a0\u0412.\u00a0\u0414\u0435\u043c\u0447\u0435\u043d\u043a\u043e, \u0404.\u00a0\u0412.\u00a0\u0411\u043e\u0440\u043e\u0434\u0430\u0432\u043a\u0430, \u0412.\u00a0\u0412.\u00a0\u0421\u043c\u0438\u0440\u043d\u043e\u0432; \u041a\u0438\u0457\u0432\u0441\u044c\u043a\u0438\u0439 \u043d\u0430\u0446\u0456\u043e\u043d\u0430\u043b\u044c\u043d\u0438\u0439 \u0443\u043d\u0456\u0432\u0435\u0440\u0441\u0438\u0442\u0435\u0442 \u0431\u0443\u0434\u0456\u0432\u043d\u0438\u0446\u0442\u0432\u0430 \u0456 \u0430\u0440\u0445\u0456\u0442\u0435\u043a\u0442\u0443\u0440\u0438; 2008, 2009), \u0430 \u0440\u0430\u0437\u043e\u043c \u0437 \u043d\u0438\u043c \u0456 \u0412\u0456\u043a\u0456\u043f\u0435\u0434\u0456\u044f, \u043a\u0430\u0436\u0435 [\u0437\u0440\u0456\u0437\u0430\u043d\u0438\u0439] \u043e\u0431'\u0454\u043c \u0432\u0438\u0434\u0438\u043c\u043e\u0441\u0442\u0456 \u2014 \u0432 \u0437\u0430\u0433\u0430\u043b\u044c\u043d\u043e\u043c\u0443 \u0432\u0438\u043f\u0430\u0434\u043a\u0443, \u0430 \u044f\u043a \u0447\u0430\u0441\u0442\u043a\u043e\u0432\u0456:\n\n[\u0437\u0440\u0456\u0437\u0430\u043d\u0438\u0439] \u043f\u0430\u0440\u0430\u043b\u0435\u043f\u0456\u043f\u0435\u0434 \u0432\u0438\u0434\u0438\u043c\u043e\u0441\u0442\u0456 \u2014 \u0443 \u0432\u0438\u043f\u0430\u0434\u043a\u0443 \u043e\u0440\u0442\u043e\u0433\u0440\u0430\u0444\u0456\u0447\u043d\u043e\u0457 \u043f\u0440\u043e\u0435\u043a\u0446\u0456\u0457;\n[\u0437\u0440\u0456\u0437\u0430\u043d\u0438\u0439] \u043a\u043e\u043d\u0443\u0441 \u0432\u0438\u0434\u0438\u043c\u043e\u0441\u0442\u0456 \u2014 \u0443 \u0432\u0438\u043f\u0430\u0434\u043a\u0443 \u043f\u0435\u0440\u0441\u043f\u0435\u043a\u0442\u0438\u0432\u0438.\n\n", "topic": "ukrainian", "url": "https://ukrainian.stackexchange.com/questions/1239"}, {"image": "VQAonline_00065103.png", "question": "Arbor nut keeps working itself loose on my table saw. What am I doing wong?", "context": "I recently bought a new Dewalt DWE7491RS table saw (pictured below). And on two separate occasions the arbor nut has come loose on me. Once with a dado stack that threw blades across the room! (Very SCARY!)\nThe second time it was when I was taking a standard cross-cut blade off and noticed the tension on the nut was very loose, although the blade hadn't slipped out of place yet.\nI'm almost certain that I am tightening the nut down pretty well; probably more than I should. And I do put the washer on the nut between the blade and the nut. This is also with all stock parts that are less than a month old.\nAny ideas what might be causing this to happen?\n\n", "answer": "Once the arbour nut is tight, the rotation of the spindle will tend to tighten it. So you just have to make sure it is torqued down appropriately.\nWell, it is the rotation into the material that tends to tighten the arbor nut. Once powered down and free-spinning the mass of the blade is now free to loosen the arbor nut, which is why dado sets and saws with brakes may tend to loosen the arbor. The model described here doesn't seem to have a brake, but I suppose a valid check is to make sure the blade or blade stack isn't exceeding their recommendations. (Though, they usually discuss blade size in terms of the the riving knife installed rather than mass.)\nThis is how most saws like to be treated:\n\nMake sure there is no oil or grease on the nut or the arbour. Make sure the entire spindle and all the spacers and washers are free of sawdust and other debris. If this is a new saw, I'd carefully degrease the entire spindle and blade hardware because it might still be covered in a factory anti-rust coating.\nStack up your spacers or blades correctly; there is usually an identical cupped washer on both sides of the blade, for example. What we don't want is mismatched torque on either side of the blade or blade stack.\nThread the nut on finger-tight, making sure all the slack is taken up as much as possible.\nEither lock the spindle or use two wrenches to tighten the arbour a 1/8 to 1/4 turn. The idea is to snug things down fully without stretching the threads too much -- we want to be within the natural elasticity of the spindle metal.\n\nThat should be it. If it is working loose after torquing it down then there is a problem with the arbour nut, washers or spindle.\n\nMake sure the blade is torqued down evenly. Spacers or washers must be the same mass and dimensions.\nSee if the spindle (or arbour nut) threads are obviously crossed or stripped out. Maybe they are stripped toward the end and you aren't getting a proper torque.\nIf you have over-torqued the arbour nut in the past, you may have stretched the threads on the spindle. There are gauges to check for this, but the idea is that the threads should be the same distance across the spindle. Look closely at the place where the nut tightens for really shiny spots or obvious metal movement.\nCheck for excessive run-out or vibration. Make sure the bearings aren't failing. There are ways to check for run-out without an indicator which are easy to find on-line. Vibration should be obvious while the saw is running at speed under no load. The unpowered spindle should turn easily with your fingers with no \"grumble\".\n\n", "topic": "woodworking", "url": "https://woodworking.stackexchange.com/questions/10582"}, {"image": "VQAonline_00064915.png", "question": "Would an inlay affect the structure of an end grain cutting board?", "context": "I am in the process of creating an end grain cutting board which is made up of Maple only, but would like to decorate it by inlaying other types of wood into it (Walnut for example). The cutting board size is 18\" by 12\" and 1.2\" thick, and the inlay will be near the a corner of the board with a size of 5\" by 5\" and .35\" deep. Would having this inlay affect the structural integrity of the board? \nI am doing the inlay by using the v-inlay method as described in the image below at and angle of 45 degrees which most probably means that the flat base of the inlay wont be touching the board, just the edges:\n\n", "answer": "\nWould having this inlay affect the structural integrity of the board? \n\nI wouldn't have thought so judging from the size you indicate. It will locally affect the strength, but not enough to be a major factor I would have thought (in light of the following).\n\nwhich most probably means that the flat base of the inlay wont be touching the board, just the edges:\n\nThis is a problem I think. The edges of both the inlay and the rebate are end grain, and joints of end grain to end grain are the weakest glue joint. I would highly recommend laying a bed of epoxy underneath the inlay, so that it basically fills the void underneath. Not only will this hugely improve the bond to the inlay it will add strength to that part of the board as well so you get a two-for-one benefit.\n", "topic": "woodworking", "url": "https://woodworking.stackexchange.com/questions/2329"}, {"image": "VQAonline_00065001.png", "question": "FIlling up gaps in an old wooden floor", "context": "I'm currently in the process of renewing an old wooden floor. What would one use to fill in the gaps between the planks?\n\nI've asked in the local home repair store and they gave me a silicone like mass that resembles the color of the wood.\nThat seems to work ok for the nail holes, but is kind of soft and somewhat expensive for filling out all space between the boards.\nI have seen some youtube videos in which people are mixing sand dust with either wood glue or some special liquid. Does anyone know something on the pros and conns of these approaches?\n", "answer": "\nI'm currently in the process of renewing an old wooden floor. What would one use to fill in the gaps between the planks?\n\nNothing unless there's a draft problem. The gaps aren't generally an issue except visually but they can make for a cold floor depending on the rest of the structure of the house and how much air movement can take place through the floor.\nSome argue such gaps are supposed to be there, and if you look at old floors 99 times out of 100 you'll see gaps between boards, sometimes quite wide ones, and in many cases there's no filler and no trace that there ever was any present.\nIn the past where gaps were filled they used various methods, including the same way gaps were dealt with on ship decks, using oakum and tar (!), as well as with melted wax or wax/resin compositions (also used as a filler material for furniture in this era).\nSee more discussion on the issue and solutions at these links:\nHow to Fill Gaps In a Wide-Plank Wood Floor on This Old House.\nMy home's pine flooring has large gaps in between some of the planks, especially in the kitchen. What can be done to fill or seal them? on Quora.\nHow to fill gaps in a wood floor? on HomeOwnersHub.\nFiller for ancient wood floors? on the WoodenBoat forum.\n\nI've asked in the local home repair store and they gave me a silicone like mass that resembles the color of the wood.\nThat seems to work ok for the nail holes\n\nThat stuff is no good for filling nail holes on a floor. This type of flexible filler is for filling gaps that require some movement to take place without cracking, as in gappy installations of skirting to walls, one wooden moulding piece to the next where some shrinkage and expansion will take place through the year.\nIf you must fill the nail holes (you don't have to, it's quite normal for them not to be filled) you want a rigid filler and ideally a strong one if it's to take direct foot traffic, i.e. if there's no carpet going on or any rugs being put down.\n", "topic": "woodworking", "url": "https://woodworking.stackexchange.com/questions/5619"}, {"image": "VQAonline_00065156.png", "question": "Will the center have enough support for my custom desk/table?", "context": "I'm trying to build my first large desk for my computer setup. My plan is to set a 74\" \u00d7 42\" laminate counter top on top of four Alex drawers and I am concerned about the center of the table sagging or breaking without an apron for support. I drew a top down view of the table quickly and was wondering what I should add for support to strengthen the center if needed.\nThe countertop,\nThe drawers, The design\nMost of the weight will be located near the top of the table with three monitors. Other than that I'm planning on having just a keyboard, mouse, and maybe some light nick-knacks scattered throughout.\n\n", "answer": "Laminate countertop material is very stiff. 46\" is just over 1m and such spans are normal in many kitchen designs. After 1 year of use I expect laminate will deflect, but you will not notice (maybe 1-2mm).\nIf deflection can be seen after many months easy to install central leg same height as Alex drawers for support.\n", "topic": "woodworking", "url": "https://woodworking.stackexchange.com/questions/14063"}, {"image": "VQAonline_00065084.png", "question": "How to freshen wooden box", "context": "I have an old Chess set which has been in storage in what would appear to be less than ideal conditions. (Photo taken with flash.)\n\nOne half of the box (on one side only, ie the half that is not shown in the image is as fresh as the left side here) is very dull looking. Other than the colour there is no obvious damage. I do not know what has happened to it.\nWhat is the easiest way to achieve uniform appearance?\nEdit: the \"damaged\" side was uppermost I believe, and there were other things on top. I did not notice excessive amounts of dust around it. Wiping with alcohol does not have any effect.\n", "answer": "I would first clean the whole thing up with mineral spirits (not alcohol) and then try something like feed n wax on it. Definitely test this only on a small section (preferably a non visible section) first to see how it looks compared to the existing finish. \nAlternatively I would recommend a natural danish oil. I think any colorless oil finish will darken the damaged side enough to blend it right into the non-damaged side, and danish oil has the advantage of also being a varnish so it will provide additional protection from water.\nEither way whatever you use, you should try to test it on a small area that is not easily visible to see the results before finishing the entire piece.\n", "topic": "woodworking", "url": "https://woodworking.stackexchange.com/questions/9720"}, {"image": "VQAonline_00065044.png", "question": "Can this be a very simple cross cut sled with replaceable zero inserts?", "context": "I am trying to reuse an older crosscut sled.\nI will have problems with using the old cut for the blade so I googled for a solution and I came across a couple of complicated designs with replaceable zero inserts. Those would add another piece of plywood on top of the one that makes the sled base.\nSince my fences are very solid (both front and back) I could practically cut out the piece of plywood 1\" or 2\" to the left and right of the blade cut and install a replacement screwed to the fence as shown below. Wood nuts will be installed int the fence and then I will use screws to attach a new insert every time I need a new one like below and then make a new cut. I wonder what I am missing since I never seen this simple design\n", "answer": "This will work. This is how mine works. People like to make things very complicated... a lot of people like making jigs more than anything else.\n", "topic": "woodworking", "url": "https://woodworking.stackexchange.com/questions/7249"}, {"image": "VQAonline_00064978.png", "question": "Convert a minimalist bench design into a desk", "context": "I saw this tutorial and I really want to build it for a workspace area at my job. The measurements of this bench were 16\u2033 high x 40\u2033 long x 13.5\u2033 deep, but I'd like to make a table that's 29\" high x 107\" long x 20\" deep. The tutorial bench is built using a modified box joint with interlocking 2\"x4\" pieces that are glued together with liquid nails. Each of the 2\"x4\" pieces have a little bit removed on the 2\" side to create a square look with a table saw.\nAre any structural concerns with making the table this long? Would the table need to be reinforced underneath with hidden brackets? How much weight should it be able to hold?\nI appreciate your help!\n", "answer": "\nAre any structural concerns with making the table this long? \n\nNo. Even with a top much thinner than you're going to end up with you can have a >100\" span.\n\nWould the table need to be reinforced underneath with hidden brackets? \n\nNot at all, the interlocking box joint is very strong (lots of surface area for the glue) and the table will be naturally stiff.\n\nHow much weight should it be able to hold?\n\nYou could try using The Sagulator to calculate this, but it won't be easy since this isn't any material that it includes (because of the way the boards are oriented). Let's just say a lot.\nThis design orients the boards edge-up, which is using the wood at its maximum strength as regards resistance to sagging. Couple that with them being glued together what you're making here is in effect a laminated beam, so its strength is immense. I bet a heavy person could stand right in the centre and it wouldn't bow enough to notice.\nThere is a weight issue that you might not have thought of and that's the weight of the table itself. Obviously you like the chunky style but do be aware scaled up to table size this makes for a great mass of solid wood and it'll weight quite a bit and may be difficult to move around even just made from 2x material. Made from hardwood it would be a two-man or three-man job in a lot of cases!\n", "topic": "woodworking", "url": "https://woodworking.stackexchange.com/questions/4900"}, {"image": "VQAonline_00065185.png", "question": "Seating arrangements face to face makes me stressed. What can I do about it?", "context": "\nI had been rejecting jobs in the past which had tables with such arrangement like in the picture. I mean when there is no partition by which you could have privacy from another person seeing you constantly.\nBut the problem is - I see many big companies, which might be good jobs have such seating.\nIt is especially hard when I feel very stressed and feel shame to show I am so much stressed. And its bad when you have to give up good job just because seating arrangement.\nI usually avoid sitting that way in public transport also. Sometimes I try to sit for a bit - to train myself. Usually I get stressed after that, so when I know I need to feel relaxed, I avoid those training. This happens not with all people. Mostly happens with young, cool people. With old people its easier. Also with my family members its not big deal if its not too close face to face. Same is with girls, they are surprised why I avoid eye contact. But with some who I spend more time, I try to get eye contact and after long time I get used, also because I know that girl likes me the way I am if she is long time with me.\nSome say that I would get used. Maybe this is true. Maybe with some guys who like me I could get used. But if there sits a guy who does not like me, and is angry at me often, then I am really not sure. \nIn some companies they state that monitor will hide my face and people are not staring at me all the time, they have work and they look at their monitors. Which is true. \nI might hide behind monitor, but I am afraid that I would not start to hunch.\nAnd also in the IT industry I believe I should not be alone who has problem with eye contact. So I do not get why offices are that way when they know that not all IT guys are good at social interactions.\nUpdate\nMy question is - have you any suggestions how could I solve this problem? How to feel better? Or maybe solve it somehow technically?\n", "answer": "I have a seating arrangement like this (it isn't a problem for me), for other reasons I requested a 2nd monitor and two monitor stands. My screen wall effectively forms a partition across the desk. The person who sits across from me has three monitors so we actually have to actively stand up to hold a face to face conversation.\nIn all honesty every job I have ever worked in has this style of seating (UK, cultures may vary) so it is probably a very common problem and one you'll struggle to avoid. \n", "topic": "workplace", "url": "https://workplace.stackexchange.com/questions/71629"}, {"image": "VQAonline_00065180.png", "question": "Why would a job application ask about residency history?", "context": "Recently I looked at the online job application for Barnes & Noble (a US bookstore chain) and they asked for a list of your residence history: \n\nIt was not a required field, but this seemed like a very odd thing for me to ask, as the locations I have lived seem like they would have nothing to do with whether or not I would be a good worker. \nSo why would a job application ask for residence history?\n", "answer": "Its because, during the background check process, some companies will go through the criminal records and court cases against the combination of your name and address. If they find a match or a close match, they will come back to you.\nThis is important, reason being that, if a company processes your VISA for any other country, they need to be sure that you don't have any criminal records at any places.\n", "topic": "workplace", "url": "https://workplace.stackexchange.com/questions/56354"}, {"image": "VQAonline_00065208.png", "question": "Why would a US company tell their employees that they will pay for a union?", "context": "I recently read about a union effort in Amazon and a photo of an Amazon flyer was mentioning that accepting a union at Amazon means paying for it:\n\n(source)\nI am French and do not know how unions work in the US, so apologies if the question is obvious: are all employees of a company automatically paying for a union once there is one in the company?\nBy \"paying\", I mean \"having an entry on the payslip with \"union fees\"\".\nI can imagine that there are some indirect payments possible (the company must co-pay for the union, and this may ultimately mean that the salaries are lower (this is an extreme example)), or that some employees want to join the union (and pay their fees).\nThe flyer seems to suggest however that once a union is formed, everyone must pay for it. Is that correct?\n", "answer": "Employees pay something called \"union dues\" in the United States\nUnions are not free to run. They hire benefit administrators. They sometimes manage pensions. They definitely need to hire negotiators. They pay for this by charging their members a fee.\n\nare all employees of a company automatically paying for a union once there is one in the company?\n\nAll employees in the bargaining unit, which is nearly always smaller than the number of employees in the company and in this case much smaller. Bargaining units might form around a location, a particular skillset, or even job function.\n\nBy \"paying\", I mean \"having an entry on the payslip with \"union fees\"\".\n\nQuite often. The unions often prefer to deduct dues directly from paycheques.\n\nThe flyer seems to suggest however that once a union is formed, everyone must pay for it. Is that correct?\n\nDepends on the particular laws of the state. In some states, if a majority vote for a union, everyone must pay. In \"right-to-work\" states, official membership in the union is optional and so is paying union dues. So the union would still exist, it is just that a large number of people could opt-out of being members.\n", "topic": "workplace", "url": "https://workplace.stackexchange.com/questions/169883"}, {"image": "VQAonline_00065203.png", "question": "Can I take any recourse after being told to take down ' Xinjiang \"Reeducation\" Camps ' as my location?", "context": "Background\nToday at work, I got a talking to from my superior about something on my Skype for Business account being too political. Below is what caused the issue:\n[\nMy location was set to Xinjiang \"Reeducation\" Camp as an (admittedly) edgy joke. I understand this is unprofessional, but our work tends to be informal (we have rubber chickens in my work area). The thing is, I have seen other people put up funny things as locations like \"Where we are going, we won't need roads\" or something like \"Iraq\" as their location as a joke. Regardless, this is what my superior stated to me:\n\nAt a meeting my superior was at, someone at the meeting had me as their contact.\nThey saw that I had \"Xinjiang \"Reeducation\" Camps\" as my location.\nThey (presumably) told my manager to tell me to take this down as my location.\nMy manager meets with me one-on-one and tells me to take this down as soon as possible and \"be as bland as possible, we need to act in accordance with policies\", the typical schpiel. She mentioned she meant to told me earlier when I had \"Saudi Arabia\" as my location, but that she didn't get to it because it didn't stay up too long. \n\nSolution\nI took this down as requested, since this was a direct order from my manager and replaced it with a Star Trek reference (Romulan Empire Outpost). However, I don't understand why this is political. At my job, you are required to be a U.S. citizen. The Uighur Human Rights Policy Act of 2019 passed unanimously in the senate (meaning bascially all U.S. citizens are on board). The place I work also refers to Taiwan as Taiwan, not Chinese Taipei as the PRC is so apt to call it. When she asked me to explain I said \"it is a joke but is meant to refer to the Xinjiang Province concentration camps that China is using to oppress the Uighur minority\". She then asked me what word (Uighur) I used. She didn't even know what it was, but yet she came to me saying it was political.\nQuestion\nApologies if this reads like a rant, I really just don't understand the fundamental issue with a viewpoint that most U.S. citizens hold. What reasonable recourse of any kind do I have here given the situation above?\n", "answer": "I'd like to point out a few things here, that I think are a little off in your assessment of the situation.\n(Edit: All of this is assuming that your Skype is used for official work)\nFirst\nIt seems like you think that a joke on concentration camps (Especially one downplaying them like calling them an education camp) can be equated to a silly joke like \"Where we are going, we won't need roads\". This is not the case at all. One is harsh and should always be kept to a more private setting where you know it won't offend. The other is... well, just silly, non offensive.\nWith humor it is always advice to use non-offensive humor in work-settings, unless you specifically know that harsh humor is accepted by everyone at your work place.\nSecond\nYou talk about the relaxed setting at your work with rubber ducks. This is great and shows that humor is acceptable. Keep in mind that rubber ducks are also innocent in nature, so it doesn't show that everything goes, just that humor like \"Where we are going, we won't need roads\"is fine.\nThird\nYou talk about human rights and US citizens. This has nothing to do with the situation really. Because even if you are a US citizen, you can get offended by stuff. Someone obviously did here, the human rights act of 2019 has nothing to do with this.\nFourth\nYou say that your workplace Taiwan as Taiwan. This is fine, but keep in mind that there's no reason to assume that the person was offended by the pro Taiwanese opinion. They might as well just be offended by the joke because of it's dark nature.\nFifth\nYou mention your manager didn't even understand the joke as if that means she can't ask you to stop. Someone complained about a dark, out of place joke that was political. She probably knew there are no rules specifically stating \"No dark jokes\" but there's one saying \"keep political opinions to a minimum\" so she took the route that the rules allowed, to help this person.\nTLDR:\nYou made a joke that was not appropriate. It doesn't matter how many arguments you try to find that supports it being okay, because your manager told you it was not. Keep your humor more light and innocent.\n", "topic": "workplace", "url": "https://workplace.stackexchange.com/questions/147026"}, {"image": "VQAonline_00065171.png", "question": "Should I add my photo to CV if I get benefit from it?", "context": "I think everyone will agree with this statement: \"A picture is worth a thousand words\". It should also be applied to CV too. Searching on the internet, I find many advices tell me to avoid this, most of them are opinions. I don't understand this. The only one article I found which encourage you to adding photo in CV is from Forbes. \nAn example of a CV with photo: \n\nThis is a question that also ask the same thing, and I notice that I have a slightly different question.\nThe main reason to stay away from putting photo into CV is discrimination. However my country doesn't have that law, and in fact I have benefit from this.\n\nAttractive males received a 19.9% callback rate, almost 50% higher\n than the 13.7% response for plain men and more than twice the 9.2%\n response to those with no photo.\n\nI am not an attractive one, I have acne. But many of my friends say that I have feature that if I use Photoshop, I will have a nice-looking. So sorry if you feel I'm over confident.\nIf you have many chances to apply CVs with photos, does adding photo make your CVs have more chance to be accepted?\n", "answer": "Putting a picture on your cv is almost always a bad thing. First it wastes space that can be better used to sell your skills. CVs should never exceed 2 pages and if you put a picture there , you have less room for describing your skills.\nSecond, it appears naive as if you don't understand how things are done. Being thought of as naive is rarely a good thing when trying to imporess someone enought to schedule an interview. \nThird, it could very well make it easy for someone to screen you out based on appearance. So if you are not model-beautifiul, it doesn't work in your favor to have a picture. It is a known problem that short people, overweight people, older people, people of color, and unattractive people are judged more harshly in interviews. If you put the picture out there before the interview, you may get judged that way long before it gets to the interview and you have lost your chance to overcome that by selling yourself with your enthusiasm and technical ability. If you did choose to put a picture in, then it had better be of the highest qualty and that means hiring a really good professional photographer which can be costly.\nFourth, what would you gain from it? Yes a picture is worth a thousand words but how would having a picture help sell you to a potential employer? Are you selling your looks or your skills? A picture tells me nothing about your skills. Might as well put a picture of a cute puppy there, it would have as much positive impact for you.\nIt is true there are some jobs where a photo would be expected. Jobs where your looks are a large part of what you are selling such as acting or modeling.\n", "topic": "workplace", "url": "https://workplace.stackexchange.com/questions/33639"}, {"image": "VQAonline_00065190.png", "question": "Advocating for space. Is it rude?", "context": "My workplace follows the open office layout. My team consists of a product manager and 7 developers (including myself). We have one large table to ourselves and its a cozy setup we have managed. I sit next to the window at the edge of the table with the PM to my left.\nJust last week a designer was moved to our team from one of the other teams. He had to sit close to the PM and he decided to squeeze in between the him and me. That left very little wiggle room for me. I requested him to move to some other spot or make some space to which he obliged by moving away a few inches. Good enough.\n\nThe next day he got along all the stuff from his previous desk which included a lot of table accessories that take up space. My laptop was about 4 inches away from his mousepad. I had to request him again to make space as this arrangement was not comfortable for me. This time he moved but not without making remarks about how I am being such a baby. This was said in a friendly tone and I would not have given much thought to it, but then 2 other developers sitting across the table made the same comments (still in friendly tones).\nCurrently the setup is back to being cozy for all of us and I have no complaints but I feel the way I handled this situation ended up making me look stubborn and inflexible. The other developers may or may be okay with having very little breathing room and I would not like to broach the subject for a few days. \nShould I clear the air around why I had to make the request I made? What should I do if a similar situation presents itself again?\n", "answer": "Just leave it. If the situation is comfortable now, there is no need to bring it up again. Others probably don't think about this as much as you.\nIf the situation arises again, tell your team's manager that your team needs a bigger table and let him/her deal with it. If you need to explain yourself, do so firmly and without excuses. It's important to have a comfortable and ergonomic workplace where you can focus and not get distracted!\n", "topic": "workplace", "url": "https://workplace.stackexchange.com/questions/86460"}, {"image": "VQAonline_00065206.png", "question": "Strange job interview -- What were they really looking for?", "context": "This was the strangest job interview I ever had. I was interviewing for a position as an engineer at a US based tire manufacturing plant. The interview started pretty normally, and then the interviewer says \"please follow me\". I'm all dressed up in suit and tie. He walks me into the basement of the building where there were eight large 10x10\" timbers, each maybe six or seven feet long. There were two other observers there. He handed me a clipboard, and said you are to supervise the building of this cube structure, using these two workers. \nThe clipboard had a drawing with a structure looked pretty much like this... (obviously without the swimsuit models and yellow awning...) Each timber was six or seven feet long. They were notched at the ends and drilled for large metal locking pins. Pins were included. \n\nThe stated goal of the exercise was to get the structure complete in as quick a time as possible, while still being safe. He gives me gloves and hard hats. We started into it, and I realized it was really a three person hands-on job. I jumped in grabbed the timber lifted it over my head and placed it into position. The doggone things weight 60 or 70 lbs each. All the other beams could be pre-assembled and then tilted from the ground, but not the last two, at least not that I could figure out. Oh well.\nI did ask for advice, they absolutely remained silent. We could have easily built the thing upside down, but the clipboard guide showed it right side up. I have thought about alternative building methods (i.e. lean one side up against the wall, and that would have gotten us closer to completion, but that's still a bit unsafe... Note, I was coming out of the military where the mission was get it done, and I took the (remote) risk, not anyone else... I knew I could press 70 lbs easily... even while wearing a suit.\nAs it turns out I was leaving the US Army Combat Engineers, so I had a lot of familiarity with some of this stuff. At no time did I place either of the two volunteers even remotely at risk. And it wasn't clear to me if they wanted \"salaried\" folks to get their hands dirty when necessary. This was not a union labor force. At the time, I thought that was part of the test. I was totally in \"get it done\" mode. \nWe built the thing quickly, but I did NOT get the job offer. I have no clue what they were really looking for. It was a very odd interview. (Turns out I think I lucked out not working there... that plant had massive layoffs one year later... er... the grapes were probably sour?)\nMy questions: \n\nHas anybody ever seen a similar exercise at a job interview? \nWhat skills and traits are the interviewers really looking for?\nIs there a better way to address / prepare for such an odd interview? \nHow does one do well? \n\n", "answer": "\nHe handed me a clipboard, and said you are to supervise the building of this cube structure, using these two workers.\n\n...then...\n\nI jumped in grabbed the timber lifted it over my head and placed it into position.\n\nYou didn't do what was asked.\nYou were asked to supervise building a structure, but instead you jumped in and actually helped build the structure.\nIn addition:\n\nDid you have the relevant H&S training to ensure that the structure was being built safely by the workers?\nDid you check weights of timbers, ensure that you had enough people to lift them safely, etc.?\nDid you check the workers had the relevant H&S training?\n\nHindsight is everything of course - but in these situations, always clarify if in doubt, and always question what they're actually trying to get out of you here. They probably don't give two hoots if you can actually put together a bunch of wood, so there's an ulterior motive there somewhere - you just need to work out what it is, and play it to your advantage. That's easier said than done of course, and I hate \"mind game\" interviews (aside from a power trip for the interviewer, I don't see the point) - but that's sadly the reality of many interviews these days.\n", "topic": "workplace", "url": "https://workplace.stackexchange.com/questions/153792"}, {"image": "VQAonline_00065793.png", "question": "Why are potions always brewed in extreme bulk?", "context": "This isn't a question intended for any specific setting. Rather, this is a general point of curiosity that occurred to me while brainstorming my story. Potion brewing is almost inexorably tied to the image of a cauldron. Whether it be witches standing over a cauldron big enough to boil someone alive in and stirring it with a big stick, or more reasonably-sized personal cauldrons in, say, Harry Potter, potions are very, very frequently being brewed in these bulky, thick metal pots, to the point that some people don't realize they ever historically saw mundane use.\nThe thing is... cauldrons are big. Even the small ones for personal use, as far as I can tell, can hold a lot of liquid. Even if we assume half a liter, about the size of a commercial water bottle you'd find for sale at a deli or pizza place, is an appropriate serving size of the average potion, then even tiny cauldrons, when filled with potion ingredients, would make at least a dozen servings of such a thing. And yet in, say, Harry Potter, we're told that on Harry's first day of potions class in the sixth book, Professor Slughorn had a \"large, bubbling cauldron\" of Felix Felicis on display for the class... a potion where a single serving size looks like this:\n\nThe unspoken implication is crystal clear: any world where cauldrons are heavily associated with potion-making is a world where potions are almost always brewed in bulk. Where dozens, maybe even hundreds of servings of a single potion are brewed at a time.\nWhat might the reason be that brewing potions in extreme bulk is such a natural and widespread form of potion-making that cauldrons could possibly become so heavily associated with it?\n", "answer": "Exact Proportions.\nThe ingredient ratios in a potion must be very precise. The easiest way to achieve this is to brew a large amount.\nSuppose you need 1 litre of swamp water, 10 grams of ant juice and 0.5 grams of butterfly testicle powder, and can reliably measure anything up to the closest half gram. Then due to measuring errors you will end up with anywhere from 0 to 1 grams of butterfly testicle powder. In particularly you might use double the correct amount and ruin the potion.\nIf instead you brew 100 litres of potion, then you need 50 grams of powder, and the error is only between 49.5 to 50.5. So you have at most a 1% error.\nThis also applies when creating butterfly testicle powder in the first place. The potency varies between butterflies. So if you only need 1 butterfly's worth and I only need 1 butterfly's worth, and we each grind up one butterfly, then we might end up with different potency and cannot trade recipes. Better everyone in the area pools their butterflies. Then the resulting brew is measured for potency and redistributed.\n", "topic": "worldbuilding", "url": "https://worldbuilding.stackexchange.com/questions/214194"}, {"image": "VQAonline_00065870.png", "question": "what would the wind patterns on a world that has a constant \"midnight sun\"", "context": "I created a world that's axial tilt is on a constant 45 degree angle in the northern hemisphere it would look something like this. The problem is I can't figure out how the wind currents would work.\nthe planet has the same mass, size, orbit speed and rotation speed as earth. If anyone can explain it to me it would be a big help.\n", "answer": "Almost an Eyeball Earth\nAn Eyeball Earth is a hypothetical type of planet where one side constantly faces the star. There are theories about how the climate would work, mostly on how there are persistent winds between the two sides.\nSpecifically, the colder, denser air would move closer to the ground, get into the \"eye\", become warm air, rise up, and then move to the cold side higher up in the atmosphere. I would expect something similar on this planet.\nSimilar is Not the Same\nIgnoring the effects of terrain and ocean currents, both of which are pretty important to climate models, you essentially have a \"tropics\" ring at very high latitudes. Storms would gain energy there, move to the cold side, and cause rain/snow as they cool off. I would expect something like Hadley Cells to form, but built around the \"tropics ring\" instead of the equator.\nAdditionally, since the planet is also rotating, you get Coriolis effects on these storm systems. It wouldn't be a straight shot over to the cold side: everything would drift and spin. This would cause interesting variations in weather patterns.\nInside the Tropics\nAdditionally, the area inside the \"tropics\", the cap formed by the ring where the sun is directly overhead, would be very interesting! It would have near constant exposure to the sun and be on the tail end of cold air systems. Surface winds would likely be calm, like the doldrums on earth. There could possibly be a desert or very warm sea there!\n", "topic": "worldbuilding", "url": "https://worldbuilding.stackexchange.com/questions/235801"}, {"image": "VQAonline_00065375.png", "question": "How I explain New France not having their Middle East?", "context": "In an alternate history I am designing, France wins the Seven Years War and does not lose their land claims in America. A problem I have reached though is explaining why land east of the Mississippi winds up in American control. \nHow can I explain why the land east of the Mississippi (represented by the red line) would come into American control?\n\n", "answer": "The American Revolution Happened Earlier\nThe reason that the French won the 7 Years War is that the America Revolution happened in the midst of it, and France recognized the territory of the newly independent USA in exchange for a cessation of hostilities on that front.\n", "topic": "worldbuilding", "url": "https://worldbuilding.stackexchange.com/questions/59962"}, {"image": "VQAonline_00065364.png", "question": "Modern soldiers carry axes instead of combat knives. Why?", "context": "In my alternative history story, taking place at the end of the 20th century, one country has an army with platoons of airborne scouts. But unlike modern special forces they don't use combat knives as side-arms, but combat axes. Like this one:\n\nSo, question is: why do they use them instead of combat knives? \nIs there any advantage?\nMy ideas are:\n\nTraditions (maybe the ancestors of these soldiers were lumberjacks)\nWhen you have to drop in forested areas an axe can be much more useful to chop wood, make shelters, including long-lived shooting points, or traversing young forest areas.\nThere was a Soviet Union soldier who captured a Nazi tank using an axe during World War II\n\nI know that in close-quarter combat killing with a knife is usually faster; you can kill with a single well-placed jab, and an axe requires space to make a swing. An axe is also harder to conceal. But the main soldier's weapon is an automatic rifle, not a knife or axe.\nUPD: As Nex Terren mentioned, an axe, like a shovel, can be used as entrenching tool. Furthermore, Soviet Army used small sapper shovels:\n\nSoviet Spetsnaz units had advanced training with the MPL-50, which\n they mostly used not for entrenching, but for close quarters combat.\n The spade is well balanced, which allows it to be used as a throwing\n weapon.\n\nSo using axe not just as a tool but also as a last-chance melee and throwing weapon looks possible.\n", "answer": "Breaching\nI'm surprised no one has mentioned breaching.\nUrban combat is much more common than it used to be, especially with anti-insurgency suppression and targeted rescue/kill operations. \nAn infantry company may all have to perform breaching or they may have a special unit for breaching, but in these scenarios an axe has much more utility and versatility as a tool than a knife.\nClose Quarters Combat\nAn axe also has more reach and requires less dexterity than a knife, which can be useful with less training where visibility is low and corners are common. It can also cause more damage with a glancing blow and can be reversed to cause the same puncture damage with less force due to levering action with the handle. \nReferences\nGoogling around, I was easily able to find references to axes as mechanical breaching tools, often used when explosives are overkill, too dangerous to use in enclosed spaces, or collateral material damage is unacceptable (like public spaces or inhabited buildings).\nFor example, this is from a US military Infantry Rifle Company handbook chapter on urban operations:\n\nExplosive breaching includes using nonelectrical demolition systems; ballistic breaching includes using direct fire weapons; and mechanical breaching includes using crowbars, axes, saws, hooligan's tools, and sledgehammers.\n\nThere's also more online on WWII Axes\n\nThe main role of an axe during WWII was to cut wood or material for things such as building a fire, building shelters, etc. \n\nThe US Army Ranger Handbook also mentions using hatchets as standard issue (tho headlines with a quote from 1759).\n\nLet the enemy come till he's almost close enough to touch, then let him have it and jump out and finish him up with\n your hatchet.\n\nLater it mentions a hatchet as in the Level 3 Survival Kit:\n\nLevel 3 Kit (carried in assault pack or ruck) water proof container with more of the materials listed in the level 1 and 2\n kits plus shelter making materials (poncho, tarp, bungee cords, or space blanket) and a hatchet or saw.\n\n", "topic": "worldbuilding", "url": "https://worldbuilding.stackexchange.com/questions/57100"}, {"image": "VQAonline_00065297.png", "question": "Cutting down giant trees", "context": "Inspired by Nick Staab's recent work \"Jack\" (pictured below), how would you go around cutting down massive trees?\n \nIn this scenario the simpler the tools the better, as it would allow a primitive civilization to capitalize on their plentiful resource. \n", "answer": "A primitive society which was keen to cut down such a tree might try killing it first.\nA strip of bark is removed in a continuous ring around the base of the tree, and then the team cuts into the exposed wood in order to cut off the flow of sap to the tree. After a while it dies, and then the wood can be allowed to dry out and age for a while.\nAfter that, the tree can be cut down in the usual way using axes, wedges and saws, but since the wood is no longer living \"green\" wood it won't be so elastic and difficult to cut, making it easier to finish the job even with copper axes of other soft metal tools. (Once you get to bronze axes and saws, you are essentially using modern technology, our great refinements is steel can hold an edge for much longer and cross cut saws now have much better designs for the teeth).\n", "topic": "worldbuilding", "url": "https://worldbuilding.stackexchange.com/questions/33106"}, {"image": "VQAonline_00065618.png", "question": "Could there be something like aerobatic smoke trails in the vacuum of space?", "context": "Whether exhaust from a solid rocket engine, or something released by spaceships in flight (like aerobatic smoke in the atmosphere), could extended trails behind a ship be created that would not dissipate in the vacuum of space?\nCould a trail be formed that a person close enough to see the ship with the naked eye would be able to see the trail?\n \n", "answer": "Using fumed silica. It is stable in vacuum, after crystallizing it won't spread out and it requires little source material to create large volume agglomerates.\n", "topic": "worldbuilding", "url": "https://worldbuilding.stackexchange.com/questions/146988"}, {"image": "VQAonline_00065368.png", "question": "Fossils on the Moon: Panspermia meets Conspiracy Theories!", "context": "This is an answer I found while looking up random factoids. And it's gotten me wondering.\n((If I should take down the picture, and just paraphrase the text, please let me know.))\n\nCharlie here is right, in that this is a fairly ridiculous theory. However, the sheer ridiculousness of the concept reminded me of something that, while not quite as absurd as dinosaur fossils on Luna, is very similar in some aspects: Panspermia.\nFor those who need reminding, Panspermia is the concept that tiny and simplistic microbial life forms can be 'ejected' from their homeworlds by way of impact ejecta, hitch-hike on these wayward meteoroids, and then spread to other worlds when their host meteor impacts another planetary body. Scientific experimentation on this theory (as of this date) has proven a bit iffy, but it's still a popular idea in sci-fi stories and video games.\nThat being said, let's look back at the lunacy (nyuk-nyuk) of the Dinosaurs on the Moon idea. Suppose an incident similar to this happened earlier on in this planets history - say, when only microbial life forms exist on the surface. As several documentaries have claimed, there are potential places where life forms can survive extreme heat, even in nigh-apocalyptic conditions.\nCould a \"Rapid Centrifugal Disintegration\" event similar to this result in a planet and a moon with life forms derived from the same common ancestors? How could life survive it? Assuming they don't, could fossils of these microbial survivors even be found/survive to the civilized age?\n", "answer": "No. The rapid centrifugal stuff is nonsense. \nThere are ways to blast rock off a planet though, including an incoming impact. We have found rocks that came from other bodies and fallen to Earth, so it happens!\nSee the extended quote I included in this answer for current thoughts on how could life survive it.\n", "topic": "worldbuilding", "url": "https://worldbuilding.stackexchange.com/questions/57738"}, {"image": "VQAonline_00065930.png", "question": "How to format those little blurbs at the start of chapters?", "context": "At the start of each chapter in Dune there are quotes from Maudib and Fremen or Bene Gesserit sayings. There are similar epistolary entries at the start of Mist born.\nWhat is the proper term for them and how do I properly format them in my manuscript?\n", "answer": "You must be talking about Epigraph.\nThe formatting of epigrams varies widely. Refer to the relevant style guide for your publication.\nBroadly speaking, this guidance from an APA style editor, How to Format an Epigraph, can suit most purposes.\n\nThe text of the epigraph is indented from the left margin in the same way as a block quote. On the line below the end of the epigraph, the author\u2019s name (and only the author\u2019s last name if he or she is well-known) and the source\u2019s title should be given. This credit line should be flush right, preceded by an em dash. An epigraph\u2019s source is not listed in the References section.\n\n", "topic": "writers", "url": "https://writers.stackexchange.com/questions/58894"}, {"image": "VQAonline_00065893.png", "question": "Author order and grouping on technical document", "context": "This question pertains to author/contributor order and grouping on a technical document.\n\nI performed tests on some equipment and had two other people directly involved.\nMy boss and his boss need to both be listed on the document as they ordered the testing and the paper needs them on it to have any validity.\nThere were several other people who were not actively involved but made contributions ranging from from a comment like \"hey, don't forget to test XYZ\" to providing a part needed in testing\n\nHow do I order the people involved in testing and do I separate the active testers from 'other' contributors?\nBelow is a screenshot of what I've got right now. (names and subject are changed)\n\n", "answer": "1) A comment like \"hey, don't forget to test XYZ\" gets an acknowledgement at the end.\n2) I've never seen a format like this, where you actually separate the author list into two sections. I like it. Just have never seen it.\n3) Bosses go at the end of the author list. In academia or government, they should be left off if they had no real involvement. But well-known people often get author credit just for their name recognition. Everyone agrees it is unethical, but many people do it anyway. Industry may be different.\n4) Providing a part: Not enough info to decide. Normally, someone who just made something to your specs, using only standard techniques, would not get any share of the authorship. That goes for machining, mixing chemicals, soldering a circuit, etc. Use an acknowledgement.\n5) Some journals require an explanation of what each author did, and it gets published with the paper (in a boxed footnote on the first page, in a tiny font). I love that; helps keep people honest. In your case, it would probably defeat the purpose of your publication. \"All the real work was done by the first three guys. Our boss told us to take the measurements. The Vice-President of Research did nothing whatsoever.\" Not a good career strategy. Sadly.\n", "topic": "writers", "url": "https://writers.stackexchange.com/questions/10836"}, {"image": "VQAonline_00065915.png", "question": "Using real words from a foreign culture feels like 'Calling a rabbit a \"smeerp\"'", "context": "I'm working on a novel, that's set in pre-Islamic Persia, in the same general way that The Lord of the Rings is set in Britain. (Meaning, it's set in a world all its own, but there's this source of inspiration.)\nHere's my conundrum: the land is ruled by the Shah - that's a given, that's expected if the setting is Persian rather than European. But what happens under the Shah? Knights, barons, counts and dukes are all titles associated with the European court. They appear to clash with a setting, as if I'm telling a basically European story, only recoloured Middle-eastern.\nSo the knights are asvarans (it's actually amazing how much the position of the asvarans in 5th century Persia is reminiscent of 10th century European knights). And after much research, I've got vaspahrs, sardars and ostandars. At which point, I'm looking at the trope Calling a Rabbit a \"Smeerp\" - I'm just giving different names to something that has a perfectly good English word.\nMoreover, I have only recently pointed others to this xkcd:\n\n(source)\nI do not believe it is relevant that I found the words I'm using in an encyclopedia rather than made them up; to the reader, they are equally unfamiliar.\nHow do I balance realism against readability in this particular case? I do not want to break the readers' suspension of disbelief by using words that are too European, but I don't want to weigh on the reader with heaps of foreign-language vocabulary either.\n(Note: Bioware's Dragon Age franchise uses 'Teyrn', 'Arl', 'Bahn' instead of 'Duke', 'Earl', 'Baron'. However, in their example the replacement words are not too far from the English words, and thus much easier to remember, avoiding confusion. Also, the names they use are for the most part English enough. Consequently, looking at something like 'Arl Eamon', one doesn't have to wonder which part is title and which part is name. As opposed to 'Vaspahr Narseh', for instance.)\n", "answer": "I've found that the main key to unfamiliar words -- and this applies to jargon in technical writing as much as it does to foreign or made-up words in fiction -- is density. The example in the XKCD comic is irritating because it can't get through a single sentence without three new words. The situation is very different if three unfamiliar words are introduced over the span of a chapter.\nAnother key is how naturally you supply the explanation. Instead of \"translating\" or explaining, provide context -- introduce the asvarans in a setting where their martial role is apparent, show your sardars in leadership roles, show your ostandars ruling, etc. This might be direct (you show those characters doing those things) or indirect (people refer to them in connection with illustrative events or attitudes).\nImagine if the XKCD example were instead handled like this:\n\nThe six fra'ars stood solemnly in front of the gate, their gray beards all reaching nearly to their waists. Despite their years they stood strong and alert. $Name, standing in front of the others, held a large sword aloft in one hand, seemingly effortlessly. $POV-character involuntarily took a step back; he knew that the krytosis was normally wielded two-handed because of its weight.\nHe heard the din of the many farmlings running and playing beyond the gate. He envied them; they had no cares, were not affected by the ill tidings in the land, and had no idea of their eventual fate. He wished he could be young and oblivious again. [...]\n\nThis is more jargon-dense than I would write for \"real\", but I hope it illustrates the point that you can introduce terms without falling into the \"pass the dictionary\" trap.\n", "topic": "writers", "url": "https://writers.stackexchange.com/questions/40637"}, {"image": "VQAonline_00065928.png", "question": "In an action scene, what is it called when a flying/thrown object changes rotation quickly after hitting something?", "context": "I am writing an action scene in which a character is thrown/falls and while they are flying through the air either the top or bottom half of their body hits an immovable object mid air, causing them to continue in the same direction but with their body at a different rotation now, without starting to spin because of it. It's essentially like a rug pull but happening in mid air if that makes sense? I think I have read this as \"jackknifing\" in other writing but I'm not sure if that is the correct use case and meaning? Here is a video example except the character spins after impact, which is not desired.\nI have included a diagram to help illustrate this as well (side on view, top of arrows are character's heads, grey object is the ground, green object is stationary and immovable):\n\nExample sentence:\nRed guy hit blue guy, sending him flying back into a green object, which ______ his body.\n", "answer": "Fun word games are fun . Some options:\n\nGo simple\n\n\n... which twisted his body\n\n\nChange the wording\n\n\n... he hit the object and that caused him to swivel mid-air\n\n\n... he collided with the object which made his body veer sharply to the left\n\n\nChain more than one action\n\n\n... his body ricocheted off the object and spun.\n\n\nFind the exact verb you want...?\n\nOverall, if we're speaking about verbs and their synonyms, I think \"turn\" is a good starting point in this case:\nhttps://www.merriam-webster.com/thesaurus/turn\nOr maybe \"twist\":\nhttps://www.merriam-webster.com/thesaurus/twist\nIn such cases, I usually start general, then click on the synonym that is the closest in meaning to what I'm looking for, and follow the links.\n", "topic": "writers", "url": "https://writers.stackexchange.com/questions/55576"}, {"image": "VQAonline_00065922.png", "question": "A flower's head or heart?", "context": "I\u2019m currently writing a story and one of my characters is a butterfly. In one of the scenes, she manages to run away from a bunch of hungry lizards and hides into a flower; let\u2019s say a Zinnia. \nWhen describing the scene I wrote: \n\n\u201c\u2026\u2026she dived into the Zinnia\u2019s head and hastily rolled into the\n pollen\u2026.\u201d\n\nBut then, I found that the expression \u201cflower head\u201d means: \u201cA dense, compact cluster of small flowers that appear to be a single flower, as of a dandelion or clover.\u201d \nThus, I changed the word to: \n\n\u201c\u2026she dived into the Zinnia\u2019s heart and hastily rolled into the pollen\u2026.\u201d\n\nWould it be correct?\n\n", "answer": "A few alternatives:\n\nShe dived into the flower.\nShe dived into the zinnia's flower.\nShe dived into the petals.\nShe dived into the zinnia's center.\n\nOr, simply:\n\nShe dived into the pollen.\n\nAdd the rolling in pollen parts if needed.\nI realize you want to differentiate between zinnia the plant and the actual flower. You don't want your readers thinking she's going to be hiding under the leaves.\nBoth \"head\" and \"heart\" have other connotations; using either of those terms might pull the reader out of the story. My guess is that your story is for children (because of the butterfly character). If this is the case, then you really want your language to be clear and easy to follow. In any case, avoid terms that aren't perfectly clear.\n", "topic": "writers", "url": "https://writers.stackexchange.com/questions/45934"}, {"image": "VQAonline_00065904.png", "question": "Why do typesetters capitalize the first few words of a new chapter?", "context": "Something I've always wondered. You've probably noticed that in some newspapers or books, the first few words of a chapter/story are bolded or capitalized, similar to initials. For example:\n\nThis image isn't from a real book, it's from an SE puzzle parodying a newspaper. It should convey my point though.\nI've heard that this is a typesetting convention and it is not the writers who capitalize these words. However, what is the point? Is it recommended in publishing? Does it actually help readability, or is there some historical reason?\n", "answer": "That's called a lead-in. The general idea is to use special formatting (e.g. all caps, small caps, italics) to gently guide the reader to recognize where the text begins (or resumes after a break).\nIf sections are marked with headers, guidance like that isn't strictly necessary. In those cases, the use of lead-ins is a style choice.\nAnd, yes, that kind of formatting is the publisher's job, not the writer's.\n", "topic": "writers", "url": "https://writers.stackexchange.com/questions/27309"}, {"image": "VQAonline_00065892.png", "question": "Do publishers use LaTeX to produce books for the blind?", "context": "What software tools do publishers use to produce books for the blind, i.e. containing embossed Braille?\nIs LaTeX one of them? There is a braille LaTeX package, which can be used to print Braille, but I'm not sure it can be used to emboss Braille. If not LaTex, what do publishers use?\nI'd be keen to work on a LaTeX package for both printing and embossing Braille but, first, I need to know whether there would be an interest for such a solution...\n\n", "answer": "I've seen this up for quite a while now and will take a stab at answering your question. However, the answer is much more complicated than the question.\nThe simple answer is yes, Python is used to convert text to braille tags, and a LaTex package is used to print out the symbols. IF the brailler you are using is not designed for use with a proprietary package. I think Bookmaker and Express machines come with open ports.\nAs a GNU/Linux user myself, I find it hard to find open source drivers for most hardware. So I have a lot of boat anchors sitting around here.\nNow for the complicated answer, simplified.\nWhat do Publishers use?\nThe Duxbury Braille Translator for Windoze is fairly universal among publishers. It can drive a Thermoform machine, most Embossers, as well as the top plate embossing machines, such as the ET PED-30.\nIt also works perfectly with the ET BraillePlace double-sided embossing for making books.\nIf you are interested in producing books, the lowest cost commercial grade double-sided Brailler I am aware of is the ET Bookmaker.\nFor the final part of your question about interest in software.\nI would say no, because of the proprietary nature of the hardware.\nAs you are well aware, even SANE cannot obtain enough information from manufacturers to create drivers for most of the scanners, printers, and other hardware available. Most of the manufacturers are in bed with Mickey$oft and could care less about the rest of the world.\nVTY\nDutch\n", "topic": "writers", "url": "https://writers.stackexchange.com/questions/10359"}]